Beginning Algebra

Download as pdf or txt
Download as pdf or txt
You are on page 1of 1601

Beginning Algebra

v. 1.0
This is the book Beginning Algebra (v. 1.0).

This book is licensed under a Creative Commons by-nc-sa 3.0 (http://creativecommons.org/licenses/by-nc-sa/


3.0/) license. See the license for more details, but that basically means you can share this book as long as you
credit the author (but see below), don't make money from it, and do make it available to everyone else under the
same terms.

This book was accessible as of December 29, 2012, and it was downloaded then by Andy Schmitz
(http://lardbucket.org) in an effort to preserve the availability of this book.

Normally, the author and publisher would be credited here. However, the publisher has asked for the customary
Creative Commons attribution to the original publisher, authors, title, and book URI to be removed. Additionally,
per the publisher's request, their name has been removed in some passages. More information is available on this
project's attribution page (http://2012books.lardbucket.org/attribution.html?utm_source=header).

For more information on the source of this book, or why it is available for free, please see the project's home page
(http://2012books.lardbucket.org/). You can browse or download additional books there.

ii
Table of Contents
About the Author .................................................................................................................. 1
Acknowledgments................................................................................................................. 2
Preface..................................................................................................................................... 3
Chapter 1: Real Numbers and Their Operations ............................................................ 5
Real Numbers and the Number Line ............................................................................................................ 6
Adding and Subtracting Integers ............................................................................................................... 31
Multiplying and Dividing Integers ............................................................................................................. 47
Fractions ....................................................................................................................................................... 67
Review of Decimals and Percents ............................................................................................................... 99
Exponents and Square Roots .................................................................................................................... 127
Order of Operations ................................................................................................................................... 149
Review Exercises and Sample Exam......................................................................................................... 167
Chapter 2: Linear Equations and Inequalities ............................................................ 184
Introduction to Algebra............................................................................................................................. 185
Simplifying Algebraic Expressions........................................................................................................... 206
Solving Linear Equations: Part I ............................................................................................................... 227
Solving Linear Equations: Part II .............................................................................................................. 249
Applications of Linear Equations ............................................................................................................. 270
Ratio and Proportion Applications .......................................................................................................... 305
Introduction to Inequalities and Interval Notation ............................................................................... 327
Linear Inequalities (One Variable) ........................................................................................................... 346
Review Exercises and Sample Exam......................................................................................................... 372
Chapter 3: Graphing Lines .............................................................................................. 390
Rectangular Coordinate System ............................................................................................................... 391
Graph by Plotting Points ........................................................................................................................... 420
Graph Using Intercepts ............................................................................................................................. 447
Graph Using the y-Intercept and Slope ................................................................................................... 470
Finding Linear Equations .......................................................................................................................... 502
Parallel and Perpendicular Lines ............................................................................................................. 529
Introduction to Functions ......................................................................................................................... 548
Linear Inequalities (Two Variables) ......................................................................................................... 578
Review Exercises and Sample Exam......................................................................................................... 597

iii
Chapter 4: Solving Linear Systems................................................................................ 619
Solving Linear Systems by Graphing ....................................................................................................... 620
Solving Linear Systems by Substitution .................................................................................................. 644
Solving Linear Systems by Elimination ................................................................................................... 664
Applications of Linear Systems ................................................................................................................ 689
Solving Systems of Linear Inequalities (Two Variables)........................................................................ 715
Review Exercises and Sample Exam......................................................................................................... 732
Chapter 5: Polynomials and Their Operations ........................................................... 746
Rules of Exponents..................................................................................................................................... 747
Introduction to Polynomials..................................................................................................................... 775
Adding and Subtracting Polynomials ...................................................................................................... 797
Multiplying Polynomials ........................................................................................................................... 816
Dividing Polynomials................................................................................................................................. 841
Negative Exponents ................................................................................................................................... 864
Review Exercises and Sample Exam......................................................................................................... 887
Chapter 6: Factoring and Solving by Factoring .......................................................... 903
Introduction to Factoring ......................................................................................................................... 904
Factoring Trinomials of the Form x^2 + bx + c........................................................................................ 935
Factoring Trinomials of the Form ax^2 + bx + c...................................................................................... 955
Factoring Special Binomials...................................................................................................................... 978
General Guidelines for Factoring Polynomials ....................................................................................... 999
Solving Equations by Factoring .............................................................................................................. 1013
Applications Involving Quadratic Equations ........................................................................................ 1038
Review Exercises and Sample Exam....................................................................................................... 1063
Chapter 7: Rational Expressions and Equations....................................................... 1077
Simplifying Rational Expressions........................................................................................................... 1078
Multiplying and Dividing Rational Expressions ................................................................................... 1107
Adding and Subtracting Rational Expressions...................................................................................... 1127
Complex Rational Expressions................................................................................................................ 1152
Solving Rational Equations ..................................................................................................................... 1172
Applications of Rational Equations ........................................................................................................ 1192
Variation ................................................................................................................................................... 1219
Review Exercises and Sample Exam....................................................................................................... 1241

iv
Chapter 8: Radical Expressions and Equations......................................................... 1256
Radicals ..................................................................................................................................................... 1257
Simplifying Radical Expressions............................................................................................................. 1285
Adding and Subtracting Radical Expressions ....................................................................................... 1313
Multiplying and Dividing Radical Expressions ..................................................................................... 1330
Rational Exponents .................................................................................................................................. 1360
Solving Radical Equations ....................................................................................................................... 1385
Review Exercises and Sample Exam....................................................................................................... 1409
Chapter 9: Solving Quadratic Equations and Graphing Parabolas ....................... 1425
Extracting Square Roots .......................................................................................................................... 1426
Completing the Square ............................................................................................................................ 1449
Quadratic Formula ................................................................................................................................... 1469
Guidelines for Solving Quadratic Equations and Applications............................................................ 1488
Graphing Parabolas.................................................................................................................................. 1514
Introduction to Complex Numbers and Complex Solutions................................................................ 1553
Review Exercises and Sample Exam....................................................................................................... 1575
Appendix: Geometric Figures ....................................................................................... 1592
Plane .......................................................................................................................................................... 1593
Solid ........................................................................................................................................................... 1595

v
About the Author
John Redden earned his degrees at California State
University–Northridge and Glendale Community
College. He is now a professor of mathematics at the
College of the Sequoias, located in Visalia, California.
With over a decade of experience working with students
to develop their algebra skills, he knows just where they
struggle and how to present complex techniques in
more understandable ways. His student-friendly and
commonsense approach carries over to his writing of Elementary Algebra and various
other open-source learning resources.

Author site: http://edunettech.blogspot.com/

1
Acknowledgments
I would like to thank the following reviewers whose feedback helped improve the
final product:

• Katherine Adams, Eastern Michigan University


• Sheri Berger, Los Angeles Valley College
• Seung Choi, Northern Virginia Community College
• Stephen DeLong, Colorado Mountain College
• Keith Eddy, College of the Sequoias
• Solomon Emeghara, William Patterson University
• Audrey Gillant, SUNY–Maritime
• Barbara Goldner, North Seattle Community College
• Joseph Grich, William Patterson University
• Caroll Hobbs, Pensacola State College
• Clark Ingham, Mott Community College
• Valerie LaVoice, NHTI, Concord’s Community College
• Sandra Martin, Brevard Schools
• Bethany Mueller, Pensacola State College
• Tracy Redden, College of the Sequoias
• James Riley, Northern Arizona University
• Bamdad Samii, California State University–Northridge
• Michael Scott, California State University–Monterey Bay
• Nora Wheeler, Santa Rosa Junior College

I would also like to acknowledge Michael Boezi and Vanessa Gennarelli of Unnamed
Publisher. The success of this project is in large part due to their vision and
expertise. Finally, a special heartfelt thank-you is due to my wife, Tracy, who spent
countless hours proofreading and editing these pages—all this while maintaining a
tight schedule for our family. Without her, this textbook would not have been
possible.

2
Preface
It is essential to lay a solid foundation in mathematics if a student is to be
competitive in today’s global market. The importance of algebra, in particular,
cannot be overstated, as it is the basis of all mathematical modeling used in
applications found in all disciplines. Traditionally, the study of algebra is separated
into a two parts, elementary algebra and intermediate algebra. This textbook,
Elementary Algebra, is the first part, written in a clear and concise manner, making
no assumption of prior algebra experience. It carefully guides students from the
basics to the more advanced techniques required to be successful in the next
course.

This text is, by far, the best elementary algebra textbook offered under a Creative
Commons license. It is written in such a way as to maintain maximum flexibility
and usability. A modular format was carefully integrated into the design. For
example, certain topics, like functions, can be covered or omitted without
compromising the overall flow of the text. An introduction of square roots in
Chapter 1 is another example that allows for instructors wishing to include the
quadratic formula early to do so. Topics such as these are carefully included to
enhance the flexibility throughout. This textbook will effectively enable traditional
or nontraditional approaches to elementary algebra. This, in addition to robust and
diverse exercise sets, provides the base for an excellent individualized textbook
instructors can use free of needless edition changes and excessive costs! A few other
differences are highlighted below:

• Equivalent mathematical notation using standard text found on a


keyboard
• A variety of applications and word problems included in most exercise
sets
• Clearly enumerated steps found in context within carefully chosen
examples
• Alternative methods and notation, modularly integrated, where
appropriate
• Video examples available, in context, within the online version of the
textbook
• Robust and diverse exercise sets with discussion board questions
• Key words and key takeaways summarizing each section

This text employs an early-and-often approach to real-world applications, laying


the foundation for students to translate problems described in words into

3
Preface

mathematical equations. It also clearly lays out the steps required to build the skills
needed to solve these equations and interpret the results. With robust and diverse
exercise sets, students have the opportunity to solve plenty of practice problems. In
addition to embedded video examples and other online learning resources, the
importance of practice with pencil and paper is stressed. This text respects the
traditional approaches to algebra pedagogy while enhancing it with the technology
available today. In addition, textual notation is introduced as a means to
communicate solutions electronically throughout the text. While it is important to
obtain the skills to solve problems correctly, it is just as important to communicate
those solutions with others effectively in the modern era of instant
communications.

Unnamed Publisher is the only publisher today willing to put in the resources that
it takes to produce a quality, peer-reviewed textbook and allow it to be published
under a Creative Commons license. They have the system that implements the
customizable, affordable, and open textbook of the twenty-first century. In fact, this
textbook was specifically designed and written to fully maximize the potential of
the Unnamed Publisher system. I feel that my partnership with Unnamed Publisher
has produced a truly fine example in Elementary Algebra, which demonstrates what
is possible in the future of publishing.

4
Chapter 1
Real Numbers and Their Operations

5
Chapter 1 Real Numbers and Their Operations

1.1 Real Numbers and the Number Line

LEARNING OBJECTIVES

1. Construct a number line and graph points on it.


2. Use a number line to determine the order of real numbers.
3. Determine the opposite of a real number.
4. Determine the absolute value of a real number.

Definitions

A set1 is a collection of objects, typically grouped within braces { }, where each


object is called an element2. For example, {red, green, blue} is a set of colors. A
subset3 is a set consisting of elements that belong to a given set. For example,
{green, blue} is a subset of the color set above. A set with no elements is called the
empty set4 and has its own special notation, { } or ∅.

When studying mathematics, we focus on special sets of numbers. The set of


natural (or counting) numbers5, denoted N, is

1. Any collection of objects.


The three periods (…) is called an ellipsis and indicates that the numbers continue
2. An object within a set. without bound. The set of whole numbers6, denoted W , is the set of natural
3. A set consisting of elements
numbers combined with zero.
that belong to a given set.

4. A subset with no elements,


denoted ∅ or { }.

5. The set of counting numbers


{1, 2, 3, 4, 5, …}.

6. The set of natural numbers The set of integers7, denoted Z, consists of both positive and negative whole
combined with zero {0, 1, 2, 3, numbers, as well as zero.
4, 5, …}.

7. The set of positive and


negative whole numbers
combined with zero {…, −3, −2,
−1, 0, 1, 2, 3, …}.

6
Chapter 1 Real Numbers and Their Operations

Notice that the sets of natural and whole numbers are both subsets of the set of
integers.

Rational numbers8, denoted Q, are defined as any number of the form ab, where a
and b are integers and b is nonzero. Decimals that repeat or terminate are rational.
For example,

The set of integers is a subset of the set of rational numbers because every integer
can be expressed as a ratio of the integer and 1. In other words, any integer can be
written over 1 and can be considered a rational number. For example,

Irrational numbers9 are defined as any number that cannot be written as a ratio of
two integers. Nonterminating decimals that do not repeat are irrational. For
example,

The set of real numbers10, denoted R, is defined as the set of all rational numbers
combined with the set of all irrational numbers. Therefore, all the numbers defined
so far are subsets of the set of real numbers. In summary,

8. Numbers of the form ba , where


a and b are integers and b is
nonzero.

9. Numbers that cannot be


written as a ratio of two
integers.

10. The set of all rational and


irrational numbers.

1.1 Real Numbers and the Number Line 7


Chapter 1 Real Numbers and Their Operations

Number Line

A real number line11, or simply number line, allows us to visually display real
numbers by associating them with unique points on a line. The real number
associated with a point is called a coordinate12. A point on the real number line
that is associated with a coordinate is called its graph13.

To construct a number line, draw a horizontal line with arrows on both ends to
indicate that it continues without bound. Next, choose any point to represent the
number zero; this point is called the origin14.

Mark off consistent lengths on both sides of the origin and label each tick mark to
define the scale. Positive real numbers lie to the right of the origin and negative
real numbers lie to the left. The number zero (0) is neither positive nor negative.
Typically, each tick represents one unit.

11. A line that allows us to visually


represent real numbers by
associating them with points
on the line. As illustrated below, the scale need not always be one unit. In the first number line,
each tick mark represents two units. In the second, each tick mark represents 17.
12. The real number associated
with a point on a number line.

13. A point on the number line


associated with a coordinate.

14. The point on the number line


that represtents zero.

1.1 Real Numbers and the Number Line 8


Chapter 1 Real Numbers and Their Operations

The graph of each real number is shown as a dot at the appropriate point on the
number line. A partial graph of the set of integers Z follows:

Example 1: Graph the following set of real numbers: {−1, − 1


3
, 0, 5
3 }.

Solution: Graph the numbers on a number line with a scale where each tick mark
represents 13 unit.

Ordering Real Numbers

When comparing real numbers on a number line, the larger number will always lie
to the right of the smaller one. It is clear that 15 is greater than 5, but it may not be
so clear to see that −1 is greater than −5 until we graph each number on a number
line.

We use symbols to help us efficiently communicate relationships between numbers


15. Express equality with the on the number line. The symbols used to describe an equality relationship15
symbol =. If two quantities are between numbers follow:
not equal, use the symbol ≠.

1.1 Real Numbers and the Number Line 9


Chapter 1 Real Numbers and Their Operations

These symbols are used and interpreted in the following manner:

We next define symbols that denote an order relationship between real numbers.

These symbols allow us to compare two numbers. For example,

Since the graph of −120 is to the left of the graph of –10 on the number line, that
number is less than −10. We could write an equivalent statement as follows:

Similarly, since the graph of zero is to the right of the graph of any negative
number on the number line, zero is greater than any negative number.

1.1 Real Numbers and the Number Line 10


Chapter 1 Real Numbers and Their Operations

The symbols < and > are used to denote strict inequalities16, and the symbols ≤ and
≥ are used to denote inclusive inequalities17. In some situations, more than one
symbol can be correctly applied. For example, the following two statements are
both true:

In addition, the “or equal to” component of an inclusive inequality allows us to


correctly write the following:

The logical use of the word “or” requires that only one of the conditions need be
true: the “less than” or the “equal to.”

Example 2: Fill in the blank with <, =, or >: −2 ____ −12.

Solution: Use > because the graph of −2 is to the right of the graph of −12 on a
number line. Therefore, −2 > −12, which reads “negative two is greater than negative
twelve.”

16. Express ordering relationships


using the symbol < for “less Answer: −2 > −12
than” and > for “greater than.”

17. Use the symbol ≤ to express


quantities that are “less than
or equal to” and ≥ for
quantities that are “greater
than or equal to” each other.

1.1 Real Numbers and the Number Line 11


Chapter 1 Real Numbers and Their Operations

In this text, we will often point out the equivalent notation used to express
mathematical quantities electronically using the standard symbols available on a
keyboard. We begin with the equivalent textual notation for inequalities:

Many calculators, computer algebra systems, and programming languages use this
notation.

Opposites

The opposite18 of any real number a is −a. Opposite real numbers are the same
distance from the origin on a number line, but their graphs lie on opposite sides of
the origin and the numbers have opposite signs.

For example, we say that the opposite of 10 is −10.

Next, consider the opposite of a negative number. Given the integer −7, the integer
the same distance from the origin and with the opposite sign is +7, or just 7.

Therefore, we say that the opposite of −7 is −(−7) = 7. This idea leads to what is often
referred to as the double-negative property19. For any real number a,

18. Real numbers whose graphs


are on opposite sides of the
origin with the same distance
to the origin.

19. The opposite of a negative


number is positive: −(−a) = a.

1.1 Real Numbers and the Number Line 12


Chapter 1 Real Numbers and Their Operations

Example 3: What is the opposite of − 34?

Solution: Here we apply the double-negative property.

3
Answer: 4

Example 4: Simplify: − (− (4)).

Solution: Start with the innermost parentheses by finding the opposite of +4.

Answer: 4

Example 5: Simplify: −(−(−2)).

Solution: Apply the double-negative property starting with the innermost


parentheses.

1.1 Real Numbers and the Number Line 13


Chapter 1 Real Numbers and Their Operations

Answer: −2

Tip

If there is an even number of consecutive negative signs, then the result is


positive. If there is an odd number of consecutive negative signs, then the
result is negative.

Try this! Simplify: − (− (− (5))).

Answer: −5

Video Solution

(click to see video)


Absolute Value

The absolute value20 of a real number a, denoted |a|, is defined as the distance
between zero (the origin) and the graph of that real number on the number line.
Since it is a distance, it is always positive. For example,

Both 4 and −4 are four units from the origin, as illustrated below:

20. The absolute value of a number


is the distance from the graph
of the number to zero on a
number line.

1.1 Real Numbers and the Number Line 14


Chapter 1 Real Numbers and Their Operations

Example 6: Simplify:

a. |−12|

b. |12|

Solution: Both −12 and 12 are twelve units from the origin on a number line.
Therefore,

Answers: a. 12; b. 12

Also, it is worth noting that

The absolute value can be expressed textually using the notation abs(a). We often
encounter negative absolute values, such as − |3| or −abs(3). Notice that the
negative sign is in front of the absolute value symbol. In this case, work the absolute
value first and then find the opposite of the result.

Try not to confuse this with the double-negative property, which states that
−(−7) = +7.

Example 7: Simplify: −|| − (−7)||.

1.1 Real Numbers and the Number Line 15


Chapter 1 Real Numbers and Their Operations

Solution: First, find the opposite of −7 inside the absolute value. Then find the
opposite of the result.

Answer: −7

At this point, we can determine what real numbers have a particular absolute value.
For example,

Think of a real number whose distance to the origin is 5 units. There are two

origin, namely, {±5}. The symbol (±) is read “plus or minus” and indicates that
solutions: the distance to the right of the origin and the distance to the left of the

there are two answers, one positive and one negative.

Now consider the following:

Here we wish to find a value for which the distance to the origin is negative. Since
negative distance is not defined, this equation has no solution. If an equation has no
solution, we say the solution is the empty set: Ø.

1.1 Real Numbers and the Number Line 16


Chapter 1 Real Numbers and Their Operations

KEY TAKEAWAYS

• Any real number can be associated with a point on a line.


• Create a number line by first identifying the origin and marking off a
scale appropriate for the given problem.
• Negative numbers lie to the left of the origin and positive numbers lie to
the right.
• Smaller numbers always lie to the left of larger numbers on the number
line.
• The opposite of a positive number is negative and the opposite of a
negative number is positive.
• The absolute value of any real number is always positive because it is
defined to be the distance from zero (the origin) on a number line.
• The absolute value of zero is zero.

1.1 Real Numbers and the Number Line 17


Chapter 1 Real Numbers and Their Operations

TOPIC EXERCISES

Part A: Real Numbers

Use set notation to list the described elements.

1. The hours on a clock.

2. The days of the week.

3. The first ten whole numbers.

4. The first ten natural numbers.

5. The first five positive even integers.

6. The first five positive odd integers.

Determine whether the following real numbers are integers, rational, or irrational.

1
7. 2

8. −3

9. 4.5

10. −5

⎯⎯⎯⎯
11. 0.36

⎯⎯
12. 0.3

13. 1.001000100001 …

⎯⎯⎯⎯⎯⎯
14. 1.001

15. e = 2.71828 …
⎯⎯
16. √ 7 = 2.645751 …

1.1 Real Numbers and the Number Line 18


Chapter 1 Real Numbers and Their Operations

17. −7

18. 3.14

22
19. 7

20. 1.33

21. 0

22. 8,675,309

True or false.

23. All integers are rational numbers.

24. All integers are whole numbers.

25. All rational numbers are whole numbers.

26. Some irrational numbers are rational.

27. All terminating decimal numbers are rational.

28. All irrational numbers are real.

Part B: Real Number Line

Choose an appropriate scale and graph the following sets of real numbers on a
number line.

29. {−3, 0 3}

30. {−2, 2, 4, 6, 8, 10}

31. {−2,
3}
1 2 5
− 3
, 3
,

32. {− , 2}
5 1 1
2
, − 2
, 0, 2

1.1 Real Numbers and the Number Line 19


Chapter 1 Real Numbers and Their Operations

33. {− 7 , 1}
5 2
, 0, 7

34. { – 5, – 2, – 1, 0}

35. { − 3, − 2, 0, 2, 5}

36. {−2.5, −1.5, 0, 1, 2.5}

37. {0, 0.3, 0.6, 0.9, 1.2}

38. {−10, 30, 50}

39. {−6, 0, 3, 9, 12}

40. {−15, −9, 0, 9, 15}

Part C: Ordering Real Numbers

Fill in the blank with <, =, or >.

41. −7 ___ 0

42. 30 ___ 2

43. 10 ___ −10

44. −150 ___ −75

45. −0.5 ___ −1.5

46. 0 ___ 0

47. −500 ___ 200

48. −1 ___ −200

49. −10 ___ −10

50. −40 ___ −41

1.1 Real Numbers and the Number Line 20


Chapter 1 Real Numbers and Their Operations

True or false.

51. 5 ≠7

52. 4 =5

53. 1 ≠1

54. −5 > −10

55. 4 ≤4

56. −12 ≥0

57. −10 = −10

58. 3 >3

59. −1000 < −20

60. 0 =0

61. List three integers less than −5.

62. List three integers greater than −10.

63. List three rational numbers less than zero.

64. List three rational numbers greater than zero.

65. List three integers between −20 and −5.

66. List three rational numbers between 0 and 1.

Translate each statement into an English sentence.

67. 10 < 20

68. −50 ≤ −10

1.1 Real Numbers and the Number Line 21


Chapter 1 Real Numbers and Their Operations

69. −4 ≠0

70. 30 ≥ −1

71. 0 =0

72. e ≈ 2.718

Translate the following into a mathematical statement.

73. Negative seven is less than zero.

74. Twenty-four is not equal to ten.

75. Zero is greater than or equal to negative one.

76. Four is greater than or equal to negative twenty-one.

77. Negative two is equal to negative two.

78. Negative two thousand is less than negative one thousand.

Part D: Opposites

Simplify.

79. −(−9)

80. − (−
5)
3

81. −(10)

82. −(3)

83. −(5)

84. − (
4)
3

85. − (−1)

1.1 Real Numbers and the Number Line 22


Chapter 1 Real Numbers and Their Operations

86. − (− (−1))

87. − (− (1))

88. − (− (−3))

89. − (− (− (−11)))

1
90. What is the opposite of −
2

91. What is the opposite of π ?

92. What is the opposite −0.01?

93. Is the opposite of −12 smaller or larger than −11?

94. Is the opposite of 7 smaller or larger than −6?

Fill in the blank with <, =, or >.

95. −7 ___ −(−8)

96. 6 ___ −(6)

97. 13 ___ − (−12)

98. −(−5) ___ −(−2)

99. −100 ___ −(−(−50))

100. 44 ___ −(−44)

Part E: Absolute Value

Simplify.

101. |20|

102. |−20|

1.1 Real Numbers and the Number Line 23


Chapter 1 Real Numbers and Their Operations

103. |−33|

104. |−0.75||

105. |−
2|
| 5|

106. |
3|
|8|
107. |0|

108. |1|

109. − |12|

110. − |−20|

111. − |20|

112. − |−8|

113. − |7|

114. − |− |
3
| 16 |

115. − (− |
| 9 |)
8|

116. |−(−2)||

117. − |−(−3)||

118. −(− |5||)

119. − (− |−45||)

120. − |− (−21)||

121. abs(6)

1.1 Real Numbers and the Number Line 24


Chapter 1 Real Numbers and Their Operations

122. abs(−7)

123. −abs(5)

124. −abs(−19)

125. − (−abs(9))

126. −abs(−(−12))

Determine the unknown.

127. || ? || =9

128. || ? || = 15

129. || ? || =0

130. || ? || =1

131. || ? || = −8

132. || ? || = −20

133. |?| − 10 = −2

134. |?|| + 5 = 14

Fill in the blank with <, = , or >.

135. |−2| ____ 0

136. |−7| ____ |−10|

137. −10 ____ − |−2|

138. |−6|| ____ |−(−6)||

139. − |3| ____ |−(−5)||

1.1 Real Numbers and the Number Line 25


Chapter 1 Real Numbers and Their Operations

140. 0 ____ − |−(−4)||

Part F: Discussion Board Topics

141. Research and discuss the history of the number zero.

142. Research and discuss the various numbering systems throughout


history.

143. Research and discuss the definition and history of π .

144. Research the history of irrational numbers. Who is credited with


proving that the square root of 2 is irrational and what happened to him?

145. Research and discuss the history of absolute value.

146. Discuss the “just make it positive” definition of absolute value.

1.1 Real Numbers and the Number Line 26


Chapter 1 Real Numbers and Their Operations

ANSWERS

1: {1, 2, 3, 4, 5, 6, 7, 8, 9, 10, 11, 12}

3: {0, 1, 2, 3, 4, 5, 6, 7, 8, 9}

5: {2, 4, 6, 8, 10}

7: Rational

9: Rational

11: Rational

13: Irrational

15: Irrational

17: Integer, Rational

19: Rational

21: Integer, Rational

23: True

25: False

27: True

29:

31:

1.1 Real Numbers and the Number Line 27


Chapter 1 Real Numbers and Their Operations

33:

35:

37:

39:

41: <

43: >

45: >

47: <

49: =

51: True

53: False

55: True

57: True

59: True

61: −10, −7, −6 (answers may vary)

1.1 Real Numbers and the Number Line 28


Chapter 1 Real Numbers and Their Operations

63: −1, −2/3, −1/3 (answers may vary)

65: −15, −10, −7 (answers may vary)

67: Ten is less than twenty.

69: Negative four is not equal to zero.

71: Zero is equal to zero.

73: −7 <0

75: 0 ≥ −1

77: −2 = −2

79: 9

81: −10

83: −5

85: 1

87: 1

89: 11

91: −π

93: Larger

95: <

97: >

99: <

101: 20

1.1 Real Numbers and the Number Line 29


Chapter 1 Real Numbers and Their Operations

103: 33

105: 2/5

107: 0

109: −12

111: −20

113: −7

115: 8/9

117: −3

119: 45

121: 6

123: −5

125: 9

127: ±9

129: 0

131: Ø, No solution

133: ±8

135: >

137: <

139: <

1.1 Real Numbers and the Number Line 30


Chapter 1 Real Numbers and Their Operations

1.2 Adding and Subtracting Integers

LEARNING OBJECTIVES

1. Add and subtract signed integers.


2. Translate English sentences involving addition and subtraction into
mathematical statements.
3. Calculate the distance between two numbers on a number line.

Addition and Subtraction (+, −)

Visualize adding 3 + 2 on the number line by moving from zero three units to the
right then another two units to the right, as illustrated below:

The illustration shows that 3 + 2 = 5. Similarly, visualize adding two negative


numbers (−3) + (−2) by first moving from the origin three units to the left and then
moving another two units to the left.

In this example, the illustration shows (−3) + (−2) = −5, which leads to the following
two properties of real numbers.

Next, we will explore addition of numbers with unlike signs. To add 3 + (−7), first
move from the origin three units to the right, then move seven units to the left as
shown:

31
Chapter 1 Real Numbers and Their Operations

In this case, we can see that adding a negative number is equivalent to subtraction:

It is tempting to say that a positive number plus a negative number is negative, but
that is not always true: 7 + (−3) = 7 − 3 = 4. The result of adding numbers with
unlike signs may be positive or negative. The sign of the result is the same as the
sign of the number with the greatest distance from the origin. For example, the
following results depend on the sign of the number 12 because it is farther from
zero than 5:

Example 1: Simplify: 14 + (−25).

Solution: Here −25 is the greater distance from the origin. Therefore, the result is
negative.

Answer: −11

1.2 Adding and Subtracting Integers 32


Chapter 1 Real Numbers and Their Operations

Given any real numbers a, b, and c, we have the following properties of addition:

Additive identity property21: a + 0 = 0 + a = a

Additive inverse property22: a + (−a) = (−a) + a = 0

Associative property23: (a + b) + c = a + (b + c)

Commutative property24: a+b=b+a

Example 2: Simplify:

a. 5 + 0

b. 10 + (−10)

Solution:

a. Adding zero to any real number results in the same real number.

21. Given any real number a,


a+0=0+a=a.
22. Given any real number a,
a + (−a) = (−a) + a = 0. b. Adding opposites results in zero.

(a + b) + c = a + (b + c).
23. Given real numbers a, b and c,

24. Given real numbers a and b,


a + b = b + a.

1.2 Adding and Subtracting Integers 33


Chapter 1 Real Numbers and Their Operations

Answers: a. 5; b. 0

Example 3: Simplify:

a. (3 + 7) + 4

b. 3 + (7 + 4)

Solution: Parentheses group the operations that are to be performed first.

a.

b.

These two examples both result in 14: changing the grouping of the numbers does
not change the result.

Answers: a. 14; b. 14

1.2 Adding and Subtracting Integers 34


Chapter 1 Real Numbers and Their Operations

At this point, we highlight the fact that addition is commutative: the order in which
we add does not matter and yields the same result.

On the other hand, subtraction is not commutative.

We will use these properties, along with the double-negative property for real
numbers, to perform more involved sequential operations. To simplify things, we
will make it a general rule to first replace all sequential operations with either
addition or subtraction and then perform each operation in order from left to right.

Example 4: Simplify: 4 − (−10) + (−5).

Solution: Replace the sequential operations and then perform them from left to
right.

Answer: 9

1.2 Adding and Subtracting Integers 35


Chapter 1 Real Numbers and Their Operations

Example 5: Simplify: −3 + (−8) − (−7).

Solution:

Answer: −4

Try this! Simplify: 12 − (−9) + (−6).

Answer: 15

Video Solution

(click to see video)

Often we find the need to translate English sentences involving addition and
subtraction to mathematical statements. Listed below are some key words that
translate to the given operation.

Key Words Operation

Sum, increased by, more than, plus, added to, total +

Difference, decreased by, subtracted from, less, minus −

Example 6: What is the difference of 7 and −3?

Solution: The key word “difference” implies that we should subtract the numbers.

1.2 Adding and Subtracting Integers 36


Chapter 1 Real Numbers and Their Operations

Answer: The difference of 7 and −3 is 10.

Example 7: What is the sum of the first five positive integers?

Solution: The initial key word to focus on is “sum”; this means that we will be
adding the five numbers. The first five positive integers are {1, 2, 3, 4, 5}. Recall that
0 is neither positive nor negative.

Answer: The sum of the first five positive integers is 15.

Example 8: What is 10 subtracted from the sum of 8 and 6?

Solution: We know that subtraction is not commutative; therefore, we must take


care to subtract in the correct order. First, add 8 and 6 and then subtract 10 as
follows:

a mathematical statement in the order it appears. In other words, 10 − (8 + 6)


It is important to notice that the phrase “10 subtracted from” does not translate to

would be an incorrect translation and leads to an incorrect answer. After


translating the sentence, perform the operations.

1.2 Adding and Subtracting Integers 37


Chapter 1 Real Numbers and Their Operations

Answer: Ten subtracted from the sum of 8 and 6 is 4.

Distance on a Number Line

One application of the absolute value is to find the distance between any two points
on a number line. For real numbers a and b, the distance formula for a number
line25 is given as,

Example 9: Determine the distance between 2 and 7 on a number line.

Solution: On the graph we see that the distance between the two given integers is 5
units.

Using the distance formula we obtain the same result.

25. The distance between any two


real numbers a and b on a
number line can be calculated
using the formula
d = |b − a||.

1.2 Adding and Subtracting Integers 38


Chapter 1 Real Numbers and Their Operations

Answer: 5 units

Example 10: Determine the distance between −4 and 7 on a number line.

Solution: Use the distance formula for a number line d = |b − a||, where a = −4
and b = 7.

Answer: 11 units

It turns out that it does not matter which points are used for a and b; the absolute
value always ensures a positive result.

Using a = −4 and b = 7 Using a = 7 and b = −4

d = |7 − (−4)||
d = |−4 − 7|
= |7 + 4||
= |−11|
= |11|
= 11
= 11

1.2 Adding and Subtracting Integers 39


Chapter 1 Real Numbers and Their Operations

Try this! Determine the distance between −12 and −9 on the number line.

Answer: 3

Video Solution

(click to see video)

KEY TAKEAWAYS

• A positive number added to a positive number is positive. A negative


number added to a negative number is negative.
• The sign of a positive number added to a negative number is the same as
the sign of the number with the greatest distance from the origin.
• Addition is commutative and subtraction is not.
• When simplifying, it is a best practice to first replace sequential
operations and then work the operations of addition and subtraction
from left to right.
• The distance between any two numbers on a number line is the absolute
value of their difference. In other words, given any real numbers a and
b, use the formula d = |b − a||to calculate the distance d between
them.

1.2 Adding and Subtracting Integers 40


Chapter 1 Real Numbers and Their Operations

TOPIC EXERCISES

Part A: Addition and Subtraction

Add and subtract.

1. 24 + (−18)

2. 9 + (−11)

3. −31 +5

4. −12 + 15

5. −30 + (−8)

6. −50 + (−25)

7. −7 + (−7)

8. −13 − (−13)

9. 8 − 12 + 5

10. −3 −7+4

11. −1 −2−3−4

12. 6 − (−5) + (−10) − 14

13. −5 + (−3) − (−7)

14. 2 − 7 + (−9)

15. −30 + 20 − 8 − (−18)

16. 10 − (−12) + (−8) − 20

17. 5 − (−2) + (−6)

1.2 Adding and Subtracting Integers 41


Chapter 1 Real Numbers and Their Operations

18. −3 + (−17) − (−13)

19. −10 + (−12) − (−20)

20. −13 + (−5) − (−25)

21. 20 − (−4) − (−5)

22. 17 + (−12) − (−2)

Translate each sentence to a mathematical statement and then simplify.

23. Find the sum of 3, 7, and −8.

24. Find the sum of −12, −5, and 7.

25. Determine the sum of the first ten positive integers.

26. Determine the sum of the integers in the set {−2, −1, 0, 1, 2}.

27. Find the difference of 10 and 6.

28. Find the difference of 10 and −6.

29. Find the difference of −16 and −5.

30. Find the difference of −19 and 7.

31. Subtract 12 from 10.

32. Subtract −10 from −20.

33. Subtract 5 from −31.

34. Subtract −3 from 27.

35. Two less than 8.

36. Five less than −10.

1.2 Adding and Subtracting Integers 42


Chapter 1 Real Numbers and Their Operations

37. Subtract 8 from the sum of 4 and 7.

38. Subtract −5 from the sum of 10 and −3.

39. Subtract 2 from the difference of 8 and 5.

40. Subtract 6 from the difference of −1 and 7.

41. Mandy made a $200 deposit into her checking account on Tuesday. She
then wrote 4 checks for $50.00, $125.00, $60.00, and $45.00. How much more
than her deposit did she spend?

42. The quarterback ran the ball three times in last Sunday’s football game.
He gained 7 yards on one run but lost 3 yards and 8 yards on the other two.
What was his total yardage running for the game?

43. The revenue for a local photographer for the month is $1,200. His costs
include a studio rental of $600, props costing $105, materials fees of $135,
and a make-up artist who charges $120. What is his total profit for the
month?

44. An airplane flying at 30,000 feet lost 2,500 feet in altitude and then rose
1,200 feet. What is the new altitude of the plane?

45. The temperature was 22° at 6:00 p.m. and dropped 26° by midnight. What
was the temperature at midnight?

46. A nurse has 30 milliliters of saline solution but needs 75 milliliters of the
solution. How much more does she need?

47. The width of a rectangle is 2 inches less than its length. If the length
measures 16 inches, determine the width.

48. The base of a triangle is 3 feet shorter than its height. If the height
measures 5 feet, find the length of the base.

Part B: Distance on a Number Line

Find the distance between the given numbers on a number line.

49. −3 and 12

1.2 Adding and Subtracting Integers 43


Chapter 1 Real Numbers and Their Operations

50. 8 and −13

51. −25 and −10

52. −100 and −130

53. −7 and −20

54. 0 and −33

55. −10 and 10

56. −36 and 36

57. The coldest temperature on earth, −129°F, was recorded in 1983 at


Vostok Station, Antarctica. The hottest temperature on earth, 136°F, was
recorded in 1922 at Al ’Aziziyah, Libya. Calculate earth’s temperature range.

58. The daily high temperature was recorded as 91°F and the low was
recorded as 63°F. What was the temperature range for the day?

59. A student earned 67 points on his lowest test and 87 points on his best.
Calculate his test score range.

60. On a busy day, a certain website may have 12,500 hits. On a slow day, it
may have as few as 750 hits. Calculate the range of the number of hits.

Part C: Discussion Board Topics

61. Share an example of adding signed numbers in a real-world application.

62. Demonstrate the associative property of addition with any three real
numbers.

63. Show that subtraction is not commutative.

1.2 Adding and Subtracting Integers 44


Chapter 1 Real Numbers and Their Operations

ANSWERS

1: 6

3: −26

5: −38

7: −14

9: 1

11: −10

13: −1

15: 0

17: 1

19: −2

21: 29

23: 2

25: 55

27: 4

29: −11

31: −2

33: −36

35: 6

37: 3

1.2 Adding and Subtracting Integers 45


Chapter 1 Real Numbers and Their Operations

39: 1

41: $80

43: $240

45: −4°

47: 14 inches

49: 15 units

51: 15 units

53: 13 units

55: 20 units

57: 265°F

59: 20 points

1.2 Adding and Subtracting Integers 46


Chapter 1 Real Numbers and Their Operations

1.3 Multiplying and Dividing Integers

LEARNING OBJECTIVES

1. Multiply and divide signed integers.


2. Translate English sentences involving multiplication and division into
mathematical statements.
3. Determine the prime factorization of composite numbers.
4. Interpret the results of quotients involving zero.

Multiplication and Division

We begin with a review of what it means to multiply and divide signed numbers.
The result of multiplying real numbers is called the product26 and the result of
dividing is called the quotient27. Recall that multiplication is equivalent to adding:

Clearly, the product of two positive numbers is positive. Similarly, the product of a
positive number and negative number can be written as shown:

We see that the product of a positive number and a negative number is negative.
Next, explore the results of multiplying two negative numbers. Consider the
products in the following illustration and try to identify the pattern:

26. The result of multiplying.

27. The result after dividing.

47
Chapter 1 Real Numbers and Their Operations

This shows that the product of two negative numbers is positive. To summarize,

The rules for division are the same because division can always be rewritten as
multiplication:

The rules for multiplication and division should not be confused with the fact that
the sum of two negative numbers is negative.

Example 1: Simplify:

a. (−3) + (−5)

b. (−3) (−5)

1.3 Multiplying and Dividing Integers 48


Chapter 1 Real Numbers and Their Operations

Solution: Here we add and multiply the same two negative numbers.

a. The result of adding two negative numbers is negative.

b. The result of multiplying two negative numbers is positive.

Answers: a. −8; b. 15

Given any real numbers a, b, and c, we have the following properties of


multiplication:

Zero factor property28: a⋅0=0⋅a=0

Multiplicative identity property29: a⋅1=1⋅a=a

28. Given any real number a, Associative property30: (a ⋅ b) ⋅ c = a ⋅ (b ⋅ c)


a ⋅ 0 = 0 ⋅ a = 0.
29. Given any real number a,
a ⋅ 1 = 1 ⋅ a = a.
Commutative property31: a⋅b=b⋅a
30. Given any real numbers a, b,

(a ⋅ b) ⋅ c = a ⋅ (b ⋅ c) .
and c,

31. Given any real numbers a and


b, a ⋅ b = b ⋅ a.

1.3 Multiplying and Dividing Integers 49


Chapter 1 Real Numbers and Their Operations

Example 2: Simplify:

a. 5 ⋅ 0

b. 10 ⋅ 1

Solution:

a. Multiplying by zero results in zero.

b. Multiplying any real number by one results in the same real number.

Answers: a. 0; b. 10

Example 3: Simplify:

a. (3 ⋅ 7) ⋅ 2

b. 3 ⋅ (7 ⋅ 2)

Solution:

a.

1.3 Multiplying and Dividing Integers 50


Chapter 1 Real Numbers and Their Operations

b.

The value of each expression is 42. Changing the grouping of the numbers does not
change the result.

Answers: a. 42; b. 42

At this point, we highlight that multiplication is commutative: the order in which


we multiply does not matter and yields the same result.

On the other hand, division is not commutative.

Use these properties to perform sequential operations involving multiplication and


division. When doing so, it is important to perform these operations in order from
left to right.

1.3 Multiplying and Dividing Integers 51


Chapter 1 Real Numbers and Their Operations

Example 4: Simplify: 3 (−2) (−5) (−1).

Solution: Multiply two numbers at a time as follows:

Answer: −30

Because multiplication is commutative, the order in which we multiply does not


affect the final answer. When sequential operations involve multiplication and
division, order does matter; hence we must work the operations from left to right to
obtain a correct result.

Example 5: Simplify: 10 ÷ (−2) (−5).

Solution: Perform the division first; otherwise, the result will be incorrect.

Answer: 25

1.3 Multiplying and Dividing Integers 52


Chapter 1 Real Numbers and Their Operations

Notice that the order in which we multiply and divide does affect the final result.
Therefore, it is important to perform the operations of multiplication and division
as they appear from left to right.

Example 6: Simplify: −6 (3) ÷ (−2) (−3).

Solution: Work the operations one at a time from left to right.

Try this! Simplify: −5 ÷ 5 ⋅ 2 (−3).

Answer: 6

Video Solution

(click to see video)

Within text-based applications, the symbol used for multiplication is the asterisk32
(*) and the symbol used for division is the forward slash (/).

32. The symbol (*) that indicates


multiplication within text-
based applications.

33. Integers that are divisible by


The set of even integers33 is the set of all integers that are evenly divisible by 2. We
two or are multiples of two. can also obtain the set of even integers by multiplying each integer by 2.

1.3 Multiplying and Dividing Integers 53


Chapter 1 Real Numbers and Their Operations

The set of odd integers34 is the set of all integers that are not evenly divisible by 2.

A prime number35 is an integer greater than 1 that is divisible only by 1 and itself.
The smallest prime number is 2 and the rest are necessarily odd.

Any integer greater than 1 that is not prime is called a composite number36 and
can be written as a product of primes. When a composite number, such as 30, is
written as a product, 30 = 2 ⋅ 15, we say that 2 ⋅ 15 is a factorization37 of 30 and
that 2 and 15 are factors38. Note that factors divide the number evenly. We can
continue to write composite factors as products until only a product of primes
remains.

34. Integers that are not divisible


by 2.

35. Integers greater than 1 that are


divisible only by 1 and itself. The prime factorization39 of 30 is 2 ⋅ 3 ⋅ 5.

36. Integers greater than 1 that are


not prime.

37. Any combination of factors,


multiplied together, resulting
Example 7: Determine the prime factorization of 70.
in the product.

38. Any of the numbers or


expressions that form a Solution: Begin by writing 70 as a product with 2 as a factor. Then express any
product. composite factor as a product of prime numbers.
39. The unique factorization of a
natural number written as a
product of primes.

1.3 Multiplying and Dividing Integers 54


Chapter 1 Real Numbers and Their Operations

Since the prime factorization is unique, it does not matter how we choose to
initially factor the number because the end result is the same.

Answer: The prime factorization of 70 is 2 ⋅ 5 ⋅ 7.

Some tests (called divisibility tests) useful for finding prime factors of composite
numbers follow:

1. If the integer is even, then 2 is a factor.


2. If the sum of the digits is evenly divisible by 3, then 3 is a factor.
3. If the last digit is a 5 or 0, then 5 is a factor.

Often we find the need to translate English sentences that include multiplication
and division terms to mathematical statements. Listed below are some key words
that translate to the given operation.

Key Words Operation

Product, multiplied by, of, times * or ⋅

Quotient, divided by, ratio, per / or ÷

1.3 Multiplying and Dividing Integers 55


Chapter 1 Real Numbers and Their Operations

Example 8: Calculate the quotient of 20 and −10.

Solution: The key word “quotient” implies that we should divide.

Answer: The quotient of 20 and −10 is −2.

Example 9: What is the product of the first three positive even integers?

Solution: The first three positive even integers are {2, 4, 6} and the key word
“product” implies that we should multiply.

Answer: The product of the first three positive even integers is 48.

Example 10: Joe is able to drive 342 miles on 18 gallons of gasoline. How many miles
per gallon of gas is this?

Solution: The key word “per” indicates that we must divide the number of miles
driven by the number of gallons used:

Answer: Joe gets 19 miles per gallon from his vehicle.

1.3 Multiplying and Dividing Integers 56


Chapter 1 Real Numbers and Their Operations

In everyday life, we often wish to use a single value that typifies a set of values. One
way to do this is to use what is called the arithmetic mean40 or average41. To
calculate an average, divide the sum of the values in the set by the number of values
in that set.

Example 11: A student earns 75, 86, and 94 on his first three exams. What is the
student’s test average?

Solution: Add the scores and divide the sum by 3.

Answer: The student’s test average is 85.

Zero and Division

Recall the relationship between multiplication and division:

In this case, the dividend42 12 is evenly divided by the divisor43 6 to obtain the
40. A numerical value that typifies quotient, 2. It is true in general that if we multiply the divisor by the quotient we
a set of numbers. It is obtain the dividend. Now consider the case where the dividend is zero and the
calculated by adding up the divisor is nonzero:
numbers in the set and
dividing by the number of
elements in the set.

41. Used in reference to the


arithmetic mean.

42. The numerator of a quotient.

43. The denominator of a quotient.

1.3 Multiplying and Dividing Integers 57


Chapter 1 Real Numbers and Their Operations

This demonstrates that zero divided by any nonzero real number must be zero. Now
consider a nonzero number divided by zero:

The zero-factor property of multiplication states that any real number times 0 is 0.
We conclude that there is no real number such that 0⋅? = 12 and thus, the
quotient is left undefined44. Try 12 ÷ 0 on a calculator. What does it say? For our
purposes, we will simply write “undefined.”

To summarize, given any real number a ≠ 0, then

We are left to consider the case where the dividend and divisor are both zero.

Here any real number seems to work. For example, 0 ⋅ 5 = 0 and 0 ⋅ 3 = 0.


Therefore, the quotient is uncertain or indeterminate45.

In this course, we state that 0 ÷ 0 is undefined.

44. A quotient such as 50 , which is


left without meaning and is not
assigned an interpretation.

45. A quotient such as 00 , which is a


quantity that is uncertain or
ambiguous.

1.3 Multiplying and Dividing Integers 58


Chapter 1 Real Numbers and Their Operations

KEY TAKEAWAYS

• A positive number multiplied by a negative number is negative. A


negative number multiplied by a negative number is positive.
• Multiplication is commutative and division is not.
• When simplifying, work the operations of multiplication and division in
order from left to right.
• Even integers are numbers that are evenly divisible by 2 or multiples of
2, and all other integers are odd.
• A prime number is an integer greater than 1 that is divisible only by 1
and itself.
• Composite numbers are integers greater than 1 that are not prime.
Composite numbers can be written uniquely as a product of primes.
• The prime factorization of a composite number is found by continuing
to divide it into factors until only a product of primes remains.
• To calculate an average of a set of numbers, divide the sum of the values
in the set by the number of values in the set.
• Zero divided by any nonzero number is zero. Any number divided by
zero is undefined.

1.3 Multiplying and Dividing Integers 59


Chapter 1 Real Numbers and Their Operations

TOPIC EXERCISES

Part A: Multiplication and Division

Multiply and divide.

1. 5(−7)

2. −3(−8)

3. 2 (−4) (−9)

4. −3 ⋅2⋅5

5. −12 (3) (0)

6. 0 (−12) (−5)

7. (−1) (−1) (−1) (−1)

8. (−1) (−1) (−1)

9. −100 ÷ 25

10. 25 ÷ 5(−5)

11. −15(−2) ÷ 10(−3)

12. −5 ⋅ 10 ÷ 2(−5)

13. (−3) (25) ÷ (−5)

14. 6*(−3)/(−9)

15. 20/(−5)*2

16. −50/2*5

17. Determine the product of 11 and −3.

1.3 Multiplying and Dividing Integers 60


Chapter 1 Real Numbers and Their Operations

18. Determine the product of −7 and −22.

19. Find the product of 5 and −12.

20. Find the quotient of negative twenty-five and five.

21. Determine the quotient of −36 and 3.

22. Determine the quotient of 26 and −13.

23. Calculate the product of 3 and −8 divided by −2.

24. Calculate the product of −1 and −3 divided by 3.

25. Determine the product of the first three positive even integers.

26. Determine the product of the first three positive odd integers.

Determine the prime factorization of the following integers.

27. 105

28. 78

29. 138

30. 154

31. 165

32. 330

Calculate the average of the numbers in each of the following sets.

33. {50, 60, 70}

34. {9, 12, 30}

35. {3, 9, 12, 30, 36}

1.3 Multiplying and Dividing Integers 61


Chapter 1 Real Numbers and Their Operations

36. {72, 84, 69, 71}

37. The first four positive even integers.

38. The first four positive odd integers.

The distance traveled D is equal to the average rate r times the time traveled t at
that rate: D = rt. Determine the distance traveled given the rate and the time.

39. 60 miles per hour for 3 hours

40. 55 miles per hour for 3 hours

41. 15 miles per hour for 5 hours

42. 75 feet per second for 5 seconds

43. 60 kilometers per hour for 10 hours

44. 60 meters per second for 30 seconds

45. A student club ran a fund-raiser in the quad selling hot dogs. The
students sold 122 hot dog meals for $3.00 each. Their costs included $50.00
for the hot dogs and buns, $25.00 for individually wrapped packages of
chips, and $35.00 for the sodas. What was their profit?

46. A 230-pound man loses 4 pounds each week for 8 weeks. How much does
he weigh at the end of 8 weeks?

47. Mary found that she was able to drive 264 miles on 12 gallons of gas. How
many miles per gallon does her car get?

48. After filling his car with gasoline, Bill noted that his odometer reading
was 45,346 miles. After using his car for a week, he filled up his tank with 14
gallons of gas and noted that his odometer read 45,724 miles. In that week,
how many miles per gallon did Bill’s car get?

Part B: Zero and Division with Mixed Practice

Perform the operations.

1.3 Multiplying and Dividing Integers 62


Chapter 1 Real Numbers and Their Operations

49. 0 ÷9

50. 15 ÷0

51. 4(−7) ÷0

52. 7 (0) ÷ (−15)

53. −5(0) ÷ 9(0)

54. 5 ⋅ 2 (−3) (−5)

55. −8 − 5 + (−13)

56. −4(−8) ÷ 16(−2)

57. 50 ÷ (−5) ÷ (−10)

58. 49 ÷ 7 ÷ (−1)

59. 3 ⋅ 4 ÷ 12

60. 0 − (−8) − 12

61. −8 ⋅ 4(−3) ÷ 2

62. 0/(−3*8*5)

63. (−4*3)/(2*(−3))

64. −16/(−2*2)*3

65. −44/11*2

66. −5*3/(−15)

67. 4*3*2/6

68. −6*7/( −2)

1.3 Multiplying and Dividing Integers 63


Chapter 1 Real Numbers and Their Operations

69. During 5 consecutive winter days, the daily lows were −7°, −3°, 0°, −5°, and
−10°. Calculate the average low temperature.

70. On a very cold day the temperature was recorded every 4 hours with the
following results: −16°, −10°, 2°, 6°, −5°, and −13°. Determine the average
temperature.

71. A student earns 9, 8, 10, 7, and 6 points on the first 5 chemistry quizzes.
What is her quiz average?

72. A website tracked hits on its homepage over the Thanksgiving holiday.
The number of hits for each day from Thursday to Sunday was 12,250; 4,400;
7,750; and 10,200, respectively. What was the average number of hits per day
over the holiday period?

Part C: Discussion Board Topics

73. Demonstrate the associative property of multiplication with any three


real numbers.

74. Show that division is not commutative.

75. Discuss the importance of working multiplication and division


operations from left to right. Make up an example where order does matter
and share the solution.

76. Discuss division involving 0. With examples, explain why the result is
sometimes 0 and why it is sometimes undefined.

77. Research and discuss the fundamental theorem of arithmetic.

78. Research and discuss other divisibility tests. Provide an example for each
test.

79. The arithmetic mean is one way to typify a set of values. Research other
methods used to typify a set of values.

1.3 Multiplying and Dividing Integers 64


Chapter 1 Real Numbers and Their Operations

ANSWERS

1: −35

3: 72

5: 0

7: 1

9: −4

11: −9

13: 15

15: −8

17: −33

19: −60

21: −12

23: 12

25: 48

27: 3 ⋅5⋅7

29: 2 ⋅ 3 ⋅ 23

31: 3 ⋅ 5 ⋅ 11

33: 60

35: 18

37: 5

1.3 Multiplying and Dividing Integers 65


Chapter 1 Real Numbers and Their Operations

39: 180 miles

41: 75 miles

43: 600 kilometers

45: $256.00

47: 22 miles per gallon

49: 0

51: Undefined

53: 0

55: −26

57: 1

59: 1

61: 48

63: 2

65: −8

67: 4

69: −5°

71: 8 points

1.3 Multiplying and Dividing Integers 66


Chapter 1 Real Numbers and Their Operations

1.4 Fractions

LEARNING OBJECTIVES

1. Reduce a fraction to lowest terms.


2. Multiply and divide fractions.
3. Add and subtract fractions.

Reducing

A fraction46 is a real number written as a quotient, or ratio47, of two integers a and


b, where b ≠ 0.

The integer above the fraction bar is called the numerator48 and the integer below
is called the denominator49. The numerator is often called the “part” and the
denominator is often called the “whole.” Equivalent fractions50 are two equal
ratios expressed using different numerators and denominators. For example,

46. A rational number written as a


quotient of two integers: ba ,
where b is nonzero.

47. Relationship between two


Fifty parts out of 100 is the same ratio as 1 part out of 2 and represents the same
numbers or quantities usually
expressed as a quotient. real number. Consider the following factorizations of 50 and 100:

48. The number above the fraction


bar.

49. The number below the fraction


bar.

50. Two equal fractions expressed


using different numerators and
denominators.
The numbers 50 and 100 share the factor 25. A shared factor is called a common
50
51. A factor that is shared by more factor51. We can rewrite the ratio 100 as follows:
than one real number.

67
Chapter 1 Real Numbers and Their Operations

Making use of the multiplicative identity property and the fact that 25
25
= 1, we have

Dividing 25
25
and replacing this factor with a 1 is called canceling52. Together, these
basic steps for finding equivalent fractions define the process of reducing53. Since
factors divide their product evenly, we achieve the same result by dividing both the
numerator and denominator by 25 as follows:

Finding equivalent fractions where the numerator and denominator have no


common factor other than 1 is called reducing to lowest terms54. When learning
how to reduce to lowest terms, it is helpful to first rewrite the numerator and
denominator as a product of primes and then cancel. For example,

52. The process of dividing out


common factors in the
numerator and the
denominator.

53. The process of finding


equivalent fractions by
dividing the numerator and the
denominator by common
factors.

54. Finding equivalent fractions We achieve the same result by dividing the numerator and denominator by the
where the numerator and the greatest common factor (GCF)55. The GCF is the largest number that divides both
denominator share no common the numerator and denominator evenly. One way to find the GCF of 50 and 100 is to
integer factor other than 1.
list all the factors of each and identify the largest number that appears in both lists.
55. The largest shared factor of Remember, each number is also a factor of itself.
any number of integers.

1.4 Fractions 68
Chapter 1 Real Numbers and Their Operations

Common factors are listed in bold, and we see that the greatest common factor is
50. We use the following notation to indicate the GCF of two numbers: GCF(50, 100) =
50. After determining the GCF, reduce by dividing both the numerator and the
denominator as follows:

Example 1: Reduce to lowest terms: 105


300
.

Solution: Rewrite the numerator and denominator as a product of primes and then
cancel.

Alternatively, we achieve the same result if we divide both the numerator and
denominator by the GCF(105, 300). A quick way to find the GCF of the two numbers
requires us to first write each as a product of primes. The GCF is the product of all
the common prime factors.

1.4 Fractions 69
Chapter 1 Real Numbers and Their Operations

In this case, the common prime factors are 3 and 5 and the greatest common factor
of 105 and 300 is 15.

7
Answer: 20

Try this! Reduce to lowest terms: 32


96
.

1
Answer: 3

Video Solution

(click to see video)

An improper fraction56 is one where the numerator is larger than the


denominator. A mixed number57 is a number that represents the sum of a whole
number and a fraction. For example, 5 12 is a mixed number that represents the sum
5 + 12. Use long division to convert an improper fraction to a mixed number; the
remainder is the numerator of the fractional part.

56. A fraction where the Example 2: Write 23


5
as a mixed number.
numerator is larger than the
denominator.
Solution: Notice that 5 divides into 23 four times with a remainder of 3.
57. A number that represents the
sum of a whole number and a
fraction.

1.4 Fractions 70
Chapter 1 Real Numbers and Their Operations

We then can write

Note that the denominator of the fractional part of the mixed number remains the
same as the denominator of the original fraction.

Answer: 4 3
5

To convert mixed numbers to improper fractions, multiply the whole number by


the denominator and then add the numerator; write this result over the original
denominator.

Example 3: Write 3 57 as an improper fraction.

Solution: Obtain the numerator by multiplying 7 times 3 and then add 5.

1.4 Fractions 71
Chapter 1 Real Numbers and Their Operations

26
Answer: 7

It is important to note that converting to a mixed number is not part of the


reducing process. We consider improper fractions, such as 267, to be reduced to
lowest terms. In algebra it is often preferable to work with improper fractions,
although in some applications, mixed numbers are more appropriate.

Try this! Convert 10 12 to an improper fraction.

21
Answer: 2

Video Solution

(click to see video)


Multiplying and Dividing Fractions

In this section, assume that a, b, c, and d are all nonzero integers. The product of
two fractions is the fraction formed by the product of the numerators and the
product of the denominators. In other words, to multiply fractions, multiply the
numerators and multiply the denominators:

1.4 Fractions 72
Chapter 1 Real Numbers and Their Operations

Example 4: Multiply: 23 ⋅ 57.

Solution: Multiply the numerators and multiply the denominators.

Answer: 10
21

Example 5: Multiply: 59 (− 4 ).
1

Solution: Recall that the product of a positive number and a negative number is
negative.

5
Answer: − 36

Example 6: Multiply: 23 ⋅ 5 34.

Solution: Begin by converting 5 34 to an improper fraction.

1.4 Fractions 73
Chapter 1 Real Numbers and Their Operations

In this example, we noticed that we could reduce before we multiplied the


numerators and the denominators. Reducing in this way is called cross canceling58,
and can save time when multiplying fractions.

Answer: 3 56

Two real numbers whose product is 1 are called reciprocals59. Therefore, ab and b
a
are reciprocals because ab ⋅ ba = ab
ab
= .1For example,

Because their product is 1, 23 and 32 are reciprocals. Some other reciprocals are listed
below:

58. Cancelling common factors in This definition is important because dividing fractions requires that you multiply
the numerator and the the dividend by the reciprocal of the divisor.
denominator of fractions
before multiplying.

59. The reciprocal of a nonzero


number n is 1/n.

1.4 Fractions 74
Chapter 1 Real Numbers and Their Operations

Example 7: Divide: 23 ÷ 57.

Solution: Multiply 23 by the reciprocal of 57 .

14
Answer: 15

You also need to be aware of other forms of notation that indicate division: / and —.
For example,

Or

The latter is an example of a complex fraction60, which is a fraction whose


numerator, denominator, or both are fractions.
60. A fraction where the
numerator or denominator
consists of one or more
fractions.

1.4 Fractions 75
Chapter 1 Real Numbers and Their Operations

Note

Students often ask why dividing is equivalent to multiplying by the reciprocal


of the divisor. A mathematical explanation comes from the fact that the
product of reciprocals is 1. If we apply the multiplicative identity property and
multiply numerator and denominator by the reciprocal of the denominator,
then we obtain the following:

Before multiplying, look for common factors to cancel; this eliminates the need to
reduce the end result.

5
2
Example 8: Divide: 7
.
4

Solution:

1.4 Fractions 76
Chapter 1 Real Numbers and Their Operations

10
Answer: 7

When dividing by an integer, it is helpful to rewrite it as a fraction over 1.

Example 9: Divide: 23 ÷ 6.

Solution: Rewrite 6 as 61 and multiply by its reciprocal.

1.4 Fractions 77
Chapter 1 Real Numbers and Their Operations

1
Answer: 9

Also, note that we only cancel when working with multiplication. Rewrite any
division problem as a product before canceling.

Try this! Divide: 5 ÷ 2 35.

Answer: 1 12
13

Video Solution

(click to see video)


Adding and Subtracting Fractions

Negative fractions are indicated with the negative sign in front of the fraction bar,
in the numerator, or in the denominator. All such forms are equivalent and
interchangeable.

1.4 Fractions 78
Chapter 1 Real Numbers and Their Operations

Adding or subtracting fractions requires a common denominator61. In this section,


assume the common denominator c is a nonzero integer.

It is good practice to use positive common denominators by expressing negative


fractions with negative numerators. In short, avoid negative denominators.

3
Example 10: Subtract: 12
15
− 15 .

Solution: The two fractions have a common denominator 15. Therefore, subtract
the numerators and write the result over the common denominator:

3
Answer: 5

Most problems that you are likely to encounter will have unlike denominators62.
In this case, first find equivalent fractions with a common denominator before
adding or subtracting the numerators. One way to obtain equivalent fractions is to
61. A denominator that is shared divide the numerator and the denominator by the same number. We now review a
by more than one fraction. technique for finding equivalent fractions by multiplying the numerator and the
denominator by the same number. It should be clear that 5/5 is equal to 1 and that 1
62. Denominators of fractions that
are not the same. multiplied times any number is that number:

1.4 Fractions 79
Chapter 1 Real Numbers and Their Operations

We have equivalent fractions 12 = 105. Use this idea to find equivalent fractions with
a common denominator to add or subtract fractions. The steps are outlined in the
following example.

7 3
Example 11: Subtract: 15 − 10 .

Solution:

Step 1: Determine a common denominator. To do this, use the least common


multiple (LCM)63 of the given denominators. The LCM of 15 and 10 is indicated by
LCM(15, 10). Try to think of the smallest number that both denominators divide
into evenly. List the multiples of each number:

Common multiples are listed in bold, and the least common multiple is 30.

Step 2: Multiply the numerator and the denominator of each fraction by values that
result in equivalent fractions with the determined common denominator.

63. The smallest number that is


evenly divisible by a set of
numbers.

1.4 Fractions 80
Chapter 1 Real Numbers and Their Operations

Step 3: Add or subtract the numerators, write the result over the common
denominator and then reduce if possible.

1
Answer: 6

The least common multiple of the denominators is called the least common
denominator (LCD)64. Finding the LCD is often the difficult step. It is worth finding
because if any common multiple other than the least is used, then there will be
more steps involved when reducing.

5 1
Example 12: Add: 10 + 18 .

Solution: First, determine that the LCM(10, 18) is 90 and then find equivalent
fractions with 90 as the denominator.

64. The least common multiple of a


set of denominators.

1.4 Fractions 81
Chapter 1 Real Numbers and Their Operations

5
Answer: 9

2 5
Try this! Add: 30 + 21 .

32
Answer: 105

Video Solution

(click to see video)

Example 13: Simplify: 2 13 + 35 − 12.

Solution: Begin by converting 2 13 to an improper fraction.

1.4 Fractions 82
Chapter 1 Real Numbers and Their Operations

Answer: 2 13
30

In general, it is preferable to work with improper fractions. However, when the


original problem involves mixed numbers, if appropriate, present your answers as
mixed numbers. Also, mixed numbers are often preferred when working with
numbers on a number line and with real-world applications.

Try this! Subtract: 57 − 2 17.

Answer: −1 37

Video Solution

(click to see video)

1.4 Fractions 83
Chapter 1 Real Numbers and Their Operations

Example 14: How many 12 inch thick paperback books can be stacked to fit on a
shelf that is 1 12 feet in height?

Solution: First, determine the height of the shelf in inches. To do this, use the fact
that there are 12 inches in 1 foot and multiply as follows:

Next, determine how many notebooks will fit by dividing the height of the shelf by
the thickness of each book.

Answer: 36 books can be stacked on the shelf.

1.4 Fractions 84
Chapter 1 Real Numbers and Their Operations

KEY TAKEAWAYS

• Fractions are not unique; there are many ways to express the same ratio.
Find equivalent fractions by multiplying or dividing the numerator and
the denominator by the same real number.
• Equivalent fractions in lowest terms are generally preferred. It is a good
practice to always reduce.
• In algebra, improper fractions are generally preferred. However, in real-
life applications, mixed number equivalents are often preferred. We may
present answers as improper fractions unless the original question
contains mixed numbers, or it is an answer to a real-world or geometric
application.
• Multiplying fractions does not require a common denominator; multiply
the numerators and multiply the denominators to obtain the product. It
is a best practice to cancel any common factors in the numerator and
the denominator before multiplying.
• Reciprocals are rational numbers whose product is equal to 1. Given a
fraction ba , its reciprocal is ba .
• Divide fractions by multiplying the dividend by the reciprocal of the
divisor. In other words, multiply the numerator by the reciprocal of the
denominator.
• Rewrite any division problem as a product before canceling.
• Adding or subtracting fractions requires a common denominator. When
the denominators of any number of fractions are the same, simply add
or subtract the numerators and write the result over the common
denominator.
• Before adding or subtracting fractions, ensure that the denominators
are the same by finding equivalent fractions with a common
denominator. Multiply the numerator and the denominator of each
fraction by the appropriate value to find the equivalent fractions.
• Typically, it is best to convert all mixed numbers to improper fractions
before beginning the process of adding, subtracting, multiplying, or
dividing.

1.4 Fractions 85
Chapter 1 Real Numbers and Their Operations

TOPIC EXERCISES

Part A: Working with Fractions

Reduce each fraction to lowest terms.

5
1.
30

6
2.
24

30
3.
70

18
4.
27

44
5.
84

54
6.
90

135
7.
30

105
8.
300

18
9.
6

256
10.
16

126
11.
45

52
12.
234

54
13.
162

2000
14.
3000

1.4 Fractions 86
Chapter 1 Real Numbers and Their Operations

270
15.
360

Rewrite as an improper fraction.

3
16. 4
4

1
17. 2
2

7
18. 5
15

1
19. 1
2

5
20. 3
8

3
21. 1
4

1
22. −2
2

3
23. −1
4

Rewrite as a mixed number.

15
24.
2

9
25.
2

40
26.
13

103
27.
25

73
28.
10

52
29. − 7

1.4 Fractions 87
Chapter 1 Real Numbers and Their Operations

59
30. −
6

Part B: Multiplying and Dividing

Multiply and reduce to lowest terms.

31. 2
3
⋅ 5
7

32. 1
5
⋅ 4
8

33. 1
2
⋅ 1
3

34. 3
4
⋅ 20
9

35. 5
7
⋅ 49
10

36. 2
3
⋅ 9
12

37. 6
14
⋅ 21
12

38. 44
15
⋅ 15
11

39. 3 3
4
⋅2 1
3

40. 2 7
10
⋅5 5
6

(− 2 )
3 5
41.
11

(5)
4 9
42. −
5

43. (−
5) (− )
9 3
10

(− )
6 14
44. 7
3

45. (− ) (− 8 )
9 4
12

1.4 Fractions 88
Chapter 1 Real Numbers and Their Operations

(− )
3 4
46. −
8 15

1 1 1
47. 7 ⋅ 2
⋅ 3

3 15 7
48.
5
⋅ 21
⋅ 27

2 1 4
49.
5
⋅3 8
⋅ 5

4 2 5
50. 2 ⋅ ⋅2
9 5 11

Determine the reciprocal of the following numbers.

1
51.
2

8
52.
5

2
53. −
3

4
54. −
3

55. 10

56. −4

1
57. 2
3

5
58. 1
8

Divide and reduce to lowest terms.

1 2
59.
2
÷ 3

5 1
60.
9
÷ 3

61.
5
8
÷ (− 45 )

1.4 Fractions 89
Chapter 1 Real Numbers and Their Operations

62. (−
5)
2 15
÷ 3

− 67
63.
− 67

− 12
64. 1
4

− 10
3
65. 5
− 20

2
3
66. 9
2

30
50
67. 5
3

1
2
68.
2

5
69. 2
5

−6
70. 5
4

71. 2 1
2
÷ 5
3

72. 4 2
3
÷3 1
2

73. 5 ÷2 3
5

74. 4 3
5
÷ 23

Part C: Adding and Subtracting Fractions

Add or subtract and reduce to lowest terms.

1.4 Fractions 90
Chapter 1 Real Numbers and Their Operations

75. 17
20
− 5
20

76. 4
9
− 13
9

77. 3
5
+ 1
5

78. 11
15
+ 9
15

79. 5
7
−2 1
7

80. 5 1
8
−1 1
8

81. 1
2
+ 1
3

82. 1
5
− 1
4

83. 3
4
− 5
2

84. 3
8
+ 7
16

85. 7
15
− 3
10

86. 3
10
+ 2
14

87. 2
30
+ 5
21

88. 3
18
− 1
24

89. 5 1
2
+2 1
3

90. 1 3
4
+2 1
10

1 1 1
91.
2
+ 3
+ 6

2 3 2
92.
3
+ 5
− 9

7 3 2
93.
3
− 2
+ 15

1.4 Fractions 91
Chapter 1 Real Numbers and Their Operations

9 3 3
94.
4
− 2
+ 8

1 2 1
95. 1 +2 −1
3 5 15

2 1 1
96.
3
−4 2
+3 6

6 3
97. 1 − +
16 18

1 1
98. 3 − −
21 15

Part D: Mixed Exercises

Perform the operations. Reduce answers to lowest terms.

3 7 1
99.
14
⋅ 3
÷ 8

100.
1
2
⋅ (− 45 ) ÷ 14
15

1 3 1
101.
2
÷ 4
⋅ 5

5 5 5
102. − ÷ ⋅
9 3 2

5 9 3
103.
12
− 21
+ 9

3 5 1
104. − − +
10 12 20

4 1
105.
5
÷4⋅ 2

5 2
106.
3
÷ 15 ⋅ 3

3 4
107. What is the product of 16
and 9
?

108. What is the product of − 24


5
and 25
8
?

5 25
109. What is the quotient of 9
and 3
?

1.4 Fractions 92
Chapter 1 Real Numbers and Their Operations

110. What is the quotient of − 16


5
and 32?

1 9 2
111. Subtract 6
from the sum of 2
and 3
.

1 3 6
112. Subtract 4
from the sum of 4
and 5
.

113. What is the total width when 3 boards, each with a width of 2 5
8
inches,
are glued together?

114. The precipitation in inches for a particular 3-day weekend was


published as 103 inches on Friday, 1 12 inches on Saturday, and 34 inches on
Sunday. Calculate the total precipitation over this period.

115. A board that is 5 14 feet long is to be cut into 7 pieces of equal length.
What is length of each piece?

3
116. How many 4
inch thick notebooks can be stacked into a box that is 2
feet high?

117. In a mathematics class of 44 students, one-quarter of the students


signed up for a special Saturday study session. How many students signed
up?

118. Determine the length of fencing needed to enclose a rectangular pen


with dimensions 35 12 feet by 20 23 feet.

1
119. Each lap around the track measures 4
mile. How many laps are
required to complete a 2 12 mile run?

120. A retiree earned a pension that consists of three-fourths of his regular


monthly salary. If his regular monthly salary was $5,200, then what monthly
payment can the retiree expect from the pension plan?

Part E: Discussion Board Topics

121. Does 0 have a reciprocal? Explain.

122. Explain the difference between the LCM and the GCF. Give an example.

123. Explain the difference between the LCM and LCD.

1.4 Fractions 93
Chapter 1 Real Numbers and Their Operations

124. Why is it necessary to find an LCD in order to add or subtract fractions?

7 1
125. Explain how to determine which fraction is larger, 16
or 2
.

1.4 Fractions 94
Chapter 1 Real Numbers and Their Operations

ANSWERS

1: 1/6

3: 3/7

5: 11/21

7: 9/2

9: 3

11: 14/5

13: 1/3

15: 3/4

17: 5/2

19: 3/2

21: 7/4

23: −7/4

1
25: 4
2

3
27: 4
25

3
29: −7 7

31: 10/21

33: 1/6

35: 7/2

37: 3/4

1.4 Fractions 95
Chapter 1 Real Numbers and Their Operations

39: 8 3
4

41: −15/22

43: 27/50

45: 3/8

47: 1/42

49: 1

51: 2

53: −3/2

55: 1/10

57: 3/7

59: 3/4

61: −25/32

63: 1

65: 40/3

67: 9/25

69: 25/2

71: 1 1
2

73: 1 12
13

75: 3/5

77: 4/5

1.4 Fractions 96
Chapter 1 Real Numbers and Their Operations

79: −1 3
7

81: 5/6

83: −7/4

85: 1/6

87: 32/105

5
89: 7
6

91: 1

93: 29/30

95: 2 2
3

97: 19/24

99: 4

101: 2/15

103: 9/28

105: 1/10

107: 1/12

109: 1/15

111: 5

113: 7 7
8
inches

3
115: 4
feet

117: 11 students

1.4 Fractions 97
Chapter 1 Real Numbers and Their Operations

119: 10 laps

1.4 Fractions 98
Chapter 1 Real Numbers and Their Operations

1.5 Review of Decimals and Percents

LEARNING OBJECTIVES

1. Convert fractions to decimals and back.


2. Perform operations with decimals.
3. Round off decimals to a given place.
4. Define a percent.
5. Convert percents to decimals and back.
6. Convert fractions to percents and back.

Decimals

In this section, we provide a brief review of the decimal system. A real number in
decimal form, a decimal65 consists of a decimal point, digits (0 through 9) to the left
of the decimal point representing the whole number part, and digits to the right of
the decimal point representing the fractional part. The digits represent powers of
10 as shown in the set {…, 1,000, 100, 10, 1, 1/10, 1/100, 1/1,000, …} according to the
following diagram:

For example, the decimal 538.3 can be written in the following expanded form:

65. A real number expressed using After simplifying, we obtain the mixed number 538 103 . Use this process to convert
the decimal system. decimals to mixed numbers.

99
Chapter 1 Real Numbers and Their Operations

Example 1: Write as a mixed number: 32.15.

Solution: In this example, 32 is the whole part and the decimal ends in the
hundredths place. Therefore, the fractional part will be 15/100, and we can write

Answer: 32.15 = 32 3
20

To convert fractions to decimal equivalents, divide.

Example 2: Write as a decimal: 34.

Solution: Use long division to convert to a decimal.

Answer: 3
4
= 0.75

1.5 Review of Decimals and Percents 100


Chapter 1 Real Numbers and Their Operations

If the division never ends, then use a bar over the repeating digit (or block of digits)
to indicate a repeating decimal.

Example 3: Write as a decimal: 2 56.

Solution: Use long division to convert the fractional part to a decimal and then add
the whole part.

At this point, we can see that the long division will continue to repeat. When this is
the case, use a bar over the repeating digit to indicate that it continues forever:

Then write

⎯⎯
Answer: 2 5
6
= 2.83

1.5 Review of Decimals and Percents 101


Chapter 1 Real Numbers and Their Operations

To add or subtract decimals, align them vertically with the decimal point and add
corresponding place values. Recall that sometimes you need to borrow from or
carry over to the adjoining column (regrouping).

Example 4: Subtract: 54.328 − 23.25.

Solution: Note that trailing zeros to the right of the decimal point do not change
the value of the decimal, 23.25 = 23.250. In this case, you need to borrow from the
tenths place (regroup) to subtract the digits in the hundredths place.

Answer: 31.078

Multiply decimals the same way you multiply whole numbers. The number of
decimal places in the product will be the sum of the decimal places found in each of
the factors.

Example 5: Multiply: 5.36 × 7.4.

Solution: The total number of decimal places of the two factors is 2 + 1 = 3.


Therefore, the result has 3 decimal places.

1.5 Review of Decimals and Percents 102


Chapter 1 Real Numbers and Their Operations

Answer: 39.664

When dividing decimals, move the decimal points of both the dividend and the
divisor so that the divisor is a whole number. Remember to move the decimal the
same number of places for both the dividend and divisor.

Example 6: Divide: 33.3216 ÷ 6.24.

Solution: Move the decimal point to turn the divisor into a whole number: 624.
Move the decimal points of both the divisor and dividend two places to the right.

Next, divide.

Answer: 5.34

It is often necessary to round off66 decimals to a specified number of decimal


66. A means of approximating places. Rounding off allows us to approximate decimals with fewer significant
decimals with a specified
number of significant digits.
digits. To do this, look at the digit to the right of the specified place value.

1.5 Review of Decimals and Percents 103


Chapter 1 Real Numbers and Their Operations

1. If the digit to the right of the specified place is 4 or less, then leave the
specified digit unchanged and drop all subsequent digits.
2. If the digit to the right of the specified place is 5 or greater, then
increase the value of the digit in the specified place by 1 and drop all
subsequent digits.

Recall that decimals with trailing zeros to the right of the decimal point can be
dropped. For example, round 5.635457 to the nearest thousandth:

Round the same number 5.635457 to the nearest hundredth:

After rounding off, be sure to use the appropriate notation ( ≈ ) to indicate that the
number is now an approximation. When working with US currency, we typically
round off to two decimal places, or the nearest hundredth.

Example 7: Calculate and round off to the nearest hundredth.

a. 1/3 of $10.25.

b. 1/4 of $10.25.

Solution: In this context, the key word “of” indicates that we should multiply.

a. Multiplying by 13 is equivalent to dividing by 3.

1.5 Review of Decimals and Percents 104


Chapter 1 Real Numbers and Their Operations

b. Multiplying by 14 is equivalent to dividing by 4.

Answers: a. $3.42; b. $2.56

Definition of Percent

A percent67 is a representation of a number as a part of one hundred. The word


“percent” can be written “per cent” which means “per 100” or “/100.” We use the
symbol ( % ) to denote a percentage:

For example,

67. A representation of a number


N
as part of 100: N% = 100
.
Percents are an important part of our everyday life and show up often in our study
68. A circular graph divided into of algebra. Percents can be visualized using a pie chart68 (or circle graph), where
sectors whose area is each sector gives a visual representation of a percentage of the whole. For example,
proportional to the relative
the following pie chart shows the percentage of students in certain age categories
size of the ratio of the part to
the total. of all US community colleges.

1.5 Review of Decimals and Percents 105


Chapter 1 Real Numbers and Their Operations

Source: American Association of Community Colleges.

Each sector is proportional to the size of the part out of the whole. The sum of the
percentages presented in a pie chart must be 100%. To work with percentages
effectively, you have to know how to convert percents to decimals or fractions and
back again.

Percents to Decimals

Applying the definition of percent, you see that 58% = 100 58


= 0.58. The same result
can be obtained by moving the decimal two places to the left. To convert percents to
decimals, either apply the definition or move the decimal two places to the left.

Example 8: Convert to a decimal: 152%.

Solution: Treat 152% as 152.0% and move the decimal two places to the left.

Answer: 1.52

1.5 Review of Decimals and Percents 106


Chapter 1 Real Numbers and Their Operations

Example 9: Convert to a decimal: 2 3


4
%.

Solution: First, write the decimal percent equivalent,

Next, move the decimal two places to the left,

At this point, fill in the tenths place with a zero.

Answer: 0.0275

Try this! Convert to a decimal: 215%.

Answer: 2.15

Video Solution

(click to see video)


Decimals and Fractions to Percents

To convert a decimal to a percent, convert the decimal to a fraction of 100 and


apply the definition of percent, or equivalently, move the decimal to the right two
places and add a percent sign.

1.5 Review of Decimals and Percents 107


Chapter 1 Real Numbers and Their Operations

Example 10: Convert 0.23 to a percent.

Solution: First, convert the decimal to a fraction of 100 and apply the definition.

You can achieve the same result by moving the decimal two places to the right and
adding a percent sign.

Answer: 23%.

Alternatively, you can multiply by 1 in the form of 100%.

Example 11: Convert 2.35 to a percent.

Solution: Recall that 1 = 100%.

You can achieve the same result by moving the decimal two places to the right and
adding a percent sign.

1.5 Review of Decimals and Percents 108


Chapter 1 Real Numbers and Their Operations

Answer: 235%

Example 12: Convert 5 15 to a percent.

Solution:

Answer: 520%

Sometimes we can use the definition of percent and find an equivalent fraction with
a denominator of 100.

13
Example 13: Convert 25
to a percent.

Solution: Notice that the denominator 25 is a factor of 100. Use the definition of
percent by finding an equivalent fraction with 100 in the denominator.

Answer: 52%

1.5 Review of Decimals and Percents 109


Chapter 1 Real Numbers and Their Operations

This is a very specialized technique because 100 may not be a multiple of the
denominator.

Example 14: Convert 13 to a percent.

Solution: Notice that the denominator 3 is not a factor of 100. In this case, it is best
to multiply by 1 in the form of 100%.

Answer: 33 1
3
%

Try this! Convert to a percent: 23 .

Answer: 66 2
3
%

Video Solution

(click to see video)


Percents to Fractions

When converting percents to fractions, apply the definition of percent and then
reduce.

1.5 Review of Decimals and Percents 110


Chapter 1 Real Numbers and Their Operations

Example 15: Convert to a fraction: 28%.

Solution:

7
Answer: 25

Applying the definition of percent is equivalent to removing the percent sign and
1
multiplying by 100 .

Example 16: Convert to a fraction: 66 2


3
%.

Solution: First, convert to an improper fraction and then apply the definition of
percent.

1.5 Review of Decimals and Percents 111


Chapter 1 Real Numbers and Their Operations

2
Answer: 3

Try this! Convert to a fraction: 3 7


31
%.

1
Answer: 31

Video Solution

(click to see video)

Example 17: Using the given pie chart, calculate the total number of students that
were 21 years old or younger if the total US community college enrollment in 2009
was 11.7 million.

Solution: From the pie chart we can determine that 47% of the total 11.7 million
students were 21 years old or younger.

Source: American Association of Community Colleges.

Convert 47% to a decimal and multiply as indicated by the key word “of.”

1.5 Review of Decimals and Percents 112


Chapter 1 Real Numbers and Their Operations

Answer: In 2009, approximately 5.5 million students enrolled in US community


colleges were 21 years old or younger.

KEY TAKEAWAYS

• To convert a decimal to a mixed number, add the appropriate fractional


part indicated by the digits to the right of the decimal point to the whole
part indicated by the digits to the left of the decimal point and reduce if
necessary.
• To convert a mixed number to a decimal, convert the fractional part of
the mixed number to a decimal using long division and then add it to the
whole number part.
• To add or subtract decimals, align them vertically with the decimal
point and add corresponding place values.
• To multiply decimals, multiply as usual for whole numbers and count
the number of decimal places of each factor. The number of decimal
places in the product will be the sum of the decimal places found in each
of the factors.
• To divide decimals, move the decimal in both the divisor and dividend
until the divisor is a whole number and then divide as usual.
• When rounding off decimals, look to the digit to the right of the
specified place value. If the digit to the right is 4 or less, round down by
leaving the specified digit unchanged and dropping all subsequent
digits. If the digit to the right is 5 or more, round up by increasing the
specified digit by one and dropping all subsequent digits.
N
• A percent represents a number as part of 100: N% = .
100
• To convert a percent to a decimal, apply the definition of percent and
write that number divided by 100. This is equivalent to moving the
decimal two places to the left.
• To convert a percent to a fraction, apply the definition of percent and
then reduce.
• To convert a decimal or fraction to a percent, multiply by 1 in the form
of 100%. This is equivalent to moving the decimal two places to the right
and adding a percent sign.
• Pie charts are circular graphs where each sector is proportional to the
size of the part out of the whole. The sum of the percentages must total
100%.

1.5 Review of Decimals and Percents 113


Chapter 1 Real Numbers and Their Operations

TOPIC EXERCISES

Part A: Decimals

Write as a mixed number.

1. 45.8

2. 15.4

3. 1.82

4. 2.55

5. 4.72

6. 3.14

Write as a decimal.

7. 2 4
5

8. 5 1
5

9. 3 1
8

10. 1 3
20

3
11. 8

5
12. 8

13. 1 1
3

14. 2 1
6

Perform the operations. Round dollar amounts to the nearest hundredth.

15. 13.54 − 4.6

1.5 Review of Decimals and Percents 114


Chapter 1 Real Numbers and Their Operations

16. 16.8 − 4.845

17. 45.631 + 7.82

18. 256.34 + 51.771

19. 12.82 × 5.9

20. 123.5 × 0.17

21. 0.451 × 1.5

22. 0.836 × 9.3

23. 38.319 ÷ 5.3

24. 52.6551 ÷ 5.01

25. 0.9338 ÷ 0.023

26. 4.6035 ÷ 0.045


1
27. Find 6
of $20.00.

1
28. Find 5
of $33.26.

2
29. Find 3
of $15.25.

3
30. Find 4
of $15.50.

31. A gymnast scores 8.8 on the vault, 9.3 on the uneven bars, 9.1 on the
balance beam, and 9.8 on the floor exercise. What is her overall average?

32. To calculate a batting average, divide the player’s number of hits by the
total number of at-bats and round off the result to three decimal places. If a
player has 62 hits in 195 at-bats, then what is his batting average?

Part B: Percents to Decimals

Convert each percent to its decimal equivalent.

1.5 Review of Decimals and Percents 115


Chapter 1 Real Numbers and Their Operations

33. 43%

34. 25%

35. 33%

36. 100%

37. 150%

38. 215%

1
39.
2
%

3
40. 2 %
4

1
41. 1 %
2

2
42. 3 %
3

43. 0.025%

44. 0.0001%

45. 1.75%

46. 20.34%

47. 0%

48. 1%

49. 3.05%

50. 5.003%

51. Convert one-half of one percent to a decimal.

1.5 Review of Decimals and Percents 116


Chapter 1 Real Numbers and Their Operations

52. Convert three-quarter percent to a decimal.

53. What is 20% of zero?

54. What is 50% of one hundred?

55. What is 150% of 100?

56. What is 20% of $20.00?

57. What is 112% of $210?

58. What is 9 1
2
% of $1,200?

59. If the bill at a restaurant comes to $32.50, what is the amount of a 15%
tip?

60. Calculate the total cost, including a 20% tip, of a meal totaling $37.50.

61. If an item costs $45.25, then what is the total after adding 8.25% for tax?

62. If an item costs $36.95, then what is the total after adding 9¼% tax?

63. A retail outlet is offering 15% off the original $29.99 price of branded
sweaters. What is the price after the discount?

64. A solar technology distribution company expects a 12% increase in first


quarter sales as a result of a recently implemented rebate program. If the
first quarter sales last year totaled $350,000, then what are the sales
projections for the first quarter of this year?

65. If a local mayor of a town with a population of 40,000 people enjoys a 72%
favorable rating in the polls, then how many people view the mayor
unfavorably?

66. If a person earning $3,200 per month spends 32% of his monthly income
on housing, then how much does he spend on housing each month?

Part C: Decimals and Fractions to Percents

Convert the following decimals and fractions to percents.

1.5 Review of Decimals and Percents 117


Chapter 1 Real Numbers and Their Operations

67. 0.67

68. 0.98

69. 1.30

70. 2.25

57
71.
100

99
72.
100

1
73.
5

2
74.
3

25
75.
8

1
76. 3
4

17
77.
50

1
78. 7

79. 0.0023

80. 0.000005

81. 20

82. 100

Part D: Percents to Fractions

Use the definition of percent to convert to fractions.

83. 20%

1.5 Review of Decimals and Percents 118


Chapter 1 Real Numbers and Their Operations

84. 80%

85. 57%

86. 97%

1
87. 5 %
2

2
88. 1 %
3

89. 75%

90. 32%

91. 400%

92. 230%

93. 100%

1
94.
8
%

5
95.
12
%

5
96. 5 7 %

1
97. 33 %
3

7
98. 3 %
31

99. 0.7%

100. 0.05%

101. 1.2%

102. 12.5%

1.5 Review of Decimals and Percents 119


Chapter 1 Real Numbers and Their Operations

The course grade weighting for a traditional mathematics course with 1,200 total
points is shown in the pie chart below. Use the chart to answer the following
questions.

103. How many points will the final exam be worth?

104. How many points will the homework be worth?

105. How many points will each of the four regular exams be worth?

106. How many 10-point homework assignments can be assigned?

A website had 12,000 unique users in the fall of 2009. Answer the questions based on
the pie chart below depicting total Web browser usage.

107. How many users used the Firefox Web browser?

108. How many users used a browser other than Internet Explorer?

109. How many users used either Firefox or Internet Explorer?

110. How many users used Google Chrome or Safari?

The 2009 employment status of 11.7 million full-time community college students is
given in the following pie chart. Use the chart to answer the following questions.
Round off each answer to the nearest hundredth.

1.5 Review of Decimals and Percents 120


Chapter 1 Real Numbers and Their Operations

Source: American
Association of
Community Colleges.

111. How many full-time students were employed full time?

112. How many full-time students were employed part time?

113. How many full-time students were unemployed or employed part time?

114. How many full-time students also worked part time or full time?

The pie chart below depicts all US households categorized by income. The total
number of households in 2007 was about 111,600,000. Use the chart to answer the
following questions.

Source: US Census
Bureau.

115. How many households reported an income from $50,000 to $74,999?

116. How many households reported an income from $75,000 to $99,999?

117. How many households reported an income of $100,000 or more?

118. How many households reported an income of less than $25,000?

1.5 Review of Decimals and Percents 121


Chapter 1 Real Numbers and Their Operations

Part E: Discussion Board Topics

119. The decimal system is considered a base-10 numeral system. Explain


why. What other numeral systems are in use today?

120. Research and discuss the history of the symbol %.

121. Research and discuss simple interest and how it is calculated. Make up
an example and share the solution.

122. Discuss methods for calculating tax and total bills.

123. Research and discuss the history of the pie chart.

124. Research and discuss the idea of a weighted average.

1.5 Review of Decimals and Percents 122


Chapter 1 Real Numbers and Their Operations

ANSWERS

1: 45 4
5

3: 1 41
50

5: 4 18
25

7: 2.8

9: 3.125

11: 0.375

⎯⎯
13: 1.3

15: 8.94

17: 53.451

19: 75.638

21: 0.6765

23: 7.23

25: 40.6

27: $3.33

29: $10.17

31: 9.25

33: 0.43

35: 0.33

37: 1.5

1.5 Review of Decimals and Percents 123


Chapter 1 Real Numbers and Their Operations

39: 0.005

41: 0.015

43: 0.00025

45: 0.0175

47: 0

49: 0.0305

51: 0.005

53: 0

55: 150

57: $235.20

59: $4.88

61: $48.98

63: $25.49

65: 11,200 people

67: 67%

69: 130%

71: 57%

73: 20%

75: 312.5%

77: 34%

1.5 Review of Decimals and Percents 124


Chapter 1 Real Numbers and Their Operations

79: 0.23%

81: 2,000%

1
83:
5

57
85:
100

11
87:
200

3
89:
4

91: 4

93: 1

1
95:
240

1
97:
3

7
99:
1000

3
101:
250

103: 360 points

105: 180 points

107: 3,000 users

109: 10,560 users

111: 3.16 million

113: 8.54 million

115: 20,980,800 households

1.5 Review of Decimals and Percents 125


Chapter 1 Real Numbers and Their Operations

117: 21,204,000 households

1.5 Review of Decimals and Percents 126


Chapter 1 Real Numbers and Their Operations

1.6 Exponents and Square Roots

LEARNING OBJECTIVES

1. Interpret exponential notation with positive integer exponents.


2. Calculate the nth power of a real number.
3. Calculate the exact and approximate value of the square root of a real
number.

Exponential Notation and Positive Integer Exponents

If a number is repeated as a factor numerous times, then we can write the product
in a more compact form using exponential notation69. For example,

The base70 is the factor, and the positive integer exponent71 indicates the number
of times the base is repeated as a factor. In the above example, the base is 5 and the
exponent is 4. In general, if a is the base that is repeated as a factor n times, then

69. The compact notation When the exponent is 2, we call the result a square72. For example,
ax 2 + bx + c = 0. used
when a factor is repeated
multiple times.

70. The factor a in the exponential


notation an .

71. The positive integer n in the


exponential notation an that The number 3 is the base and the integer 2 is the exponent. The notation 32 can be
indicates the number of times read two ways: “three squared” or “3 raised to the second power.” The base can be
the base is used as a factor. any real number.
72. The result when the exponent
of any real number is 2.

127
Chapter 1 Real Numbers and Their Operations

It is important to study the difference between the ways the last two examples are
calculated. In the example (−7)2 , the base is −7 as indicated by the parentheses. In
the example −52 , the base is 5, not −5, so only the 5 is squared and the result
remains negative. To illustrate this, write

This subtle distinction is very important because it determines the sign of the
result.

The textual notation for exponents is usually denoted using the caret73 (^) symbol
as follows:

The square of an integer is called a perfect square74. The ability to recognize


perfect squares is useful in our study of algebra. The squares of the integers from 1
to 15 should be memorized. A partial list of perfect squares follows:

73. The symbol ^ that indicates


exponents on many
calculators, an = a ^ n.

74. The result of squaring an Try this! Simplify (−12)2 .


integer.

1.6 Exponents and Square Roots 128


Chapter 1 Real Numbers and Their Operations

Answer: 144

Video Solution

(click to see video)

When the exponent is 3 we call the result a cube75. For example,

The notation 33 can be read two ways: “three cubed” or “3 raised to the third
power.” As before, the base can be any real number.

Note that the result of cubing a negative number is negative. The cube of an integer
is called a perfect cube76. The ability to recognize perfect cubes is useful in our
study of algebra. The cubes of the integers from 1 to 10 should be memorized. A
partial list of perfect cubes follows:

Try this! Simplify (−2)3 .

Answer: −8

75. The result when the exponent Video Solution


of any real number is 3.

76. The result of cubing an integer. (click to see video)

1.6 Exponents and Square Roots 129


Chapter 1 Real Numbers and Their Operations

If the exponent is greater than 3, then the notation an is read “a raised to the nth
power.”

Notice that the result of a negative base with an even exponent is positive. The
result of a negative base with an odd exponent is negative. These facts are often
confused when negative numbers are involved. Study the following four examples
carefully:

The base is (−2) The base is 2

(−2) 4 = (−2) ⋅ (−2) ⋅ (−2) ⋅ (−2) = +16 −2 4 = −2 ⋅ 2 ⋅ 2 ⋅ 2 = −16


(−2) 3 = (−2) ⋅ (−2) ⋅ (−2) = −8 −2 3 = −2 ⋅ 2 ⋅ 2 = −8

The parentheses indicate that the negative number is to be used as the base.

Example 1: Calculate:

a. (− 13 )
3

b. (− 13 )
4

Solution: The base is − 13 for both problems.

a. Use the base as a factor three times.

1.6 Exponents and Square Roots 130


Chapter 1 Real Numbers and Their Operations

b. Use the base as a factor four times.

Answers: a. − 271 ; b. 1
81

Try this! Simplify: −104 and (−10)4 .

Answers: −10,000 and 10,000

Video Solution

(click to see video)


Square Root of a Real Number

Think of finding the square root77 of a number as the inverse of squaring a number.
In other words, to determine the square root of 25 the question is, “What number
squared equals 25?” Actually, there are two answers to this question, 5 and −5.

77. The number that, when


multiplied by itself, yields the
original number.

1.6 Exponents and Square Roots 131


Chapter 1 Real Numbers and Their Operations

When asked for the square root of a number, we implicitly mean the principal
(nonnegative) square root78. Therefore we have,

⎯⎯⎯⎯
As an example, √25 = 5, which is read “square root of 25 equals 5.” The symbol √
is called the radical sign79 and 25 is called the radicand80. The alternative textual
notation for square roots follows:

It is also worthwhile to note that

This is the case because 12 = 1 and 02 = 0.

⎯⎯⎯⎯⎯⎯⎯⎯⎯⎯⎯
Example 2: Simplify: √10,000.

Solution: 10,000 is a perfect square because 100 ⋅ 100 = 10,000.

78. The non-negative square root.


Answer: 100
79. The symbol √ used to denote a
square root.

80. The expression a within a


n ⎯⎯
radical sign, √ a.

1.6 Exponents and Square Roots 132


Chapter 1 Real Numbers and Their Operations

Example 3: Simplify: √ 19 .
⎯⎯⎯

Solution: Here we notice that 19 is a square because 1


3
⋅ 1
3
= .19

1
Answer: 3

Given a and b as positive real numbers, use the following property to simplify
square roots whose radicands are not squares:

The idea is to identify the largest square factor of the radicand and then apply the
⎯⎯
property shown above. As an example, to simplify √8 notice that 8 is not a perfect
square. However, 8 = 4 ⋅ 2 and thus has a perfect square factor other than 1. Apply
the property as follows:

⎯⎯
Here 2√2 is a simplified irrational number. You are often asked to find an
approximate answer rounded off to a certain decimal place. In that case, use a

1.6 Exponents and Square Roots 133


Chapter 1 Real Numbers and Their Operations

calculator to find the decimal approximation using either the original problem or
the simplified equivalent.

On a calculator, try 2.83^2. What do you expect? Why is the answer not what you
would expect?

⎯⎯⎯⎯⎯
It is important to mention that the radicand must be positive. For example, √−9 is
undefined since there is no real number that when squared is negative. Try taking
the square root of a negative number on your calculator. What does it say? Note:
taking the square root of a negative number is defined later in the course.

Example 4: Simplify and give an approximate answer rounded to the nearest


⎯⎯⎯⎯
hundredth: √75.

Solution: The radicand 75 can be factored as 25 ⋅ 3 where the factor 25 is a perfect


square.

⎯⎯⎯⎯
Answer: √75 ≈ 8.66

⎯⎯⎯⎯ ⎯⎯
As a check, calculate √75 and 5√3 on a calculator and verify that the both results
are approximately 8.66.

1.6 Exponents and Square Roots 134


Chapter 1 Real Numbers and Their Operations

⎯⎯⎯⎯⎯⎯
Example 5: Simplify: √180.

Solution:

Since the question did not ask for an approximate answer, we present the exact
answer.

⎯⎯
Answer: 6√5

⎯⎯⎯⎯⎯⎯
Example 5: Simplify: −5√162.

Solution:

⎯⎯
Answer: −45√2

1.6 Exponents and Square Roots 135


Chapter 1 Real Numbers and Their Operations

Try this! Simplify and give an approximate answer rounded to the nearest
⎯⎯⎯⎯⎯⎯
hundredth: √128.

⎯⎯
Answer: 8√2 ≈ 11.31

Video Solution

(click to see video)

A right triangle81 is a triangle where one of the angles


Figure 1.1 Pythagoras
measures 90°. The side opposite the right angle is the
longest side, called the hypotenuse82, and the other two
sides are called legs83. Numerous real-world
applications involve this geometric figure. The
Pythagorean theorem84 states that given any right
triangle with legs measuring a and b units, the square of
the measure of the hypotenuse c is equal to the sum of
the squares of the measures of the legs: a2 + b2 = c2 .
In other words, the hypotenuse of any right triangle is
equal to the square root of the sum of the squares of its
legs.
Source: Detail of The School of
Athens by Raffaello Sanzio, 1509,
from
http://commons.wikimedia.org/
wiki/
File:Sanzio_01_Pythagoras.jpg.

81. A triangle with an angle that


measures 90°.

82. The longest side of a right


triangle, it will always be the
side opposite the right angle.

83. The sides of a right triangle


that are not the hypotenuse.

84. Given any right triangle with Example 6: If the two legs of a right triangle measure 3 units and 4 units, then find
legs measuring a and b units the length of the hypotenuse.
and hypotenuse measuring c
units, then a2 + b2 = c2.

1.6 Exponents and Square Roots 136


Chapter 1 Real Numbers and Their Operations

Solution: Given the lengths of the legs of a right triangle, use the formula
⎯⎯⎯⎯⎯⎯⎯⎯⎯⎯⎯
c = √a2 + b2 to find the length of the hypotenuse.

Answer: c = 5 units

When finding the hypotenuse of a right triangle using the Pythagorean theorem,
the radicand is not always a perfect square.

Example 7: If the two legs of a right triangle measure 2 units and 6 units, find the
length of the hypotenuse.

Solution:

1.6 Exponents and Square Roots 137


Chapter 1 Real Numbers and Their Operations

⎯⎯⎯⎯
Answer: c = 2√10 units

KEY TAKEAWAYS

• When using exponential notation an , the base a is used as a factor n


times.
• When the exponent is 2, the result is called a square. When the exponent
is 3, the result is called a cube.
• Memorize the squares of the integers up to 15 and the cubes of the
integers up to 10. They will be used often as you progress in your study
of algebra.
• When negative numbers are involved, take care to associate the
exponent with the correct base. Parentheses group a negative number
raised to some power.
• A negative base raised to an even power is positive.
• A negative base raised to an odd power is negative.
• The square root of a number is a number that when squared results in
the original number. The principal square root is the positive square
root.
• Simplify a square root by looking for the largest perfect square factor of
the radicand. Once a perfect square is found, apply the property
⎯⎯⎯⎯⎯⎯⎯ ⎯⎯ ⎯⎯
√ a ⋅ b = √ a ⋅ √ b, where a and b are nonnegative, and simplify.
• Check simplified square roots by calculating approximations of the
answer using both the original problem and the simplified answer on a
calculator to verify that the results are the same.
• Find the length of the hypotenuse of any right triangle given the lengths
of the legs using the Pythagorean theorem.

1.6 Exponents and Square Roots 138


Chapter 1 Real Numbers and Their Operations

TOPIC EXERCISES

Part A: Square of a Number

Simplify.

2
1. 10

2
2. 12

2
3. (−9)

2
4. −12

5. 11 ^2

6. (−20) ^2
2
7. 0

2
8. 1

2
9. −(−8)

2
10. −(13)

11. (
2)
1 2

12. (−
3)
2 2

13. 0.5^2

14. 1.25^2

15. (−2.6)^2

16. −(−5.1)^2

1.6 Exponents and Square Roots 139


Chapter 1 Real Numbers and Their Operations

17. (2
3)
1 2

18. (5
4)
3 2

If s is the length of the side of a square, then the area is given by A = s2 .

19. Determine the area of a square given that a side measures 5 inches.

20. Determine the area of a square given that a side measures 2.3 feet.

21. List all the squares of the integers 0 through 15.

22. List all the squares of the integers from −15 to 0.

{0, , 2}.
23. List the squares of all the rational numbers in the set
1 2 4 5
3
, 3
, 1, 3
, 3

{0, 2}
24. List the squares of all the rational numbers in the set
1 3 5
2
, 1, 2
, 2, .

Part B: Integer Exponents

Simplify.

3
25. 5

6
26. 2

4
27. (−1)

3
28. (−3)

4
29. −1

4
30. (−2)

3
31. −7

1.6 Exponents and Square Roots 140


Chapter 1 Real Numbers and Their Operations

3
32. (−7)

3
33. −(−3)

4
34. −(−10)

20
35. (−1)

21
36. (−1)

37. (−6) ^3

38. −3 ^4

39. 1 ^ 100

40. 0 ^ 100

41. −(
2)
1 3

42. (
2)
1 6

43. (
2)
5 3

44. (−
4)
3 4

45. List all the cubes of the integers −5 through 5.

46. List all the cubes of the integers from −10 to 0.

{− 3}
47. List all the cubes of the rational numbers in the set
2 1 1 2
3
, − 3
, 0, 3
, .

{− 7}
48. List all the cubes of the rational numbers in the set
3 1 1 3
7
, − 7
, 0, 7
, .

1.6 Exponents and Square Roots 141


Chapter 1 Real Numbers and Their Operations

Part C: Square Root of a Number

Determine the exact answer in simplified form.

⎯⎯⎯⎯⎯⎯
49. √ 121

⎯⎯⎯⎯
50. √ 81

⎯⎯⎯⎯⎯⎯
51. √ 100

⎯⎯⎯⎯⎯⎯
52. √ 169

⎯⎯⎯⎯
53. −√ 25

⎯⎯⎯⎯⎯⎯
54. −√ 144

⎯⎯⎯⎯
55. √ 12

⎯⎯⎯⎯
56. √ 27

⎯⎯⎯⎯
57. √ 45

⎯⎯⎯⎯
58. √ 50

⎯⎯⎯⎯
59. √ 98

⎯⎯⎯⎯⎯⎯⎯⎯
60. √ 2000

61. √
⎯1⎯⎯
4

62. √
⎯⎯⎯⎯
9

16

63. √
⎯5⎯⎯
9

1.6 Exponents and Square Roots 142


Chapter 1 Real Numbers and Their Operations

64. √
⎯⎯⎯⎯
8

36

⎯⎯⎯⎯⎯⎯⎯
65. √ 0.64

⎯⎯⎯⎯⎯⎯⎯
66. √ 0.81

⎯⎯⎯⎯⎯2⎯
67. √ 30

⎯⎯⎯⎯⎯2⎯
68. √ 15

⎯⎯⎯⎯⎯⎯⎯⎯2⎯
69. √ (−2)

70. √ (−5)
⎯⎯⎯⎯⎯⎯⎯⎯⎯2⎯

⎯⎯⎯⎯⎯
71. √ −9

⎯⎯⎯⎯⎯⎯⎯
72. √ −16

⎯⎯⎯⎯
73. 3√ 16

⎯⎯⎯⎯
74. 5√ 18

⎯⎯⎯⎯
75. −2√ 36

⎯⎯⎯⎯
76. −3√ 32

⎯⎯⎯⎯⎯⎯
77. 6√ 200

⎯⎯⎯⎯
78. 10√ 27

Approximate the following to the nearest hundredth.

⎯⎯
79. √ 2

1.6 Exponents and Square Roots 143


Chapter 1 Real Numbers and Their Operations

⎯⎯
80. √ 3

⎯⎯⎯⎯
81. √ 10

⎯⎯⎯⎯
82. √ 15

⎯⎯
83. 2√ 3

⎯⎯
84. 5√ 2

⎯⎯
85. −6√ 5

⎯⎯
86. −4√ 6

87. sqrt(79)

88. sqrt(54)

89. −sqrt(162)

90. −sqrt(86)

91. If the two legs of a right triangle measure 6 units and 8 units, then find
the length of the hypotenuse.

92. If the two legs of a right triangle measure 5 units and 12 units, then find
the length of the hypotenuse.

93. If the two legs of a right triangle measure 9 units and 12 units, then find
the length of the hypotenuse.

3
94. If the two legs of a right triangle measure 2
units and 2 units, then find
the length of the hypotenuse.

95. If the two legs of a right triangle both measure 1 unit, then find the
length of the hypotenuse.

1.6 Exponents and Square Roots 144


Chapter 1 Real Numbers and Their Operations

96. If the two legs of a right triangle measure 1 unit and 5 units, then find
the length of the hypotenuse.

97. If the two legs of a right triangle measure 2 units and 4 units, then find
the length of the hypotenuse.

98. If the two legs of a right triangle measure 3 units and 9 units, then find
the length of the hypotenuse.

Part D: Discussion Board Topics

99. Why is the result of an exponent of 2 called a square? Why is the result of
an exponent of 3 called a cube?

100. Research and discuss the history of the Pythagorean theorem.

101. Research and discuss the history of the square root.

102. Discuss the importance of the principal square root.

1.6 Exponents and Square Roots 145


Chapter 1 Real Numbers and Their Operations

ANSWERS

1: 100

3: 81

5: 121

7: 0

9: −64

11: 1/4

13: .25

15: 6.76

4
17: 5
9

19: 25 square inches

21: {0, 1, 4, 9, 16, 25, 36, 49, 64, 81, 100, 121, 144, 169, 196, 225}

23: {0, 1/9, 4/9, 1, 16/9, 25/9, 4}

25: 125

27: 1

29: −1

31: −343

33: 27

35: 1

37: −216

1.6 Exponents and Square Roots 146


Chapter 1 Real Numbers and Their Operations

39: 1

1
41: −
8

125
43:
8

45: {−125, −64, −27, −8, −1, 0, 1, 8, 27, 64, 125}

47: {− }
8 1 1 8
27
, − 27
, 0, 27
, 27

49: 11

51: 10

53: −5

⎯⎯
55: 2√ 3

⎯⎯
57: 3√ 5

⎯⎯
59: 7√ 2

1
61:
2

√5
63:
3

65: 0.8

67: 30

69: 2

71: Not real

73: 12

75: −12

1.6 Exponents and Square Roots 147


Chapter 1 Real Numbers and Their Operations

⎯⎯
77: 60√ 2

79: 1.41

81: 3.16

83: 3.46

85: −13.42

87: 8.89

89: −12.73

91: 10 units

93: 15 units

⎯⎯
95: √ 2 units

⎯⎯
97: 2√ 5 units

1.6 Exponents and Square Roots 148


Chapter 1 Real Numbers and Their Operations

1.7 Order of Operations

LEARNING OBJECTIVES

1. Identify and work with grouping symbols.


2. Understand the order of operations.
3. Simplify using the order of operations.

Grouping Symbols

In a computation where more than one operation is involved, grouping symbols


help tell us which operations to perform first. The grouping symbols85 commonly
used in algebra are

All of the above grouping symbols, as well as absolute value, have the same order of
precedence. Perform operations inside the innermost grouping symbol or absolute
value first.

Example 1: Simplify: 5 − (4 − 12).

Solution: Perform the operations within the parentheses first. In this case, first
subtract 12 from 4.

85. Parentheses, brackets, braces,


and the fraction bar are the
common symbols used to
group expressions and
mathematical operations
within a computation.

149
Chapter 1 Real Numbers and Their Operations

Answer: 13

Example 2: Simplify: 3 {−2 [− (−3 − 1)]}.

Solution:

Answer: −24

5−||4−(−3)||
|−3|−(5−7)
Example 3: Simplify: .

Solution: The fraction bar groups the numerator and denominator. They should be
simplified separately.

1.7 Order of Operations 150


Chapter 1 Real Numbers and Their Operations

Answer: − 25

Try this! Simplify: − [−3 (2 + 3)].

Answer: 15

Video Solution

(click to see video)


Order of Operations

When several operations are to be applied within a calculation, we must follow a


specific order86 to ensure a single correct result.

1. Perform all calculations within the innermost parentheses or


grouping symbols.
2. Evaluate all exponents.
3. Perform multiplication and division operations from left to right.
4. Finally, perform all remaining addition and subtraction operations
from left to right.

Caution: Note that multiplication and division operations must be worked from left to
right.

Example 4: Simplify: 52 − 4 ⋅ 3 ÷ 12.

Solution: First, evaluate 52 and then perform multiplication and division as they
appear from left to right.

86. To ensure a single correct


result, perform mathematical
operations in a specific order.

1.7 Order of Operations 151


Chapter 1 Real Numbers and Their Operations

Answer: 24

Because multiplication and division operations should be worked from left to right,
it is sometimes correct to perform division before multiplication.

Example 5: Simplify: 24 − 12 ÷ 3 ⋅ 2 + 11.

Solution: Begin by evaluating the exponent, 24 = 2 ⋅ 2 ⋅ 2 ⋅ 2 = 16.

Multiplying first leads to an incorrect result.

1.7 Order of Operations 152


Chapter 1 Real Numbers and Their Operations

Answer: 19

Example 6: Simplify: −3 − 52 + (−7)2 .

Solution: Take care to correctly identify the base when squaring.

Answer: 21

Example 7: Simplify: 5 − 3 [23 − 5 + 7(−3)].

Solution: It is tempting to first subtract 5 − 3, but this will lead to an incorrect


result. The order of operations requires us to simplify within the brackets first.

1.7 Order of Operations 153


Chapter 1 Real Numbers and Their Operations

Subtracting 5 − 3 first leads to an incorrect result.

Answer: 59

Example 8: Simplify: −32 − [5 − (4 − 10)].


2

Solution: Perform the operations within the innermost parentheses first.

Answer: −8

1.7 Order of Operations 154


Chapter 1 Real Numbers and Their Operations

Example 9: Simplify: (− 23 ) ÷ [ 53 − (− 12 ) ].
2 3

Solution:

32
Answer: 129

We are less likely to make a mistake if we work one operation at a time. Some
problems may involve an absolute value, in which case we assign it the same order
of precedence as parentheses.

Example 10: Simplify: 2 − 4 |−4 − 3| + (−2)4.

Solution: We begin by evaluating the absolute value and then the exponent
(−2)4 = (−2) (−2) (−2) (−2) = +16.

1.7 Order of Operations 155


Chapter 1 Real Numbers and Their Operations

Answer: −10

Try this! Simplify: 10 ÷ 5 ⋅ 2 |(−4) + |−3||| + (−3)2.

Answer: 13

Video Solution

(click to see video)

1.7 Order of Operations 156


Chapter 1 Real Numbers and Their Operations

KEY TAKEAWAYS

• Grouping symbols indicate which operations to perform first. We usually


group mathematical operations with parentheses, brackets, braces, and
the fraction bar. We also group operations within absolute values. All
groupings have the same order of precedence: the operations within the
innermost grouping are performed first.

• When applying operations within a calculation, follow the order


of operations to ensure a single correct result.

1. Address innermost parentheses or groupings first.


2. Simplify all exponents.
3. Perform multiplication and division operations from left to
right.
4. Finally, perform addition and subtraction operations from
left to right.
• It is important to highlight the fact that multiplication and division
operations should be applied as they appear from left to right. It is a
common mistake to always perform multiplication before division,
which, as we have seen, in some cases produces incorrect results.

1.7 Order of Operations 157


Chapter 1 Real Numbers and Their Operations

TOPIC EXERCISES

Part A: Order of Operations

Simplify.

1. −7 −3⋅5

2. 3 +2⋅3

3. −3(2) − 62
2
4. 2(−3) + 5(−4)

5. 6/3 *2

6. 6/(3 * 2)
1 3 2
7. − − ⋅
2 5 3

5 1 5
8.
8
÷ 2
− 6

2
9. 3.2 − 6.9 ÷ 2.3

10. 8.2 − 3 ÷ 1.2 ⋅ 2.1

11. 2 + 3(−2) − 7

12. 8 ÷2−3⋅2

13. 3 + 6 2 ÷ 12

14. 5 − 4 2 ÷ (−8)

15. −9 − 3 ⋅ 2 ÷ 3(−2)

16. −2 − 3 2 + (−2) 2

1.7 Order of Operations 158


Chapter 1 Real Numbers and Their Operations

17. 12 ÷ 6 ⋅ 2 − 22

18. 4 ⋅ 3 ÷ 12 ⋅ 2 − (−2) 2
2
19. (−5) − 2(5)2 ÷ 10

20. −3(4 − 7) + 2

21. (−2 + 7) 2 − 10 2

22. 10 − 7(3 + 2) + 7 2

23. −7 − 3 (4 − 2 ⋅ 8)

24. 5 − 3 [6 − (2 + 7)]

25. 1 + 2 [(−2) 3 − (−3) 2 ]

[2 (7 − 5) ÷ 4 ⋅ (−2) + (−3) ]
3
26. −3

27. −7
2
− [−20 − (−3) 2 ] − (−10)

28. 4.7 − 3.2 (4 − 1.2 3 )

29. −5.4 (6.1 − 3.1 ÷ 0.1) − 8.2 2

2
30. −7.3 + (−9.3) 2 − 37.8 ÷ 1.8

31. 2 − 7 ( 3 2 − 3 + 4 ⋅ 3)

32. (
2)
1 2
− (− 23 )
2

33. (
2)
1 3
+ (−2) 3

1.7 Order of Operations 159


Chapter 1 Real Numbers and Their Operations

34. (−
3)
1 2
− (− 23 )
3

1 1 1
35.
3
− 2
⋅ 5

5 3 14
36.
8
÷ 2
⋅ 15

− ( 12 )
2 3
37. 5 ⋅ 15

(5 − )
5 3 4
38.
17 35

39.
3
16
÷ ( 12
5
− 1
2
+ 23 ) ⋅ 4

40. (
3)
2 2
− ( 12 )
2

[ 4 ⋅ (−4) − 2]
1 3 2 2
41. 2

⋅ [( 23 ) − ( 12 ) ] ÷ (−2) 2
2 2
42. 6

(−5) +3
22

43.
−4 2 +2⋅7

(−3.2−3.3)(8.7−4.7)
44.
(−4.7+3.9+2.1)

2−[3−(5−7) ]
2

3(6−3 2 )
45.

2+3⋅6−4⋅3
46.
2 2 −3 2

(2+7)⋅2−2 3
47.
10+9 2 +3 3

(−1−3)2 −15
−3⋅(−7+2 2 )−5
48.

1.7 Order of Operations 160


Chapter 1 Real Numbers and Their Operations

49. (7 + 4 * (−2)) / (−3 + (−2) ^ 2)

50. 4 + 3 * ((−3) ^ 3 + 5 ^ 2) / 6 − 2 ^ 2

51. Mary purchased 14 bottles of water at $0.75 per bottle, 4 pounds of


assorted candy at $3.50 per pound, and 16 packages of microwave popcorn
costing $0.50 each for her party. What was her total bill?

52. Joe bought four 8-foot 2-by-4 boards for $24.00. How much did he spend
per linear foot?

53. Margaret bought two cases of soda at the local discount store for $23.52.
If each case contained 24 bottles, how much did she spend per bottle?

54. Billy earns $12.00 per hour and “time and a half” for every hour he works
over 40 hours a week. What is his pay for 47 hours of work this week?

55. Audry bought 4 bags of marbles each containing 15 assorted marbles. If


she wishes to divide them up evenly between her 3 children, how many will
each child receive?

56. Mark and Janet carpooled home from college for the Thanksgiving
holiday. They shared the driving, but Mark drove twice as far as Janet. If
Janet drove 135 miles, then how many miles was the entire trip?

Part B: Order of Operations with Absolute Values

Simplify.

57. 3 + 2 |−5||

58. 9 − 4 |−3|

59. − (− |2| + |−10|)

60. − (|−6|| − |−8|)

61. |− (40 − |−22|)||

62. ||−5|| − |10|||

1.7 Order of Operations 161


Chapter 1 Real Numbers and Their Operations

63. −(|−8| − 5)
2

64. (|−1| − |−2|) 2

65. −4 + 2 ||2 2 − 3 2 ||

66. −10 − ||4 − 5 2 ||

67. − |(−5)
| 2
+ 4 2 ÷ 8||
| |

68. − (−3 − [ 6 − |−7|])

69. −2 [7 − (4 + |−7|)]

70. 3 − 7 |−2 − 3| + 4 3

71. 7 − 5 ||6 2 − 5 2 || + (−7) 2

72. (−4)
2
− ||−5 + (−2) 3 || − 3 2

− || 12 − (− 43 ) ||
2 2
73.
3 | |

74. −30 | ÷ 15 ||
10 1
| 3
− 2

75. (−4)
3
− (2 − |−4|) ÷ ||−3 2 + 7||

76. [10 − 3 (6 − |−8|)] ÷ 4 − 5 2

Find the distance between the given numbers on a number line.

1 1
77. and −
2 4

3 2
78. − and −
4 3

1.7 Order of Operations 162


Chapter 1 Real Numbers and Their Operations

5 3
79. − and −
8 4

7 3
80. − and 7
5

81. −0.5 and 8.3

82. 10.7 and −2.8

1 1
83. 3 and −2
5 3

3
84. 5 and 0
4

Part C: Discussion Board Topics

85. Convert various examples in this section to equivalent expressions using


text-based symbols.

86. What is PEMDAS and what is it missing?

87. Discuss the importance of proper grouping and give some examples.

88. Experiment with the order of operations on a calculator and share your
results.

1.7 Order of Operations 163


Chapter 1 Real Numbers and Their Operations

ANSWERS

1: −22

3: −42

5: 4

9
7: −
10

9: 7.24

11: −11

13: 6

15: −5

17: 0

19: 20

21: −75

23: 29

25: −33

27: −10

29: 67.22

31: −124

63
33: −
8

7
35:
30

1.7 Order of Operations 164


Chapter 1 Real Numbers and Their Operations

13
37:
24

9
39: 7

41: 50

43: −17

1
45: −
3

5
47:
59

49: −1

51: $32.50

53: $0.49

55: 20 marbles

57: 13

59: −8

61: 18

63: −9

65: 6

67: −27

69: 8

71: 1

11
73: −
18

1.7 Order of Operations 165


Chapter 1 Real Numbers and Their Operations

75: −63

3
77: unit
4

1
79: unit
8

81: 8.8 units

8
83: 5 units
15

1.7 Order of Operations 166


Chapter 1 Real Numbers and Their Operations

1.8 Review Exercises and Sample Exam

167
Chapter 1 Real Numbers and Their Operations

REVIEW EXERCISES

Real Numbers and the Number Line

Choose an appropriate scale and graph the following sets of real numbers on a
number line.

1. {−4, 0, 4}

2. {−30, 10, 40}

3. {−12, −3, 9}

4. {−10, 8, 10}

Fill in the blank with <, = , or >.

5. 0 ___ −9

6. −75 ___ −5

7. −12 ___ − (−3)

8. − (−23) ___ 23

9. |−20| ____ − |−30|

10. − |6|| ____ − |− (−8)||

Determine the unknown.

11. || ? || =2

12. || ? || =1

13. || ? || = −7

14. || ? || =0

1.8 Review Exercises and Sample Exam 168


Chapter 1 Real Numbers and Their Operations

Translate the following into a mathematical statement.

15. Negative eight is less than or equal to zero.

16. Seventy-eight is not equal to twelve.

17. Negative nine is greater than negative ten.

18. Zero is equal to zero.

Adding and Subtracting Integers

Simplify.

19. 12 + (−7)

20. 20 + (−32)

21. −23 − (−7)

22. −8 − (−8)

23. −3 − (−13) + (−1)

24. 9 + (−15) − (−8)

25. (7 − 10) − 3

26. (−19 + 6) − 2

Find the distance between the given numbers on a number line.

27. −8 and 14

28. −35 and −6

29. What is 2 less than 17?

30. What is 3 less than −20?

1.8 Review Exercises and Sample Exam 169


Chapter 1 Real Numbers and Their Operations

31. Subtract 30 from the sum of 8 and 12.

32. Subtract 7 from the difference of −5 and 7.

33. An airplane flying at 22,000 feet descended 8,500 feet and then ascended
5,000 feet. What is the new altitude of the airplane?

34. The width of a rectangle is 5 inches less than its length. If the length
measures 22 inches, then determine the width.

Multiplying and Dividing Integers

Simplify.

35. 10 ÷5⋅2

36. 36 ÷6⋅2

37. −6 (4) ÷ 2 (−3)

38. 120 ÷ (−5) (−3) (−2)

39. −8 (−5) ÷0

40. −24 (0) ÷8

41. Find the product of −6 and 9.

42. Find the quotient of −54 and −3.

43. James was able to drive 234 miles on 9 gallons of gasoline. How many
miles per gallon did he get?

44. If a bus travels at an average speed of 54 miles per hour for 3 hours, then
how far does the bus travel?

Fractions

Reduce each fraction to lowest terms.

1.8 Review Exercises and Sample Exam 170


Chapter 1 Real Numbers and Their Operations

180
45.
300

252
46.
324

23
47. Convert to a mixed number: .
8

48. Convert to an improper fraction: 3 5


9
.

Simplify.

(− 7 )
3 2
49.
5

(− 3 )
5 1
50. −
8

3 6
51. − ÷ 7
4

4 28
52.
15
÷ 3

53. 4 4
5
÷6

54. 5 ÷8 1
3

5 15
55.
4
÷ 2
⋅6

5 3 5
56.
24
÷ 2
÷ 12

1 1
57.
12
− 4

5 3
58.
6
− 14

3 2 1
59.
4
+ 3
− 12

3 5 1
60.
10
+ 12
− 6

61. Subtract 2
3
from the sum of − 1
2
and 2
9
.

1.8 Review Exercises and Sample Exam 171


Chapter 1 Real Numbers and Their Operations

5 1 7
62. Subtract 6
from the difference of 3
and 2
.

63. If a bus travels at an average speed of 54 miles per hour for 2 1


3
hours,
then how far does the bus travel?

64. Determine the length of fencing needed to enclose a rectangular pen


with dimensions 12 12 feet by 8 34 feet.

Decimals and Percents

65. Write as a mixed number: 5.32.

66. Write as a decimal: 7 3


25
.

Perform the operations.

67. 6.032 + 2.19

68. 12.106 − 9.21

69. 4.23 × 5.13

70. 9.246 ÷ 4.02

Convert to a decimal.

71. 7.2%

72. 5 3
8
%

73. 147%

74. 27 1
2
%

Convert to a percent.

75. 0.055

76. 1.75

1.8 Review Exercises and Sample Exam 172


Chapter 1 Real Numbers and Their Operations

9
77. 10

5
78. 6

79. Mary purchased 3 boxes of t-shirts for a total of $126. If each box
contains 24 t-shirts, then what is the cost of each t-shirt?

80. A retail outlet is offering 12% off the original $39.99 price of tennis shoes.
What is the price after the discount?

81. If an item costs $129.99, then what is the total after adding 7 1
4
% sales
tax?

82. It is estimated that 8.3% of the total student population carpools to


campus each day. If there are 13,000 students, then estimate the number of
students that carpool to campus.

Exponents and Square Roots

Simplify.

2
83. 8

84. (−5)
2

2
85. −4

2
86. −(−3)

87. (
9)
2 2

88. (1 )
2
2
3

3
89. 3

3
90. (−4)

1.8 Review Exercises and Sample Exam 173


Chapter 1 Real Numbers and Their Operations

91. (
5)
2 3

92. (−
6)
1 3

4
93. −(−2)

5
94. −(−1)

⎯⎯⎯⎯
95. √ 49

⎯⎯⎯⎯⎯⎯
96. √ 225

⎯⎯⎯⎯
97. 2√ 25

⎯⎯⎯⎯⎯⎯
98. −√ 121

⎯⎯⎯⎯
99. 3√ 50

⎯⎯⎯⎯
100. −4√ 12

101. √
⎯4⎯⎯
9

102. √
⎯⎯⎯⎯
8

25

103. Calculate the area of a square with sides measuring 3 centimeters.


(A = s2 )

104. Calculate the volume of a cube with sides measuring 3 centimeters.


(V = s3 )

105. Determine the length of the diagonal of a square with sides measuring 3
centimeters.

106. Determine the length of the diagonal of a rectangle with dimensions 2


inches by 4 inches.

1.8 Review Exercises and Sample Exam 174


Chapter 1 Real Numbers and Their Operations

Order of Operations

Simplify.

107. −5 (2) − 72

108. 1 − 4 2 + 2(−3) 2

+ 3(6 − 2 ⋅ 4)
3
109. 2

110. 5 − 3(8 − 3 ⋅ 4)2

111. −2
3
+ 6 (3 2 − 4) + (−3) 2

2
112. 5 − 40 ÷ 5(−2) 2 − (−4)

[ 9 (−3) − 4]
3 2 2 2
113.
4

114. (
2)
1 2 3 9 1
− 4
÷ 16
− 3

2−3(6−3 2 )
2

115.
4⋅5−5 2

(2⋅8−6 ) −10
2 2 2

7 3 −(2(−5) −7)
116. 3

117. 8 − 5 ||3 ⋅ 4 − (−2) 4 ||

118. ||14 − ||−3 − 5 2 ||||

Find the distance between the given numbers on a number line.

119. −14 and 22

120. −42 and −2

1.8 Review Exercises and Sample Exam 175


Chapter 1 Real Numbers and Their Operations

121. 7
8
and − 1
5

122. −5 1
2
and −1 1
4

1.8 Review Exercises and Sample Exam 176


Chapter 1 Real Numbers and Their Operations

SAMPLE EXAM

1. List three integers greater than −10.

2. Determine the unknown(s): || ? || = 13.

3. Fill in the blank with <, =, or >: − |−100| ___ 9 2 .

4. Convert to a fraction: 33 1
3
%.

5. Convert to a percent: 2 3
4
.

75
6. Reduce: .
225

Calculate the following.

2 2 2
7. a. (−7) ; b. −(−7) ; c. −7

3 3 3
8. a. (−3) ; b. −(−3) ; c. −3

9. a. |10| ; b. − |−10| ; c. − |10|

Simplify.

10. − (− (−1))

11. 2
3
+ 1
5
− 3
10

12. 10 − (−12) + (−8) − 20

13. −8 (4) (−3) ÷ 2

14.
1
2
⋅ (− 45 ) ÷ 14
15

3 1 2
15.
5
⋅ 2
− 3

16. 4 ⋅ 5 − 20 ÷ 5 ⋅ 2

1.8 Review Exercises and Sample Exam 177


Chapter 1 Real Numbers and Their Operations

17. 10 − 7 (3 − 8) − 5 2

18. 3 + 2 ||−2 2 − (−1)|| + (−2) 2

[5 − (7 − |−2|) + 15 ⋅ 2 ÷ 3]
1 2
19. 3

20. √
⎯⎯⎯⎯
1

16

⎯⎯⎯⎯
21. 3√ 72

22. Subtract 2 from the sum of 8 and −10.

23. Subtract 10 from the product of 8 and −10.

24. A student earns 9, 8, 10, 7, and 8 points on the first 5 chemistry quizzes.
What is her quiz average?

25. An 8 34 foot plank is cut into 5 pieces of equal length. What is the length
of each piece?

1.8 Review Exercises and Sample Exam 178


Chapter 1 Real Numbers and Their Operations

REVIEW EXERCISES ANSWERS

1:

3:

5: >

7: <

9: >

11: ±2

13: Ø, No Solution

15: −8 ≤0

17: −9 > −10

19: 5

21: −16

23: 9

25: −6

27: 22 units

29: 15

31: −10

1.8 Review Exercises and Sample Exam 179


Chapter 1 Real Numbers and Their Operations

33: 18,500 feet

35: 4

37: 36

39: Undefined

41: −54

43: 26 miles per gallon

3
45:
5

47: 2 7
8

6
49: −
35

7
51: −
8

4
53:
5

55: 1

1
57: −
6

4
59:
3

17
61: −
18

63: 126 miles

65: 5 8
25

67: 8.222

69: 21.6999

1.8 Review Exercises and Sample Exam 180


Chapter 1 Real Numbers and Their Operations

71: 0.072

73: 1.47

75: 5.5%

77: 90%

79: $1.75

81: $139.41

83: 64

85: −16

87: 4/81

89: 27

8
91:
125

93: −16

95: 7

97: 10

⎯⎯
99: 15√ 2

2
101:
3

103: 9 square centimeters

⎯⎯
105: 3√ 2 centimeters

107: −59

109: −22

1.8 Review Exercises and Sample Exam 181


Chapter 1 Real Numbers and Their Operations

111: 31

113: 3

115: 5

117: −12

119: 36 units

43
121: units
40

1.8 Review Exercises and Sample Exam 182


Chapter 1 Real Numbers and Their Operations

SAMPLE EXAM ANSWERS

1: {−5, 0, 5} (answers may vary)

3: <

5: 275%

7:a. 49; b. −49; c. −49

9:a. 10; b. −10; c. −10

17
11:
30

13: 48

11
15: −
30

17: 20

19: 10

⎯⎯
21: 18√ 2

23: −90

25: 1 3
4
feet

1.8 Review Exercises and Sample Exam 183


Chapter 2
Linear Equations and Inequalities

184
Chapter 2 Linear Equations and Inequalities

2.1 Introduction to Algebra

LEARNING OBJECTIVES

1. Identify an algebraic expression and its parts.


2. Evaluate algebraic expressions.
3. Use formulas to solve problems in common applications.

Preliminary Definitions

In algebra, letters are used to represent numbers. The letters used to represent
these numbers are called variables1. Combinations of variables and numbers along
with mathematical operations form algebraic expressions2, or just expressions.
The following are some examples of expressions with one variable, x:

Terms3 in an algebraic expression are separated by addition operators, and factors4


are separated by multiplication operators. The numerical factor of a term is called
the coefficient5. For example, the algebraic expression 3x 2 + 2x − 1 can be
1. Letters or symbols used in
algebra to represent numbers. thought of as 3x 2 + 2x + (−1) and has three terms. The first term, 3x 2 ,
represents the quantity 3 ⋅ x ⋅ x , where 3 is the coefficient and x is the variable. All
2. Combinations of variables and of the variable factors, with their exponents, form the variable part of a term6. If a
numbers along with
mathematical operations used term is written without a variable factor, then it is called a constant term7.
to generalize specific Consider the components of 3x 2 + 2x − 1,
arithmetic operations.

3. Components of an algebraic Terms Coefficient Variable Part


expression separated by
addition operators.
3x 2 3 x2
4. Any of the numbers or
expressions that form a
product.
2x 2 x
5. The numerical factor of a term.

6. All the variable factors with −1 −1


their exponents.

7. A term written without a


variable factor.

185
Chapter 2 Linear Equations and Inequalities

The third term in this expression, −1, is a constant term because it is written
without a variable factor. While a variable represents an unknown quantity and
may change, the constant term does not change.

Example 1: List all coefficients and variable parts of each term: 5x 2 − 4xy − y 2 .

Solution: Think of the third term in this example, −y 2 , as −1y 2 .

Terms Coefficient Variable Part

5x 2 5 x2

−4xy −4 xy

−y 2 −1 y2

Answer: Coefficients: {−4, − 1, 5}; variable parts: {x 2 , xy, y 2 }

Some terms, such as y 2 and −y 2 , appear not to have a coefficient. The


multiplicative identity property states that 1 times anything is itself and occurs so
often that it is customary to omit this factor and write

Therefore, the coefficient of y 2 is actually 1 and the coefficient of −y 2 is −1. In


addition, you will encounter terms that have variable parts composed of algebraic
expressions as factors.

2.1 Introduction to Algebra 186


Chapter 2 Linear Equations and Inequalities

−3(x + y) + (x + y) .
Example 2: List all coefficients and variable parts of each term:
3 2

Solution: This is an expression with two terms:

Terms Coefficient Variable Part

−3(x + y) (x + y)
3 3
−3

(x + y) (x + y)
2 2
1

Answer: Coefficients: {−3, 1}; variable parts: {(x + y) , (x + y) }


3 2

In our study of algebra, we will encounter a wide variety of algebraic expressions.


Typically, expressions use the two most common variables, x and y. However,
expressions may use any letter (or symbol) for a variable, even Greek letters, such
as alpha (α) and beta (β). Some letters and symbols are reserved for constants, such
as π ≈ 3.14159 and e ≈ 2.71828. Since there is only a limited number of letters,
you will also use subscripts, x 1 , x 2 , x 3 , x 4 , …, to indicate different variables.

Try this! List all coefficients and variable parts of the expression:
−5a2 + ab − 2b2 − 3.

Answer: Coefficients: {−5, − 3, − 2, 1}; variable parts: {a2 , ab, b2 }

Video Solution

(click to see video)

2.1 Introduction to Algebra 187


Chapter 2 Linear Equations and Inequalities

Evaluating Algebraic Expressions

Think of an algebraic expression as a generalization of particular arithmetic


operations. Performing these operations after substituting given values for
variables is called evaluating8. In algebra, a variable represents an unknown value.
However, if the problem specifically assigns a value to a variable, then you can
replace that letter with the given number and evaluate using the order of
operations.

Example 3: Evaluate:

a. 2x + 3 , where x = −4

b. 23 y, where y = 9

Solution: To avoid common errors, it is a best practice to first replace all variables
with parentheses and then replace, or substitute9, the given value.

a.

b.

8. The process of performing the


operations of an algebraic
expression for given values of
the variables.

9. The act of replacing a variable


with an equivalent quantity.

2.1 Introduction to Algebra 188


Chapter 2 Linear Equations and Inequalities

Answers: a. −5; b. 6

If parentheses are not used in part (a) of the previous example, the result is quite
different: 2x + 3 = 2 − 4 + 4. Without parentheses, the first operation is
subtraction, which leads to an incorrect result.

Example 4: Evaluate: −2x − y , where x = −5 and y = −3 .

Solution: After substituting the given values for the variables, simplify using the
order of operations.

Answer: 13

Example 5: Evaluate: 9a2 − b2 , where a = 2 and b = −5.

Solution:

2.1 Introduction to Algebra 189


Chapter 2 Linear Equations and Inequalities

Answer: 11

Example 6: Evaluate: −x 2 − 4x + 1, where x = − 12 .

Solution:

Answer: 11/4

The answer to the previous example is 114, which can be written as a mixed number
2 34. In algebra, improper fractions are generally preferred. Unless the original
problem has mixed numbers in it, or it is an answer to a real-world application,
solutions will be expressed as reduced improper fractions.

Example 7: Evaluate: (3x − 2) (x − 7), where x = 23 .

Solution: The order of operations requires us to perform the operations within the
parentheses first.

2.1 Introduction to Algebra 190


Chapter 2 Linear Equations and Inequalities

Answer: 0

Example 8: Evaluate: b2 − 4ac, where a = −1, b = −3, and c = 2.

Solution: The expression b2 − 4ac is called the discriminant10; it is an essential


quantity seen later in our study of algebra.

Answer: 17

Try this! Evaluate a3 − b3 , where a = 2 and b = −3.

Answer: 35
10. The algebraic expression
b2 − 4ac.

2.1 Introduction to Algebra 191


Chapter 2 Linear Equations and Inequalities

Video Solution

(click to see video)


Using Formulas

The main difference between algebra and arithmetic is the organized use of
variables. This idea leads to reusable formulas11, which are mathematical models
using algebraic expressions to describe common applications. For example, the area
of a rectangle is modeled by the formula:

In this equation, variables are used to describe the relationship between the area of
a rectangle and the length of its sides. The area is the product of the length and
width of the rectangle. If the length of a rectangle measures 3 meters and the width
measures 2 meters, then the area can be calculated using the formula as follows:

Example 9: The cost of a daily truck rental is $48.00 plus an additional $0.45 for
every mile driven. This cost in dollars can be modeled by the formula
cost = 0.45x + 48, where x represents the number of miles driven in one day. Use
this formula to calculate the cost to rent the truck for a day and drive it 120 miles.

Solution: Use the formula to find the cost when the number of miles x = 120.

11. A reusable mathematical model


using algebraic expressions to Substitute 120 into the given formula for x and then simplify.
describe a common
application.

2.1 Introduction to Algebra 192


Chapter 2 Linear Equations and Inequalities

Answer: The rental costs $102.

Uniform motion12 is modeled by the formula D = rt, which expresses distance D in


terms of the average rate r, or speed, and the time t traveled at that rate. This
formula, D = rt, is used often and is read “distance equals rate times time.”

Example 10: Jim’s road trip takes 2 12 hours at an average speed of 66 miles per
hour. How far does he travel?

Solution: Substitute the appropriate values into the formula and then simplify.

Answer: Jim travels 165 miles.

12. Described by the formula


D = rt, where the distance D
is given as the product of the The volume in cubic units of a rectangular box is given by the formula V = lwh ,
average rate r and the time t where l represents the length, w represents the width, and h represents the height.
traveled at that rate.

2.1 Introduction to Algebra 193


Chapter 2 Linear Equations and Inequalities

Example 11: A wooden box is 1 foot in length, 5 inches wide, and 6 inches high.
Find the volume of the box in cubic inches.

Solution: Take care to ensure that all the units are consistent and use 12 inches for
the length instead of 1 foot.

Answer: The volume of the box is 360 cubic inches.

Simple interest13 I is given by the formula I = prt, where p represents the


principal amount invested at an annual interest rate r for t years.

Example 12: Calculate the simple interest earned on a 2-year investment of $1,250
at an annual interest rate of 3 34 %.
13. Modeled by the formula
I = prt, where p represents
the principal amount invested Solution: Convert 3 3
4
%to a decimal number before using it in the formula.
at an annual interest rate r for
t years.

2.1 Introduction to Algebra 194


Chapter 2 Linear Equations and Inequalities

Use this value for r and the fact that p = $1,250 and t = 2 years to calculate the simple
interest.

Answer: The simple interest earned is $93.75.

Try this! The perimeter of a rectangle is given by the formula P = 2l + 2w, where
l represents the length and w represents the width. Use the formula to calculate the
perimeter of a rectangle with a length of 5 feet and a width of 2 12 feet.

Answer: 15 feet

Video Solution

(click to see video)

KEY TAKEAWAYS

• Think of algebraic expressions as generalizations of common arithmetic


operations that are formed by combining numbers, variables, and
mathematical operations.
• It is customary to omit the coefficient if it is 1, as in x 2 = 1x 2 .
• To avoid common errors when evaluating, it is a best practice to replace
all variables with parentheses and then substitute the appropriate
values.
• The use of algebraic expressions allows us to create useful and reusable
formulas that model common applications.

2.1 Introduction to Algebra 195


Chapter 2 Linear Equations and Inequalities

TOPIC EXERCISES

Part A: Definitions

List all of the coefficients and variable parts of the following expressions.

1. 4x −1

2. – 7x 2 − 2x + 1

3. −x 2 + 5x − 3
2
4. 3x 2 y 2 − xy + 7
3

1 1 5
5.
3
y2 − 2
y+ 7

6. −4a2 b + 5ab 2 − ab + 1

7. 2(a + b) − 3(a + b)
3 5

8. 5(x + 2) 2 − 2 (x + 2) − 7

9. m 2 n − mn 2 + 10mn − 27

10. x 4 − 2x 3 − 3x 2 − 4x − 1

Part B: Evaluating Algebraic Expressions

Evaluate.

11. x + 3 , where x = −4

12. 2x − 3 , where x = −3

13. −5x + 20 , where x = 4

14. −5y , where y = −1

2.1 Introduction to Algebra 196


Chapter 2 Linear Equations and Inequalities

3
15.
4
a, where a = 32

16. 2(a − 4), where a = −1

17. −10(5 − z) , where z = 14


1
18. 5y − 1 , where y = − 5

1
19. −2a + 1, where a = − 3

3
20. 4x + 3 , where x = 16

1
21. −x + , where x = −2
2

2 1 1
22.
3
x− 2
, where x =− 4

2
For each problem below, evaluate b − 4ac, given the following values for a, b,
and c.

23. a = 1, b = 2, c = 3

24. a = 3, b =– 4 , c =– 1

25. a =– 6, b = 0, c =– 2
1 2
26. a = ,b = 1, c =
2 3

1 1
27. a = −3, b = − ,c =
2 9

2
28. a = −13, b = − ,c =0
3

Evaluate.

29. −4xy 2 , where x = −3 and y = 2


5
30.
8
x 2 y , where x = −1 and y = 16

2.1 Introduction to Algebra 197


Chapter 2 Linear Equations and Inequalities

31. a2 − b 2, where a = 2 and b = 3.

32. a2 − b 2, where a = −1 and b = −2


1 1
33. x 2 − y 2, where x = and y =−
2 2

34. 3x 2 − 5x + 1 , where x = −3

35. y 2 − y − 6, where y = 0
1
36. 1 − y 2 , where y = − 2

37. (x + 3) (x − 2) , where x = −4

38. (y − 5) (y + 6) , where y = 5

39. 3 (α − β) + 4, where α = −1 and β = 6

40. 3α2 − β 2 , where α = 2 and β = −3

41. Evaluate 4 (x + h) , given x = 5 and h = 0.01 .

42. Evaluate −2 (x + h) + 3 , given x = 3 and h = 0.1 .

43. Evaluate 2(x + h) − 5 (x + h) + 3 , given x = 2 and h = 0.1 .


2

44. Evaluate 3(x + h) + 2 (x + h) − 1 , given x = 1 and


2

h = 0.01 .

Part C: Using Formulas

Convert the following temperatures to degrees Celsius given C = 5


9
(F − 32),
where F represents degrees Fahrenheit.

45. 86°F

2.1 Introduction to Algebra 198


Chapter 2 Linear Equations and Inequalities

46. 95°F

47. −13°F

48. 14°F

49. 32°F

50. 0°F

Given the base and height of a triangle, calculate the area. (A = 1


2
bh )

51. b = 25 centimeters and h = 10 centimeters

52. b = 40 inches and h = 6 inches


1
53. b = foot and h = 2 feet
2

3 5
54. b = inches and h = inches
4 8

55. A certain cellular phone plan charges $23.00 per month plus $0.09 for
each minute of usage. The monthly charge is given by the formula
monthly charge = 0.09x + 23 , where x represents the number of
minutes of usage per month. What is the charge for a month with 5 hours of
usage?

56. A taxi service charges $3.75 plus $1.15 per mile given by the formula
charge = 1.15x + 3.75 , where x represents the number of miles
driven. What is the charge for a 17-mile ride?

57. If a calculator is sold for $14.95, then the revenue in dollars, R, generated
by this item is given by the formula R = 14.95q , where q represents the
number of calculators sold. Use the formula to determine the revenue
generated by this item if 35 calculators are sold.

58. Yearly subscriptions to a tutoring website can be sold for $49.95. The
revenue in dollars, R, generated by subscription sales is given by the formula
R = 49.95q , where q represents the number of yearly subscriptions sold.
Use the formula to calculate the revenue generated by selling 250
subscriptions.

2.1 Introduction to Algebra 199


Chapter 2 Linear Equations and Inequalities

59. The cost of producing pens with the company logo printed on them
consists of a onetime setup fee of $175 plus $0.85 for each pen produced.
This cost can be calculated using the formula C = 175 + 0.85q , where q
represents the number of pens produced. Use the formula to calculate the
cost of producing 2,000 pens.

60. The cost of producing a subscription website consists of an initial


programming and setup fee of $4,500 plus a monthly Web hosting fee of
$29.95. The cost of creating and hosting the website can be calculated using
the formula C = 4500 + 29.95n , where n represents the number of
months the website is hosted. How much will it cost to set up and host the
website for 1 year?

61. The perimeter of a rectangle is given by the formula P = 2l + 2w ,


where l represents the length and w represents the width. What is the
perimeter of a fenced-in rectangular yard measuring 70 feet by 100 feet?

62. Calculate the perimeter of an 8-by-10-inch picture.

63. Calculate the perimeter of a room that measures 12 feet by 18 feet.

64. A computer monitor measures 57.3 centimeters in length and 40.9


centimeters high. Calculate the perimeter.

65. The formula for the area of a rectangle in square units is given by
A = l ⋅ w , where l represents the length and w represents the width. Use
this formula to calculate the area of a rectangle with length 12 centimeters
and width 3 centimeters.

66. Calculate the area of an 8-by-12-inch picture.

67. Calculate the area of a room that measures 12 feet by 18 feet.

68. A computer monitor measures 57.3 centimeters in length and 40.9


centimeters in height. Calculate the total area of the screen.

69. A concrete slab is poured in the shape of a rectangle for a shed


measuring 8 feet by 10 feet. Determine the area and perimeter of the slab.

70. Each side of a square deck measures 8 feet. Determine the area and
perimeter of the deck.

2.1 Introduction to Algebra 200


Chapter 2 Linear Equations and Inequalities

71. The volume of a rectangular solid is given by V = lwh , where l


represents the length, w represents the width, and h is the height of the
solid. Find the volume of a rectangular solid if the length is 2 inches, the
width is 3 inches, and the height is 4 inches.

72. If a trunk measures 3 feet by 2 feet and is 2½ feet tall, then what is the
volume of the trunk?

73. The interior of an industrial freezer measures 3 feet wide by 3 feet deep
and 4 feet high. What is the volume of the freezer?

74. A laptop case measures 1 feet 2 inches by 10 inches by 2 inches. What is


the volume of the case?

75. If the trip from Fresno to Sacramento can be made by car in 2½ hours at
an average speed of 67 miles per hour, then how far is Sacramento from
Fresno?

76. A high-speed train averages 170 miles per hour. How far can it travel in
1½ hours?

77. A jumbo jet can cruises at an average speed of 550 miles per hour. How
far can it travel in 4 hours?

78. A fighter jet reaches a top speed of 1,316 miles per hour. How far will the
jet travel if it can sustain this speed for 15 minutes?

79. The Hubble Space Telescope is in low earth orbit traveling at an average
speed of 16,950 miles per hour. What distance does it travel in 1½ hours?

80. Earth orbits the sun a speed of about 66,600 miles per hour. How far does
earth travel around the sun in 1 day?

81. Calculate the simple interest earned on a $2,500 investment at 3% annual


interest rate for 4 years.

82. Calculate the simple interest earned on a $1,000 investment at 5% annual


interest rate for 20 years.

83. How much simple interest is earned on a $3,200 investment at a 2.4%


annual interest for 1 year?

2.1 Introduction to Algebra 201


Chapter 2 Linear Equations and Inequalities

84. How much simple interest is earned on a $500 investment at a 5.9%


annual interest rate for 3 years?

85. Calculate the simple interest earned on a $10,500 investment at a 4 1


4
%
annual interest rate for 4 years.

86. Calculate the simple interest earned on a $6,250 investment at a 6 3


4
%
annual interest rate for 1 year.

Part D: Discussion Board Topics

87. Research and discuss the history of the symbols for addition (+) and
subtraction (−).

88. What are mathematical models and why are they useful in everyday life?

89. Find and post a useful formula. Demonstrate its use with some values.

90. Discuss the history and importance of the variable. How can you denote a
variable when you run out of letters?

91. Find and post a useful resource describing the Greek alphabet.

2.1 Introduction to Algebra 202


Chapter 2 Linear Equations and Inequalities

ANSWERS

1: Coefficients: {−1, 4} ; variable parts: {x}

3: Coefficients: {−3, − 1, 5} ; variable parts: {x 2 , x}

5: Coefficients: {− 7}
; variable parts: {y 2 , y}
1 1 5
2
, 3
,

7: Coefficients: {−3, 2} ; variable parts: {(a + b) , (a + b) }


3 5

9: Coefficients: {−27, − 1, 1, 10} ; variable parts:


{m n, mn , mn}
2 2

11: −1

13: 0

15: 24

17: 90

19: 5/3

21: 5/2

23: −8

25: −48

27: 19/12

29: 48

31: −5

33: 0

2.1 Introduction to Algebra 203


Chapter 2 Linear Equations and Inequalities

35: −6

37: 6

39: −17

41: 20.04

43: 1.32

45: 30°C

47: −25°C

49: 0°C

51: 125 square centimeters

53: 1/2 square feet

55: $50

57: $523.25

59: $1,875.00

61: 340 feet

63: 60 feet

65: 36 square centimeters

67: 216 square feet

69: Area: 80 square feet; Perimeter: 36 feet

71: 24 cubic inches

73: 36 cubic feet

2.1 Introduction to Algebra 204


Chapter 2 Linear Equations and Inequalities

75: 167.5 miles

77: 2,200 miles

79: 25,425 miles

81: $300

83: $76.80

85: $1,785

2.1 Introduction to Algebra 205


Chapter 2 Linear Equations and Inequalities

2.2 Simplifying Algebraic Expressions

LEARNING OBJECTIVES

1. Apply the distributive property to simplify an algebraic expression.


2. Identify and combine like terms.

Distributive Property

The properties of real numbers are important in our study of algebra because a
variable is simply a letter that represents a real number. In particular, the
distributive property14 states that given any real numbers a, b, and c,

This property is applied when simplifying algebraic expressions. To demonstrate


how it is used, we simplify 2(5 − 3) in two ways, and observe the same correct
result.

Certainly, if the contents of the parentheses can be simplified, do that first. On the
other hand, when the contents of parentheses cannot be simplified, multiply every
term within the parentheses by the factor outside of the parentheses using the
distributive property. Applying the distributive property allows you to multiply and
remove the parentheses.

and c, a (b + c) = ab + ac
14. Given any real numbers a, b,

or (b + c) a = ba + ca. Example 1: Simplify: 5 (7y + 2).

206
Chapter 2 Linear Equations and Inequalities

Solution: Multiply 5 times each term inside the parentheses.

Answer: 35y + 10

Example 2: Simplify: −3 (2x 2 + 5x + 1).

Solution: Multiply −3 times each of the coefficients of the terms inside the
parentheses.

Answer: −6x 2 − 15x − 3

Example 3: Simplify: 5 (−2a + 5b) − 2c.

Solution: Apply the distributive property by multiplying only the terms grouped
within the parentheses by 5.

Answer: −10a + 25b − 2c

2.2 Simplifying Algebraic Expressions 207


Chapter 2 Linear Equations and Inequalities

in the following manner: (b + c) a = ba + ca.


Because multiplication is commutative, we can also write the distributive property

Example 4: Simplify: (3x − 4y + 1) ⋅ 3.

Solution: Multiply each term within the parentheses by 3.

Answer: 9x − 12y + 3

Division in algebra is often indicated using the fraction bar rather than with the
symbol (÷). And sometimes it is useful to rewrite expressions involving division as
products:

Rewriting algebraic expressions as products allows us to apply the distributive


property.

2.2 Simplifying Algebraic Expressions 208


Chapter 2 Linear Equations and Inequalities

25x 2 −5x+10
Example 5: Divide: 5
.

Solution: First, treat this as 15 times the expression in the numerator and then
distribute.

Alternate Solution: Think of 5 as a common denominator and divide each of the


terms in the numerator by 5:

Answer: 5x 2 − x + 2

We will discuss the division of algebraic expressions in more detail as we progress


through the course.

Try this! Simplify: 13 (−9x + 27y − 3).

Answer: −3x + 9y − 1

2.2 Simplifying Algebraic Expressions 209


Chapter 2 Linear Equations and Inequalities

Video Solution

(click to see video)


Combining Like Terms

Terms with the same variable parts are called like terms15, or similar terms16.
Furthermore, constant terms are considered to be like terms. If an algebraic
expression contains like terms, apply the distributive property as follows:

In other words, if the variable parts of terms are exactly the same, then we may add
or subtract the coefficients to obtain the coefficient of a single term with the same
variable part. This process is called combining like terms17. For example,

Notice that the variable factors and their exponents do not change. Combining like
terms in this manner, so that the expression contains no other similar terms, is
called simplifying the expression18. Use this idea to simplify algebraic expressions
with multiple like terms.

15. Constant terms or terms with


the same variable parts.
Example 6: Simplify: 3a + 2b − 4a + 9b.
16. Used when referring to like
terms.
Solution: Identify the like terms and combine them.
17. Adding or subtracting like
terms within an algebraic
expression to obtain a single
term with the same variable
part.

18. The process of combining like


terms until the expression
contains no more similar
terms.

2.2 Simplifying Algebraic Expressions 210


Chapter 2 Linear Equations and Inequalities

Answer: −a + 11b

In the previous example, rearranging the terms is typically performed mentally and
is not shown in the presentation of the solution.

Example 7: Simplify: x 2 + 3x + 2 + 4x 2 − 5x − 7.

Solution: Identify the like terms and add the corresponding coefficients.

Answer: 5x 2 − 2x − 5

Example 8: Simplify: 5x 2 y − 3xy 2 + 4x 2 y − 2xy 2 .

Solution: Remember to leave the variable factors and their exponents unchanged
in the resulting combined term.

Answer: 9x 2 y − 5xy 2

Example 9: Simplify: 12 a − 13 b + 34 a + .b

2.2 Simplifying Algebraic Expressions 211


Chapter 2 Linear Equations and Inequalities

Solution: To add the fractional coefficients, use equivalent coefficients with


common denominators for each like term.

Answer: 54 a + 23 b

Example 10: Simplify: −12x(x + y) + 26x(x + y) .


3 3

Solution: Consider the variable part to be x(x + y) . Then this expression has two
3

like terms with coefficients −12 and 26.

Answer: 14x(x + y)
3

Try this! Simplify: −7x + 8y − 2x − 3y .

Answer: −9x + 5y

Video Solution

(click to see video)

2.2 Simplifying Algebraic Expressions 212


Chapter 2 Linear Equations and Inequalities

Distributive Property and Like Terms

When simplifying, we will often have to combine like terms after we apply the
distributive property. This step is consistent with the order of operations:
multiplication before addition.

Example 11: Simplify: 2 (3a − b) − 7 (−2a + 3b).

Solution: Distribute 2 and −7 and then combine like terms.

Answer: 20a − 23b

In the example above, it is important to point out that you can remove the
parentheses and collect like terms because you multiply the second quantity by −7,

times the given quantity, 2 (3a − b) + (−7) (−2a + 3b).


not just by 7. To correctly apply the distributive property, think of this as adding −7

Try this! Simplify: −5 (2x − 3) + 7x.

Answer: −3x + 15

Video Solution

(click to see video)

2.2 Simplifying Algebraic Expressions 213


Chapter 2 Linear Equations and Inequalities

Often we will encounter algebraic expressions like + (a + b) or − (a + b). As we


have seen, the coefficients are actually implied to be +1 and −1, respectively, and
therefore, the distributive property applies using +1 or –1 as the factor. Multiply
each term within the parentheses by these factors:

This leads to two useful properties,

Example 12: Simplify: 5x − (−2x 2 + 3x − 1).

Solution: Multiply each term within the parentheses by −1 and then combine like
terms.

Answer: 2x 2 + 2x + 1

When distributing a negative number, all of the signs within the parentheses will
change. Note that 5x in the example above is a separate term; hence the
distributive property does not apply to it.

2.2 Simplifying Algebraic Expressions 214


Chapter 2 Linear Equations and Inequalities

Example 13: Simplify: 5 − 2 (x 2 − 4x − 3).

Solution: The order of operations requires that we multiply before subtracting.


Therefore, distribute −2 and then combine the constant terms. Subtracting 5 − 2
first leads to an incorrect result, as illustrated below:

Answer: −2x2 + 8x + 11

Caution

It is worth repeating that you must follow the order of operations: multiply and
divide before adding and subtracting!

Try this! Simplify: 8 − 3 (−x 2 + 2x − 7).

Answer: 3x 2 − 6x + 29

Video Solution

(click to see video)

Example 14: Subtract 3x − 2 from twice the quantity −4x 2 + 2x − 8.

2.2 Simplifying Algebraic Expressions 215


Chapter 2 Linear Equations and Inequalities

Solution: First, group each expression and treat each as a quantity:

Next, identify the key words and translate them into a mathematical expression.

Finally, simplify the resulting expression.

Answer: −8x 2 + x − 14

KEY TAKEAWAYS

• The properties of real numbers apply to algebraic expressions, because


variables are simply representations of unknown real numbers.
• Combine like terms, or terms with the same variable part, to simplify
expressions.

expressions, a (b + c) = ab + ac.
• Use the distributive property when multiplying grouped algebraic

• It is a best practice to apply the distributive property only when the


expression within the grouping is completely simplified.
• After applying the distributive property, eliminate the parentheses and
then combine any like terms.
• Always use the order of operations when simplifying.

2.2 Simplifying Algebraic Expressions 216


Chapter 2 Linear Equations and Inequalities

TOPIC EXERCISES

Part A: Distributive Property

Multiply.

1. 3 (3x − 2)

2. 12(−5y + 1)

3. −2 (x + 1)

4. 5(a − b)

(8x − 16)
5
5.
8

(10x − 5)
3
6. −
5

7. (2x + 3) ⋅ 2

8. (5x − 1) ⋅ 5

9. (−x + 7) (−3)

10. (−8x + 1) (−2)

11. −(2a − 3b)

12. −(x − 1)
1
13.
3
(2x + 5)

3
14. − (y − 2)
4

15. −3 (2a + 5b − c)

16. − (2y 2 − 5y + 7)

2.2 Simplifying Algebraic Expressions 217


Chapter 2 Linear Equations and Inequalities

17. 5 (y 2 − 6y − 9)

18. −6 (5x 2 + 2x − 1)

19. 7x 2 − (3x − 11)

20. − (2a − 3b) + c

21. 3 (7x 2 − 2x) − 3

(4a − 6a + 4)
1 2
22.
2

(9y − 3y + 27)
1 2
23. −
3

24. (5x 2 − 7x + 9) (−5)

25. 6 ( x + 12 )
1 1
3
x2 − 6

26. −2 (3x 3 − 2x 2 + x − 3)

20x+30y−10z
27.
10

−4a+20b−8c
28.
4

3x 2 −9x+81
29.
−3

−15y 2 +20y−5
30.
5

Translate the following sentences into algebraic expressions and then simplify.

31. Simplify two times the expression 25x 2 − 9.

32. Simplify the opposite of the expression 6x 2 + 5x − 1 .

2.2 Simplifying Algebraic Expressions 218


Chapter 2 Linear Equations and Inequalities

33. Simplify the product of 5 and x 2 − 8.

34. Simplify the product of −3 and −2x 2 + x − 8.

Part B: Combining Like Terms

Simplify.

35. 2x − 3x

36. −2a + 5a − 12a

37. 10y − 30 − 15y


1 5
38.
3
x+ 12
x

1 4 3
39. − x+ + x
4 5 8

40. 2x − 4x + 7x − x

41. −3y − 2y + 10y − 4y

42. 5x − 7x + 8y + 2y

43. −8α + 2β − 5α − 6β

44. −6α + 7β − 2α + β

45. 3x + 5 − 2y + 7 − 5x + 3y

46. – y + 8x − 3 + 14x + 1 − y

47. 4xy − 6 + 2xy + 8

48. −12ab − 3 + 4ab − 20


1 2 2 3
49.
3
x− 5
y+ 3
x− 5
y

2.2 Simplifying Algebraic Expressions 219


Chapter 2 Linear Equations and Inequalities

3 2 1 3
50.
8
a− 7
b− 4
a+ 14
b

51. −4x 2 − 3xy + 7 + 4x 2 − 5xy − 3

52. x 2 + y 2 − 2xy − x 2 + 5xy − y 2

53. x 2 − y 2 + 2x 2 − 3y
1 2 1 1
54.
2
x2 − 3
y2 − 8
x2 + 5
y2

3 4 1 1
55.
16
a2 − 5
+ 4
a2 − 4

1 3 7 1
56.
5
y2 − 4
+ 10
y2 − 2

57. 6x 2 y − 3xy 2 + 2x 2 y − 5xy 2

58. 12x 2 y 2 + 3xy − 13x 2 y 2 + 10xy


2
59. −ab + a2 b − 2ab 2 + 5a2 b

60. m 2 n 2 − mn + mn − 3m 2 n + 4m 2 n 2

61. 2(x + y) + 3(x + y)


2 2

1 2
62.
5
(x + 2) 3 − 3
(x + 2) 3

63. −3x (x 2 − 1) + 5x (x 2 − 1)

64. 5 (x − 3) − 8 (x − 3)

65. −14 (2x + 7) + 6 (2x + 7)

66. 4xy(x + 2) 2 − 9xy(x + 2) 2 + xy(x + 2) 2

Part C: Mixed Practice

2.2 Simplifying Algebraic Expressions 220


Chapter 2 Linear Equations and Inequalities

Simplify.

67. 5 (2x − 3) + 7

68. −2 (4y + 2) − 3y

69. 5x − 2(4x − 5)

70. 3 − (2x + 7)

71. 2x − (3x − 4y − 1)

72. (10y − 8) − (40x + 20y − 7)

73.
1
2
y− 3
4
x − ( 23 y − 1
5
x)

1 3 3 1
74.
5
a− 4
b+ 15
a− 2
b

(x − y) + x − 2y
2
75.
3

(6x − 1) + (4y − 1) − (−2x + 2y − 6 )


1 1 1
76. −
3 2

77. (2x 2 − 7x + 1) + (x 2 + 7x − 5)

78. 6 (−2x 2 + 3x − 1) + 10x 2 − 5x

79. − (x 2 − 3x + 8) + x 2 − 12

80. 2 (3a − 4b) + 4 (−2a + 3b)

81. −7 (10x − 7y) − 6 (8x + 4y)

82. 10 (6x − 9) − (80x − 35)

83. 10 − 5 (x 2 − 3x − 1)

2.2 Simplifying Algebraic Expressions 221


Chapter 2 Linear Equations and Inequalities

84. 4 + 6 (y 2 − 9)

85.
3
4
x − ( 12 x 2 + 2
3
x − 75 )

86. −
7
3
x 2 + (− 1
6
x 2 + 7x − 1)

87. (2y 2 − 3y + 1) − (5y 2 + 10y − 7)

88. (−10a2 − b 2 + c) + (12a2 + b 2 − 4c)

89. −4 (2x 2 + 3x − 2) + 5 (x 2 − 4x − 1)

90. 2 (3x 2 − 7x + 1) − 3 (x 2 + 5x − 1)

91. x 2 y + 3xy 2 − (2x 2 y − xy 2 )

92. 3 (x 2 y 2 − 12xy) − (7x 2 y 2 − 20xy + 18)

93. 3 − 5 (ab − 3) + 2 (ba − 4)

94. −9 − 2 (xy + 7) − (yx − 1)

95. −5 (4α − 2β + 1) + 10 (α − 3β + 2)

(100α − 50αβ + 2β ) − (50α + 10αβ − 5β )


1 2 2 1 2 2
96.
2 5

Translate the following sentences into algebraic expressions and then simplify.

97. What is the difference of 3x −4 and −2x + 5?

98. Subtract 2x −3 from 5x + 7.

99. Subtract 4x +3 from twice the quantity x − 2.

100. Subtract three times the quantity −x + 8 from 10x − 9 .

2.2 Simplifying Algebraic Expressions 222


Chapter 2 Linear Equations and Inequalities

Part D: Discussion Board Topics

101. Do we need a distributive property for division, (a + b) ÷ c? Explain.

102. Do we need a separate distributive property for three terms,


a(b + c + d)? Explain.

103. Explain how to subtract one expression from another. Give some
examples and demonstrate the importance of the order in which subtraction
is performed.

104. Given the algebraic expression 8 − 5(3x + 4) , explain why


subtracting 8 − 5 is not the first step.

105. Can you apply the distributive property to the expression 5(abc)?
Explain why or why not and give some examples.

106. How can you check to see if you have simplified an expression
correctly? Give some examples.

2.2 Simplifying Algebraic Expressions 223


Chapter 2 Linear Equations and Inequalities

ANSWERS

1: 9x −6

3: −2x −2

5: 5x − 10

7: 4x +6

9: 3x − 21

11: −2a + 3b
2 5
13:
3
x+ 3

15: −6a − 15b + 3c

17: 5y 2 − 30y − 45

19: 7x 2 − 3x + 11

21: 21x 2 − 6x − 3

23: −3y 2 +y−9

25: 2x 2 −x+3

27: 2x + 3y − z

29: −x 2 + 3x − 27

31: 50x 2 − 18

33: 5x 2 − 40

35: −x

2.2 Simplifying Algebraic Expressions 224


Chapter 2 Linear Equations and Inequalities

37: −5y − 30
1 4
39:
8
x+ 5

41: y

43: −13α − 4β

45: −2x + y + 12

47: 6xy +2

49: x −y

51: −8xy +4

53: 3x 2 − y 2 − 3y
7 21
55:
16
a2 − 20

57: 8x 2 y − 8xy 2

59: 6a2 b − 3ab 2

61: 5(x + y)
2

63: 2x (x 2 − 1)

65: −8 (2x + 7)

67: 10x −8

69: −3x + 10

71: −x + 4y + 1
11 1
73: − x− y
20 6

2.2 Simplifying Algebraic Expressions 225


Chapter 2 Linear Equations and Inequalities

5 8
75:
3
x− 3
y

77: 3x 2 −4

79: 3x − 20

81: −118x + 25y

83: −5x 2 + 15x + 15


1 1 7
85: −
2
x2 + 12
x+ 5

87: −3y 2 − 13y + 8

89: −3x 2 − 32x + 3

91: −x 2 y + 4xy 2

93: −3ab + 10

95: −10α − 20β + 15

97: 5x −9

99: −2x −7

2.2 Simplifying Algebraic Expressions 226


Chapter 2 Linear Equations and Inequalities

2.3 Solving Linear Equations: Part I

LEARNING OBJECTIVES

1. Identify linear equations with one variable and verify their solutions.
2. Use the properties of equality to solve basic linear equations.
3. Use multiple steps to solve linear equations by isolating the variable.
4. Solve linear equations where the coefficients are fractions or decimals.

Linear Equations with One Variable and Their Solutions

Learning how to solve various algebraic equations is one of the main goals in
algebra. This section introduces the basic techniques used for solving linear
equations with one variable.

An equation19 is a statement indicating that two algebraic expressions are equal. A


linear equation with one variable20, x, is an equation that can be written in the
general form ax + b = 0, where a and b are real numbers and a ≠ 0. Here are
some examples of linear equations, all of which are solved in this section:

A solution21 to a linear equation is any value that can replace the variable to
produce a true statement. The variable in the linear equation 2x + 3 = 13 is x, and
the solution is x = 5 . To verify this, substitute the value 5 for x and check that you
obtain a true statement.

19. Statement indicating that two


algebraic expressions are
equal.

20. An equation that can be


written in the general form
ax + b = 0, where a and b
are real numbers and a ≠ 0.

21. Any value that can replace the


variable in an equation to
produce a true statement.

227
Chapter 2 Linear Equations and Inequalities

Alternatively, when an equation is equal to a constant, we can verify a solution by


substituting the value for the variable and show that the result is equal to that
constant. In this sense, we say that solutions satisfy the equation22.

Example 1: Is x = 3 a solution to −2x − 3 = −9?

Answer: Yes, it is a solution, because x = 3 satisfies the equation.

Example 2: Is a = − 12 a solution to −10a + 5 = 25?

Answer: No, it is not a solution, because a = − 12 does not satisfy the equation.

Recall that when evaluating expressions, it is a good practice to first replace all
variables with parentheses, then substitute the appropriate values. By making use
of parentheses we avoid some common errors when working the order of
operations.

Example 3: Is y = −3 a solution to 2y − 5 = −y − 14 ?

22. After replacing the variable Solution:


with a solution and
simplifying, it produces a true
statement.

2.3 Solving Linear Equations: Part I 228


Chapter 2 Linear Equations and Inequalities

Answer: Yes, it is a solution, because y = −3 produces a true statement.

Try this! Is x = −3 a solution to −2x + 5 = −1?

Answer: No

Video Solution

(click to see video)


Solving Basic Linear Equations

We begin by defining equivalent equations23 as equations with the same solution


set. Consider the following two linear equations and check to see if the solution is
x = 7.

Here we can see that the two linear equations 3x − 5 = 16 and 3x = 21 are
equivalent because they share the same solution set, namely, {7}. The goal is to
develop a systematic process to find equivalent equations until the variable is
23. Equations with the same isolated:
solution set.

2.3 Solving Linear Equations: Part I 229


Chapter 2 Linear Equations and Inequalities

To do this, use the properties of equality24. Given algebraic expressions A and B,


where c is a real number, we have the following:

Note

Multiplying or dividing both sides of an equation by 0 is carefully avoided.


Dividing by 0 is undefined and multiplying both sides by 0 results in the
equation 0 = 0.

To summarize, equality is retained and you obtain an


equivalent equation if you add, subtract, multiply, or
divide both sides of an equation by any nonzero real
number. The technique for solving linear equations
involves applying these properties in order to isolate the
variable on one side of the equation. If the linear
equation has a constant term, then we add to or
subtract it from both sides of the equation to obtain an
equivalent equation where the variable term is isolated.

24. Properties that allow us to


obtain equivalent equations by
adding, subtracting,
multiplying, and dividing both
sides of an equation by Example 4: Solve: x + 3 = −5.
nonzero real numbers.

2.3 Solving Linear Equations: Part I 230


Chapter 2 Linear Equations and Inequalities

Solution: To isolate the variable x on the left side, subtract 3 from both sides.

Answer: The solution is x = −8 . To check that this is true, substitute −8 into the
original equation and simplify to see that it is satisfied: x + 3 = −8 + 3 = −5 ✓.

In the previous example, after subtracting 3 from both sides, you get x + 0 = −8.
By the additive identity property of real numbers, this is equivalent to x = −8 .
This step is often left out in the presentation of the solution.

If the variable term of the equation (including the coefficient) is isolated, then apply
the multiplication or division property of equality to obtain an equivalent equation
with the variable isolated. In other words, our goal is to obtain an equivalent
equation with x or 1x isolated on one side of the equal sign.

Example 5: Solve: −5x = −35.

Solution: The coefficient of x is –5, so divide both sides by −5.

Answer: The solution is x = 7 . Perform the check mentally by substituting 7 for x in


the original equation.

2.3 Solving Linear Equations: Part I 231


Chapter 2 Linear Equations and Inequalities

In the previous example, after dividing both sides by −5, x is left with a coefficient of
1, because −5−5
= 1. In fact, when we say “isolate the variable,” we mean to change
the coefficient of the variable to 1, because 1x = 7 is equivalent to x = 7 . This step
is often left out of the instructional examples even though its omission is sometimes
a source of confusion.

Another important property is the symmetric property25: for any algebraic


expressions A and B,

The equation 2 = x is equivalent to x = 2 . It does not matter on which side we


choose to isolate the variable.

Example 6: Solve: 2 = 5 + x.

Solution: Isolate the variable x by subtracting 5 from both sides of the equation.

Answer: The solution is −3, and checking the solution shows that 2 = 5 − 3.

25. Allows you to solve for the Try this! Solve: 6 = x − 4.


variable on either side of the
equal sign, because 5 = x is
equivalent to x = 5 . Answer: x = 10

2.3 Solving Linear Equations: Part I 232


Chapter 2 Linear Equations and Inequalities

Video Solution

(click to see video)


Isolating the Variable in Two Steps

A linear equation of the form ax + b = c takes two steps to solve. First, use the
appropriate equality property of addition or subtraction to isolate the variable
term. Next, isolate the variable using the equality property of multiplication or
division. Checking solutions in the following examples is left to the reader.

Example 7: Solve: 2x − 5 = 15.

Solution:

Answer: The solution is 10.

Example 8: Solve: −3x − 2 = 9.

Solution:

2.3 Solving Linear Equations: Part I 233


Chapter 2 Linear Equations and Inequalities

Answer: The solution is − 113.

Example 9: Solve: 6 − 5y = −14.

Solution: When no sign precedes the term, it is understood to be positive. In other


words, think of this as +6 − 5y = −14. Begin by subtracting 6 from both sides of
the equal sign.

Answer: The solution is 4.

Example 10: Solve: 3x + 12 = 23.

Solution:

2.3 Solving Linear Equations: Part I 234


Chapter 2 Linear Equations and Inequalities

1
Answer: The solution is 18
.

Example 11: Solve: 3 − y = 1.

Solution:

Recall that −y is equivalent to −1y ; divide both sides of the equation by −1.

2.3 Solving Linear Equations: Part I 235


Chapter 2 Linear Equations and Inequalities

Alternatively, multiply both sides of −y = −2 by −1 and achieve the same result:

Answer: The solution is 2.

In summary, to retain equivalent equations, we must perform the same operation


on both sides of the equation. First, apply the addition or subtraction property of
equality to isolate the variable term and then apply the multiplication or division
property of equality to isolate the variable on one side of the equation.

Try this! Solve: −7x + 6 = 27.

Answer: x = −3

Video Solution

(click to see video)


Multiplying by the Reciprocal

To solve an equation like 34 x = 1, we can isolate the variable by dividing both sides
by the coefficient. For example,

2.3 Solving Linear Equations: Part I 236


Chapter 2 Linear Equations and Inequalities

On the left side of the equal sign, the fraction cancels. On the right side, we have a
complex fraction and multiply by the reciprocal of the coefficient. You can save a
step by recognizing this and start by multiplying both sides of the equation by the
reciprocal of the coefficient.

Recall that the product of reciprocals is 1, in this case 4


3
⋅ 3
4
= 1, leaving the
variable isolated.

Example 12: Solve: 53 x + 2 = −8.

Solution: Isolate the variable term using the addition property of equality and then
multiply both sides of the equation by the reciprocal of the coefficient 53 .

2.3 Solving Linear Equations: Part I 237


Chapter 2 Linear Equations and Inequalities

Answer: The solution is −6.

Example 13: Solve: − 45 x − 5 = 15.

Solution:

The reciprocal of − 45 is − 54 because (− 54 ) (− 45 ) = + 20


20
= .1Therefore, to
isolate the variable x, multiply both sides by − 54.

2.3 Solving Linear Equations: Part I 238


Chapter 2 Linear Equations and Inequalities

Answer: The solution is −25.

Try this! Solve: 23 x − 9 = −4.

Answer: x = 15
2

Video Solution

(click to see video)

KEY TAKEAWAYS

• Linear equations with one variable can be written in the form


ax + b = 0, where a and b are real numbers and a ≠ 0.
• To “solve a linear equation” means to find a numerical value that can
replace the variable and produce a true statement.
• The properties of equality provide tools for isolating the variable and
solving equations.
• To solve a linear equation, first isolate the variable term by adding the
opposite of the constant term to both sides of the equation. Then isolate
the variable by dividing both sides of the equation by its coefficient.
• After isolating a variable term with a fraction coefficient, solve by
multiplying both sides by the reciprocal of the coefficient.

2.3 Solving Linear Equations: Part I 239


Chapter 2 Linear Equations and Inequalities

TOPIC EXERCISES

Part A: Solutions to Linear Equations

Is the given value a solution to the linear equation?

1. x − 6 = 20; x = 26

2. y + 7 = −6; y = −13

3. −x + 5 = 17; x = 12

4. −2y = 44; y = 11

5. 4x = −24; x = −6

6. 5x − 1 = 34; x = −7

7. −2a − 7 = −7; a = 0
1
8. − x − 4 = −5; x = −3
3

1 2 1 11
9. − x+ =− ; x=
2 3 4 6

10. −8x − 33 = 3x; x = 3

11. 3y − 5 = −2y − 15; y = −2


1
12. 3 (2x + 1) = −4x − 3; x = − 2

1 1 1 1
13.
2
y− 3
= 3
y+ 6
;y =3

4 1 2 1 1
14. − y+ =− y− ;y =
3 9 3 9 3

Part B: Solving Basic Linear Equations

Solve.

2.3 Solving Linear Equations: Part I 240


Chapter 2 Linear Equations and Inequalities

15. x + 3 = 13

16. y − 4 = 22

17. −6 + x = 12

18. 9 + y = −4
1 1
19. x − =
2 3

2 1
20. x + =−
3 5

1 1
21. x +2 =3
2 3

22. −37 + x = −37

23. 4x = −44

24. −9x = 63

25. −y = 13

26. −x = −10

27. −9x =0

28. −3a = −33

29. 27 = 18y

30. 14 = −7x

31. 5.6a = −39.2

32. −1.2y = 3.72


1 1
33.
3
x=− 2

2.3 Solving Linear Equations: Part I 241


Chapter 2 Linear Equations and Inequalities

t 1
34. − =
12 4

7 1
35. − x=
3 2

x
36.
5
= −3

4 2
37.
9
y=− 3

5 5
38. − y=−
8 2

Part C: Solving Linear Equations

Solve.

39. 5x + 7 = 32

40. 4x − 3 = 21

41. 3a − 7 = 23

42. 12y +1=1

43. 21x −7=0

44. −3y + 2 = −13

45. −5x +9=8

46. 22x − 55 = −22

47. 4.5x − 2.3 = 6.7

48. 1.4 − 3.2x = 3

49. 9.6 − 1.4y = −10.28

50. 4.2y − 3.71 = 8.89

2.3 Solving Linear Equations: Part I 242


Chapter 2 Linear Equations and Inequalities

51. 3 − 2y = −11

52. −4 − 7a = 24

53. −10 = 2x − 5

54. 24 = 6 − 12y
5 1 2
55.
6
x− 2
= 3

1 1 2
56.
2
x+ 3
= 5

2 1
57. 4a − =−
3 6

3 1 1
58.
5
x− 2
= 10

4 1 1
59. − y+ =
5 3 15

9 4 4
60. − x+ =
16 3 3

61. −x + 5 = 14

62. −y − 7 = −12

63. 75 − a = 200

64. 15 =5−x

65. −8 = 4 − 2x

66. 33 − x = 33

67. 18 =6−y

68. −12 = −2x + 3

69. −3 = 3.36 − 1.2a

2.3 Solving Linear Equations: Part I 243


Chapter 2 Linear Equations and Inequalities

70. 0 = −3.1a + 32.55


1 3
71.
4
=− 8
+ 10x

1
72. 70 = 50 − y
2

Translate the following sentences into linear equations and then solve.

73. The sum of 2x and 5 is equal to 15.

74. The sum of −3x and 7 is equal to 14.

75. The difference of 5x and 6 is equal to 4.

76. Twelve times x is equal to 36.

77. A number n divided by 8 is 5.

78. Six subtracted from two times a number x is 12.

79. Four added to three times a number n is 25.

80. Three-fourths of a number x is 9.

81. Negative two-thirds times a number x is equal to 20.

82. One-half of a number x plus 3 is equal to 10.

Find a linear equation of the form ax + b = 0 with the given solution, where a
and b are integers. (Answers may vary.)

83. x =2

84. x = −3
1
85. x =− 2

2
86. x = 3

2.3 Solving Linear Equations: Part I 244


Chapter 2 Linear Equations and Inequalities

Part D: Discussion Board Topics

87. How many steps are needed to solve any equation of the form
ax + b = c? Explain.

88. Instead of dividing by 6 when 6x = 12 , could you multiply by the


reciprocal of 6? Does this always work?

2.3 Solving Linear Equations: Part I 245


Chapter 2 Linear Equations and Inequalities

ANSWERS

1: Yes

3: No

5: Yes

7: Yes

9: Yes

11: Yes

13: Yes

15: 10

17: 18

19: 5/6

21: 5/6

23: −11

25: −13

27: 0

29: 3/2

31: −7

33: −3/2

35: −3/14

37: −3/2

2.3 Solving Linear Equations: Part I 246


Chapter 2 Linear Equations and Inequalities

39: 5

41: 10

43: 1/3

45: 1/5

47: 2

49: 14.2

51: 7

53: −5/2

55: 7/5

57: 1/8

59: 1/3

61: −9

63: −125

65: 6

67: −12

69: 5.3

71: 1/16

73: 2x + 5 = 15 ; x = 5

75: 5x − 6 = 4; x = 2
n
77:
8
= 5; n = 40

2.3 Solving Linear Equations: Part I 247


Chapter 2 Linear Equations and Inequalities

79: 3n + 4 = 25 ; n = 7
2
81: − x = 20 ; x = −30
3

83: x −2=0

85: 2x +1=0

2.3 Solving Linear Equations: Part I 248


Chapter 2 Linear Equations and Inequalities

2.4 Solving Linear Equations: Part II

LEARNING OBJECTIVES

1. Solve general linear equations.


2. Identify and solve conditional equations, identities, and contradictions.
3. Clear decimals and fractions from equations.
4. Solve literal equations or formulas for a given variable.

Combining Like Terms and Simplifying

Linear equations typically are not given in standard form, so solving them requires
additional steps. These additional steps include simplifying expressions on each
side of the equal sign using the order of operations.

Same-Side Like Terms

We will often encounter linear equations where the expressions on each side of the
equal sign can be simplified. Typically, this involves combining same-side like
terms26. If this is the case, then it is best to simplify each side first before solving.

Example 1: Solve: −4a + 2 − a = 3 − 2.

Solution: First, combine the like terms on each side of the equal sign.

26. Like terms of an equation on


the same side of the equal sign.

249
Chapter 2 Linear Equations and Inequalities

Answer: The solution is 15.

Opposite-Side Like Terms

Given a linear equation in the form ax + b = cx + d, we begin by combining like


terms on opposite sides of the equal sign. To combine opposite-side like terms27,
use the addition or subtraction property of equality to effectively “move terms”
from one side to the other so that they can be combined.

Example 2: Solve: −2y − 3 = 5y + 11 .

Solution: To “move” the term 5y to the left side, subtract it on both sides.

From here, solve using the techniques developed previously.

Always check to see that the solution is correct by substituting the solution back
into the original equation and simplifying to see if you obtain a true statement.

27. Like terms of an equation on


opposite sides of the equal
sign.

2.4 Solving Linear Equations: Part II 250


Chapter 2 Linear Equations and Inequalities

Answer: The solution is −2.

General Guidelines for Solving Linear Equations

When solving linear equations, the goal is to determine what value, if any, will
produce a true statement when substituted in the original equation. Do this by
isolating the variable using the following steps:

Step 1: Simplify both sides of the equation using the order of operations and
combine all same-side like terms.

Step 2: Use the appropriate properties of equality to combine opposite-side like


terms with the variable term on one side of the equation and the constant term on
the other.

Step 3: Divide or multiply as needed to isolate the variable.

Step 4: Check to see if the answer solves the original equation.

Example 3: Solve: − 12 (10y − 2) + 3 = 14.

Solution: Simplify the linear expression on the left side before solving.

To check,

2.4 Solving Linear Equations: Part II 251


Chapter 2 Linear Equations and Inequalities

Answer: The solution is −2.

Example 4: Solve: 5 (3x + 2) − 2 = −2 (1 − 7x).

Solution: First, simplify the expressions on both sides of the equal sign.

Answer: The solution is −10. The check is left as an exercise.

Try this! Solve: 6 − 3 (4x − 1) = 4x − 7.

Answer: x = 1

2.4 Solving Linear Equations: Part II 252


Chapter 2 Linear Equations and Inequalities

Video Solution

(click to see video)


Conditional Equations, Identities, and Contradictions

There are three different types of equations. Up to this point, we have been solving
conditional equations28. These are equations that are true for particular values. An
identity29 is an equation that is true for all possible values of the variable. For
example,

has a solution set consisting of all real numbers, R. A contradiction30 is an equation


that is never true and thus has no solutions. For example,

has no solution. We use the empty set, ∅, to indicate that there are no solutions.

If the end result of solving an equation is a true statement, like 0 = 0, then the
equation is an identity and any real number is a solution. If solving results in a false
statement, like 0 = 1, then the equation is a contradiction and there is no solution.

Example 5: Solve: 4 (x + 5) + 6 = 2 (2x + 3).

Solution:

28. Equations that are true for


particular values.

29. An equation that is true for all


possible values.

30. An equation that is never true


and has no solution.

2.4 Solving Linear Equations: Part II 253


Chapter 2 Linear Equations and Inequalities

Answer: ∅. Solving leads to a false statement; therefore, the equation is a


contradiction and there is no solution.

Example 6: Solve: 3 (3y + 5) + 5 = 10 (y + 2) − y.

Solution:

Answer: R. Solving leads to a true statement; therefore, the equation is an identity


and any real number is a solution.

If it is hard to believe that any real number is a solution to the equation in the
previous example, then choose your favorite real number, and substitute it in the
equation to see that it leads to a true statement. Choose x = 7 and check:

Try this! Solve: −2 (3x + 1) − (x − 3) = −7x + 1.

2.4 Solving Linear Equations: Part II 254


Chapter 2 Linear Equations and Inequalities

Answer: R

Video Solution

(click to see video)


Clearing Decimals and Fractions

The coefficients of linear equations may be any real number, even decimals and
fractions. When decimals and fractions are used, it is possible to use the
multiplication property of equality to clear the coefficients in a single step. If given
decimal coefficients, then multiply by an appropriate power of 10 to clear the
decimals. If given fractional coefficients, then multiply both sides of the equation by
the least common multiple of the denominators (LCD).

Example 7: Solve: 2.3x + 2.8 = −1.2x + 9.8.

Solution: Notice that all decimal coefficients are expressed with digits in the tenths
place; this suggests that we can clear the decimals by multiplying both sides by 10.
Take care to distribute 10 to each term on both sides of the equation.

Answer: The solution is 2.

2.4 Solving Linear Equations: Part II 255


Chapter 2 Linear Equations and Inequalities

Example 8: Solve: 13 x + 15 = 15 x − 1
.

multiple of the given denominators. In this case, the LCM (3, 5) = 15.
Solution: Clear the fractions by multiplying both sides by the least common

Answer: The solution is −9.

It is important to know that these techniques only work for equations. Do not try to
clear fractions when simplifying expressions. As a reminder,

Solve equations and simplify expressions. If you multiply an expression by 6, you


will change the problem. However, if you multiply both sides of an equation by 6,
you obtain an equivalent equation.

2.4 Solving Linear Equations: Part II 256


Chapter 2 Linear Equations and Inequalities

Literal Equations (Linear Formulas)

Algebra lets us solve whole classes of applications using literal equations31, or


formulas. Formulas often have more than one variable and describe, or model, a
particular real-world problem. For example, the familiar formula D = rt describes
the distance traveled in terms of the average rate and time; given any two of these
quantities, we can determine the third. Using algebra, we can solve the equation for
any one of the variables and derive two more formulas.

If we divide both sides by r, we obtain the formula t = Dr. Use this formula to find
the time, given the distance and the rate.

If we divide both sides by t, we obtain the formula r = Dt. Use this formula to find
the rate, given the distance traveled and the time it takes to travel that distance.
Using the techniques learned up to this point, we now have three equivalent
31. A formula that summarizes formulas relating distance, average rate, and time:
whole classes of problems.

2.4 Solving Linear Equations: Part II 257


Chapter 2 Linear Equations and Inequalities

When given a literal equation, it is often necessary to solve for one of the variables
in terms of the others. Use the properties of equality to isolate the indicated
variable.

Example 9: Solve for a: P = 2a + b.

Solution: The goal is to isolate the variable a.

Answer: a = P−b
2

x+y
Example 10: Solve for y: z = 2
.

Solution: The goal is to isolate the variable y.

2.4 Solving Linear Equations: Part II 258


Chapter 2 Linear Equations and Inequalities

Answer: y = 2z − x

Try this! Solve for b: 2a − 3b = c.

Answer: b = 2a−c
3

Video Solution

(click to see video)

2.4 Solving Linear Equations: Part II 259


Chapter 2 Linear Equations and Inequalities

KEY TAKEAWAYS

• Solving general linear equations involves isolating the variable, with


coefficient 1, on one side of the equal sign.

• The steps for solving linear equations are:

1. Simplify both sides of the equation and combine all same-


side like terms.
2. Combine opposite-side like terms to obtain the variable term
on one side of the equal sign and the constant term on the
other.
3. Divide or multiply as needed to isolate the variable.
4. Check the answer.
• Most linear equations that you will encounter are conditional and have
one solution.
• If solving a linear equation leads to a true statement like 0 = 0, then the
equation is an identity and the solution set consists of all real numbers,
R.
• If solving a linear equation leads to a false statement like 0 = 5, then the
equation is a contradiction and there is no solution, ∅.
• Clear fractions by multiplying both sides of a linear equation by the least
common multiple of all the denominators. Distribute and multiply all
terms by the LCD to obtain an equivalent equation with integer
coefficients.
• Given a formula, solve for any variable using the same techniques for
solving linear equations. This works because variables are simply
representations of real numbers.

2.4 Solving Linear Equations: Part II 260


Chapter 2 Linear Equations and Inequalities

TOPIC EXERCISES

Part A: Checking for Solutions

Is the given value a solution to the linear equation?

1. 2 (3x + 5) − 6 = 3x − 8; x = −4

2. −x + 17 − 8x = 9 − x; x = −1
1
3. 4 (3x − 7) − 3 (x + 2) = −1; x = 3

4. −5 − 2 (x − 5) = − (x + 3) ; x = −8

5. 7 − 2 ( 12 x − 6) = x − 1; x = 10

2 4
6. 3x − (9x − 2) = 0; x =
3 9

Part B: Solving Linear Equations

Solve.

7. 4x − 7 = 7x + 5

8. −5x + 3 = −8x − 9

9. 3x − 5 = 2x − 17

10. −2y − 52 = 3y + 13

11. −4x + 2 = 7x − 20

12. 4x − 3 = 6x − 15

13. 9x − 25 = 12x − 25

14. 12y + 15 = −6y + 23

2.4 Solving Linear Equations: Part II 261


Chapter 2 Linear Equations and Inequalities

15. 1.2x − 0.7 = 3x + 4.7

16. 2.1x + 6.1 = −1.3x + 4.4

17. 2.02x + 4.8 = 14.782 − 1.2x

18. −3.6x + 5.5 + 8.2x = 6.5 + 4.6x


1 2 1
19.
2
x− 3
=x+ 5

1 1 1 1
20.
3
x− 2
=− 4
x− 3

1 2 1 3
21. − y+ = y+
10 5 5 10

20 5 5
22. x − = x+
3 2 6

2 1 5 37
23.
3
y+ 2
= 8
y+ 24

1 4 10 1 2
24.
3
+ 3
x= 7
x+ 3
− 21
x

8 11 7 1
25.
9
− 18
x= 6
− 2
x

1 4 1
26.
3
− 9x = 9
+ 2
x

27. 12x − 5 + 9x = 44

28. 10 − 6x − 13 = 12

29. −2 + 4x + 9 = 7x + 8 − 2x

30. 20x − 5 + 12x = 6 − x + 7

31. 3a + 5 − a = 2a + 7

32. −7b + 3 = 2 − 5b + 1 − 2b

33. 7x − 2 + 3x = 4 + 2x − 2

2.4 Solving Linear Equations: Part II 262


Chapter 2 Linear Equations and Inequalities

34. −3x + 8 − 4x + 2 = 10

35. 6x + 2 − 3x = −2x − 13

36. 3x − 0.75 + 0.21x = 1.24x + 7.13

37. −x − 2 + 4x = 5 + 3x − 7

38. −2y − 5 = 8y − 6 − 10y


1 1 1 1 7
39.
10
x− 3
= 30
− 15
x− 15

5 4 1 3 1 1
40.
8
− 3
x+ 3
=− 9
x− 4
+ 3
x

Part C: Solving Linear Equations Involving Parentheses

Solve.

41. −5 (2y − 3) + 2 = 12

42. 3 (5x + 4) + 5x = −8

43. 4 − 2 (x − 5) = −2

44. 10 − 5 (3x + 1) = 5 (x − 4)

45. 9 − (x + 7) = 2 (x − 1)

46. −5 (2x − 1) + 3 = −12

47. 3x − 2 (x + 1) = x + 5

48. 5x − 3 (2x − 1) = 2 (x − 3)

49. −6 (x − 1) − 3x = 3 (x + 8)

(5x + 10) =
3 1
50. − (4x − 12)
5 2

2.4 Solving Linear Equations: Part II 263


Chapter 2 Linear Equations and Inequalities

51. 3.1 (2x − 3) + 0.5 = 22.2

52. 4.22 − 3.13 (x − 1) = 5.2 (2x + 1) − 11.38

53. 6 (x − 2) − (7x − 12) = 14

54. −9 (x − 3) − 3x = −3 (4x + 9)

55. 3 − 2 (x + 4) = −3 (4x − 5)

56. 12 − 2 (2x + 1) = 4 (x − 1)

57. 3 (x + 5) − 2 (2x + 3) = 7x + 9

58. 3 (2x − 1) − 4 (3x − 2) = −5x + 10

59. −3 (2a − 3) + 2 = 3 (a + 7)

60. −2 (5x − 3) − 1 = 5 (−2x + 1)

1 1
61.
2
(2x + 1) − 4
(8x + 2) = 3 (x − 4)

(6x − 3) −
2 1 3
62. − = (4x + 1)
3 2 2

(2x − 5) = 0
1 1
63.
2
(3x − 1) + 3

1 1 1
64.
3
(x − 2) + 5
= 9
(3x + 3)

65. −2 (2x − 7) − (x + 3) = 6 (x − 1)

66. 10 (3x + 5) − 5 (4x + 2) = 2 (5x + 20)

67. 2 (x − 3) − 6 (2x + 1) = −5 (2x − 4)

68. 5 (x − 2) − (4x − 1) = −2 (3 − x)

69. 6 (3x − 2) − (12x − 1) + 4 = 0

2.4 Solving Linear Equations: Part II 264


Chapter 2 Linear Equations and Inequalities

70. −3 (4x − 2) − (9x + 3) − 6x = 0

Part D: Literal Equations

Solve for the indicated variable.

71. Solve for w: A = l ⋅ w.

72. Solve for a: F = ma.

73. Solve for w: P = 2l + 2w .

74. Solve for r: C = 2πr.

75. Solve for b: P = a + b + c.


9
76. Solve for C: F= 5
C + 32.

1
77. Solve for h: A= 2
bh .

78. Solve for t: I = Prt.

79. Solve for y: ax + by = c .

80. Solve for h: S = 2πr2 + 2πrh.


2x+y
81. Solve for x: z= 5
.

3b−2c
82. Solve for c: a= 3
.

83. Solve for b: y = mx + b .

84. Solve for m: y = mx + b .

85. Solve for y: 3x − 2y = 6 .

86. Solve for y: −5x + 2y = 12 .

2.4 Solving Linear Equations: Part II 265


Chapter 2 Linear Equations and Inequalities

x y
87. Solve for y:
3
− 5
= 1.

3 1 1
88. Solve for y:
4
x− 5
y= 2
.

Translate the following sentences into linear equations and then solve.

89. The sum of 3x and 5 is equal to the sum of 2x and 7.

90. The sum of −5x and 6 is equal to the difference of 4x and 2.

91. The difference of 5x and 25 is equal to the difference of 3x and 51.

1 3 2
92. The sum of
2
x and 4
is equal to
3
x.

93. A number n divided by 5 is equal to the sum of twice the number and 3.

94. Negative ten times a number n is equal to the sum of three times the
number and 13.

Part E: Discussion Board Topics

95. What is the origin of the word algebra?

96. What is regarded as the main business of algebra?

97. Why is solving equations such an important algebra topic?

98. Post some real-world linear formulas not presented in this section.

99. Research and discuss the contributions of Diophantus of Alexandria.

100. Create an identity or contradiction of your own and share on the


discussion board. Provide a solution and explain how you found it.

2.4 Solving Linear Equations: Part II 266


Chapter 2 Linear Equations and Inequalities

ANSWERS

1: Yes

3: No

5: Yes

7: −4

9: −12

11: 2

13: 0

15: −3

17: 3.1

19: −26/15

21: 1/3

23: 25

25: −5/2

27: 7/3

29: −1

31: ∅

33: 1/2

35: −3

37: R

2.4 Solving Linear Equations: Part II 267


Chapter 2 Linear Equations and Inequalities

39: −3/5

41: 1/2

43: 8

45: 4/3

47: ∅

49: −3/2

51: 5

53: −14

55: 2

57: 0

59: −10/9

61: 3

63: 1

65: 17/11

67: ∅

69: 7/6

A
71: w = l

P−2l
73: w = 2

75: b =P−a−c
2A
77: h = b

2.4 Solving Linear Equations: Part II 268


Chapter 2 Linear Equations and Inequalities

−ax+c
79: y = b

5z−y
81: x = 2

83: b = y − mx
3x−6
85: y = 2

5x−15
87: y = 3

89: 3x + 5 = 2x + 7 ; x = 2

91: 5x − 25 = 3x − 51 ; x = −13
n 5
93:
5
= 2n + 3; n = − 3

2.4 Solving Linear Equations: Part II 269


Chapter 2 Linear Equations and Inequalities

2.5 Applications of Linear Equations

LEARNING OBJECTIVES

1. Identify key words and phrases, translate sentences to mathematical


equations, and develop strategies to solve problems.
2. Solve word problems involving relationships between numbers.
3. Solve geometry problems involving perimeter.
4. Solve percent and money problems including simple interest.
5. Set up and solve uniform motion problems.

Key Words, Translation, and Strategy

Algebra simplifies the process of solving real-world problems. This is done by using
letters to represent unknowns, restating problems in the form of equations, and
offering systematic techniques for solving those equations. To solve problems using
algebra, first translate the wording of the problem into mathematical statements
that describe the relationships between the given information and the unknowns.
Usually, this translation to mathematical statements is the difficult step in the
process. The key to the translation is to carefully read the problem and identify
certain key words and phrases.

Key Words Translation

Sum, increased by, more than, plus, added to, total +

Difference, decreased by, subtracted from, less, minus −

Product, multiplied by, of, times, twice *

Quotient, divided by, ratio, per /

Is, total, result =

Here are some examples of translated key phrases.

Key Phrases Translation

The sum of a number and 7.


x+7
Seven more than a number.

270
Chapter 2 Linear Equations and Inequalities

Key Phrases Translation

The difference of a number and 7.

Seven less than a number. x−7


Seven subtracted from a number.

The product of 2 and a number.


2x
Twice a number.

One-half of a number.
1
2
x

The quotient of a number and 7. x/7

When translating sentences into mathematical statements, be sure to read the


sentence several times and identify the key words and phrases.

Example 1: Translate: Four less than twice some number is 16.

Solution: First, choose a variable for the unknown number and identify the key
words and phrases.

Let x represent the unknown indicated by “some number.”

Remember that subtraction is not commutative. For this reason, take care when
setting up differences. In this example, 4 − 2x = 16 is an incorrect translation.

Answer: 2x − 4 = 16

2.5 Applications of Linear Equations 271


Chapter 2 Linear Equations and Inequalities

It is important to first identify the variable—let x represent…—and state in words


what the unknown quantity is. This step not only makes your work more readable
but also forces you to think about what you are looking for. Usually, if you know
what you are asked to find, then the task of finding it is achievable.

Example 2: Translate: When 7 is subtracted from 3 times the sum of a number and
12, the result is 20.

Solution: Let n represent the unknown number.

Answer: 3 (n + 12) − 7 = 20

To understand why parentheses are needed, study the structures of the following
two sentences and their translations:

“3 times the sum of a number and 12” 3(n + 12)

“the sum of 3 times a number and 12” 3n + 12

The key is to focus on the phrase “3 times the sum.” This prompts us to group the
sum within parentheses and then multiply by 3. Once an application is translated
into an algebraic equation, solve it using the techniques you have learned.

Guidelines for Setting Up and Solving Word Problems

Step 1: Read the problem several times, identify the key words and phrases, and
organize the given information.

2.5 Applications of Linear Equations 272


Chapter 2 Linear Equations and Inequalities

Step 2: Identify the variables by assigning a letter or expression to the unknown


quantities.

Step 3: Translate and set up an algebraic equation that models the problem.

Step 4: Solve the resulting algebraic equation.

Step 5: Finally, answer the question in sentence form and make sure it makes sense
(check it).

For now, set up all of your equations using only one variable. Avoid two variables by
looking for a relationship between the unknowns.

Problems Involving Relationships between Real Numbers

We classify applications involving relationships between real numbers broadly as


number problems. These problems can sometimes be solved using some creative
arithmetic, guessing, and checking. Solving in this manner is not a good practice
and should be avoided. Begin by working through the basic steps outlined in the
general guidelines for solving word problems.

Example 3: A larger integer is 2 less than 3 times a smaller integer. The sum of the
two integers is 18. Find the integers.

Solution:

Identify variables: Begin by assigning a variable to the smaller integer.

Use the first sentence to identify the larger integer in terms of the variable x: “A
larger integer is 2 less than 3 times a smaller.”

2.5 Applications of Linear Equations 273


Chapter 2 Linear Equations and Inequalities

Set up an equation: Add the expressions that represent the two integers, and set the
resulting expression equal to 18 as indicated in the second sentence: “The sum of
the two integers is 18.”

Solve: Solve the equation to obtain the smaller integer x.

Back substitute: Use the expression 3x − 2 to find the larger integer—this is called
back substituting32.

Answer the question: The two integers are 5 and 13.

Check: 5 + 13 = 18. The answer makes sense.

32. The process of finding the


answers to other unknowns
after one has been found.

2.5 Applications of Linear Equations 274


Chapter 2 Linear Equations and Inequalities

Example 4: The difference between two integers is 2. The larger integer is 6 less
than twice the smaller. Find the integers.

Solution: Use the relationship between the two integers in the second sentence,
“The larger integer is 6 less than twice the smaller,” to identify the unknowns in
terms of one variable.

Since the difference is positive, subtract the smaller integer from the larger.

Solve.

Use 2x − 6 to find the larger integer.

Answer: The two integers are 8 and 10. These integers clearly solve the problem.

It is worth mentioning again that you can often find solutions to simple problems
by guessing and checking. This is so because the numbers are chosen to simplify the
process of solving, so that the algebraic steps are not too tedious. You learn how to

2.5 Applications of Linear Equations 275


Chapter 2 Linear Equations and Inequalities

set up algebraic equations with easier problems, so that you can use these ideas to
solve more difficult problems later.

Example 5: The sum of two consecutive even integers is 46. Find the integers.

Solution: The key phrase to focus on is “consecutive even integers.”

Add the even integers and set them equal to 46.

Solve.

Use x + 2 to find the next even integer.

Answer: The consecutive even integers are 22 and 24.

2.5 Applications of Linear Equations 276


Chapter 2 Linear Equations and Inequalities

It should be clear that consecutive even integers are separated by two units.
However, it may not be so clear that odd integers are as well.

Example 6: The sum of two consecutive odd integers is 36. Find the integers.

Solution: The key phrase to focus on is “consecutive odd integers.”

Add the two odd integers and set the expression equal to 36.

Solve.

Use x + 2 to find the next odd integer.

2.5 Applications of Linear Equations 277


Chapter 2 Linear Equations and Inequalities

Answer: The consecutive odd integers are 17 and 19.

The algebraic setup for even and odd integer problems is the same. A common
mistake is to use x and x + 3 when identifying the variables for consecutive odd
integers. This is incorrect because adding 3 to an odd number yields an even
number: for example, 5 + 3 = 8. An incorrect setup is very likely to lead to a decimal
answer, which may be an indication that the problem was set up incorrectly.

Example 7: The sum of three consecutive integers is 24. Find the integers.

Solution: Consecutive integers are separated by one unit.

Add the integers and set the sum equal to 24.

Solve.

2.5 Applications of Linear Equations 278


Chapter 2 Linear Equations and Inequalities

Back substitute to find the other two integers.

Answer: The three consecutive integers are 7, 8 and 9, where 7 + 8 + 9 = 24.

Try this! The sum of three consecutive odd integers is 87. Find the integers.

Answer: The integers are 27, 29, and 31.

Video Solution

(click to see video)


Geometry Problems (Perimeter)

Recall that the perimeter33 of a polygon is the sum of the lengths of all the outside
edges. In addition, it is helpful to review the following perimeter formulas
(π ≈ 3.14159).

33. The sum of the lengths of all Perimeter of a rectangle34: P = 2l + 2w


the outside edges of a polygon.

34. P = 2l + 2w, where l


represents the length and w
represents the width. Perimeter of a square35: P = 4s
35. P = 4s, where s represents
the length of a side.

2.5 Applications of Linear Equations 279


Chapter 2 Linear Equations and Inequalities

Perimeter of a triangle36: P=a+b+c

Perimeter of a circle (circumference37): C = 2πr

Keep in mind that you are looking for a relationship between the unknowns so that
you can set up algebraic equations using only one variable. When working with
geometry problems, it is often helpful to draw a picture.

Example 8: A rectangle has a perimeter measuring 64 feet. The length is 4 feet more
than 3 times the width. Find the dimensions of the rectangle.

Solution: The sentence “The length is 4 feet more than 3 times the width” gives
the relationship between the two variables.

The sentence “A rectangle has a perimeter measuring 64 feet” suggests an


algebraic setup. Substitute 64 for the perimeter and the expression for the length
into the appropriate formula as follows:
36. P = a + b + c, where a, b,
and c each represents the
length of a different side.

37. The perimeter of a circle given


by C = 2πr, where r
represents the radius of the
circle and π ≈ 3.14159 .

2.5 Applications of Linear Equations 280


Chapter 2 Linear Equations and Inequalities

Once you have set up an algebraic equation with one variable, solve for the width,
w.

Use 3w + 4 to find the length.

Answer: The rectangle measures 7 feet by 25 feet. To check, add all of the sides:

Example 9: Two sides of a triangle are 5 and 7 inches longer than the third side. If
the perimeter measures 21 inches, find the length of each side.

Solution: The first sentence describes the relationships between the unknowns.

2.5 Applications of Linear Equations 281


Chapter 2 Linear Equations and Inequalities

Substitute these expressions into the appropriate formula and use 21 for the
perimeter P.

You now have an equation with one variable to solve.

Back substitute.

Answer: The three sides of the triangle measure 3 inches, 8 inches, and 10 inches.
The check is left to the reader.

Try this! The length of a rectangle is 1 foot less than twice its width. If the
perimeter is 46 feet, find the dimensions.

2.5 Applications of Linear Equations 282


Chapter 2 Linear Equations and Inequalities

Answer: Width: 8 feet; length: 15 feet

Video Solution

(click to see video)


Problems Involving Money and Percents

Whenever setting up an equation involving a percentage, we usually need to


convert the percentage to a decimal or fraction. If the question asks for a
percentage, then do not forget to convert your answer to a percent at the end. Also,
when money is involved, be sure to round off to two decimal places.

Example 10: If a pair of shoes costs $52.50 including a 7 1


4
%tax, what is the
original cost of the item before taxes are added?

Solution: Begin by converting 7 1


4
%to a decimal.

The amount of tax is this rate times the original cost of the item. The original cost
of the item is what you are asked to find.

Use this equation to solve for c, the original cost of the item.

2.5 Applications of Linear Equations 283


Chapter 2 Linear Equations and Inequalities

Answer: The cost of the item before taxes is $48.95. Check this by multiplying $48.95
by 0.0725 to obtain the tax and add it to this cost.

Example 11: Given a 5 18 %annual interest rate, how long will it take $1,200 to yield
$307.50 in simple interest?

Solution:

Organize the data needed to use the simple interest formula I = prt.

Given interest for the time period: I = $307.50

Given principal: p = $1200

Given rate: r=5 1


8
% = 5.125% = 0.05125

Next, substitute all of the known quantities into the formula and then solve for the
only unknown, t.

2.5 Applications of Linear Equations 284


Chapter 2 Linear Equations and Inequalities

Answer: It takes 5 years for $1,200 invested at 5 1


8
%to earn $307.50 in simple
interest.

Example 12: Mary invested her total savings of $3,400 in two accounts. Her mutual
fund account earned 8% last year and her CD earned 5%. If her total interest for the
year was $245, how much was in each account?

Solution: The relationship between the two unknowns is that they total $3,400.
When a total is involved, a common technique used to avoid two variables is to
represent the second unknown as the difference of the total and the first unknown.

The total interest is the sum of the interest earned from each account.

Interest
earned in
I = Prt = x ⋅ 0.08 ⋅ 1 = 0.08x
the mutual
fund:

Interest
earned in I = Prt = (3,400 − x) ⋅ 0.05 ⋅ 1 = 0.05(3,400 − x)
the CD:

2.5 Applications of Linear Equations 285


Chapter 2 Linear Equations and Inequalities

This equation models the problem with one variable. Solve for x.

Back substitute.

Answer: Mary invested $2,500 at 8% in a mutual fund and $900 at 5% in a CD.

Example 13: Joe has a handful of dimes and quarters that values $5.30. He has one
fewer than twice as many dimes than quarters. How many of each coin does he
have?

Solution: Begin by identifying the variables.

2.5 Applications of Linear Equations 286


Chapter 2 Linear Equations and Inequalities

To determine the total value of a number of coins, multiply the number of coins by
the value of each coin. For example, 5 quarters have a value $0.25 ⋅ 5 = $1.25.

Solve for the number of quarters, q.

Back substitute into 2q − 1 to find the number of dimes.

Answer: Joe has 12 quarters and 23 dimes. Check by multiplying $0.25 ⋅ 12 = $3.00
and $0.10 ⋅ 23 = $2.30. Then add to obtain the correct amount: $3.00 + $2.30 = $5.30.

Try this! A total amount of $5,900 is invested in two accounts. One account earns
3.5% interest and another earns 4.5%. If the interest for 1 year is $229.50, then how
much is invested in each account?

Answer: $3,600 is invested at 3.5% and $2,300 at 4.5%.

2.5 Applications of Linear Equations 287


Chapter 2 Linear Equations and Inequalities

Video Solution

(click to see video)


Uniform Motion Problems (Distance Problems)

Uniform motion refers to movement at a speed, or rate, that does not change. We
can determine the distance traveled by multiplying the average rate by the time
traveled at that rate with the formula D = r ⋅ t. Applications involving uniform
motion usually have a lot of data, so it helps to first organize the data in a chart and
then set up an algebraic equation that models the problem.

Example 14: Two trains leave the station at the same time traveling in opposite
directions. One travels at 70 miles per hour and the other at 60 miles per hour. How
long does it take for the distance between them to reach 390 miles?

Solution: First, identify the unknown quantity and organize the data.

The given information is filled in on the following chart. The time for each train is
equal.

2.5 Applications of Linear Equations 288


Chapter 2 Linear Equations and Inequalities

To avoid introducing two more variables, use the formula D = r ⋅ t to fill in the
unknown distances traveled by each train.

We can now completely fill in the chart.

The algebraic setup is defined by the distance column. The problem asks for the
time it takes for the total distance to reach 390 miles.

Solve for t.

2.5 Applications of Linear Equations 289


Chapter 2 Linear Equations and Inequalities

Answer: It takes 3 hours for the distance between the trains to reach 390 miles.

Example 15: A train traveling nonstop to its destination is able to make the trip at
an average speed of 72 miles per hour. On the return trip, the train makes several
stops and is only able to average 48 miles per hour. If the return trip takes 2 hours
longer than the initial trip to the destination, then what is the travel time each
way?

Solution: First, identify the unknown quantity and organize the data.

The given information is filled in the following chart:

2.5 Applications of Linear Equations 290


Chapter 2 Linear Equations and Inequalities

Use the formula D = r ⋅ t to fill in the unknown distances.

Use these expressions to complete the chart.

The algebraic setup is again defined by the distance column. In this case, the
distance to the destination and back is the same, and the equation is

Solve for t.

2.5 Applications of Linear Equations 291


Chapter 2 Linear Equations and Inequalities

The return trip takes t + 2 = 4 + 2 = 6 hours.

Answer: It takes 4 hours to arrive at the destination and 6 hours to return.

Try this! Mary departs for school on a bicycle at an average rate of 6 miles per
hour. Her sister Kate, running late, leaves 15 minutes later and cycles at twice that
speed. How long will it take Kate to catch up to Mary? Be careful! Pay attention to
the units given in the problem.

Answer: It takes 15 minutes for Kate to catch up.

Video Solution

(click to see video)

2.5 Applications of Linear Equations 292


Chapter 2 Linear Equations and Inequalities

KEY TAKEAWAYS

• Simplify the process of solving real-world problems by creating


mathematical models that describe the relationship between unknowns.
Use algebra to solve the resulting equations.
• Guessing and checking for solutions is a poor practice. This technique
might sometimes produce correct answers, but is unreliable, especially
when problems become more complex.
• Read the problem several times and search for the key words and
phrases. Identify the unknowns and assign variables or expressions to
the unknown quantities. Look for relationships that allow you to use
only one variable. Set up a mathematical model for the situation and use
algebra to solve the equation. Check to see if the solution makes sense
and present the solution in sentence form.
• Do not avoid word problems: solving them can be fun and rewarding.
With lots of practice you will find that they really are not so bad after
all. Modeling and solving applications is one of the major reasons to
study algebra.
• Do not feel discouraged when the first attempt to solve a word problem
does not work. This is part of the process. Try something different and
learn from incorrect attempts.

2.5 Applications of Linear Equations 293


Chapter 2 Linear Equations and Inequalities

TOPIC EXERCISES

Part A: Translate

Translate the following into algebraic equations.

1. The sum of a number and 6 is 37.

2. When 12 is subtracted from twice some number the result is 6.

3. Fourteen less than 5 times a number is 1.

4. Twice some number is subtracted from 30 and the result is 50.

5. Five times the sum of 6 and some number is 20.

6. The sum of 5 times some number and 6 is 20.

7. When the sum of a number and 3 is subtracted from 10 the result is 5.

8. The sum of three times a number and five times that same number is 24.

9. Ten is subtracted from twice some number and the result is the sum of the
number and 2.

10. Six less than some number is ten times the sum of that number and 5.

Part B: Number Problems

Set up an algebraic equation and then solve.

11. A larger integer is 1 more than twice another integer. If the sum of the
integers is 25, find the integers.

12. If a larger integer is 2 more than 4 times another integer and their
difference is 32, find the integers.

13. One integer is 30 more than another integer. If the difference between
the larger and twice the smaller is 8, find the integers.

2.5 Applications of Linear Equations 294


Chapter 2 Linear Equations and Inequalities

14. The quotient of some number and 4 is 22. Find the number.

15. Eight times a number is decreased by three times the same number,
giving a difference of 20. What is the number?

16. One integer is two units less than another. If their sum is −22, find the
two integers.

17. The sum of two consecutive integers is 139. Find the integers.

18. The sum of three consecutive integers is 63. Find the integers.

19. The sum of three consecutive integers is 279. Find the integers.

20. The difference of twice the smaller of two consecutive integers and the
larger is 39. Find the integers.

21. If the smaller of two consecutive integers is subtracted from two times
the larger, then the result is 17. Find the integers.

22. The sum of two consecutive even integers is 46. Find the integers.

23. The sum of two consecutive even integers is 238. Find the integers.

24. The sum of three consecutive even integers is 96. Find the integers.

25. If the smaller of two consecutive even integers is subtracted from 3 times
the larger the result is 42. Find the integers.

26. The sum of three consecutive even integers is 90. Find the integers.

27. The sum of two consecutive odd integers is 68. Find the integers.

28. The sum of two consecutive odd integers is 180. Find the integers.

29. The sum of three consecutive odd integers is 57. Find the integers.

30. If the smaller of two consecutive odd integers is subtracted from twice
the larger the result is 23. Find the integers.

31. Twice the sum of two consecutive odd integers is 32. Find the integers.

2.5 Applications of Linear Equations 295


Chapter 2 Linear Equations and Inequalities

32. The difference between twice the larger of two consecutive odd integers
and the smaller is 59. Find the integers.

Part C: Geometry Problems

Set up an algebraic equation and then solve.

33. If the perimeter of a square is 48 inches, then find the length of each
side.

34. The length of a rectangle is 2 inches longer than its width. If the
perimeter is 36 inches, find the length and width.

35. The length of a rectangle is 2 feet less than twice its width. If the
perimeter is 26 feet, find the length and width.

36. The width of a rectangle is 2 centimeters less than one-half its length. If
the perimeter is 56 centimeters, find the length and width.

37. The length of a rectangle is 3 feet less than twice its width. If the
perimeter is 54 feet, find the dimensions of the rectangle.

38. If the length of a rectangle is twice as long as the width and its perimeter
measures 72 inches, find the dimensions of the rectangle.

39. The perimeter of an equilateral triangle measures 63 centimeters. Find


the length of each side.

40. An isosceles triangle whose base is one-half as long as the other two
equal sides has a perimeter of 25 centimeters. Find the length of each side.

41. Each of the two equal legs of an isosceles triangle are twice the length of
the base. If the perimeter is 105 centimeters, then how long is each leg?

42. A triangle has sides whose measures are consecutive even integers. If the
perimeter is 42 inches, find the measure of each side.

43. A triangle has sides whose measures are consecutive odd integers. If the
perimeter is 21 inches, find the measure of each side.

2.5 Applications of Linear Equations 296


Chapter 2 Linear Equations and Inequalities

44. A triangle has sides whose measures are consecutive integers. If the
perimeter is 102 inches, then find the measure of each side.

45. The circumference of a circle measures 50π units. Find the radius.

46. The circumference of a circle measures 10π units. Find the radius.

47. The circumference of a circle measures 100 centimeters. Determine the


radius to the nearest tenth.

48. The circumference of a circle measures 20 centimeters. Find the


diameter rounded off to the nearest hundredth.

49. The diameter of a circle measures 5 inches. Determine the circumference


to the nearest tenth.

50. The diameter of a circle is 13 feet. Calculate the exact value of the
circumference.

Part D: Percent and Money Problems

Set up an algebraic equation and then solve.

51. Calculate the simple interest earned on a 2-year investment of $1,550 at a


8¾% annual interest rate.

52. Calculate the simple interest earned on a 1-year investment of $500 at a


6% annual interest rate.

53. For how many years must $10,000 be invested at an 8½% annual interest
rate to yield $4,250 in simple interest?

54. For how many years must $1,000 be invested at a 7.75% annual interest
rate to yield $503.75 in simple interest?

55. At what annual interest rate must $2,500 be invested for 3 years in order
to yield $412.50 in simple interest?

56. At what annual interest rate must $500 be invested for 2 years in order to
yield $93.50 in simple interest?

2.5 Applications of Linear Equations 297


Chapter 2 Linear Equations and Inequalities

57. If the simple interest earned for 1 year was $47.25 and the annual rate
was 6.3%, what was the principal?

58. If the simple interest earned for 2 years was $369.60 and the annual rate
was 5¼%, what was the principal?

59. Joe invested last year’s $2,500 tax return in two different accounts. He
put most of the money in a money market account earning 5% simple
interest. He invested the rest in a CD earning 8% simple interest. How much
did he put in each account if the total interest for the year was $138.50?

60. James invested $1,600 in two accounts. One account earns 4.25% simple
interest and the other earns 8.5%. If the interest after 1 year was $85, how
much did he invest in each account?

61. Jane has her $5,400 savings invested in two accounts. She has part of it in
a CD at 3% annual interest and the rest in a savings account that earns 2%
annual interest. If the simple interest earned from both accounts is $140 for
the year, then how much does she have in each account?

62. Marty put last year’s bonus of $2,400 into two accounts. He invested part
in a CD with 2.5% annual interest and the rest in a money market fund with
1.3% annual interest. His total interest for the year was $42.00. How much
did he invest in each account?

63. Alice puts money into two accounts, one with 2% annual interest and
another with 3% annual interest. She invests 3 times as much in the higher
yielding account as she does in the lower yielding account. If her total
interest for the year is $27.50, how much did she invest in each account?

64. Jim invested an inheritance in two separate banks. One bank offered 5.5%
annual interest rate and the other 6¼%. He invested twice as much in the
higher yielding bank account than he did in the other. If his total simple
interest for 1 year was $4,860, then what was the amount of his inheritance?

65. If an item is advertised to cost $29.99 plus 9.25% tax, what is the total
cost?

66. If an item is advertised to cost $32.98 plus 8¾% tax, what is the total cost?

67. An item, including an 8.75% tax, cost $46.49. What is the original pretax
cost of the item?

2.5 Applications of Linear Equations 298


Chapter 2 Linear Equations and Inequalities

68. An item, including a 5.48% tax, cost $17.82. What is the original pretax
cost of the item?

69. If a meal costs $32.75, what is the total after adding a 15% tip?

70. How much is a 15% tip on a restaurant bill that totals $33.33?

71. Ray has a handful of dimes and nickels valuing $3.05. He has 5 more
dimes than he does nickels. How many of each coin does he have?

72. Jill has 3 fewer half-dollars than she has quarters. The value of all 27 of
her coins adds to $9.75. How many of each coin does Jill have?

73. Cathy has to deposit $410 worth of five- and ten-dollar bills. She has 1
fewer than three times as many tens as she does five-dollar bills. How many
of each bill does she have to deposit?

74. Billy has a pile of quarters, dimes, and nickels that values $3.75. He has 3
more dimes than quarters and 5 more nickels than quarters. How many of
each coin does Billy have?

75. Mary has a jar with one-dollar bills, half-dollar coins, and quarters
valuing $14.00. She has twice as many quarters than she does half-dollar
coins and the same amount of half-dollar coins as one-dollar bills. How
many of each does she have?

76. Chad has a bill-fold of one-, five-, and ten-dollar bills totaling $118. He
has 2 more than 3 times as many ones as he does five-dollar bills and 1 fewer
ten- than five-dollar bills. How many of each bill does Chad have?

Part D: Uniform Motion (Distance Problems)

Set up an algebraic equation then solve.

77. Two cars leave a location traveling in opposite directions. If one car
averages 55 miles per hour and the other averages 65 miles per hour, then
how long will it take for them to separate a distance of 300 miles?

78. Two planes leave the airport at the same time traveling in opposite
directions. The average speeds for the planes are 450 miles per hour and 395

2.5 Applications of Linear Equations 299


Chapter 2 Linear Equations and Inequalities

miles per hour. How long will it take the planes to be a distance of 1,478.75
miles apart?

79. Bill and Ted are racing across the country. Bill leaves 1 hour earlier than
Ted and travels at an average rate of 60 miles per hour. If Ted intends to
catch up at a rate of 70 miles per hour, then how long will it take?

80. Two brothers leave from the same location, one in a car and the other on
a bicycle, to meet up at their grandmother’s house for dinner. If one brother
averages 30 miles per hour in the car and the other averages 12 miles per
hour on the bicycle, then it takes the brother on the bicycle 1 hour less than
3 times as long as the other in the car. How long does it take each of them to
make the trip?

81. A commercial airline pilot flew at an average speed of 350 miles per hour
before being informed that his destination airfield may be closed due to
poor weather conditions. In an attempt to arrive before the storm, he
increased his speed 400 miles per hour and flew for another 3 hours. If the
total distance flown was 2,950 miles, then how long did the trip take?

82. Two brothers drove the 2,793 miles from Los Angeles to New York. One of
the brothers, driving during the day, was able to average 70 miles per hour,
and the other, driving at night, was able to average 53 miles per hour. If the
brother driving at night drove 3 hours less than the brother driving in the
day, then how many hours did they each drive?

83. Joe and Ellen live 21 miles apart. Departing at the same time, they cycle
toward each other. If Joe averages 8 miles per hour and Ellen averages 6
miles per hour, how long will it take them to meet?

84. If it takes 6 minutes to drive to the automobile repair shop at an average


speed of 30 miles per hour, then how long will it take to walk back at an
average rate of 4 miles per hour?

85. Jaime and Alex leave the same location and travel in opposite directions.
Traffic conditions enabled Alex to average 14 miles per hour faster than
Jaime. After 1½ hours they are 159 miles apart. Find the speed at which each
was able to travel.

86. Jane and Holly live 51 miles apart and leave at the same time traveling
toward each other to meet for lunch. Jane traveled on the freeway at twice

2.5 Applications of Linear Equations 300


Chapter 2 Linear Equations and Inequalities

the average speed as Holly. They were able to meet in a half hour. At what
rate did each travel?

Part F: Discussion Board Topics

87. Discuss ideas for calculating taxes and tips mentally.

88. Research historical methods for representing unknowns.

89. Research and compare simple interest and compound interest. What is
the difference?

90. Discuss why algebra is a required subject.

91. Research ways to show that a repeating decimal is rational. Share your
findings on the discussion board.

2.5 Applications of Linear Equations 301


Chapter 2 Linear Equations and Inequalities

ANSWERS

1: x + 6 = 37

3: 5x − 14 = 1

5: 5 (x + 6) = 20

7: 10 − (x + 3) = 5

9: 2x − 10 = x + 2

11: 8, 17

13: 22, 52

15: 4

17: 69, 70

19: 92, 93, 94

21: 15, 16

23: 118, 120

25: 18, 20

27: 33, 35

29: 17, 19, 21

31: 7, 9

33: 12 inches

35: Width: 5 feet; length: 8 feet

37: Width: 10 feet; length: 17 feet

2.5 Applications of Linear Equations 302


Chapter 2 Linear Equations and Inequalities

39: 21 centimeters

41: 21 centimeters, 42 centimeters, 42 centimeters

43: 5 inches, 7 inches, 9 inches

45: 25 units

47: 15.9 centimeters

49: 15.7 inches

51: $271.25

53: 5 years

55: 5.5%

57: $750.00

59: Joe invested $2,050 in the money market account and $450 in the CD.

61: Jane has $3,200 in the CD and $2,200 in savings.

63: Alice invested $250 at 2% and $750 at a 3%.

65: $32.76

67: $42.75

69: $37.66

71: He has 17 nickels and 22 dimes.

73: Cathy has 12 fives and 35 ten-dollar bills.

75: Mary has 7 one-dollar bills, 7 half-dollar coins, and 14 quarters.

77: 2.5 hours

2.5 Applications of Linear Equations 303


Chapter 2 Linear Equations and Inequalities

79: 6 hours

81: 8 hours

83: 1½ hours

85: Jaime: 46 miles per hour; Alex: 60 miles per hour

2.5 Applications of Linear Equations 304


Chapter 2 Linear Equations and Inequalities

2.6 Ratio and Proportion Applications

LEARNING OBJECTIVES

1. Understand the difference between a ratio and a proportion.


2. Solve proportions using cross multiplication.
3. Solve applications involving proportions, including similar triangles.

Definitions

A ratio38 is a relationship between two numbers or quantities usually expressed as a


quotient. Ratios are typically expressed using the following notation:

All of the above are equivalent forms used to express a ratio. However, the most
familiar way to express a ratio is in the form of a fraction. When writing ratios, it is
important to pay attention to the units. If the units are the same, then the ratio can
be written without them.

Example 1: Express the ratio 12 feet to 48 feet in reduced form.

Solution:

38. Relationship between two


numbers or quantities usually
expressed as a quotient.

305
Chapter 2 Linear Equations and Inequalities

Answer: 1 to 4

If the units are different, then we must be sure to include them because the ratio
represents a rate39.

Example 2: Express the ratio 220 miles to 4 hours in reduced form.

Solution:

Answer: 55 miles to 1 hour (or 55 miles per hour)

Rates are useful when determining unit cost40, or the price of each unit. We use the
unit cost to compare values when the quantities are not the same. To determine the
unit cost, divide the cost by the number of units.

Example 3: A local supermarket offers a pack of 12 sodas for $3.48 on sale, and the
local discount warehouse offers the soda in a 36-can case for $11.52. Which is the
better value?
39. A ratio where the units for the
numerator and the
denominator are different. Solution: Divide the cost by the number of cans to obtain the unit price.
40. The price of each unit.

2.6 Ratio and Proportion Applications 306


Chapter 2 Linear Equations and Inequalities

Supermarket Discount warehouse

$3.48 $11.52
= $0.29/can = $0.32/can
12 cans 36 cans

Answer: The supermarket sale price of $3.48 for a 12-pack is a better value at $0.29
per can.

A proportion41 is a statement of equality of two ratios.

This proportion is often read “a is to b as c is to d.” Here is an example of a simple


proportion,

If we clear the fractions by multiplying both sides of the proportion by the product
of the denominators, 8, then we obtain the following true statement:

Given any nonzero real numbers a, b, c, and d that satisfy a proportion, multiply
both sides by the product of the denominators to obtain the following:

41. A statement of the equality of


two ratios.

2.6 Ratio and Proportion Applications 307


Chapter 2 Linear Equations and Inequalities

This shows that cross products are equal, and is commonly referred to as cross
multiplication42.

Solving Proportions

Cross multiply to solve proportions where terms are unknown.

Example 4: Solve: 58 = n4.

Solution: Cross multiply and then solve for n.

a c
42. If b = d
, then ad = bc.

2.6 Ratio and Proportion Applications 308


Chapter 2 Linear Equations and Inequalities

Answer: n = 5
2

Example 5: Solve: 15 5
x = 6.

Solution: Cross multiply then solve for x.

Answer: x = 18

n+3
Example 6: Solve: 5 = 72.

Solution: When cross multiplying, be sure to group n + 3. Apply the distributive


property in the next step.

2.6 Ratio and Proportion Applications 309


Chapter 2 Linear Equations and Inequalities

Answer: n = 29
2

Try this! Solve: 53 = 3n−1


2
.

Answer: n = 13
9

Video Solution

(click to see video)


Applications

When setting up proportions, consistency with the units of each ratio is critical.
Units for the numerators should be the same and units for the denominators should
also be the same.

Example 7: It is claimed that 2 out of 3 dentists prefer a certain brand of


toothpaste. If 600 dentists are surveyed, then how many will say they prefer that
brand?

Solution: First, identify the unknown and assign it a variable.

2.6 Ratio and Proportion Applications 310


Chapter 2 Linear Equations and Inequalities

Since you are looking for the number of dentists who prefer the brand name out of
a total of 600 surveyed, construct the ratios with the number of dentists who prefer
the brand in the numerator and the total number surveyed in the denominator.

Cross multiply and solve for n,

Answer: The claim suggests that 400 out of 600 dentists surveyed prefer the brand
name.

Example 8: In Tulare County, 3 out of every 7 voters said yes to Proposition 40. If
42,000 people voted, how many said no to Proposition 40?

Solution: The problem gives the ratio of voters who said yes, but it asks for the
number who said no.

2.6 Ratio and Proportion Applications 311


Chapter 2 Linear Equations and Inequalities

If 3 out of 7 said yes, then we can assume 4 out of 7 said no. Set up the ratios with
the number of voters who said no in the numerator and the total number of voters
in the denominator.

Cross multiply and solve for n.

Answer: 24,000 voters out of 42,000 said no.

Example 9: The sum of two integers in the ratio of 4 to 5 is 27. Find the integers.

Solution: The sum of two integers is 27; use this relationship to avoid two variables.

The integers are given to be in the ratio of 4 to 5. Set up the following proportion:

2.6 Ratio and Proportion Applications 312


Chapter 2 Linear Equations and Inequalities

Use 27 − n to determine the other integer.

Answer: The integers are 12 and 15.

Try this! A recipe calls for 5 tablespoons of sugar for every 8 cups of flour. How
many tablespoons of sugar are required for 32 cups of flour?

Answer: 20 tablespoons of sugar

Video Solution

(click to see video)


Similar Triangles

We will often encounter proportion problems in geometry and trigonometry. One


application involves similar triangles43, which have the same shape, but not
necessarily the same size. The measures of the corresponding angles of similar
43. Triangles with the same shape triangles are equal, and the measures of the corresponding sides are proportional.
but not necessarily the same Given similar triangles ABC and RST,
size. The measures of
corresponding angles are equal
and the corresponding sides
are proportional.

2.6 Ratio and Proportion Applications 313


Chapter 2 Linear Equations and Inequalities

We may write ABC ~ RST and conclude that all of the corresponding angles are
equal. The notation indicates that angle A corresponds to angle R and that the
measures of these angles are equal: A = R.

In addition, the measures of other pairs of corresponding angles are equal: B = S and
C = T.

Use uppercase letters for angles and a lowercase letter to denote the side opposite
of the given angle. Denote the proportionality of the sides as follows:

Example 10: If triangle ABC is similar to RST, where a = 3, b = 4, c = 5, and


r = 9, then find the remaining two sides.

Solution: Draw a picture and identify the variables pictorially. Represent the
remaining unknown sides by s and t. Set up proportions and solve for the missing
sides.

2.6 Ratio and Proportion Applications 314


Chapter 2 Linear Equations and Inequalities

Answer: The two remaining sides measure 12 units and 15 units.

The reduced ratio of any two corresponding sides of similar triangles is called the
scale factor44. In the previous example, the ratio of the two given sides a and r is

Therefore, triangle ABC is similar to triangle RST with a scale factor of 1/3. This
means that each leg of triangle ABC is 1/3 of the measure of the corresponding legs
of triangle RST. Also, another interesting fact is that the perimeters of similar
triangles are in the same proportion as their sides and share the same scale factor.

44. The reduced ratio of any two


corresponding sides of similar
triangles.

2.6 Ratio and Proportion Applications 315


Chapter 2 Linear Equations and Inequalities

Example 11: If a triangle ABC has a perimeter of 12 units and is similar to RST with a
scale factor of 1/3, then find the perimeter of triangle RST.

Solution:

Scale factor 1/3 implies that the perimeters are in proportion to this ratio. Set up a
proportion as follows:

Cross multiply and solve for x.

Answer: The perimeter of triangle RST is 36 units.

KEY TAKEAWAYS

• Solve proportions by multiplying both sides of the equation by the


product of the denominators, or cross multiply.
• When setting up a proportion, it is important to ensure consistent units
in the numerators and denominators.
• The corresponding angles of similar triangles are equal and their
corresponding sides are proportional. The ratio of any two
corresponding sides determines the scale factor, which can be used to
solve many applications involving similar triangles.

2.6 Ratio and Proportion Applications 316


Chapter 2 Linear Equations and Inequalities

TOPIC EXERCISES

Part A: Ratios and Rates

Express each ratio in reduced form.

1. 100 inches : 250 inches

2. 480 pixels : 320 pixels

3. 96 feet : 72 feet

240 miles
4.
4 hours

96 feet
5.
3 seconds

6,000 revolutions
6.
4 minutes

7. Google’s average 2008 stock price and earnings per share were $465.66 and
$14.89, respectively. What was Google’s average price-to-earnings ratio in
2008? (Source: Wolfram Alpha)

8. The F-22 Raptor has two engines that each produce 35,000 pounds of
thrust. If the takeoff weight of this fighter jet is 50,000 pounds, calculate the
plane’s thrust-to-weight ratio. (Source: USAF)

9. A discount warehouse offers a box of 55 individual instant oatmeal


servings for $11.10. The supermarket offers smaller boxes of the same
product containing 12 individual servings for $3.60. Which store offers the
better value?

10. Joe and Mary wish to take a road trip together and need to decide whose
car they will take. Joe calculated that his car is able to travel 210 miles on 12
gallons of gasoline. Mary calculates that her car travels 300 miles on 19
gallons. Which of their cars gets more miles to the gallon?

Part B: Solving Proportions

2.6 Ratio and Proportion Applications 317


Chapter 2 Linear Equations and Inequalities

Solve.

2 n
11.
3
= 150

7 21
12. n = 5

1 5
13.
3
= n

12 6
14.
5
= n

n 3
15.
8
=− 2

n 5
16.
3
=− 7

2n
17. 8 = 3

5
18. n = −30

1
19. 1 = n−1

1
20. −1 =− n+1

40 5
21. − n =− 3

2n+1 3
22.
3
=− 5

5 2
23.
3n+3
= 3

n+1 1
24.
2n−1
= 3

5n+7 n−1
25.
5
= 2

n+7
26. −2n +3= 6

2.6 Ratio and Proportion Applications 318


Chapter 2 Linear Equations and Inequalities

27. Find two numbers in the ratio of 3 to 5 whose sum is 160. (Hint: Use n and
160 − n to represent the two numbers.)

28. Find two numbers in the ratio of 2 to 7 whose sum is 90.

29. Find two numbers in the ratio of −3 to 7 whose sum is 80.

30. Find two numbers in the ratio of −1 to 3 whose sum is 90.

31. A larger integer is 5 more than a smaller integer. If the two integers have
a ratio of 6 to 5 find the integers.

32. A larger integer is 7 less than twice a smaller integer. If the two integers
have a ratio of 2 to 3 find the integers.

Given the following proportions, determine each ratio, x : y.


x y
33.
3
= 4

x−2y 3y
34.
3
=− 5

2x+4y 3
35.
2x−4y
= 2

x+y 3
36. x−y = 5

Part C: Applications

Set up a proportion and then solve.

37. If 4 out of every 5 voters support the governor, then how many of the
1,200 people surveyed support the governor?

38. If 1 out of every 3 voters surveyed said they voted yes on Proposition 23,
then how many of the 600 people surveyed voted yes?

39. Out of 460 students surveyed, the ratio to support the student union
remodel project was 3 to 5. How many students were in favor of the
remodel?

2.6 Ratio and Proportion Applications 319


Chapter 2 Linear Equations and Inequalities

40. An estimated 5 out of 7 students carry credit card debt. Estimate the
number of students that carry credit card debt out of a total of 14,000
students.

41. If the ratio of female to male students at the college is 6 to 5, then


determine the number of male students out of 11,000 total students.

42. In the year 2009 it was estimated that there would be 838 deaths in the
United States for every 100,000 people. If the total US population was
estimated to be 307,212,123 people, then how many deaths in the United
States were expected in 2009? (Source: CIA World Factbook)

43. In the year 2009 it was estimated that there would be 1,382 births in the
United States for every 100,000 people. If the total US population was
estimated to be 307,212,123 people, then how many births in the United
States were expected in 2009? (Source: CIA World Factbook)

44. If 2 out of every 7 voters approve of a sales tax increase then determine
the number of voters out of the 588 surveyed who do not support the
increase.

45. A recipe calls for 1 cup of lemon juice to make 4 cups of lemonade. How
much lemon juice is needed to make 2 gallons of lemonade?

46. The classic “Shirley Temple” cocktail requires 1 part cherry syrup to 4
parts lemon-lime soda. How much cherry syrup is needed to mix the cocktail
given a 12-ounce can of lemon-lime soda?

47. A printer prints 30 pages in 1 minute. How long will it take to print a
720-page booklet?

48. A typist can type 75 words per minute. How long will it take to type 72
pages if there are approximately 300 words per page?

1
49. On a particular map, every 16
inch represents 1 mile. How many miles
does 3 12 inches represent?

50. On a graph every 1 centimeter represents 100 feet. What measurement


on the map represents one mile?

2.6 Ratio and Proportion Applications 320


Chapter 2 Linear Equations and Inequalities

51. A candy store offers mixed candy at $3.75 for every half-pound. How
much will 2.6 pounds of candy cost?

52. Mixed nuts are priced at $6.45 per pound. How many pounds of mixed
nuts can be purchased with $20.00?

53. Corn at the farmers market is bundled and priced at $1.33 for 6 ears. How
many ears can be purchased with $15.00?

54. If 4 pizzas cost $21.00, then how much will 16 pizzas cost?

3
55. A sweetened breakfast cereal contains 110 calories in one 4
-cup serving.
How many calories are in a 1 78 -cup serving?

56. Chicken-flavored rice contains 300 calories in each 2.5-ounce serving.


How many calories are in a 4-ounce scoop of chicken-flavored rice?

57. A 200-pound man would weigh about 33.2 pounds on the moon. How
much will a 150-pound man weigh on the moon?

58. A 200-pound man would weigh about 75.4 pounds on Mars. How much
will a 150-pound man weigh on Mars?

59. There is a 1 out of 6 chance of rolling a 1 on a six-sided die. How many


times can we expect a 1 to come up in 360 rolls of the die?

60. There is a 1 out of 6 chance of rolling a 7 with two six-sided dice. How
many times can we expect a 7 to come up in 300 rolls?

61. The ratio of peanuts to all nuts in a certain brand of packaged mixed nuts
is 3 to 5. If the package contains 475 nuts, then how many peanuts can we
expect?

62. A mixed bag of marbles is packaged with a ratio of 6 orange marbles for
every 5 red marbles. If the package contains 216 orange marbles, then how
many red marbles can we expect?

63. A graphic designer wishes to create a 720-pixel-wide screen capture. If


the width to height ratio is to be 3:2, then to how many pixels should he set
the height?

2.6 Ratio and Proportion Applications 321


Chapter 2 Linear Equations and Inequalities

64. If a video monitor is produced in the width to height ratio of 16:9 and the
width of the monitor is 40 inches, then what is the height?

Part D: Similar Triangles

If triangle ABC is similar to triangle RST, find the remaining two sides given the
information.

65. a = 6, b = 8, c = 10, and s = 16

66. b = 36 , c = 48, r = 20, and t = 32

67. b = 2, c = 4, r = 6, and s = 4

68. b = 3, c = 2, r = 10, and t = 12

69. a = 40, c = 50, s = 3, and t = 10

70. c = 2, r = 7, s = 9, and t = 4

71. At the same time of day, a tree casts a 12-foot shadow while a 6-foot man
casts a 3-foot shadow. Estimate the height of the tree.

72. At the same time of day, a father and son, standing side by side, cast a
4-foot and 2-foot shadow, respectively. If the father is 6 feet tall, then how
tall is his son?

73. If the 6-8-10 right triangle ABC is similar to RST with a scale factor of 2/3,
then find the perimeter of triangle RST.

74. If the 3-4-5 right triangle ABC is similar to RST with a scale factor of 5,
then find the perimeter of triangle RST.

75. An equilateral triangle with sides measuring 6 units is similar to another


with scale factor 3:1. Find the length of each side of the unknown triangle.

76. The perimeter of an equilateral triangle ABC measures 45 units. If


triangle ABC ~ RST and r = 20, then what is the scale factor?

2.6 Ratio and Proportion Applications 322


Chapter 2 Linear Equations and Inequalities

77. The perimeter of an isosceles triangle ABC, where the two equal sides
each measure twice that of the base, is 60 units. If the base of a similar
triangle measures 6 units, then find its perimeter.

78. The perimeter of an isosceles triangle ABC measures 11 units and its two
equal sides measure 4 units. If triangle ABC is similar to triangle RST and
triangle RST has a perimeter of 22 units, then find all the sides of triangle
RST.

79. A 6-8-10 right triangle ABC is similar to a triangle RST with perimeter 72
units. Find the length of each leg of triangle RST.

80. The perimeter of triangle ABC is 60 units and b = 20 units. If


ABC ~ RST and s = 10 units, then find the perimeter of triangle RST.

Part E: Discussion Board Topics

81. What is the golden ratio and where does it appear?

82. Research and discuss the properties of similar triangles.

83. Discuss the mathematics of perspective.

84. Research and discuss the various aspect ratios that are available in
modern media devices.

2.6 Ratio and Proportion Applications 323


Chapter 2 Linear Equations and Inequalities

ANSWERS

1: 2:5

3: 4:3

5: 32 feet per second

7: 31.27

9: The discount warehouse

11: n = 100

13: n = 15

15: n = −12

17: n = 12

19: n =2

21: n = 24
3
23: n = 2

19
25: n =− 5

27: 60, 100

29: −60, 140

31: 25, 30

33: 3/4

35: 10

37: 960 people

2.6 Ratio and Proportion Applications 324


Chapter 2 Linear Equations and Inequalities

39: 276 students

41: 5,000 male students

43: 4,245,672 births

45: 8 cups of lemon juice

47: 24 minutes

49: 56 miles

51: $19.50

53: 66 ears

55: 275 calories

57: 24.9 pounds

59: 60 times

61: 285 peanuts

63: 480 pixels

65: t = 20, r = 12

67: a = 3, t = 8

69: r = 8, b = 15

71: 24 feet

73: 36 units

75: 2 units

77: 30 units

2.6 Ratio and Proportion Applications 325


Chapter 2 Linear Equations and Inequalities

79: r = 18 units, s = 24 units, t = 30 units

2.6 Ratio and Proportion Applications 326


Chapter 2 Linear Equations and Inequalities

2.7 Introduction to Inequalities and Interval Notation

LEARNING OBJECTIVES

1. Graph the solutions of a single inequality on a number line and express


the solutions using interval notation.
2. Graph the solutions of a compound inequality on a number line, and
express the solutions using interval notation.

Unbounded Intervals

An algebraic inequality45, such as x ≥ 2 , is read “x is greater than or equal to 2.”


This inequality has infinitely many solutions for x. Some of the solutions are 2, 3,
3.5, 5, 20, and 20.001. Since it is impossible to list all of the solutions, a system is
needed that allows a clear communication of this infinite set. Two common ways of
expressing solutions to an inequality are by graphing them on a number line46
and using interval notation47.

To express the solution graphically, draw a number line and shade in all the values
that are solutions to the inequality. Interval notation is textual and uses specific
notation as follows:

45. Expressions related with the


symbols ≤, <, ≥, and >.

46. Solutions to an algebraic


inequality expressed by
shading the solution on a
number line.

47. A textual system of expressing


solutions to an algebraic
inequality.
Determine the interval notation after graphing the solution set on a number line.
The numbers in interval notation should be written in the same order as they
48. An inequality that includes the appear on the number line, with smaller numbers in the set appearing first. In this
boundary point indicated by
the “or equal” part of the
example, there is an inclusive inequality48, which means that the lower-bound 2 is
symbols ≤ and ≥ and a closed included in the solution. Denote this with a closed dot on the number line and a
dot on the number line. square bracket in interval notation. The symbol (∞) is read as infinity49 and
49. The symbol (∞) indicates the
indicates that the set is unbounded to the right on a number line. Interval notation
interval is unbounded to the requires a parenthesis to enclose infinity. The square bracket indicates the
right. boundary is included in the solution. The parentheses indicate the boundary is not

327
Chapter 2 Linear Equations and Inequalities

included. Infinity is an upper bound to the real numbers, but is not itself a real
number: it cannot be included in the solution set.

Now compare the interval notation in the previous example to that of the strict, or
noninclusive, inequality that follows:

Strict inequalities50 imply that solutions may get very close to the boundary point,
in this case 2, but not actually include it. Denote this idea with an open dot on the
number line and a round parenthesis in interval notation.

Example 1: Graph and give the interval notation equivalent: x < 3 .

Solution: Use an open dot at 3 and shade all real numbers strictly less than 3. Use
negative infinity51 (−∞) to indicate that the solution set is unbounded to the left on
a number line.

Answer: Interval notation: (−∞, 3)

Example 2: Graph and give the interval notation equivalent: x ≤ 5 .


50. Express ordering relationships
using the symbol < for “less
than” and > for “greater than.” Solution: Use a closed dot and shade all numbers less than and including 5.
51. The symbol (−∞) indicates the
interval is unbounded to the
left.

2.7 Introduction to Inequalities and Interval Notation 328


Chapter 2 Linear Equations and Inequalities

Answer: Interval notation: (−∞, 5]

It is important to see that 5 ≥ x is the same as x ≤ 5 . Both require values of x to be


smaller than or equal to 5. To avoid confusion, it is good practice to rewrite all

shortened form of infinity. For example, (−∞, 5] can be expressed textually as


inequalities with the variable on the left. Also, when using text, use “inf” as a

(−inf, 5].

A compound inequality52 is actually two or more inequalities in one statement


joined by the word “and” or by the word “or.” Compound inequalities with the
logical “or” require that either condition must be satisfied. Therefore, the solution
set of this type of compound inequality consists of all the elements of the solution
sets of each inequality. When we join these individual solution sets it is called the
union53, denoted ∪. For example, the solutions to the compound inequality
x < 3 or x ≥ 6 can be graphed as follows:

Sometimes we encounter compound inequalities where the separate solution sets


overlap. In the case where the compound inequality contains the word “or,” we
combine all the elements of both sets to create one set containing all the elements
of each.

52. Two inequalities in one


Example 3: Graph and give the interval notation equivalent: x ≤ −1 or x < 3.
statement joined by the word
“and” or by the word “or.”
Solution: Combine all solutions of both inequalities. The solutions to each
53. The set formed by joining the inequality are sketched above the number line as a means to determine the union,
individual solution sets
which is graphed on the number line below.
indicated by the logical use of
the word “or” and denoted
with the symbol ∪.

2.7 Introduction to Inequalities and Interval Notation 329


Chapter 2 Linear Equations and Inequalities

Answer: Interval notation: (−∞, 3)

Any real number less than 3 in the shaded region on the number line will satisfy at
least one of the two given inequalities.

Example 4: Graph and give the interval notation equivalent: x < 3 or x ≥ −1.

Solution: Both solution sets are graphed above the union, which is graphed below.

Answer: Interval notation: R = (−∞, ∞)

When you combine both solution sets and form the union, you can see that all real
numbers satisfy the original compound inequality.

In summary,

2.7 Introduction to Inequalities and Interval Notation 330


Chapter 2 Linear Equations and Inequalities

and

Bounded Intervals

An inequality such as

reads “−1 one is less than or equal to x and x is less than three.” This is a compound
inequality because it can be decomposed as follows:

2.7 Introduction to Inequalities and Interval Notation 331


Chapter 2 Linear Equations and Inequalities

The logical “and” requires that both conditions must be true. Both inequalities are
satisfied by all the elements in the intersection54, denoted ∩, of the solution sets of
each.

Example 5: Graph and give the interval notation equivalent: x < 3 and x ≥ −1.

Solution: Determine the intersection, or overlap, of the two solution sets. The
solutions to each inequality are sketched above the number line as a means to
determine the intersection, which is graphed on the number line below.

Here x = 3 is not a solution because it solves only one of the inequalities.

Answer: Interval notation: [−1, 3)

Alternatively, we may interpret −1 ≤ x < 3 as all possible values for x between or


bounded by −1 and 3 on a number line. For example, one such solution is x = 1 .
Notice that 1 is between −1 and 3 on a number line, or that −1 < 1 < 3. Similarly, we
can see that other possible solutions are −1, −0.99, 0, 0.0056, 1.8, and 2.99. Since

solution graphically and/or with interval notation, in this case [−1, 3).
there are infinitely many real numbers between −1 and 3, we must express the

54. The set formed by the shared


values of the individual
solution sets that is indicated
by the logical use of the word
“and,” denoted with the
symbol ∩.

2.7 Introduction to Inequalities and Interval Notation 332


Chapter 2 Linear Equations and Inequalities

Example 6: Graph and give the interval notation equivalent: − 32 < x < 2.

Solution: Shade all real numbers bounded by, or strictly between, − 3


2
= −1 12and
2.

Answer: Interval notation: (− 3


2
, 2)

Example 7: Graph and give the interval notation equivalent: −5 < x ≤ 15.

Solution: Shade all real numbers between −5 and 15, and indicate that the upper
bound, 15, is included in the solution set by using a closed dot.

Answer: Interval notation: (−5, 15]

In the previous two examples, we did not decompose the inequalities; instead we
chose to think of all real numbers between the two given bounds.

In summary,

2.7 Introduction to Inequalities and Interval Notation 333


Chapter 2 Linear Equations and Inequalities

Set-Builder Notation

In this text, we use interval notation. However, other resources that you are likely
to encounter use an alternate method for describing sets called set-builder
notation55. We have used set notation to list the elements such as the integers

The braces group the elements of the set and the ellipsis marks indicate that the
integers continue forever. In this section, we wish to describe intervals of real
numbers—for example, the real numbers greater than or equal to 2.

Since the set is too large to list, set-builder notation allows us to describe it using
familiar mathematical notation. An example of set-builder notation follows:

55. A system for describing sets


using familiar mathematical
notation.

2.7 Introduction to Inequalities and Interval Notation 334


Chapter 2 Linear Equations and Inequalities

Here x∈R describes the type of number, where the symbol (∈) is read “element of.”
This implies that the variable x represents a real number. The vertical bar (|) is read
“such that.” Finally, the statement x ≥ 2 is the condition that describes the set
using mathematical notation. At this point in our study of algebra, it is assumed
that all variables represent real numbers. For this reason, you can omit the “∈R”
and write {x||x ≥ 2} , which is read “the set of all real numbers x such that x is
greater than or equal to 2.”

To describe compound inequalities such as x < 3 or x ≥ 6, write


{x||x < 3 or x ≥ 6} , which is read “the set of all real numbers x such that x is less
than 3 or x is greater than or equal to 6.”

Write bounded intervals, such as −1 ≤ x < 3, as {x|| − 1 ≤ x < 3} , which is read


“the set of all real numbers x such that x is greater than or equal to −1 and less than
3.”

2.7 Introduction to Inequalities and Interval Notation 335


Chapter 2 Linear Equations and Inequalities

KEY TAKEAWAYS

• Inequalities usually have infinitely many solutions, so rather than


presenting an impossibly large list, we present such solutions sets either
graphically on a number line or textually using interval notation.
• Inclusive inequalities with the “or equal to” component are indicated
with a closed dot on the number line and with a square bracket using
interval notation.
• Strict inequalities without the “or equal to” component are indicated
with an open dot on the number line and a parenthesis using interval
notation.
• Compound inequalities that make use of the logical “or” are solved by
solutions of either inequality. The solution set is the union of each
individual solution set.
• Compound inequalities that make use of the logical “and” require that
all inequalities are solved by a single solution. The solution set is the
intersection of each individual solution set.
• Compound inequalities of the form n < A < m can be decomposed
into two inequalities using the logical “and.” However, it is just as valid
to consider the argument A to be bounded between the values n and m.

2.7 Introduction to Inequalities and Interval Notation 336


Chapter 2 Linear Equations and Inequalities

TOPIC EXERCISES

Part A: Simple Inequalities

Graph all solutions on a number line and provide the corresponding interval
notation.

1. x ≤ 10

2. x > −5

3. x >0

4. x ≤0

5. x ≤ −3

6. x ≥ −1

7. −4 <x

8. 1 ≥x
1
9. x <− 2

3
10. x ≥− 2

3
11. x ≥ −1 4

3
12. x < 4

Part B: Compound Inequalities

Graph all solutions on a number line and give the corresponding interval notation.

13. −2 <x<5

14. −5 ≤ x ≤ −1

2.7 Introduction to Inequalities and Interval Notation 337


Chapter 2 Linear Equations and Inequalities

15. −5 < x ≤ 20

16. 0 ≤ x < 15

17. 10 < x ≤ 40

18. −40 ≤ x < −10

19. 0 < x ≤ 50

20. −30 <x<0


5 1
21. − <x<
8 8

3 1
22. − ≤x≤
4 2

1
23. −1 ≤x<1 2

1 1
24. −1 <x<−
2 2

25. x < −3 or x > 3

26. x < −2 or x ≥ 4

27. x ≤ 0 or x > 10

28. x ≤ −20 or x ≥ −10


2 1
29. x <− or x >
3 3

4 1
30. x ≤− or x > −
3 3

31. x > −5 or x < 5

32. x < 12 or x > −6

33. x < 3 or x ≥ 3

2.7 Introduction to Inequalities and Interval Notation 338


Chapter 2 Linear Equations and Inequalities

34. x ≤ 0 or x > 0

35. x < −7 or x < 2

36. x ≥ −3 or x > 0

37. x ≥ 5 or x > 0

38. x < 15 or x ≤ 10

39. x > −2 and x < 3

40. x ≥ 0 and x < 5

41. x ≥ −5 and x ≤ −1

42. x < −4 and x > 2

43. x ≤ 3 and x > 3

44. x ≤ 5 and x ≥ 5

45. x ≤ 0 and x ≥ 0

46. x < 2 and x ≤ −1

47. x > 0 and x ≥ −1

48. x < 5 and x < 2

Part C: Interval Notation

Determine the inequality given the answers expressed in interval notation.

49. (−∞, 7]

50. (−4, ∞)

51. [− 1
2
, ∞)

2.7 Introduction to Inequalities and Interval Notation 339


Chapter 2 Linear Equations and Inequalities

52. (−∞, − 3)

53. (−8, 10]

54. (−20, 0]

55. (−14, − 2)

56. [
3]
2 4
3
,

57. (−
2)
3 1
4
,

58. (−∞, − 8)

59. (8, ∞)

60. (−∞, 4) ∪ [8, ∞)

61. (−∞, − 2] ∪ [0, ∞)

62. (−∞, − 5] ∪ (5, ∞)

63. (−∞, 0) ∪ (2, ∞)

64. (−∞, − 15) ∪ (−5, ∞)

Write an equivalent inequality.

65. All real numbers less than 27.

66. All real numbers less than or equal to zero.

67. All real numbers greater than 5.

68. All real numbers greater than or equal to −8.

2.7 Introduction to Inequalities and Interval Notation 340


Chapter 2 Linear Equations and Inequalities

69. All real numbers strictly between −6 and 6.

70. All real numbers strictly between −80 and 0.

Part D: Discussion Board Topics

71. Compare interval notation with set-builder notation. Share an example


of a set described using both systems.

72. Explain why we do not use a bracket in interval notation when infinity is
an endpoint.

73. Research and discuss the different compound inequalities, particularly


unions and intersections.

74. Research and discuss the history of infinity.

75. Research and discuss the contributions of Georg Cantor.

76. What is a Venn diagram? Explain and post an example.

2.7 Introduction to Inequalities and Interval Notation 341


Chapter 2 Linear Equations and Inequalities

ANSWERS

1: (−∞, 10]

3: (0, ∞)

5: (−∞, − 3]

7: (−4, ∞)

9: (−∞, − 12 )

11: [−1 3
4
, ∞)

13: (−2, 5)

15: (−5, 20]

2.7 Introduction to Inequalities and Interval Notation 342


Chapter 2 Linear Equations and Inequalities

17: (10, 40]

19: (0, 50]

21: (− 5
8
, 1
8 )

23: [−1, 1 12 )

25: (−∞, − 3) ∪ (3, ∞)

27: (−∞, 0] ∪ (10, ∞)

29: (−∞, − 23 ) ∪ ( 13 , ∞)

31: R

2.7 Introduction to Inequalities and Interval Notation 343


Chapter 2 Linear Equations and Inequalities

33: R

35: (−∞, 2)

37: (0, ∞)

39: (−2, 3)

41: [−5, − 1]

43: ∅

45: {0}

47: (0, ∞)

2.7 Introduction to Inequalities and Interval Notation 344


Chapter 2 Linear Equations and Inequalities

49: x ≤7
1
51: x ≥− 2

53: −8 < x ≤ 10

55: −14 < x < −2


3 1
57: − <x<
4 2

59: x >8

61: x ≤ −2 or x ≥ 0

63: x < 0 or x > 2

65: x < 27

67: x >5

69: −6 <x<6

2.7 Introduction to Inequalities and Interval Notation 345


Chapter 2 Linear Equations and Inequalities

2.8 Linear Inequalities (One Variable)

LEARNING OBJECTIVES

1. Identify linear inequalities and check solutions.


2. Solve linear inequalities and express the solutions graphically on a
number line and in interval notation.
3. Solve compound linear inequalities and express the solutions
graphically on a number line and in interval notation.
4. Solve applications involving linear inequalities and interpret the results.

Definition of a Linear Inequality

A linear inequality56 is a mathematical statement that relates a linear expression


as either less than or greater than another. The following are some examples of
linear inequalities, all of which are solved in this section:

A solution to a linear inequality57 is a real number that will produce a true


statement when substituted for the variable. Linear inequalities have either
infinitely many solutions or no solution. If there are infinitely many solutions,
graph the solution set on a number line and/or express the solution using interval
notation.

Example 1: Are x = −2 and x = 4 solutions to 3x + 7 < 16?

Solution: Substitute the values for x, simplify, and check to see if we obtain a true
56. A mathematical statement statement.
relating a linear expression as
either less than or greater than
another.

57. A real number that produces a


true statement when its value
is substituted for the variable.

346
Chapter 2 Linear Equations and Inequalities

Answer: x = −2 is a solution and x = 4 is not.

Algebra of Linear Inequalities

All but one of the techniques learned for solving linear equations apply to solving
linear inequalities. You may add or subtract any real number to both sides of an
inequality, and you may multiply or divide both sides by any positive real number to
create equivalent inequalities. For example,

Both subtracting 7 from each side and dividing each side by +5 results in an
equivalent inequality that is true.

Example 2: Solve and graph the solution set: 3x + 7 < 16.

Solution:

2.8 Linear Inequalities (One Variable) 347


Chapter 2 Linear Equations and Inequalities

It is helpful to take a minute and choose a few values in and out of the solution set,
substitute them into the original inequality, and then verify the results. As
indicated, you should expect x = 0 to solve the original inequality, but x = 5
should not.

Checking in this manner gives a good indication that the inequality is solved
correctly. This can be done mentally.

Answer: Interval notation: (−∞, 3)

When working with linear inequalities, a different rule applies when multiplying or
dividing by a negative number. To illustrate the problem, consider the true
statement 10 > −5 and divide both sides by −5.

2.8 Linear Inequalities (One Variable) 348


Chapter 2 Linear Equations and Inequalities

Dividing by −5 results in a false statement. To retain a true statement, the inequality


must be reversed.

The same problem occurs when multiplying by a negative number. This leads to the
following new rule: when multiplying or dividing by a negative number, reverse the
inequality. It is easy to forget to do this so take special care to watch for negative
coefficients.

In general, given algebraic expressions A and B, where c is a positive nonzero real


number, we have the following properties of inequalities58:

58. Properties used to obtain


equivalent inequalities and
used as a means to solve them.

2.8 Linear Inequalities (One Variable) 349


Chapter 2 Linear Equations and Inequalities

We use these properties to obtain an equivalent inequality59, one with the same
solution set, where the variable is isolated. The process is similar to solving linear
equations.

Example 3: Solve: −2x + 1 ≥ 21.

Solution:

Answer: Interval notation: (−∞, − 10]

Example 4: Solve: −7 (2x + 1) < 1.

Solution:

59. Inequalities that share the


same solution set.

2.8 Linear Inequalities (One Variable) 350


Chapter 2 Linear Equations and Inequalities

Answer: Interval notation: (− 4


7
, ∞)

Example 5: Solve: 5x − 3 (2x − 1) ≥ 2(x − 3).

Solution:

2.8 Linear Inequalities (One Variable) 351


Chapter 2 Linear Equations and Inequalities

Answer: Interval notation: (−∞, 3]

Try this! Solve: 3 − 5(x − 1) ≤ 28.

Answer: [−4, ∞)

Video Solution

(click to see video)


Compound Inequalities

Following are some examples of compound linear inequalities:

These compound inequalities60 are actually two inequalities in one statement


joined by the word “and” or by the word “or.” For example,

is a compound inequality because it can be decomposed as follows:

60. Two or more inequalities in


one statement joined by the
word “and” or by the word
“or.”

2.8 Linear Inequalities (One Variable) 352


Chapter 2 Linear Equations and Inequalities

Solve each inequality individually, and the intersection of the two solution sets
solves the original compound inequality. While this method works, there is another
method that usually requires fewer steps. Apply the properties of this section to all
three parts of the compound inequality with the goal of isolating the variable in the
middle of the statement to determine the bounds of the solution set.

Example 6: Solve: −3 < 2x + 5 < 17.

Solution:

Answer: Interval notation: (−4, 6)

Example 7: Solve: −1 ≤ 12 x − 3 < 1.

Solution:

2.8 Linear Inequalities (One Variable) 353


Chapter 2 Linear Equations and Inequalities

Answer: Interval notation: [4, 8)

It is important to note that when multiplying or dividing all three parts of a


compound inequality by a negative number, you must reverse all of the inequalities
in the statement. For example,

The answer above can be written in an equivalent form, where smaller numbers lie
to the left and the larger numbers lie to the right, as they appear on a number line.

Using interval notation, write (−10, 5).

2.8 Linear Inequalities (One Variable) 354


Chapter 2 Linear Equations and Inequalities

Try this! Solve: −8 ≤ 2 (−3x + 5) < 34.

Answer: (−4, 3]

Video Solution

(click to see video)

For compound inequalities with the word “or” you must work both inequalities
separately and then consider the union of the solution sets. Values in this union
solve either inequality.

Example 8: Solve: 3x + 1 < 10 or 2x − 1 ≥ 11

Solution: Solve each inequality and form the union by combining the solution sets.

Answer: Interval notation: (−∞, 3) ∪ [6, ∞)

Try this! Solve: 4x − 1 < −5 or 4x − 1 > 5.

Answer: (−∞, −1) ∪ ( 32 , ∞)

2.8 Linear Inequalities (One Variable) 355


Chapter 2 Linear Equations and Inequalities

Video Solution

(click to see video)


Applications of Linear Inequalities

Some of the key words and phrases that indicate inequalities are summarized
below:

Key Phrases Translation

A number is at least 5.
x≥5
A number is 5 or more inclusive.

A number is at most 3.
x≤3
A number is 3 or less inclusive.

A number is strictly less than 4.


x<4
A number is less than 4, noninclusive.

A number is greater than 7.


x>7
A number is more than 7, noninclusive.

A number is in between 2 and 10. 2 < x < 10

A number is at least 5 and at most 15.


5 ≤ x ≤ 15
A number may range from 5 to 15.

As with all applications, carefully read the problem several times and look for key
words and phrases. Identify the unknowns and assign variables. Next, translate the
wording into a mathematical inequality. Finally, use the properties you have
learned to solve the inequality and express the solution graphically or in interval
notation.

Example 9: Translate: Five less than twice a number is at most 25.

Solution: First, choose a variable for the unknown number and identify the key
words and phrases.

2.8 Linear Inequalities (One Variable) 356


Chapter 2 Linear Equations and Inequalities

Answer: 2n − 5 ≤ 25. The key phrase “is at most” indicates that the quantity has a
maximum value of 25 or smaller.

Example 10: The temperature in the desert can range from 10°C to 45°C in one
24-hour period. Find the equivalent range in degrees Fahrenheit, F, given that
C = 59 (F − 32).

Solution: Set up a compound inequality where the temperature in Celsius is


inclusively between 10°C and 45°C. Then substitute the expression equivalent to the
Celsius temperature in the inequality and solve for F.

Answer: The equivalent Fahrenheit range is from 50°F to 113°F.

Example 11: In the first four events of a meet, a gymnast scores 7.5, 8.2, 8.5, and 9.0.
What must she score on the fifth event to average at least 8.5?

Solution: The average must be at least 8.5; this means that the average must be
greater than or equal to 8.5.

2.8 Linear Inequalities (One Variable) 357


Chapter 2 Linear Equations and Inequalities

Answer: She must score at least 9.3 on the fifth event.

KEY TAKEAWAYS

• Inequalities typically have infinitely many solutions. The solutions are


presented graphically on a number line or using interval notation or
both.
• All but one of the rules for solving linear inequalities are the same as for
solving linear equations. If you divide or multiply an inequality by a
negative number, reverse the inequality to obtain an equivalent
inequality.
• Compound inequalities involving the word “or” require us to solve each
inequality and form the union of each solution set. These are the values
that solve at least one of the given inequalities.
• Compound inequalities involving the word “and” require the
intersection of the solution sets for each inequality. These are the values
that solve both or all of the given inequalities.
• The general guidelines for solving word problems apply to applications
involving inequalities. Be aware of a new list of key words and phrases
that indicate a mathematical setup involving inequalities.

2.8 Linear Inequalities (One Variable) 358


Chapter 2 Linear Equations and Inequalities

TOPIC EXERCISES

Part A: Checking for Solutions

Determine whether the given number is a solution to the given inequality.

1. 2x − 3 < 6; x = −1

2. −3x + 1 ≤ 0; x = −2

3. 5x − 20 > 0; x = 3
1 3 1
4.
2
x+1>− 4
; x=− 4

5. −5 < 7x + 1 < 9; x = 0

6. −20 ≤ −3x − 5 ≤ −10; x = 5

7. x < −3 or x > 3 ; x = −10


1
8. x < 0 or x ≥ 1; x = 2

9. 2x + 1 < −3 or 2x + 1 ≥ 5 ; x = 2

10. 4x − 1 < −17 or 3x + 2 ≥ 6 ; x = 1

Part B: Solving Linear Inequalities

Solve and graph the solution set. In addition, present the solution set in interval
notation.

11. x +5>1

12. x − 3 < −4

13. 6x ≤ 24

14. 4x > −8

2.8 Linear Inequalities (One Variable) 359


Chapter 2 Linear Equations and Inequalities

15. −7x ≤ 14

16. −2x +5>9

17. 7x − 3 ≤ 25

18. 12x + 7 > −53

19. −2x + 5 < −7

20. −2x +4≤4

21. −15x + 10 > 20

22. −8x + 1 ≤ 29
1
23. 7 x−3<1

1 1 2
24.
2
x− 3
> 3

5 1 1
25.
3
x+ 2
≤ 3

3 1 5
26. − x− ≥
4 2 2

1 3 1
27. − x+ <−
5 4 5

2
28. − x + 1 < −3
3

29. 2 (−3x + 1) < 14

30. −7 (x − 2) + 1 < 15

31. 9x − 3 (3x + 4) > −12

32. 12x − 4 (3x + 5) ≤ −2

33. 5 − 3 (2x − 6) ≥ −1

2.8 Linear Inequalities (One Variable) 360


Chapter 2 Linear Equations and Inequalities

34. 9x − (10x − 12) < 22

35. 2 (x − 7) − 3 (x + 3) ≤ −3

36. 5x − 3 > 3x + 7

37. 4 (3x − 2) ≤ −2 (x + 3) + 12

38. 5 (x − 3) ≥ 15x − (10x + 4)

39. 12x + 1 > 2 (6x − 3) − 5

40. 3 (x − 2) + 5 > 2 (3x + 5) + 2

41. −4 (3x − 1) + 2x ≤ 2 (4x − 1) − 3

42. −2(x − 2) + 14x < 7(2x + 1)

Set up an algebraic inequality and then solve it.

43. The sum of three times a number and 4 is greater than negative 8.

44. The sum of 7 and three times a number is less than or equal to 1.

45. When a number is subtracted from 10, the result is at most 12.

46. When 5 times a number is subtracted from 6, the result is at least 26.

47. If five is added to three times a number, then the result is less than
twenty.

48. If three is subtracted from two times a number, then the result is greater
than or equal to nine.

49. Bill earns $12.00 for the day plus $0.25 for every person he gets to
register to vote. How many people must he register to earn at least $50.00
for the day?

2.8 Linear Inequalities (One Variable) 361


Chapter 2 Linear Equations and Inequalities

50. With a golf club membership costing $100 per month, each round of golf
costs only $25.00. How many rounds of golf can a member play if he wishes
to keep his costs to $250 per month at most?

51. Joe earned scores of 72, 85, and 75 on his first three algebra exams. What
must he score on the fourth exam to average at least 80?

52. Maurice earned 4, 7, and 9 points out of 10 on the first three quizzes.
What must he score on the fourth quiz to average at least 7?

53. A computer is set to shut down if the temperature exceeds 40°C. Give an
equivalent statement using degrees Fahrenheit. (Hint: C = 59 (F − 32).)

54. A certain brand of makeup is guaranteed not to run if the temperature is


less than 35°C. Give an equivalent statement using degrees Fahrenheit.

Part C: Compound Inequalities

Solve and graph the solution set. In addition, present the solution set in interval
notation.

55. −1 <x+3<5

56. −10 ≤ 5x < 20

57. −2 ≤ 4x + 6 < 10

58. −10 ≤ 3x − 1 ≤ −4

59. −15 < 3x − 6 ≤ 6

60. −22 < 5x + 3 ≤ 3


1
61. −1 ≤ x−5≤1
2

62. 1 < 8x + 5 < 5


1 2 1 4
63. − ≤ x− <
5 3 5 5

2.8 Linear Inequalities (One Variable) 362


Chapter 2 Linear Equations and Inequalities

1 3 2 1
64. − < x− ≤
2 4 3 2

65. −3 ≤ 3(x − 1) ≤ 3

66. −12 < 6(x − 3) ≤ 0

67. 4 < −2 (x + 3) < 6

68. −5 ≤ 5 (−x + 1) < 15

( 2 x − 1) +
3 1 1 3 3
69. − ≤ <
2 4 4 2

1
70. −4 ≤− (3x + 12) < 4
3

71. −2 ≤ 12 − 2(x − 3) ≤ 20

72. −5 < 2 (x − 1) − 3 (x + 2) < 5

73. 3x ≤ −15 or 2x > 6

74. 4x − 1 < −17 or 3x + 2 ≥ 8

75. −2x + 1 < −1 or − 2x + 1 > 1

76. 7x + 4 ≤ 4 or 6x − 5 ≥ 1

77. 3x − 7 < 14 or 2x + 3 > 7

78. −3x + 1 < −5 or − 4x − 3 > −23


1 1
79.
2
x − 2 < −1 or 2
x−2>1

1 1
80.
3
x + 3 ≥ −2 or 3
x+3≤2

81. 3x + 7 ≤ 7 or − 5x + 6 > 6

82. −10x − 3 ≤ 17 or 20x − 6 > −26

2.8 Linear Inequalities (One Variable) 363


Chapter 2 Linear Equations and Inequalities

83. 2x − 10 < −2 or − 3x + 4 > −5

84. 5x + 3 < 4 or 5 − 10x > 4

85. 3x < 18 and 5x > −20

86. x + 7 ≤ 5 and x − 3 ≥ −10

87. 2x − 1 < 5 and 3x − 1 < 10

88. 5x + 2 < −13 and 3x + 4 > 13

Set up a compound inequality for the following and then solve.

89. Five more than two times some number is between 15 and 25.

90. Four subtracted from three times some number is between −4 and 14.

91. Clint wishes to earn a B, which is at least 80 but less than 90. What range
must he score on the fourth exam if the first three were 65, 75, and 90?

92. A certain antifreeze is effective for a temperature range of −35°C to


120°C. Find the equivalent range in degrees Fahrenheit.

93. The average temperature in London ranges from 23°C in the summer to
14°C in the winter. Find the equivalent range in degrees Fahrenheit.

94. If the base of a triangle measures 5 inches, then in what range must the
height be for the area to be between 10 square inches and 20 square inches?

95. A rectangle has a length of 7 inches. Find all possible widths if the area is
to be at least 14 square inches and at most 28 square inches.

96. A rectangle has a width of 3 centimeters. Find all possible lengths, if the
perimeter must be at least 12 centimeters and at most 26 centimeters.

97. The perimeter of a square must be between 40 feet and 200 feet. Find the
length of all possible sides that satisfy this condition.

2.8 Linear Inequalities (One Variable) 364


Chapter 2 Linear Equations and Inequalities

98. If two times an angle is between 180 degrees and 270 degrees, then what
are the bounds of the original angle?

99. If three times an angle is between 270 degrees and 360 degrees then what
are the bounds of the original angle?

Part D: Discussion Board Topics

100. Research and discuss the use of set-builder notation with intersections
and unions.

101. Can we combine logical “or” into one statement like we do for logical
“and”?

2.8 Linear Inequalities (One Variable) 365


Chapter 2 Linear Equations and Inequalities

ANSWERS

1: Yes

3: No

5: Yes

7: Yes

9: Yes

11: x > −4 ; (−4, ∞)

13: x ≤ 4 ; (−∞, 4]

15: x ≥ −2 ; [−2, ∞)

17: x ≤ 4 ; (−∞, 4]

19: x > 6 ; (6, ∞)

3 (
21: x <− 2
; −∞, − 23 )

2.8 Linear Inequalities (One Variable) 366


Chapter 2 Linear Equations and Inequalities

23: x < 28 ; (−∞, 28)

10 ( ]
1
25: x ≤− ; −∞, − 1
10

4 ( 4
27: x > 19
; 19 , ∞)

29: x > −2 ; (−2, ∞)

31: ∅

33: x ≤ 4 ; (−∞, 4]

35: x ≥ −20 ; [−20, ∞)

37: x ≤ 1 ; (−∞, 1]

2.8 Linear Inequalities (One Variable) 367


Chapter 2 Linear Equations and Inequalities

39: R

2 [2
41: x ≥ 1
; 1 , ∞)

43: n > −4

45: n ≥ −2

47: n <5

49: Bill must register at least 152 people.

51: Joe must earn at least an 88 on the fourth exam.

53: The computer will shut down when the temperature exceeds 104°F.

55: −4 < x < 2 ; (−4, 2)

57: −2 ≤ x < 1 ; [−2, 1)

59: −3 < x ≤ 4 ; (−3, 4]

2.8 Linear Inequalities (One Variable) 368


Chapter 2 Linear Equations and Inequalities

61: 8 ≤ x ≤ 12 ; [8, 12]

2 [ )
3
63: 0 ≤x< ; 0, 3
2

65: 0 ≤ x ≤ 2 ; [0, 2]

67: −6 < x < −5 ; (−6, − 5)

69: −16 ≤ x < 8 ; [−16, 8)

71: −1 ≤ x ≤ 10 ; [−1, 10]

73: x ≤ −5 or x > 3 ; (−∞, − 5] ∪ (3, ∞)

75: x > 1 or x < 0 ; (−∞, 0) ∪ (1, ∞)

2.8 Linear Inequalities (One Variable) 369


Chapter 2 Linear Equations and Inequalities

77: R

79: x < 2 or x > 6 ; (−∞, 2) ∪ (6, ∞)

81: x ≤ 0 ; (−∞, 0]

83: x < 4 ; (−∞, 4)

85: −4 < x < 6 ; (−4, 6)

87: x < 3 ; (−∞, 3)

89: 5 < n < 20

91: Clint must earn a score in the range from 90 to 100.

93: The average temperature in London ranges from 57.2°F to 73.4°F.

95: The width must be at least 2 inches and at most 4 inches.

97: Sides must be between 10 feet and 50 feet.

2.8 Linear Inequalities (One Variable) 370


Chapter 2 Linear Equations and Inequalities

99: The angle is between 90 degrees and 120 degrees.

2.8 Linear Inequalities (One Variable) 371


Chapter 2 Linear Equations and Inequalities

2.9 Review Exercises and Sample Exam

372
Chapter 2 Linear Equations and Inequalities

REVIEW EXERCISES

Introduction to Algebra

Evaluate.

1. 2x + 7 , where x = −4

2. −4x + 1 , where x = −2
2 1 3
3.
3
y− 2
, where y = 5

3 5 2
4. − y+ , where y =
4 3 3

2 1
5. b − 4ac, where a = 5, b = −2 , and c = 2

2 1
6. b − 4ac, where a = − ,b = −1 , and c = −3
4

7. 2x 2 − x + 3 , where x = −3

8. 5x 2 − 2x + 4 , where x = −1

9. Calculate the simple interest earned on a 3-year investment of $750 at an


annual interest rate of 8%.

10. A bus traveled for 1 23 hours at an average speed of 48 miles per hour.
What distance did the bus travel?

11. Calculate the area of a rectangle with dimensions 4½ feet by 6 feet.

12. Calculate the volume of a rectangular box with dimensions 4½ feet by 6


feet by 1 foot.

Simplifying Algebraic Expressions

Multiply.

13. −5 (3x − 2)

2.9 Review Exercises and Sample Exam 373


Chapter 2 Linear Equations and Inequalities

14. (6x − 9) ⋅ 3

(4x − 8x + 32)
3 2
15.
4

16. −20 ( x − 54 )
1 2
10
x2 − 5

17. − (3a − 2b + 5c − 1)

18. −6 (y 3 + 3y 2 − 7y + 5)

Simplify.

19. 5a − 7b − 3a + 5b

20. 6x 2 − 4x + 7x 2 − 3x
3 1 1 1
21.
5
xy + 2
− 10
xy − 4

3 4 1 1
22. − a− b+ a− 7
b
4 21 3

23. a2 b + 2ab 2 − 7a2 b + 9ab 2

24. y 2 − 3y + 5 − y 2 + 9

25. −8 (8x − 3) − 7

26. 7 − (6x − 9)

27. 2 (3x 2 − 2x + 1) − (5x − 7)

28. (2y 2 + 6y − 8) − (5y 2 − 12y + 1)

29. 6 − 3 (a − 2b) + 7 (5a − 3b)

30. 10 − 5 (x 2 − x + 1) − (3x 2 + 5x − 1)

2.9 Review Exercises and Sample Exam 374


Chapter 2 Linear Equations and Inequalities

31. Subtract 5x −1 from 2x − 3.

32. Subtract x − 3 from twice the quantity x − 1 .

Solving Linear Equations: Part I

Is the given value a solution to the linear equation?

33. −x + 3 = −18 ; x = −15

34. 4x − 3 = −3x ; x = −2
1
35. 8x + 2 = 5x + 1 ; x = − 3

36. 2x + 4 = 3x − 2 ; x = −1

Solve.

37. y + 23 = 25

38. −3x = 54
x
39.
4
=8

5 2
40.
2
x= 3

41. 7x − 5 = −54

42. −2x + 7 = 43

43. 7x +3=0

44. 4x +5=5

45. 1 = 10 − 3x

46. 10 − 5y = 15

47. 7 − y = 28

2.9 Review Exercises and Sample Exam 375


Chapter 2 Linear Equations and Inequalities

48. 33 − x = 16
5 1 3
49.
6
x+ 3
= 2

2 1 1
50. − y+ =−
3 5 3

51. The sum of 9x and 6 is 51.

52. The difference of 3x and 8 is 25.

Solving Linear Equations: Part II

Solve.

53. 5x − 2 = 3x + 6

54. 7x + 1 = 2x − 29

55. 14x + 1 = 15x − 11

56. 6y − 13 = 3 + 7y

57. 8y + 6 − 3y = 22 − 3y

58. 12 − 5y + 6 = y − 6

59. 5 − 2 (7x − 1) = 2x + 1

60. 10 − 5 (x − 1) = 5 − x

61. 2x − (3x − 4) = 7 − x

62. 9x − 3 (2x + 1) = 3x − 3

63. 2 (5x − 2) − 3 (2x + 1) = 5 (x − 3)

64. 3 (5x − 1) − 4 (x − 4) = −5 (2x + 10)

2.9 Review Exercises and Sample Exam 376


Chapter 2 Linear Equations and Inequalities

3 1
65.
2
(4x − 3) + 4
=1

3 1
66.
4
− 6
(4x − 9) = 2

67.
2
3
(9x − 3) + 1
2
= 3 (2x − 12 )

68. 1 − 5
4
(4x − 1) = 5 ( 12 − x)

69. The sum of 4x and 3 is equal to the difference of 7x and 8.

70. The difference of 5x and 1 is equal to the sum of 12x and 1.

71. Solve for x: y = 9x + 1

72. Solve for y: 5x + 2y = 3

73. Solve for l: P = 2l + 2w


1
74. Solve for b: A= 2
bh

Applications of Linear Equations

75. A larger integer is 3 more than twice a smaller integer. If their sum is 39,
then find the integers.

76. A larger integer is 5 more than 3 times a smaller integer. If their sum is
49, then find the integers.

77. The sum of three consecutive odd integers is 45. Find the integers.

78. The sum of three consecutive even integers is 72. Find the integers.

79. The sum of three consecutive integers is 60. Find the integers.

80. The length of a rectangle is 7 centimeters less than twice its width. If the
perimeter measures 46 centimeters, then find the dimensions of the
rectangle.

2.9 Review Exercises and Sample Exam 377


Chapter 2 Linear Equations and Inequalities

81. A triangle has sides whose measures are consecutive even integers. If the
perimeter is 24 meters, then find the measure of each side.

82. The circumference of a circle measures 24π inches. Find the radius of the
circle.

83. Mary invested $1,800 in two different accounts. One account earned 3.5%
simple interest and the other earned 4.8%. If the total interest after 1 year
was $79.25, then how much did she invest in each account?

84. James has $6 in dimes and quarters. If he has 4 fewer quarters than he
does dimes, then how many of each coin does he have?

85. Two brothers leave the house at the same time traveling in opposite
directions. One averages 40 miles per hour and the other 36 miles per hour.
How long does it take for the distance between them to reach 114 miles?

86. Driving to her grandmother’s house, Jill made several stops and was only
able to average 40 miles per hour. The return trip took 2 hours less time
because she drove nonstop and was able to average 60 miles per hour. How
long did it take Jill to drive home from her grandmother’s house?

Ratio and Proportion Applications

Solve.

3 n
87.
4
= 8

7 28
88.
3
= n

6 30
89. n = 11

n 2
90.
5
= 3

3n−1 1
91.
3
= 2

4 1
92.
2n+5
=− 3

2.9 Review Exercises and Sample Exam 378


Chapter 2 Linear Equations and Inequalities

1
93. −3 = n−1

2 1
94.
n−6
= 2n+1

95. Find two numbers in the proportion 4 to 5 whose sum is 27.

96. A larger number is 2 less than twice a smaller number. If the two
numbers are in the proportion 5 to 9, then find the numbers.

97. A recipe calls for 1½ teaspoons of vanilla extract for every 3 cups of
batter. How many teaspoons of vanilla extract should be used with 7 cups of
batter?

98. The ratio of female to male employees at a certain bank is 4 to 5. If there


are 80 female employees at the bank, then determine the total number of
employees.

If triangle ABC is similar to triangle RST, then find the remaining two sides given the
following.

99. a = 4, b = 9, c = 12, and s = 3

100. b = 7, c = 10, t = 15, and r = 6

101. At the same time of day, a pole casts a 27-foot shadow and 4-foot boy
casts a 6-foot shadow. Calculate the height of the pole.

102. An equilateral triangle with sides measuring 10 units is similar to


another equilateral triangle with scale factor of 2:3. Find the perimeter of
the unknown triangle.

Introduction to Inequalities and Interval Notation

Graph all solutions on a number line and provide the corresponding interval
notation.

103. x < −1

104. x ≤ 10

2.9 Review Exercises and Sample Exam 379


Chapter 2 Linear Equations and Inequalities

105. x ≥0

106. x > −2
1 3
107. − ≤x<
2 2

108. −20 < x < 30

109. x < 5 or x ≥ 15

110. x < 2 or x > 0

Determine the inequality given the answers expressed in interval notation.

111. (−∞, 3)

112. [−4, ∞)

113. (−2, 2)

114. (−3, 8]

115. (−∞, 1) ∪ [3, ∞)

116. (−∞, − 8] ∪ [8, ∞)

Linear Inequalities (One Variable)

Solve and graph. In addition, present the solution set in interval notation.

117. x + 2 > −1

118. −4x ≥ 16

119. 9x + 4 ≤ −5

120. 5x − 7 < 13

2.9 Review Exercises and Sample Exam 380


Chapter 2 Linear Equations and Inequalities

121. 7x + 5 − 8x ≥ 15

122. 5x − 6 + 3x < 2 + 9x − 5

123. 3x − (x − 4) > x + 4

124. 3 (2x − 1) − 3 (x − 2) ≤ 2 (x + 4)

125. 2 − 5 (x − 4) > 12

126. 3x − 5 (x − 2) ≥ 11 − 5x

127. −1 < 2x + 5 ≤ 11
1 7
128. −2 ≤ x− ≤2
4 2

129. 5x + 3 < −2 or 6x − 5 ≥ 7

130. 20 − 3x ≤ 5 or 5 − 2x ≥ 25

2.9 Review Exercises and Sample Exam 381


Chapter 2 Linear Equations and Inequalities

SAMPLE EXAM

2
1. Evaluate b − 4ac, where a = −1, b = −2 , and c = 1
2
.

2. Determine the area of a triangle given that the base measures 10


centimeters and the height measures 5 centimeters. (A = 12 bh )

Simplify.

3. 5 − 2 (4x − 1)
1 2 1 3
4.
4
x− 3
y+ 2
x− 5
y

5. (5a + 4ab − 2b) − (3a + 2ab − 3b)

6. 3x − (x 2 + 5x − 1) + (x 2 − x + 4)

Solve.

7. 2 − 5x = 27
1 3 1
8.
2
x− 4
=− 8

9. 5x − 7 = 3x − 5

10. 3 (y − 3) − (4y + 2) = 1

11. 5 (x − 2) − 3 (x + 2) = 2x − 3
5 n
12.
8
= 32

3 6
13.
n+1
=− 4

14. Solve for b: A = a + 2b .

Solve and graph the solution set. In addition, present the solution set in interval
notation.

2.9 Review Exercises and Sample Exam 382


Chapter 2 Linear Equations and Inequalities

15. 2x + 3 > 23

16. 5 (−2x + 1) ≤ 35

17. 4 (3x − 2) < 3 (2x + 1) + 1

18. −9 ≤ 3 (x + 4) ≤ 21

19. 6 (x − 13 ) < −2 or 1
5
(x + 10) ≥ 3

20. An algebra student earns 75, 79, and 89 points on the first three quizzes.
What must she score on the fourth quiz to earn an average of at least 80?

21. The sum of three consecutive odd integers is 117. Find the integers.

22. The length of a rectangle is 6 inches less than twice the width. If the
perimeter measures 39 inches, then find the dimensions of the rectangle.

23. Millie invested her $5,350 savings in two accounts. One account earns 5%
annual interest and the other earns 6.2% in annual interest. If she earned
$317.30 simple interest in 1 year, then how much was in each account?

24. Because of traffic, Joe was only able to drive an average of 42 miles per
hour on the trip to a conference. He was able to average 63 miles per hour
on the return trip and it took 1 hour less time. How long did it take Joe to
drive home from the conference?

25. A graphic designer wishes to crop an image in the width-to-height ratio


of 3:2. If the height is required to be 400 pixels, then to how many pixels
should the width be set?

2.9 Review Exercises and Sample Exam 383


Chapter 2 Linear Equations and Inequalities

REVIEW EXERCISES ANSWERS

1: −1

3: −1/10

5: −6

7: 24

9: $180

11: 27 square feet

13: −15x + 10

15: 3x 2 − 6x + 24

17: −3a + 2b − 5c + 1

19: 2a − 2b
1 1
21:
2
xy + 4

23: −6a2 b + 11ab 2

25: −64x + 17

27: 6x 2 − 9x + 9

29: 32a − 15b + 6

31: −3x −2

33: No

35: Yes

37: 2

2.9 Review Exercises and Sample Exam 384


Chapter 2 Linear Equations and Inequalities

39: 32

41: −7

43: −3/7

45: 3

47: −21

49: 7/5

51: 5

53: 4

55: 12

57: 2

59: 3/8

61: Ø

63: 8

65: 7/8

67: R

69: 11/3

y−1
71: x = 9

P−2w
73: l = 2

75: 12, 27

77: 13, 15, 17

2.9 Review Exercises and Sample Exam 385


Chapter 2 Linear Equations and Inequalities

79: 19, 20, 21

81: 6 meters, 8 meters, 10 meters

83: Mary invested $550 at 3.5% and $1,250 at 4.8%.

85: They will be 114 miles apart in 1½ hours.

87: 6

89: 11/5

91: 5/6

93: 2/3

95: 12, 15

97: 3½ teaspoons

99: t = 4, r = 4/3

101: 18 feet

103: (−∞, − 1)

105: [0, ∞)

107: [− 1
2
, 3
2 )

2.9 Review Exercises and Sample Exam 386


Chapter 2 Linear Equations and Inequalities

109: (−∞, 5) ∪ [15, ∞)

111: x <3

113: −2 <x<2

115: x < 1 or x ≥ 3

117: x > −3 ; (−3, ∞)

119: x ≤ −1 ; (−∞, − 1]

121: x ≤ −10 ; (−∞, − 10]

123: x > 0 ; (0, ∞)

125: x < 2 ; (−∞, 2)

127: −3 < x ≤ 3 ; (−3, 3]

2.9 Review Exercises and Sample Exam 387


Chapter 2 Linear Equations and Inequalities

129: x < −1 or x ≥ 2 ; (−∞, − 1) ∪ [2, ∞)

2.9 Review Exercises and Sample Exam 388


Chapter 2 Linear Equations and Inequalities

SAMPLE EXAM ANSWERS

1: 6

3: −8x +7

5: 2ab + 2a + b

7: −5

9: 1

11: Ø

13: −3

15: x > 10 ; (10, ∞)

17: x < 2 ; (−∞, 2)

19: x < 0 or x ≥ 5 ; (−∞, 0) ∪ [5, ∞)

21: The three odd integers are 37, 39, and 41.

23: Millie invested $1,200 in the account earning 5% annual interest and
$4,150 in the account earning 6.2%.

25: The width should be set to 600 pixels.

2.9 Review Exercises and Sample Exam 389


Chapter 3
Graphing Lines

390
Chapter 3 Graphing Lines

3.1 Rectangular Coordinate System

LEARNING OBJECTIVES

1. Plot points using the rectangular coordinate system.


2. Calculate the distance between any two points in the rectangular
coordinate plane.
3. Determine the midpoint between any two points.

Rectangular Coordinate System

The rectangular coordinate system1 consists of two real number lines that
intersect at a right angle. The horizontal number line is called the x-axis2, and the
vertical number line is called the y-axis3. These two number lines define a flat
surface called a plane4, and each point on this plane is associated with an ordered
pair5 of real numbers (x, y). The first number is called the x-coordinate, and the
second number is called the y-coordinate. The intersection of the two axes is
known as the origin6, which corresponds to the point (0, 0).

1. A system with two number


lines at right angles uniquely
specifying points in a plane
using ordered pairs (x, y).

2. The horizontal number line


used as reference in the
rectangular coordinate system.

3. The vertical number line used


as reference in the rectangular
coordinate system.

4. The flat surface defined by the


x- and y-axes.

5. A pair (x, y) that identifies


position relative to the origin
on a rectangular coordinate
plane.

6. The point where the x- and y-


axes cross, denoted by (0, 0).

391
Chapter 3 Graphing Lines

An ordered pair (x, y) represents the position of a point relative to the origin. The x-
coordinate represents a position to the right of the origin if it is positive and to the
left of the origin if it is negative. The y-coordinate represents a position above the
origin if it is positive and below the origin if it is negative. Using this system, every
position (point) in the plane is uniquely identified. For example, the pair (2, 3)
denotes the position relative to the origin as shown:

This system is often called the Cartesian coordinate


system7, named after the French mathematician René
Descartes (1596–1650).

The x- and y-axes break the plane into four regions


7. Used in honor of René called quadrants8, named using roman numerals I, II,
Descartes when referring to III, and IV, as pictured. In quadrant I, both coordinates
the rectangular coordinate are positive. In quadrant II, the x-coordinate is negative
system.
and the y-coordinate is positive. In quadrant III, both
8. The four regions of a coordinates are negative. In quadrant IV, the x-
rectangular coordinate plane coordinate is positive and the y-coordinate is negative.
partly bounded by the x- and y-
axes and numbered using the
roman numerals I, II, III, and
IV.

3.1 Rectangular Coordinate System 392


Chapter 3 Graphing Lines

Portrait of René Descartes


(1596–1650) after Frans Hals,
from
http://commons.wikimedia.org/
wiki/File:Frans_Hals_-
_Portret_van_Ren%C3%A9_Desca
rtes.jpg.

Example 1: Plot the ordered pair (−3, 5) and determine the quadrant in which it
lies.

Solution: The coordinates x = −3 and y = 5 indicate a point 3 units to the left of


and 5 units above the origin.

3.1 Rectangular Coordinate System 393


Chapter 3 Graphing Lines

Answer: The point is plotted in quadrant II (QII) because the x-coordinate is


negative and the y-coordinate is positive.

Ordered pairs with 0 as one of the coordinates do not lie in a quadrant; these points
are on one axis or the other (or the point is the origin if both coordinates are 0).
Also, the scale indicated on the x-axis may be different from the scale indicated on
the y-axis. Choose a scale that is convenient for the given situation.

Example 2: Plot this set of ordered pairs: {(4, 0), (−6, 0), (0, 3), (−2, 6), (−4, −6)}.

Solution: Each tick mark on the x-axis represents 2 units and each tick mark on the
y-axis represents 3 units.

3.1 Rectangular Coordinate System 394


Chapter 3 Graphing Lines

Example 3: Plot this set of ordered pairs: {(−6, −5), (−3, −3), (0, −1), (3, 1), (6, 3)}.

Solution:

3.1 Rectangular Coordinate System 395


Chapter 3 Graphing Lines

In this example, the points appear to be collinear9, or to lie on the same line. The
entire chapter focuses on finding and expressing points with this property.

Try this! Plot the set of points {(5, 3), (−3, 2), (−2, −4), (4, −3)} and indicate in which
quadrant they lie. ([Link: Click here for printable graph paper in PDF.])

Answer:

Video Solution

(click to see video)

9. Describes points that lie on the Graphs are used in everyday life to display data visually. A line graph10 consists of a
same line. set of related data values graphed on a coordinate plane and connected by line
10. A set of related data values
segments. Typically, the independent quantity, such as time, is displayed on the x-
graphed on a coordinate plane axis and the dependent quantity, such as distance traveled, on the y-axis.
and connected by line
segments.

3.1 Rectangular Coordinate System 396


Chapter 3 Graphing Lines

Example 4: The following line graph shows the number of mathematics and
statistics bachelor’s degrees awarded in the United States each year since 1970.

Source: Digest of Education Statistics.

a. How many mathematics and statistics bachelor’s degrees were awarded in 1975?

b. In which years were the number of mathematics and statistics degrees awarded
at the low of 11,000?

Solution:

a. The scale on the x-axis represents time since 1970, so to determine the number of
degrees awarded in 1975, read the y-value of the graph at x = 5.

3.1 Rectangular Coordinate System 397


Chapter 3 Graphing Lines

Source: Digest of Education Statistics.

The y-value corresponding to x = 5 is 18. The graph indicates that this is in


thousands; there were 18,000 mathematics and statistics degrees awarded in 1975.

b. To find the year a particular number of degrees was awarded, first look at the y-
axis. In this case, 11,000 degrees is represented by 11 on the y-axis; look to the right
to see in which years this occurred.

3.1 Rectangular Coordinate System 398


Chapter 3 Graphing Lines

Source: Digest of Education Statistics.

The y-value of 11 occurs at two data points, one where x = 10 and the other where x
= 30. These values correspond to the years 1980 and 2000, respectively.

Answers:

a. In the year 1975, 18,000 mathematics and statistics degrees were awarded.

b. In the years 1980 and 2000, the lows of 11,000 mathematics and statistics degrees
were awarded.

Distance Formula

Frequently you need to calculate the distance between two points in a plane. To do
this, form a right triangle using the two points as vertices of the triangle and then
11. Given any right triangle with
legs measuring a and b units apply the Pythagorean theorem. Recall that the Pythagorean theorem11 states that
and hypotenuse measuring c if given any right triangle with legs measuring a and b units, then the square of the
units, then a2 + b2 = c2. measure of the hypotenuse c is equal to the sum of the squares of the legs:

3.1 Rectangular Coordinate System 399


Chapter 3 Graphing Lines

a2 + b2 = c2 . In other words, the hypotenuse of any right triangle is equal to the


square root of the sum of the squares of its legs.

Example 5: Find the distance between (−1, 2) and (3, 5).

Solution: Form a right triangle by drawing horizontal and vertical lines through
the two points. This creates a right triangle as shown below:

3.1 Rectangular Coordinate System 400


Chapter 3 Graphing Lines

The length of leg b is calculated by finding the distance between the x-values of the
given points, and the length of leg a is calculated by finding the distance between
the given y-values.

Next, use the Pythagorean theorem to find the length of the hypotenuse.

Answer: The distance between the two points is 5 units.

Generalize this process to produce a formula that can be used to algebraically


calculate the distance between any two given points.

Given two points, (x 1 , y 1 ) and (x 2 , y 2 ) , then the distance, d, between them is


given by the distance formula12:
12. Given two points (x 1 , y 1 ) and

d = √(x 2 − x 1 )2 + (y 2 − y 1 )
(x 2 , y 2 ), calculate the ⎯⎯⎯⎯⎯⎯⎯⎯⎯⎯⎯⎯⎯⎯⎯⎯⎯⎯⎯⎯⎯⎯⎯⎯⎯⎯⎯⎯⎯⎯⎯⎯⎯⎯⎯⎯2⎯
distance d between them using

d = √(x 2 − x 1 )2 + (y 2 − y 1 ) .
the formula
⎯⎯⎯⎯⎯⎯⎯⎯⎯⎯⎯⎯⎯⎯⎯⎯⎯⎯⎯⎯⎯⎯⎯⎯⎯⎯⎯⎯⎯⎯⎯⎯⎯⎯⎯⎯2⎯

3.1 Rectangular Coordinate System 401


Chapter 3 Graphing Lines

Example 6: Calculate the distance between (−3, −1) and (−2, 4).

Solution: Use the distance formula.

It is a good practice to include the formula in its general form as a part of the
written solution before substituting values for the variables. This improves
readability and reduces the chance for errors.

⎯⎯⎯⎯
Answer: √26 units

Try this! Calculate the distance between (−7, 5) and (−1, 13).

Answer: 10 units

Video Solution

(click to see video)

3.1 Rectangular Coordinate System 402


Chapter 3 Graphing Lines

Example 7: Do the three points (1, −1), (3, −3), and (3, 1) form a right triangle?

Solution: The Pythagorean theorem states that having side lengths that satisfy the
property a2 + b2 = c2 is a necessary and sufficient condition of right triangles. In
other words, if you can show that the sum of the squares of the leg lengths of the
triangle is equal to the square of the length of the hypotenuse, then the figure must
be a right triangle. First, calculate the length of each side using the distance
formula.

Now we check to see if a2 + b2 = c2 .

3.1 Rectangular Coordinate System 403


Chapter 3 Graphing Lines

Answer: Yes, the three points form a right triangle. In fact, since two of the legs are
equal in length, the points form an isosceles right triangle.

Midpoint Formula

The point that bisects the line segment formed by two points, (x 1 , y 1 ) and
(x 2 , y 2 ), is called the midpoint13 and is given by the following formula:

The midpoint is an ordered pair formed by finding the average of the x-values and
the average of the y-values of the given points.

Example 8: Calculate the midpoint between (−1, −2) and (7, 4).

Solution: First, calculate the average of the x- and y-values of the given points.

13. Given two points, (x 1 , y 1 )


and (x 2 , y 2 ), the midpoint is

( 2 )
an ordered pair given by
x 1 +x 2 y 1 +y 2
2
, .

3.1 Rectangular Coordinate System 404


Chapter 3 Graphing Lines

Next, form the midpoint as an ordered pair using the averaged coordinates.

Answer: (3, 1)

To verify that this is indeed the midpoint, calculate the distance between the two
given points and verify that the result is equal to the sum of the two equal distances
from the endpoints to this midpoint. This verification is left to the reader as an
exercise.

Try this! Find the midpoint between (−6, 5) and (6, −11).

Answer: (0, −3)

3.1 Rectangular Coordinate System 405


Chapter 3 Graphing Lines

Video Solution

(click to see video)

KEY TAKEAWAYS

• Use the rectangular coordinate system to uniquely identify points in a


plane using ordered pairs (x, y). Ordered pairs indicate position relative
to the origin. The x-coordinate indicates position to the left and right of
the origin. The y-coordinate indicates position above or below the
origin.
• The scales on the x-axis and y-axis may be different. Choose a scale for
each axis that is appropriate for the given problem.
• Graphs are used to visualize real-world data. Typically, independent
data is associated with the x-axis and dependent data is associated with
the y-axis.
• The Pythagorean theorem gives us a necessary and sufficient condition
of right triangles. Given a right triangle, then the measures of the sides
2
satisfy a2 + b = c2. Conversely, if the sides satisfy a2 + b 2 = c2,

= √(x 2 − x 1 )2 + (y 2 − y 1 ) , is derived
then the triangle must be a right triangle.
⎯⎯⎯⎯⎯⎯⎯⎯⎯⎯⎯⎯⎯⎯⎯⎯⎯⎯⎯⎯⎯⎯⎯⎯⎯⎯⎯⎯⎯⎯⎯⎯⎯⎯⎯⎯2⎯
• The distance formula, d
from the Pythagorean theorem and gives us the distance between any
two points, (x 1 , y 1 ) and (x 2 , y 2 ), in a rectangular coordinate plane.

( ), is derived by taking the


x 1 +x 2 y 1 +y 2
• The midpoint formula,
2
, 2
average of each coordinate and forming an ordered pair.

3.1 Rectangular Coordinate System 406


Chapter 3 Graphing Lines

TOPIC EXERCISES

Part A: Ordered Pairs

Give the coordinates of points A, B, C, D, and E.

1.

2.

3.

4.

3.1 Rectangular Coordinate System 407


Chapter 3 Graphing Lines

5.

6.

Graph the given set of ordered pairs.

7. {(−4, 5), (−1, 1), (−3, −2), (5, −1)}

8. {(−15, −10), (−5, 10), (15, 10), (5, −10)}

9. {(−2, 5), (10, 0), (2, −5), (6, −10)}

10. {(−8, 3), (−4, 6), (0, −6), (6, 9)}

11. {(−10, 5), (20, −10), (30, 15), (50, 0)}

12. {(− , − 12 ) , (− 2) ( 3 , − 1) , ( 3 , 1)}


5 1 1 2 5
3 3
, ,

3.1 Rectangular Coordinate System 408


Chapter 3 Graphing Lines

13. {(− , − 43 ) , ( 25 , 3) (1, − 3 ) , (0, 1)}


3 4 2
5
,

14. {(−3.5, 0), (−1.5, 2), (0, 1.5), (2.5, −1.5)}

15. {(−0.8, 0.2), (−0.2, −0.4), (0, −1), (0.6, −0.4)}

16. {(−1.2, −1.2), (−0.3, −0.3), (0, 0), (0.6, 0.6), (1.2, 1.2)}

State the quadrant in which the given point lies.

17. (−3, 2)

18. (5, 7)

19. (−12, −15)

20. (7, −8)

21. (−3.8, 4.6)

22. (17.3, 1.9)

23. (− 1
8
, − 58 )

24. ( 34 , − 14 )

25. x > 0 and y < 0

26. x < 0 and y < 0

27. x < 0 and y > 0

28. x > 0 and y > 0

The average price of a gallon of regular unleaded gasoline in US cities is given in the
following line graph. Use the graph to answer the following questions.

3.1 Rectangular Coordinate System 409


Chapter 3 Graphing Lines

Source: Bureau of
Labor Statistics.

29. What was the average price of a gallon of unleaded gasoline in 2004?

30. What was the average price of a gallon of unleaded gasoline in 1976?

31. In which years were the average price of a gallon of unleaded gasoline
$1.20?

32. What is the price increase of a gallon of gasoline from 1980 to 2008?

33. What was the percentage increase in the price of a gallon of unleaded
gasoline from 1976 to 1980?

34. What was the percentage increase in the price of a gallon of unleaded
gasoline from 2000 to 2008?

The average price of all-purpose white flour in US cities from 1980 to 2008 is given in
the following line graph. Use the graph to answer the questions that follow.

Source: Bureau of
Labor Statistics.

3.1 Rectangular Coordinate System 410


Chapter 3 Graphing Lines

35. What was the average price per pound of all-purpose white flour in 2000?

36. What was the average price per pound of all-purpose white flour in 2008?

37. In which year did the price of flour average $0.25 per pound?

38. In which years did the price of flour average $0.20 per pound?

39. What was the percentage increase in flour from the year 2000 to 2008?

40. What was the percentage increase in flour from the year 1992 to 2000?

Given the following data, create a line graph.

41. The percentage of total high school graduates who enrolled in college.

Year Percentage

1969 36%

1979 40%

1989 47%

1999 42%

Source: Digest of Education Statistics.

42. The average daily temperature given in degrees Fahrenheit in May.

Exam Temperature

8:00 am 60

12:00 pm 72

4:00 pm 75

8:00 pm 67

12:00 am 60

4:00 am 55

Calculate the area of the shape formed by connecting the following set of vertices.

3.1 Rectangular Coordinate System 411


Chapter 3 Graphing Lines

43. {(0, 0), (0, 3), (5, 0), (5, 3)}

44. {(−1, −1), (−1, 1), (1, −1), (1, 1)}

45. {(−2, −1), (−2, 3), (5, 3), (5, −1)}

46. {(−5, −4), (−5, 5), (3, 5), (3, −4)}

47. {(0, 0), (4, 0), (2, 2)}

48. {(−2, −2), (2, −2), (0, 2)}

49. {(0, 0), (0, 6), (3, 4)}

50. {(−2, 0), (5, 0), (3, −3)}

Part B: Distance Formula

Calculate the distance between the given two points.

51. (−5, 3) and (−1, 6)

52. (6, −2) and (−2, 4)

53. (0, 0) and (5, 12)

54. (−6, −8) and (0, 0)

55. (−7, 8) and (5, −1)

56. (−1, −2) and (9, 22)

57. (−1, 2) and (−7/2, −4)

58. (−
3)
and ( )
1 1 5 11
2
, 2
, − 3

59. (−
3)
and (1, − 13 )
1 2
3
,

60. ( , − 34 )and ( 32 , 4)
1 1
2

3.1 Rectangular Coordinate System 412


Chapter 3 Graphing Lines

61. (1, 2) and (4, 3)

62. (2, −4) and (−3, −2)

63. (−1, 5) and (1, −3)

64. (1, −7) and (5, −1)

65. (−7, −3) and (−1, 6)

66. (0, 1) and (1, 0)

67. (−0.2, −0.2) and (1.8, 1.8)

68. (1.2, −3.3) and (2.2, −1.7)

For each problem, show that the three points form a right triangle.

69. (−3, −2), (0, −2), and (0, 4)

70. (7, 12), (7, −13), and (−5, −4)

71. (−1.4, 0.2), (1, 2), and (1, −3)

72. (2, −1), (−1, 2), and (6, 3)

73. (−5, 2), (−1, −2), and (−2, 5)

74. (1, −2), (2, 3), and (−3, 4)

Isosceles triangles have two legs of equal length. For each problem, show that the
following points form an isosceles triangle.

75. (1, 6), (−1, 1), and (3, 1)

76. (−6, −2), (−3, −5), and (−9, −5)

77. (−3, 0), (0, 3), and (3, 0)

78. (0, −1), (0, 1), and (1, 0)

3.1 Rectangular Coordinate System 413


Chapter 3 Graphing Lines

Calculate the area and the perimeter of the triangles formed by the following set of
vertices.

79. {(−4, −5), (−4, 3), (2, 3)}

80. {(−1, 1), (3, 1), (3, −2)}

81. {(−3, 1), (−3, 5), (1, 5)}

82. {(−3, −1), (−3, 7), (1, −1)}

Part C: Midpoint Formula

Find the midpoint between the given two points.

83. (−1, 6) and (−7, −2)

84. (8, 0) and (4, −3)

85. (−10, 0) and (10, 0)

86. (−3, −6) and (−3, 6)

87. (−10, 5) and (14, −5)

88. (0, 1) and (2, 2)

89. (5, −3) and (4, −5)

90. (0, 0) and (1, 1)

91. (−1, −1) and (4, 4)

92. (3, −5) and (3, 5)

93. (− , − 13 )and ( 32 , 3)
1 7
2

94. ( , − 23 )and ( 18 , − 12 )
3
4

3.1 Rectangular Coordinate System 414


Chapter 3 Graphing Lines

95. (
4)
and (− , − 32 )
5 1 1
3
, 6

96. (− , − 52 )and ( 10 , − 14 )
1 7
5

97. Given the right triangle formed by the vertices (0, 0), (6, 0), and (6, 8),
show that the midpoints of the sides form a right triangle.

98. Given the isosceles triangle formed by the vertices (−10, −12), (0, 12), and
(10, −12), show that the midpoints of the sides also form an isosceles
triangle.

99. Calculate the area of the triangle formed by the vertices (−4, −3), (−1, 1),
and (2, −3). (Hint: The vertices form an isosceles triangle.)

100. Calculate the area of the triangle formed by the vertices (−2, 1), (4, 1),
and (1, −5).

Part D: Discussion Board Topics

101. Research and discuss the life and contributions to mathematics of René
Descartes.

102. Research and discuss the history of the right triangle and the
Pythagorean theorem.

103. What is a Pythagorean triple? Provide some examples.

104. Explain why you cannot use a ruler to calculate distance on a graph.

105. How do you bisect a line segment with only a compass and a
straightedge?

3.1 Rectangular Coordinate System 415


Chapter 3 Graphing Lines

ANSWERS

1: A: (3, 5); B: (−2, 3); C: (−5, 0); D: (1, −3); E: (−3, −4)

3: A: (0, 6); B: (−4, 3); C: (−8, 0); D: (−6, −6); E: (8, −9)

5: A: (−10, 25); B: (30, 20); C: (0, 10); D: (15, 0); E: (25, −10)

7:

9:

11:

13:

3.1 Rectangular Coordinate System 416


Chapter 3 Graphing Lines

15:

17: QII

19: QIII

21: QII

23: QIII

25: QIV

27: QII

29: $1.80

31: 1980 to 1984, 1996

33: 100%

35: $0.30

37: 1992

39: 67%

3.1 Rectangular Coordinate System 417


Chapter 3 Graphing Lines

41:

43: 15 square units

45: 28 square units

47: 4 square units

49: 9 square units

51: 5 units

53: 13 units

55: 15 units

57: 13/2 units

59: 5/3 units

⎯⎯⎯⎯
61: √ 10 units

⎯⎯⎯⎯
63: 2√ 17 units

⎯⎯⎯⎯
65: 3√ 13 units

67: 2.8 units

69: Proof

3.1 Rectangular Coordinate System 418


Chapter 3 Graphing Lines

71: Proof

73: Proof

75: Proof

77: Proof

79: Perimeter: 24 units; area: 24 square units

⎯⎯
81: Perimeter: 8 + 4√ 2 units; area: 8 square units

83: (−4, 2)

85: (0, 0)

87: (2, 0)

89: (9/2, −4)

91: (3/2, 3/2)

93: (1/2, 1)

95: (3/4, −5/8)

99: 12 square units

3.1 Rectangular Coordinate System 419


Chapter 3 Graphing Lines

3.2 Graph by Plotting Points

LEARNING OBJECTIVES

1. Verify solutions to linear equations with two variables.


2. Graph lines by plotting points.
3. Identify and graph horizontal and vertical lines.

Solutions to Equations with Two Variables

A linear equation with two variables14 has standard form ax + by = c, where a,


b, and c are real numbers and a and b are not both 0. Solutions to equations of this
form are ordered pairs (x, y), where the coordinates, when substituted into the
equation, produce a true statement.

Example 1: Determine whether (1, −2) and (−4, 1) are solutions to 6x − 3y = 12 .

Solution: Substitute the x- and y-values into the equation to determine whether the
ordered pair produces a true statement.

Answer: (1, −2) is a solution, and (−4, 1) is not.

14. An equation with two variables


that can be written in the
standard form ax + by = c ,
It is often the case that a linear equation is given in a form where one of the
where the real numbers a and b
are not both zero. variables, usually y, is isolated. If this is the case, then we can check that an ordered

420
Chapter 3 Graphing Lines

pair is a solution by substituting in a value for one of the coordinates and


simplifying to see if we obtain the other.

Example 2: Are ( 1
2
, − 3)and (−5, 14) solutions to y = 2x − 4 ?

Solution: Substitute the x-values and simplify to see if the corresponding y-values
are obtained.

Answer: ( 1
2
, − 3)is a solution, and (−5, 14) is not.

Try this! Is (6, −1) a solution to y = − 23 x + 3?

Answer: Yes

Video Solution

(click to see video)

When given linear equations with two variables, we can solve for one of the
variables, usually y, and obtain an equivalent equation as follows:

3.2 Graph by Plotting Points 421


Chapter 3 Graphing Lines

Written in this form, we can see that y depends on x. Here x is the independent
variable15 and y is the dependent variable16.

The linear equation y = 2x − 4 can be used to find ordered pair solutions. If we


substitute any real number for x, then we can simplify to find the corresponding y-
value. For example, if x = 3 , then y = 2(3) − 4 = 6 − 4 = 2, and we can form an
ordered pair solution, (3, 2). Since there are infinitely many real numbers to choose
for x, the linear equation has infinitely many ordered pair solutions (x, y).

Example 3: Find ordered pair solutions to the equation 5x − y = 14 with the given
x-values {−2, −1, 0, 4, 6}.

Solution: First, solve for y.

15. The variable that determines


the values of other variables.
Usually we think of the x-value
as the independent variable.

16. The variable whose value is


determined by the value of the
independent variable. Usually
we think of the y-value as the
dependent variable.

3.2 Graph by Plotting Points 422


Chapter 3 Graphing Lines

Next, substitute the x-values in the equation y = 5x − 14 to find the


corresponding y-values.

Answer: {(−2, −24), (−1, −19), (0, −14), (4, 6), (6, 16)}

In the previous example, certain x-values are given, but that is not always going to
be the case. When treating x as the independent variable, we can choose any values
for x and then substitute them into the equation to find the corresponding y-values.
This method produces as many ordered pair solutions as we wish.

Example 4: Find five ordered pair solutions to 6x + 2y = 10 .

Solution: First, solve for y.

3.2 Graph by Plotting Points 423


Chapter 3 Graphing Lines

Next, choose any set of x-values. Usually we choose some negative values and some
positive values. In this case, we will find the corresponding y-values when x is {−2,
−1, 0, 1, 2}. Make the substitutions required to fill in the following table (often
referred to as a t-chart):

Answer: {(−2, 11), (−1, 8), (0, 5), (1, 2), (2, −1)}. Since there are infinitely many
ordered pair solutions, answers may vary depending on the choice of values for the
independent variable.

Try this! Find five ordered pair solutions to 10x − 2y = 2 .

Answer: {(−2, −11), (−1, −6), (0, −1), (1, 4), (2, 9)} (answers may vary)

Video Solution

(click to see video)

3.2 Graph by Plotting Points 424


Chapter 3 Graphing Lines

Graph by Plotting Points

Since the solutions to linear equations are ordered pairs, they can be graphed using
the rectangular coordinate system. The set of all solutions to a linear equation can
be represented on a rectangular coordinate plane using a straight line connecting at
least two points; this line is called its graph17. To illustrate this, plot five ordered
pair solutions, {(−2, 11), (−1, 8), (0, 5), (1, 2), (2, −1)}, to the linear equation
6x + 2y = 10 .

Notice that the points are collinear; this will be the case for any linear equation.
Draw a line through the points with a straightedge, and add arrows on either end to
indicate that the graph extends indefinitely.

17. A point on the number line


associated with a coordinate.

3.2 Graph by Plotting Points 425


Chapter 3 Graphing Lines

The resulting line represents all solutions to 6x + 2y = 10 , of which there are


infinitely many. The steps for graphing lines by plotting points are outlined in the
following example.

Example 5: Find five ordered pair solutions and graph: 10x − 5y = 10 .

Solution:

Step 1: Solve for y.

3.2 Graph by Plotting Points 426


Chapter 3 Graphing Lines

Step2: Choose at least two x-values and find the corresponding y-values. In this
section, we will choose five real numbers to use as x-values. It is a good practice to
choose 0 and some negative numbers, as well as some positive numbers.

Five ordered pair solutions are {(−2, −6), (−1, −4), (0, −2), (1, 0), (2, 2)}

Step 3: Choose an appropriate scale, plot the points, and draw a line through them
using a straightedge. In this case, choose a scale where each tick mark on the y-axis
represents 2 units because all the y-values are multiples of 2.

Answer:

3.2 Graph by Plotting Points 427


Chapter 3 Graphing Lines

It will not always be the case that y can be solved in terms of x with integer
coefficients. In fact, the coefficients often turn out to be fractions.

Example 6: Find five ordered pair solutions and graph: −5x + 2y = 10 .

Solution:

Remember that you can choose any real number for the independent variable x, so
choose wisely here. Since the denominator of the coefficient of the variable x is 2,
you can avoid fractions by choosing multiples of 2 for the x-values. In this case,
choose the set of x-values {−6, −4, −2, 0, 2} and find the corresponding y-values.

3.2 Graph by Plotting Points 428


Chapter 3 Graphing Lines

Five solutions are {(−6, −10), (−4, −5), (−2, 0), (0, 5), (2, 10)}. Here we choose to scale
the x-axis with multiples of 2 and the y-axis with multiples of 5.

Answer:

Try this! Find five ordered pair solutions and graph: x + 2y = 6 .

Answer: {(−2, 4), (0, 3), (2, 2), (4, 1), (6, 0)}

3.2 Graph by Plotting Points 429


Chapter 3 Graphing Lines

Video Solution

(click to see video)


Horizontal and Vertical Lines

We need to recognize by inspection linear equations that represent a vertical or


horizontal line.

Example 7: Graph by plotting five points: y = −2 .

Solution: Since the given equation does not have a variable x, we can rewrite it with
a 0 coefficient for x.

Choose any five values for x and see that the corresponding y-value is always −2.

3.2 Graph by Plotting Points 430


Chapter 3 Graphing Lines

We now have five ordered pair solutions to plot {(−2, −2), (−1, −2), (0, −2), (1, −2),
(2, −2)}.

Answer:

When the coefficient for the variable x is 0, the graph is a horizontal line. In
general, the equation for a horizontal line18 can be written in the form y = k ,
where k represents any real number.

18. Any line whose equation can be


written in the form y = k, where Example 8: Graph by plotting five points: x = 3.
k is a real number.

3.2 Graph by Plotting Points 431


Chapter 3 Graphing Lines

Solution: Since the given equation does not have a variable y, rewrite it with a 0
coefficient for y.

Choose any five values for y and see that the corresponding x-value is always 3.

We now have five ordered pair solutions to plot: {(3, −2), (3, −1), (3, 0), (3, 1), (3, 2)}.

Answer:

3.2 Graph by Plotting Points 432


Chapter 3 Graphing Lines

When the coefficient for the variable y is 0, the graph is a vertical line. In general,
the equation for a vertical line19 can be written as x = k , where k represents any
real number.

To summarize, if k is a real number,

Try this! Graph y = 5 and x = −2 on the same set of axes and determine where
they intersect.

Answer: (−2, 5)

Video Solution

(click to see video)

KEY TAKEAWAYS

• Solutions to linear equations with two variables ax + by = c are


ordered pairs (x, y), where the coordinates, when substituted into the
equation, result in a true statement.
• Linear equations with two variables have infinitely many ordered pair
solutions. When the solutions are graphed, they are collinear.
• To find ordered pair solutions, choose values for the independent
variable, usually x, and substitute them in the equation to find the
corresponding y-values.
• To graph linear equations, determine at least two ordered pair solutions
and draw a line through them with a straightedge.
• Horizontal lines are described by y = k, where k is any real number.
• Vertical lines are described by x = k, where k is any real number.

19. Any line whose equation can be


written in the form x = k, where
k is a real number.

3.2 Graph by Plotting Points 433


Chapter 3 Graphing Lines

TOPIC EXERCISES

Part A: Solutions to Linear Systems

Determine whether the given point is a solution.

1. 5x − 2y = 4 ; (−1, 1)

2. 3x − 4y = 10 ; (2, −1)

3. −3x + y = −6 ; (4, 6)

4. −8x − y = 24 ; (−2, −3)

5. −x + y = −7 ; (5, −2)

6. 9x − 3y = 6 ; (0, −2)
1 1 1
7.
2
x+ 3
y=− 6
; (1, −2)

3 1
8.
4
x− 2
y = −1; (2, 1)

9. 4x − 3y = 1 ; ( 12 , 1
3 )

5 (5 )
9
10. −10x + 2y = − ; 1 , 1
10

1
11. y = x + 3 ; (6, 3)
3

12. y = −4x + 1 ; (−2, 9)


2
13. y = x − 3 ; (0, −3)
3

5
14. y =− x + 1 ; (8, −5)
8

4 (
15. y =− 1
2
x+ 3
; − 1
2
, 1)

3.2 Graph by Plotting Points 434


Chapter 3 Graphing Lines

2 (2
16. y =− 1
3
x− 1
; 1 , − 23 )

17. y = 2 ; (−3, 2)

18. y = 4 ; (4, −4)

19. x = 3 ; (3, −3)

20. x = 0 ; (1, 0)

Find the ordered pair solutions given the set of x-values.

21. y = −2x + 4 ; {−2, 0, 2}


1
22. y = x − 3 ; {−4, 0, 4}
2

3 1
23. y =− x+ ; {−2, 0, 2}
4 2

24. y = −3x + 1 ; {−1/2, 0, 1/2}

25. y = −4 ; {−3, 0, 3}
1 3
26. y = x+ ; {−1/4, 0, 1/4}
2 4

27. 2x − 3y = 1 ; {0, 1, 2}

28. 3x − 5y = −15 ; {−5, 0, 5}

29. – x + y = 3 ; {−5, −1, 0}


1 1
30.
2
x− 3
y = −4; {−4, −2, 0}

3 1
31.
5
x+ 10
y = 2; {−15, −10, −5}

32. x − y = 0 ; {10, 20, 30}

Find the ordered pair solutions, given the set of y-values.

3.2 Graph by Plotting Points 435


Chapter 3 Graphing Lines

1
33. y = x − 1 ; {−5, 0, 5}
2

3
34. y =− x + 2 ; {0, 2, 4}
4

35. 3x − 2y = 6 ; {−3, −1, 0}

36. −x + 3y = 4 ; {−4, −2, 0}


1 1
37.
3
x− 2
y = −4; {−1, 0, 1}

3 1
38.
5
x+ 10
y = 2; {−20, −10, −5}

Part B: Graphing Lines

Given the set of x-values {−2, −1, 0, 1, 2}, find the corresponding y-values and graph
them.

39. y =x+1

40. y = −x + 1

41. y = 2x − 1

42. y = −3x + 2

43. y = 5x − 10

44. 5x + y = 15

45. 3x −y=9

46. 6x − 3y = 9

47. y = −5

48. y =3

Find at least five ordered pair solutions and graph.

3.2 Graph by Plotting Points 436


Chapter 3 Graphing Lines

49. y = 2x − 1

50. y = −5x + 3

51. y = −4x + 2

52. y = 10x − 20
1
53. y =− x+2
2

1
54. y = x−1
3

2
55. y = x−6
3

2
56. y =− x+2
3

57. y =x

58. y = −x

59. −2x + 5y = −15

60. x + 5y = 5

61. 6x −y=2

62. 4x + y = 12

63. −x + 5y = 0

64. x + 2y = 0
1
65.
10
x−y=3

3
66.
2
x + 5y = 30

Part C: Horizontal and Vertical Lines

3.2 Graph by Plotting Points 437


Chapter 3 Graphing Lines

Find at least five ordered pair solutions and graph them.

67. y =4

68. y = −10

69. x =4

70. x = −1

71. y =0

72. x =0
3
73. y = 4

5
74. x =− 4

75. Graph the lines y = −4 and x = 2 on the same set of axes. Where do
they intersect?

76. Graph the lines y = 5 and x = −5 on the same set of axes. Where do
they intersect?

77. What is the equation that describes the x-axis?

78. What is the equation that describes the y-axis?

Part D: Mixed Practice

Graph by plotting points.

3
79. y =− x+6
5

3
80. y = x−3
5

81. y = −3

82. x = −5

3.2 Graph by Plotting Points 438


Chapter 3 Graphing Lines

83. 3x − 2y = 6

84. −2x + 3y = −12

Part E: Discussion Board Topics

85. Discuss the significance of the relationship between algebra and


geometry in describing lines.

86. Give real-world examples relating two unknowns.

3.2 Graph by Plotting Points 439


Chapter 3 Graphing Lines

ANSWERS

1: No

3: Yes

5: Yes

7: Yes

9: Yes

11: No

13: Yes

15: Yes

17: Yes

19: Yes

21: {(−2, 8), (0, 4), (2, 0)}

23: {(−2, 2), (0, 1/2), (2, −1)}

25: {(−3, −4), (0, −4), (3, −4)}

27: {(0, −1/3), (1, 1/3), (2, 1)}

29: {(−5, −2), (−1, 2), (0, 3)}

31: {(−15, 110), (−10, 80), (−5, 50)}

33: {(−8, −5), (2, 0), (12, 5)}

35: {(0, −3), (4/3, −1), (2, 0)}

37: {(−27/2, −1), (−12, 0), (−21/2, 1)}

3.2 Graph by Plotting Points 440


Chapter 3 Graphing Lines

39:

41:

43:

45:

47:

3.2 Graph by Plotting Points 441


Chapter 3 Graphing Lines

49:

51:

53:

55:

3.2 Graph by Plotting Points 442


Chapter 3 Graphing Lines

57:

59:

61:

63:

3.2 Graph by Plotting Points 443


Chapter 3 Graphing Lines

65:

67:

69:

71:

3.2 Graph by Plotting Points 444


Chapter 3 Graphing Lines

73:

75:

77: y =0

79:

81:

3.2 Graph by Plotting Points 445


Chapter 3 Graphing Lines

83:

3.2 Graph by Plotting Points 446


Chapter 3 Graphing Lines

3.3 Graph Using Intercepts

LEARNING OBJECTIVES

1. Identify and find x- and y-intercepts of a graph.


2. Graph a line using x- and y-intercepts.

Definition of x- and y-Intercepts

The x-intercept20 is the point where the graph of a line intersects the x-axis. The y-
intercept21 is the point where the graph of a line intersects the y-axis. These points
have the form (x, 0) and (0, y), respectively.

To find the x- and y-intercepts algebraically, use the fact that all x-intercepts have a
20. The point (or points) where a y-value of zero and all y-intercepts have an x-value of zero. To find the y-intercept,
graph intersects the x-axis, set x = 0 and determine the corresponding y-value. Similarly, to find the x-
expressed as an ordered pair intercept, set y = 0 and determine the corresponding x-value.
(x, 0).

21. The point (or points) where a


graph intersects the y-axis,
expressed as an ordered pair
(0, y).

447
Chapter 3 Graphing Lines

Example 1: Find the x- and y-intercepts: −3x + 2y = 12 .

Solution: To find the x-intercept, set y = 0.

Therefore, the x-intercept is (−4, 0). To find the y-intercept, set x = 0.

Hence the y-intercept is (0, 6). Note that this linear equation is graphed above.

Answer: x-intercept: (−4, 0); y-intercept: (0, 6)

Example 2: Find the x- and y-intercepts: y = −3x + 9 .

Solution: Begin by finding the x-intercept.

3.3 Graph Using Intercepts 448


Chapter 3 Graphing Lines

The x-intercept is (3, 0). Next, determine the y-intercept.

The y-intercept is (0, 9).

Answer: x-intercept: (3, 0); y-intercept: (0, 9)

Keep in mind that the intercepts are ordered pairs and not numbers. In other
words, the x-intercept is not x = 2 but rather (2, 0). In addition, not all graphs
necessarily have both intercepts: for example,

The horizontal line graphed above has a y-intercept of (0, −2) and no x-intercept.

3.3 Graph Using Intercepts 449


Chapter 3 Graphing Lines

The vertical line graphed above has an x-intercept (3, 0) and no y-intercept.

Try this! Find the x- and y-intercepts: 4x − y = 2 .

Answer: x-intercept: (1/2, 0); y-intercept: (0, −2)

Video Solution

(click to see video)


Graphing Lines Using Intercepts

Since two points determine a line, we can use the x- and y-intercepts to graph linear
equations. We have just outlined an easy method for finding intercepts; now we
outline the steps for graphing lines using the intercepts.

Example 3: Graph using intercepts: 2x − 3y = 12 .

Solution:

3.3 Graph Using Intercepts 450


Chapter 3 Graphing Lines

Step 1: Find the x- and y-intercepts.

Step 2: Plot the intercepts and draw the line through them. Use a straightedge to
create a nice straight line. Add an arrow on either end to indicate that the line
continues indefinitely in either direction.

Answer:

Example 4: Graph using intercepts: y = − 15 x + 3.

3.3 Graph Using Intercepts 451


Chapter 3 Graphing Lines

Solution: Begin by determining the x- and y-intercepts.

Next, graph the two points and draw a line through them with a straightedge.

Answer:

3.3 Graph Using Intercepts 452


Chapter 3 Graphing Lines

Example 5: Graph using intercepts: y = −2x .

Solution:

Here the x- and y-intercepts are actually the same point, the origin. We will need at
least one more point so that we can graph the line. Choose any value for x and
determine the corresponding value for y.

Use the ordered pair solutions (0, 0), (−1, 2), and (1, −2) to graph the line.

Answer:

3.3 Graph Using Intercepts 453


Chapter 3 Graphing Lines

To summarize, any linear equation can be graphed by finding two points and
connecting them with a line drawn with a straightedge. Two important and useful
points are the x- and y-intercepts; find these points by substituting y = 0 and x = 0,
respectively. This method for finding intercepts will be used throughout our study
of algebra.

Try this! Graph using intercepts: 3x − 5y = 15 .

Answer: x-intercept: (5, 0); y-intercept: (0, −3)

Video Solution

(click to see video)


Finding Intercepts Given the Graph

The x- and y-intercepts are important points on any graph. This chapter will focus
on the graphs of linear equations. However, at this point, we can use these ideas to
determine intercepts of nonlinear graphs. Remember that intercepts are ordered
pairs that indicate where the graph intersects the axes.

3.3 Graph Using Intercepts 454


Chapter 3 Graphing Lines

Example 6: Find the x- and y-intercepts given the following graph:

Solution: We see that the graph intersects the x-axis in two places. This graph has
two x-intercepts, namely, (−4, 0) and (2, 0). Furthermore, the graph intersects the y-
axis in one place. The only y-intercept is (0, −3).

Answer: x-intercepts: (−4, 0), (2, 0); y-intercept: (0, −3)

In our study of algebra, we will see that some graphs have many intercepts. Also, we
will see that some graphs do not have any.

Example 7: Given the following graph, find the x- and y-intercepts:

3.3 Graph Using Intercepts 455


Chapter 3 Graphing Lines

Solution: This is a graph of a circle; we can see that it does not intersect either axis.
Therefore, this graph does not have any intercepts.

Answer: None

KEY TAKEAWAYS

• Since two points determine any line, we can graph lines using the x- and
y-intercepts.
• To find the x-intercept, set y = 0 and solve for x.
• To find the y-intercept, set x = 0 and solve for y.
• This method of finding x- and y-intercepts will be used throughout our
study of algebra because it works for any equation.
• To graph a line, find the intercepts, if they exist, and draw a straight line
through them. Use a straightedge to create the line and include arrows
on either end to indicate that the line extends infinitely in either
direction.
• Horizontal and vertical lines do not always have both x- and y-
intercepts.

3.3 Graph Using Intercepts 456


Chapter 3 Graphing Lines

TOPIC EXERCISES

Part A: Intercepts

Given the graph, find the x- and y-intercepts.

1.

2.

3.

4.

3.3 Graph Using Intercepts 457


Chapter 3 Graphing Lines

5.

6.

Find the x- and y-intercepts.

7. 5x − 4y = 20

8. −2x + 7y = −28

9. x −y=3

10. −x +y=0

11. 3x − 4y = 1

12. −2x + 5y = 3

3.3 Graph Using Intercepts 458


Chapter 3 Graphing Lines

13. 1
4
x− 1
3
y=1

14. − 2
5
x+ 3
4
y=2

15. y =6

16. y = −3

17. x =2

18. x = −1

19. y = mx + b

20. ax + by = c

Part B: Graph Using Intercepts

Find the intercepts and graph them.

21. 3x + 4y = 12

22. −2x + 3y = 6

23. 5x − 2y = 10

24. −4x − 8y = 16
1 1
25. − x+ y=1
2 3

3 1
26.
4
x− 2
y = −3

5
27. 2x − y = 10
2

7
28. 2x − y = −14
3

29. 4x − y = −8

3.3 Graph Using Intercepts 459


Chapter 3 Graphing Lines

30. 6x −y=6

31. – x + 2y = 1

32. 3x + 4y = 6

33. 2x + y = −1

34. −2x + 6y = 3

35. 15x + 4y = −60

36. −25x + 3y = 75

37. 4x + 2y = 0

38. 3x −y=0

39. −12x + 6y = −4

40. 3x + 12y = −4

41. y = 2x + 4

42. y = −x + 3
1
43. y = x+1
2

2
44. y = x−3
3

2
45. y =− x+1
5

5 5
46. y =− x−
8 4

7 7
47. y =− x−
8 2

3
48. y = −x + 2

3.3 Graph Using Intercepts 460


Chapter 3 Graphing Lines

49. y =3
3
50. y = 2

51. x =5

52. x = −2

53. y = 5x

54. y = −x

Part C: Intercepts of Nonlinear Graphs

Given the graph find the x- and y-intercepts.

55.

56.

57.

3.3 Graph Using Intercepts 461


Chapter 3 Graphing Lines

58.

59.

60.

61.

3.3 Graph Using Intercepts 462


Chapter 3 Graphing Lines

62.

63.

64.

3.3 Graph Using Intercepts 463


Chapter 3 Graphing Lines

Part D: Discussion Board Topics

65. What are the x-intercepts of the line y = 0?

66. What are the y-intercepts of the line x = 0?

67. Do all lines have intercepts?

68. How many intercepts can a circle have? Draw circles showing all possible
numbers of intercepts.

69. Research and post the definitions of line segment, ray, and line. Why are
the arrows important?

3.3 Graph Using Intercepts 464


Chapter 3 Graphing Lines

ANSWERS

1: y-intercept: (0, −3); x-intercept: (4, 0)

3: y-intercept: (0, −3); x-intercept: none

5: y-intercept: (0, 0); x-intercept: (0, 0)

7: x-intercept: (4, 0); y-intercept: (0, −5)

9: x-intercept: (3, 0); y-intercept: (0, −3)

11: x-intercept: (1/3, 0); y-intercept: (0, −1/4)

13: x-intercept: (4, 0); y-intercept: (0, −3)

15: x-intercept: none; y-intercept: (0, 6)

17: x-intercept: (2, 0); y-intercept: none

19: x-intercept: (−b/m, 0); y-intercept: (0, b)

21:

23:

3.3 Graph Using Intercepts 465


Chapter 3 Graphing Lines

25:

27:

29:

31:

33:

3.3 Graph Using Intercepts 466


Chapter 3 Graphing Lines

35:

37:

39:

41:

3.3 Graph Using Intercepts 467


Chapter 3 Graphing Lines

43:

45:

47:

49:

3.3 Graph Using Intercepts 468


Chapter 3 Graphing Lines

51:

53:

55: x-intercepts: (−3, 0), (3, 0); y-intercept: (0, −3)

57: x-intercepts: (−4, 0), (0, 0); y-intercept: (0, 0)

59: x-intercepts: (−2, 0), (2, 0); y-intercept: (0, −1)

61: x-intercepts: (−3, 0), (0, 0), (2, 0); y-intercept: (0, 0)

63: x-intercepts: (−4, 0), (4, 0); y-intercepts: (0, −4), (0, 4)

3.3 Graph Using Intercepts 469


Chapter 3 Graphing Lines

3.4 Graph Using the y-Intercept and Slope

LEARNING OBJECTIVES

1. Identify and find the slope of a line.


2. Graph a line using the slope and y-intercept.

Slope

The steepness of any incline can be measured as the ratio of the vertical change to
the horizontal change. For example, a 5% incline can be written as 5/100, which
means that for every 100 feet forward, the height increases 5 feet.

In mathematics, we call the incline of a line the slope22 and use the letter m to
denote it. The vertical change is called the rise23 and the horizontal change is called
the run24.

22. The incline of a line measured


The rise and the run can be positive or negative. A positive rise corresponds to a
as the ratio of the vertical vertical change up and a negative rise corresponds to a vertical change down. A
change to the horizontal positive run denotes a horizontal change to the right and a negative run
change, often referred to as rise corresponds to a horizontal change to the left. Given the graph, we can calculate
over run.
the slope by determining the vertical and horizontal changes between any two
23. The vertical change between points.
any two points on a line.

24. The horizontal change between


any two points on a line.

470
Chapter 3 Graphing Lines

Example 1: Find the slope of the given line:

Solution: From the given points on the graph, count 3 units down and 4 units right.

Answer: m = − 34

Here we have a negative slope, which means that for every 4 units of movement to
the right, the vertical change is 3 units downward. There are four geometric cases
for the value of the slope.

3.4 Graph Using the y-Intercept and Slope 471


Chapter 3 Graphing Lines

Reading the graph from left to right, we see that lines with an upward incline have
positive slopes and lines with a downward incline have negative slopes.

If the line is horizontal, then the rise is 0:

The slope of a horizontal line is 0. If the line is vertical, then the run is 0:

The slope of a vertical line is undefined.

Try this! Find the slope of the given line:

3.4 Graph Using the y-Intercept and Slope 472


Chapter 3 Graphing Lines

Answer: m = 23

Video Solution

(click to see video)

Calculating the slope can be difficult if the graph does not have points with integer
coordinates. Therefore, we next develop a formula that allows us to calculate the
slope algebraically. Given any two points (x 1 , y 1 ) and (x 2 , y 2 ), we can obtain the
rise and run by subtracting the corresponding coordinates.

This leads us to the slope formula25. Given any two points (x 1 , y 1 ) and (x 2 , y 2 ),
25. Given two points (x 1 , y 1 ) and the slope is given by
(x 2 , y 2 ), then the slope of the
line is given by the formula
rise y 2 −y 1
m= run = x 2 −x 1.

3.4 Graph Using the y-Intercept and Slope 473


Chapter 3 Graphing Lines

Example 2: Find the slope of the line passing through (−3, −5) and (2, 1).

Solution: Given (−3, −5) and (2, 1), calculate the difference of the y-values divided by
the difference of the x-values. Since subtraction is not commutative, take care to be
consistent when subtracting the coordinates.

Answer: m = 65

We can graph the line described in the previous example and verify that the slope is
6/5.

3.4 Graph Using the y-Intercept and Slope 474


Chapter 3 Graphing Lines

Certainly the graph is optional; the beauty of the slope formula is that we can
obtain the slope, given two points, using only algebra.

Example 3: Find the slope of the line passing through (−4, 3) and (−1, −7).

Solution:

Answer: m = − 10
3

3.4 Graph Using the y-Intercept and Slope 475


Chapter 3 Graphing Lines

When using the slope formula, take care to be consistent since order does matter.
You must subtract the coordinates of the first point from the coordinates of the
second point for both the numerator and the denominator in the same order.

Example 4: Find the slope of the line passing through (7, −2) and (−5, −2).

Solution:

Answer: m = 0. As an exercise, plot the given two points and verify that they lie on
a horizontal line.

Example 5: Find the slope of the line passing through (−4, −3) and (−4, 5).

Solution:

Answer: The slope m is undefined. As an exercise, plot the given two points and
verify that they lie on a vertical line.

3.4 Graph Using the y-Intercept and Slope 476


Chapter 3 Graphing Lines

Try this! Calculate the slope of the line passing through (−2, 3) and (5, −5).

Answer: m = − 87

Video Solution

(click to see video)

When considering the slope as a rate of change it is important to include the correct
units.

Example 6: A Corvette Coupe was purchased new in 1970 for about $5,200 and
depreciated in value over time until it was sold in 1985 for $1,300. At this point, the
car was beginning to be considered a classic and started to increase in value. In the
year 2000, when the car was 30 years old, it sold at auction for $10,450. The
following line graph depicts the value of the car over time.

a. Determine the rate at which the car depreciated in value from 1970 to 1985.

3.4 Graph Using the y-Intercept and Slope 477


Chapter 3 Graphing Lines

b. Determine the rate at which the car appreciated in value from 1985 to 2000.

Solution: Notice that the value depends on the age of the car and that the slope
measures the rate in dollars per year.

a. The slope of the line segment depicting the value for the first 15 years is

Answer: The value of the car depreciated $260 per year from 1970 to 1985.

b. The slope of the line segment depicting the value for the next 15 years is

Answer: The value of the car appreciated $610 per year from 1985 to 2000.

Slope-Intercept Form of a Line

To this point, we have learned how to graph lines by plotting points and by using
the x- and y-intercepts. In addition, we have seen that we need only two points to
graph a line. In this section, we outline a process to easily determine two points
using the y-intercept and the slope. The equation of any nonvertical line can be
written in slope-intercept form26 y = mx + b . In this form, we can identify the
slope, m, and the y-intercept, (0, b).

Example 7: Determine the slope and y-intercept: y = − 45 x + 7.


26. Any nonvertical line can be
written in the form
y = mx + b , where m is the Solution: In this form, the coefficient of x is the slope, and the constant is the y-
slope and (0, b) is the y- value of the y-intercept. Therefore, by inspection, we have
intercept.

3.4 Graph Using the y-Intercept and Slope 478


Chapter 3 Graphing Lines

Answer: The y-intercept is (0, 7), and the slope is m = − 45 .

It is not always the case that the linear equation is given in slope-intercept form.
When it is given in standard form, you have to first solve for y to obtain slope-
intercept form.

Example 8: Express 3x + 5y = 30 in slope-intercept form and then identify the


slope and y-intercept.

Solution: Begin by solving for y. To do this, apply the properties of equality to first
isolate 5y and then divide both sides by 5.

Answer: Slope-intercept form: y = − 35 x + 6; y-intercept: (0, 6); slope: m = − 35

3.4 Graph Using the y-Intercept and Slope 479


Chapter 3 Graphing Lines

Once the equation is in slope-intercept form, we immediately have one point to


plot, the y-intercept. From the intercept, you can mark off the slope to plot another
point on the line. From the previous example we have

Starting from the point (0, 6), use the slope to mark another point 3 units down and
5 units to the right.

It is not necessary to check that the second point, (5, 3), solves the original linear
equation. However, we do it here for the sake of completeness.

3.4 Graph Using the y-Intercept and Slope 480


Chapter 3 Graphing Lines

Marking off the slope in this fashion produces as many ordered pair solutions as we
desire. Notice that if we mark off the slope again, from the point (5, 3), then we
obtain the x-intercept, (10, 0).

Example 9: Graph: −x + 2y = 4 .

Solution: In this example, we outline the general steps for graphing a line using
slope-intercept form.

Step 1: Solve for y to obtain slope-intercept form.

Step 2: Identify the y-intercept and slope.

3.4 Graph Using the y-Intercept and Slope 481


Chapter 3 Graphing Lines

Step 3: Plot the y-intercept and use the slope to find another ordered pair solution.
Starting from the y-intercept, mark off the slope and identify a second point. In this
case, mark a point after a rise of 1 unit and a run of 2 units.

Step 4: Draw the line through the two points with a straightedge.

Answer:

3.4 Graph Using the y-Intercept and Slope 482


Chapter 3 Graphing Lines

In this example, we notice that we could get the x-intercept by marking off the
slope in a different but equivalent manner. Consider the slope as the ratio of two
negative numbers as follows:

We could obtain another point on the line by marking off the equivalent slope down
1 unit and left 2 units. We do this twice to obtain the x-intercept, (−4, 0).

3.4 Graph Using the y-Intercept and Slope 483


Chapter 3 Graphing Lines

Marking off the slope multiple times is not necessarily always going to give us the x-
intercept, but when it does, we obtain a valuable point with little effort. In fact, it is
a good practice to mark off the slope multiple times; doing so allows you to obtain
more points on the line and produce a more accurate graph.

Example 10: Graph and find the x-intercept: y = 34 x − 2.

Solution: The equation is given in slope-intercept form. Therefore, by inspection,


we have the y-intercept and slope.

3.4 Graph Using the y-Intercept and Slope 484


Chapter 3 Graphing Lines

We can see that the x-value of the x-intercept is a mixed number between 2 and 3.
To algebraically find x-intercepts, recall that we must set y = 0 and solve for x.

Answer: The x-intercept is (2 2


3
, 0).

Example 11: Graph: x − y = 0 .

3.4 Graph Using the y-Intercept and Slope 485


Chapter 3 Graphing Lines

Solution: Begin by solving for y.

The equation y = x can be written y = 1x + 0 , and we have

Answer:

3.4 Graph Using the y-Intercept and Slope 486


Chapter 3 Graphing Lines

Try this! Graph −2x + 5y = 20 and label the x-intercept.

Answer:

Video Solution

(click to see video)

3.4 Graph Using the y-Intercept and Slope 487


Chapter 3 Graphing Lines

KEY TAKEAWAYS

• Slope measures the steepness of a line as rise over run. A positive rise
denotes a vertical change up, and a negative rise denotes a vertical
change down. A positive run denotes a horizontal change right, and a
negative run denotes a horizontal change left.
• Horizontal lines have a slope of zero, and vertical lines have undefined
slopes.
• Given any two points on a line, we can algebraically calculate the slope
rise y 2 −y 1
using the slope formula, m = run = x 2 −x 1 .
• Any nonvertical line can be written in slope-intercept form,
y = mx + b , from which we can determine, by inspection, the slope
m and y-intercept (0, b).
• If we know the y-intercept and slope of a line, then we can easily graph
it. First, plot the y-intercept, and from this point use the slope as rise
over run to mark another point on the line. Finally, draw a line through
these two points with a straightedge and add an arrow on either end to
indicate that it extends indefinitely.
• We can obtain as many points on the line as we wish by marking off the
slope multiple times.

3.4 Graph Using the y-Intercept and Slope 488


Chapter 3 Graphing Lines

TOPIC EXERCISES

Part A: Slope

Determine the slope and the y-intercept of the given graph.

1.

2.

3.

4.

3.4 Graph Using the y-Intercept and Slope 489


Chapter 3 Graphing Lines

5.

6.

Determine the slope, given two points.

7. (3, 2) and (5, 1)

8. (7, 8) and (−3, 5)

9. (2, −3) and (−3, 2)

10. (−3, 5) and (7, −5)

11. (−1, −6) and (3, 2)

12. (5, 3) and (4, 12)

13. (−9, 3) and (−6, −5)

3.4 Graph Using the y-Intercept and Slope 490


Chapter 3 Graphing Lines

14. (−22, 4) and (−8, −12)

15. ( 12 , − 13 ) and (− 1
2
, 2
3 )

16. (− 3
4
, 3
2 ) and ( 4 , − 2 )
1 1

17. (− 1
3
, 5
8 ) and ( 2 , − 4 )
1 3

18. (− 3
5
, − 32 ) and ( 101 , 4
5 )

19. (3, −5) and (5, −5)

20. (−3, 1) and (−14, 1)

21. (−2, 3) and (−2, −4)

22. (−4, −4) and (5, 5)

23. A roof drops 4 feet for every 12 feet forward. Determine the slope of the
roof.

24. A road drops 300 feet for every 5,280 feet forward. Determine the slope of
the road.

25. The following graph gives the US population of persons 65 years old and
over. At what rate did this population increase from 2000 to 2008?

Source: US Census
Bureau.

3.4 Graph Using the y-Intercept and Slope 491


Chapter 3 Graphing Lines

26. The following graph gives total consumer credit outstanding in the
United States. At what rate did consumer credit increase from 2002 to 2008?

Source: US Census
Bureau.

27. A commercial van was purchased new for $20,000 and is expected to be
worth $4,000 in 8 years. Determine the rate at which the van depreciates in
value.

28. A commercial-grade copy machine was purchased new for $4,800 and
will be considered worthless in 6 years. Determine the rate at which the
copy machine depreciates in value.

29. Find y if the slope of the line passing through (−2, 3) and (4, y) is 12.

30. Find y if the slope of the line passing through (5, y) and (6, −1) is 10.

31. Find y if the slope of the line passing through (5, y) and (−4, 2) is 0.

32. Find x if the slope of the line passing through (−3, 2) and (x, 5) is
undefined.

Part B: Slope-Intercept Form

Express the given linear equation in slope-intercept form and identify the slope and
y-intercept.

33. 6x − 5y = 30

34. −2x + 7y = 28

3.4 Graph Using the y-Intercept and Slope 492


Chapter 3 Graphing Lines

35. 9x − y = 17

36. x − 3y = 18

37. 2x − 3y = 0

38. −6x + 3y = 0

39. 2
3
x− 5
4
y = 10

40. − 4
3
x+ 1
5
y = −5

Graph the line given the slope and the y-intercept.

1
41. m = and (0, −2)
3

2
42. m =− and (0, 4)
3

43. m = 3 and (0, 1)

44. m = −2 and (0, −1)

45. m = 0 and (0, 5)

46. m undefined and (0, 0)

47. m = 1 and (0, 0)

48. m = −1 and (0, 0)


15
49. m =− and (0, 20)
3

50. m = −10 and (0, −5)

Graph using the slope and y-intercept.

2
51. y = x−2
3

3.4 Graph Using the y-Intercept and Slope 493


Chapter 3 Graphing Lines

1
52. y =− x+1
3

53. y = −3x + 6

54. y = 3x + 1
3
55. y = x
5

3
56. y =− 7
x

57. y = −8

58. y =7

59. y = −x + 2

60. y =x+1
1 3
61. y = x+
2 2

3 5
62. y =− x+
4 2

63. 4x +y=7

64. 3x −y=5

65. 5x − 2y = 10

66. −2x + 3y = 18

67. x −y=0

68. x +y=0
1 1
69.
2
x− 3
y=1

2 1
70. − x+ y=2
3 2

3.4 Graph Using the y-Intercept and Slope 494


Chapter 3 Graphing Lines

71. 3x + 2y = 1

72. 5x + 3y = 1

73. On the same set of axes, graph the three lines, where y = 3
2
x + b and
b = {−2, 0, 2}.

74. On the same set of axes, graph the three lines, where y = mx + 1 and
m = {−1/2, 0, 1/2}.

Part C: Discussion Board Topics

75. Name three methods for graphing lines. Discuss the pros and cons of
each method.

76. Choose a linear equation and graph it three different ways. Scan the
work and share it on the discussion board.

77. Why do we use the letter m for slope?

78. How are equivalent fractions useful when working with slopes?

79. Can we graph a line knowing only its slope?

Δy
80. Research and discuss the alternative notation for slope: m = Δx
.

81. What strategies for graphing lines should be brought to an exam?


Explain.

3.4 Graph Using the y-Intercept and Slope 495


Chapter 3 Graphing Lines

ANSWERS

1: y-intercept: (0, 3); slope: m = −3/4

3: y-intercept: (0, 2); slope: m = 0

5: y-intercept: (0, 0); slope: m = 2

7: −1/2

9: −1

11: 2

13: −8/3

15: −1

17: −33/20

19: 0

21: Undefined

23: −1/3

25: ½ million per year

27: $2,000 per year

29: 75

31: 2

6
33: y = x − 6 ; slope: 6/5; y-intercept: (0, −6)
5

35: y = 9x − 17 ; slope: 9; y-intercept: (0, −17)


2
37: y = x ; slope: 2/3; y-intercept: (0, 0)
3

3.4 Graph Using the y-Intercept and Slope 496


Chapter 3 Graphing Lines

8
39: y = x − 8 ; slope: 8/15; y-intercept: (0, −8)
15

41:

43:

45:

47:

49:

3.4 Graph Using the y-Intercept and Slope 497


Chapter 3 Graphing Lines

51:

53:

55:

57:

3.4 Graph Using the y-Intercept and Slope 498


Chapter 3 Graphing Lines

59:

61:

63:

65:

3.4 Graph Using the y-Intercept and Slope 499


Chapter 3 Graphing Lines

67:

69:

71:

73:

3.4 Graph Using the y-Intercept and Slope 500


Chapter 3 Graphing Lines

3.4 Graph Using the y-Intercept and Slope 501


Chapter 3 Graphing Lines

3.5 Finding Linear Equations

LEARNING OBJECTIVES

1. Given a graph, identify the slope and y-intercept.


2. Find the equation of the line using the slope and y-intercept.
3. Find the equation of the line using point-slope form.

Finding Equations Using Slope-Intercept Form

Given the algebraic equation of a line, we are able to graph it in a number of ways.
In this section, we will be given a geometric description of a line and be asked to
find the algebraic equation. Finding the equation of a line can be accomplished in a
number of ways, the first of which makes use of slope-intercept form, y = mx + b .
If we know the slope, m, and the y-intercept, (0, b), we can construct the equation.

Example 1: Find the equation of a line with slope m = − 58 and y-intercept (0, 1).

Solution: The given y-intercept implies that b = 1. Substitute the slope m and the
y-value of the y-intercept b into the equation y = mx + b .

Answer: y = − 58 x + 1

Finding a linear equation is very straightforward if the slope and y-intercept are
given. This is certainly not always the case; however, the example demonstrates

502
Chapter 3 Graphing Lines

that the algebraic equation of a line depends on these two pieces of information. If
the graph is given, then we can often read it to determine the y-intercept and slope.

Example 2: Find the equation of the line given the graph:

Solution: By reading the graph, we can see that the y-intercept is (0, 4), and thus

Furthermore, from the points (0, 4) to (4, 2), we can see that the rise is −2 units and
the run is 4 units.

Now substitute m and b into slope-intercept form:

3.5 Finding Linear Equations 503


Chapter 3 Graphing Lines

Answer: y = − 12 x + 4

Often the y-intercept and slope will not be given or are not easily discernible from
the graph. For this reason, we will develop some algebraic techniques that allow us
to calculate these quantities.

Example 3: Find the equation of the line with slope m = − 23 passing through
(−6, 3).

Solution: Begin by substituting the given slope into slope-intercept form.

For the ordered pair (−6, 3) to be a solution, it must solve the equation. Therefore,
we can use it to find b. Substitute the appropriate x- and y-values as follows:

3.5 Finding Linear Equations 504


Chapter 3 Graphing Lines

After substituting the appropriate values, solve for the only remaining variable, b.

Once we have b, we can then complete the equation:

As a check, verify that (−6, 3) solves this linear equation as follows:

Answer: y = − 23 x − 1

Example 4: Find the equation of the line given the graph:

3.5 Finding Linear Equations 505


Chapter 3 Graphing Lines

Solution: Use the graph to determine the slope. From the points (−5, 2) to (−1, 0), we
can see that the rise between the points is −2 units and the run is 4 units. Therefore,
we calculate the slope as follows:

Substitute the slope into slope-intercept form.

Now substitute the coordinates of one of the given points to find b. It does not
matter which one you choose. Here choose (−1, 0):

3.5 Finding Linear Equations 506


Chapter 3 Graphing Lines

Next, put it all together.

Answer: y = − 12 x − 12

As an exercise, substitute the coordinates of the point (−5, 2) to see that b will turn
out to be the same value. In fact, you can substitute any ordered pair solution of the
line to find b. We next outline an algebraic technique for finding the equation of a
nonvertical line passing through two given points.

Example 5: Find the equation of the line passing through (−4, −2) and (1, 3).

Solution: When finding a linear equation using slope-intercept form y = mx + b ,


the goal is to find m and then b.

Step 1: Find the slope m. In this case, given two points, use the slope formula.

3.5 Finding Linear Equations 507


Chapter 3 Graphing Lines

Substitute m = 1 into slope-intercept form.

Step 2: Find b. To do this, substitute the coordinates of any given ordered pair
solution. Use (1, 3):

Step 3: Finish building the equation by substituting in the value for b. In this case,
we use b = 2.

Answer: y = x + 2

3.5 Finding Linear Equations 508


Chapter 3 Graphing Lines

These three steps outline the process for finding the equation of any nonvertical
line in slope-intercept form. This is a completely algebraic method, but always keep
in mind the geometry behind the technique.

Note that the line has a y-intercept at (0, 2), with slope m = 1.

Example 6: Find the equation of the line passing through (−1, 3) and (5, 1).

Solution: First, find m, the slope. Given two points, use the slope formula as follows:

3.5 Finding Linear Equations 509


Chapter 3 Graphing Lines

Substitute m = − 13 into slope-intercept form.

Next, find b. Substitute the coordinates of the point (−1, 3).

Finally, substitute b = 8
3
into the equation.

3.5 Finding Linear Equations 510


Chapter 3 Graphing Lines

Answer: y = − 13 x + 83

Try this! Find the equation of the line passing through (−3, 4) and (6, −2).

Answer: y = − 23 x + 2

Video Solution

(click to see video)


Finding Equations Using a Point and the Slope

Given any point on a line and its slope, we can find the equation of that line. Begin
by applying the slope formula with a given point (x 1 , y 1 ) and a variable point
(x, y) .

The equation y − y 1 = m (x − x 1 ) is called the point-slope form of a line27. Any


nonvertical linear equation can be written in this form. It is useful for finding the
27. Any nonvertical line can be equation of a line given the slope and any ordered pair solution.
written in the form
y − y 1 = m (x − x 1 ),
where m is the slope and
(x 1 , y 1 ) is any point on the
line.

3.5 Finding Linear Equations 511


Chapter 3 Graphing Lines

Example 7: Find the equation of the line with slope m = 12 passing through (4, −1).

Solution: Use point-slope form, where m = 12 and (x 1 , y 1 ) = (4, −1).

At this point, we must choose to present the equation of our line in either standard
form or slope-intercept form.

In this textbook, we will present our lines in slope-intercept form. This facilitates
future graphing.

Answer: y = 12 x − 3

Example 8: Find the equation of the line passing through (−5, 3) with slope
m = − 25.

3.5 Finding Linear Equations 512


Chapter 3 Graphing Lines

Solution: Substitute (−5, 3) and m = − 25 into point-slope form.

Answer: y = − 25 x + 1

It is always important to understand what is occurring geometrically. Compare the


answer for the last example to the corresponding graph below.

3.5 Finding Linear Equations 513


Chapter 3 Graphing Lines

The geometric understanding is important because you will often be given graphs
from which you will need to determine a point on the line and the slope.

Example 9: Find an equation of the given graph:

Solution: Between the points (1, 1) to (3, 0), we can see that the rise is −1 unit and
the run is 2 units. The slope of the line is m = rise −1 1
run = 2 = − .2 Use this and the
point (3, 0) to find the equation as follows:

Answer: y = − 12 x + 32

3.5 Finding Linear Equations 514


Chapter 3 Graphing Lines

Example 10: Find the equation of the line passing through (−1, 1) and (7, −1).

Solution: Begin by calculating the slope using the slope formula.

Next, substitute into point-slope form using one of the given points; it does not
matter which point is used. Use m = − 14 and the point (−1, 1).

Answer: y = − 14 x + 34

Try this! Find the equation of the line passing through (4, −5) and (−4, 1).

3.5 Finding Linear Equations 515


Chapter 3 Graphing Lines

Answer: y = − 34 x − 2

Video Solution

(click to see video)

KEY TAKEAWAYS

• Given the graph of a line, you can determine the equation in two ways,
using slope-intercept form, y = mx + b , or point-slope form,
y − y 1 = m (x − x 1 ) .
• The slope and one point on the line is all that is needed to write the
equation of a line.
• All nonvertical lines are completely determined by their y-intercept and
slope.
• If the slope and y-intercept can be determined, then it is best to use
slope-intercept form to write the equation.
• If the slope and a point on the line can be determined, then it is best to
use point-slope form to write the equation.

3.5 Finding Linear Equations 516


Chapter 3 Graphing Lines

TOPIC EXERCISES

Part A: Slope-Intercept Form

Determine the slope and y-intercept.

1. 5x − 3y = 18

2. −6x + 2y = 12

3. x −y=5

4. −x +y=0

5. 4x − 5y = 15

6. −7x + 2y = 3

7. y =3
3
8. y =− 4

1 1
9.
5
x− 3
y = −1

5 3
10.
16
x+ 8
y=9

2 5 5
11. − x+ y=
3 2 4

1 3 1
12.
2
x− 4
y=− 2

Part B: Finding Equations in Slope-Intercept Form

Given the slope and y-intercept, determine the equation of the line.

13. m = 1/2; (0, 5)

14. m = 4; (0, −1)

3.5 Finding Linear Equations 517


Chapter 3 Graphing Lines

15. m = −2/3; (0, −4)

16. m = −3; (0, 9)

17. m = 0; (0, −1)

18. m = 5; (0, 0)

Given the graph, find the equation in slope-intercept form.

19.

20.

21.

22.

3.5 Finding Linear Equations 518


Chapter 3 Graphing Lines

23.

24.

Find the equation, given the slope and a point.

25. m = 2/3; (−9, 2)

26. m = −1/5; (5, −5)

27. m = 0; (−4, 3)

28. m = 3; (−2, 1)

29. m = −5; (−2, 8)

30. m = −4; (1/2, −3/2)

31. m = −1/2; (3, 2)

3.5 Finding Linear Equations 519


Chapter 3 Graphing Lines

32. m = 3/4; (1/3, 5/4)

33. m = 0; (3, 0)

34. m undefined; (3, 0)

Given two points, find the equation of the line.

35. (−6, 6), (2, 2)

36. (−10, −3), (5, 0)

37. (0, 1/2), (1/2, −1)

38. (1/3, 1/3), (2/3, 1)

39. (3, −4), (−6, −7)

40. (−5, 2), (3, 2)

41. (−6, 4), (−6, −3)

42. (−4, −4), (−1, −1)

43. (3, −3), (−5, 5)

44. (0, 8), (−4, 0)

Part C: Equations Using Point-Slope Form

Find the equation, given the slope and a point.

45. m = 1/2; (4, 3)

46. m = −1/3; (9, −2)

47. m = 6; (1, −5)

48. m = −10; (1, −20)

3.5 Finding Linear Equations 520


Chapter 3 Graphing Lines

49. m = −3; (2, 3)

50. m = 2/3; (−3, −5)

51. m = −3/4; (−8, 3)

52. m = 5; (1/5, −3)

53. m = −3; (−1/9, 2)

54. m = 0; (4, −6)

55. m = 0; (−5, 10)

56. m = 5/8; (4, 3)

57. m = −3/5; (−2, −1)

58. m = 1/4; (12, −2)

59. m = 1; (0, 0)

60. m = −3/4; (0, 0)

Given the graph, use the point-slope formula to find the equation.

61.

62.

3.5 Finding Linear Equations 521


Chapter 3 Graphing Lines

63.

64.

65.

66.

3.5 Finding Linear Equations 522


Chapter 3 Graphing Lines

Use the point-slope formula to find the equation of the line passing through the two
points.

67. (−4, 0), (0, 5)

68. (−1, 2), (0, 3)

69. (−3, −2), (3, 2)

70. (3, −1), (2, −3)

71. (−2, 4), (2, −4)

72. (−5, −2), (5, 2)

73. (−3, −1), (3, 3)

74. (1, 5), (0, 5)

75. (1, 2), (2, 4)

76. (6, 3), (2, −3)

77. (10, −3), (5, −4)

78. (−3, 3), (−1, 12)

79. (4/5, −1/3), (−1/5, 2/3)

80. (5/3, 1/3), (−10/3, −5/3)

81. (3, −1/4), (4, −1/2)

3.5 Finding Linear Equations 523


Chapter 3 Graphing Lines

82. (0, 0), (−5, 1)

83. (2, −4), (0, 0)

84. (3, 5), (3, −2)

85. (−4, 7), (−1, 7)

86. (−8, 0), (6, 0)

Part D: Applications

87. Joe has been keeping track of his cellular phone bills for the last two
months. The bill for the first month was $38.00 for 100 minutes of usage. The
bill for the second month was $45.50 for 150 minutes of usage. Find a linear
equation that gives the total monthly bill based on the minutes of usage.

88. A company in its first year of business produced 150 training manuals for
a total cost of $2,350. The following year, the company produced 50 more
manuals at a cost of $1,450. Use this information to find a linear equation
that gives the total cost of producing training manuals from the number of
manuals produced.

89. A corn farmer in California was able to produce 154 bushels of corn per
acre 2 years after starting his operation. Currently, after 7 years of
operation, he has increased his yield to 164 bushels per acre. Use this
information to write a linear equation that gives the total yield per acre
based on the number of years of operation, and use it to predict the yield for
next year.

90. A Webmaster has noticed that the number of registered users has been
steadily increasing since beginning an advertising campaign. Before starting
to advertise, he had 1,200 registered users, and after 3 months of advertising
he now has 1,590 registered users. Use this data to write a linear equation
that gives the total number of registered users, given the number of months
after starting to advertise. Use the equation to predict the number of users 7
months into the advertising campaign.

91. A car purchased new cost $22,000 and was sold 10 years later for $7,000.
Write a linear equation that gives the value of the car in terms of its age in
years.

3.5 Finding Linear Equations 524


Chapter 3 Graphing Lines

92. An antique clock was purchased in 1985 for $1,500 and sold at auction in
1997 for $5,700. Determine a linear equation that models the value of the
clock in terms of years since 1985.

Part E: Discussion Board Topics

93. Discuss the merits and drawbacks of point-slope form and y-intercept
form.

94. Research and discuss linear depreciation. In a linear depreciation model,


what do the slope and y-intercept represent?

3.5 Finding Linear Equations 525


Chapter 3 Graphing Lines

ANSWERS

1: m = 5/3; (0, −6)

3: m = 1; (0, −5)

5: m = 4/5; (0, −3)

7: m = 0; (0, 3)

9: m = 3/5; (0, 3)

11: m = 4/15; (0, 1/2)

13: y = 1
2
x+5

15: y =− 2
3
x−4

17: y = −1

19: y = −x + 3

21: y = −1

23: y = 1
2
x

25: y = 2
3
x+8

27: y =3

29: y = −5x − 2

31: y =− 1
2
x+ 7
2

33: y =0

35: y =− 1
2
x+3

37: y = −3x + 1
2

3.5 Finding Linear Equations 526


Chapter 3 Graphing Lines

39: y = 1
3
x−5

41: x = −6

43: y = −x

45: y = 1
2
x+1

47: y = 6x − 11

49: y = −3x + 9

51: y =− 3
4
x−3

53: y = −3x + 5
3

55: y = 10

57: y =− 3
5
x− 11
5

59: y =x

61: y = −2x + 5

63: y = 2
3
x+ 17
3

65: y =− 3
5
x− 2
5

67: y = 5
4
x+5

69: y = 2
3
x

71: y = −2x

73: y = 2
3
x+1

75: y = 2x

77: y = 1
5
x−5

3.5 Finding Linear Equations 527


Chapter 3 Graphing Lines

79: y = −x + 7
15

81: y =− 1
4
x+ 1
2

83: y = −2x

85: y =7

87: cost = 0.15x + 23

89: yield = 2x + 150 ; 166 bushels

91: value = −1,500x + 22,000

3.5 Finding Linear Equations 528


Chapter 3 Graphing Lines

3.6 Parallel and Perpendicular Lines

LEARNING OBJECTIVES

1. Determine the slopes of parallel and perpendicular lines.


2. Find equations of parallel and perpendicular lines.

Definition of Parallel and Perpendicular

Parallel lines28 are lines in the same plane that never intersect. Two nonvertical
lines in the same plane, with slopes m1 and m2 , are parallel if their slopes are the
same, m1 = m2 . Consider the following two lines:

Consider their corresponding graphs:

28. Lines in the same plane that do


not intersect; their slopes are Both lines have a slope m = 3
4
and thus are parallel.
the same.

529
Chapter 3 Graphing Lines

Perpendicular lines29 are lines in the same plane that intersect at right angles (90
degrees). Two nonvertical lines in the same plane, with slopes m1 and m2 , are
perpendicular if the product of their slopes is −1: m1 ⋅ m2 = −1. We can solve for
m1 and obtain m1 = − m1 . In this form, we see that perpendicular lines have
2
slopes that are negative reciprocals, or opposite reciprocals30. For example, if
given a slope

then the slope of a perpendicular line is the opposite reciprocal:

The mathematical notation m⊥ reads “m perpendicular.” We can verify that two


slopes produce perpendicular lines if their product is −1.

Geometrically, we note that if a line has a positive slope, then any perpendicular
line will have a negative slope. Furthermore, the rise and run between two
perpendicular lines are interchanged.

29. Lines in the same plane that


intersect at right angles; their
slopes are opposite reciprocals.

30. Two real numbers whose


product is −1. Given a real
number ba , the opposite
reciprocal is − ba .

3.6 Parallel and Perpendicular Lines 530


Chapter 3 Graphing Lines

Perpendicular lines have slopes that are opposite reciprocals, so remember to find
the reciprocal and change the sign. In other words,

Determining the slope of a perpendicular line can be performed mentally. Some


examples follow.

Given slope Slope of perpendicular line


1
m= 2
m ⊥ = −2
3 4
m=− 4
m⊥ = 3
1
m=3 m⊥ = − 3
1
m = −4 m⊥ = 4

3.6 Parallel and Perpendicular Lines 531


Chapter 3 Graphing Lines

Example 1: Determine the slope of a line parallel to y = −5x + 3 .

Solution: Since the given line is in slope-intercept form, we can see that its slope is
m = −5. Thus the slope of any line parallel to the given line must be the same,
m∥ = −5. The mathematical notation m∥ reads “m parallel.”

Answer: m∥ = −5

Example 2: Determine the slope of a line perpendicular to 3x − 7y = 21 .

Solution: First, solve for y and express the line in slope-intercept form.

In this form, we can see that the slope of the given line is m = 37, and thus
m⊥ = − 73.

Answer: m⊥ = − 7
3

Try this! Find the slope of the line perpendicular to 15x + 5y = 20 .

Answer: m⊥ = 1
3

3.6 Parallel and Perpendicular Lines 532


Chapter 3 Graphing Lines

Video Solution

(click to see video)


Finding Equations of Parallel and Perpendicular Lines

We have seen that the graph of a line is completely determined by two points or one
point and its slope. Often you will be asked to find the equation of a line given some
geometric relationship—for instance, whether the line is parallel or perpendicular
to another line.

Example 3: Find the equation of the line passing through (6, −1) and parallel to
y = 12 x + 2.

Solution: Here the given line has slope m = 12 , and the slope of a line parallel is
m∥ = 12. Since you are given a point and the slope, use the point-slope form of a
line to determine the equation.

Answer: y = 12 x − 4

3.6 Parallel and Perpendicular Lines 533


Chapter 3 Graphing Lines

It is important to have a geometric understanding of this question. We were asked


to find the equation of a line parallel to another line passing through a certain
point.

Through the point (6, −1) we found a parallel line, y = 12 x − 4, shown dashed.
Notice that the slope is the same as the given line, but the y-intercept is different. If
we keep in mind the geometric interpretation, then it will be easier to remember
the process needed to solve the problem.

Example 4: Find the equation of the line passing through (−1, −5) and perpendicular
to y = − 14 x + 2.

Solution: The given line has slope m = − 14 , and thus m⊥ = + 41 = 4. Substitute


this slope and the given point into point-slope form.

3.6 Parallel and Perpendicular Lines 534


Chapter 3 Graphing Lines

Answer: y = 4x − 1

Geometrically, we see that the line y = 4x − 1 , shown dashed below, passes


through (−1, −5) and is perpendicular to the given line.

It is not always the case that the given line is in slope-intercept form. Often you
have to perform additional steps to determine the slope. The general steps for
finding the equation of a line are outlined in the following example.

3.6 Parallel and Perpendicular Lines 535


Chapter 3 Graphing Lines

Example 5: Find the equation of the line passing through (8, −2) and perpendicular
to 6x + 3y = 1 .

Solution:

Step 1: Find the slope m. First, find the slope of the given line. To do this, solve for y
to change standard form to slope-intercept form, y = mx + b .

In this form, you can see that the slope is m = −2 = −2


1
, and thus
1 1
m⊥ = − −2 = + 2.

Step 2: Substitute the slope you found and the given point into the point-slope form
of an equation for a line. In this case, the slope is m⊥ = 12 and the given point is
(8, −2).

Step 3: Solve for y.

3.6 Parallel and Perpendicular Lines 536


Chapter 3 Graphing Lines

Answer: y = 12 x − 6

Example 6: Find the equation of the line passing through ( 72 , 1)and parallel to
2x + 14y = 7 .

Solution: Find the slope m by solving for y.

The given line has the slope m = − 17 , and so m∥ = − 17. We use this and the point
( 2 , 1)in point-slope form.
7

3.6 Parallel and Perpendicular Lines 537


Chapter 3 Graphing Lines

Answer: y = − 17 x + 32

through (− 12 , 2).
Try this! Find the equation of the line perpendicular to x − 3y = 9 and passing

Answer: y = −3x + 12

Video Solution

(click to see video)

When finding an equation of a line perpendicular to a horizontal or vertical line, it


is best to consider the geometric interpretation.

Example 7: Find the equation of the line passing through (−3, −2) and perpendicular
to y = 4 .

Solution: We recognize that y = 4 is a horizontal line and we want to find a


perpendicular line passing through (−3, −2).

3.6 Parallel and Perpendicular Lines 538


Chapter 3 Graphing Lines

If we draw the line perpendicular to the given horizontal line, the result is a vertical
line.

Equations of vertical lines look like x = k . Since it must pass through (−3, −2), we
conclude that x = −3 is the equation. All ordered pair solutions of a vertical line
must share the same x-coordinate.

3.6 Parallel and Perpendicular Lines 539


Chapter 3 Graphing Lines

Answer: x = −3

We can rewrite the equation of any horizontal line, y = k , in slope-intercept form


as follows:

Written in this form, we see that the slope is m = 0 = 01. If we try to find the slope
of a perpendicular line by finding the opposite reciprocal, we run into a problem:
m⊥ = − 10, which is undefined. This is why we took care to restrict the definition to
two nonvertical lines. Remember that horizontal lines are perpendicular to vertical
lines.

KEY TAKEAWAYS

• Parallel lines have the same slope.


• Perpendicular lines have slopes that are opposite reciprocals. In other
words, if m = ba , then m ⊥ = − ba .
• To find an equation of a line, first use the given information to
determine the slope. Then use the slope and a point on the line to find
the equation using point-slope form.
• Horizontal and vertical lines are perpendicular to each other.

3.6 Parallel and Perpendicular Lines 540


Chapter 3 Graphing Lines

TOPIC EXERCISES

Part A: Parallel and Perpendicular Lines

Determine the slope of parallel lines and perpendicular lines.

3
1. y =− x+8
4

1
2. y = x−3
2

3. y = 4x + 4

4. y = −3x + 7
5 1
5. y =− x−
8 2

7 3
6. y = x+
3 2

2
7. y = 9x − 5

1
8. y = −10x + 5

9. y =5
1
10. x =− 2

11. x −y=0

12. x +y=0

13. 4x + 3y = 0

14. 3x − 5y = 10

15. −2x + 7y = 14

16. −x − y = 15

3.6 Parallel and Perpendicular Lines 541


Chapter 3 Graphing Lines

1 1
17.
2
x− 3
y = −1

2 4
18. − x+ y=8
3 5

1 1
19. 2x − y=
5 10

4
20. − x − 2y = 7
5

Determine if the lines are parallel, perpendicular, or neither.

2
y= x+3
{y =
3
21.
2
3
x−3

3
y= x−1
{y =
4
22.
4
3
x+3

y = −2x + 1
{y=
23. 1
2
x+8

{ y = 3x + 2
y = 3x − 2
24.

{ x = −2
y=5
25.

y=7
{y = −
26. 1
7

{ 5x + 3y = 9
3x − 5y = 15
27.

3.6 Parallel and Perpendicular Lines 542


Chapter 3 Graphing Lines

{ 3x + 3y = 2
x−y=7
28.

{ −x + 3y = −2
2x − 6y = 4
29.

{ 6x − 3y = −3
−4x + 2y = 3
30.

{ 2x + 3y = 6
x + 3y = 9
31.

{ x − 10 = 0
y − 10 = 0
32.

{ 2y − 10 = 0
y+2=0
33.

{ 2x + 3y = 6
3x + 2y = 6
34.

{ 10x − 8y = 16
−5x + 4y = 20
35.

1
x − 13 y = 1
{
2
36.
1
6
x + 14 y = −2

Part B: Equations in Point-Slope Form

Find the equation of the line.

3.6 Parallel and Perpendicular Lines 543


Chapter 3 Graphing Lines

1
37. Parallel to y = x + 2 and passing through (6, −1).
2

3
38. Parallel to y =− x − 3 and passing through (−8, 2).
4

39. Perpendicular to y = 3x − 1 and passing through (−3, 2).

1
40. Perpendicular to y =− x + 2 and passing through (4, −3).
3

41. Perpendicular to y = −2 and passing through (−1, 5).

42. Perpendicular to x = 1
5
and passing through (5, −3).

43. Parallel to y = 3 and passing through (2, 4).

44. Parallel to x = 2 and passing through (7, −3).

45. Perpendicular to y =x and passing through (7, −13).

46. Perpendicular to y = 2x + 9 and passing through (3, −1).

1
47. Parallel to y = x − 5 and passing through (−2, 1).
4

3
48. Parallel to y =− x + 1 and passing through (4, 1/4).
4

49. Parallel to 2x − 3y = 6 and passing through (6, −2).

50. Parallel to −x +y=4 and passing through (9, 7).

51. Perpendicular to 5x − 3y = 18 and passing through (−9, 10).

52. Perpendicular to x − y = 11 and passing through (6, −8).

1 1
53. Parallel to
5
x− 3
y = 2 and passing through (−15, 6).

5 1
54. Parallel to −10x − 7
y= and passing through (−1, 1/2).
2

1 1
55. Perpendicular to
2
x− 3
y = 1 and passing through (−10, 3).

3.6 Parallel and Perpendicular Lines 544


Chapter 3 Graphing Lines

56. Perpendicular to −5x + y = −1 and passing through (−4, 0).

57. Parallel to x + 4y = 8 and passing through (−1, −2).

58. Parallel to 7x − 5y = 35 and passing through (2, −3).

59. Perpendicular to 6x + 3y = 1 and passing through (8, −2).

60. Perpendicular to −4x − 5y = 1 and passing through (−1, −1).

and passing through ( , − 14 ).


1
61. Parallel to −5x − 2y = 4 5

62. Parallel to 6x − 3
2
y = 9 and passing through ( 13 , 3)
2
.

63. Perpendicular to y − 3 = 0 and passing through (−6, 12).

64. Perpendicular to x + 7 = 0 and passing through (5, −10).

3.6 Parallel and Perpendicular Lines 545


Chapter 3 Graphing Lines

ANSWERS

3 4
1: m ∥ =− and m ⊥ =
4 3

1
3: m ∥ = 4 and m ⊥ = − 4

5 8
5: m ∥ =− and m ⊥ =
8 5

1
7: m ∥ = 9 and m ⊥ = − 9

9: m ∥ = 0 and m ⊥ undefined

11: m ∥ = 1 and m ⊥ = −1

4 3
13: m ∥ =− and m ⊥ =
3 4

2 7
15: m ∥ = 7
and m ⊥ =− 2

3 2
17: m ∥ = and m ⊥ =−
2 3

1
19: m ∥ = 10 and m ⊥ = − 10

21: Parallel

23: Perpendicular

25: Perpendicular

27: Perpendicular

29: Parallel

31: Neither

33: Parallel

35: Parallel

3.6 Parallel and Perpendicular Lines 546


Chapter 3 Graphing Lines

37: y = 1
2
x−4

39: y =− 1
3
x+1

41: x = −1

43: y =4

45: y = −x − 6

47: y = 1
4
x+ 3
2

49: y = 2
3
x−6

51: y =− 3
5
x+ 23
5

53: y = 3
5
x + 15

55: y =− 2
3
x− 11
3

57: y =− 1
4
x− 9
4

59: y = 1
2
x−6

61: y =− 5
2
x+ 1
4

63: x = −6

3.6 Parallel and Perpendicular Lines 547


Chapter 3 Graphing Lines

3.7 Introduction to Functions

LEARNING OBJECTIVES

1. Identify a function.
2. State the domain and range of a function.
3. Use function notation.

Relations, Functions, Domain, and Range

Relationships between sets occur often in everyday life. For example, for each
month in Cape Canaveral, we can associate an average amount of rainfall. In this
case, the amount of precipitation depends on the month of the year, and the data
can be written in tabular form or as a set of ordered pairs.

Month Precipitation Ordered pairs

January 2.4 in (January, 2.4)

February 3.3 in (February, 3.3)

March 3.1 in (March, 3.1)

April 2.0 in (April, 2.0)

May 3.8 in (May, 3.8)

June 6.8 in (June, 6.8)

July 8.1 in (July, 8.1)

August 7.6 in (August, 7.6)

September 7.3 in (September, 7.3)

October 4.1 in (October, 4.1)

November 3.3 in (November, 3.3)

December 2.4 in (December, 2.4)

31. Any set of ordered pairs.


We define a relation31 as any set of ordered pairs. Usually we write the independent
32. The set of first components of a component of the relation in the first column and the dependent component in the
relation. The x-values define
second column. In the opening example, notice that it makes sense to relate the
the domain in relations
consisting of points (x, y) in the average amount of precipitation as dependent on the month of year. The set of all
rectangular coordinate plane. elements in the first column of a relation is called the domain32. The set of all

548
Chapter 3 Graphing Lines

elements that compose the second column is called the range33. In this example,
the domain consists of the set of all months of the year, and the range consists of
the values that represent the average rainfall for each month.

In the context of algebra, the relations of interest are sets of ordered pairs (x, y) in
the rectangular coordinate plane. In this case, the x-values define the domain and
the y-values define the range. Of special interest are relations where every x-value
corresponds to exactly one y-value; these relations are called functions34.

Example 1: Determine the domain and range of the following relation and state
whether or not it is a function: {(−1, 4), (0, 7), (2, 3), (3, 3), (4, −2)}.

Solution: Here we separate the domain and range and depict the correspondence
between the values with arrows.

Answer: The domain is {−1, 0, 2, 3, 4}, and the range is {−2, 3, 4, 7}. The relation is a
function because each x-value corresponds to exactly one y-value.

33. The set of second components


of a relation. The y-values
define the range in relations
consisting of points (x, y) in the Example 2: Determine the domain and range of the following relation and state
rectangular coordinate plane.
whether or not it is a function: {(−4, −3), (−2, 6), (0, 3), (3, 5), (3, 7)}.
34. Relations where every x-value
corresponds to exactly one y-
value. With the definition Solution:
comes new notation:
f (x) = y , which is read “f of
x is equal to y.”

3.7 Introduction to Functions 549


Chapter 3 Graphing Lines

Answer: The domain is {−4, −2, 0, 3}, and the range is {−3, 3, 5, 6, 7}. This relation is
not a function because the x-value 3 has two corresponding y-values.

In the previous example, the relation is not a function because it contains ordered
pairs with the same x-value, (3, 5) and (3, 7). We can recognize functions as relations
where no x-values are repeated.

In algebra, equations such as y = 3


4
x − 2 define relations. This linear equation can
be graphed as follows:

3.7 Introduction to Functions 550


Chapter 3 Graphing Lines

The graph is a relation since it represents the infinite set of ordered pair solutions
to y = 34 x − 2. The domain is the set of all x-values, and in this case consists of all
real numbers. The range is the set of all possible y-values, and in this case also
consists of all real numbers. Furthermore, the graph is a function because for each
x-value there is only one corresponding y-value. In fact, any nonvertical or
nonhorizontal line is a function with domain and range consisting of all real
numbers.

Any graph is a set of ordered pairs and thus defines a relation. Consider the
following graph of a circle:

Here the graph represents a relation where many x-values in the domain
correspond to two y-values. If we draw a vertical line, as illustrated, we can see that
(3, 2) and (3, −2) are two ordered pairs with the same x-value. Therefore, the x-value
3 corresponds to two y-values; hence the graph does not represent a function. The
illustration suggests that if any vertical line intersects a graph more than once,
then the graph does not represent a function. This is called the vertical line test35.

35. If a vertical line intersects a Example 3: Given the following graph, determine the domain and range and state
graph more than once, then whether or not it is a function.
the graph does not represent a
function.

3.7 Introduction to Functions 551


Chapter 3 Graphing Lines

Solution: The given shape is called a parabola and extends indefinitely to the left
and right as indicated by the arrows. This suggests that if we choose any x-value,
then we will be able to find a corresponding point on the graph; therefore, the
domain consists of all real numbers. Furthermore, the graph shows that −1 is the
minimum y-value, and any y-value greater than that is represented in the relation.
Hence the range consists of all y-values greater than or equal to −1, or in interval
notation, [−1, ∞) .

3.7 Introduction to Functions 552


Chapter 3 Graphing Lines

Lastly, any vertical line will intersect the graph only once; therefore, it is a function.

Answer: The domain is all real numbers R = (−∞, ∞), and the range is [−1, ∞) . The
graph represents a function because it passes the vertical line test.

Try this! Given the graph, determine the domain and range and state whether or
not it is a function:

Answer: Domain: [−4, ∞) ; range: (−∞, ∞) ; function: no

Video Solution

(click to see video)


Function Notation and Linear Functions

With the definition of a function comes special notation. If we consider each x-value
to be the input that produces exactly one output, then we can use the notation

3.7 Introduction to Functions 553


Chapter 3 Graphing Lines

The notation f (x) reads “f of x” and should not be confused with multiplication.
Most of our study of algebra involves functions, so the notation becomes very useful
when performing common tasks. Functions can be named with different letters;
some common names for functions are g(x), h(x), C(x), and R(x). First, consider
nonvertical lines that we know can be expressed using slope-intercept form,
y = mx + b . For any real numbers m and b, the equation defines a function, and we
can replace y with the new notation f (x) as follows:

Therefore, a linear function36 is any function that can be written in the form
f (x) = mx + b . In particular, we can write the following:

The notation also shows values to evaluate in the equation. If the value for x is given
as 8, then we know that we can find the corresponding y-value by substituting 8 in
for x and simplifying. Using function notation, this is denoted f (8) and can be
interpreted as follows:

Finally, simplify:

36. Any function that can be


written in the form
f(x) = mx + b.

3.7 Introduction to Functions 554


Chapter 3 Graphing Lines

We have f (8) = 4. This notation tells us that when x = 8 (the input), the function
results in 4 (the output).

Example 4: Given the linear function f (x) = −5x + 7, find f (−2).

Solution: In this case, f (−2) indicates that we should evaluate when x = −2 .

Answer: f (−2) = 17

Example 5: Given the linear function f (x) = −5x + 7, find x when f (x) = 10.

Solution: In this case, f (x) = 10 indicates that the function should be set equal to
10.

3.7 Introduction to Functions 555


Chapter 3 Graphing Lines

Answer: Here x = − 35 , and we can write f (− 35 ) = 10.

Example 6: Given the graph of a linear function g(x),

a. Find g(2).

b. Find x when g(x) = 3.

3.7 Introduction to Functions 556


Chapter 3 Graphing Lines

Solution:

a. The notation g(2) implies that x = 2. Use the graph to determine the
corresponding y-value.

Answer: g(2) = 1

b. The notation g(x) = 3 implies that the y-value is given as 3. Use the graph to
determine the corresponding x-value.

3.7 Introduction to Functions 557


Chapter 3 Graphing Lines

Answer: x = 4

Example 7: Graph the linear function f (x) = − 53 x + 6 and state the domain and
range.

Solution: From the function, we see that b = 6 and thus the y-intercept is (0, 6). Also,
we can see that the slope is m = − 53 = −53 = rise. Starting from the y-intercept,
run
mark a second point down 5 units and right 3 units.

3.7 Introduction to Functions 558


Chapter 3 Graphing Lines

Given any coordinate on the x-axis, we can find a corresponding point on the graph;
the domain consists of all real numbers. Also, for any coordinate on the y-axis, we
can find a point on the graph; the range consists of all real numbers.

Answer: Both the domain and range consist of all real numbers R.

Try this! Given the linear function g(x) = −x + 5,

a. Find g (− 12 ).

b. Find x when g(x) = 18.

Answers:

a. g (− 12 ) = 11
2

b. x = −13

3.7 Introduction to Functions 559


Chapter 3 Graphing Lines

Video Solution

(click to see video)

KEY TAKEAWAYS

• A relation is any set of ordered pairs. However, in the context of this


course, we will be working with sets of ordered pairs (x, y) in the
rectangular coordinate system. The set of x-values defines the domain
and the set of y-values defines the range.
• Special relations where every x-value (input) corresponds to exactly one
y-value (output) are called functions.
• We can easily determine whether an equation represents a function by
performing the vertical line test on its graph. If any vertical line
intersects the graph more than once, then the graph does not represent
a function. In this case, there will be more than one point with the same
x-value.

• Any nonvertical or nonhorizontal line is a function and can be


written using function notation f (x) = mx + b . Both the
domain and range consist of all real numbers.

◦ If asked to find f (a), we substitute a in for the variable and


then simplify.
◦ If asked to find x when f (x) = a, we set the function equal
to a and then solve for x .

3.7 Introduction to Functions 560


Chapter 3 Graphing Lines

TOPIC EXERCISES

Part A: Functions

For each problem below, does the correspondence represent a function?

1. Algebra students to their scores on the first exam.

2. Family members to their ages.

3. Lab computers to their users.

4. Students to the schools they have attended.

5. People to their citizenships.

6. Local businesses to their number of employees.

Determine the domain and range and state whether the relation is a function or not.

7. {(3, 2), (5, 3), (7, 4)}

8. {(−5, −3), (0, 0), (5, 0)}

9. {(−10, 2), (−8, 1), (−8, 0)}

10. {(9, 12), (6, 6), (6, 3)}

11.

12.

3.7 Introduction to Functions 561


Chapter 3 Graphing Lines

13.

14.

15.

16.

3.7 Introduction to Functions 562


Chapter 3 Graphing Lines

17.

18.

19.

20.

3.7 Introduction to Functions 563


Chapter 3 Graphing Lines

21.

22.

23.

24.

3.7 Introduction to Functions 564


Chapter 3 Graphing Lines

25.

26.

Part B: Function Notation

Given the following functions, find the function values.

27. f (x) = 3x , find f (−2) .

28. f (x) = −5x + 1 , find f (−1) .

29. f (x) = 3
5
x − 4 , find f (15) .

30. f (x) = 2
5
x− 1
5
, find f (3) .

31. f (x) = 5
2
x− 1
3
, find f (− 1
3
).

3.7 Introduction to Functions 565


Chapter 3 Graphing Lines

32. f (x) = −6 , find f (7) .

33. g(x) = 5 , find g(−4) .

34. g(x) = −5x , find g(−3) .

35. g(x) =− 1
8
x+ 5
8
, find g ( 58 ) .

36. g(x) = 5
3
x − 5 , find g (3) .

37. f (x) = 5x − 9 , find x when f (x) = 1 .

38. f (x) = −7x + 2 , find x when f (x) = 0 .

39. f (x) =− 7
5
x − 2 , find x when f (x) = −9 .

40. f (x) = −x − 4 , find x when f (x) = 1


2
.

41. g(x) = x , find x when g(x) = 12 .

42. g(x) = −x + 1 , find x when g(x) = 2


3
.

43. g(x) = −5x + 1


3
, find x when g(x) =− 1
2
.

44. g(x) =− 5
8
x + 3 , find x when g(x) = 3 .

Given f (x) = 2
3
x − 1 and g(x) = −3x + 2 calculate the following.

45. f (6)

46. f (−12)

47. f (0)

48. f (1)

49. g ( 23 )

50. g (0)

3.7 Introduction to Functions 566


Chapter 3 Graphing Lines

51. g (−1)

52. g (− 1
2 )

53. Find x when f (x) = 0.

54. Find x when f (x) = −3 .

55. Find x when g(x) = −1 .

56. Find x when g(x) = 0.

Given the graph, find the function values.

57. Given the graph of f (x) , find f (−4) , f (−1) , f (0) , and f (2) .

58. Given the graph of g(x) , find g(−3) , g(−1) , g(0) , and g(1) .

59. Given the graph of f (x) , find f (−4) , f (−1) , f (0) , and f (2) .

3.7 Introduction to Functions 567


Chapter 3 Graphing Lines

60. Given the graph of g(x) , find g(−4) , g(−1) , g(0) , and g(2) .

61. Given the graph of f (x) , find f (−1) , f (0) , f (1) , and f (3) .

62. Given the graph of g(x) , find g(−2) , g(0) , g(2) , and g(6) .

63. Given the graph of g(x) , find g(−4) , g(−3) , g(0) , and g(4) .

3.7 Introduction to Functions 568


Chapter 3 Graphing Lines

64. Given the graph of f (x) , find f (−4) , f (0) , f (1) , and f (3) .

Given the graph, find the x-values.

65. Given the graph of f (x) , find x when f (x) = 3 , f (x) = 1 , and
f (x) = −3 .

66. Given the graph of g(x) , find x when g(x) = −1 , g(x) = 0 , and
g(x) = 1 .

67. Given the graph of f (x) , find x when f (x) = 3.

3.7 Introduction to Functions 569


Chapter 3 Graphing Lines

68. Given the graph of g(x) , find x when g(x) = −2 , g(x) = 0 , and
g(x) = 4 .

69. Given the graph of f (x) , find x when f (x) = −16 , f (x) = −12 , and
f (x) = 0 .

70. Given the graph of g(x) , find x when g(x) = −3 , g(x) = 0 , and
g(x) = 1 .

71. Given the graph of f (x) , find x when f (x) = −4 , f (x) = 0 , and
f (x) = −2 .

3.7 Introduction to Functions 570


Chapter 3 Graphing Lines

72. Given the graph of g(x) , find x when g(x) = 5 , g(x) = 3 , and
g(x) = 2 .

73. The cost in dollars of producing pens with a company logo is given by the
function C(x) = 1.65x + 120, where x is the number of pens
produced. Use the function to calculate the cost of producing 200 pens.

74. The revenue in dollars from selling sweat shirts is given by the function
R(x) = 29.95x , where x is the number of sweat shirts sold. Use the
function to determine the revenue if 20 sweat shirts are sold.

75. The value of a new car in dollars is given by the function


V(t) = −2,500t + 18,000 , where t represents the age of the car in
years. Use the function to determine the value of the car when it is 5 years
old. What was the value of the car when new?

76. The monthly income in dollars of a commissioned car salesman is given


by the function I(n) = 550n + 1,250 , where n represents the number
of cars sold in the month. Use the function to determine the salesman’s
monthly income if he sells 3 cars this month. What is his income if he does
not sell any cars in a month?

77. The perimeter of an isosceles triangle with a base measuring 10


centimeters is given by the function P(x) = 2x + 10 , where x represents
the length of each of the equal sides. Find the length of each side if the
perimeter is 40 centimeters.

3.7 Introduction to Functions 571


Chapter 3 Graphing Lines

78. The perimeter of a square depends on the length of each side s and is
modeled by the function P(s) = 4s. If the perimeter of a square measures
140 meters, then use the function to calculate the length of each side.

79. A certain cellular phone plan charges $18 per month and $0.10 per
minute of usage. The cost of the plan is modeled by the function
C(x) = 0.10x + 18 , where x represents the number of minutes of usage
per month. Determine the minutes of usage if the cost for the month was
$36.

80. The monthly revenue generated by selling subscriptions to a tutoring


website is given by the function R(x) = 29x , where x represents the
number of subscription sales per month. How many subscriptions were sold
if the revenues for the month totaled $1,508?

Graph the linear function and state the domain and range.

81. f (x) =− 5
2
x + 10

82. f (x) = 3
5
x − 10

83. g(x) = 6x + 2

84. g(x) = −4x + 6

85. h(t) = 1
2
t−3

86. h(t) =− 3
4
t+3

87. C(x) = 100 + 50x

88. C(x) = 50 + 100x

Part C: Discussion Board Topics

89. Is a vertical line a function? What are the domain and range of a vertical
line?

90. Is a horizontal line a function? What are the domain and range of a
horizontal line?

3.7 Introduction to Functions 572


Chapter 3 Graphing Lines

91. Come up with your own correspondence between real-world sets.


Explain why it does or does not represent a function.

92. Can a function have more than one y-intercept? Explain.

3.7 Introduction to Functions 573


Chapter 3 Graphing Lines

ANSWERS

1: Yes

3: No

5: No

7: Domain: {3, 5, 7}; range: {2, 3, 4}; function: yes

9: Domain: {−10,−8}; range: {0, 1, 2}; function: no

11: Domain: {−4, −1, 2}; range: {1, 2, 3}; function: yes

13: Domain: {−2, 2}; range: {2, 3, 5}; function: no

15: Domain: (−∞, ∞) ; range: {2}; function: yes

17: Domain: (−∞, ∞) ; range: (−∞, ∞) ; function: yes

19: Domain: [−2, ∞) ; range: (−∞, ∞) ; function: no

21: Domain: [−4, ∞) ; range: [0, ∞) ; function: yes

23: Domain: (−∞, ∞) ; range: [0, ∞) ; function: yes

25: Domain: (−∞, ∞) ; range: [2, ∞) ; function: yes

27: f (−2) = −6

29: f (15) =5

31: f (− 1
3
)=− 7
6

33: g(−4) =5

35: g ( 58 ) = 35
64

3.7 Introduction to Functions 574


Chapter 3 Graphing Lines

37: x =2

39: x =5

41: x = 12

43: x = 1
6

45: f (6) =3

47: f (0) = −1

49: g ( 23 ) =0

51: g (−1) =5

53: x = 3
2

55: x =1

57: f (−4) = −3 , f (−1) = 0 , f (0) = 1, and f (2) = 3

59: f (−4) = −4 , f (−1) = −4 , f (0) = −4 , and f (2) = −4

61: f (−1) = 1 , f (0) = −2 , f (1) = −3 , and f (3) = 1

63: g(−4) = 0 , g(−3) = 1 , g(0) = 2 , and g(4) = 3

65: f (−1) = 3 , f (0) = 1, and f (2) = −3

67: f (1) = 3 (answers may vary)

69: f (−4) = −16 ; f (−6) = −12 and f (−2) = −12 ; f (−8) = 0


and f (0) = 0

71: f (−4)
= −4 and f (4) = −4 ; f (0) = 0; f (−2) = −2 and
f (2) = −2

73: $450

3.7 Introduction to Functions 575


Chapter 3 Graphing Lines

75: New: $18,000; 5 years old: $5,500

77: 15 centimeters

79: 180 minutes

81: Domain and range: R

83: Domain and range: R

85: Domain and range R

87: Domain and range: R

3.7 Introduction to Functions 576


Chapter 3 Graphing Lines

3.7 Introduction to Functions 577


Chapter 3 Graphing Lines

3.8 Linear Inequalities (Two Variables)

LEARNING OBJECTIVES

1. Identify and check solutions to linear inequalities with two variables.


2. Graph solution sets of linear inequalities with two variables.

Solutions to Linear Inequalities

We know that a linear equation with two variables has infinitely many ordered pair
solutions that form a line when graphed. A linear inequality with two variables37,
on the other hand, has a solution set consisting of a region that defines half of the
plane.

For the inequality, the line defines one boundary of the region that is shaded. This
indicates that any ordered pair that is in the shaded region, including the boundary
line, will satisfy the inequality. To see that this is the case, choose a few test
37. An inequality relating linear points38 and substitute them into the inequality.
expressions with two variables.
The solution set is a region
defining half of the plane.

38. A point not on the boundary of


the linear inequality used as a
means to determine in which
half-plane the solutions lie.

578
Chapter 3 Graphing Lines

Also, we can see that ordered pairs outside the shaded region do not solve the linear
inequality.

The graph of the solution set to a linear inequality is always a region. However, the
boundary may not always be included in that set. In the previous example, the line
was part of the solution set because of the “or equal to” part of the inclusive
inequality ≤. If we have a strict inequality <, we would then use a dashed line to
indicate that those points are not included in the solution set.

3.8 Linear Inequalities (Two Variables) 579


Chapter 3 Graphing Lines

Consider the point (0, 3) on the boundary; this ordered pair satisfies the linear
equation. It is the “or equal to” part of the inclusive inequality that makes it part of
the solution set.

So far, we have seen examples of inequalities that were “less than.” Now consider
the following graphs with the same boundary:

3.8 Linear Inequalities (Two Variables) 580


Chapter 3 Graphing Lines

Given the graphs above, what might we expect if we use the origin (0, 0) as a test
point?

Try this! Which of the ordered pairs (−2, −1), (0, 0), (−2, 8), (2, 1), and (4, 2) solve the
inequality y > − 12 x + 2?

Answer: (−2, 8) and (4, 2)

Video Solution

(click to see video)

3.8 Linear Inequalities (Two Variables) 581


Chapter 3 Graphing Lines

Graphing Solutions to Linear Inequalities

Solutions to linear inequalities are a shaded half-plane, bounded by a solid line or a


dashed line. This boundary is either included in the solution or not, depending on
the given inequality. If we are given a strict inequality, we use a dashed line to
indicate that the boundary is not included. If we are given an inclusive inequality,
we use a solid line to indicate that it is included. The steps for graphing the solution
set for an inequality with two variables are outlined in the following example.

Example 1: Graph the solution set: y > −3x + 1 .

Solution:

Step 1: Graph the boundary line. In this case, graph a dashed line y = −3x + 1
because of the strict inequality. By inspection, we see that the slope is
m = −3 = −3 1
= rise
runand the y-intercept is (0, 1).

Step 2: Test a point not on the boundary. A common test point is the origin (0, 0).
The test point helps us determine which half of the plane to shade.

3.8 Linear Inequalities (Two Variables) 582


Chapter 3 Graphing Lines

Step 3: Shade the region containing the solutions. Since the test point (0, 0) was not
a solution, it does not lie in the region containing all the ordered pair solutions.
Therefore, shade the half of the plane that does not contain this test point. In this
case, shade above the boundary line.

Answer:

Consider the problem of shading above or below the boundary line when the
inequality is in slope-intercept form. If y > mx + b , then shade above the line. If
y < mx + b , then shade below the line. Use this with caution; sometimes the
boundary is given in standard form, in which case these rules do not apply.

Example 2: Graph the solution set: 2x − 5y ≥ −10 .

3.8 Linear Inequalities (Two Variables) 583


Chapter 3 Graphing Lines

Solution: Here the boundary is defined by the line 2x − 5y = −10 . Since the
inequality is inclusive, we graph the boundary using a solid line. In this case, graph
the boundary line using intercepts.

Next, test a point; this helps decide which region to shade.

3.8 Linear Inequalities (Two Variables) 584


Chapter 3 Graphing Lines

Since the test point is in the solution set, shade the half of the plane that contains it.

Answer:

In this example, notice that the solution set consists of all the ordered pairs below
the boundary line. This may be counterintuitive because of the original ≥ in the
inequality. This illustrates that it is a best practice to actually test a point. Solve for
y and you see that the shading is correct.

In slope-intercept form, you can see that the region below the boundary line should
be shaded. An alternate approach is to first express the boundary in slope-intercept
form, graph it, and then shade the appropriate region.

3.8 Linear Inequalities (Two Variables) 585


Chapter 3 Graphing Lines

Example 3: Graph the solution set: y < 2 .

Solution: First, graph the boundary line y = 2 with a dashed line because of the
strict inequality.

Next, test a point.

In this case, shade the region that contains the test point.

Answer:

3.8 Linear Inequalities (Two Variables) 586


Chapter 3 Graphing Lines

Try this! Graph the solution set: 5x − y ≤ 10 .

Answer:

3.8 Linear Inequalities (Two Variables) 587


Chapter 3 Graphing Lines

Video Solution

(click to see video)

KEY TAKEAWAYS

• Linear inequalities with two variables have infinitely many ordered pair
solutions, which can be graphed by shading in the appropriate half of a
rectangular coordinate plane.
• To graph the solution set of a linear inequality with two variables, first
graph the boundary with a dashed or solid line depending on the
inequality. If given a strict inequality, use a dashed line for the
boundary. If given an inclusive inequality, use a solid line. Next, choose
a test point not on the boundary. If the test point solves the inequality,
then shade the region that contains it; otherwise, shade the opposite
side.
• When graphing the solution sets of linear inequalities, it is a good
practice to test values in and out of the solution set as a check.

3.8 Linear Inequalities (Two Variables) 588


Chapter 3 Graphing Lines

TOPIC EXERCISES

Part A: Solutions to Linear Inequalities (Two Variables)

Is the ordered pair a solution to the given inequality?

1. y < 5x + 1 ; (0, 0)
1
2. y >− x − 4 ; (0, −2)
2

2
3. y ≤ x + 1 ; (6, 5)
3

1
4. y ≥− x − 5 ; (−3, −8)
3

3 ( 3
5. y < 1
5
x− 1
; −
1
, − 1)

6. 4x − 3y ≤ 2 ; (−2, −1)

7. −x + 4y > 7 ; (0, 0)

8. 7x − 3y < 21 ; (5, −3)

9. y > −5 ; (−3, −1)

10. x ≤ 0 ; (0, 7)

Part B: Graphing Solutions to Linear Inequalities

Graph the solution set.

11. y < −3x + 3


2
12. y <− x+4
3

1
13. y ≥− x
2

4
14. y ≥ x−8
5

3.8 Linear Inequalities (Two Variables) 589


Chapter 3 Graphing Lines

15. y ≤ 8x − 7

16. y > −5x + 3

17. y > −x + 4

18. y >x−2

19. y ≥ −1

20. y < −3

21. x <2

22. x ≥2
3 1
23. y ≤ x−
4 2

3 5
24. y >− x+
2 2

25. −2x + 3y > 6

26. 7x − 2y > 14

27. 5x − y < 10

28. x −y<0

29. 3x − 2y ≥ 0

30. x − 5y ≤ 0

31. −x + 2y ≤ −4

32. −x + 2y ≤ 3

33. 2x − 3y ≥ −1

34. 5x − 4y < −3

3.8 Linear Inequalities (Two Variables) 590


Chapter 3 Graphing Lines

1 1
35.
2
x− 3
y<1

1 1 1
36.
2
x− 10
y≥ 2

37. x ≥ −2y

38. x < 2y + 3

39. 3x −y+2>0

40. 3 − y − 2x < 0

41. −4x ≤ 12 − 3y

42. 5x ≤ −4y − 12

43. Write an inequality that describes all points in the upper half-plane
above the x-axis.

44. Write an inequality that describes all points in the lower half-plane
below the x-axis.

45. Write an inequality that describes all points in the half-plane left of the
y-axis.

46. Write an inequality that describes all points in the half-plane right of the
y-axis.

47. Write an inequality that describes all ordered pairs whose y-coordinates
are at least 2.

48. Write an inequality that describes all ordered pairs whose x-coordinate is
at most 5.

3.8 Linear Inequalities (Two Variables) 591


Chapter 3 Graphing Lines

ANSWERS

1: Yes

3: Yes

5: Yes

7: No

9: Yes

11:

13:

15:

17:

3.8 Linear Inequalities (Two Variables) 592


Chapter 3 Graphing Lines

19:

21:

23:

25:

3.8 Linear Inequalities (Two Variables) 593


Chapter 3 Graphing Lines

27:

29:

31:

33:

3.8 Linear Inequalities (Two Variables) 594


Chapter 3 Graphing Lines

35:

37:

39:

41:

3.8 Linear Inequalities (Two Variables) 595


Chapter 3 Graphing Lines

43: y >0

45: x <0

47: y ≥2

3.8 Linear Inequalities (Two Variables) 596


Chapter 3 Graphing Lines

3.9 Review Exercises and Sample Exam

597
Chapter 3 Graphing Lines

REVIEW EXERCISES

Rectangular Coordinate System

Graph the given set of ordered pairs.

1. {(−3, 4) , (−4, 0) , (0, 3) , (2, 4)}

2. {(−5, 5) , (−3, − 1) , (0, 0) , (3, 2)}

3. Graph the points (−3, 5), (−3, −3), and (3, −3) on a rectangular coordinate
plane. Connect the points and calculate the area of the shape.

4. Graph the points (−4, 1), (0, 1), (0, −2), and (−4, −2) on a rectangular
coordinate plane. Connect the points and calculate the area of the shape.

5. Graph the points (1, 0), (4, 0), (1, −5), and (4, −5) on a rectangular
coordinate plane. Connect the points and calculate the perimeter of the
shape.

6. Graph the points (−5, 2), (−5, −3), (1, 2), and (1, −3) on a rectangular
coordinate plane. Connect the points and calculate the perimeter of the
shape.

Calculate the distance between the given two points.

7. (−1, −2) and (5, 6)

8. (2, −5) and (−2, −2)

9. (−9, −3) and (−8, 4)

10. (−1, 3) and (1, −3)

Calculate the midpoint between the given points.

11. (−1, 3) and (5, −7)

12. (6, −3) and (−8, −11)

3.9 Review Exercises and Sample Exam 598


Chapter 3 Graphing Lines

13. (7, −2) and (−6, −1)

14. (−6, 0) and (0, 0)

15. Show algebraically that the points (−1, −1), (1, −3), and (2, 0) form an
isosceles triangle.

16. Show algebraically that the points (2, −1), (6, 1), and (5, 3) form a right
triangle.

Graph by Plotting Points

Determine whether the given point is a solution.

17. −5x + 2y = 7 ; (1, − 1)

18. 6x − 5y = 4 ; (−1, − 2)

19. y = 3
4
x + 1 ; (− 2
3
, 1
2 )

20. y =− 3
5
x − 2 ; (10, − 8)

Find at least five ordered pair solutions and graph.

21. y = −x + 2

22. y = 2x − 3
1
23. y = x−2
2

2
24. y =− x
3

25. y =3

26. x = −3

27. x − 5y = 15

3.9 Review Exercises and Sample Exam 599


Chapter 3 Graphing Lines

28. 2x − 3y = 12

Graph Using Intercepts

Given the graph, find the x- and y- intercepts.

29.

30.

31.

32.

3.9 Review Exercises and Sample Exam 600


Chapter 3 Graphing Lines

Find the intercepts and graph them.

33. 3x − 4y = 12

34. 2x − y = −4
1 1
35.
2
x− 3
y=1

1 2
36. − x+ y=2
2 3

5
37. y =− x+5
3

38. y = −3x + 4

Graph Using the y-Intercept and Slope

Given the graph, determine the slope and y-intercept.

39.

40.

3.9 Review Exercises and Sample Exam 601


Chapter 3 Graphing Lines

Determine the slope, given two points.

41. (−3, 8) and (5, −6)

42. (0, −5) and (−6, 3)

43. (1/2, −2/3) and (1/4, −1/3)

44. (5, −3/4) and (2, −3/4)

Express in slope-intercept form and identify the slope and y-intercept.

45. 12x − 4y = 8

46. 3x − 6y = 24
1 3
47. − x+ y=1
3 4

48. −5x + 3y = 0

Graph using the slope and y-intercept.

49. y = −x + 3

50. y = 4x − 1

51. y = −2x
5
52. y =− x+3
2

53. 2x − 3y = 9

3.9 Review Exercises and Sample Exam 602


Chapter 3 Graphing Lines

3
54. 2x + y=3
2

55. y =0

56. x − 4y = 0

Finding Linear Equations

Given the graph, determine the equation of the line.

57.

58.

59.

60.

3.9 Review Exercises and Sample Exam 603


Chapter 3 Graphing Lines

Find the equation of a line, given the slope and a point on the line.

61. m = 1/2; (−4, 8)

62. m = −1/5; (−5, −9)

63. m = 2/3; (1, −2)

64. m = −3/4; (2, −3)

Find the equation of the line given two points on the line.

65. (−5, −5) and (10, 7)

66. (−6, 12) and (3, −3)

67. (2, −1) and (−2, 2)

68. (5/2, −2) and (−5, 5/2)

69. (7, −6) and (3, −6)

70. (10, 1) and (10, −3)

Parallel and Perpendicular Lines

Determine if the lines are parallel, perpendicular, or neither.

{ 6x − 14y = 42
−3x + 7y = 14
71.

3.9 Review Exercises and Sample Exam 604


Chapter 3 Graphing Lines

{ 2x − 3y = 36
2x + 3y = 18
72.

{ 8x − 2y = −1
x + 4y = 2
73.

{x = 2
y=2
74.

Find the equation of the line in slope-intercept form.

75. Parallel to 5x − y = 15 and passing through (−10, −1).

76. Parallel to x − 3y = 1 and passing through (2, −2).

77. Perpendicular to 8x − 6y = 4 and passing through (8, −1).

78. Perpendicular to 7x + y = 14 and passing through (5, 1).

79. Parallel to y = 1 and passing through (4, −1).

80. Perpendicular to y = 1 and passing through (4, −1).

Introduction to Functions

Determine the domain and range and state whether it is a function or not.

81. {(−10, −1), (−5, 2), (5, 2)}

82. {(−12, 4), (−1, −3), (−1, −2)}

83.

3.9 Review Exercises and Sample Exam 605


Chapter 3 Graphing Lines

84.

85.

86.

Given the following,

87. f (x) = 9x − 4 , find f (−1) .

88. f (x) = −5x + 1 , find f (−3) .

3.9 Review Exercises and Sample Exam 606


Chapter 3 Graphing Lines

, find g (− ).
1 1
89. g(x) = x− 1
2 3 3

, find g ( ).
3 1 2
90. g(x) =− x+
4 3 3

91. f (x) = 9x − 4 , find x when f (x) = 0 .

92. f (x) = −5x + 1 , find x when f (x) = 2 .


1 1
93. g(x) = x− , find x when g(x) = 1.
2 3

3 1
94. g(x) =− x+ , find x when g(x) = −1 .
4 3

Given the graph of a function f (x) , determine the following.

95. f (3)

96. x when f (x) =4

Linear Inequalities (Two Variables)

Is the ordered pair a solution to the given inequality?

97. 6x − 2y ≤ 1 ; (−3, −7)

98. −3x + y > 2 ; (0, 2)

99. 6x − 10y < −1 ; (5, −3)


1
100. x − y > 0 ; (1, 4)
3

101. y > 0 ; (−3, −1)

3.9 Review Exercises and Sample Exam 607


Chapter 3 Graphing Lines

102. x ≤ −5 ; (−6, 4)

Graph the solution set.

103. y ≥ −2x + 1

104. y < 3x − 4

105. −x +y≤3
5 1
106.
2
x+ 2
y≤2

107. 3x − 5y > 0

108. y >0

3.9 Review Exercises and Sample Exam 608


Chapter 3 Graphing Lines

SAMPLE EXAM

1. Graph the points (−4, −2), (−4, 1), and (0, −2) on a rectangular coordinate
plane. Connect the points and calculate the area of the shape.

2. Is (−2, 4) a solution to 3x − 4y = −10 ? Justify your answer.

Given the set of x-values {−2, −1, 0, 1, 2}, find the corresponding y-values and graph
the following.

3. y =x−1

4. y = −x + 1

5. On the same set of axes, graph y = 4 and x = −3 . Give the point where
they intersect.

Find the x- and y-intercepts and use those points to graph the following.

6. 2x −y=8

7. 12x + 5y = 15

8. Calculate the slope of the line passing through (−4, −5) and (−3, 1).

Determine the slope and y-intercept. Use them to graph the following.

3
9. y =− x+6
2

10. 5x − 2y = 6

11. Given m = −3 , determine m ⊥ .

12. Are the given lines parallel, perpendicular, or neither?

{ 4x − 6y = 30
−2x + 3y = −12

3.9 Review Exercises and Sample Exam 609


Chapter 3 Graphing Lines

13. Determine the slope of the given lines.

a. y = −2

b. x = 1
3

c. Are these lines parallel, perpendicular, or neither?

15. Determine the equation of the line with slope m =− 3


4
passing
through (8, 1).

16. Find the equation to the line passing through (−2, 3) and (4, 1).

17. Find the equation of the line parallel to 5x −y=6 passing through
(−1, −2).

18. Find the equation of the line perpendicular to −x + 2y = 4 passing


through (1/2, 5).

4
Given a linear function f (x) =− x + 2 , determine the following.
5

19. f (10)

20. x when f (x) =0

21. Graph the solution set: 3x − 4y > 4 .

22. Graph the solution set: y − 2x ≥ 0 .

23. A rental car company charges $32.00 plus $0.52 per mile driven. Write an
equation that gives the cost of renting the car in terms of the number of
miles driven. Use the formula to determine the cost of renting the car and
driving it 46 miles.

24. A car was purchased new for $12,000 and was sold 5 years later for
$7,000. Write a linear equation that gives the value of the car in terms of its
age in years.

25. The area of a rectangle is 72 square meters. If the width measures 4


meters, then determine the length of the rectangle.

3.9 Review Exercises and Sample Exam 610


Chapter 3 Graphing Lines

REVIEW EXERCISES ANSWERS

1:

3: Area: 24 square units

5: Perimeter: 16 units

7: 10 units

⎯⎯
9: 5√ 2 units

11: (2, −2)

13: (1/2, −3/2)

17: No

3.9 Review Exercises and Sample Exam 611


Chapter 3 Graphing Lines

19: Yes

21:

23:

25:

27:

29: y-intercept: (0, −2); x-intercept: (−4, 0)

3.9 Review Exercises and Sample Exam 612


Chapter 3 Graphing Lines

31: y-intercept: none; x-intercept: (5, 0)

33:

35:

37:

39: y-intercept: (0, 1); slope: −2

41: −7/4

43: −4/3

45: y = 3x − 2 ; slope: 3; y-intercept (0, −2)


4 4
47: y = x+ ; slope: 4/9; y-intercept (0, 4/3)
9 3

49:

3.9 Review Exercises and Sample Exam 613


Chapter 3 Graphing Lines

51:

53:

55:

57: y = −2x + 1

59: y = −5
1
61: y = x + 10
2

3.9 Review Exercises and Sample Exam 614


Chapter 3 Graphing Lines

2 8
63: y = x−
3 3

4
65: y = x−1
5

3 1
67: y =− x+
4 2

69: y = −6

71: Parallel

73: Perpendicular

75: y = 5x + 49
3
77: y =− x+5
4

79: y = −1

81: Domain: {−10, −5, 5}; range: {−1, 2}; function: yes

83: Domain: R; range: R; function: yes

85: Domain: [−3, ∞) ; range: R; function: no

87: f (−1) = −13

89: g (− 1
3 )=−
1
2

91: x = 4
9

93: x = 8
3

95: f (3) = −2

97: Yes

99: No

3.9 Review Exercises and Sample Exam 615


Chapter 3 Graphing Lines

101: No

103:

105:

107:

3.9 Review Exercises and Sample Exam 616


Chapter 3 Graphing Lines

SAMPLE EXAM ANSWERS

1: Area: 6 square units

3:

5: Intersection: (−3, 4)

7:

9: Slope: −3/2; y-intercept: (0, 6)

3.9 Review Exercises and Sample Exam 617


Chapter 3 Graphing Lines

11: m ⊥ = 1
3

13:a. 0; b. Undefined; c. Perpendicular

3
15: y =− x+7
4

17: y = 5x + 3

19: f (10) = −6

21:

23: cost = 0.52x + 32 ; $55.92

25: 18 meters

3.9 Review Exercises and Sample Exam 618


Chapter 4
Solving Linear Systems

619
Chapter 4 Solving Linear Systems

4.1 Solving Linear Systems by Graphing

LEARNING OBJECTIVES

1. Check solutions to systems of linear equations.


2. Solve linear systems using the graphing method.
3. Identify dependent and inconsistent systems.

Definition of a Linear System

Real-world applications are often modeled using more than one variable and more
than one equation. A system of equations1 consists of a set of two or more
equations with the same variables. In this section, we will study linear systems2
consisting of two linear equations each with two variables. For example,

A solution to a linear system3, or simultaneous solution4, to a linear system is an


ordered pair (x, y) that solves both of the equations. In this case, (3, 2) is the only
solution. To check that an ordered pair is a solution, substitute the corresponding x-
and y-values into each equation and then simplify to see if you obtain a true
statement for both equations.

1. A set of two or more equations


with the same variables.

2. In this section, we restrict our


study to systems of two linear
equations with two variables.

3. An ordered pair that satisfies


both equations and
corresponds to a point of
intersection.

4. Used when referring to a


solution of a system of
equations.

620
Chapter 4 Solving Linear Systems

{ −2x + 3y = 5
x−y=1
Example 1: Determine whether (1, 0) is a solution to the system .

Solution: Substitute the appropriate values into both equations.

Answer: Since (1, 0) does not satisfy both equations, it is not a solution.

{ −2x + 3y = 16
x − y = −6
Try this! Is (−2, 4) a solution to the system ?

Answer: Yes

Video Solution

(click to see video)


Solve by Graphing

Geometrically, a linear system consists of two lines, where a solution is a point of


intersection. To illustrate this, we will graph the following linear system with a
solution of (3, 2):

4.1 Solving Linear Systems by Graphing 621


Chapter 4 Solving Linear Systems

First, rewrite the equations in slope-intercept form so that we may easily graph
them.

Next, replace these forms of the original equations in the system to obtain what is
called an equivalent system5. Equivalent systems share the same solution set.

If we graph both of the lines on the same set of axes, then we can see that the point
of intersection is indeed (3, 2), the solution to the system.

5. A system consisting of
equivalent equations that
share the same solution set.

4.1 Solving Linear Systems by Graphing 622


Chapter 4 Solving Linear Systems

To summarize, linear systems described in this section consist of two linear


equations each with two variables. A solution is an ordered pair that corresponds to
a point where the two lines in the rectangular coordinate plane intersect.
Therefore, we can solve linear systems by graphing both lines on the same set of
axes and determining the point where they cross. When graphing the lines, take
care to choose a good scale and use a straightedge to draw the line through the
points; accuracy is very important here. The steps for solving linear systems using
the graphing method6 are outlined in the following example.

{ 2x + y = 1
x − y = −4
Example 2: Solve by graphing: .

Solution:

Step 1: Rewrite the linear equations in slope-intercept form.

6. A means of solving a system by


graphing the equations on the
same set of axes and
determining where they
intersect.

4.1 Solving Linear Systems by Graphing 623


Chapter 4 Solving Linear Systems

Step 2: Write the equivalent system and graph the lines on the same set of axes.

4.1 Solving Linear Systems by Graphing 624


Chapter 4 Solving Linear Systems

Step 3: Use the graph to estimate the point where the lines intersect and check to
see if it solves the original system. In the above graph, the point of intersection
appears to be (−1, 3).

Answer: (−1, 3)

{ −2x + 3y = −18
2x + y = 2
Example 3: Solve by graphing: .

Solution: We first solve each equation for y to obtain an equivalent system where
the lines are in slope-intercept form.

Graph the lines and determine the point of intersection.

4.1 Solving Linear Systems by Graphing 625


Chapter 4 Solving Linear Systems

Answer: (3, −4)

{ y = −3
3x + y = 6
Example 4: Solve by graphing: .

Solution:

4.1 Solving Linear Systems by Graphing 626


Chapter 4 Solving Linear Systems

Answer: (3, −3)

The graphing method for solving linear systems is not ideal when the solution
consists of coordinates that are not integers. There will be more accurate algebraic
methods in sections to come, but for now, the goal is to understand the geometry
involved when solving systems. It is important to remember that the solutions to a
system correspond to the point, or points, where the graphs of the equations
intersect.

4.1 Solving Linear Systems by Graphing 627


Chapter 4 Solving Linear Systems

{ 5x + 2y = −2
−x + y = 6
Try this! Solve by graphing: .

Answer: (−2, 4)

Video Solution

(click to see video)


Dependent and Inconsistent Systems

Systems with at least one solution are called consistent systems7. Up to this point,
all of the examples have been of consistent systems with exactly one ordered pair
solution. It turns out that this is not always the case. Sometimes systems consist of
two linear equations that are equivalent. If this is the case, the two lines are the
same and when graphed will coincide. Hence the solution set consists of all the
points on the line. This is a dependent system8. Given a consistent linear system
with two variables, there are two possible results:

The solutions to independent systems9 are ordered pairs (x, y). We need some way
to express the solution sets to dependent systems, since these systems have
infinitely many solutions, or points of intersection. Recall that any line can be
written in slope-intercept form, y = mx + b . Here, y depends on x. So we may
express all the ordered pair solutions (x, y) in the form (x, mx + b) , where x is
any real number.
7. A system with at least one
solution.

8. A system that consists of


equivalent equations with
infinitely many ordered pair

{ 4x − 6y = 18
solutions, denoted by −2x + 3y = −9
(x, mx + b). Example 5: Solve by graphing: .

9. A system of equations with one


ordered pair solution (x, y).

4.1 Solving Linear Systems by Graphing 628


Chapter 4 Solving Linear Systems

Solution: Determine slope-intercept form for each linear equation in the system.

In slope-intercept form, we can easily see that the system consists of two lines with
the same slope and same y-intercept. They are, in fact, the same line. And the
system is dependent.

4.1 Solving Linear Systems by Graphing 629


Chapter 4 Solving Linear Systems

Answer: (x, 2
3
x − 3)

In this example, it is important to notice that the two lines have the same slope and
same y-intercept. This tells us that the two equations are equivalent and that the
simultaneous solutions are all the points on the line y = 23 x − 3. This is a
dependent system, and the infinitely many solutions are expressed using the form
(x, mx + b) . Other resources may express this set using set notation, {(x, y) |
y = 23 x − 3}, which reads “the set of all ordered pairs (x, y) such that y equals two-
thirds x minus 3.”

Sometimes the lines do not cross and there is no point of intersection. Such systems
have no solution, Ø, and are called inconsistent systems10.

{ −4x + 10y = 10
−2x + 5y = −15
Example 6: Solve by graphing: .

Solution: Determine slope-intercept form for each linear equation.

10. A system with no simultaneous


solution.

4.1 Solving Linear Systems by Graphing 630


Chapter 4 Solving Linear Systems

In slope-intercept form, we can easily see that the system consists of two lines with
the same slope and different y-intercepts. Therefore, they are parallel and will
never intersect.

Answer: There is no simultaneous solution, Ø.

4.1 Solving Linear Systems by Graphing 631


Chapter 4 Solving Linear Systems

{ −2x − 2y = 2
x + y = −1
Try this! Solve by graphing: .

Answer: (x, − x − 1)

Video Solution

(click to see video)

KEY TAKEAWAYS

• In this section, we limit our study to systems of two linear equations


with two variables. Solutions to such systems, if they exist, consist of
ordered pairs that satisfy both equations. Geometrically, solutions are
the points where the graphs intersect.
• The graphing method for solving linear systems requires us to graph
both of the lines on the same set of axes as a means to determine where
they intersect.
• The graphing method is not the most accurate method for determining
solutions, particularly when the solutions have coordinates that are not
integers. It is a good practice to always check your solutions.
• Some linear systems have no simultaneous solution. These systems
consist of equations that represent parallel lines with different y-
intercepts and do not intersect in the plane. They are called inconsistent
systems and the solution set is the empty set, Ø.
• Some linear systems have infinitely many simultaneous solutions. These
systems consist of equations that are equivalent and represent the same
line. They are called dependent systems and their solutions are
expressed using the notation (x, mx + b) , where x is any real
number.

4.1 Solving Linear Systems by Graphing 632


Chapter 4 Solving Linear Systems

TOPIC EXERCISES

Part A: Solutions to Linear Systems

Determine whether the given ordered pair is a solution to the given system.

{ −2x − 2y = 2
x + y = −1
1. (3, −2);

{ −2x − 2y = 2
x + y = −1
2. (−5, 0);

{ 3x − y = −12
−x + y = −4
3. (−2, −6);

{ −5x − 3y = 11
3x + 2y = −8
4. (2, −7);

{ −13x + 2y = −6
5x − 5y = 15
5. (0, −3);

{ −2x − 4y = 0
x+y=− 4
6. (− 1
2
, 1
4
);

{ −4x − 8y = 5
−x − y = −1
7. ( 34 , 1
4
);

1
x + 12 y = 1
{
3
8. (−3, 4);
2
3
x − 32 y = −8

4.1 Solving Linear Systems by Graphing 633


Chapter 4 Solving Linear Systems

{ 5x − 10y = 5
y = −3
9. (−5, −3);

{ −7x + 4y = 8
x=4
10. (4, 2);

Given the graph, determine the simultaneous solution.

11.

12.

13.

14.

4.1 Solving Linear Systems by Graphing 634


Chapter 4 Solving Linear Systems

15.

16.

17.

18.

4.1 Solving Linear Systems by Graphing 635


Chapter 4 Solving Linear Systems

19.

20.

Part B: Solving Linear Systems

Solve by graphing.

{ y = −x + 1
y= 2
x+6
21.

3
y= x+2
{y = −
4
22.
1
4
x−2

4.1 Solving Linear Systems by Graphing 636


Chapter 4 Solving Linear Systems

{ y = −x + 2
y=x−4
23.

{ y = 4x − 5
y = −5x + 4
24.

y = 2 x + 1

25. 
 y= 3x

5

2
y=− x+6
{y =
5
26.
2
5
x + 10

{y = x + 1
y = −2
27.

{ x = −3
y=3
28.

y=0
{y =
29. 2
5
x−4

{ y = 3x
x=2
30.

y =

31. 
y =
3
x−6

5
3
5
x−3

1
y=− x+1
{y = −
2
32.
1
2
x+1

4.1 Solving Linear Systems by Graphing 637


Chapter 4 Solving Linear Systems

{ −6x + 3y = −6
2x + 3y = 18
33.

{ 2x + 8y = 8
−3x + 4y = 20
34.

{ 2x − 3y = 9
−2x + y = 1
35.

{ 5x + 4y = −4
x + 2y = −8
36.

{ 2x − 3y = 6
4x + 6y = 36
37.

{ 6x − 3y = −6
2x − 3y = 18
38.

{ −6x − 10y = −10


3x + 5y = 30
39.

{ 5x − 15y = −15
−x + 3y = 3
40.

{ −x + y = 0
x−y=0
41.

{y − x = 1
y=x
42.

4.1 Solving Linear Systems by Graphing 638


Chapter 4 Solving Linear Systems

{
3x + 2y = 0
43.
x=2

1 2
2x + y=
{ −3x +
3 3
44.
1
2
y = −2

1 1
x+ y=2
{− 1
10 5
45.
1
5
x+ 5
y = −1

1 1
x− y=1
{
3 2
46.
1 1
3
x+ 5
y=1

1 1
x+ y=0
{
9 6
47.
1 1 1
9
x+ 4
y= 2

 5 x− 1 y=5
 16
48. 
− 5 x + 1 y = 5
 16
2

2 2



49. 
−
1
x− 1
y = 92

6 2
1
18
x+ 1
6
y = − 32

1 1 1
x− y=−
{
2 4 2
50.
1 1
3
x− 2
y=3

{ x = −5
y=4
51.

{x = 2
y = −3
52.

4.1 Solving Linear Systems by Graphing 639


Chapter 4 Solving Linear Systems

{x = 0
y=0
53.

{ y=3
y = −2
54.

{ y = −5
y=5
55.

{y − 2 = 0
y=2
56.

{x = 1
x = −5
57.

{x = 0
y=x
58.

{ −x + y = −2
4x + 6y = 3
59.

{ 3x + 10y = −10
−2x + 20y = 20
60.

Set up a linear system of two equations and two variables and solve it using the
graphing method.

61. The sum of two numbers is 20. The larger number is 10 less than five
times the smaller.

62. The difference between two numbers is 12 and their sum is 4.

4.1 Solving Linear Systems by Graphing 640


Chapter 4 Solving Linear Systems

63. Where on the graph of 3x − 2y = 6 does the x-coordinate equal the y-


coordinate?

64. Where on the graph of −5x + 2y = 30 does the x-coordinate equal


the y-coordinate?

A regional bottled water company produces and sells bottled water. The following
graph depicts the supply and demand curves of bottled water in the region. The
horizontal axis represents the weekly tonnage of product produced, Q. The vertical
axis represents the price per bottle in dollars, P.

Use the graph to answer the following questions.

65. Determine the price at which the quantity demanded is equal to the
quantity supplied.

66. If production of bottled water slips to 20 tons, then what price does the
demand curve predict for a bottle of water?

67. If production of bottled water increases to 40 tons, then what price does
the demand curve predict for a bottle of water?

68. If the price of bottled water is set at $2.50 dollars per bottle, what
quantity does the demand curve predict?

Part C: Discussion Board Topics

69. Discuss the weaknesses of the graphing method for solving systems.

70. Explain why the solution set to a dependent linear system is denoted by
(x, mx + b).

4.1 Solving Linear Systems by Graphing 641


Chapter 4 Solving Linear Systems

ANSWERS

1: No

3: No

5: Yes

7: No

9: Yes

11: (5, 0)

13: (2, 1)

15: (0, 0)

17: (x, 2x − 2)

19: ∅

21: (−2, 3)

23: (3, −1)

25: (5, 3)

27: (−3, −2)

29: (10, 0)

31: ∅

33: (3, 4)

35: (−3, −5)

37: (6, 2)

4.1 Solving Linear Systems by Graphing 642


Chapter 4 Solving Linear Systems

39: ∅

41: (x, x)

43: (2, −3)

45: (10, 5)

47: (−9, 6)

49: (x, 1
3
x − 9)

51: (−5, 4)

53: (0, 0)

55: ∅

57: ∅

59: (3/2, −1/2)

61: The two numbers are 5 and 15.

63: (6, 6)

65: $1.25

67: $1.00

4.1 Solving Linear Systems by Graphing 643


Chapter 4 Solving Linear Systems

4.2 Solving Linear Systems by Substitution

LEARNING OBJECTIVE

1. Solve linear systems using the substitution method.

The Substitution Method

In this section, we will define a completely algebraic technique for solving systems.
The idea is to solve one equation for one of the variables and substitute the result
into the other equation. After performing this substitution step, we will be left with
a single equation with one variable, which can be solved using algebra. This is called
the substitution method11, and the steps are outlined in the following example.

{ 3x − 2y = −7
2x + y = 7
Example 1: Solve by substitution: .

Solution:

Step 1: Solve for either variable in either equation. If you choose the first equation,
you can isolate y in one step.

Step 2: Substitute the expression −2x + 7 for the y variable in the other equation.

11. A means of solving a linear


system by solving for one of
the variables and substituting
the result into the other
equation.

644
Chapter 4 Solving Linear Systems

This leaves you with an equivalent equation with one variable, which can be solved
using the techniques learned up to this point.

Step 3: Solve for the remaining variable. To solve for x, first distribute −2:

Step 4: Back substitute12 to find the value of the other coordinate. Substitute x = 1
into either of the original equations or their equivalents. Typically, we use the
equivalent equation that we found when isolating a variable in step 1.

12. Once a value is found for a


variable, substitute it back into
one of the original equations,
or their equivalent equations, The solution to the system is (1, 5). Be sure to present the solution as an ordered
to determine the pair.
corresponding value of the
other variable.

4.2 Solving Linear Systems by Substitution 645


Chapter 4 Solving Linear Systems

Step 5: Check. Verify that these coordinates solve both equations of the original
system:

The graph of this linear system follows:

The substitution method for solving systems is a completely algebraic method. Thus
graphing the lines is not required.

Answer: (1, 5)

4.2 Solving Linear Systems by Substitution 646


Chapter 4 Solving Linear Systems

{ x−y=3
2x − y = 12
Example 2: Solve by substitution: .

Solution: In this example, we can see that x has a coefficient of 1 in the second
equation. This indicates that it can be isolated in one step as follows:

Substitute 3 + y for x in the first equation. Use parentheses and take care to
distribute.

Use x = 3 + y to find x.

Answer: (9, 6). The check is left to the reader.

4.2 Solving Linear Systems by Substitution 647


Chapter 4 Solving Linear Systems

{ x = −1
3x − 5y = 17
Example 3: Solve by substitution: .

Solution: In this example, the variable x is already isolated. Hence we can substitute
x = −1 into the first equation.

Answer: (−1, −4). It is a good exercise to graph this particular system to compare the
substitution method to the graphing method for solving systems.

{ 8x + 2y = 10
3x + y = 4
Try this! Solve by substitution: .

Answer: (1, 1)

Video Solution

(click to see video)

Solving systems algebraically frequently requires work with fractions.

4.2 Solving Linear Systems by Substitution 648


Chapter 4 Solving Linear Systems

{ 24x − 4y = −15
2x + 8y = 5
Example 4: Solve by substitution: .

Solution: Begin by solving for x in the first equation.

Next, substitute into the second equation and solve for y.

Back substitute into the equation used in the substitution step:

4.2 Solving Linear Systems by Substitution 649


Chapter 4 Solving Linear Systems

Answer: (−1/2, 3/4)

As we know, not all linear systems have only one ordered pair solution. Recall that
some systems have infinitely many ordered pair solutions and some do not have
any solutions. Next, we explore what happens when using the substitution method
to solve a dependent system.

{ 10x − 2y = 2
−5x + y = −1
Example 5: Solve by substitution: .

Solution: Since the first equation has a term with coefficient 1, we choose to solve
for that first.

4.2 Solving Linear Systems by Substitution 650


Chapter 4 Solving Linear Systems

Next, substitute this expression in for y in the second equation.

This process led to a true statement; hence the equation is an identity and any real
number is a solution. This indicates that the system is dependent. The simultaneous
solutions take the form (x, mx + b), or in this case, (x, 5x − 1), where x is any real
number.

Answer: (x, 5x − 1)

To have a better understanding of the previous example, rewrite both equations in


slope-intercept form and graph them on the same set of axes.

4.2 Solving Linear Systems by Substitution 651


Chapter 4 Solving Linear Systems

We can see that both equations represent the same line, and thus the system is
dependent. Now explore what happens when solving an inconsistent system using
the substitution method.

{ 14x − 6y = −16
−7x + 3y = 3
Example 6: Solve by substitution: .

Solution: Solve for y in the first equation.

4.2 Solving Linear Systems by Substitution 652


Chapter 4 Solving Linear Systems

Substitute into the second equation and solve.

Solving leads to a false statement. This indicates that the equation is a


contradiction. There is no solution for x and hence no solution to the system.

Answer: No solution, Ø

A false statement indicates that the system is inconsistent, or in geometric terms,


that the lines are parallel and do not intersect. To illustrate this, determine the
slope-intercept form of each line and graph them on the same set of axes.

4.2 Solving Linear Systems by Substitution 653


Chapter 4 Solving Linear Systems

In slope-intercept form, it is easy to see that the two lines have the same slope but
different y-intercepts.

{ 4x − 10y = 6
2x − 5y = 3
Try this! Solve by substitution: .

Answer: (x, 2
5
x − 35 )

Video Solution

(click to see video)

4.2 Solving Linear Systems by Substitution 654


Chapter 4 Solving Linear Systems

KEY TAKEAWAYS

• The substitution method is a completely algebraic method for solving a


system of equations.
• The substitution method requires that we solve for one of the variables
and then substitute the result into the other equation. After performing
the substitution step, the resulting equation has one variable and can be
solved using the techniques learned up to this point.
• When the value of one of the variables is determined, go back and
substitute it into one of the original equations, or their equivalent
equations, to determine the corresponding value of the other variable.
• Solutions to systems of two linear equations with two variables, if they
exist, are ordered pairs (x, y).
• If the process of solving a system of equations leads to a false statement,
then the system is inconsistent and there is no solution, Ø.
• If the process of solving a system of equations leads to a true statement,
then the system is dependent and there are infinitely many solutions
that can be expressed using the form (x, mx + b).

4.2 Solving Linear Systems by Substitution 655


Chapter 4 Solving Linear Systems

TOPIC EXERCISES

Part A: Substitution Method

Solve by substitution.

{ −3x + y = 1
y = 4x − 1
1.

{ 4x − y = 2
y = 3x − 8
2.

{ x + 3y = −8
x = 2y − 3
3.

{ 2x + 3y = 12
x = −4y + 1
4.

{ −5x + 2y = 2
y = 3x
5.

{ 2x + 3y = 10
y=x
6.

{ −4x + y = 2
y = 4x + 1
7.

{ 3x + y = 5
y = −3x + 5
8.

4.2 Solving Linear Systems by Substitution 656


Chapter 4 Solving Linear Systems

{ 2x − y = −3
y = 2x + 3
9.

{ x − 2y = 5
y = 5x − 1
10.

{ 3x − y = 4
y = −7x + 1
11.

{ 5x − 2y = 0
x = 6y + 2
12.

{ −2x − y = −6
y = −2
13.

{ x − 4y = −3
x = −3
14.

{ 7x − 5y = 9
y=− 5
x+3
15.

{ 6x − 9y = 0
y= 3
x−1
16.

1 1

{ x − 6y = 4
y= 2
x+ 3
17.

3 1

{ 2x + 4y = 1
y=− 8
x+ 2
18.

4.2 Solving Linear Systems by Substitution 657


Chapter 4 Solving Linear Systems

{ 2x + 3y = 16
x+y=6
19.

{ −2x + 3y = −2
x−y=3
20.

{ 3x − 2y = 17
2x + y = 2
21.

{ 3x + 5y = −5
x − 3y = −11
22.

{ 3x − 4y = −2
x + 2y = −3
23.

{ 9x − y = 10
5x − y = 12
24.

{ −4x − 8y = 24
x + 2y = −6
25.

{ −2x − 6y = −12
x + 3y = −6
26.

{ 6x − 2y = −2
−3x + y = −4
27.

{ 2x − 10y = −20
x − 5y = −10
28.

4.2 Solving Linear Systems by Substitution 658


Chapter 4 Solving Linear Systems

{ 4x + 3y = −1
3x − y = 9
29.

{ 4x + 2y = −2
2x − y = 5
30.

{ 2x − 5y = −6
−x + 4y = 0
31.

{ 5x + 2y = −8
3y − x = 5
32.

{ 4x + 10y = 2
2x − 5y = 1
33.

{ 6x + 14y = 0
3x − 7y = −3
34.

10x − y = 3
{ −5x +
35. 1
2
y=1

1 1 2
− x+ y=
{
3 6 3
36.
1 1 3
2
x− 3
y=− 2

1 2
x+ y=1
{
3 3
37.
1 1 1
4
x− 3
y=− 12

1
x − y = 12
{
7
38.
1
4
x + 12 y = 2

4.2 Solving Linear Systems by Substitution 659


Chapter 4 Solving Linear Systems

3 2 1
− x+ y=
{
5 5 2
39.
1 1
3
x− 12
y= − 13

1 2
x= y
{x −
2 3
40.
2
3
y=2

1 1 5
− x+ y=
{
2 2 8
41.
1 1 1
4
x+ 2
y= 4

{ −x + 2y = 3
x−y=0
42.

{ 2x − 3y = 0
y = 3x
43.

{ −6x + 3y = −6
2x + 3y = 18
44.

{ 2x + 8y = 8
−3x + 4y = 20
45.

{ 3x + 2y = 7
5x − 3y = −1
46.

{ 2x + 7y = 1
−3x + 7y = 2
47.

{ y = −3
y=3
48.

4.2 Solving Linear Systems by Substitution 660


Chapter 4 Solving Linear Systems

{ x = −2
x=5
49.

{y = 4
y=4
50.

Set up a linear system and solve it using the substitution method.

51. The sum of two numbers is 19. The larger number is 1 less than three
times the smaller.

52. The sum of two numbers is 15. The larger is 3 more than twice the
smaller.

53. The difference of two numbers is 7 and their sum is 1.

54. The difference of two numbers is 3 and their sum is −7.

55. Where on the graph of −5x + 3y = 30 does the x-coordinate equal


the y-coordinate?

56. Where on the graph of 1


2
x− 1
3
y = 1 does the x-coordinate equal the
y-coordinate?

Part B: Discussion Board Topics

57. Describe what drives the choice of variable to solve for when beginning
the process of solving by substitution.

58. Discuss the merits and drawbacks of the substitution method.

4.2 Solving Linear Systems by Substitution 661


Chapter 4 Solving Linear Systems

ANSWERS

1: (2, 7)

3: (−5, −1)

5: (2, 6)

7: ∅

9: (x, 2x + 3)

11: (1/2, −5/2)

13: (4, −2)

15: (3, 12/5)

17: (−3, −7/6)

19: (2, 4)

21: (3, −4)

23: (−8/5, −7/10)

25: (x, − 1
2
x − 3)

27: ∅

29: (2, −3)

31: (−8, −2)

33: (1/2, 0)

35: ∅

37: (1, 1)

4.2 Solving Linear Systems by Substitution 662


Chapter 4 Solving Linear Systems

39: (−11/10, −2/5)

41: (−1/2, 3/4)

43: (0, 0)

45: (−4, 2)

47: (−1/5, 1/5)

49: ∅

51: The two numbers are 5 and 14.

53: The two numbers are 4 and −3.

55: (−15, −15)

4.2 Solving Linear Systems by Substitution 663


Chapter 4 Solving Linear Systems

4.3 Solving Linear Systems by Elimination

LEARNING OBJECTIVES

1. Solve linear systems using the elimination method.


2. Solve linear systems with fractions and decimals.
3. Identify the weaknesses and strengths of each method for solving linear
systems.

The Elimination Method

In this section, the goal is to develop another completely algebraic method for
solving a system of linear equations. We begin by defining what it means to add
equations together. In the following example, notice that if we add the expressions
on both sides of the equal sign, we obtain another true statement.

This is true in general: if A, B, C, and D are algebraic expressions, then we have the
following addition property of equations13:

For the system

13. If A, B, C, and D are algebraic


expressions, where A = B and C
= D, then A + C = B + D.

664
Chapter 4 Solving Linear Systems

we add the two equations together:

The sum of y and −y is zero and that term is eliminated. This leaves us with a linear
equation with one variable that can be easily solved:

At this point, we have the x coordinate of the simultaneous solution, so all that is
left to do is back substitute to find the corresponding y-value.

Hence the solution to the system is (3, 2). This process describes the elimination
(or addition) method14 for solving linear systems. Of course, the variable is not
always so easily eliminated. Typically, we have to find an equivalent system by
applying the multiplication property of equality to one or both of the equations as a
means to line up one of the variables to eliminate. The goal is to arrange that either
the x terms or the y terms are opposites, so that when the equations are added, the
terms eliminate. The steps for the elimination method are outlined in the following
example.

{ 3x − 2y = −7
14. A means of solving a system by 2x + y = 7
Example 1: Solve by elimination: .
adding equivalent equations in
such a way as to eliminate a
variable.

4.3 Solving Linear Systems by Elimination 665


Chapter 4 Solving Linear Systems

Solution:

Step 1: Multiply one, or both, of the equations to set up the elimination of one of
the variables. In this example, we will eliminate the variable y by multiplying both
sides of the first equation by 2. Take care to distribute.

This leaves us with an equivalent system where the variable y is lined up to


eliminate.

Step 2: Add the equations together to eliminate one of the variables.

Step 3: Solve for the remaining variable.

Step 3: Back substitute into either equation or its equivalent equation.

4.3 Solving Linear Systems by Elimination 666


Chapter 4 Solving Linear Systems

Step 4: Check. Remember that the solution must solve both of the original
equations.

Answer: (1, 5)

Occasionally, we will have to multiply both equations to line up one of the variables
to eliminate. We want the resulting equivalent equations to have terms with
opposite coefficients.

{ 3x + 2y = 7
5x − 3y = −1
Example 2: Solve by elimination: .

Solution: We choose to eliminate the terms with variable y because the coefficients
have different signs. To do this, we first determine the least common multiple of
the coefficients; in this case, the LCM(3, 2) is 6. Therefore, multiply both sides of
both equations by the appropriate values to obtain coefficients of −6 and 6.

4.3 Solving Linear Systems by Elimination 667


Chapter 4 Solving Linear Systems

This results in the following equivalent system:

The y terms are now lined up to eliminate.

Back substitute.

4.3 Solving Linear Systems by Elimination 668


Chapter 4 Solving Linear Systems

Answer: (1, 2)

Sometimes linear systems are not given in standard form. When this is the case, it is
best to first rearrange the equations before beginning the steps to solve by
elimination.

{ 3y = 4x + 1
5x + 12y = 11
Example 3: Solve by elimination: .

Solution: First, rewrite the second equation in standard form.

This results in the following equivalent system where like terms are aligned in
columns:

We can eliminate the term with variable y if we multiply the second equation by −4.

Next, we add the equations together,

4.3 Solving Linear Systems by Elimination 669


Chapter 4 Solving Linear Systems

Back substitute.

Answer: (1/3, 7/9)

{ −3x − 2y = 4
2x + y = −3
Try this! Solve by elimination: .

Answer: (−2, 1)

4.3 Solving Linear Systems by Elimination 670


Chapter 4 Solving Linear Systems

Video Solution

(click to see video)

At this point, we explore what happens when solving dependent and inconsistent
systems using the elimination method.

{ 6x − 2y = 14
3x − y = 7
Example 4: Solve by elimination: .

Solution: To eliminate the variable x, we could multiply the first equation by −2.

Now adding the equations we have

A true statement indicates that this is a dependent system. The lines coincide, and
we need y in terms of x to present the solution set in the form (x, mx + b) . Choose
one of the original equations and solve for y. Since the equations are equivalent, it
does not matter which one we choose.

4.3 Solving Linear Systems by Elimination 671


Chapter 4 Solving Linear Systems

Answer: (x, 3x − 7)

{ 2x − 6y = 12
−x + 3y = 9
Example 5: Solve by elimination: .

Solution: We can eliminate x by multiplying the first equation by 2.

Now adding the equations we have

A false statement indicates that the system is inconsistent. The lines are parallel
and do not intersect.

Answer: No solution, ∅

{ 2x + 10y = 30
3x + 15y = −15
Try this! Solve by elimination: .

Answer: No solution, ∅

4.3 Solving Linear Systems by Elimination 672


Chapter 4 Solving Linear Systems

Video Solution

(click to see video)


Clearing Fractions and Decimals

Given a linear system where the equations have fractional coefficients, it is usually
best to clear the fractions before beginning the elimination method.

 − 1 x+ 1 y= 4
 10
Example 6: Solve: 
 1 x + 1 y = − 2.
 7
2 5

3 21

Solution: Recall that we can clear fractions by multiplying both sides of an equation
by the least common denominator (LCD). Take care to distribute and then simplify.

This results in an equivalent system where the equations have integer coefficients,

Solve using the elimination method.

4.3 Solving Linear Systems by Elimination 673


Chapter 4 Solving Linear Systems

Back substitute.

Answer: (−3, 1)

We can use a similar technique to clear decimals before solving.

{ −0.5x + 0.12y = 0.16


3x − 0.6y = −0.9
Example 7: Solve: .

4.3 Solving Linear Systems by Elimination 674


Chapter 4 Solving Linear Systems

Solution: Multiply each equation by the lowest power of 10 necessary to result in


integer coefficients. In this case, multiply the first equation by 10 and the second
equation by 100.

This results in an equivalent system where the equations have integer coefficients:

Solve using the elimination method.

Back substitute.

4.3 Solving Linear Systems by Elimination 675


Chapter 4 Solving Linear Systems

Answer: (−0.2, 0.5)



Try this! Solve using elimination: 

1 2
x− y=3

3 3
.
1 1 8
3
x− 2
y= 3

Answer: (5, −2)

Video Solution

(click to see video)


Summary of the Methods for Solving Linear Systems

We have developed three methods for solving linear systems of two equations with
two variables. In this section, we summarize the strengths and weaknesses of each
method.

The graphing method is useful for understanding what a system of equations is and
what the solutions must look like. When the equations of a system are graphed on
the same set of axes, we can see that the solution is the point where the graphs
intersect. The graphing is made easy when the equations are in slope-intercept
form. For example,

4.3 Solving Linear Systems by Elimination 676


Chapter 4 Solving Linear Systems

The simultaneous solution (−1, 10) corresponds to the point of intersection. One
drawback of this method is that it is very inaccurate. When the coordinates of the
solution are not integers, the method is practically unusable. If we have a choice, we
typically avoid this method in favor of the more accurate algebraic techniques.

The substitution method, on the other hand, is a completely algebraic method. It


requires you to solve for one of the variables and substitute the result into the other
equation. The resulting equation has one variable for which you can solve. This
method is particularly useful when there is a variable within the system with
coefficient of 1. For example,

In this case, it is easy to solve for y in the first equation and then substitute the
result into the other equation. One drawback of this method is that it often leads to
equivalent equations with fractional coefficients, which are tedious to work with. If
there is not a coefficient of 1, then it usually is best to choose the elimination
method.

4.3 Solving Linear Systems by Elimination 677


Chapter 4 Solving Linear Systems

The elimination method is a completely algebraic method that makes use of the
addition property of equations. We multiply one or both of the equations to obtain
equivalent equations where one of the variables is eliminated if we add them
together. For example,

Here we multiply both sides of the first equation by 5 and both sides of the second
equation by −2. This results in an equivalent system where the variable x is
eliminated when we add the equations together. Of course, there are other
combinations of numbers that achieve the same result. We could even choose to
eliminate the variable y. No matter which variable is eliminated first, the solution
will be the same. Note that the substitution method, in this case, would require
tedious calculations with fractional coefficients. One weakness of the elimination
method, as we will see later in our study of algebra, is that it does not always work
for nonlinear systems.

KEY TAKEAWAYS

• The elimination method is a completely algebraic method for solving a


system of equations.
• Multiply one or both of the equations in a system by certain numbers to
obtain an equivalent system consisting of like terms with opposite
coefficients. Adding these equivalent equations together eliminates a
variable, and the resulting equation has one variable for which you can
solve.
• It is a good practice to first rewrite the equations in standard form
before beginning the elimination method.
• When the value of one of the variables is determined, back substitute
into one of the original equations, or their equivalent equations, and
determine the corresponding value of the other variable.

4.3 Solving Linear Systems by Elimination 678


Chapter 4 Solving Linear Systems

TOPIC EXERCISES

Part A: Elimination Method

Solve by elimination.

{ 2x − y = 9
x+y=3
1.

{ 5x + y = −18
x − y = −6
2.

{ −x − 2y = 0
x + 3y = 5
3.

{ x − y = −7
−x + 4y = 4
4.

{ x − y = −3
−x + y = 2
5.

{ 6x + 4y = 2
3x − y = −2
6.

{ 10x − y = 4
5x + 2y = −3
7.

{ x − 7y = 21
−2x + 14y = 28
8.

4.3 Solving Linear Systems by Elimination 679


Chapter 4 Solving Linear Systems

{ 12x − 6y = −24
−2x + y = 4
9.

{ 3x + 12y = 6
x + 8y = 3
10.

{ 4x + 10y = 14
2x − 3y = 15
11.

{ 3x − 9y = 15
4x + 3y = −10
12.

{ 8x + 3y = −15
−4x − 5y = −3
13.

{ 4x − 2y = −112
−2x + 7y = 56
14.

{ 3x + 5y = −10
−9x − 15y = −15
15.

{ 2x + 6y = −7
6x − 7y = 4
16.

{ −5x − 3y = −7
4x + 2y = 4
17.

{ 3x + 2y = 7
5x − 3y = −1
18.

4.3 Solving Linear Systems by Elimination 680


Chapter 4 Solving Linear Systems

{ 2x + 5y = −14
7x + 3y = 9
19.

{ 7x + 2y = −15
9x − 3y = 3
20.

{ −7x + 6y = 11
5x − 3y = −7
21.

{ 3x + 7y = −1
2x + 9y = 8
22.

{ 3x + 3y = −5
2x + 2y = 5
23.

{ 2x − 4y = 8
−3x + 6y = −12
24.

{ 15x + 10y = −1
25x + 15y = −1
25.

{ 18x − 12y = 5
2x − 3y = 2
26.

{ −3x − 2y = 4
y = −2x − 3
27.

{ 6y = 4x − 15
28x + 6y = 9
28.

4.3 Solving Linear Systems by Elimination 681


Chapter 4 Solving Linear Systems

{ y = −5x + 5
y = 5x + 15
29.

{ 5x − 8y = −16
2x − 3y = 9
30.

1 1 1
x− y=
{
2 3 6
31.
5 7
2
x+y= 2

1 1
x− y=1
{ x+y=
4 9
32.
3
4

1 1 1
x− y=
{
2 4 3
33.
1
4
x+ 1
2
y= − 19
6

14
− x + 2y = 4
{−
3
34.
1
3
x + 17 y = 21
4

{ 0.11x + 0.04y = −0.2


0.025x + 0.1y = 0.5
35.

{ 0.5x + y = −2.75
1.3x + 0.1y = 0.35
36.

{ 0.02x + 0.03y = 0.125


x+y=5
37.

{ 0.05x + 0.1y = 2.4


x + y = 30
38.

4.3 Solving Linear Systems by Elimination 682


Chapter 4 Solving Linear Systems

Set up a linear system and solve it using the elimination method.

39. The sum of two numbers is 14. The larger number is 1 less than two times
the smaller.

40. The sum of two numbers is 30. The larger is 2 more than three times the
smaller.

41. The difference of two numbers is 13 and their sum is 11.

42. The difference of two numbers is 2 and their sum is −12.

Part B: Mixed Exercises

Solve using any method.

{ 3x + y = 12
y = 2x − 3
43.

x + 3y = −5
{y=
44. 1
3
x+5

{ y=3
x = −1
45.

{x + 9 = 0
y= 2
46.

{ −x + y = 1
y=x
47.

{ y = −10
y = 5x
48.

4.3 Solving Linear Systems by Elimination 683


Chapter 4 Solving Linear Systems

{ 3x + 4y = 2
3y = 2x − 24
49.

{ −2y + 2 = 3x
y=− 2
x+1
50.

{ 7x = 2y + 23
7y = −2x − 1
51.

{ 3x + 2y − 5 = 0
5x + 9y − 14 = 0
52.

 y = − 5 x + 10

53. 
 y = 5 x − 10

16

16

{
y=− 5
x + 12
54.
x=6

2 (x − 3) + y = 0
{ 3 (2x + y − 1) = 15
55.

3 − 2 (x − y) = −3
{ 4x − 3 (y + 1) = 8
56.

2 (x + 1) = 3 (2y − 1) − 21
{ 3 (x + 2) = 1 − (3y − 2)
57.

x y
− = −7
{
2 3
58. y
x
3
− 2
= −8

4.3 Solving Linear Systems by Elimination 684


Chapter 4 Solving Linear Systems

x y 3
− =
{
4 2 4
59. y
x 1
3
+ 6
= 6

1 2
x− y=3
{
3 3
60.
1 1 8
3
x− 2
y= 3

1 1 4
− x+ y=
{
10 2 5
61.
1 1 2
7
x+ 3
y=− 21

 y=− 5 x+ 1

62. 
 1 x+ 1 y= 1
3
3 2

5 10

1 2
− x+y=−
{−
7 3
63.
1 1 1
14
x+ 2
y= 3

1 1 1
x− y=
{− 3
15 12 3
64.
3 3
10
x+ 8
y=− 2

{ 0.03x + 0.0525y = 193.5


x + y = 4,200
65.

{ 0.2x + 0.1y = 52.5


x + y = 350
66.

{ 0.3x + 0.1y = −0.3


0.2x − 0.05y = 0.43
67.

{ 0.05x − 0.5y = −0.63


0.1x + 0.3y = 0.3
68.

4.3 Solving Linear Systems by Elimination 685


Chapter 4 Solving Linear Systems

{ −0.75x + 1.25y = −4
0.15x − 0.25y = −0.3
69.

{ −0.03x + 0.25y = 0.08


−0.15x + 1.25y = 0.4
70.

Part C: Discussion Board Topics

71. How do we choose the best method for solving a linear system?

72. What does it mean for a system to be dependent? How can we tell if a
given system is dependent?

4.3 Solving Linear Systems by Elimination 686


Chapter 4 Solving Linear Systems

ANSWERS

1: (4, −1)

3: (−10, 5)

5: ∅

7: (1/5, −2)

9: (x, 2x + 4)

11: (6, −1)

13: (−3, 3)

15: ∅

17: (−1, 4)

19: (3, −4)

21: (−1, 2/3)

23: ∅

25: (1/5, −2/5)

27: (−2, 1)

29: (−1, 10)

31: (1, 1)

33: (−2, −16/3)

35: (−4, 6)

37: (2.5, 2.5)

4.3 Solving Linear Systems by Elimination 687


Chapter 4 Solving Linear Systems

39: The two numbers are 5 and 9.

41: The two numbers are 12 and −1.

43: (3, 3)

45: (−1, 3)

47: Ø

49: (6, −4)

51: (3, −1)

53: (32, 0)

55: (x, − 2x + 6)

57: (−4, 3)

59: (1, −1)

61: (−3, 1)

63: ∅

65: (1,200, 3,000)

67: (0.8, −5.4)

69: Ø

4.3 Solving Linear Systems by Elimination 688


Chapter 4 Solving Linear Systems

4.4 Applications of Linear Systems

LEARNING OBJECTIVES

1. Set up and solve applications involving relationships between numbers.


2. Set up and solve applications involving interest and money.
3. Set up and solve mixture problems.
4. Set up and solve uniform motion problems (distance problems).

Problems Involving Relationships between Real Numbers

We now have the techniques needed to solve linear systems. For this reason, we are
no longer limited to using one variable when setting up equations that model
applications. If we translate an application to a mathematical setup using two
variables, then we need to form a linear system with two equations.

Example 1: The sum of two numbers is 40 and their difference is 8. Find the
numbers.

Solution:

Identify variables.

Set up equations: When using two variables, we need to set up two equations. The
first key phrase, “the sum of the two numbers is 40,” translates as follows:

689
Chapter 4 Solving Linear Systems

And the second key phrase, “the difference is 8,” leads us to the second equation:

Therefore, our algebraic setup consists of the following system:

Solve: We can solve the resulting system using any method of our choosing. Here we
choose to solve by elimination. Adding the equations together eliminates the
variable y.

Once we have x, back substitute to find y.

Check: The sum of the two numbers should be 42 and their difference 8.

4.4 Applications of Linear Systems 690


Chapter 4 Solving Linear Systems

Answer: The two numbers are 24 and 16.

Example 2: The sum of 9 times a larger number and twice a smaller is 6. The
difference of 3 times the larger and the smaller is 7. Find the numbers.

Solution: Begin by assigning variables to the larger and smaller number.

The first sentence describes a sum and the second sentence describes a difference.

This leads to the following system:

Solve using the elimination method. Multiply the second equation by 2 and add.

4.4 Applications of Linear Systems 691


Chapter 4 Solving Linear Systems

Back substitute to find y.

Answer: The larger number is 4/3 and the smaller number is −3.

Try this! The sum of two numbers is 3. When twice the smaller number is
subtracted from 6 times the larger the result is 22. Find the numbers.

Answer: The two numbers are −1/2 and 7/2.

Video Solution

(click to see video)

4.4 Applications of Linear Systems 692


Chapter 4 Solving Linear Systems

Interest and Money Problems

In this section, the interest and money problems15 should seem familiar. The
difference is that we will be making use of two variables when setting up the
algebraic equations.

Example 3: A roll of 32 bills contains only $5 bills and $10 bills. If the value of the
roll is $220, then how many of each bill are in the roll?

Solution: Begin by identifying the variables.

When using two variables, we need to set up two equations. The first equation is
created from the fact that there are 32 bills.

The second equation sums the value of each bill: the total value is $220.

Present both equations as a system; this is our algebraic setup.

15. Applications involving simple


interest and money.

4.4 Applications of Linear Systems 693


Chapter 4 Solving Linear Systems

Here we choose to solve by elimination, although substitution would work just as


well. Eliminate x by multiplying the first equation by −5.

Now add the equations together:

Once we have y, the number of $10 bills, back substitute to find x.

Answer: There are twenty $5 bills and twelve $10 bills. The check is left to the
reader.

Example 4: A total of $6,300 was invested in two accounts. Part was invested in a CD
at a 4 12 %annual interest rate and part was invested in a money market fund at a
3 34 %annual interest rate. If the total simple interest for one year was $267.75, then
how much was invested in each account?

Solution:

4.4 Applications of Linear Systems 694


Chapter 4 Solving Linear Systems

The total amount in both accounts can be expressed as

To set up a second equation, use the fact that the total interest was $267.75. Recall
that the interest for one year is the interest rate times the principal
(I = prt = pr ⋅ 1 = pr ). Use this to add the interest in both accounts. Be sure to use
the decimal equivalents for the interest rates given as percentages.

These two equations together form the following linear system:

Eliminate y by multiplying the first equation by −0.0375.

Next, add the equations together to eliminate the variable y.

4.4 Applications of Linear Systems 695


Chapter 4 Solving Linear Systems

Back substitute.

Answer: $4,200 was invested at 4 1


2
%and $2,100 was invested at 3 34 %.

At this point, we should be able to solve these types of problems in two ways: with
one variable and now with two variables. Setting up word problems with two
variables often simplifies the entire process, particularly when the relationships
between the variables are not so clear.

Try this! On the first day of a two-day meeting, 10 coffees and 10 doughnuts were
purchased for a total of $20.00. Since nobody drank the coffee and all the doughnuts
were eaten, the next day only 2 coffees and 14 doughnuts were purchased for a total
of $13.00. How much did each coffee and each doughnut cost?

Answer: Coffee: $1.25; doughnut: $0.75

Video Solution

(click to see video)

4.4 Applications of Linear Systems 696


Chapter 4 Solving Linear Systems

Mixture Problems

Mixture problems16 often include a percentage and some total amount. It is


important to make a distinction between these two types of quantities. For
example, if a problem states that a 20-ounce container is filled with a 2% saline
(salt) solution, then this means that the container is filled with a mixture of salt and
water as follows:

Percentage Amount

Salt 2% = 0.02 0.02(20 ounces) = 0.4 ounces

Water 98% = 0.98 0.98(20 ounces) = 19.6 ounces

In other words, we multiply the percentage times the total to get the amount of
each part of the mixture.

Example 5: A 2% saline solution is to be combined and mixed with a 5% saline


solution to produce 72 ounces of a 2.5% saline solution. How much of each is
needed?

Solution:

The total amount of saline solution needed is 72 ounces. This leads to one equation,

The second equation adds up the amount of salt in the correct percentages. The
amount of salt is obtained by multiplying the percentage times the amount, where
16. Applications involving a the variables x and y represent the amounts of the solutions.
mixture of amounts usually
given as a percentage of some
total.

4.4 Applications of Linear Systems 697


Chapter 4 Solving Linear Systems

The algebraic setup consists of both equations presented as a system:

Solve.

Back substitute.

Answer: We need 60 ounces of the 2% saline solution and 12 ounces of the 5% saline
solution.

4.4 Applications of Linear Systems 698


Chapter 4 Solving Linear Systems

Example 6: A 50% alcohol solution is to be mixed with a 10% alcohol solution to


create an 8-ounce mixture of a 32% alcohol solution. How much of each is needed?

Solution:

The total amount of the mixture must be 8 ounces.

The second equation adds up the amount of alcohol from each solution in the
correct percentages. The amount of alcohol in the end result is 32% of 8 ounces, or
0.032(8).

Now we can form a system of two linear equations and two variables as follows:

In this example, multiply the second equation by 100 to eliminate the decimals. In
addition, multiply the first equation by −10 to line up the variable y to eliminate.

4.4 Applications of Linear Systems 699


Chapter 4 Solving Linear Systems

We obtain the following equivalent system:

Add the equations and then solve for x:

Back substitute.

Answer: To obtain 8 ounces of a 32% alcohol mixture we need to mix 4.4 ounces of
the 50% alcohol solution and 3.6 ounces of the 10% solution.

4.4 Applications of Linear Systems 700


Chapter 4 Solving Linear Systems

Try this! A 70% antifreeze concentrate is to be mixed with water to produce a


5-gallon mixture containing 28% antifreeze. How much water and antifreeze
concentrate is needed?

Answer: We need to mix 3 gallons of water with 2 gallons of antifreeze concentrate.

Video Solution

(click to see video)


Uniform Motion Problems (Distance Problems)

Recall that the distance traveled is equal to the average rate times the time traveled
at that rate, D = r ⋅ t. These uniform motion problems17 usually have a lot of
data, so it helps to first organize that data in a chart and then set up a linear
system. In this section, you are encouraged to use two variables.

Example 7: An executive traveled a total of 8 hours and 1,930 miles by car and by
plane. Driving to the airport by car, she averaged 60 miles per hour. In the air, the
plane averaged 350 miles per hour. How long did it take her to drive to the airport?

Solution: We are asked to find the time it takes her to drive to the airport; this
indicates that time is the unknown quantity.

17. Applications relating distance, Use the formula D = r ⋅ t to fill in the unknown distances.
average rate, and time.

4.4 Applications of Linear Systems 701


Chapter 4 Solving Linear Systems

The distance column and the time column of the chart help us to set up the
following linear system.

Solve.

4.4 Applications of Linear Systems 702


Chapter 4 Solving Linear Systems

Now back substitute to find the time it took to drive to the airport x:

Answer: It took her 3 hours to drive to the airport.

It is not always the case that time is the unknown quantity. Read the problem
carefully and identify what you are asked to find; this defines your variables.

Example 8: Flying with the wind, an airplane traveled 1,365 miles in 3 hours. The
plane then turned against the wind and traveled another 870 miles in 2 hours. Find
the speed of the airplane and the speed of the wind.

Solution: There is no obvious relationship between the speed of the plane and the
speed of the wind. For this reason, use two variables as follows:

4.4 Applications of Linear Systems 703


Chapter 4 Solving Linear Systems

Use the following chart to organize the data:

With the wind, the airplane’s total speed is x + w . Flying against the wind, the total
speed is x − w .

Use the rows of the chart along with the formula D = r ⋅ t to construct a linear
system that models this problem. Take care to group the quantities that represent
the rate in parentheses.

4.4 Applications of Linear Systems 704


Chapter 4 Solving Linear Systems

If we divide both sides of the first equation by 3 and both sides of the second
equation by 2, then we obtain the following equivalent system:

Back substitute.

Answer: The speed of the airplane is 445 miles per hour and the speed of the wind is
10 miles per hour.

Try this! A boat traveled 24 miles downstream in 2 hours. The return trip, which
was against the current, took twice as long. What are the speeds of the boat and of
the current?

Answer: The speed of the boat is 9 miles per hour and the speed of the current is 3
miles per hour.

Video Solution

(click to see video)

4.4 Applications of Linear Systems 705


Chapter 4 Solving Linear Systems

KEY TAKEAWAYS

• Use two variables as a means to simplify the algebraic setup of


applications where the relationship between unknowns is unclear.
• Carefully read the problem several times. If two variables are used, then
remember that you need to set up two linear equations in order to solve
the problem.
• Be sure to answer the question in sentence form and include the correct
units for the answer.

4.4 Applications of Linear Systems 706


Chapter 4 Solving Linear Systems

TOPIC EXERCISES

Part A: Applications Involving Numbers

Set up a linear system and solve.

1. The sum of two integers is 54 and their difference is 10. Find the integers.

2. The sum of two integers is 50 and their difference is 24. Find the integers.

3. The sum of two positive integers is 32. When the smaller integer is
subtracted from twice the larger, the result is 40. Find the two integers.

4. The sum of two positive integers is 48. When twice the smaller integer is
subtracted from the larger, the result is 12. Find the two integers.

5. The sum of two integers is 74. The larger is 26 more than twice the
smaller. Find the two integers.

6. The sum of two integers is 45. The larger is 3 less than three times the
smaller. Find the two integers.

7. The sum of two numbers is zero. When 4 times the smaller number is
added to 8 times the larger, the result is 1. Find the two numbers.

8. The sum of a larger number and 4 times a smaller number is 5. When 8


times the smaller is subtracted from twice the larger, the result is −2. Find
the numbers.

9. The sum of 12 times the larger number and 11 times the smaller is −36.
The difference of 12 times the larger and 7 times the smaller is 36. Find the
numbers.

10. The sum of 4 times the larger number and 3 times the smaller is 7. The
difference of 8 times the larger and 6 times the smaller is 10. Find the
numbers.

Part B: Interest and Money Problems

Set up a linear system and solve.

4.4 Applications of Linear Systems 707


Chapter 4 Solving Linear Systems

11. A $7,000 principal is invested in two accounts, one earning 3% interest


and another earning 7% interest. If the total interest for the year is $262,
then how much is invested in each account?

12. Mary has her total savings of $12,500 in two different CD accounts. One
CD earns 4.4% interest and another earns 3.2% interest. If her total interest
for the year is $463, then how much does she have in each CD account?

13. Sally’s $1,800 savings is in two accounts. One account earns 6% annual
interest and the other earns 3%. Her total interest for the year is $93. How
much does she have in each account?

14. Joe has two savings accounts totaling $4,500. One account earns 3 34 %
annual interest and the other earns 2 58 %. If his total interest for the year
is $141.75, then how much is in each account?

15. Millicent has $10,000 invested in two accounts. For the year, she earns
$535 more in interest from her 7% mutual fund account than she does from
her 4% CD. How much does she have in each account?

16. A small business has $85,000 invested in two accounts. If the account
earning 3% annual interest earns $825 more in interest than the account
earning 4.5% annual interest, then how much is invested in each account?

17. Jerry earned a total of $284 in simple interest from two separate
accounts. In an account earning 6% interest, Jerry invested $1,000 more than
twice the amount he invested in an account earning 4%. How much did he
invest in each account?

18. James earned a total of $68.25 in simple interest from two separate
accounts. In an account earning 2.6% interest, James invested one-half as
much as he did in the other account that earned 5.2%. How much did he
invest in each account?

19. A cash register contains $10 bills and $20 bills with a total value of $340.
If there are 23 bills total, then how many of each does the register contain?

20. John was able to purchase a pizza for $10.80 with quarters and dimes. If
he uses 60 coins to buy the pizza, then how many of each did he have?

4.4 Applications of Linear Systems 708


Chapter 4 Solving Linear Systems

21. Dennis mowed his neighbor’s lawn for a jar of dimes and nickels. Upon
completing the job, he counted the coins and found that there were 4 less
than twice as many dimes as there were nickels. The total value of all the
coins is $6.60. How many of each coin did he have?

22. Two families bought tickets for the big football game. One family ordered
2 adult tickets and 3 children’s tickets for a total of $26.00. Another family
ordered 3 adult tickets and 4 children’s tickets for a total of $37.00. How
much did each adult ticket cost?

23. Two friends found shirts and shorts on sale at a flea market. One bought
5 shirts and 3 shorts for a total of $51.00. The other bought 3 shirts and 7
shorts for a total of $80.00. How much was each shirt and each pair of
shorts?

24. On Monday Joe bought 10 cups of coffee and 5 doughnuts for his office at
a cost of $16.50. It turns out that the doughnuts were more popular than the
coffee. Therefore, on Tuesday he bought 5 cups of coffee and 10 doughnuts
for a total of $14.25. How much was each cup of coffee?

Part C: Mixture Problems

Set up a linear system and solve.

25. A 15% acid solution is to be mixed with a 25% acid solution to produce 12
gallons of a 20% acid solution. How much of each is needed?

26. One alcohol solution contains 12% alcohol and another contains 26%
alcohol. How much of each should be mixed together to obtain 5 gallons of a
14.8% alcohol solution?

27. A nurse wishes to obtain 40 ounces of a 1.2% saline solution. How much
of a 1% saline solution must she mix with a 2% saline solution to achieve the
desired result?

28. A customer ordered 20 pounds of fertilizer that contains 15% nitrogen.


To fill the customer’s order, how much of the stock 30% nitrogen fertilizer
must be mixed with the 10% nitrogen fertilizer?

29. A customer ordered 2 pounds of a mixed peanut product containing 15%


cashews. The inventory consists of only two mixes containing 10% and 30%
cashews. How much of each type must be mixed to fill the order?

4.4 Applications of Linear Systems 709


Chapter 4 Solving Linear Systems

30. How many pounds of pure peanuts must be combined with a 20% peanut
mix to produce 10 pounds of a 32% peanut mix?

31. How much cleaning fluid with 20% alcohol content, must be mixed with
water to obtain a 24-ounce mixture with 10% alcohol content?

32. A chemist wishes to create a 32-ounce solution with 12% acid content. He
uses two types of stock solutions, one with 30% acid content and another
with 10% acid content. How much of each does he need?

33. A concentrated cleaning solution that contains 50% ammonia is mixed


with another solution containing 10% ammonia. How much of each is mixed
to obtain 8 ounces of a 32% ammonia cleaning formula?

34. A 50% fruit juice concentrate can be purchased wholesale. Best taste is
achieved when water is mixed with the concentrate in such a way as to
obtain a 12% fruit juice mixture. How much water and concentrate is needed
to make a 50-ounce fruit juice drink?

35. A 75% antifreeze concentrate is to be mixed with water to obtain 6


gallons of a 25% antifreeze solution. How much water is needed?

36. Pure sugar is to be mixed with a fruit salad containing 10% sugar to
produce 48 ounces of a salad containing 16% sugar. How much pure sugar is
required?

Part D: Uniform Motion Problems

Set up a linear system and solve.

37. An airplane averaged 460 miles per hour on a trip with the wind behind
it and 345 miles per hour on the return trip against the wind. If the total
round trip took 7 hours, then how long did the airplane spend on each leg of
the trip?

38. The two legs of a 330-mile trip took 5 hours. The average speed for the
first leg of the trip was 70 miles per hour and the average speed for the
second leg of the trip was 60 miles per hour. How long did each leg of the
trip take?

4.4 Applications of Linear Systems 710


Chapter 4 Solving Linear Systems

39. An executive traveled 1,200 miles, part by helicopter and part by private
jet. The jet averaged 320 miles per hour while the helicopter averaged 80
miles per hour. If the total trip took 4½ hours, then how long did she spend
in the private jet?

40. Joe took two buses on the 463-mile trip from San Jose to San Diego. The
first bus averaged 50 miles per hour and the second bus was able to average
64 miles per hour. If the total trip took 8 hours, then how long was spent in
each bus?

41. Billy canoed downstream to the general store at an average rate of 9


miles per hour. His average rate canoeing back upstream was 4 miles per
hour. If the total trip took 6½ hours, then how long did it take Billy to get
back on the return trip?

42. Two brothers drove the 2,793 miles from Los Angeles to New York. One of
the brothers, driving in the day, was able to average 70 miles per hour, and
the other, driving at night, was able to average 53 miles per hour. If the total
trip took 45 hours, then how many hours did each brother drive?

43. A boat traveled 24 miles downstream in 2 hours. The return trip took
twice as long. What was the speed of the boat and the current?

44. A helicopter flying with the wind can travel 525 miles in 5 hours. On the
return trip, against the wind, it will take 7 hours. What are the speeds of the
helicopter and of the wind?

45. A boat can travel 42 miles with the current downstream in 3 hours.
Returning upstream against the current, the boat can only travel 33 miles in
3 hours. Find the speed of the current.

46. A light aircraft flying with the wind can travel 180 miles in 1½ hours. The
aircraft can fly the same distance against the wind in 2 hours. Find the speed
of the wind.

Part E: Discussion Board

47. Compose a number or money problem that can be solved with a system
of equations of your own and share it on the discussion board.

48. Compose a mixture problem that can be solved with a system of


equations of your own and share it on the discussion board.

4.4 Applications of Linear Systems 711


Chapter 4 Solving Linear Systems

49. Compose a uniform motion problem that can be solved with a system of
equations of your own and share it on the discussion board.

4.4 Applications of Linear Systems 712


Chapter 4 Solving Linear Systems

ANSWERS

1: The integers are 22 and 32.

3: The integers are 8 and 24.

5: The integers are 16 and 58.

7: The two numbers are −1/4 and 1/4.

9: The smaller number is −4 and the larger is 2/3.

11: $5,700 at 3% and $1,300 at 7%

13: $1,300 at 6% and $500 at 3%

15: $8,500 at 7% and $1,500 at 4%

17: $1,400 at 4% and $3,800 at 6%

19: 12 tens and 11 twenties

21: 52 dimes and 28 nickels

23: Shirts: $4.50; shorts: $9.50

25: 6 gallons of each

27: 32 ounces of the 1% saline solution and 8 ounces of the 2% saline solution

29: 1.5 pounds of the 10% cashew mix and 0.5 pounds of the 30% cashew mix

31: 12 ounces of cleaning fluid

33: 4.4 ounces of the 50% ammonia solution and 3.6 ounces of the 10%
ammonia solution

35: 4 gallons

37: The airplane flew 3 hours with the wind and 4 hours against the wind.

4.4 Applications of Linear Systems 713


Chapter 4 Solving Linear Systems

39: 3.5 hours

41: 4.5 hours

43: Boat: 9 miles per hour; current: 3 miles per hour

45: 1.5 miles per hour

4.4 Applications of Linear Systems 714


Chapter 4 Solving Linear Systems

4.5 Solving Systems of Linear Inequalities (Two Variables)

LEARNING OBJECTIVES

1. Check solutions to systems of linear inequalities with two variables.


2. Solve systems of linear inequalities.

Solutions to Systems of Linear Inequalities

A system of linear inequalities18 consists of a set of two or more linear inequalities


with the same variables. The inequalities define the conditions that are to be
considered simultaneously. For example,

We know that each inequality in the set contains infinitely many ordered pair
solutions defined by a region in a rectangular coordinate plane. When considering
two of these inequalities together, the intersection of these sets defines the set of
simultaneous ordered pair solutions. When we graph each of the above inequalities
separately, we have

18. A set of two or more linear


inequalities that define the
conditions to be considered When graphed on the same set of axes, the intersection can be determined.
simultaneously.

715
Chapter 4 Solving Linear Systems

The intersection is shaded darker and the final graph of the solution set is
presented as follows:

The graph suggests that (3, 2) is a solution because it is in the intersection. To verify
this, show that it solves both of the original inequalities:

4.5 Solving Systems of Linear Inequalities (Two Variables) 716


Chapter 4 Solving Linear Systems

Points on the solid boundary are included in the set of simultaneous solutions and
points on the dashed boundary are not. Consider the point (−1, 0) on the solid
boundary defined by y = 2x + 2 and verify that it solves the original system:

Notice that this point satisfies both inequalities and thus is included in the solution
set. Now consider the point (2, 0) on the dashed boundary defined by y = x − 2
and verify that it does not solve the original system:

This point does not satisfy both inequalities and thus is not included in the solution
set.

Solving Systems of Linear Inequalities

Solutions to a system of linear inequalities are the ordered pairs that solve all the
inequalities in the system. Therefore, to solve these systems, graph the solution sets

4.5 Solving Systems of Linear Inequalities (Two Variables) 717


Chapter 4 Solving Linear Systems

of the inequalities on the same set of axes and determine where they intersect. This
intersection, or overlap, defines the region of common ordered pair solutions.

{ 3x − 6y ≥ 6
−2x + y > −4
Example 1: Graph the solution set: .

Solution: To facilitate the graphing process, we first solve for y.

For the first inequality, we use a dashed boundary defined by y = 2x − 4 and


shade all points above the line. For the second inequality, we use a solid boundary
defined by y = 12 x − 1 and shade all points below. The intersection is darkened.

Now we present the solution with only the intersection shaded.

4.5 Solving Systems of Linear Inequalities (Two Variables) 718


Chapter 4 Solving Linear Systems

Answer:

{ 4x − 6y > 12
−2x + 3y > 6
Example 2: Graph the solution set: .

Solution: Begin by solving both inequalities for y.

Use a dashed line for each boundary. For the first inequality, shade all points above
the boundary. For the second inequality, shade all points below the boundary.

4.5 Solving Systems of Linear Inequalities (Two Variables) 719


Chapter 4 Solving Linear Systems

As you can see, there is no intersection of these two shaded regions. Therefore,
there are no simultaneous solutions.

Answer: No solution, ∅

 y ≥ −4

Example 3: Graph the solution set:  y < x + 3 .

 y ≤ −3x + 3

Solution: The intersection of all the shaded regions forms the triangular region as
pictured darkened below:

4.5 Solving Systems of Linear Inequalities (Two Variables) 720


Chapter 4 Solving Linear Systems

After graphing all three inequalities on the same set of axes, we determine that the
intersection lies in the triangular region pictured.

Answer:

4.5 Solving Systems of Linear Inequalities (Two Variables) 721


Chapter 4 Solving Linear Systems

The graphic suggests that (−1, 1) is a common point. As a check, substitute that
point into the inequalities and verify that it solves all three conditions.

KEY TAKEAWAY

• To solve systems of linear inequalities, graph the solution sets of each


inequality on the same set of axes and determine where they intersect.

4.5 Solving Systems of Linear Inequalities (Two Variables) 722


Chapter 4 Solving Linear Systems

TOPIC EXERCISES

Part A: Solving Systems of Linear Inequalities

Determine whether the given point is a solution to the given system of linear
equations.

{ y ≥ −x + 3
y≤x+3
1. (3, 2);

{ y ≥ 2x − 1
y < −3x + 4
2. (−3, −2);

y > −x + 5
{y ≤
3. (5, 0); 3
4
x−2

2
y< x+1
{y ≥
3
4. (0, 1);
5
2
x−2

{ 2x + 3y < 6
−4x + 3y ≥ −12
5. (−1, 8
3
);

 −x + y < 0

6. (−1, −2);  x + y < 0

 x + y < −2

Part B: Solving Systems of Linear Inequalities

Graph the solution set.

{ y ≥ −x + 3
y≤x+3
7.

4.5 Solving Systems of Linear Inequalities (Two Variables) 723


Chapter 4 Solving Linear Systems

{ y ≥ 2x − 1
y < −3x + 4
8.

{ y < −1
y>x
9.

2
y< x+1
{y ≥
3
10.
5
2
x−2

y > −x + 5
{y ≤
11. 3
4
x−2

y >

12. 
y <
3
x+3

5
3
5
x−3

{ −3x + 12y ≥ −12


x + 4y < 12
13.

{ 2x + y ≥ 1
−x + y ≤ 6
14.

{ 4x − 3y < 15
−2x + 3y > 3
15.

{ 2x + 3y < 6
−4x + 3y ≥ −12
16.

{ −4x + 3y < −6
5x + y ≤ 4
17.

4.5 Solving Systems of Linear Inequalities (Two Variables) 724


Chapter 4 Solving Linear Systems

{ −x + 2y ≤ 0
3x + 5y < 15
18.

{ 5x + y > 5
x≥0
19.

{ y≥1
x ≥ −2
20.

{y + 2 ≥ 0
x−3<0
21.

{ −2x < −5y − 5


5y ≥ 2x + 5
22.

{ −x + y < 1
x−y≥0
23.

{ y−x<1
−x + y ≥ 0
24.

{x ≤ 2
x > −2
25.

{y < 2
y > −1
26.

{ 3x − 6y ≥ 18
−x + 2y > 8
27.

4.5 Solving Systems of Linear Inequalities (Two Variables) 725


Chapter 4 Solving Linear Systems

{ 6x − 8y > −8
−3x + 4y ≤ 4
28.

2x + y < 3
{ −x ≤
29. 1
2
y

2x + 6y ≤ 6
{−
30. 1
3
x−y≤3

 y<3

31.  y > x

 x > −4

y < 1

32.  y ≥ x − 1

 y < −3x + 3

 −4x + 3y > −12



33. 

 2x + 3y > 6
y≥2

 −x + y < 0

34.  x + y ≤ 0

 x + y > −2

 x+y<2

35. 

 −x + y ≤ 2
x<3

4.5 Solving Systems of Linear Inequalities (Two Variables) 726


Chapter 4 Solving Linear Systems

 y+4≥0
 1
36.  2 x + 3 y ≤ 1
 1
1

− 2 x + 3 y ≤ 1
1

37. Construct a system of linear inequalities that describes all points in the
first quadrant.

38. Construct a system of linear inequalities that describes all points in the
second quadrant.

39. Construct a system of linear inequalities that describes all points in the
third quadrant.

40. Construct a system of linear inequalities that describes all points in the
fourth quadrant.

4.5 Solving Systems of Linear Inequalities (Two Variables) 727


Chapter 4 Solving Linear Systems

ANSWERS

1: Yes

3: No

5: No

7:

9:

11:

13:

4.5 Solving Systems of Linear Inequalities (Two Variables) 728


Chapter 4 Solving Linear Systems

15:

17:

19:

21:

4.5 Solving Systems of Linear Inequalities (Two Variables) 729


Chapter 4 Solving Linear Systems

23:

25:

27: No solution, ∅

29:

31:

4.5 Solving Systems of Linear Inequalities (Two Variables) 730


Chapter 4 Solving Linear Systems

33:

35:

{y > 0
x>0
37:

{y < 0
x<0
39:

4.5 Solving Systems of Linear Inequalities (Two Variables) 731


Chapter 4 Solving Linear Systems

4.6 Review Exercises and Sample Exam

732
Chapter 4 Solving Linear Systems

REVIEW EXERCISES

Solving Linear Systems by Graphing

Determine whether the given ordered pair is a solution to the given system.

{ −3x + y = −6
5x − y = 8
1. (1, −3);

{ 6x − 5y = −2
4x − 2
y = −10
2. (−3, −4);

 3 x− 1 y=− 2
 5
3. (−1, 1/5); 
− 1 x − 1 y = 1
 5
3 3

2 10

3 1
x+ y=−
{
4 4
4. (1/2, −1);
2 4
3
x−y= 3

Given the graph, determine the simultaneous solution.

5.

6.

4.6 Review Exercises and Sample Exam 733


Chapter 4 Solving Linear Systems

7.

8.

Solve by graphing.

1
y= x−3
{y = −
2
9.
3
4
x+2

y=5
{y = −
10. 4
5
x+1

{ 2x − 3y = 3
x − 2y = 0
11.

4.6 Review Exercises and Sample Exam 734


Chapter 4 Solving Linear Systems

{ −4x + 2y = 16
5x − y = −11
12.

{ 5x + 4y = 12
2
x + 2y = 6
13.

{ 3x − 5y = 5
6x − 10y = −2
14.

Solving Linear Systems by Substitution

Solve by substitution.

{x + y = 6
y = 7x − 2
15.

{ x = −2y − 1
2x − 4y = 10
16.

{ 5x − 7y = −8
x−y=0
17.

{ −x + y = 7
9x + 2y = −41
18.

{ 2x − 9y = 4
6x − 3y = 4
19.

{ 12x + 3y = 6
8x − y = 7
20.

4.6 Review Exercises and Sample Exam 735


Chapter 4 Solving Linear Systems

20x − 4y = −3
{ −5x + y = −
21. 1
2

3x − y = 6
{x −
22. 1
3
y=2

{ 8x − 4y = −10
x = −1
23.

{ 14x − 4y = 0
y = −7
24.

Solving Linear Systems by Elimination

Solve by elimination.

{ 3x − 8y = 5
x−y=5
25.

{ 9x + 4y = −30
7x + 2y = −10
26.

{ 2x − 5y = 17
9x − 6y = −6
27.

{ 3x + 7y = 14
4x − 2y = 30
28.

5 11
x − 2y = −
{
2 4
29.
1 1
6
x− 3
y= − 13

4.6 Review Exercises and Sample Exam 736


Chapter 4 Solving Linear Systems

 2x − 3 y = 20

30. 
 3 x − 1 y = 11
2
2 3

3 6

{ 0.6x + 0.5y = −0.13


0.1x − 0.3y = 0.17
31.

{
−1.25x − 0.45y = −12.23
32.
0.5x − 1.5y = 5.9

{ −12x + 5y = 10
6x − 2
y = −5
33.

{ 9x + 4y = 3
27x + 12y = −2
34.

{ 4x − 3y = 2
6x − 5y = 0
35.

{ 10x + 3y = 6
5x = 1
36.

{ 3x = 6y − 18
8y = −2x + 6
37.

{ 9x − 27y − 3 = 0
6y = 3x + 1
38.

Applications of Linear Systems

Set up a linear system and solve.

4.6 Review Exercises and Sample Exam 737


Chapter 4 Solving Linear Systems

39. The sum of two numbers is 74 and their difference is 38. Find the
numbers.

40. The sum of two numbers is 34. When the larger is subtracted from twice
the smaller, the result is 8. Find the numbers.

41. A jar full of 40 coins consisting of dimes and nickels has a total value of
$2.90. How many of each coin are in the jar?

42. A total of $9,600 was invested in two separate accounts earning 5.5% and
3.75% annual interest. If the total simple interest earned for the year was
$491.25, then how much was invested in each account?

43. A 1% saline solution is to be mixed with a 3% saline solution to produce 6


ounces of a 1.8% saline solution. How much of each is needed?

44. An 80% fruit juice concentrate is to be mixed with water to produce 10


gallons of a 20% fruit juice mixture. How much of each is needed?

45. An executive traveled a total of 4½ hours and 435 miles to a conference


by car and by light aircraft. Driving to the airport by car, he averaged 50
miles per hour. In the air, the light aircraft averaged 120 miles per hour.
How long did it take him to drive to the airport?

46. Flying with the wind, an airplane traveled 1,065 miles in 3 hours. On the
return trip, against the wind, the airplane traveled 915 miles in 3 hours.
What is the speed of the wind?

Systems of Linear Inequalities (Two Variables)

Determine whether the given point is a solution to the system of linear inequalities.

{ −3x + y ≤ −6
5x − y > 8
47. (5, −2);

{ −5x + y > 1
2x − 3y > −10
48. (2, 3);

4.6 Review Exercises and Sample Exam 738


Chapter 4 Solving Linear Systems

{x − y ≥ 0
y < −10
49. (2, −10);

1
y> x−4
{y < −
2
50. (0, −2);
3
4
x+2

Graph the solution set.

{ 2x + 3y < 12
8x + 3y ≤ 24
51.

{ 4x − y ≥ 0
x+y≥7
52.

{ −2x + 6y > −6
x − 3y > −12
53.

{x − y > 0
y≤7
54.

 y<4

55.  y ≥ 3 x + 1

4

 y > −x − 1

 x − y ≥ −3

56.  x − y ≤ 3

 x+y<1

4.6 Review Exercises and Sample Exam 739


Chapter 4 Solving Linear Systems

SAMPLE EXAM

{ −4x + y = 14
2x − 3y = −12
1. Is (−3, 2) a solution to the system ?

{ 4x − y < 0
x+y≥7
2. Is (−2, 9) a solution to the system ?

Given the graph, determine the simultaneous solution.

3.

4.

Solve using the graphing method.

{ y = − 12 x + 3
y=x−3
5.

{ −x + 6y = −18
2x + 3y = 6
6.

4.6 Review Exercises and Sample Exam 740


Chapter 4 Solving Linear Systems

{x + y = 3
y=2
7.

{ x = −5
y=x
8.

Solve using the substitution method.

{ 2x − 3y = −9
5x + y = −14
9.

{ x − 2y = 2
4x − 3y = 1
10.

{ 10x + 2y = 4
5x + y = 1
11.

{ 3x − 6y = 12
x − 2y = 4
12.

Solve using the elimination method.

{ −5x + 2y = −17
4x − y = 13
13.

{ 4x + 5y = 7
7x − 3y = −23
14.

{ x − 6y = 6
−3x + 18y = 18
15.

4.6 Review Exercises and Sample Exam 741


Chapter 4 Solving Linear Systems

{ 8x − 6y = 6
−4x + 3y = −3
16.

1 3 7
x+ y=
{ 4x −
2 4 4
17.
1 4
3
y= 3

{ −0.3x + 0.5y = 0.08


0.2x − 0.1y = −0.24
18.

Graph the solution set.

{ x − 4y < 8
3x + 4y < 24
19.

{ 3x − 8y ≤ 0
x≤8
20.

Set up a linear system of two equations and two variables and solve it using any
method.

21. The sum of two integers is 23. If the larger integer is one less than twice
the smaller, then find the two integers.

22. James has $2,400 saved in two separate accounts. One account earns 3%
annual interest and the other earns 4%. If his interest for the year totals $88,
then how much is in each account?

23. Mary drives 110 miles to her grandmother’s house in a total of 2 hours.
On the freeway, she averages 62 miles per hour. In the city she averages 34
miles per hour. How long does she spend on the freeway?

24. A 15% acid solution is to be mixed with a 35% acid solution to produce 12
ounces of a 22% acid solution. How much of each is needed?

25. Joey has bag full of 52 dimes and quarters with a total value of $8.35. How
many of each coin does Joey have?

4.6 Review Exercises and Sample Exam 742


Chapter 4 Solving Linear Systems

REVIEW EXERCISES ANSWERS

1: Yes

3: Yes

5: (−3, 1)

7: Ø

9: (4, −1)

11: (6, 3)

13: (x, − 5
4
x + 3)

15: (1, 5)

17: (4, 4)

19: (1/2, −1/3)

21: Ø

23: (−1, 1/2)

25: (7, 2)

27: (−4, −5)

29: (−1/2, 3/4)

31: (0.2, −0.5)

33: (x, 12
5
x + 2)

35: (5, 6)

37: (−3, 3/2)

4.6 Review Exercises and Sample Exam 743


Chapter 4 Solving Linear Systems

39: 18 and 56

41: 18 dimes and 22 nickels

43: 3.6 ounces of the 1% saline solution and 2.4 ounces of the 3% saline
solution

45: It took him 1½ hours to drive to the airport.

47: Yes

49: No

51:

53:

55:

4.6 Review Exercises and Sample Exam 744


Chapter 4 Solving Linear Systems

SAMPLE EXAM ANSWERS

1: Yes

3: (−1, −2)

5: (4, 1)

7: (1, 2)

9: (−3, 1)

11: Ø

13: (3, −1)

15: Ø

17: (1/2, 2)

19:

21: 8 and 15

23: She drives 1½ hours on the freeway.

25: 21 quarters and 31 dimes

4.6 Review Exercises and Sample Exam 745


Chapter 5
Polynomials and Their Operations

746
Chapter 5 Polynomials and Their Operations

5.1 Rules of Exponents

LEARNING OBJECTIVES

1. Simplify expressions using the rules of exponents.


2. Simplify expressions involving parentheses and exponents.
3. Simplify expressions involving 0 as an exponent.

Product, Quotient, and Power Rule for Exponents

If a factor is repeated multiple times, then the product can be written in


exponential form1 x n . The positive integer exponent n indicates the number of
times the base x is repeated as a factor.

For example,

Here the base is 5 and the exponent is 4. Exponents are sometimes indicated with
the caret (^) symbol found on the keyboard: 5^4 = 5*5*5*5.

Next consider the product of 23 and 25 ,

Expanding the expression using the definition produces multiple factors of the
1. An equivalent expression base, which is quite cumbersome, particularly when n is large. For this reason, we
written using a rational
exponent.

747
Chapter 5 Polynomials and Their Operations

will develop some useful rules to help us simplify expressions with exponents. In
this example, notice that we could obtain the same result by adding the exponents.

In general, this describes the product rule for exponents2. If m and n are positive
integers, then

In other words, when multiplying two expressions with the same base, add the
exponents.

Example 1: Simplify: 105 ⋅ 1018 .

Solution:

Answer: 1023

In the previous example, notice that we did not multiply the base 10 times itself.
When applying the product rule, add the exponents and leave the base unchanged.

2. x m ⋅ x n = x m+n; the
product of two expressions
with the same base can be
simplified by adding the
exponents. Example 2: Simplify: x 6 ⋅ x 12 ⋅ x .

5.1 Rules of Exponents 748


Chapter 5 Polynomials and Their Operations

Solution: Recall that the variable x is assumed to have an exponent of 1: x = x 1 .

Answer: x 19

The base could be any algebraic expression.

Example 3: Simplify: (x + y) (x + y) .
9 13

Solution: Treat the expression (x + y) as the base.

Answer: (x + y) 22

The commutative property of multiplication allows us to use the product rule for
exponents to simplify factors of an algebraic expression.

Example 4: Simplify: 2x 8 y ⋅ 3x 4 y 7 .

5.1 Rules of Exponents 749


Chapter 5 Polynomials and Their Operations

Solution: Multiply the coefficients and add the exponents of variable factors with
the same base.

Answer: 6x 12 y 8

Next, we will develop a rule for division by first looking at the quotient of 27 and
23 .

Here we can cancel factors after applying the definition of exponents. Notice that
the same result can be obtained by subtracting the exponents.

This describes the quotient rule for exponents3. If m and n are positive integers
and x ≠ 0 , then

xm In other words, when you divide two expressions with the same base, subtract the
3. x n = x m−n; the quotient of
two expressions with the same exponents.
base can be simplified by
subtracting the exponents.

5.1 Rules of Exponents 750


Chapter 5 Polynomials and Their Operations

12y 15
Example 5: Simplify: .
4y 7

Solution: Divide the coefficients and subtract the exponents of the variable y.

Answer: 3y 8

20x 10 (x+5)
6

10x 9 (x+5)
Example 6: Simplify: 2
.

Solution:

Answer: 2x(x + 5)
4

Now raise 23 to the fourth power as follows:

5.1 Rules of Exponents 751


Chapter 5 Polynomials and Their Operations

After writing the base 23 as a factor four times, expand to obtain 12 factors of 2. We
can obtain the same result by multiplying the exponents.

In general, this describes the power rule for exponents4. Given positive integers m
and n, then

In other words, when raising a power to a power, multiply the exponents.

Example 7: Simplify: (y 6 ) .
7

Solution:

n
4. (x m ) = x mn ; a power Answer: y 42
raised to a power can be
simplified by multiplying the
exponents.

5.1 Rules of Exponents 752


Chapter 5 Polynomials and Their Operations

To summarize, we have developed three very useful rules of exponents that are
used extensively in algebra. If given positive integers m and n, then

Product rule: x m ⋅ x n = x m+n

xm
Quotient rule: = x m−n , x ≠ 0
x n

(x ) = x
m n
m⋅n
Power rule:

Try this! Simplify: y 5 ⋅ (y 4 ) .


6

Answer: y 29

Video Solution

(click to see video)


Power Rules for Products and Quotients

Now we consider raising grouped products to a power. For example,

After expanding, we have four factors of the product xy. This is equivalent to raising
each of the original factors to the fourth power. In general, this describes the
5. (xy) = x n y n; if a product
n power rule for a product5. If n is a positive integer, then
is raised to a power, then apply
that power to each factor in
the product.

5.1 Rules of Exponents 753


Chapter 5 Polynomials and Their Operations

Example 8: Simplify: (2ab) .


7

Solution: We must apply the exponent 7 to all the factors, including the coefficient,
2.

If a coefficient is raised to a relatively small power, then present the real number
equivalent, as we did in this example: 27 = 128.

Answer: 128a7 b7

In many cases, the process of simplifying expressions involving exponents requires


the use of several rules of exponents.

Example 9: Simplify: (3xy 3 ) .


4

Solution:

5.1 Rules of Exponents 754


Chapter 5 Polynomials and Their Operations

Answer: 81x 4 y 12

Example 10: Simplify: (4x 2 y 5 z) .


3

Solution:

Answer: 64x 6 y 15 z 3

Example 11: Simplify: [5(x + y) ] .


3 3

Solution:

Answer: 125(x + y)
9

Next, consider a quotient raised to a power.

5.1 Rules of Exponents 755


Chapter 5 Polynomials and Their Operations

Here we obtain four factors of the quotient, which is equivalent to the numerator
and the denominator both raised to the fourth power. In general, this describes the
power rule for a quotient6. If n is a positive integer and y ≠ 0 , then

In other words, given a fraction raised to a power, we can apply that exponent to
the numerator and the denominator. This rule requires that the denominator is
nonzero. We will make this assumption for the remainder of the section.

Example 12: Simplify: ( 3a


b )
3
.

Solution: First, apply the power rule for a quotient and then the power rule for a
product.

(y ) =
n
x xn
6. y n ; if a quotient is
raised to a power, then apply
that power to the numerator
and the denominator.

5.1 Rules of Exponents 756


Chapter 5 Polynomials and Their Operations

3
Answer: 27a3
b

In practice, we often combine these two steps by applying the exponent to all
factors in the numerator and the denominator.

Example 13: Simplify: ( ab3 ) .


2 5
2c

Solution: Apply the exponent 5 to all of the factors in the numerator and the
denominator.

5 10
Answer: a b15
32c

Example 14: Simplify: ( ).


5x 5 (2x−1) 4 2
3y 7

Solution:

5.1 Rules of Exponents 757


Chapter 5 Polynomials and Their Operations

25x 10 (2x−1) 8
Answer:
9y 14

It is a good practice to simplify within parentheses before using the power rules;
this is consistent with the order of operations.

Example 15: Simplify: (


xy 2 )
−2x 3 y 4 z 4
.

Solution:

Answer: 16x 8 y 8 z 4

5.1 Rules of Exponents 758


Chapter 5 Polynomials and Their Operations

To summarize, we have developed two new rules that are useful when grouping
symbols are used in conjunction with exponents. If given a positive integer n, where
y is a nonzero number, then

(xy) = x y
n n n
Power rule for a product:

(y )
n
x xn
Power rule for a quotient: = n
y

4x 2 (x−y)
( )
3 3
Try this! Simplify: .
3yz 5

64x 6 (x−y)
9

Answer:
27y 3 z 15

Video Solution

(click to see video)


Zero as an Exponent

Using the quotient rule for exponents, we can define what it means to have 0 as an
exponent. Consider the following calculation:

Eight divided by 8 is clearly equal to 1, and when the quotient rule for exponents is
applied, we see that a 0 exponent results. This leads us to the definition of zero as
an exponent7, where x ≠ 0 :
7. x 0 = 1; any nonzero base
raised to the 0 power is defined
to be 1.

5.1 Rules of Exponents 759


Chapter 5 Polynomials and Their Operations

It is important to note that 00 is undefined. If the base is negative, then the result is
still +1. In other words, any nonzero base raised to the 0 power is defined to be 1. In
the following examples, assume all variables are nonzero.

Example 16: Simplify:

a. (−5)
0

b. −50

Solution:

a. Any nonzero quantity raised to the 0 power is equal to 1.

b. In the example −50 , the base is 5, not −5.

Answers: a. 1; b. −1

Example 17: Simplify: (5x 3 y 0 z 2 ) .


2

5.1 Rules of Exponents 760


Chapter 5 Polynomials and Their Operations

Solution: It is good practice to simplify within the parentheses first.

Answer: 25x 6 z 4

Example 18: Simplify: (− 8a12 b14 ) .


10 5 0

5c d

Solution:

Answer: 1

Try this! Simplify: 5x 0 and (5x) .


0

Answer: 5x 0 = 5 and (5x) = 1


0

Video Solution

(click to see video)

5.1 Rules of Exponents 761


Chapter 5 Polynomials and Their Operations

KEY TAKEAWAYS

• The rules of exponents allow you to simplify expressions involving


exponents.
• When multiplying two quantities with the same base, add exponents:
x m ⋅ x n = x m+n.
• When dividing two quantities with the same base, subtract exponents:
xm
xn = x m−n.
n
• When raising powers to powers, multiply exponents: (x m ) = x m⋅n .
• When a grouped quantity involving multiplication and division is raised

the denominator: (xy)


to a power, apply that power to all of the factors in the numerator and
= x n y n and ( xy ) =
n n
xn
yn .
• Any nonzero quantity raised to the 0 power is defined to be equal to 1:
x 0 = 1.

5.1 Rules of Exponents 762


Chapter 5 Polynomials and Their Operations

TOPIC EXERCISES

Part A: Product, Quotient, and Power Rule for Exponents

Write each expression using exponential form.

1. (2x) (2x) (2x) (2x) (2x)

2. (−3y) (−3y) (−3y)

3. −10 ⋅a⋅a⋅a⋅a⋅a⋅a⋅a

4. 12 ⋅x⋅x⋅y⋅y⋅y⋅y⋅y⋅y

5. −6 ⋅ (x − 1)(x − 1)(x − 1)

6. (9ab) (9ab) (9ab) (a − b) (a − b)


2 2

Simplify.

7
7. 2 ⋅ 25
9
8. 3 ⋅3
4
9. −2

4
10. (−2)

3
11. −3

4
12. (−3)

13
13. 10 ⋅ 10 5 ⋅ 10 4
8
14. 10 ⋅ 10 7 ⋅ 10

5 12
15.
52

5.1 Rules of Exponents 763


Chapter 5 Polynomials and Their Operations

10 7
16.
10

10 12
17.
10 9

18. (7 )
3 5

19. (4 )
8 4

⋅ (10 5 )
6 4
20. 10

Simplify.

6
21. (−x)

22. a5 ⋅ (−a)2

23. x 3 ⋅ x5 ⋅ x

24. y 5 ⋅ y4 ⋅ y2

25. (a5 ) ⋅ (a3 ) ⋅ a


2 4

+ 1) 4 (y 5 ) ⋅ y 2
4
26. (x

27. (x + 1) 5 (x + 1) 8

28. (2a − b) (2a − b)


12 9

(3x−1) 5
29.
(3x−1) 2

(a−5)
37

(a−5)
30. 13

5.1 Rules of Exponents 764


Chapter 5 Polynomials and Their Operations

31. xy 2 ⋅ x 2y

32. 3x 2 y 3 ⋅ 7xy 5

33. −8a2 b ⋅ 2ab


2 3
34. −3ab c ⋅ 9a4 b 5 c6
4
35. 2a2 b c (−3abc)

36. 5a2 (b ) c ⋅ (−2) a (b )


3 3 3 2 3 2 4

37. 2x 2 (x + y) ⋅ 3x 5 (x + y)
5 4

38. −5xy 6 (2x − 1) 6 ⋅ x 5 y(2x − 1) 3

39. x 2 y ⋅ xy 3 ⋅ x 5 y 5

40. −2x 10 y ⋅ 3x 2 y 12 ⋅ 5xy 3


2 4 2
41. 3 x y z ⋅ 3xy 4 z 4

42. (−x 2 ) (x ) (x )
3 3 4 2 3

a10 ⋅(a6 )
3

43.
a3

10x 9 (x 3 )
5

44.
2x 5

a6 b 3
45.
a2 b 2

m 10 n 7
46.
m3 n4

20x 5 y 12 z 3
47.
10x 2 y 10 z

5.1 Rules of Exponents 765


Chapter 5 Polynomials and Their Operations

−24a16 b 12 c3
48.
6a6 b 11 c

16 x 4 (x+2) 3
49.
4x(x+2)

50y 2 (x+y)
20

10y(x+y)
50. 17

Part B: Power Rules for Products and Quotients

Simplify.

5
51. (2x)

52. (−3y)
4

53. (−xy)
3

54. (5xy)
3

55. (−4abc)
2

56. (
2x )
7 2

(− )
3
5
57.
3y

58. ( c )
3ab 3

(− )
2xy 4
59.
3z

( x )
5y(2x−1) 3
60.

5.1 Rules of Exponents 766


Chapter 5 Polynomials and Their Operations

61. (3x 2 )
3

62. (−2x 3 )
2

63. (xy 5 )
7

64. (x 2 y 10 )
2

65. (3x 2 y)
3

66. (2x 2 y 3 z 4 )
5

67. (−7ab c )
4 2 2

[x y (x + y) ]
5 4 4 5
68.

69. [2y(x + 1) 5 ]
3

( b3 )
3
a
70.

( 3b )
4
5a2
71.

(− )
2
2x 3
72.
3y 2

(− )
3
x2
73.
y3

( 3c3 d 2 )
4
ab 2
74.

5.1 Rules of Exponents 767


Chapter 5 Polynomials and Their Operations

( )
6
2x 7 y(x−1) 3
75.
z5

76. (2x 4 ) ⋅ (x 5 )
3 2

77. (x 3 y) ⋅ (xy 4 )
2 3

78. (−2a2 b ) ⋅ (2a b)


3 2 5 4

79. (−a2 b) (3ab )


3 4 4

(2x (x + y) ) ⋅ (2x (x + y) )
4 5 2 3
3 4
80.

( xy 2 )
−3x 5 y 4 3
81.

( xy 2 )
−3x 5 y 4 2
82.

( 5x 5 y 10 )
−25x 10 y 15 3
83.

( 5xy 2 )
10x 3 y 5 2
84.

(− )
5
24ab 3
85.
6bc

( 16x 2 y )
−2x 3 y 2
86.

( 3abc )
3
30ab 3
87.

( 2s2 t )
3
3s3 t 2
88.

5.1 Rules of Exponents 768


Chapter 5 Polynomials and Their Operations

6xy 5 (x+y)
( 3y 2 z(x+y) 2 )
6 5
89.

(− )
4
64a5 b 12 c2 (2ab−1)14
90.
32a2 b 10 c2 (2ab−1)7

(2)
91. The probability of tossing a fair coin and obtaining n heads in a row is
n
given by the formula P = 1
. Determine the probability, as a percent, of
tossing 5 heads in a row.

same faces up in a row is given by the formula P = ( 16 ) . Determine the


92. The probability of rolling a single fair six-sided die and obtaining n of the
n

probability, as a percent, of obtaining the same face up two times in a row.

93. If each side of a square measures 2x 3 units, then determine the area in
terms of the variable x.

94. If each edge of a cube measures 5x 2 units, then determine the volume
in terms of the variable x.

Part C: Zero Exponents

Simplify. (Assume variables are nonzero.)

0
95. 7

0
96. (−7)

0
97. −10

0
98. −3 ⋅ (−7) 0
0
99. 8675309

2
100. 5 ⋅ 30 ⋅ 23
0
101. −3 ⋅ (−2) 2 ⋅ (−3) 0

5.1 Rules of Exponents 769


Chapter 5 Polynomials and Their Operations

102. 5x 0 y 2

2 2 0 5
103. (−3) x y z

(x ) y (z )
2 3 2 3 2 0
104. −3

105. 2x 3 y 0 z ⋅ 3x 0 y 3 z 5

c ⋅ 3a2 (b 3 c2 )
2 0 0
106. −3ab

107. (−8xy 2 )
0

( y3 )
0
2x 2
108.

9x 0 y 4
109.
3y 3

Part D: Discussion Board Topics

110. René Descartes (1637) established the usage of exponential form: a2 ,


a3 , and so on. Before this, how were exponents denoted?

111. Discuss the accomplishments accredited to Al-Karismi.

0
112. Why is 0 undefined?

4
113. Explain to a beginning student why 3 ⋅ 3 2 ≠ 9 6.

5.1 Rules of Exponents 770


Chapter 5 Polynomials and Their Operations

ANSWERS

5
1: (2x)

3: −10a7

5: −6(x − 1) 3
12
7: 2

9: −16

11: −27

22
13: 10

10
15: 5

3
17: 10

32
19: 4

21: x 6

23: x 9

25: a23

27: (x + 1) 13

29: (3x − 1) 3

31: x 3 y 3

2
33: −16a3 b

5 2
35: −6a3 b c

5.1 Rules of Exponents 771


Chapter 5 Polynomials and Their Operations

37: 6x 7 (x + y)
9

39: x 8 y 9

41: 27x 5 y 6 z 5

43: a25

45: a4 b

47: 2x 3 y 2 z 2

49: 4x 3 (x + 2) 2

51: 32x 5

53: −x 3 y 3

2 2
55: 16a2 b c
125
57: −
27y 3

16x 4 y 4
59:
81z 4

61: 27x 6

63: x 7 y 35

65: 27x 6 y 3

8 4
67: 49a2 b c

69: 8y 3 (x + 1) 15

625a8
71:
81b 4

5.1 Rules of Exponents 772


Chapter 5 Polynomials and Their Operations

x6
73: −
y9

64x 42 y 6 (x−1) 18
75:
z 30

77: x 9 y 14

19
79: −81a10 b

81: −27x 12 y 6

83: −125x 15 y 15

1024a5 b 10
85: −
c5

1000b 6
87:
c3

32x 5 y 15 (x+y)
20

89:
z5

91: 3 1
8
%

93: A = 4x 6

95: 1

97: −1

99: 1

101: −4

103: 9x 2 z 5

105: 6x 3 y 3 z 6

107: 1

5.1 Rules of Exponents 773


Chapter 5 Polynomials and Their Operations

109: 3y

5.1 Rules of Exponents 774


Chapter 5 Polynomials and Their Operations

5.2 Introduction to Polynomials

LEARNING OBJECTIVES

1. Identify a polynomial and determine its degree.


2. Evaluate a polynomial for given values of the variables.
3. Evaluate a polynomial using function notation.

Definitions

A polynomial8 is a special algebraic expression with terms that consist of real


number coefficients and variable factors with whole number exponents.

Polynomials do not have variables in the denominator of any term.

8. An algebraic expression
consisting of terms with real The degree of a term9 in a polynomial is defined to be the exponent of the variable,
number coefficients and or if there is more than one variable in the term, the degree is the sum of their
variables with whole number
exponents. Recall that x 0 = 1; any constant term can be written as a product of x 0
exponents.
and itself. Hence the degree of a constant term is 0.
9. The exponent of the variable; if
there is more than one variable
in the term, the degree of the
term is the sum their
exponents.

775
Chapter 5 Polynomials and Their Operations

Term Degree

3x 2 2

6x 2 y 2+1=3

7a2 b3 2+3=5

0
8
, since 8 = 8x 0

2x 1, since x = x1

The degree of a polynomial10 is the largest degree of all of its terms.

Polynomial Degree

4x 5 − 3x 3 + 2x − 1 5
10. The largest degree of all of its
terms.

5.2 Introduction to Polynomials 776


Chapter 5 Polynomials and Their Operations

Polynomial Degree

6x 2 y − 5xy 3 + 7 4, because 5xy 3 has degree 4.

1 5
x+ 1, because x = x1
2 4

We classify polynomials by the number of terms and the degree as follows:

Expression Classification Degree

5x 7 Monomial11 (one term) 7

8x 6 − 1 Binomial12 (two terms) 6

−3x 2 + x − 1 Trinomial13 (three terms) 2

11. Polynomial with one term.

12. Polynomial with two terms.

13. Polynomial with three terms.

5.2 Introduction to Polynomials 777


Chapter 5 Polynomials and Their Operations

Expression Classification Degree

5x 3 − 2x 2 + 3x − 6 Polynomial14 (many terms) 3

In this text, we will call polynomials with four or more terms simply polynomials.

Example 1: Classify and state the degree: 7x 2 − 4x 5 − 1.

Solution: Here there are three terms. The highest variable exponent is 5. Therefore,
this is a trinomial of degree 5.

Answer: Trinomial; degree 5

Example 2: Classify and state the degree: 12a5 bc3 .

Solution: Since the expression consists of only multiplication, it is one term, a


monomial. The variable part can be written as a5 b1 c3 ; hence its degree is
5 + 1 + 3 = 9.

Answer: Monomial; degree 9

Example 3: Classify and state the degree: 4x 2 y − 6xy 4 + 5x 3 y 3 + 4.

Solution: The term 4x 2 y has degree 3; −6xy 4 has degree 5; 5x 3 y 3 has degree 6;
14. An algebraic expression and the constant term 4 has degree 0. Therefore, the polynomial has 4 terms with
consisting of terms with real
number coefficients and degree 6.
variables with whole number
exponents.

5.2 Introduction to Polynomials 778


Chapter 5 Polynomials and Their Operations

Answer: Polynomial; degree 6

Of particular interest are polynomials with one variable15, where each term is of
the form an x n . Here an is any real number and n is any whole number. Such
polynomials have the standard form

Typically, we arrange terms of polynomials in descending order based on the


degree of each term. The leading coefficient16 is the coefficient of the variable with
the highest power, in this case, an .

Example 4: Write in standard form: 3x − 4x 2 + 5x 3 + 7 − 2x 4 .

Solution: Since terms are separated by addition, write the following:

In this form, we can see that the subtraction in the original corresponds to negative
coefficients. Because addition is commutative, we can write the terms in descending
order based on the degree of each term as follows:

15. A polynomial where each term


has the form an x n , where an
is any real number and n is any
whole number. Answer: −2x 4 + 5x 3 − 4x 2 + 3x + 7
16. The coefficient of the term
with the largest degree.

5.2 Introduction to Polynomials 779


Chapter 5 Polynomials and Their Operations

We can further classify polynomials with one variable by their degree as follows:

Polynomial Name

5 Constant (degree 0)

2x + 1 Linear (degree 1)

3x 2 + 5x − 3 Quadratic (degree 2)

x3 + x2 + x + 1 Cubic (degree 3)

7x 4 + 3x 3 − 7x + 8 Fourth-degree polynomial

In this text, we call any polynomial of degree n ≥ 4 an nth-degree polynomial. In


other words, if the degree is 4, we call the polynomial a fourth-degree polynomial.
If the degree is 5, we call it a fifth-degree polynomial, and so on.

5.2 Introduction to Polynomials 780


Chapter 5 Polynomials and Their Operations

Evaluating Polynomials

Given the values for the variables in a polynomial, we can substitute and simplify
using the order of operations.

Example 5: Evaluate: 3x − 1 , where x = − 32 .

Solution: First, replace the variable with parentheses and then substitute the given
value.

Answer: −11/2

Example 6: Evaluate: 3x 2 + 2x − 1, where x = −1 .

Solution:

Answer: 0

5.2 Introduction to Polynomials 781


Chapter 5 Polynomials and Their Operations

Example 7: Evaluate: −2a2 b + ab2 − 7, where a = 3 and b = −2.

Solution:

Answer: 41

Example 8: The volume of a sphere in cubic units is given by the formula


V = 43 πr3, where r is the radius. Calculate the volume of a sphere with radius
r = 32 meters.

Solution:

5.2 Introduction to Polynomials 782


Chapter 5 Polynomials and Their Operations

Answer: 9
2
πcubic meters

Try this! Evaluate: x 3 − x 2 + 4x − 2, where x = −3 .

Answer: −50

Video Solution

(click to see video)


Polynomial Functions

Polynomial functions with one variable are functions that can be written in the
form

where an is any real number and n is any whole number. Some examples of the
different classes of polynomial functions are listed below:

5.2 Introduction to Polynomials 783


Chapter 5 Polynomials and Their Operations

Polynomial function Name

f (x) = 5 Constant function17 (degree 0)

f (x) = −2x + 1 Linear function18 (degree 1)

f (x) = 5x 2 + 4x − 3 Quadratic function19 (degree 2)

f (x) = x 3 − 1 Cubic function20 (degree 3)

f (x) = 4x 5 + 3x 4 − 7 Polynomial function

Since there are no restrictions on the values for x, the domain of any polynomial
function consists of all real numbers.
17. A polynomial function with
degree 0.

18. A polynomial function with


degree 1.
Example 9: Calculate: f (5), given f (x) = −2x 2 + 5x + 10.
19. A polynomial function with
degree 2.

20. A polynomial function with Solution: Recall that the function notation f (5) indicates we should evaluate the
degree 3. function when x = 5 . Replace every instance of the variable x with the value 5.

5.2 Introduction to Polynomials 784


Chapter 5 Polynomials and Their Operations

Answer: f (5) = −15

Example 10: Calculate: f (−1), given f (x) = −x 3 + 2x 2 − 4x + 1.

Solution: Replace the variable x with −1.

Answer: f (−1) = 8

Try this! Given g (x) = x 3 − 2x 2 − x − 4, calculate g(−1).

Answer: g (−1) = −6

Video Solution

(click to see video)

5.2 Introduction to Polynomials 785


Chapter 5 Polynomials and Their Operations

KEY TAKEAWAYS

• Polynomials are special algebraic expressions where the terms are the
products of real numbers and variables with whole number exponents.
• The degree of a polynomial with one variable is the largest exponent of
the variable found in any term.
• The terms of a polynomial are typically arranged in descending order
based on the degree of each term.
• When evaluating a polynomial, it is a good practice to replace all
variables with parentheses and then substitute the appropriate values.
• All polynomials are functions.

5.2 Introduction to Polynomials 786


Chapter 5 Polynomials and Their Operations

TOPIC EXERCISES

Part A: Definitions

Classify the given polynomial as linear, quadratic, or cubic.

1. 2x +1

2. x 2 + 7x + 2

3. 2 − 3x 2 + x

4. 4x

5. x 2 − x3 + x + 1

6. 5 − 10x 3

Classify the given polynomial as a monomial, binomial, or trinomial and state the
degree.

7. x 3 −1

8. x 2 y 2

9. x − x5 + 1

10. x 2 + 3x − 1
4
11. 5ab

1 1
12.
3
x− 2

13. −5x 3 + 2x + 1

14. 8x 2 −9

15. 4x 5 − 5x 3 + 6x

5.2 Introduction to Polynomials 787


Chapter 5 Polynomials and Their Operations

16. 8x 4 − x 5 + 2x − 3

17. 9x +7

18. x 5 + x4 + x3 + x2 − x + 1

19. 6x −1 + 5x 4 − 8
4
20. x − 3x 2 + 3

21. 7

22. x 2

23. 4x 2 y − 3x 3 y 3 + xy 3
2
24. a3 b − 6ab
3
25. a3 b

x2 y2
26. y − x

27. xy −3

28. a5 bc2 + 3a9 − 5a4 b 3 c

29. −3x 10 y 2 z − xy 12 z + 9x 13 + 30

30. 7x 0

Write the following polynomials in standard form.

31. 1 − 6x + 7x 2

32. x − 9x 2 − 8

33. 7 − x 3 + x 7 − x 2 + x − 5x 5

5.2 Introduction to Polynomials 788


Chapter 5 Polynomials and Their Operations

34. a3 − a9 + 6a5 − a + 3 − a4

Part B: Evaluating Polynomials

35. Fill in the following chart:

36. Fill in the following chart:

Evaluate.

37. 2x − 3 , where x = 3

38. x 2 − 3x + 5 , where x = −2
1 1 1
39. − x+ , where x =−
2 3 3

1
40. −x 2 + 5x − 1 , where x = − 2

41. −2x 2 + 3x − 5 , where x = 0

5.2 Introduction to Polynomials 789


Chapter 5 Polynomials and Their Operations

42. 8x 5 − 27x 3 + 81x − 17 , where x = 0

43. y 3 − 2y + 1 , where y = −2

44. y 4 + 2y 2 − 32, where y = 2

45. a3 + 2a2 + a − 3, where a = −3

46. x 3 − x 2, where x = 5
3 1 3 2
47.
4
x2 − 2
x+ 6
, where x =− 3

5 1 1
48.
8
x2 − 4
x+ 2
, where x =4

49. x 2 y + xy 2 , where x = 2 and y = −3

50. 2a5 b − ab 4 + a2 b 2, where a = −1 and b = −2

51. a2 − b 2, where a = 5 and b = −6


3 1
52. a2 − b 2, where a = 4
and b =− 4

53. a3 − b 3, where a = −2 and b = 3

54. a3 + b 3, where a = 5 and b = −5


2
For each problem, evaluate b − 4ac, given the following values.

55. a = −1, b = 2, and c = −1


1
56. a = 2, b = −2 , and c = 2

57. a = 3, b = −5 , c = 0

58. a = 1, b = 0, and c = −4

5.2 Introduction to Polynomials 790


Chapter 5 Polynomials and Their Operations

1
59. a = ,b = −4 , and c = 2
4

60. a = 1, b = 5, and c = 6

The volume of a sphere in cubic units is given by the formula V = 43 πr3 , where r
is the radius. For each problem, calculate the volume of a sphere given the following
radii.

61. r = 3 centimeters

62. r = 1 centimeter

63. r = 1/2 feet

64. r = 3/2 feet

65. r = 0.15 in

66. r = 1.3 inches

The height in feet of a projectile launched vertically from the ground with an initial
velocity v 0 in feet per second is given by the formula h = −16t 2 + v 0 t, where
t represents time in seconds. For each problem, calculate the height of the projectile
given the following initial velocity and times.

67. v 0 = 64 feet/second , at times t = 0, 1, 2, 3, 4 seconds

68. v 0 = 80 feet/second , at times t = 0, 1, 2, 2.5, 3, 4, 5 seconds

The stopping distance of a car, taking into account an average reaction time, can be
estimated with the formula d = 0.05v 2 + 1.5 , where d is in feet and v is the
speed in miles per hour. For each problem, calculate the stopping distance of a car
traveling at the given speeds.

69. 20 miles per hour

70. 40 miles per hour

71. 80 miles per hour

5.2 Introduction to Polynomials 791


Chapter 5 Polynomials and Their Operations

72. 100 miles per hour

Part C: Polynomial Functions

2
Given the linear function f (x) = x + 6, evaluate each of the following.
3

73. f (−6)

74. f (−3)

75. f (0)

76. f (3)

77. Find x when f (x) = 10 .

78. Find x when f (x) = −4 .

Given the quadratic function f (x) = 2x 2 − 3x + 5 , evaluate each of the


following.

79. f (−2)

80. f (−1)

81. f (0)

82. f (2)

Given the cubic function g (x) = x 3 − x 2 + x − 1, evaluate each of the


following.

83. g(−2)

84. g (−1)

85. g (0)

86. g (1)

5.2 Introduction to Polynomials 792


Chapter 5 Polynomials and Their Operations

The height in feet of a projectile launched vertically from the ground with an initial
velocity of 128 feet per second is given by the function h(t) = −16t 2 + 128t ,
where t is in seconds. Calculate and interpret the following.

87. h(0)

88. h ( 12 )

89. h(1)

90. h(3)

91. h(4)

92. h(5)

93. h(7)

94. h(8)

Part D: Discussion Board Topics

95. Find and share some graphs of polynomial functions.

96. Explain how to convert feet per second into miles per hour.

97. Find and share the names of fourth-degree, fifth-degree, and higher
polynomials.

5.2 Introduction to Polynomials 793


Chapter 5 Polynomials and Their Operations

ANSWERS

1: Linear

3: Quadratic

5: Cubic

7: Binomial; degree 3

9: Trinomial; degree 5

11: Monomial; degree 5

13: Trinomial; degree 3

15: Trinomial; degree 5

17: Binomial; degree 1

19: Not a polynomial

21: Monomial; degree 0

23: Trinomial; degree 6

25: Monomial; degree 6

27: Binomial; degree 2

29: Polynomial; degree 14

31: 7x 2 − 6x + 1

33: x 7 − 5x 5 − x 3 − x 2 + x + 7

35:

5.2 Introduction to Polynomials 794


Chapter 5 Polynomials and Their Operations

37: 3

39: 1/2

41: −5

43: −3

45: −15

47: 7/6

49: 6

51: −11

53: −35

55: 0

57: 25

59: 14

61: 36π cubic centimeters

63: π/6 cubic feet

65: 0.014 cubic inches

5.2 Introduction to Polynomials 795


Chapter 5 Polynomials and Their Operations

67:

Time Height

t = 0 seconds h = 0 feet

t = 1 second h = 48 feet

t = 2 seconds h = 64 feet

t = 3 seconds h = 48 feet

t = 4 seconds h = 0 feet

69: 21.5 feet

71: 321.5 feet

73: 2

75: 6

77: x =6

79: 19

81: 5

83: −15

85: −1

87: The projectile is launched from the ground.

89: The projectile is 112 feet above the ground 1 second after launch.

91: The projectile is 256 feet above the ground 4 seconds after launch.

93: The projectile is 112 feet above the ground 7 seconds after launch.

5.2 Introduction to Polynomials 796


Chapter 5 Polynomials and Their Operations

5.3 Adding and Subtracting Polynomials

LEARNING OBJECTIVES

1. Add polynomials.
2. Subtract polynomials.
3. Add and subtract polynomial functions.

Adding Polynomials

Recall that we combine like terms, or terms with the same variable part, as a means
to simplify expressions. To do this, add the coefficients of the terms to obtain a
single term with the same variable part. For example,

Notice that the variable part, x 2 , does not change. This, in addition to the
commutative and associative properties of addition, allows us to add
polynomials21.

Example 1: Add: 3x + (4x − 5).

Solution: The property + (a + b) = a + b, which was derived using the


distributive property, allows us to remove the parentheses so that we can add like
terms.

21. The process of combining all


like terms of two or more Answer: 7x − 5
polynomials.

797
Chapter 5 Polynomials and Their Operations

Example 2: Add: (3x 2 + 3x + 5) + (2x 2 − x − 2).

Solution: Remove the parentheses and then combine like terms.

Answer: 5x 2 + 2x + 3

Example 3: Add: (−5x 2 y − 2xy 2 + 7xy) + (4x 2 y + 7xy 2 − 3xy) .

Solution: Remember that the variable parts have to be exactly the same before we
can add the coefficients.

Answer: −x 2 y + 5xy 2 + 4xy

It is common practice to present the terms of the simplified polynomial expression


in descending order based on their degree. In other words, we typically present
polynomials in standard form, with terms in order from highest to lowest degree.

5.3 Adding and Subtracting Polynomials 798


Chapter 5 Polynomials and Their Operations

Example 4: Add: (a − 4a3 + a5 − 8) + (−9a5 + a4 − 7a + 5 + a3 .)

Solution:

Answer: −8a5 + a4 − 3a3 − 6a − 3

Try this! Add: (6 − 5x 3 + x 2 − x) + (x 2 + x + 6x 3 − 1).

Answer: x 3 + 2x 2 + 5

Video Solution

(click to see video)


Subtracting Polynomials

When subtracting polynomials22, we see that the parentheses become very


important. Recall that the distributive property allowed us to derive the following:

In other words, when subtracting an algebraic expression, we remove the


parentheses by subtracting each term.

Example 5: Subtract: 10x − (3x + 5).


22. The process of subtracting all
the terms of one polynomial
from another and combining
like terms.

5.3 Adding and Subtracting Polynomials 799


Chapter 5 Polynomials and Their Operations

Solution: Subtract each term within the parentheses and then combine like terms.

Answer: 7x − 5

Subtracting a quantity is equivalent to multiplying it by −1.

Example 6: Subtract: (3x 2 + 3x + 5) − (2x 2 − x − 2).

Solution: Distribute the −1, remove the parentheses, and then combine like terms.

Answer: x 2 + 4x + 7

Multiplying the terms of a polynomial by −1 changes all the signs.

Example 7: Subtract: (−5x 3 − 2x 2 + 7) − (4x 3 + 7x 2 − 3x + 2).

Solution: Distribute the −1, remove the parentheses, and then combine like terms.

5.3 Adding and Subtracting Polynomials 800


Chapter 5 Polynomials and Their Operations

Answer: −9x 3 − 9x 2 + 3x + 5

Example 8: Subtract 6x 2 − 3x − 1 from 2x 2 + 5x − 2.

difference correctly. First, write the quantity (2x 2 + 5x − 2); from this, subtract
Solution: Since subtraction is not commutative, we must take care to set up the

the quantity (6x 2 − 3x − 1).

Answer: −4x 2 + 8x − 1

Example 9: Simplify: (2x 2 − 3x + 5) − (x 2 − 3x + 1) + (5x 2 − 4x − 8).

Solution: Apply the distributive property, remove the parentheses, and then
combine like terms.

5.3 Adding and Subtracting Polynomials 801


Chapter 5 Polynomials and Their Operations

Answer: 6x 2 − 4x − 4

Try this! Subtract: (8x 2 y − 5xy 2 + 6) − (x 2 y + 2xy 2 − 1).

Answer: 7x 2 y − 7xy 2 + 7

Video Solution

(click to see video)


Adding and Subtracting Polynomial Functions

We use function notation to indicate addition and subtraction of functions as


follows:

Addition of functions: (f + g) (x) = f (x) + g(x)

Subtraction of functions: (f − g) (x) = f (x) − g(x)

When using function notation, be careful to group the entire function and add or
subtract accordingly.

Example 10: Calculate: (f + g)(x), given f (x) = −x 2 − 3x + 5 and


g (x) = 3x 2 + 2x + 1.

Solution: The notation (f + g)(x) indicates that you should add the functions
f (x) + g(x) and collect like terms.

5.3 Adding and Subtracting Polynomials 802


Chapter 5 Polynomials and Their Operations

Answer: (f + g)(x) = 2x 2 − x + 6

Example 11: Calculate: (f − g)(x), given f (x) = 2x − 3 and


g (x) = −2x 2 + 2x + 5.

Solution: The notation (f − g)(x) indicates that you should subtract the functions
f (x) − g(x):

Answer: (f − g)(x) = 2x 2 − 8

We may be asked to evaluate the sum or difference of two functions. We have the
option to first find the sum or difference and use the resulting function to evaluate
for the given variable, or to first evaluate each function and then find the sum or
difference.

Example 12: Calculate: (f − g)(5), given f (x) = x 2 + x − 7 and g (x) = 4x + 10.

Solution: First, find (f − g) (x) = f (x) − g(x).

5.3 Adding and Subtracting Polynomials 803


Chapter 5 Polynomials and Their Operations

Therefore,

Next, substitute 5 for the variable x.

Answer: (f − g) (5) = −7

Alternate Solution: Since (f − g)(5) = f (5) − g(5), we can find f (5) and g(5) and
then subtract the results.

Therefore, we have

Answer: (f − g) (5) = −7

5.3 Adding and Subtracting Polynomials 804


Chapter 5 Polynomials and Their Operations

KEY TAKEAWAYS

• When adding polynomials, remove the associated parentheses and then


combine like terms.
• When subtracting polynomials, distribute the −1 and subtract all the

• The notation (f + g) (x) indicates that you add the functions.


terms before removing the parentheses and combining like terms.

• The notation (f − g) (x) indicates that you subtract the functions.

5.3 Adding and Subtracting Polynomials 805


Chapter 5 Polynomials and Their Operations

TOPIC EXERCISES

Part A: Addition of Polynomials

Add.

1. (2x + 1) + (−x + 7)

2. (−6x + 5) + (3x − 1)

3. ( x + 12 ) + ( 13 x − 2)
2
3

4. ( x − 34 ) + ( 56 x + 18 )
1
3

5. (2x + 1) + (x − 3) + (5x − 2)

6. (2x − 8) + (−3x 2 + 7x − 5)

7. (x 2 − 3x + 7) + (3x 2 − 8x − 5)

8. (−5x 2 − 1 + x) + (−x + 7x 2 − 9)

9. ( x + 16 ) + (− x − 1)
1 1 3 2
2
x2 − 3 2
x2 + 3

10. (− x − 6) + (2x 2 − x + 52 )
3 1 3
5
x2 + 4 8

11. (x 2 + 5) + (3x 2 − 2x + 1) + (x 2 + x − 3)

12. (a3 − a2 + a − 8) + (a3 + a2 + 6a − 2)

13. (a3 − 8) + (−3a3 + 5a2 − 2)

14. (4a5 + 5a3 − a) + (3a4 − 2a2 + 7)

15. (2x 2 + 5x − 12) + (7x − 5)

5.3 Adding and Subtracting Polynomials 806


Chapter 5 Polynomials and Their Operations

16. (3x + 5) + (x 2 − x + 1) + (x 3 + 2x 2 − 3x + 6)

17. (6x 5 − 7x 3 + x 2 − 15) + (x 4 + 2x 3 − 6x + 12)

18. (1 + 7x − 5x 3 + 4x 4 ) + (−3x 3 + 5 − x 2 + x)

19. (x 2 y 2 − 7xy + 7) + (4x 2 y 2 − 3xy − 8)

20. (x 2 + xy − y 2 ) + (7x 2 − 5xy + 2y 2 )

21. (2x 2 + 3xy − 7y 2 ) + (−5x 2 − 3xy + 8y 2 )

22. (a2 b − 100) + (2a2 b 2 − 3ab + 20)


2

23. (ab − 3a2 b + ab − 3) + (−2a2 b + ab 2 − 7ab − 1)


2

24. (10a2 b − 7ab + 8ab 2 ) + (6a2 b − ab + 5ab 2 )

25. Find the sum of 2x +8 and 7x − 1.


1 1 1 1
26. Find the sum of
3
x− 5
and
6
x+ 10
.

27. Find the sum of x 2 − 10x + 8 and 5x 2 − 2x − 6 .

28. Find the sum of a2 − 5a + 10 and −9a2 + 7a − 11.

29. Find the sum of x 2 y 2 − xy + 6 and x 2 y 2 + xy − 7 .

30. Find the sum of x 2 − 9xy + 7y 2 and −3x 2 − 3xy + 7y 2 .

Part B: Subtraction of Polynomials

Subtract.

31. (5x − 3) − (2x − 1)

5.3 Adding and Subtracting Polynomials 807


Chapter 5 Polynomials and Their Operations

32. (−4x + 1) − (7x + 10)

33. ( x − 34 ) − ( 34 x + 18 )
1
4

34. (− x + 37 ) − ( 25 x − 32 )
3
5

35. (x 2 + 7x − 5) − (4x 2 − 5x + 1)

36. (−6x 2 + 3x − 12) − (−6x 2 + 3x − 12)

37. (−3x 3 + 4x − 8) − (−x 2 + 4x + 10)

38. ( x − 34 ) − ( 32 x 2 − x + 12 )
1 1 1
2
x2 + 3 6

39. ( x − 13 ) − ( 13 x 2 + x + 59 )
5 1 3
9
x2 + 5 10

40. (a3 − 4a2 + 3a − 7) − (7a3 − 2a2 − 6a + 9)

41. (3a3 + 5a2 − 2) − (a3 − a + 8)

42. (5x 5 + 4x 3 + x 2 − 6) − (4x 4 − 3x 3 − x + 3)

43. (3 − 5x − x 3 + 5x 4 ) − (−5x 3 + 2 − x 2 − 7x)

44. (x 5 − 6x 3 + 9x) − (4x 4 + 2x 2 − 5)

45. (2x 2 y 2 − 4xy + 9) − (3x 2 y 2 − 3xy − 5)

46. (x 2 + xy − y 2 ) − (x 2 + xy − y 2 )

47. (2x 2 + 3xy − 7y 2 ) − (−5x 2 − 3xy + 8y 2 )

48. (ab − 3a2 b + ab − 3) − (−2a2 b + ab 2 − 7ab − 1)


2

5.3 Adding and Subtracting Polynomials 808


Chapter 5 Polynomials and Their Operations

49. (10a2 b − 7ab + 8ab 2 ) − (6a2 b − ab + 5ab 2 )

50. (10a2 b + 5ab − 6) − (5a2 b 2 + 5ab − 6)


2

51. Subtract 3x +1 from 5x − 9.

52. Subtract x 2 − 5x + 10 from x 2 + 5x − 5 .

53. Find the difference of 3x −7 and 8x + 6.

54. Find the difference of 2x 2 + 3x − 5 and x 2 − 9.

55. The cost in dollars of producing customized coffee mugs with a company
logo is given by the formula C = 150 + 0.10x , where x is the number of
cups produced. The revenue from selling the cups in the company store is
given by R = 10x − 0.05x 2 , where x is the number of units sold.

a. Find a formula for the profit. (profit = revenue − cost)

b. Find the profit from producing and selling 100 mugs in the company
store.

56. The cost in dollars of producing sweat shirts is given by the formula
C = 10q + 1200 , where C is the cost and q represents the quantity
produced. The revenue generated by selling the sweat shirts for $37 each is
given by R = 37q , where q represents the quantity sold. Determine the
profit generated if 125 sweat shirts are produced and sold.

57. The outer radius of a washer is 3 times the radius of the hole.

a. Derive a formula for the area of the face of the washer.

b. What is the area of the washer if the hole has a diameter of 10


millimeters?

5.3 Adding and Subtracting Polynomials 809


Chapter 5 Polynomials and Their Operations

58. Derive a formula for the surface area of the following rectangular solid.

Part C: Addition and Subtraction of Polynomial

Simplify.

59. (2x + 3) − (5x − 8) + (x − 7)

60. (3x − 5) − (7x − 11) − (5x + 2)

61. (3x − 2) − (4x − 1) + (x + 7)

62. (5x − 3) − (2x + 1) − (x − 1)

63. (5x 2 − 3x + 2) − (x 2 + x − 4) + (7x 2 − 2x − 6)

64. (−2x 3 + x 2 − 8) − (3x 2 + x − 6) − (2x − 1)

65. (2x − 7) − (x 2 + 3x − 7) + (6x − 1)

66. (6x 2 − 10x + 13) + (4x 2 − 9) − (9 − x 2 )

67. (a2 − b 2 ) − (2a2 + 3ab − 4b 2 ) + (5ab − 1)

68. (a2 − 3ab + b 2 ) − (a2 + b 2 ) − (3ab − 5)

69. ( 12 x2 − 3
4
x + 14 ) − ( 32 x − 34 ) + ( 54 x − 12 )

70. ( 95 x2 − 1
3
x + 2) − ( 103 x 2 − 45 ) − (x + 52 )

Part D: Addition and Subtraction of Polynomial Functions

5.3 Adding and Subtracting Polynomials 810


Chapter 5 Polynomials and Their Operations

Find (f + g) (x) and (f − g) (x) , given the following functions.

71. f (x) = 4x − 1 and g (x) = −3x + 1

72. f (x) = −x + 5 and g (x) = 2x − 3

73. f (x) = 3x 2 − 5x + 7 and g (x) = −2x 2 + 5x − 1

74. f (x) = x 3 + 2x 2 − 6x + 2 and g (x) = 2x 3 + 2x 2 − 5x − 1


1 1 1 3 1
75. f (x) = 2
x+ 3
and g (x) = 5
x2 − 2
x+ 6

1 2 1
76. f (x) = x 2 − 5x + 3
and g (x) = 3
x2 − x − 2

Given f (x) = 2x − 3 and g (x) = x 2 + 3x − 1 , find the following.

77. (f + g) (x)

78. (g + f ) (x)

79. (f − g) (x)

80. (g − f ) (x)

81. (g + g) (x)

82. (f + g) (3)

83. (f + g) (−2)

84. (f + g) (0)

85. (f − g) (0)

86. (f − g) (−2)

5.3 Adding and Subtracting Polynomials 811


Chapter 5 Polynomials and Their Operations

87. (g − f ) (−2)

88. (g − f ) ( 12 )

Given f (x) = 5x 2 − 3x + 2 and g (x) = 2x 2 + 6x − 4 , find the


following.

89. (f + g) (x)

90. (g + f ) (x)

91. (f − g) (x)

92. (g − f ) (x)

93. (f + g) (−2)

94. (f − g) (−2)

95. (f + g) (0)

96. (f − g) (0)

5.3 Adding and Subtracting Polynomials 812


Chapter 5 Polynomials and Their Operations

ANSWERS

1: x +8
3
3: x − 2

5: 8x −4

7: 4x 2 − 11x + 2
1 5
9: −x 2 + x−
3 6

11: 5x 2 −x+3

13: −2a3 + 5a2 − 10

15: 2x 2 + 12x − 17

17: 6x 5 + x 4 − 5x 3 + x 2 − 6x − 3

19: 5x 2 y 2 − 10xy − 1

21: −3x 2 + y2

23: −5a2 b + 2ab 2 − 6ab − 4

25: 9x +7

27: 6x 2 − 12x + 2

29: 2x 2 y 2 −1

31: 3x −2
1 7
33: − x−
2 8

35: −3x 2 + 12x − 6

5.3 Adding and Subtracting Polynomials 813


Chapter 5 Polynomials and Their Operations

37: −3x 3 + x 2 − 18
2 1 8
39:
9
x2 − 10
x− 9

41: 2a3 + 5a2 + a − 10

43: 5x 4 + 4x 3 + x 2 + 2x + 1

45: −x 2 y 2 − xy + 14

47: 7x 2 + 6xy − 15y 2

49: 4a2 b + 3ab 2 − 6ab

51: 2x − 10

53: −5x − 13

55: a. P = −0.05x 2 + 9.9x − 150 ; b. $340

57: a. A = 8πr2 ; b. 628.32 square millimeters

59: −2x +4

61: 6

63: 11x 2 − 6x

65: −x 2 + 5x − 1

67: −a2 + 2ab + 3b 2 − 1

69: 1
2
x2 − x + 1
2

71: (f + g) (x) = x and (f − g) (x) = 7x − 2

73: (f + g) (x) = x 2 + 6 and (f − g) (x) = 5x 2 − 10x + 8

5.3 Adding and Subtracting Polynomials 814


Chapter 5 Polynomials and Their Operations

75: (f
+ g) (x) = 15 x 2 − x + 12 and
(f − g) (x) = − 5 x + 2x + 6
1 2 1

77: (f + g) (x) = x 2 + 5x − 4

79: (f − g) (x) = −x 2 − x − 2

81: (g + g) (x) = 2x 2 + 6x − 2

83: (f + g) (−2) = −10

85: (f − g) (0) = −2

87: (g − f ) (−2) = 4

89: (f + g) (x) = 7x 2 + 3x − 2

91: (f − g) (x) = 3x 2 − 9x + 6

93: (f + g) (−2) = 20

95: (f + g) (0) = −2

5.3 Adding and Subtracting Polynomials 815


Chapter 5 Polynomials and Their Operations

5.4 Multiplying Polynomials

LEARNING OBJECTIVES

1. Multiply a polynomial by a monomial.


2. Multiply a polynomial by a binomial.
3. Multiply a polynomial by any size polynomial.
4. Recognize and calculate special products.
5. Multiply polynomial functions.

Multiplying by a Monomial

Recall the product rule for exponents: if m and n are positive integers, then

In other words, when multiplying two expressions with the same base, add the
exponents. This rule applies when multiplying a monomial by a monomial. To find
the product of monomials, multiply the coefficients and add the exponents of
variable factors with the same base. For example,

To multiply a polynomial by a monomial, apply the distributive property and then


simplify each term.

Example 1: Multiply: −5x (4x − 2).

816
Chapter 5 Polynomials and Their Operations

Solution: In this case, multiply the monomial, −5x , by the binomial, 4x − 2 . Apply
the distributive property and then simplify.

Answer: −20x 2 + 10x

Example 2: Multiply: 2x 2 (3x 2 − 5x + 1).

Solution: Apply the distributive property and then simplify.

Answer: 6x 4 − 10x 3 + 2x 2

Example 3: Multiply: −3ab2 (a2 b3 + 2a3 b − 6ab − 4).

Solution:

5.4 Multiplying Polynomials 817


Chapter 5 Polynomials and Their Operations

Answer: −3a3 b5 − 6a4 b3 + 18a2 b3 + 12ab2

To summarize, multiplying a polynomial by a monomial involves the distributive


property and the product rule for exponents. Multiply all of the terms of the
polynomial by the monomial. For each term, multiply the coefficients and add
exponents of variables where the bases are the same.

Try this! Multiply: −5x 2 y (2xy 2 − 3xy + 6x 2 y − 1) .

Answer: −10x 3 y 3 + 15x 3 y 2 − 30x 4 y 2 + 5x 2 y

Video Solution

(click to see video)


Multiplying by a Binomial

In the same way that we used the distributive property to find the product of a
monomial and a binomial, we will use it to to find the product of two binomials.

Here we apply the distributive property multiple times to produce the final result.
This same result is obtained in one step if we apply the distributive property to a
and b separately as follows:

5.4 Multiplying Polynomials 818


Chapter 5 Polynomials and Their Operations

This is often called the FOIL23 method. We add the products of the first terms of
each binomial ac, the outer terms ad, the inner terms bc, and finally the last terms
bd. This mnemonic device only works for products of binomials; hence it is best to
just remember that the distributive property applies.

Example 4: Multiply: (2x + 3) (5x − 2).

Solution: Distribute 2x and then distribute 3.

Simplify by combining like terms.

Answer: 10x 2 + 11x − 6

Example 5: Multiply: ( 12 x − 14 ) ( 12 x + 14. )

Solution: Distribute 1
2
x and then distribute − 14.

23. When multiplying binomials


we apply the distributive
property multiple times in
such a way as to multiply the
first terms, outer terms, inner
terms, and last terms.

5.4 Multiplying Polynomials 819


Chapter 5 Polynomials and Their Operations

Answer: 1
4
x2 − 1
16

Example 6: Multiply: (3y 2 − 1) (2y + 1).

Solution:

Answer: 6y 3 + 3y 2 − 2y − 1

After applying the distributive property, combine any like terms.

Example 7: Multiply: (x 2 − 5) (3x 2 − 2x + 2).

Solution: After multiplying each term of the trinomial by x 2 and −5, simplify.

Answer: 3x 4 − 2x 3 − 13x 2 + 10x − 10

5.4 Multiplying Polynomials 820


Chapter 5 Polynomials and Their Operations

Example 8: Multiply: (2x − 1)3 .

Solution: Perform one product at a time.

Answer: 8x 3 − 12x 2 + 6x − 1

At this point, it is worth pointing out a common mistake:

The confusion comes from the product to a power rule of exponents, where we
apply the power to all factors. Since there are two terms within the parentheses,
that rule does not apply. Care should be taken to understand what is different in the
following two examples:

5.4 Multiplying Polynomials 821


Chapter 5 Polynomials and Their Operations

Try this! Multiply: (2x − 3) (7x 2 − 5x + 4).

Answer: 14x 3 − 31x 2 + 23x − 12

Video Solution

(click to see video)


Product of Polynomials

When multiplying polynomials, we apply the distributive property many times.


Multiply all of the terms of each polynomial and then combine like terms.

Example 9: Multiply: (2x 2 + x − 3) (x 2 − 2x + 5).

Solution: Multiply each term of the first trinomial by each term of the second
trinomial and then combine like terms.

Aligning like terms in columns, as we have here, aids in the simplification process.

Answer: 2x 4 − 3x 3 + 5x 2 + 11x − 15

Notice that when multiplying a trinomial by a trinomial, we obtain nine terms


before simplifying. In fact, when multiplying an n-term polynomial by an m-term
polynomial, we will obtain n × m terms.

In the previous example, we were asked to multiply and found that

5.4 Multiplying Polynomials 822


Chapter 5 Polynomials and Their Operations

Because it is easy to make a small calculation error, it is a good practice to trace


through the steps mentally to verify that the operations were performed correctly.
Alternatively, we can check by evaluating24 any value for x in both expressions to
verify that the results are the same. Here we choose x = 2:

Because the results could coincidentally be the same, a check by evaluating does not
necessarily prove that we have multiplied correctly. However, after verifying a few
values, we can be fairly confident that the product is correct.

Try this! Multiply: (x 2 − 2x − 3) .


2

Answer: x 4 − 4x 3 − 2x 2 + 12x + 9

Video Solution
24. We can be fairly certain that (click to see video)
we have multiplied the
polynomials correctly if we Special Products
check that a few values
evaluate to the same results in
In this section, the goal is to recognize certain special products that occur often in
the original expression and in
the answer. our study of algebra. We will develop three formulas that will be very useful as we

5.4 Multiplying Polynomials 823


Chapter 5 Polynomials and Their Operations

move along. The three should be memorized. We begin by considering the following
two calculations:

This leads us to two formulas that describe perfect square trinomials25:

We can use these formulas to quickly square a binomial.

Example 10: Multiply: (3x + 5) .


2

Solution: Here a = 3x and b = 5. Apply the formula:

Answer: 9x 2 + 30x + 25
25. The trinomials obtained by

(a + b) = a + 2ab + b
squaring the binomials
2 2 2

(a − b) = a − 2ab + b . This process should become routine enough to be performed mentally.


and
2 2 2

5.4 Multiplying Polynomials 824


Chapter 5 Polynomials and Their Operations

Example 11: Multiply: (x − 4)2 .

Solution: Here a = x and b = 4. Apply the appropriate formula as follows:

Answer: x 2 − 8x + 16

Our third special product follows:

This product is called difference of squares26:

The binomials (a + b) and (a − b) are called conjugate binomials27. Therefore,

26. a2 − b = (a + b) (a − b), product is itself a binomial.


when conjugate binomials are multiplied, the middle term eliminates, and the
2

where a and b represent


algebraic expressions.

27. The binomials (a + b) and


(a − b).

5.4 Multiplying Polynomials 825


Chapter 5 Polynomials and Their Operations

Example 12: Multiply: (7x + 4) (7x − 4).

Solution:

Answer: 49x 2 − 16

Try this! Multiply: (−5x + 2) .


2

Answer: 25x 2 − 20x + 4

Video Solution

(click to see video)


Multiplying Polynomial Functions

We use function notation to indicate multiplication as follows:

Multiplication of functions: (f ⋅ g) (x) = f (x) ⋅ g (x)

Example 13: Calculate: (f ⋅ g) (x), given f (x) = 5x 2 and g (x) = −x 2 + 2x − 3.

Solution: Multiply all terms of the trinomial by the monomial function f (x).

5.4 Multiplying Polynomials 826


Chapter 5 Polynomials and Their Operations

Answer: (f ⋅ g) (x) = −5x 4 + 10x 3 − 15x 2

Example 14: Calculate: (f ⋅ g) (−1), given f (x) = −x + 3 and


g (x) = 4x 2 − 3x + 6.

Solution: First, determine (f ⋅ g) (x).

We have

Next, substitute −1 for the variable x.

5.4 Multiplying Polynomials 827


Chapter 5 Polynomials and Their Operations

Answer: (f ⋅ g) (−1) = 52

Because (f ⋅ g) (−1) = f (−1) ⋅ g (−1), we could alternatively calculate f (−1) and

we were asked to evaluate multiple values for the function (f ⋅ g) (x), it would be
g(−1) separately and then multiply the results (try this as an exercise). However, if

best to first determine the general form, as we have in the previous example.

KEY TAKEAWAYS

• To multiply a polynomial by a monomial, apply the distributive property


and then simplify each of the resulting terms.
• To multiply polynomials, multiply each term in the first polynomial
with each term in the second polynomial. Then combine like terms.
• The product of an n-term polynomial and an m-term polynomial results
in an m × n term polynomial before like terms are combined.
• Check results by evaluating values in the original expression and in your
answer to verify that the results are the same.
• Use the formulas for special products to quickly multiply binomials that
occur often in algebra.

5.4 Multiplying Polynomials 828


Chapter 5 Polynomials and Their Operations

TOPIC EXERCISES

Part A: Product of a Monomial and a Polynomial

Multiply.

1. 5x (−3x 2 y)

2. (−2x 3 y 2 ) (−3xy )
4

1
3.
2
(4x − 3)

( 3 x − 6)
3 2
4. −
4

5. 3x(5x − 2)

6. −4x(2x − 1)

7. x 2 (3x + 2)

8. −6x 2 (5x + 3)

9. 2ab(4a − 2b)

10. 5a2 b (a2 − b2 )

11. 6x 2 y 3 (−3x y + xy )
3 2

(−5ab + 6a b)
3 3 2
12. 3ab

1
13. −
2
x 2 y(4xy − 10)

14. −3x 4 y 2 (3x y )


8 3

15. 2x 2 (−5x ) (3x )


3 4

5.4 Multiplying Polynomials 829


Chapter 5 Polynomials and Their Operations

16. 4ab (a2 b c) (a4 b 2 c4 )


3

17. −2 (5x 2 − 3x + 4)

(25x − 50xy + 5y )
4 2 2
18.
5

19. 3x (5x 2 − 2x + 3)

20. −x (x 2 + x − 1)

21. x 2 (3x − 5x − 7)
2

22. x 3 (−4x − 7x + 9)
2

23.
1
4
x 4 (8x 3 − 2x 2 + 1
2
x − 5)

24. −
1
3
x 3 ( 32 x 5 − 2
3
x3 + 9
2
x − 1)

25. a2 b (a2 − 3ab + b 2 )

26. 6a2 bc3 (2a − 3b + c )


2

27.
2
3
xy 2 (9x 3 y − 27xy + 3xy 3 )

28. −3x 2 y 2 (12x − 10xy − 6y )


2 2

29. Find the product of 3x and 2x 2 − 3x + 5 .


1
30. Find the product of −8y and y 2 − 2y + .
2

31. Find the product of −4x and x 4 − 3x 3 + 2x 2 − 7x + 8.

32. Find the product of 3xy 2 and −2x 2 y + 4xy − xy 2 .

Part B: Product of a Binomial and a Polynomial

5.4 Multiplying Polynomials 830


Chapter 5 Polynomials and Their Operations

Multiply.

33. (3x − 2) (x + 4)

34. (x + 2) (x − 3)

35. (x − 1) (x + 1)

36. (3x − 1) (3x + 1)

37. (2x − 5) (x + 3)

38. (5x − 2) (3x + 4)

39. (−3x + 1) (x − 1)

40. (x + 5) (−x + 1)

41. (y − 23 ) (y + 23 )

42. ( x + 13 ) ( 32 x − 23 )
1
2

43. ( x + 15 ) ( 14 x + 25 )
3
4

44. ( ) (5 x − 2)
1 3 3 5
5
x+ 10

45. (y 2 − 2) (y + 2)

46. (y 3 − 1) (y 2 + 2)

47. (a2 − b 2 ) (a2 + b 2 )

48. (a2 − 3b)


2

49. (x − 5) (2x 2 + 3x + 4)

5.4 Multiplying Polynomials 831


Chapter 5 Polynomials and Their Operations

50. (3x − 1) (x 2 − 4x + 7)

51. (2x − 3) (4x 2 + 6x + 9)

52. (5x + 1) (25x 2 − 5x + 1)

53. (x − 12 ) (3x 2 + 4x − 1)

54. ( x − 14 ) (3x 2 + 9x − 3)
1
3

55. (x + 3) 3

56. (x − 2) 3

57. (3x − 1) 3

58. (2x + y)
3

59. (5x − 2) (2x 3 − 4x 2 + 3x − 2)

60. (x 2 − 2) (x 3 − 2x 2 + x + 1)

Part C: Product of Polynomials

Multiply.

61. (x 2 − x + 1) (x 2 + 2x + 1)

62. (3x 2 − 2x − 1) (2x 2 + 3x − 4)

63. (2x 2 − 3x + 5) (x 2 + 5x − 1)

64. (a + b + c) (a − b − c)

65. (a + 2b − c)
2

5.4 Multiplying Polynomials 832


Chapter 5 Polynomials and Their Operations

66. (x + y + z)
2

67. (x − 3) 4

68. (x + y)
4

69. Find the volume of a rectangular solid with sides measuring x , x + 2,


and x + 4 units.

70. Find the volume of a cube where each side measures x − 5 units.

Part D: Special Products

Multiply.

71. (x + 2) 2

72. (x − 3) 2

73. (2x + 5)
2

74. (3x − 7) 2

75. (−x + 2) 2

76. (−9x + 1) 2

77. (a + 6)
2

78. (2a − 3b)


2

79. ( x + 34 )
2 2
3

80. ( x − 35 )
1 2
2

5.4 Multiplying Polynomials 833


Chapter 5 Polynomials and Their Operations

81. (x 2 + 2)
2

82. (x 2 + y2)
2

83. (x + 4) (x − 4)

84. (2x + 1) (2x − 1)

85. (5x + 3) (5x − 3)

86. ( x − 13 ) ( 15 x + 13 )
1
5

87. ( x + 25 ) ( 32 x − 25 )
3
2

88. (2x − 3y) (2x + 3y)

89. (4x − y) (4x + y)

90. (a3 − b 3 ) (a3 + b 3 )

91. A box is made by cutting out the corners and folding up the edges of a
square piece of cardboard. A template for a cardboard box with a height of 2
inches is given. Find a formula for the volume, if the initial piece of
cardboard is a square with sides measuring x inches.

92. A template for a cardboard box with a height of x inches is given. Find a
formula for the volume, if the initial piece of cardboard is a square with
sides measuring 12 inches.

5.4 Multiplying Polynomials 834


Chapter 5 Polynomials and Their Operations

Part E: Multiplying Polynomial Functions

For each problem, calculate (f ⋅ g) (x) , given the functions.

93. f (x) = 8x and g (x) = 3x − 5

94. f (x) = x 2 and g (x) = −5x + 1

95. f (x) = x − 7 and g (x) = 6x − 1

96. f (x) = 5x + 3 and g (x) = x 2 + 2x − 3

97. f (x) = x 2 + 6x − 3 and g (x) = 2x 2 − 3x + 5

98. f (x) = 3x 2 − x + 1 and g (x) = −x 2 + 2x − 1

Given f (x) = 2x − 3 and g (x) = 3x − 1 , find the following.

99. (f ⋅ g) (x)

100. (g ⋅ f ) (x)

101. (f ⋅ g) (0)

102. (f ⋅ g) (−1)

103. (f ⋅ g) (1)

104. (f ⋅ g) ( 12 )

Given f (x) = 5x − 1 and g (x) = 2x 2 − 4x + 5 , find the following.

5.4 Multiplying Polynomials 835


Chapter 5 Polynomials and Their Operations

105. (f ⋅ g) (x)

106. (g ⋅ f ) (x)

107. (f ⋅ g) (0)

108. (f ⋅ g) (−1)

109. (f ⋅ g) (1)

110. (f ⋅ g) ( 12 )

111. (f ⋅ f ) (x)

112. (g ⋅ g) (x)

Part F: Discussion Board Topics

113. Explain why (x + y) ≠ x 2 + y 2.


2

114. Explain how to quickly multiply a binomial with its conjugate. Give an
example.

115. What are the advantages and disadvantages of using the mnemonic
device FOIL?

5.4 Multiplying Polynomials 836


Chapter 5 Polynomials and Their Operations

ANSWERS

1: −15x 3 y

3
3: 2x − 2

5: 15x 2 − 6x

7: 3x 3 + 2x 2

9: 8a2 b − 4ab 2

11: −18x 5 y 4 + 6x 3 y 5

13: −2x 3 y 2 + 5x 2 y

15: −30x 9

17: −10x 2 + 6x − 8

19: 15x 3 − 6x 2 + 9x

21: 3x 4 − 5x 3 − 7x 2
1 1 5
23: 2x 7 − x6 + x5 − x4
2 8 4

25: a4 b − 3a3 b 2 + a2 b 3

27: 6x 4 y 3 − 18x 2 y 3 + 2x 2 y 5

29: 6x 3 − 9x 2 + 15x

31: −4x 5 + 12x 4 − 8x 3 + 28x 2 − 32x

33: 3x 2 + 10x − 8

5.4 Multiplying Polynomials 837


Chapter 5 Polynomials and Their Operations

35: x 2 −1

37: 2x 2 + x − 15

39: −3x 2 + 4x − 1
4
41: y 2 − 9

3 7 2
43:
16
x2 + 20
x+ 25

45: y 3 + 2y 2 − 2y − 4

47: a4 − b4

49: 2x 3 − 7x 2 − 11x − 20

51: 8x 3 − 27
5 1
53: 3x 3 + x 2 − 3x +
2 2

55: x 3 + 9x 2 + 27x + 27

57: 27x 3 − 27x 2 + 9x − 1

59: 10x 4 − 24x 3 + 23x 2 − 16x + 4

61: x 4 + x3 + x + 1

63: 2x 4 + 7x 3 − 12x 2 + 28x − 5

65: a2 + 4ab − 2ac + 4b 2 − 4bc + c2

67: x 4 − 12x 3 + 54x 2 − 108x + 81

69: x 3 + 6x 2 + 8x

5.4 Multiplying Polynomials 838


Chapter 5 Polynomials and Their Operations

71: x 2 + 4x + 4

73: 4x 2 + 20x + 25

75: x 2 − 4x + 4

77: a2 + 12a + 36
4 9
79:
9
x2 + x + 16

81: x 4 + 4x 2 + 4

83: x 2 − 16

85: 25x 2 −9
9 4
87:
4
x2 − 25

89: 16x 2 − y2

91: V = 2x 2 − 16x + 32 cubic inches

93: (f ⋅ g) (x) = 24x 2 − 40x

95: (f ⋅ g) (x) = 6x 2 − 43x + 7

97: (f ⋅ g) (x) = 2x 4 + 9x 3 − 19x 2 + 39x − 15

99: (f ⋅ g) (x) = 6x 2 − 11x + 3

101: (f ⋅ g) (0) = 3

103: (f ⋅ g) (1) = −2

105: (f ⋅ g) (x) = 10x 3 − 22x 2 + 29x − 5

5.4 Multiplying Polynomials 839


Chapter 5 Polynomials and Their Operations

107: (f ⋅ g) (0) = −5

109: (f ⋅ g) (1) = 12

111: (f ⋅ f ) (x) = 25x 2 − 10x + 1

5.4 Multiplying Polynomials 840


Chapter 5 Polynomials and Their Operations

5.5 Dividing Polynomials

LEARNING OBJECTIVES

1. Divide by a monomial.
2. Divide by a polynomial using the division algorithm.
3. Divide polynomial functions.

Dividing by a Monomial

Recall the quotient rule for exponents: if x is nonzero and m and n are positive
integers, then

In other words, when dividing two expressions with the same base, subtract the
exponents. This rule applies when dividing a monomial by a monomial. In this
section, we will assume that all variables in the denominator are nonzero.

28y 3
Example 1: Divide: 7y .

Solution: Divide the coefficients and subtract the exponents of the variable y.

Answer: 4y 2

841
Chapter 5 Polynomials and Their Operations

24x 7 y 5
Example 2: Divide: .
8x 3 y 2

Solution: Divide the coefficients and apply the quotient rule by subtracting the
exponents of the like bases.

Answer: 3x 4 y 3

When dividing a polynomial by a monomial, we may treat the monomial as a


common denominator and break up the fraction using the following property:

Applying this property results in terms that can be treated as quotients of


monomials.

−5x 4 +25x 3 −15x 2


Example 3: Divide: .
5x 2

Solution: Break up the fraction by dividing each term in the numerator by the
monomial in the denominator and then simplify each term.

5.5 Dividing Polynomials 842


Chapter 5 Polynomials and Their Operations

Answer: −x 2 + 5x − 3

Check your division by multiplying the answer, the quotient28, by the monomial in
the denominator, the divisor29, to see if you obtain the original numerator, the
dividend30.

9a4 b−7a3 b2 +3a2 b


Example 4: Divide: .
−3a2 b

Solution:

28. The result after dividing.

29. The denominator of a quotient.

30. The numerator of a quotient.

5.5 Dividing Polynomials 843


Chapter 5 Polynomials and Their Operations

Answer: −3a2 + 7
3
ab − 1. The check is optional and is left to the reader.

Try this! Divide: (16x 5 − 8x 4 + 5x 3 + 2x 2 ) ÷ (2x 2 ).

Answer: 8x 3 − 4x 2 + 5
2
x+1

Video Solution

(click to see video)


Dividing by a Polynomial

The same technique outlined for dividing by a monomial does not work for
polynomials with two or more terms in the denominator. In this section, we will
outline a process called polynomial long division31, which is based on the division
algorithm for real numbers. For the sake of clarity, we will assume that all
expressions in the denominator are nonzero.

x 3 +3x 2 −8x−4
Example 5: Divide: x−2
.

31. The process of dividing two


polynomials using the division Solution: Here x − 2 is the divisor and x 3 + 3x 2 − 8x − 4 is the dividend.
algorithm.

5.5 Dividing Polynomials 844


Chapter 5 Polynomials and Their Operations

Step 1: To determine the first term of the quotient, divide the leading term of the
dividend by the leading term of the divisor.

Step 2: Multiply the first term of the quotient by the divisor, remembering to
distribute, and line up like terms with the dividend.

Step 3: Subtract the resulting quantity from the dividend. Take care to subtract
both terms.

Step 4: Bring down the remaining terms and repeat the process from step 1.

5.5 Dividing Polynomials 845


Chapter 5 Polynomials and Their Operations

Notice that the leading term is eliminated and that the result has a degree that is
one less than the dividend. The complete process is illustrated below:

Polynomial long division ends when the degree of the remainder32 is less than the
degree of the divisor. Here the remainder is 0. Therefore, the binomial divides the
polynomial evenly and the answer is the quotient shown above the division line.

To check the answer, multiply the divisor by the quotient to see if you obtain the
dividend:

Answer: x 2 + 5x + 2

Next, we demonstrate the case where there is a nonzero remainder.

32. The expression that is left after


the division algorithm ends.

5.5 Dividing Polynomials 846


Chapter 5 Polynomials and Their Operations

Just as with real numbers, the final answer adds the fraction where the remainder is
the numerator and the divisor is the denominator to the quotient. In general, when
dividing we have

If we multiply both sides by the divisor we obtain

6x 2 −5x+3
Example 6: Divide: 2x−1
.

Solution: Since the denominator is a binomial, begin by setting up polynomial long


division.

This is the quotient of the given leading terms: (6x 2 ) ÷ (2x) = 3x. Multiply 3x
To start, determine what monomial times 2x − 1 results in a leading term 6x 2 .

times the divisor 2x − 1 and line up the result with like terms of the dividend.

5.5 Dividing Polynomials 847


Chapter 5 Polynomials and Their Operations

Subtract the result from the dividend and bring down the constant term +3.

Subtracting eliminates the leading term and −5x − (−3x) = −5x + 3x = −2x .
The quotient of −2x and 2x is −1. Multiply 2x − 1 by −1 and line up the result.

Subtract again and notice that we are left with a remainder.

The constant term 2 has degree 0, and thus the division ends. We may write

5.5 Dividing Polynomials 848


Chapter 5 Polynomials and Their Operations

2
Answer: 3x − 1 + 2x−1 . To check that this result is correct, we multiply as follows:

Occasionally, some of the powers of the variables appear to be missing within a


polynomial. This can lead to errors when lining up like terms. Therefore, when first
learning how to divide polynomials using long division, fill in the missing terms
with zero coefficients, called placeholders33.

27x 3 +64
Example 7: Divide: 3x+4
.

Solution: Notice that the binomial in the numerator does not have terms with
degree 2 or 1. The division is simplified if we rewrite the expression with
placeholders:

Set up polynomial long division:

33. Terms with zero coefficients


used to fill in all missing
exponents within a polynomial.

5.5 Dividing Polynomials 849


Chapter 5 Polynomials and Their Operations

We begin with 27x 3 ÷ 3x = 9x 2 and work the rest of the division algorithm.

Answer: 9x 2 − 12x + 16

3x 4 −2x 3 +6x 2 +23x−7


Example 8: Divide: .
x 2 −2x+5

Solution:

Begin the process by dividing the leading terms to determine the leading term of
the quotient 3x 4 ÷ x 2 = 3x 2 . Take care to distribute and line up the like terms.
Continue the process until the remainder has a degree less than 2.

5.5 Dividing Polynomials 850


Chapter 5 Polynomials and Their Operations

The remainder is x − 2 . Write the answer with the remainder:

x−2
Answer: 3x 2 + 4x − 1 +
x 2 −2x+5

Polynomial long division takes time and practice to master. Work lots of problems
and remember that you may check your answers by multiplying the quotient by the
divisor (and adding the remainder if present) to obtain the dividend.

20x 4 −32x 3 +7x 2 +8x−10


Try this! Divide: 5x−3
.

7
Answer: 4x 3 − 4x 2 − x + 1 − 5x−3

Video Solution

(click to see video)

5.5 Dividing Polynomials 851


Chapter 5 Polynomials and Their Operations

Dividing Polynomial Functions

We may use function notation to indicate division as follows:

(f /g) (x) =
f (x)
Division of functions:
g(x)

The quotient of two polynomial functions does not necessarily have a domain of all
real numbers. The values for x that make the function in the denominator 0 are
restricted from the domain. This will be discussed in more detail at a later time. For
now, assume all functions in the denominator are nonzero.

Example 9: Calculate: (f /g) (x) given f (x) = 6x 5 − 36x 4 + 12x 3 − 6x 2 and


g (x) = −6x 2 .

Solution: The notation indicates that we should divide:

Answer: (f /g) (x) = −x 3 + 6x 2 − 2x + 1

Example 10: Calculate: (f /g) (−1), given f (x) = −3x 3 + 7x 2 − 11x − 1 and
g (x) = 3x − 1.

Solution: First, determine (f /g) (x).

5.5 Dividing Polynomials 852


Chapter 5 Polynomials and Their Operations

Therefore,

Substitute −1 for the variable x .

Answer: (f /g) (−1) = −4

5.5 Dividing Polynomials 853


Chapter 5 Polynomials and Their Operations

KEY TAKEAWAYS

• When dividing by a monomial, divide all terms in the numerator by the


monomial and then simplify each term. To simplify each term, divide
the coefficients and apply the quotient rule for exponents.
• When dividing a polynomial by another polynomial, apply the division
algorithm.
• To check the answer after dividing, multiply the divisor by the quotient
and add the remainder (if necessary) to obtain the dividend.
• It is a good practice to include placeholders when performing
polynomial long division.

5.5 Dividing Polynomials 854


Chapter 5 Polynomials and Their Operations

TOPIC EXERCISES

Part A: Dividing by a Monomial

Divide.

81y 5
1.
9y 2

36y 9
2.
9y 3

52x 2 y
3.
4xy

24xy 5
4.
2xy 4

25x 2 y 5 z 3
5.
5xyz

77x 4 y 9 z
6. −
22x 3 y 3 z

125a3 b 2 c
7.
−10abc

36a2 b 3 c5
8.
−6a2 b 2 c3

9x 2 +27x−3
9.
3

10x 3 −5x 2 +40x−15


10.
5

20x 3 −10x 2 +30x


11.
2x

10x 4 +8x 2 −6x


12.
24x

−6x 5 −9x 3 +3x


13.
−3x

5.5 Dividing Polynomials 855


Chapter 5 Polynomials and Their Operations

36a12 −6a9 +12a5


14.
−12a5

−12x 5 +18x 3 −6x 2


15.
−6x 2

−49a8 +7a5 −21a3


16.
7a3

9x 7 −6x 4 +12x 3 −x 2
17.
3x 2

8x 9 +16x 7 −24x 4 +8x 3


18.
−8x 3

16a7 −32a6 +20a5 −a4


19.
4a4

5a6 +2a5 +6a3 −12a2


20.
3a2

−4x 2 y 3 +16x 7 y 8 −8x 2 y 5


21.
−4x 2 y 3

100a10 b 30 c5 −50a20 b 5 c40 +20a5 b 20 c10


22.
10a5 b 5 c5

23. Find the quotient of −36x 9 y 7 and 2x 8 y 5 .

24. Find the quotient of 144x 3 y 10 z 2 and −12x 3 y 5 z .

25. Find the quotient of 3a4 − 18a3 + 27a2 and 3a2 .

26. Find the quotient of 64a2 bc3 − 16a5 bc7 and 4a2 bc3 .

Part B: Dividing by a Polynomial

Divide.

27. (2x 2 − 5x − 3) ÷ (x − 3)

28. (3x 2 + 5x − 2) ÷ (x + 2)

5.5 Dividing Polynomials 856


Chapter 5 Polynomials and Their Operations

29. (6x 2 + 11x + 3) ÷ (3x + 1)

30. (8x 2 − 14x + 3) ÷ (2x − 3)

x 3 −x 2 −2x−12
31.
x−3

2x 3 +11x 2 +4x−5
32.
x+5

2x 3 −x 2 −4x+3
33.
2x+3

−15x 3 −14x 2 +23x−6


34.
5x−2

14x 4 −9x 3 +22x 2 +4x−1


35.
7x−1

8x 5 +16x 4 −8x 3 −5x 2 −21x+10


36.
2x+5

x 2 +8x+17
37.
x+5

2x 2 −5x+5
38.
x−2

6x 2 −13x+9
39.
−2x+1

−12x 2 +x+1
40.
3x+2

x 3 +9x 2 +19x+1
41.
x+4

2x 3 −13x 2 +17x−11
42.
x−5

9x 3 −12x 2 +16x−15
43.
3x−2

3x 4 −8x 3 +5x 2 −5x+9


44.
x−2

5.5 Dividing Polynomials 857


Chapter 5 Polynomials and Their Operations

45. (6x 5 − 13x 4 + 4x 3 − 3x 2 + 13x − 2) ÷ (3x + 1)

46. (8x 5 − 22x 4 + 19x 3 − 20x 2 + 23x − 3) ÷ (2x − 3)

5x 5 +12x 4 +12x 3 −7x 2 −19x+3


47.
x 2 +2x+3

6x 5 −17x 4 +5x 3 +16x 2 −7x−3


48.
2x 2 −3x−1

x 5 +7 x 4 −x 3 −7 x 2 −49 x+9
49.
x 2 +7x−1

5x 6 −6x 4 −4x 2 +x+2


50.
5x 2 −1

x 3 −27
51.
x−3

8x 3 +125
52.
2x+5

53. (15x 5 − 9x 4 − 20x 3 + 12x 2 + 15x − 9) ÷ (5x − 3)

54. (2x 6 − 5x 5 − 4x 4 + 10x 3 + 6x 2 − 17x + 5) ÷ (2x − 5)

x 5 −2x 3 +3x−1
55.
x−1

x 4 −3x 2 +5x−13
56.
x+2

a2 −4
57.
a+2

a5 +1
58.
a5 +1

a6 −1
59.
a−1

x 5 −1
60.
x−1

5.5 Dividing Polynomials 858


Chapter 5 Polynomials and Their Operations

x 5 +x 4 +6x 3 +12x 2 −4
61.
x 2 +x−1

50x 6 −30x 5 −5x 4 +15 x 3 −5x+1


62.
5x 2 −3x+2

5x 5 −15x 3 +25x 2 −5
63.
5x

−36x 6 +12x 4 −6x 2


64.
6x 2

150x 5 y 2 z 15 −10x 3 y 6 z 5 +4x 3 y 2 z 4


65.
10x 3 y 2 z 5

27m 6 +9m 4 −81m 2 +1


66.
9m 2

67. Divide 3x 6 − 2x 5 + 27x 4 − 18x 3 − 6x 2 + 7x − 10 by


3x − 2 .

68. Divide 8x 6 + 4x 5 − 14x 4 − 5x 3 + x 2 − 2x − 3 by 2x + 1 .

Part C: Dividing Polynomial Functions

Calculate (f /g) (x) , given the functions.

69. f (x) = 40x 8 and g (x) = 10x 5

70. f (x) = 54x 5 and g (x) = 9x 3

71. f (x) = 12x 2 + 24x − 15 and g (x) = 2x + 5

72. f (x) = −8x 2 + 30x − 7 and g (x) = 2x − 7

73. f (x) = 18x 2 − 36x + 5 and g (x) = 3x − 5

74. f (x) = −7x 2 + 29x − 6 and g (x) = 7x − 1

75. f (x) = 10x 3 − 9x 2 + 27x − 10 and g (x) = 5x − 2

5.5 Dividing Polynomials 859


Chapter 5 Polynomials and Their Operations

76. f (x) = 15x 3 + 28x 2 − 11x + 56 and g (x) = 3x + 8

77. f(x) = 2x 4 + 5x 3 − 11x 2 − 19x + 20 and


g (x) = x 2 + x − 5

78. f(x) = 4x 4 − 12x 3 − 20x 2 + 26x − 3 and


g (x) = 2x 2 + 2x − 3

Given f (x) = 6x 3 + 4x 2 − 11x + 3 and g (x) = 3x − 1 , find the


following.

79. (f /g) (x)

80. (f /g) (−1)

81. (f /g) (0)

82. (f /g) (1)

Given f (x) = 5x 3 − 13x 2 + 7x + 3 and g (x) = x − 2 , find the


following.

83. (f /g) (x)

84. (f /g) (−3)

85. (f /g) (0)

86. (f /g) (7)

Part D: Discussion Board Topics

87. How do you use the distributive property when dividing a polynomial by
a monomial?

88. Compare long division of real numbers with polynomial long division.
Provide an example of each.

5.5 Dividing Polynomials 860


Chapter 5 Polynomials and Their Operations

ANSWERS

1: 9y 3

3: 13x

5: 5xy 4 z 2

25
7: −
2
a2 b

9: 3x 2 + 9x − 1

11: 10x 2 − 5x + 15

13: 2x 4 + 3x 2 − 1

15: 2x 3 − 3x + 1
1
17: 3x 5 − 2x 2 + 4x − 3

1
19: 4a3 − 8a2 + 5a − 4

21: −4x 5 y 5 + 2y 2 + 1

23: −18xy 2

25: a2 − 6a + 9

27: 2x +1

29: 2x +3

31: x 2 + 2x + 4

33: x 2 − 2x + 1

35: 2x 3 − x 2 + 3x + 1

5.5 Dividing Polynomials 861


Chapter 5 Polynomials and Their Operations

2
37: x +3+ x+5

4
39: −3x +5+ −2x+1

5
41: x 2 + 5x − 1 + x+4

7
43: 3x 2 − 2x + 4 − 3x−2

7
45: 2x 4 − 5x 3 + 3x 2 − 2x + 5 − 3x+1

47: 5x 3 + 2x 2 − 7x + 1
2
49: x 3 −7+ x 2 +7x−1

51: x 2 + 3x + 9

53: 3x 4 − 4x 2 + 3
1
55: x 4 + x3 − x2 − x + 2 + x−1

57: a −2

59: a5 + a4 + a3 + a2 + a + 1
2x+1
61: x 3 + 7x + 5 + x 2 +x−1

1
63: x 4 − 3x 2 + 5x − x

2
65: 15x 2 z 10 − y4 + 5z

8
67: x 5 + 9x 3 − 2x + 1 − 3x−2

69: (f /g) (x) = 4x 3

5.5 Dividing Polynomials 862


Chapter 5 Polynomials and Their Operations

71: (f /g) (x) = 6x − 3

73: (f /g)
5
(x) = 6x − 2 − 3x−5

75: (f /g) (x) = 2x 2 − x + 5

77: (f /g) (x) = 2x 2 + 3x − 4

79: (f /g) (x) = 2x 2 + 2x − 3

81: (f /g) (0) = −3

83: (f /g)
5
(x) = 5x 2 − 3x + 1 + x−2

85: (f /g)
3
(0) = − 2

5.5 Dividing Polynomials 863


Chapter 5 Polynomials and Their Operations

5.6 Negative Exponents

LEARNING OBJECTIVES

1. Simplify expressions with negative integer exponents.


2. Work with scientific notation.

Negative Exponents

In this section, we define what it means to have negative integer exponents. We


begin with the following equivalent fractions:

Notice that 4, 8, and 32 are all powers of 2. Hence we can write 4 = 22 , 8 = 23 , and
32 = 25 .

If the exponent of the term in the denominator is larger than the exponent of the
term in the numerator, then the application of the quotient rule for exponents
results in a negative exponent. In this case, we have the following:

We conclude that 2−3 = 13 . This is true in general and leads to the definition of
2
negative exponents34. Given any integer n and x ≠ 0 , then

1
34. x −n = x n ,given any integer
n, where x is nonzero.

864
Chapter 5 Polynomials and Their Operations

Here x ≠ 0 because 10 is undefined. For clarity, in this section, assume all variables
are nonzero.

Simplifying expressions with negative exponents requires that we rewrite the


expression with positive exponents.

Example 1: Simplify: 10−2 .

Solution:

1
Answer: 100

Example 2: Simplify: (−3)−1 .

Solution:

Answer: − 1
3

5.6 Negative Exponents 865


Chapter 5 Polynomials and Their Operations

Example 3: Simplify: 1−3 .


y

Solution:

Answer: y 3

At this point we highlight two very important examples,

If the grouped quantity is raised to a negative exponent, then apply the definition
and write the entire grouped quantity in the denominator. If there is no grouping,
then apply the definition only to the base preceding the exponent.

Example 4: Simplify: (2ab) .


−3

Solution: First, apply the definition of −3 as an exponent and then apply the power
of a product rule.

5.6 Negative Exponents 866


Chapter 5 Polynomials and Their Operations

1
Answer:
8a3 b3

Example 5: Simplify: (−3xy 3 )


−2
.

Solution:

1
Answer:
9x 2 y 6

−3
Example 6: Simplify: x −4 .
y

Solution:

5.6 Negative Exponents 867


Chapter 5 Polynomials and Their Operations

y4
Answer:
x3

The previous example suggests a property of quotients with negative


exponents35. If given any integers m and n, where x ≠ 0 and y ≠ 0 , then

In other words, negative exponents in the numerator can be written as positive


exponents in the denominator, and negative exponents in the denominator can be
written as positive exponents in the numerator.

−2x −5y 3
Example 7: Simplify: .
z −2

Solution: Take care with the coefficient −2; recognize that this is the base and that
the exponent is actually +1: −2 = (−2)1 . Hence the rules of negative exponents do
not apply to this coefficient; leave it in the numerator.

x −n ym
35. y −m = x n , given any integers −2y 3 z 2
Answer:
m and n, where x ≠ 0 and x5
y ≠ 0.

5.6 Negative Exponents 868


Chapter 5 Polynomials and Their Operations

(−3x )
−4 −3
Example 8: Simplify: .
y −2

Solution: Apply the power of a product rule before applying negative exponents.

x 12 y 2
Answer: − 27

(3x )
2 −4

(−2y −1z 3 )
Example 9: Simplify: −2
.

Solution:

5.6 Negative Exponents 869


Chapter 5 Polynomials and Their Operations

4z 6
Answer:
81x 8 y 2

(5x y)
2 3

Example 10: Simplify: .


x −5y −3

Solution: First, apply the power of a product rule and then the quotient rule.

Answer: 125x 11 y 6

To summarize, we have the following rules for negative integer exponents with
nonzero bases:

1
Negative exponents: x −n =
xn

5.6 Negative Exponents 870


Chapter 5 Polynomials and Their Operations

x −n ym
Quotients with negative exponents: = n
y −m x

(−5xy )
−3 −2
Try this! Simplify: .
5x 4 y −4

y 10
Answer:
125x 6

Video Solution

(click to see video)


Scientific Notation

Real numbers expressed in scientific notation36 have the form

where n is an integer and 1 ≤ a < 10. This form is particularly useful when the
numbers are very large or very small. For example,

It is cumbersome to write all the zeros in both of these cases. Scientific notation is
an alternative, compact representation of these numbers. The factor 10n indicates
the power of 10 to multiply the coefficient by to convert back to decimal form:

36. Real numbers expressed in the


n
form a × 10 , where n is an
integer and 1 ≤ a < 10.

5.6 Negative Exponents 871


Chapter 5 Polynomials and Their Operations

This is equivalent to moving the decimal in the coefficient fifteen places to the
right. A negative exponent indicates that the number is very small:

This is equivalent to moving the decimal in the coefficient eleven places to the left.

Converting a decimal number to scientific notation involves moving the decimal as


well. Consider all of the equivalent forms of 0.00563 with factors of 10 that follow:

While all of these are equal, 5.63 × 10−3 is the only form considered to be
expressed in scientific notation. This is because the coefficient 5.63 is between 1 and
10 as required by the definition. Notice that we can convert 5.63 × 10−3 back to
decimal form, as a check, by moving the decimal to the left three places.

Example 11: Write 1,075,000,000,000 using scientific notation.

Solution: Here we count twelve decimal places to the left of the decimal point to
obtain the number 1.075.

Answer: 1.075 × 1012

5.6 Negative Exponents 872


Chapter 5 Polynomials and Their Operations

Example 12: Write 0.000003045 using scientific notation.

Solution: Here we count six decimal places to the right to obtain 3.045.

Answer: 3.045 × 10−6

Often we will need to perform operations when using numbers in scientific


notation. All the rules of exponents developed so far also apply to numbers in
scientific notation.

Example 13: Multiply: (4.36 × 10−5 ) (5.3 × 1012 ).

Solution: Use the fact that multiplication is commutative and apply the product
rule for exponents.

Answer: 2.3108 × 108

5.6 Negative Exponents 873


Chapter 5 Polynomials and Their Operations

Example 14: Divide: (3.24 × 108 ) ÷ (9.0 × 10 ).


−3

Solution:

Answer: 3.6 × 1010

Example 15: The speed of light is approximately 6.7 × 108 miles per hour. Express
this speed in miles per second.

Solution: A unit analysis indicates that we must divide the number by 3,600.

5.6 Negative Exponents 874


Chapter 5 Polynomials and Their Operations

Answer: The speed of light is approximately 1.9 × 105 miles per second.

Example 16: By what factor is the radius of the sun larger than the radius of earth?

Solution: We want to find the number that when multiplied times the radius of
earth equals the radius of the sun.

Source: NASA and European


Space Agency, from
http://solarsystem.nasa.gov/
multimedia/
display.cfm?IM_ID=188.

Therefore,

5.6 Negative Exponents 875


Chapter 5 Polynomials and Their Operations

Answer: The radius of the sun is approximately 110 times that of earth.

Try this! Divide: (6.75 × 10−8 ) ÷ (9 × 10


−17
).

Answer: 7.5 × 108

Video Solution

(click to see video)

KEY TAKEAWAYS

• Expressions with negative exponents in the numerator can be rewritten


as expressions with positive exponents in the denominator.
• Expressions with negative exponents in the denominator can be
rewritten as expressions with positive exponents in the numerator.
• Take care to distinguish negative coefficients from negative exponents.
• Scientific notation is particularly useful when working with numbers
that are very large or very small.

5.6 Negative Exponents 876


Chapter 5 Polynomials and Their Operations

TOPIC EXERCISES

Part A: Negative Exponents

Simplify. (Assume variables are nonzero.)

−1
1. 5

−2
2. 5

−1
3. (−7)

−1
4. −7

1
5.
2 −3

5
6.
3 −2

7. (
5)
3 −2

8. (
2)
1 −5

9. (−
3)
2 −4

10. (−
3)
1 −3

11. x −4

12. y −1

13. 3x −5

−5
14. (3x)

5.6 Negative Exponents 877


Chapter 5 Polynomials and Their Operations

1
15.
y −3

5
16.
2x −1

x −1
17.
y −2

1
(x−y)
18. −4

x 2 y −3
19.
z −5

20. xy −3

21. (ab)
−1

1
22.
(ab)−1

23. −5x −3 y 2 z −4

−2 3 −5
24. 3 x y z

25. 3x −4 y 2 ⋅ 2x −1 y 3
3
26. −10a2 b ⋅ 2a−8 b −10

27. (2a−3 )
−2

28. (−3x 2 )
−1

29. (5a2 b c)
−3 −2

30. (7r3 s−5 t)


−3

5.6 Negative Exponents 878


Chapter 5 Polynomials and Their Operations

31. (−2r2 s0 t −3 )
−1

32. (2xy −3 z 2 )
−3

33. (−5a2 b c )
−3 0 4

34. (−x −2 y 3 z −4 )
−7

( 2x −3 )
−5
1
35.

( y2 )
−2
2x
36.

( 2y −1 )
−4
x
37.

( c5 )
−5
−3a2 b
38.

( 5yz −1 )
20x −3 y 2 −1
39.

( 2r3 st 0 )
−3
4r5 s−3 t 4
40.

( y 2 z3 )
2xy 3 z −1 −3
41.

(− )
2
3a2 bc
42.
ab 0 c4

(− )
xyz −4
43.
x y −2 z 3
4

( )
0
125x −3 y 4 z −5

5x 2 y 4 (x+y)
44. 3

5.6 Negative Exponents 879


Chapter 5 Polynomials and Their Operations

−2
45. (x n )

( yn )
−2
xn
46.

The value in dollars of a new MP3 player can be estimated by using the formula
V = 100(t + 1)−1 , where t is the number of years after purchase.

47. How much was the MP3 player worth new?

48. How much will the MP3 player be worth in 1 year?

49. How much will the MP3 player be worth in 4 years?

50. How much will the MP3 player be worth in 9 years?

51. How much will the MP3 player be worth in 99 years?

52. According to the formula, will the MP3 ever be worthless? Explain.

Part B: Scientific Notation

Convert to a decimal number.

53. 9.3 × 10 9

54. 1.004 × 10 4

55. 6.08 × 10 10

56. 3.042 × 10 7

57. 4.01 × 10 −7

58. 1.0 × 10 −10

59. 9.9 × 10 −3

5.6 Negative Exponents 880


Chapter 5 Polynomials and Their Operations

60. 7.0011 × 10 −5

Rewrite using scientific notation.

61. 500,000,000

62. 407,300,000,000,000

63. 9,740,000

64. 100,230

65. 0.0000123

66. 0.000012

67. 0.000000010034

68. 0.99071

Perform the indicated operations.

69. (3 × 10 5 ) (9 × 10 4 )

70. (8 × 10 −22 ) (2 × 10 −12 )

71. (2.1 × 10 −19 ) (3.0 × 10 8 )

72. (4.32 × 10 7 ) (1.50 × 10 −18 )

9.12×10 −9
73.
3.2×10 10

1.15×10 9
74.
2.3×10 −11

1.004×10 −8
75.
2.008×10 −14

5.6 Negative Exponents 881


Chapter 5 Polynomials and Their Operations

3.276×10 25
76.
5.2×10 15

77. 59,000,000,000,000 × 0.000032

78. 0.0000000000432 × 0.0000000000673

79. 1,030,000,000,000,000,000 ÷ 2,000,000

80. 6,045,000,000,000,000 ÷ 0.00000005

81. The population density of earth refers to the number of people per
× 10 7
square mile of land area. If the total land area on earth is 5.751
9
square miles and the population in 2007 was estimated to be 6.67 × 10
people, then calculate the population density of earth at that time.

82. In 2008 the population of New York City was estimated to be 8.364
million people. The total land area is 305 square miles. Calculate the
population density of New York City.

83. The mass of earth is 5.97 × 10 24 kilograms and the mass of the moon
22
is 7.35 × 10 kilograms. By what factor is the mass of earth greater than
the mass of the moon?

84. The mass of the sun is 1.99 × 10 30 kilograms and the mass of earth is
24
5.97 × 10 kilograms. By what factor is the mass of the sun greater than
the mass of earth? Express your answer in scientific notation.

× 10 5 miles and the average distance


85. The radius of the sun is 4.322
5
from earth to the moon is 2.392 × 10 miles. By what factor is the radius
of the sun larger than the average distance from earth to the moon?

15
86. One light year, 9.461 × 10 meters, is the distance that light travels
in a vacuum in one year. If the distance to the nearest star to our sun,
16
Proxima Centauri, is estimated to be 3.991 × 10 meters, then calculate
the number of years it would take light to travel that distance.

87. It is estimated that there are about 1 million ants per person on the
planet. If the world population was estimated to be 6.67 billion people in
2007, then estimate the world ant population at that time.

5.6 Negative Exponents 882


Chapter 5 Polynomials and Their Operations

88. The sun moves around the center of the galaxy in a nearly circular orbit.
The distance from the center of our galaxy to the sun is approximately
26,000 light years. What is the circumference of the orbit of the sun around
the galaxy in meters?

89. Water weighs approximately 18 grams per mole. If one mole is about
6 × 10 23 molecules, then approximate the weight of each molecule of
water.

9 6
90. A gigabyte is 1 × 10 bytes and a megabyte is 1 × 10 bytes. If the
average song in the MP3 format consumes about 4.5 megabytes of storage,
then how many songs will fit on a 4-gigabyte memory card?

5.6 Negative Exponents 883


Chapter 5 Polynomials and Their Operations

ANSWERS

1
1:
5

1
3: − 7

5: 8

25
7:
9

81
9:
16

1
11:
x4

3
13:
x5

15: y 3

y2
17: x

x 2 z5
19:
y3

1
21:
ab

−5y 2
23:
x 3 z4

6y 5
25:
x5

a6
27:
4

b6
29:
25a4 c2

t3
31: −
2r2

5.6 Negative Exponents 884


Chapter 5 Polynomials and Their Operations

625a8
33:
b 12

32
35:
x 15

16
37:
x 4y4

x3
39:
4yz

z 12
41:
8x 3 y 3

x 12 z 8
43:
y 12

1
45:
x 2n

47: $100

49: $20

51: $1

53: 9,300,000,000

55: 60,800,000,000

57: 0.000000401

59: 0.0099

61: 5 × 10 8

63: 9.74 × 10 6

65: 1.23 × 10 −5

67: 1.0034 × 10 −8

5.6 Negative Exponents 885


Chapter 5 Polynomials and Their Operations

69: 2.7 × 10 10

71: 6.3 × 10 −11

73: 2.85 × 10 −19

75: 5 × 10 5

77: 1.888 × 10 9

79: 5.15 × 10 11

81: About 116 people per square mile

83: 81.2

85: 1.807

87: 6.67 × 10 15 ants

89: 3 × 10 −23 grams

5.6 Negative Exponents 886


Chapter 5 Polynomials and Their Operations

5.7 Review Exercises and Sample Exam

887
Chapter 5 Polynomials and Their Operations

REVIEW EXERCISES

Rules of Exponents

Simplify.

3
1. 7 ⋅ 76

59
2.
56

3. y 5 ⋅ y2 ⋅ y3

4. x 3 y 2 ⋅ xy 3
2
5. −5a3 b c ⋅ 6a2 bc2

55x 2 yz 5
6.
5xyz 2

( 2c3 )
2
−3a2 b 4
7.

( 4c4 )
3
−2a3 b
8.

9. −5x 3 y 0 (z 2 ) ⋅ 2x 4 (y 3 ) z
3 2

10. (−25x 6 y 5 z)
0

11. Each side of a square measures 5x 2 units. Find the area of the square in
terms of x.

12. Each side of a cube measures 2x 3 units. Find the volume of the cube in
terms of x.

Introduction to Polynomials

5.7 Review Exercises and Sample Exam 888


Chapter 5 Polynomials and Their Operations

Classify the given polynomial as a monomial, binomial, or trinomial and state the
degree.

13. 8a3 −1

14. 5y 2 −y+1
2
15. −12ab

16. 10

Write the following polynomials in standard form.

17. 7 − x 2 − 5x

18. 5x 2 − 1 − 3x + 2x 3

Evaluate.

19. 2x 2 − x + 1 , where x = −3
1 3 1
20.
2
x− 4
, where x = 3

2 1 3
21. b − 4ac, where a = − ,b = −3 , and c = −
2 2

1 1
22. a2 − b 2, where a = − 2
and b =− 3

23. a3 − b 3, where a = −2 and b = −1

24. xy 2 − 2x 2 y , where x = −3 and y = −1

25. Given f (x) = 3x 2 − 5x + 2 , find f (−2) .

26. Given g (x) = x 3 − x 2 + x − 1, find g (−1) .

27. The surface area of a rectangular solid is given by the formula


SA = 2lw + 2wh + 2lh , where l, w, and h represent the length, width,
and height, respectively. If the length of a rectangular solid measures 2

5.7 Review Exercises and Sample Exam 889


Chapter 5 Polynomials and Their Operations

units, the width measures 3 units, and the height measures 5 units, then
calculate the surface area.

28. The surface area of a sphere is given by the formula SA = 4πr2 , where
r represents the radius of the sphere. If a sphere has a radius of 5 units, then
calculate the surface area.

Adding and Subtracting Polynomials

Perform the operations.

29. (3x − 4) + (9x − 1)

30. ( x − 19 ) + ( 16 x + 12 )
1
3

31. (7x 2 − x + 9) + (x 2 − 5x + 6)

32. (6x 2 y − 5xy 2 − 3) + (−2x 2 y + 3xy 2 + 1)

33. (4y + 7) − (6y − 2) + (10y − 1)

34. (5y 2 − 3y + 1) − (8y 2 + 6y − 11)

35. (7x 2 y 2 − 3xy + 6) − (6x 2 y 2 + 2xy − 1)

36. (a3 − b 3 ) − (a3 + 1) − (b 3 − 1)

37. (x 5 − x 3 + x − 1) − (x 4 − x 2 + 5)

38. (5x 3 − 4x 2 + x − 3) − (5x 3 − 3) + (4x 2 − x)

39. Subtract 2x −1 from 9x + 8.

40. Subtract 3x 2 − 10x − 2 from 5x 2 + x − 5.

(f + g) (x) .
(x) = 3x 2 − x + 5 and g (x) = x 2 − 9, find
41. Given f

5.7 Review Exercises and Sample Exam 890


Chapter 5 Polynomials and Their Operations

(f − g) (x) .
(x) = 3x 2 − x + 5 and g (x) = x 2 − 9, find
42. Given f

(f + g) (−2) .
(x) = 3x 2 − x + 5 and g (x) = x 2 − 9, find
43. Given f

(f − g) (−2) .
(x) = 3x 2 − x + 5 and g (x) = x 2 − 9, find
44. Given f

Multiplying Polynomials

Multiply.

45. 6x 2 (−5x )
4

(7a b)
2 2
46. 3ab

47. 2y (5y − 12)

48. −3x (3x 2 − x + 2)

49. x 2 y (2x 2 y − 5xy 2 + 2)

50. −4ab (a2 − 8ab + b 2 )

51. (x − 8) (x + 5)

52. (2y − 5) (2y + 5)

53. (3x − 1) 2

54. (3x − 1) 3

55. (2x − 1) (5x 2 − 3x + 1)

5.7 Review Exercises and Sample Exam 891


Chapter 5 Polynomials and Their Operations

56. (x 2 + 3) (x 3 − 2x − 1)

57. (5y + 7)
2

58. (y 2 − 1)
2

59. Find the product of x 2 − 1 and x 2 + 1.


3
60. Find the product of
2
x 2 y and 10x − 30y + 2 .

61. Given f (x) = 7x − 2 and g (x) = x 2 − 3x + 1 , find (f ⋅ g) (x) .

62. Given f (x) = x − 5 and g (x) = x 2 − 9, find (f ⋅ g) (x) .

(f ⋅ g) (−1).
63. Given f
(x) = 7x − 2 and g (x) = x 2 − 3x + 1 , find

64. Given f (x) = x − 5 and g (x) = x 2 − 9, find (f ⋅ g) (−1).

Dividing Polynomials

Divide.

7y 2 −14y+28
65. 7

12x 5 −30x 3 +6x


66.
6x

4a2 b−16ab 2 −4ab


67.
−4ab

6a6 −24a4 +5a2


68.
3a2

69. (10x 2 − 19x + 6) ÷ (2x − 3)

70. (2x 3 − 5x 2 + 5x − 6) ÷ (x − 2)

5.7 Review Exercises and Sample Exam 892


Chapter 5 Polynomials and Their Operations

10x 4 −21x 3 −16x 2 +23x−20


71.
2x−5

x 5 −3x 4 −28x 3 +61x 2 −12x+36


72.
x−6

10x 3 −55x 2 +72x−4


73.
2x−7

3x 4 +19x 3 +3x 2 −16x−11


74.
3x+1

5x 4 +4x 3 −5x 2 +21x+21


75.
5x+4

x 4 −4
76.
x−4

2x 4 +10x 3 −23x 2 −15x+30


77.
2x 2 −3

7x 4 −17x 3 +17x 2 −11x+2


78.
x 2 −2x+1

79. Given f (x) = x 3 − 4x + 1 and g (x) = x − 1 , find (f /g) (x) .

80. Given f (x) = x 5 − 32 and g (x) = x − 2 , find (f /g) (x) .

81. Given f (x) = x 3 − 4x + 1 and g (x) = x − 1 , find (f /g) (2) .

82. Given f (x) = x 5 − 32 and g (x) = x − 2 , find (f /g) (0) .

Negative Exponents

Simplify.

−2
83. (−10)

−2
84. −10

85. 5x −3

5.7 Review Exercises and Sample Exam 893


Chapter 5 Polynomials and Their Operations

86. (5x)
−3

1
87.
7y −3

3x −4
88.
y −2

−2a2 b −5
89.
c−8

90. (−5x 2 yz −1 )
−2

91. (−2x −3 y 0 z 2 )
−3

( 5ab2 c2 )
−1
−10a5 b 3 c2
92.

( 2a4 b−3 c )
−3
a2 b −4 c0
93.

The value in dollars of a new laptop computer can be estimated by using the formula
V = 1200(t + 1)−1 , where t represents the number of years after the
purchase.

94. Estimate the value of the laptop when it is 1½ years old.

95. What was the laptop worth new?

Rewrite using scientific notation.

96. 2,030,000,000

97. 0.00000004011

Perform the indicated operations.

98. (5.2 × 10 12 ) (1.8 × 10 −3 )

5.7 Review Exercises and Sample Exam 894


Chapter 5 Polynomials and Their Operations

99. (9.2 × 10 −4 ) (6.3 × 10 22 )

4×10 16
100.
8×10 −7

9×10 −30
101.
4×10 −10

102. 5,000,000,000,000 × 0.0000023

103. 0.0003/120,000,000,000,000

5.7 Review Exercises and Sample Exam 895


Chapter 5 Polynomials and Their Operations

SAMPLE EXAM

Simplify.

1. −5x 3 (2x y)
2

2. (x 2 )
4
⋅ x3 ⋅ x

(−2x y )
2 3 2

3.
x 2y

4. a. (−5) ; b. −5
0 0

Evaluate.

5. 2x 2 − x + 5 , where x = −5

6. a2 − b 2, where a = 4 and b = −3

Perform the operations.

7. (3x 2 − 4x + 5) + (−7x 2 + 9x − 2)

8. (8x 2 − 5x + 1) − (10x 2 + 2x − 1)

9. ( a − 12 ) − ( 23 a2 + a − 29 ) + ( 15 )
3 2 1 5
5 3
a− 18

10. 2x 2 (2x − 3x − 4x + 5)
3 2

11. (2x − 3) (x + 5)

12. (x − 1) 3

81x 5 y 2 z
13.
−3x 3 yz

5.7 Review Exercises and Sample Exam 896


Chapter 5 Polynomials and Their Operations

10x 9 −15x 5 +5x 2


14.
−5x 2

x 3 −5x 2 +7x−2
15.
x−2

6x 4 −x 3 −13x 2 −2x−1
16.
2x−1

Simplify.

−3
17. 2

18. −5x −2

19. (2x 4 y −3 z)
−2

( ab−3 c2 )
−3
−2a3 b −5 c−2
20.

21. Subtract 5x 2 y − 4xy 2 + 1 from 10x 2 y − 6xy 2 + 2 .

22. If each side of a cube measures 4x 4 units, calculate the volume in terms
of x.

23. The height of a projectile in feet is given by the formula


h = −16t 2 + 96t + 10 , where t represents time in seconds. Calculate
the height of the projectile at 1½ seconds.

24. The cost in dollars of producing custom t-shirts is given by the formula
C = 120 + 3.50x , where x represents the number of t-shirts produced.
The revenue generated by selling the t-shirts for $6.50 each is given by the
formula R = 6.50x , where x represents the number of t-shirts sold.

a. Find a formula for the profit. (profit = revenue − cost)

b. Use the formula to calculate the profit from producing and selling 150 t-
shirts.

5.7 Review Exercises and Sample Exam 897


Chapter 5 Polynomials and Their Operations

25. The total volume of water in earth’s oceans, seas, and bays is estimated
× 10 19 cubic feet. By what factor is the volume of the moon,
to be 4.73
7.76 × 10 20 cubic feet, larger than the volume of earth’s oceans? Round
to the nearest tenth.

5.7 Review Exercises and Sample Exam 898


Chapter 5 Polynomials and Their Operations

REVIEW EXERCISES ANSWERS

9
1: 7

3: y 10

3 3
5: −30a5 b c

9a4 b 8
7:
4c6

9: −10x 7 y 6 z 7

11: A = 25x 4

13: Binomial; degree 3

15: Monomial; degree 3

17: −x 2 − 5x + 7

19: 22

21: 6

23: −7

25: f (−2) = 24

27: 62 square units

29: 12x −5

31: 8x 2 − 6x + 15

33: 8y +8

35: x 2 y 2 − 5xy + 7

5.7 Review Exercises and Sample Exam 899


Chapter 5 Polynomials and Their Operations

37: x 5 − x4 − x3 + x2 + x − 6

39: 7x +9

41: (f + g) (x) = 4x 2 − x − 4

43: (f + g) (−2) = 14

45: −30x 6

47: 10y 2 − 24y

49: 2x 4 y 2 − 5x 3 y 3 + 2x 2 y

51: x 2 − 3x − 40

53: 9x 2 − 6x + 1

55: 10x 3 − 11x 2 + 5x − 1

57: 25y 2 + 70y + 49

59: x 4 −1

61: (f ⋅ g) (x) = 7x 3 − 23x 2 + 13x − 2

63: (f ⋅ g) (−1) = −45

65: y 2 − 2y + 4

67: −a + 4b + 1

69: 5x −2

71: 5x 3 + 2x 2 − 3x + 4
3
73: 5x 2 − 10x + 1 + 2x−7

5.7 Review Exercises and Sample Exam 900


Chapter 5 Polynomials and Their Operations

1
75: x 3 −x+5+ 5x+4

77: x 2 + 5x − 10

79: (f /g)
2
(x) = x 2 + x − 3 − x−1

81: (f /g) (2) = 1

1
83:
100

5
85:
x3

y3
87: 7

−2a2 c8
89:
b5

x9
91: −
8z 6

3 3
93: 8a6 b c

95: $1,200

97: 4.011 × 10 −8

99: 5.796 × 10 19

101: 2.25 × 10 −20

103: 2.5 × 10 −18

5.7 Review Exercises and Sample Exam 901


Chapter 5 Polynomials and Their Operations

SAMPLE EXAM ANSWERS

1: −10x 5 y

3: 4x 2 y 5

5: 60

7: −4x 2 + 5x + 3
2 5
9: −
3
a2 − 9

11: 2x 2 + 7x − 15

13: −27x 2 y

15: x 2 − 3x + 1
1
17:
8

y6
19:
4x 8 z 2

21: 5x 2 y − 2xy 2 + 1

23: 118 feet

25: 16.4

5.7 Review Exercises and Sample Exam 902


Chapter 6
Factoring and Solving by Factoring

903
Chapter 6 Factoring and Solving by Factoring

6.1 Introduction to Factoring

LEARNING OBJECTIVES

1. Determine the greatest common factor (GCF) of natural numbers.


2. Determine the GCF of monomials.
3. Factor out the GCF of a polynomial.
4. Factor a four-term polynomial by grouping.

GCF of Natural Numbers

The process of writing a number or expression as a product is called factoring1. If


we write 60 = 5 ⋅ 12, we say that the product 5 ⋅ 12 is a factorization2 of 60 and that
5 and 12 are factors3. Typically, there are many ways to factor a number. For
example,

Recall that a prime number is defined as a natural number with exactly two natural
number factors, 1 and itself. The first ten prime numbers follow:

1. The process of writing a


number or expression as a
product.
Any natural number greater than 1 can be uniquely written as a product of prime
2. Any combination of factors, numbers. This product is called the prime factorization4. The prime factorization
multiplied together, resulting of 60 can be determined by continuing to factor until only a product of prime
in the product.
numbers remains.
3. Any of the numbers or
expressions that form a
product.

4. The unique factorization of a


natural number written as a
product of primes.

904
Chapter 6 Factoring and Solving by Factoring

Since the prime factorization is unique, it does not matter how we choose to
initially factor the number; the end result will be the same. The prime factorization
of 60 follows:

Recall that the greatest common factor (GCF)5 of any two natural numbers is the
product of all the common prime factors.

Example 1: Find the GCF of 60 and 140.

Solution: First, determine the prime factorizations of both integers.

The product of the common prime factors is 22 ⋅ 5; hence the


GCF(60, 140) = 22 ⋅ 5 = 20. To see that it is the greatest common factor, we can
write the following:

5. The product of all the common


prime factors.

6.1 Introduction to Factoring 905


Chapter 6 Factoring and Solving by Factoring

Answer: The greatest common factor of 60 and 140 is 20.

Example 2: Find the GCF of 504 and 1,080.

Solution: First, determine the prime factorizations of both integers.

The product of the common prime factors is 23 ⋅ 32 . The


GCF(504, 1080) = 23 ⋅ 32 = 72. Note that we multiplied the common prime
factors with the smallest exponent.

The numbers 7 and 15 share no common natural number factor other than 1; we say
that they are relatively prime6.

Answer: The greatest common factor of 504 and 1,080 is 72.

GCF of Monomials

We next consider factorizations of monomials. For example, 6x and x 4 are factors


of 6x 5 because 6x 5 = 6x ⋅ x 4 . Typically, there are many ways to factor a
monomial. Some factorizations of 6x 5 follow:

6. Expressions that share no


common factors other than 1.

6.1 Introduction to Factoring 906


Chapter 6 Factoring and Solving by Factoring

Given two or more monomials, it will be useful to find the greatest common
monomial factor of each. For example, consider 6x 5 y 3 z and 8x 2 y 3 z 2 . The variable
part of these two monomials look very much like the prime factorization of natural
numbers and, in fact, can be treated the same way. Steps for finding the GCF of
monomials7 are outlined in the following example.

Example 3: Find the GCF of 6x 5 y 3 z and 8x 2 y 3 z 2 .

Solution:

Step 1: Find the GCF of the coefficients.

In this case, the GCF(6, 8) = 2.

Step 2: Determine the common variable factors with smallest exponents.

In this case, the common variables with the smallest exponents are x 2 , y 3 , and z 1 .

Step 3: The GCF of the monomials is the product of the common variable factors
and the GCF of the coefficients. Therefore,
7. The product of the GCF of the
coefficients and all common
variable factors.

6.1 Introduction to Factoring 907


Chapter 6 Factoring and Solving by Factoring

Answer: 2x 2 y 3 z

It is worth pointing out that the GCF in the previous example divides both
expressions evenly:

Furthermore, we can write the following:

The factors 3x 3 and 4z share no common monomial factors other than 1; they are
relatively prime.

Example 4: Determine the GCF of the following expressions: 30x 6 y and 18x 4 y 2 z .

Solution: The prime factorizations of the coefficients are

Thus the GCF(30, 18) = 2 ⋅ 3 = 6. Next, consider the variable part:

6.1 Introduction to Factoring 908


Chapter 6 Factoring and Solving by Factoring

The variable factors in common are x 4 and y. The factor z is not in common and we
have

Answer: 6x 4 y

Example 5: Determine the GCF of the following three expressions: 12a5 b2 (a + b) ,


5

60a4 b3 c(a + b) , and 24a2 b7 c3 (a + b) .


3 2

Solution: First, determine the GCF of the coefficients.

The GCF (12, 60, 24) = 22 ⋅ 3 = 12. Next, determine the common factors of the
variable part:

The variable factors in common are a2 , b2 , and (a + b) . Therefore,


2

6.1 Introduction to Factoring 909


Chapter 6 Factoring and Solving by Factoring

Answer: 12a2 b2 (a + b) . Note that the variable c is not common to all three
2

expressions and thus is not included in the GCF.

Try this! Determine the GCF of the following: 60x 4 y 3 (x + 2y) ,


7

45x 2 y 5 (x + 2y) , and 30x 7 y 7 (x + 2y) .


4 3

Answer: 15x 2 y 3 (x + 2y)


3

Video Solution

(click to see video)


Factoring out the GCF

We have seen that application of the distributive property is the key to multiplying
polynomials. The process of factoring a polynomial8 involves using the
distributive property in reverse to write each polynomial as a product of
polynomial factors.

To demonstrate this idea, we multiply and factor side by side. Factoring utilizes the
GCF of the terms.

8. The process of rewriting a


polynomial as a product of
polynomial factors.

6.1 Introduction to Factoring 910


Chapter 6 Factoring and Solving by Factoring

polynomial 6x 5 + 8x 2 as a product of the two factors 2x 2 and (3x 3 + 4). Note


In the previous example, we see that the distributive property allows us to write the

that in this case, 2x 2 is the GCF of the terms of the polynomial:

Factoring out the GCF9 involves rewriting a polynomial as a product where a factor
is the GCF of all of its terms:

The steps for factoring out the GCF of a polynomial10 are outlined in the following
example.

Example 6: Factor out the GCF: 7x 4 + 21x 3 − 14x 2 .

Solution:

Step 1: Identify the GCF of all the terms. In this case, the GCF(7, 21, 14) = 7, and the
common variable factor with the smallest exponent is x 2 . The GCF of the
polynomial is 7x 2 .

Step 2: Determine the terms of the missing factor by dividing each term of the
original expression by the GCF. (This step is usually performed mentally.)
9. The process of rewriting a
polynomial as a product using
the GCF of all of its terms.

10. The greatest common factor of


all the terms of the polynomial.

6.1 Introduction to Factoring 911


Chapter 6 Factoring and Solving by Factoring

Step 3: Apply the distributive property (in reverse) using the terms found in the
previous step.

Step 4: As a check, multiply using the distributive property to verify that the
product equals the original expression. (This step is optional and can be performed
mentally.)

Answer: 7x 2 (x 2 + 3x − 2)

Example 7: Factor out the GCF: 48a − 16b + 4c.

Solution: There are no variable factors in common and the GCF(48, 16, 4) = 4.

Answer: 4 (12a − 4b + c)

6.1 Introduction to Factoring 912


Chapter 6 Factoring and Solving by Factoring

Example 8: Factor out the GCF: 25x 3 + 15x 2 + 5x.

Solution: The GCF(25, 15, 5) = 5, and the common variable factor with smallest
exponents is x 1 . The GCF of all the terms is 5x .

Answer: 5x (5x 2 + 3x + 1)

If the GCF is the same as one of the terms, then, after the GCF is factored out, a
constant term 1 will remain. In the previous example, we can see that 5x 5x
= 1. The
importance of remembering the constant term becomes clear when performing the
check using the distributive property:

The constant term 1 allows us to obtain the same original expression after we
distribute.

Example 9: Factor out the GCF: 15x 6 y 4 + 10x 5 y 3 z 2 − 20xy 6 z 3.

Solution: The GCF(10, 15, 20) = 5, and the common variables with smallest
exponent are x 1 and y 3 . Therefore, the GCF of the terms is 5xy 3 . The first term
does not have a variable factor of z and thus cannot be a part of the greatest
common factor. If we divide each term by 5xy 3 , we obtain

6.1 Introduction to Factoring 913


Chapter 6 Factoring and Solving by Factoring

and can write

Answer: 5xy 3 (3x 5 y + 2x 4 z 2 − 4y 3 z 3 )

Example 10: Factor out the GCF: 24a6 b2 c5 + 8a7 b5 c.

Solution: The GCF(24, 8) = 8, and the variable factors with smallest exponents are
a6 , b2 , and c. Therefore, the GCF of all the terms is 8a6 b2 c.

Answer: 8a6 b2 c (3c4 + ab3 )

Of course, not all polynomials with integer coefficients can be factored as a product
of polynomials with integer coefficients other than 1 and itself. If this is the case,
then we say that it is a prime polynomial11.

11. A polynomial with integer


coefficients that cannot be
factored as a product of
polynomials with integer
coefficients other than 1 and Example 11: Factor: 3x − 5 .
itself.

6.1 Introduction to Factoring 914


Chapter 6 Factoring and Solving by Factoring

Solution: Prime: there are no polynomial factors other than 1 and itself.

Answer: Prime

Try this! Factor out the GCF: 16x 4 y 3 − 8x 2 y 5 − 4x 2 y.

Answer: 4x 2 y (4x 2 y 2 − 2y 4 − 1)

Video Solution

(click to see video)


Factor by Grouping

In this section, we outline a technique for factoring polynomials with four terms.
First, review some preliminary examples where the terms have a common binomial
factor.

Example 12: Factor: 5x (x − 3) + 2 (x − 3).

Solution: This expression is a binomial with terms 5x (x − 3) and 2 (x − 3). In this


case, (x − 3) is a common factor. Begin by factoring this common factor out:

To determine the terms of the remaining factor, divide each term by (x − 3):

This step is typically performed mentally. We have

6.1 Introduction to Factoring 915


Chapter 6 Factoring and Solving by Factoring

Answer: (x − 3) (5x + 2)

Recall that 1 is always a common factor. If the GCF is the same as a term, then the
factor 1 remains after we factor out that GCF.

Example 13: Factor: 3x (4x + 1) − (4x + 1).

Solution: Rewrite the second term −(4x + 1) as −1(4x + 1) and then factor out
the common binomial factor (4x + 1).

Answer: (4x + 1) (3x − 1)

Remember that the goal for this section is to develop a technique that enables us to
factor polynomials with four terms into a product of binomials. The intermediate
step of this process looks like the previous two examples. For example, we wish to
factor

6.1 Introduction to Factoring 916


Chapter 6 Factoring and Solving by Factoring

Begin by grouping the first two terms and the last two terms. Then factor out the
GCF of each grouping:

In this form, it is a binomial with a common binomial factor, (x − 3).

The steps that follow outline a technique for factoring four-term polynomials called
factor by grouping12.

Example 14: Factor: 2x 3 + 4x 2 + 3x + 6.

Solution: Group terms in such a way as to obtain a binomial with common factors.

Step 1: Group the first two and last two terms and then factor out the GCF of each.

The GCF of the first two terms is 2x 2 , and the GCF of the second two terms is 3.

12. A technique for factoring


polynomials with four terms.

6.1 Introduction to Factoring 917


Chapter 6 Factoring and Solving by Factoring

Step 2: At this point, the polynomial is a binomial. Factor out any factors common
to both terms. Here (x + 2) is a common factor.

Step 3: Optional check: multiply to verify that we obtain the original expression.

Answer: (x + 2) (2x 2 + 3)

Example 15: Factor: 2a3 − 3a2 + 2a − 3.

Solution: The GCF of the first two terms is a2 and the GCF of the second two terms
is 1.

6.1 Introduction to Factoring 918


Chapter 6 Factoring and Solving by Factoring

Answer: (2a − 3) (a2 + 1). The check is left to the reader.

Example 16: Factor: 6x 4 − 24x 3 − 5x + 20.

Solution: The GCF for the first group is 6x 3 . We have to choose 5 or −5 to factor out
of the second group.

Factoring out a +5 does not result in a common binomial factor. If we choose to


factor out −5, then we obtain a common binomial factor and can proceed. Note that
when factoring out a negative number, we change the signs of the factored terms.

6.1 Introduction to Factoring 919


Chapter 6 Factoring and Solving by Factoring

Answer: (x − 4) (6x 3 − 5). The check is left to the reader.

Tip

The sign of the leading coefficient in the second grouping usually indicates
whether or not to factor out a negative factor. If that coefficient is positive,
factor out a positive factor. If it is negative, factor out a negative factor.

When all the terms of a polynomial have a GCF other than 1, it is a best practice to
factor that out before factoring by grouping.

Example 17: Factor: 3y 4 + 9y 2 − 6y 3 − 18y.

Solution: Here we notice that the greatest common factor of all the terms is 3y .
Begin by factoring out the GCF and then factor the result by grouping.

6.1 Introduction to Factoring 920


Chapter 6 Factoring and Solving by Factoring

Answer: 3y (y 2 + 3) (y − 2)

Sometimes we must first rearrange the terms in order to obtain a common factor.

Example 18: Factor: ab − 2a2 b + a3 − 2b3.

Solution: Simply factoring the GCF out of the first group and last group does not
yield a common binomial factor.

We must rearrange the terms, searching for a grouping that produces a common
factor. In this example, we have a workable grouping if we switch the terms a3 and
ab.

6.1 Introduction to Factoring 921


Chapter 6 Factoring and Solving by Factoring

Answer: (a − 2b) (a2 + b)

Not all factorable four-term polynomials can be factored with this technique. For
example,

This four-term polynomial cannot be grouped in any way as to produce a common


binomial factor. Despite this, the polynomial is not prime and can be written as a
product of polynomials. It can be factored as follows:

Factoring such polynomials is something that we will learn to do as we move


further along in our study of algebra. For now, we will limit our attempt to factor
four-term polynomials to using the factor by grouping technique.

Try this! Factor: x 3 − x 2 y − xy + y 2 .

Answer: (x − y) (x 2 − y)

Video Solution

(click to see video)

6.1 Introduction to Factoring 922


Chapter 6 Factoring and Solving by Factoring

KEY TAKEAWAYS

• To find the greatest common factor (GCF) of any collection of natural


numbers, first find the prime factorization of each. The GCF is the
product of all the common prime factors.
• The GCF of two or more monomials is the product of the GCF of the
coefficients and the common variable factors with the smallest power.
• If the terms of a polynomial have a greatest common factor, then factor
out that GCF using the distributive property. Divide each term of the
polynomial by the GCF to determine the terms of the remaining factor.
• Some four-term polynomials can be factored by grouping the first two
terms and the last two terms. Factor out the GCF of each group and then
factor out the common binomial factor.
• When factoring by grouping, you sometimes have to rearrange the
terms to find a common binomial factor. After factoring out the GCF, the
remaining binomial factors must be the same for the technique to work.
• Not all polynomials can be factored as the product of polynomials with
integer coefficients. In this case, we call it a prime polynomial.

6.1 Introduction to Factoring 923


Chapter 6 Factoring and Solving by Factoring

TOPIC EXERCISES

Part A: GCF of Natural Numbers

Give the prime factorization of each number and determine the GCF.

1. 18, 24

2. 45, 75

3. 72, 60

4. 168, 175

5. 144, 245

6. 15, 50, 60

7. 14, 63, 70

8. 12, 48, 125

9. 60, 72, 900

10. 252, 336, 360

Part B: GCF of Variable Expressions

Determine the GCF of all the terms.

11. 15x, 30

12. 14x, 21

13. 45x 4 , 8x 3

14. 36x 5 , 35y 2

15. 6x, 27x, 36x

6.1 Introduction to Factoring 924


Chapter 6 Factoring and Solving by Factoring

16. 12x 3 , 4x 2 , 6x

17. 12x 2 y, 60xy 3


2
18. 7ab , 2a2 b, 3a3 b 3
2
19. 6a2 b , 18a3 b 2 , 9ab 2

20. 15x (x + 2) , 9 (x + 2)

21. 20x (2x − 1) , 16 (2x − 1)

22. 20x 3 (x + y) , 10x 5 (x + y)


5 2

Part C: Factoring out the GCF

Given the GCF, determine the missing factor.

23. 25x 2 + 10x = 5x ( ? )

24. 12y 5 + 7y 2 = y 2 ( ? )

25. 22x 4 − 121x 2 + 11x = 11x ( ? )

26. 30y 3 − 45y 2 − 3y = 3y ( ? )

27. 36a5 b
7
− 60a6 b 5 = 12a5 b 5 ( ? )

28. 24x 2 y + 48xy 2 − 12xy = 12xy ( ? )

Factor out the GCF.

29. 4x −8

30. 27x −9

31. 3x − 18

6.1 Introduction to Factoring 925


Chapter 6 Factoring and Solving by Factoring

32. 5x − 10

33. 25x − 16

34. 72x − 35

35. 15x 2 + 30x

36. 14a2 − 7a

37. 30a5 − 10a2

38. 8x 4 − 16x 2

39. 5x 6 + x3

40. 3x 7 − 9x 5

41. 18a2 + 30a − 6

42. 24a2 − 36a − 12

43. 27x 3 − 6x 2 + 3x

44. 8x 3 − 12x 2 + 2x

45. 9x 4 + 18x 3 − 3x 2

46. 12y 4 − 16y 3 + 20y 2

47. 7x 5 − 21x 3 − 14x 2 + 28x

48. 36y 10 + 12y 8 − 18y 4 − 6y 3

49. 12x 5 y 2 − 8x 3 y
4 3
50. 125a8 b c − 25a2 b 3 c3

6.1 Introduction to Factoring 926


Chapter 6 Factoring and Solving by Factoring

51. 6x 4 y 3 − 4x 3 y 2 + 8x 2 y

52. 15x 4 y 2 − 30x 3 y 3 + 15x 2 y 4

53. 81x 7 y 6 z 2 − 18x 2 y 8 z 4 + 9x 2 y 5 z 2

54. 4x 5 y 4 z 9 + 26x 5 y 3 z 4 − 14x 6 y 8 z 5

55. 2x (x − 3) + 5 (x − 3)

56. 3x (2x + 1) − 4 (2x + 1)

57. 5x (5x + 2) − (5x + 2)

58. 2x (3x + 4) + (3x + 4)

59. x 2 (4x − 7) − 5 (4x − 7)

60. (x + 6) − 3x 2 (x + 6)

61. (a + b) − 3a(a + b)
2 2

62. (ab + 2) + 3ab(ab + 2)


3 3

63. 7x(x + 7) 5 + 14x 2 (x + 7) 5

64. 36x 5 (3x + 2) 4 − 12x 3 (3x + 2) 4

Are the following factored correctly? Check by multiplying.

65. 4x 2 − 16x = 4x (x − 4)

66. 3a3 − 3a = 3a (a2 )

67. 3x 3 − 5x 6 = x 3 (3 − x 2 )

6.1 Introduction to Factoring 927


Chapter 6 Factoring and Solving by Factoring

68. 5x 3 − 10x 4 + 15x 5 = 5x 3 (1 − 2x + 3x 2 )

69. x 3 − x 2 + x = x (x 2 − x)

70. 12x 4 y 3 − 16x 5 y 2 + 8x 6 y 7 = 4x 4 y 2 (3y − 4x + 2x 2 y 5 )

Use polynomial long division to show that the given factor divides the polynomial
evenly.

71. Show that (x − 1) is a factor of (2x 3 − 5x 2 + 4x − 1) .

72. Show that (x + 3) is a factor of (3x 3 + 7x 2 − 4x + 6) .

73. Show that (3x − 2) is a factor of (3x 3 + 4x 2 − 7x + 2) .

74. Show that (2x + 1) is a factor of (2x 3 − 5x 2 + x + 2) .

75. The height in feet of an object tossed into the air is given by the function
h (t) = −16t 2 + 32t, where t is the time in seconds after it is tossed.
Write the function in factored form.

76. The height in feet of an object dropped from a 16‑foot ladder is given by
the function h (t) = −16t 2 + 16 , where t is the time in seconds after it
is tossed. Write the function in factored form.

77. The surface area of a cylinder is given by the formula


SA = 2πr2 + 2πrh , where r represents the radius of the base and h is
the height of the cylinder. Express this formula in factored form.

78. The surface area of a cone is given by the formula SA = πr2 + πrs,
where r represents the radius of the base and s represents the slant height.
Express this formula in factored form.

6.1 Introduction to Factoring 928


Chapter 6 Factoring and Solving by Factoring

Part D: Factor by Grouping

Factor by grouping.

79. x 2 − 10x + 2x − 20

80. x 2 − 6x − 3x + 18

81. x 3 + 2x 2 + 2x + 4

82. x 3 − 3x 2 + 5x − 15

83. x 3 + 7x 2 − 2x − 14

84. 2x 3 + 2x 2 − x − 1

85. x 3 − 5x 2 + 4x − 20

86. 6x 3 − 3x 2 + 2x − 1

87. 9x 3 − 6x 2 − 3x + 2

88. 2x 4 − x 3 − 6x + 3

89. x 5 + x 3 + 2x 2 + 2

90. 6x 5 − 4x 3 − 9x 2 + 6

91. 3a3 b + 3ab 2 + 2a2 + 2b

92. 2a3 + 2ab 3 − 3a2 b − 3b 4

6.1 Introduction to Factoring 929


Chapter 6 Factoring and Solving by Factoring

93. 2a3 − a2 b 2 − 2ab + b 3

94. a4 − 3a3 b 2 + ab 2 − 3b 4
4
95. 3a2 b − 6b 3 − a2 b + 2

96. 3x 3 + 2y 3 + x 3 y 3 + 6

97. −3x 3 − 5y 3 + x 3 y 3 + 15

98. 2x 3 y 3 + 2 − y 3 − 4x 3

99. 3x 2 − y 3 + xy 2 − 3xy

100. 2x 2 + y 3 − 2xy − xy 2

Factor out the GCF first and then factor by grouping.

101. 5x 2 − 35x − 15x + 105

102. 12x 2 − 30x − 12x + 30

103. 2x 3 + 6x 2 − 10x − 30

104. 6x 3 − 3x 2 − 42x + 21

105. 4x 4 + 4x 3 − 12x 2 − 12x

106. −9x 4 + 6x 3 − 45x 2 + 30x

107. −12x 5 + 4x 4 + 6x 3 − 2x 2

108. 24x 5 − 36x 4 + 8x 3 − 12x 2


2
109. 24a3 b − 60a3 b + 40ab 2 − 100ab
2
110. a4 b − 2a3 b 3 + a2 b 3 − 2ab 4

6.1 Introduction to Factoring 930


Chapter 6 Factoring and Solving by Factoring

Part E: Discussion Board Topics

111. Research the Euclidean algorithm for finding the GCF of two natural
numbers. Give an example that illustrates the steps.

112. Research and discuss the contributions of Euclid of Alexandria.

113. Explain what factoring is and give an example.

114. Is 5x(x + 2) − 3(x + 2) fully factored? Explain.

115. Make up a factoring problem of your own and provide the answer. Post
the problem and the solution on the discussion board.

6.1 Introduction to Factoring 931


Chapter 6 Factoring and Solving by Factoring

ANSWERS

1: 18 = 2 ⋅ 3 2 , 24 = 2 3 ⋅ 3, GCF = 6

3: 72 = 2 3 ⋅ 3 2, 60 = 2 2 ⋅ 3 ⋅ 5, GCF = 12

5: 144 = 2 4 ⋅ 3 2 , 245 = 5 ⋅ 7 2 , GCF = 1

7: 14 = 2 ⋅ 7, 63 = 3 2 ⋅ 7, 70 = 2 ⋅ 5 ⋅ 7, GCF = 7

9: 60 = 2 2 ⋅ 3 ⋅ 5, 72 = 2 3 ⋅ 3 2, 900 = 2 2 ⋅ 3 2 ⋅ 5 2, GCF = 12

11: 15

13: x 3

15: 3x

17: 12xy

2
19: 3ab

21: 4(2x − 1)

23: (5x + 2)

25: (2x 3 − 11x + 1)

27: (3b − 5a)


2

29: 4 (x − 2)

31: 3(x − 6)

33: Prime

35: 15x (x + 2)

6.1 Introduction to Factoring 932


Chapter 6 Factoring and Solving by Factoring

37: 10a2 (3a − 1)


3

39: x 3 (5x + 1)
3

41: 6 (3a2 + 5a − 1)

43: 3x (9x 2 − 2x + 1)

45: 3x 2 (3x + 6x − 1)
2

(x − 3x − 2x + 4)
4 2
47: 7x

49: 4x 3 y (3x 2 y − 2)

51: 2x 2 y (3x 2 y 2 − 2xy + 4)

53: 9x 2 y 5 z 2 (9x y − 2y z + 1)
5 3 2

55: (x − 3)(2x + 5)

57: (5x + 2)(5x − 1)

59: (4x − 7)(x 2 − 5)

61: (a + b)2 (1 − 3a)

63: 7x(x + 7) 5 (1 + 2x)

65: Yes

67: No

69: No

75: h(t) = −16t(t − 2)

6.1 Introduction to Factoring 933


Chapter 6 Factoring and Solving by Factoring

77: SA = 2πr (r + h)

79: (x − 10) (x + 2)

81: (x + 2) (x 2 + 2)

83: (x + 7) (x 2 − 2)

85: (x − 5) (x 2 + 4)

87: (3x − 2) (3x 2 − 1)

89: (x 2 + 1) (x 3 + 2)

91: (a2 + b) (3ab + 2)

93: (a2 − b) (2a − b 2 )

95: (3b − 1) (a2 b − 2)


3

97: (x 3 − 5) (y 3 − 3)

99: (x − y) (3x + y 2 )

101: 5 (x − 7) (x − 3)

103: 2 (x + 3) (x 2 − 5)

105: 4x (x + 1) (x 2 − 3)

107: −2x 2 (3x − 1) (2x 2 − 1)

109: 4ab (3a2 + 5) (2b − 5)

6.1 Introduction to Factoring 934


Chapter 6 Factoring and Solving by Factoring

6.2 Factoring Trinomials of the Form x^2 + bx + c

LEARNING OBJECTIVES

1. Factor trinomials of the form x 2 + bx + c.


2. Factor trinomials using the AC method.

Factoring Trinomials of the Form x^2 + bx + c

Some trinomials of the form x 2 + bx + c can be factored as a product of binomials.


For example,

We can verify this factorization by multiplying:

Factoring trinomials requires that we work the distributive process in reverse.


Notice that the product of the first terms of each binomial is equal to the first term
of the trinomial.

The middle term of the trinomial, 7x, is the sum of the products of the outer and
inner terms of the binomials:

935
Chapter 6 Factoring and Solving by Factoring

And the product of the last terms of each binomial is equal to the last term of the
trinomial.

This can be visually interpreted as follows:

If a trinomial of this type factors, then these relationships will be true:

This gives us

13. Describes the method of In short, if the leading coefficient of a factorable trinomial is one, then the factors
factoring a trinomial by of the last term must add up to the coefficient of the middle term. This observation
systematically checking factors is the key to factoring trinomials using the technique known as trial and error13
to see if their product is the
(or guess and check14). The steps are outlined in the following example.
original trinomial.

14. Used when referring to the


trial and error method for
factoring trinomials.

6.2 Factoring Trinomials of the Form x^2 + bx + c 936


Chapter 6 Factoring and Solving by Factoring

Example 1: Factor: x 2 + 7x + 12.

Solution: Note that the polynomial to be factored has three terms; it is a trinomial
with a leading coefficient of 1. Use trial and error to factor as follows:

Step 1: Write two sets of blank parentheses. If a trinomial of this form factors, then
it will factor into two linear binomial factors.

Step 2: Write the factors of the first term in the first space of each set of
parentheses. In this case, factor x 2 = x ⋅ x .

Step 3: Determine the factors of the last term whose sum equals the coefficient of
the middle term. To do this, list all of the factorizations of 12 and search for factors
whose sum equals the coefficient of the middle term, 7.

Choose 12 = 3 ⋅ 4 because 3 + 4 = 7.

Step 4: Write in the last term of each binomial using the factors determined in the
previous step.

Step 5: Check by multiplying the two binomials.

6.2 Factoring Trinomials of the Form x^2 + bx + c 937


Chapter 6 Factoring and Solving by Factoring

Answer: (x + 3) (x + 4)

Since multiplication is commutative, the order of the factors does not matter.

If the last term of the trinomial is positive, then either both of the constant factors
must be negative or both must be positive. Therefore, when looking at the list of
factorizations of the last term, we are searching for sums that are equal to the
coefficient of the middle term.

Example 2: Factor: x 2 − 9x + 20.

Solution: First, factor x 2 = x ⋅ x .

Next, determine which factors of 20 add up to −9:

6.2 Factoring Trinomials of the Form x^2 + bx + c 938


Chapter 6 Factoring and Solving by Factoring

In this case, choose −4 and −5 because (−4) (−5) = +20and −4 + (−5) = −9.

Check.

Answer: (x − 4) (x − 5)

If the last term of the trinomial is negative, then one of its factors must be negative.
In this case, search the list of factorizations of the last term for differences that
equal the coefficient of the middle term.

Example 3: Factor: x 2 − 4x − 12.

Solution: Begin by factoring the first term x 2 = x ⋅ x .

The factors of 12 are listed below. In this example, we are looking for factors whose
difference is −4.

6.2 Factoring Trinomials of the Form x^2 + bx + c 939


Chapter 6 Factoring and Solving by Factoring

obtained if we add 2 + (−6).


Here choose the factors 2 and −6 because the coefficient of the middle term, −4, is

Multiply to check.

Answer: (x + 2) (x − 6)

Often our first guess will not produce a correct factorization. This process may
require repeated trials. For this reason, the check is very important and is not
optional.

Example 4: Factor: x 2 + 5x − 6.

Solution: The first term of this trinomial x 2 factors as x ⋅ x .

Consider the factors of 6:

6.2 Factoring Trinomials of the Form x^2 + bx + c 940


Chapter 6 Factoring and Solving by Factoring

Suppose we choose the factors 2 and 3 because 2 + 3 = 5, the coefficient of the middle
term. Then we have the following incorrect factorization:

When we multiply to check, we find the error.

In this case, the middle term is correct but the last term is not. Since the last term
in the original expression is negative, we need to choose factors that are opposite in
sign. Therefore, we must try again. This time we choose the factors −1 and 6 because
−1 + 6 = 5.

Now the check shows that this factorization is correct.

Answer: (x − 1) (x + 6)

If we choose the factors wisely, then we can reduce much of the guesswork in this
process. However, if a guess is not correct, do not get discouraged; just try a
different set of factors.

6.2 Factoring Trinomials of the Form x^2 + bx + c 941


Chapter 6 Factoring and Solving by Factoring

Example 5: Factor: x 2 + 3x + 20.

Solution:

Here there are no factors of 20 whose sum is 3. Therefore, the original trinomial
cannot be factored as a product of two binomials. This trinomial is prime.

Answer: Prime

Try this! Factor: x 2 − 13x − 30.

Answer: (x + 2) (x − 15)

Video Solution

(click to see video)

The techniques described can also be used to factor trinomials with more than one
variable.

Example 6: Factor: x 2 − 14xy − 72y 2 .

Solution: The first term x 2 factors as x ⋅ x .

6.2 Factoring Trinomials of the Form x^2 + bx + c 942


Chapter 6 Factoring and Solving by Factoring

Next, look for factors of the coefficient of the last term, 72, whose sum is −14.

Therefore, the coefficient of the last term can be factored −72 = 4 (−18), where

4y (−18y) and try the following factorization:


4 + (−18) = −14. Because the last term has a variable factor of y 2 , factor 72y 2 as

Multiply to check.

Visually, we have the following:

6.2 Factoring Trinomials of the Form x^2 + bx + c 943


Chapter 6 Factoring and Solving by Factoring

Answer: (x + 4y) (x − 18y)

Try this! Factor: x 2 y 2 + 9xy − 10 .

Answer: (xy − 1) (xy + 10)

Video Solution

(click to see video)


Factoring Using the AC Method

An alternate technique for factoring trinomials, called the AC method15, makes use
of the grouping method for factoring four-term polynomials. If a trinomial in the
form ax 2 + bx + c can be factored, then the middle term, bx, can be replaced with
two terms with coefficients whose sum is b and product ac. This substitution results
in an equivalent expression with four terms that can be factored by grouping. The
steps are outlined in the following example.

Example 7: Factor using the AC method: x 2 − x − 30.

15. Method for factoring Solution: In this example a = 1, b = −1, and c = −30.
trinomials by replacing the
middle term with two terms
that allow us to factor the Step 1: Determine the product ac.
resulting four-term polynomial
by grouping.

6.2 Factoring Trinomials of the Form x^2 + bx + c 944


Chapter 6 Factoring and Solving by Factoring

Step 2: Find factors of ac whose sum equals the coefficient of the middle term, b.

We can see that the sum of the factors 5 and −6 is equal to the coefficient of the
middle term, −1.

Step 3: Use the factors as coefficients for the terms that replace the middle term.
Here −x = −6x + 5x . Write

Step 4: Factor the equivalent expression by grouping.

Answer: (x − 6) (x + 5)

Notice that the AC method is consistent with the trial and error method. Both
methods require that b = m + n, where c = mn. In the example above,

6.2 Factoring Trinomials of the Form x^2 + bx + c 945


Chapter 6 Factoring and Solving by Factoring

−30 = (−6) (5)and −1 = (−6) + 5. The only difference between the


methods, when the leading coefficient is 1, is in the process used to obtain the final
factorization.

Example 8: Factor: y 2 − 14x + 48.

Solution: Here ac = 48 and we search for factors whose sum is −14.

Therefore, −14x = −6x − 8x . Substitute the new terms and factor by grouping.

Answer: (x − 6) (x − 8). The check is left to the reader.

At this point, it is recommended that the reader stop and factor as many trinomials
of the form x 2 + bx + c as time allows before moving on to the next section.
Factoring trinomials is one of the more important skills that we learn in this course
and should be mastered.

6.2 Factoring Trinomials of the Form x^2 + bx + c 946


Chapter 6 Factoring and Solving by Factoring

KEY TAKEAWAYS

• Factor a trinomial by systematically guessing what factors give two


binomials whose product is the original trinomial.
• If a trinomial of the form x 2 + bx + c factors into the product of two
binomials, then the coefficient of the middle term is the sum of factors
of the last term.
• Not all trinomials can be factored as the product of binomials with
integer coefficients. In this case, we call it a prime trinomial.
• Factoring is one of the more important skills required in algebra. For
this reason, you should practice working as many problems as it takes to
become proficient.

6.2 Factoring Trinomials of the Form x^2 + bx + c 947


Chapter 6 Factoring and Solving by Factoring

TOPIC EXERCISES

Part A: Factoring Trinomials with Leading Coefficient 1

Are the following factored correctly? Check by multiplying.

1. x 2 + 5x − 6 = (x + 2) (x + 3)

2. x 2 + 6x + 16 = (x + 8) (x − 2)

3. y 2 + 2y − 8 = (y + 4) (y − 2)

4. y 2 − 10y + 21 = (y − 3) (y − 7)

− 10a + 25 = (a − 5)
2
5. a2

6. a2 + 6a + 9 = (a − 3)2

7. x 2 + 10x − 25 = (x + 5) (x − 5)

8. x 2 + 5x + 14 = (x − 2) (x + 7)

9. y 2 + 50y − 600 = (y + 60) (y − 10)

10. y 2 − 3y + 2 = (y − 2) (y − 1)

Factor.

11. x 2 + 6x + 8

12. x 2 + 4x + 3

13. x 2 + 3x + 2

14. x 2 +x−2

6.2 Factoring Trinomials of the Form x^2 + bx + c 948


Chapter 6 Factoring and Solving by Factoring

15. x 2 + 3x − 10

16. x 2 − 2x − 35

17. x 2 − 13x + 12

18. x 2 − 15x + 36

19. x 2 − 12x + 36

20. x 2 + 18x + 81

21. x 2 − 2x + 1

22. x 2 − 18x + 81

23. x 2 + 5x + 5

24. x 2 − 4x + 6

25. x 2 − 20x + 91

26. x 2 + 20x + 91

27. x 2 − 2x − 48

28. x 2 + 16x + 48

29. x 2 + 22x + 48

30. x 2 + 22x − 48

31. y 2 + 7y + 12

32. y 2 + 8y − 20

33. y 2 − 16y + 60

6.2 Factoring Trinomials of the Form x^2 + bx + c 949


Chapter 6 Factoring and Solving by Factoring

34. y 2 − 31y − 32

35. a2 − 11a − 42

36. a2 − 14a − 51

37. a2 + 26a + 25

38. a2 − 22a + 120

39. a2 + 4a − 1

40. a2 − 6a + 2

41. y 2 − 14x + 40

42. y 2 − 4y − 96

43. x 2 − 2xy + y 2

44. x 2 + 2xy + y 2

45. x 2 − 16xy + 15y 2

46. x 2 − 4xy − 32y 2

47. x 2 + 2xy − 15y 2

48. x 2 − 12xy + 32y 2

49. x 2 y 2 − 6xy + 9

50. x 2 y 2 + 25xy − 26

51. a2 + 4ab + 4b 2

52. a2 − 19ab − 20b 2

6.2 Factoring Trinomials of the Form x^2 + bx + c 950


Chapter 6 Factoring and Solving by Factoring

53. a2 − ab − 12b 2

54. a2 − 10ab − 56b 2


2
55. a2 b − 2ab − 15
2
56. a2 b − 10ab + 24

57. The area of a square is given by the function


A(x) = x 2 − 14x + 49 , where x is measured in meters. Rewrite this
function in factored form.

58. The area of a square is given by the function


A(x) = x 2 + 16x + 64 , where x is measured in meters. Rewrite this
function in factored form.

Part B: Factor Using the AC Method

Factor using the AC method.

59. x 2 + 5x − 14

60. x 2 + 2x − 48

61. x 2 − 9x + 8

62. x 2 − 14x + 24

63. x 2 − x − 72

64. x 2 − x − 90

65. y 2 − 8y + 16

66. y 2 + 16y + 64

67. x 2 + 4x + 12

6.2 Factoring Trinomials of the Form x^2 + bx + c 951


Chapter 6 Factoring and Solving by Factoring

68. x 2 + 5x − 8

69. x 2 + 3xy − 18y 2

70. x 2 − 15xy + 50y 2

Part C: Discussion Board Topics

71. Create your own trinomial of the form x 2 + bx + c that factors. Share
it along with the solution on the discussion board.

72. Write out your own list of steps for factoring a trinomial of the form
x 2 + bx + c and share your steps on the discussion board.

73. Create a trinomial that does not factor and share it along with an
explanation of why it does not factor.

6.2 Factoring Trinomials of the Form x^2 + bx + c 952


Chapter 6 Factoring and Solving by Factoring

ANSWERS

1: No

3: Yes

5: Yes

7: No

9: Yes

11: (x + 2) (x + 4)

13: (x + 1) (x + 2)

15: (x − 2) (x + 5)

17: (x − 1) (x − 12)

19: (x − 6)
2

21: (x − 1) 2

23: Prime

25: (x − 7) (x − 13)

27: (x + 6) (x − 8)

29: Prime

31: (y + 3) (y + 4)

33: (y − 6) (y − 10)

35: (a + 3) (a − 14)

6.2 Factoring Trinomials of the Form x^2 + bx + c 953


Chapter 6 Factoring and Solving by Factoring

37: (a + 1) (a + 25)

39: Prime

41: (y − 10) (y − 4)

43: (x − y)
2

45: (x − 15y) (x − y)

47: (x + 5y) (x − 3y)

49: (xy − 3)
2

51: (a + 2b)
2

53: (a + 3b) (a − 4b)

55: (ab + 3) (ab − 5)

57: A(x) = (x − 7) 2

59: (x + 7) (x − 2)

61: (x − 8) (x − 1)

63: (x − 9) (x + 8)

65: (y − 4)
2

67: Prime

69: (x + 6y) (x − 3y)

6.2 Factoring Trinomials of the Form x^2 + bx + c 954


Chapter 6 Factoring and Solving by Factoring

6.3 Factoring Trinomials of the Form ax^2 + bx + c

LEARNING OBJECTIVES

1. Factor trinomials of the form ax 2 + bx + c.


2. Factor trinomials with a common factor.

Factoring Trinomials of the Form ax^2 + bx + c

Factoring trinomials of the form ax 2 + bx + c can be challenging because the


middle term is affected by the factors of both a and c. To illustrate this, consider the
following factored trinomial:

We can multiply to verify that this is the correct factorization.

As we have seen before, the product of the first terms of each binomial is equal to
the first term of the trinomial. The middle term of the trinomial is the sum of the
products of the outer and inner terms of the binomials. The product of the last
terms of each binomial is equal to the last term of the trinomial. Visually, we have
the following:

955
Chapter 6 Factoring and Solving by Factoring

In general,

This gives us,

In short, when the leading coefficient of a trinomial is something other than 1,


there will be more to consider when determining the factors using the trial and

obtained. Multiply (2x + 5) (3x + 7) and carefully follow the formation of the
error method. The key lies in the understanding of how the middle term is

middle term.

If we think of the FOIL method for multiplying binomials, then the middle term
results from the sum of the inner product and the outer product. In this case,
14x + 15x = 29x , as illustrated below:

6.3 Factoring Trinomials of the Form ax^2 + bx + c 956


Chapter 6 Factoring and Solving by Factoring

For this reason, we need to look for products of the factors of the first and last
terms whose sum is equal to the coefficient of the middle term. For example, to
factor 6x 2 + 29x + 35, look at the factors of 6 and 35.

The combination that produces the coefficient of the middle term is


2 ⋅ 7 + 3 ⋅ 5 = 14 + 15 = 29. Make sure that the outer terms have coefficients 2
and 7, and that the inner terms have coefficients 5 and 3. Use this information to
factor the trinomial:

Example 1: Factor: 3x 2 + 7x + 2.

Solution: Since the leading coefficient and the last term are both prime, there is
only one way to factor each.

Begin by writing the factors of the first term, 3x 2 , as follows:

6.3 Factoring Trinomials of the Form ax^2 + bx + c 957


Chapter 6 Factoring and Solving by Factoring

The middle and last term are both positive; therefore, the factors of 2 are chosen as
positive numbers. In this case, the only choice is in which grouping to place these
factors.

Determine which grouping is correct by multiplying each expression.

Notice that these products differ only in their middle terms. Also, notice that the
middle term is the sum of the inner and outer product, as illustrated below:

Answer: (x + 2) (3x + 1)

6.3 Factoring Trinomials of the Form ax^2 + bx + c 958


Chapter 6 Factoring and Solving by Factoring

Example 2: Factor: 12x 2 + 38x + 20.

Solution: First, consider the factors of the first and last terms.

We search for products of factors whose sum equals the coefficient of the middle
term, 38. For brevity, the thought process is illustrated starting with the factors 2
and 6. Factoring begins at this point with the first term.

We search for factors of 20 that along with the factors of 12 produce a middle term
of 38x.

Here the last combination produces a middle term of 38x.

6.3 Factoring Trinomials of the Form ax^2 + bx + c 959


Chapter 6 Factoring and Solving by Factoring

Answer: (2x + 5) (6x + 4)

Example 3: Factor: 10x 2 − 23x + 6.

Solution: First, consider the factors of the first and last terms.

We are searching for products of factors whose sum equals the coefficient of the
middle term, −23. Factoring begins at this point with two sets of blank parentheses:

Since the last term is positive and the middle term is negative, we know that both
factors of the last term must be negative. Here we list all possible combinations
with the factors of 10x 2 = 2x ⋅ 5x .

There is no combination that produces a middle term of −23x . We then move on to


the factors of 10x 2 = 10x ⋅ x and list all possible combinations:

6.3 Factoring Trinomials of the Form ax^2 + bx + c 960


Chapter 6 Factoring and Solving by Factoring

And we can write

Answer: (10x − 3) (x − 2). The complete check is left to the reader.

We can reduce much of the guesswork involved in factoring trinomials if we


consider all of the factors of the first and last terms and their products.

Example 4: Factor: 5x 2 + 38x − 16.

Solution: We begin with the factors of 5 and 16.

Since the leading coefficient is prime, we can begin with the following:

6.3 Factoring Trinomials of the Form ax^2 + bx + c 961


Chapter 6 Factoring and Solving by Factoring

We look for products of the factors of 5 and 16 that could possibly add to 38.

Since the last term is negative, we must look for factors with opposite signs. Here
we can see that the products 2 and 40 add up to 38 if they have opposite signs:

Therefore, use −2 and 8 as the factors of 16, making sure that the inner and outer
products are −2x and 40x :

Answer: (x + 8 ) (5x − 2). The complete check is left to the reader.

After lots of practice, the process described in the previous example can be
performed mentally.

6.3 Factoring Trinomials of the Form ax^2 + bx + c 962


Chapter 6 Factoring and Solving by Factoring

Try this! Factor: 12x 2 − 31x − 30.

Answer: (3x − 10) (4x + 3)

Video Solution

(click to see video)

When given trinomials with multiple variables, the process is similar.

Example 5: Factor: 9x 2 + 30xy + 25y 2 .

Solution: Search for factors of the first and last terms such that the sum of the
inner and outer products equals the middle term.

Add the following products to obtain the middle term: 3x ⋅ 5y + 3x ⋅ 5y = 30xy .

In this example, we have a perfect square trinomial. Check.

6.3 Factoring Trinomials of the Form ax^2 + bx + c 963


Chapter 6 Factoring and Solving by Factoring

Answer: (3x + 5y)


2

Try this! Factor: 16x 2 − 24xy + 9y 2 .

Answer: (4x − 3y)


2

Video Solution

(click to see video)


Factoring Trinomials with Common Factors

It is a good practice to first factor out the GCF, if there is one. Doing this produces a
trinomial factor with smaller coefficients. As we have seen, trinomials with smaller
coefficients require much less effort to factor. This commonly overlooked step is
worth identifying early.

Example 6: Factor: 12x 2 − 27x + 6.

Solution: Begin by factoring out the GCF.

After factoring out 3, the coefficients of the resulting trinomial are smaller and
have fewer factors.

After some thought, we can see that the combination that gives the coefficient of
the middle term is 4 (−2) + 1 (−1) = −8 − 1 = −9.

6.3 Factoring Trinomials of the Form ax^2 + bx + c 964


Chapter 6 Factoring and Solving by Factoring

Check.

The factor 3 is part of the factored form of the original expression; be sure to
include it in the answer.

Answer: 3 (4x − 1) (x − 2)

It is a good practice to consistently work with trinomials where the leading


coefficient is positive.

Example 7: Factor: −x 2 + 2x + 15.

Solution: In this example, the leading coefficient is −1. Before beginning the
factoring process, factor out the −1:

At this point, factor the remaining trinomial as usual, remembering to write the −1
as a factor in your final answer. Because 3 + (−5) = −2, use 3 and 5 as the factors of 15.

6.3 Factoring Trinomials of the Form ax^2 + bx + c 965


Chapter 6 Factoring and Solving by Factoring

Answer: −1 (x + 3) (x − 5). The check is left to the reader.

Example 8: Factor: −60a2 − 5a + 30.

Solution: The GCF of all the terms is 5. However, in this case factor out −5 because
this produces a trinomial factor where the leading coefficient is positive.

Focus on the factors of 12 and 6 that combine to give the middle coefficient, 1.

After much thought, we find that 3 ⋅ 3 − 4 ⋅ 2 = 9 − 8 = 1. Factor the remaining


trinomial.

Answer: −5 (4a + 3) (3a − 2). The check is left to the reader.

6.3 Factoring Trinomials of the Form ax^2 + bx + c 966


Chapter 6 Factoring and Solving by Factoring

Try this! Factor: 24 + 2x − x 2 .

Answer: −1 (x − 6) (x + 4)

Video Solution

(click to see video)


Factoring Using the AC Method

In this section, we factor trinomials of the form ax 2 + bx + c using the AC method


described previously.

Example 9: Factor using the AC method: 18x 2 − 21x + 5.

Solution: Here a = 18, b = −21, and c = 5.

Factor 90 and search for factors whose sum is −21.

In this case, the sum of the factors −6 and −15 equals the middle coefficient, −21.
Therefore, −21x = −6x − 15x , and we can write

6.3 Factoring Trinomials of the Form ax^2 + bx + c 967


Chapter 6 Factoring and Solving by Factoring

Factor the equivalent expression by grouping.

Answer: (3x − 1) (6x − 5)

Example 10: Factor using the AC method: 9x 2 − 61x − 14.

Solution: Here a = 9, b = −61, and c = −14.

We factor −126 as follows:

The sum of factors 2 and −63 equals the middle coefficient, −61. Replace −61x with
2x − 63x :

6.3 Factoring Trinomials of the Form ax^2 + bx + c 968


Chapter 6 Factoring and Solving by Factoring

Answer: (x − 7) (9x + 2). The check is left to the reader.

KEY TAKEAWAYS

• If a trinomial of the form ax 2 + bx + c factors into the product of


two binomials, then the coefficient of the middle term will be the sum of
certain products of factors of the first and last terms.
• If the trinomial has a greatest common factor, then it is a best practice
to first factor out the GCF before attempting to factor it into a product of
binomials.
• If the leading coefficient of a trinomial is negative, then it is a best
practice to factor that negative factor out before attempting to factor
the trinomial.
• Factoring trinomials of the form ax 2 + bx + c takes lots of practice
and patience. It is extremely important to take the time to become
proficient by working lots of exercises.

6.3 Factoring Trinomials of the Form ax^2 + bx + c 969


Chapter 6 Factoring and Solving by Factoring

TOPIC EXERCISES

Part A: Factoring Trinomials

Factor.

1. 3x 2 − 14x − 5

2. 5x 2 + 7x + 2

3. 2x 2 + 5x − 3

4. 2x 2 + 13x − 7

5. 2x 2 + 9x − 5

6. 7x 2 + 20x − 3

7. 7x 2 − 46x − 21

8. 3x 2 +x−2

9. 5x 2 + 34x − 7

10. 5x 2 − 28x − 12

11. 9x 2 − 12x + 4

12. 4x 2 − 20x + 25

13. 49x 2 + 14x + 1

14. 25x 2 − 10x + 1

15. 2x 2 + 7x + 16

16. 6x 2 − 19x − 10

6.3 Factoring Trinomials of the Form ax^2 + bx + c 970


Chapter 6 Factoring and Solving by Factoring

17. 27x 2 + 66x − 16

18. 12x 2 − 88x − 15

19. 12y 2 − 8y + 1

20. 16y 2 − 66y − 27

21. 9x 2 − 12xy + 4y 2

22. 25x 2 + 40x + 16

23. 15x 2 − 26xy + 8y 2

24. 12a2 − 4ab − 5b 2

25. 4x 2 y 2 + 16xy − 9

26. 20x 2 y 2 + 4xy − 7

27. The area of a rectangle is given by the function


A(x) = 3x 2 − 10x + 3 , where x is measured in meters. Rewrite this
function in factored form.

28. The area of a rectangle is given by the function


A(x) = 10x 2 − 59x − 6 , where x is measured in meters. Rewrite this
function in factored form.

Part B: Factoring Trinomials with Common Factors

Factor.

29. 6x 2 − 20x − 16

30. 45x 2 + 27x − 18

31. 20x 2 − 20x + 5

6.3 Factoring Trinomials of the Form ax^2 + bx + c 971


Chapter 6 Factoring and Solving by Factoring

32. 3x 2 + 39x − 90

33. 16x 2 + 26x − 10

34. 54x 2 − 15x + 6

35. 45x 2 − 45x − 20

36. 90x 2 + 300x + 250

37. 40x 2 − 36xy + 8y 2


2
38. 24a2 b + 18ab − 81

39. 6x 2 y 2 + 46xy + 28

40. 2x 5 + 44x 4 + 144x 3

41. 5x 3 − 65x 2 + 60x


2
42. 15a4 b − 25a3 b − 10a2

43. 6a4 b + 2a3 b 2 − 4a2 b 3


2
44. 20a3 b − 60a2 b 3 + 45ab 4

Factor out −1 and then factor further.

45. −x 2 − 4x + 21

46. −x 2 + x + 12

47. −x 2 + 15x − 56

48. −x 2 + x + 72

49. −y 2 + 10y − 25

6.3 Factoring Trinomials of the Form ax^2 + bx + c 972


Chapter 6 Factoring and Solving by Factoring

50. −y 2 − 16y − 64

51. 36 − 9a − a2

52. 72 − 6a − a2

53. 32 + 4x − x 2

54. 200 + 10x − x 2

Factor out a negative common factor first and then factor further if possible.

55. −8x 2 + 6x + 9

56. −4x 2 + 28x − 49

57. −18x 2 − 6x + 4

58. 2 + 4x − 30x 2

59. 15 + 39x − 18x 2

60. 90 + 45x − 10x 2

61. −2x 2 + 26x + 28

62. −18x 3 − 51x 2 + 9x

63. −3x 2 y 2 + 18xy 2 − 24y 2

64. −16a4 + 16a3 b − 4a2 b 2

65. The height in feet of a projectile launched from a tower is given by the
function h(t) = −16t 2 + 64t + 80 , where t represents the number of
seconds after launch. Rewrite the given function in factored form.

6.3 Factoring Trinomials of the Form ax^2 + bx + c 973


Chapter 6 Factoring and Solving by Factoring

66. The height in feet of a projectile launched from a tower is given by the
function h(t) = −16t 2 + 64t + 192 , where t represents the number of
seconds after launch. Rewrite the given function in factored form.

Part C: Factoring Using the AC Method

Factor using the AC method.

67. 2x 2 + 5x − 7

68. 3x 2 + 7x − 10

69. 4x 2 − 25x + 6

70. 16x 2 − 38x − 5

71. 6x 2 + 23x − 18

72. 8x 2 + 10x − 25

73. 4x 2 + 28x + 40

74. −6x 2 − 3x + 30

75. 12x 2 − 56xy + 60y 2

76. 20x 2 + 80xy + 35y 2

Part D: Discussion Board Topics

77. Create your own trinomial of the form ax 2 + bx + c that factors.


Share it, along with the solution, on the discussion board.

78. Write out your own list of steps for factoring a trinomial of the form
ax 2 + bx + c and share it on the discussion board.

79. Create a trinomial of the form ax 2 + bx + c that does not factor and
share it along with the reason why it does not factor.

6.3 Factoring Trinomials of the Form ax^2 + bx + c 974


Chapter 6 Factoring and Solving by Factoring

ANSWERS

1: (x − 5) (3x + 1)

3: (x + 3) (2x − 1)

5: (x + 5) (2x − 1)

7: (x − 7) (7x + 3)

9: (x + 7) (5x − 1)

11: (3x − 2) 2

13: (7x + 1) 2

15: Prime

17: (3x + 8) (9x − 2)

19: (6y − 1) (2y − 1)

21: (3x − 2y)


2

23: (3x − 4y) (5x − 2y)

25: (2xy − 1) (2xy + 9)

27: A(x) = (3x − 1)(x − 3)

29: 2 (x − 4) (3x + 2)

31: 5(2x − 1) 2

33: 2 (8x 2 + 13x − 5)

6.3 Factoring Trinomials of the Form ax^2 + bx + c 975


Chapter 6 Factoring and Solving by Factoring

35: 5 (3x − 4) (3x + 1)

37: 4 (5x − 2y) (2x − y)

39: 2 (xy + 7) (3xy + 2)

41: 5x (x − 12) (x − 1)

43: 2a2 b (3a − 2b) (a + b)

45: −1 (x − 3) (x + 7)

47: −1 (x − 7) (x − 8)

49: −1(y − 5)
2

51: −1 (a − 3) (a + 12)

53: −1 (x − 8) (x + 4)

55: − (2x − 3) (4x + 3)

57: −2 (3x − 1) (3x + 2)

59: −3 (2x − 5) (3x + 1)

61: −2 (x − 14) (x + 1)

63: −3y 2 (x − 4) (x − 2)

65: h(t) = −16(t + 1)(t − 5)

67: (x − 1) (2x + 7)

69: (x − 6) (4x − 1)

71: (2x + 9) (3x − 2)

6.3 Factoring Trinomials of the Form ax^2 + bx + c 976


Chapter 6 Factoring and Solving by Factoring

73: 4 (x + 2) (x + 5)

75: 4 (x − 3y) (3x − 5y)

6.3 Factoring Trinomials of the Form ax^2 + bx + c 977


Chapter 6 Factoring and Solving by Factoring

6.4 Factoring Special Binomials

LEARNING OBJECTIVES

1. Factor binomials that are differences of squares.


2. Factor binomials that are sums and differences of cubes.

Difference of Squares

A binomial is a polynomial with two terms. We begin with our first special binomial
called difference of squares16:

To verify the above formula, multiply:

We use this formula to factor certain special binomials.

Example 1: Factor: x 2 − 16.

Solution:

Step 1: Identify the binomial as difference of squares and determine the square

16. a2 − b = (a + b) (a − b) ,
factors of each term.
2

where a and b represent


algebraic expressions.

978
Chapter 6 Factoring and Solving by Factoring

Here we can write

The terms are squares of x and 4. Hence a = x and b = 4.

Step 2: Substitute into the difference of squares formula.

Step 3: Multiply to check. This step is optional.

Answer: (x + 4) (x − 4)

It is worth taking some extra time at this point to review all of the squares of
integers from 1 to 12.

6.4 Factoring Special Binomials 979


Chapter 6 Factoring and Solving by Factoring

Recognizing these perfect square integers helps speed the factoring process.

Example 2: Factor: 9x 2 − 121.

Solution: The subtraction indicates that this is a difference. Furthermore, we


recognize that the terms are squares.

In this case, a = 3x and b = 11. Substitute into the formula for difference of
squares.

Answer: (3x + 11) (3x − 11)

It may be the case that the terms of the binomial have a common factor. If so, it will
be difficult to identify the perfect squares until we first factor out the GCF.

6.4 Factoring Special Binomials 980


Chapter 6 Factoring and Solving by Factoring

Example 3: Factor: 12y 2 − 75.

Solution: The terms are not perfect squares. However, notice that they do have a
common factor. First, factor out the GCF, 3.

The resulting binomial factor is a difference of squares with a = 2y and b = 5.

Answer: 3 (2y + 5) (2y − 5)

Example 4: Factor: 49x 2 − 100y 2 .

Solution: Here we have a binomial with two variables and recognize that it is a
difference of squares.

Therefore, a = 7x and b = 10y . Substitute into the formula for difference of


squares.

Answer: (7x + 10y) (7x − 10y)

6.4 Factoring Special Binomials 981


Chapter 6 Factoring and Solving by Factoring

Try this! Factor: 36x 2 − 1.

Answer: (6x + 1) (6x − 1)

Video Solution

(click to see video)

Given any real number b, a polynomial of the form x 2 + b2 is prime. Furthermore,


the sum of squares17 a2 + b2 does not have a general factored equivalent. Care
should be taken not to confuse this with a perfect square trinomial:

Therefore,

When the degree of the special binomial is greater than two, we may need to apply
the difference of squares formula multiple times. A polynomial is completely
factored when none of the factors can be factored any further.

Example 5: Factor completely: x 4 − 16.

Solution: First, identify what is being squared:

2
17. a2 + b does not have a
general factored equivalent.

6.4 Factoring Special Binomials 982


Chapter 6 Factoring and Solving by Factoring

To do this, recall the power rule for exponents, (x m )n = x mn . When exponents are
raised to a power, multiply them. With this in mind, determine that (x 2 ) = x 4
2

and write

Therefore, a = x 2 and b = 4. Substitute into the formula for difference of squares.

At this point, notice that the factor (x 2 − 4) is itself a difference of two squares
and thus can be further factored using a = x and b = 2. The factor (x 2 + 4) is a
sum of squares, which cannot be factored using real numbers.

Answer: (x 2 + 4) (x + 2) (x − 2)

Try this! Factor completely: 81x 4 − 1.

Answer: (9x 2 + 1) (3x + 1) (3x − 1)

Video Solution
18. a3 + b3 = (a + b) (a2 − ab + b2 ),
(click to see video)
where a and b represent
algebraic expressions. Sum and Difference of Cubes
19. a3 − b3 = (a − b) (a2 + ab + b2 ),
Two other special binomials of interest are the sum18 and difference of cubes19:
where a and b represent
algebraic expressions.

6.4 Factoring Special Binomials 983


Chapter 6 Factoring and Solving by Factoring

We can verify these formulas by multiplying:

The process for factoring the sum and difference of cubes is very similar to that for
the difference of squares. We first identify a and b and then substitute into the
appropriate formula. The separate formulas for sum and difference of cubes allow
us to always choose a and b to be positive.

Example 6: Factor: x 3 + 8.

Solution: The plus sign and the fact that the terms are cubes indicate to us that this
is a sum of cubes.

Next, identify what is being cubed.

6.4 Factoring Special Binomials 984


Chapter 6 Factoring and Solving by Factoring

In this case, a = x and b = 2. Substitute into the sum of cubes formula.

The resulting trinomial is prime and the factoring is complete. We can check this
factorization by multiplying.

Answer: (x + 2) (x 2 − 2x + 4)

It is helpful to review the perfect cubes of integers from 1 to 12. This will aid you in
identifying sums and differences of cubes.

6.4 Factoring Special Binomials 985


Chapter 6 Factoring and Solving by Factoring

Example 7: Factor: y 3 − 125.

Solution: In this case, we have a difference of cubes.

We can write

Substitute a = y and b = 5 into the formula for difference of cubes.

Answer: (y − 5) (y 2 + 5y + 25)

Always look for common factors when factoring. If the terms of the binomial have a
GCF other than 1, then factor that out first.

Example 8: Factor: 54x 4 + 128x.

Solution: Begin by factoring out the GCF 2x .

6.4 Factoring Special Binomials 986


Chapter 6 Factoring and Solving by Factoring

The resulting binomial factor is a sum of cubes, where a = 3x and b = 4.

Answer: 2x (3x + 4) (9x 2 − 12x + 16)

Example 9: Factor: x 3 y 3 − 1.

Solution: This binomial is a difference of cubes with two variables. Identify what is
being cubed.

Here a = xy and b = 1. Substitute into the appropriate formula and simplify.

Answer: (xy − 1) (x 2 y 2 + xy + 1)

6.4 Factoring Special Binomials 987


Chapter 6 Factoring and Solving by Factoring

Try this! Factor: 8x 3 + 343.

Answer: (2x + 7) (4x 2 − 14x + 49)

Video Solution

(click to see video)

When factoring, always look for resulting factors to factor further.

Example 10: Factor completely: x 6 − 64.

Solution: When confronted with a binomial that is both a difference of squares and
cubes, as this is, make it a rule to factor using difference of squares first.

Therefore, a = x 3 and b = 8. Substitute into the difference of squares formula.

The resulting two binomial factors are a sum and a difference of cubes. Each can be
factored further.

Therefore, we have

6.4 Factoring Special Binomials 988


Chapter 6 Factoring and Solving by Factoring

The trinomial factors are prime and the expression is completely factored.

Answer: (x + 2) (x 2 − 2x + 4) (x − 2) (x 2 + 2x + 4)

As an exercise, factor the previous example as a difference of cubes first and then
compare the results. Why do you think we make it a rule to factor using difference
of squares first?

Try this! Factor: x 6 − y 6 .

Answer: (x + y) (x 2 − xy + y 2 ) (x − y) (x 2 + xy + y 2 )

Video Solution

(click to see video)

KEY TAKEAWAYS

• When factoring special binomials, the first step is to identify it as a sum


or difference. Once we identify the binomial, we then determine the
values of a and b and substitute into the appropriate formula.
• The formulas for all of the special binomials should be memorized. In
addition, to help facilitate the identification of special binomials,
memorize the squares and cubes of integers up to at least 12.
• If a binomial is both a difference of squares and cubes, then first factor it
as a difference of squares.

6.4 Factoring Special Binomials 989


Chapter 6 Factoring and Solving by Factoring

TOPIC EXERCISES

Part A: Difference of Squares

Factor completely.

1. x 2 −9

2. x 2 − 100

3. y 2 − 36

4. y 2 − 144

5. x 2 +4

6. x 2 −5

7. m 2 − 81

8. m 2 − 64

9. 16x 2 −9

10. 25x 2 −4

11. 144x 2 −1

12. 9x 2 − 121

13. 4y 2 − 81

14. 100y 2 − 49

15. 9 − 4x 2

16. 100 − x2

6.4 Factoring Special Binomials 990


Chapter 6 Factoring and Solving by Factoring

17. 1 − y2

18. 25 − 9y 2

19. −3x 2 + 75

20. −16x 2 + 25

21. 2x 2 − 72

22. 20x 3 − 45x

23. −48x + 27x 3

24. 36x 2 − 100

25. x 2 − y2

26. 25x 2 − 9y 2

27. a2 − 4b 2
2
28. a2 b − 36

29. 4x 2 y 2 −1

30. x 2 y 2 − 25

31. 2a3 − 8ab 2


4
32. 3a3 b − 75ab 2

33. −100xy 3 + 4x 3 y

34. −18x 3 y 3 + 32xy

35. (x + 1) 2 − y 2

6.4 Factoring Special Binomials 991


Chapter 6 Factoring and Solving by Factoring

− (y − 2)
2
36. x 2

− 3) 2 − (y + 3)
2
37. (x

38. (x 2 + 2) − (x − 1) 2
2

39. (x 2 − 1) − (2x + 3) 2
2

40. x 4 −1

41. x 4 − y4

42. 16x 4 − 81
4
43. a4 b − 16

44. a4 − 16b 4

45. x 8 −1

46. 25x 8 −1

47. a8 − b2

48. a4 −9

49. x 8 − y8

50. 81x 8 −1

51. The height of a projectile dropped from a 64-foot tower is given by the
function h(t) = −16t 2 + 64 , where t represents the time in seconds
after it is dropped. Rewrite this function in factored form. (Hint: Factor out
−16 first.)

6.4 Factoring Special Binomials 992


Chapter 6 Factoring and Solving by Factoring

52. The height of a projectile dropped from a 36-foot tower is given by the
function h(t) = −16t 2 + 36 , where t represents the time in seconds
after it is dropped. Rewrite this function in factored form.

Part B: Sum and Difference of Cubes

Factor completely.

53. x 3 −1

54. x 3 +1

55. y 3 − 27

56. y 3 −8

57. 8y 3 +1

58. 27y 3 −1

59. 64a3 − 125

60. 8a3 + 27

61. a3 + 216

62. a3 − 125

63. x 3 − 1000

64. 343m 3 −1

65. 512n 3 +1

66. 8x 3 + 343

67. 40y 3 − 135

6.4 Factoring Special Binomials 993


Chapter 6 Factoring and Solving by Factoring

68. 27y 3 + 729

69. 27y 3 − 64

70. x 3 +3

71. 5x 3 +1

72. 1 − y3

73. 27 − 1,000y 3

74. 343 + 125a3

75. x 3 − y3

76. x 3 + y3

77. x 3 y 3 + 125

78. 8x 3 y 3 − 27

79. 27a3 − 8b 3

80. 16x 3 − 250y 3

81. 128x 3 + 2y 3

82. 54x 3 − 2y 3

83. 3a4 b − 24ab 4


3 3
84. a3 b c −1

85. (x + 1) 3 − 8

− (x − 5)
3
86. 8x 3

6.4 Factoring Special Binomials 994


Chapter 6 Factoring and Solving by Factoring

87. (x − 2) 3 + (x + 2) 3

+ 3)3 + (b − 3)
3
88. (a

89. x 6 −1

90. x 6 +1

91. 64a6 −1

92. a6 − 81b 6

93. x 6 − y6

94. x 6 + y6

Part C: Discussion Board Topics

95. If a binomial falls into both categories, difference of squares and


difference of cubes, which would be best to factor it as, and why? Create an
example that illustrates this situation and factor it using both formulas.

96. What can be said about the degrees of the factors of a polynomial? Give
an example.

97. Make up your own difference of squares factoring exercise and provide
the answer. Explain how you solved it.

98. Make up your own sum or difference of cubes factoring exercise and
provide the answer. Explain how you solved it.

6.4 Factoring Special Binomials 995


Chapter 6 Factoring and Solving by Factoring

ANSWERS

1: (x + 3) (x − 3)

3: (y + 6) (y − 6)

5: Prime

7: (m + 9) (m − 9)

9: (4x + 3) (4x − 3)

11: (12x + 1) (12x − 1)

13: (2y + 9) (2y − 9)

15: (3 + 2x) (3 − 2x)

17: (1 + y) (1 − y)

19: −3 (x + 5) (x − 5)

21: 2 (x + 6) (x − 6)

23: 3x (3x + 4) (3x − 4)

25: (x + y) (x − y)

27: (a + 2b) (a − 2b)

29: (2xy + 1) (2xy − 1)

31: 2a (a + 2b) (a − 2b)

33: 4xy (x + 5y) (x − 5y)

6.4 Factoring Special Binomials 996


Chapter 6 Factoring and Solving by Factoring

35: (x + 1 + y) (x + 1 − y)

37: (x + y) (x − y − 6)

39: (x 2 + 2x + 2) (x 2 − 2x − 4)

41: (x 2 + y 2 ) (x + y) (x − y)

43: (a2 b + 4) (ab + 2) (ab − 2)


2

45: (x 4 + 1) (x 2 + 1) (x + 1) (x − 1)

47: (a4 + b) (a4 − b)

49: (x 4 + y 4 ) (x 2 + y 2 ) (x + y) (x − y)

51: h(t) = −16(t + 2)(t − 2)

53: (x − 1) (x 2 + x + 1)

55: (y − 3) (y 2 + 3y + 9)

57: (2y + 1) (4y 2 − 2y + 1)

59: (4a − 5) (16a2 + 20a + 25)

61: (a + 6) (a2 − 6a + 36)

63: (x − 10) (x 2 + 10x + 100)

65: (8n + 1) (64n 2 − 8n + 1)

67: 5 (2y − 3) (4y 2 + 6y + 9)

69: (3y − 4) (9y 2 + 12y + 16)

6.4 Factoring Special Binomials 997


Chapter 6 Factoring and Solving by Factoring

71: Prime

73: (3 − 10y) (9 + 30y + 100y 2 )

75: (x − y) (x 2 + xy + y 2 )

77: (xy + 5) (x 2 y 2 − 5xy + 25)

79: (3a − 2b) (9a2 + 6ab + 4b 2 )

81: 2 (4x + y) (16x 2 − 4xy + y 2 )

83: 3ab (a − 2b) (a2 + 2ab + 4b 2 )

85: (x − 1) (x 2 + 4x + 7)

87: 2x (x 2 + 12)

89: (x + 1) (x 2 − x + 1) (x − 1) (x 2 + x + 1)

91: (2a + 1) (4a2 − 2a + 1) (2a − 1) (4a2 + 2a + 1)

93: (x + y) (x 2 − xy + y 2 ) (x − y) (x 2 + xy + y 2 )

6.4 Factoring Special Binomials 998


Chapter 6 Factoring and Solving by Factoring

6.5 General Guidelines for Factoring Polynomials

LEARNING OBJECTIVE

1. Develop a general strategy for factoring polynomials.

General Factoring Strategy

We have learned various techniques for factoring polynomials with up to four


terms. The challenge is to identify the type of polynomial and then decide which
method to apply. The following outlines a general guideline for factoring
polynomials:

1. Check for common factors. If the terms have common factors, then
factor out the greatest common factor (GCF) and look at the resulting
polynomial factors to factor further.
2. Determine the number of terms in the polynomial.

a. Factor four-term polynomials by grouping.

b. Factor trinomials (three terms) using “trial and error” or the AC


method.

c. Factor binomials (two terms) using the following special products:

a2 − b2 = (a + b) (a − b)
Difference of
squares:

Sum of squares: a2 + b2 no general formula

a3 − b3 = (a − b) (a2 + ab + b2 )
Difference of
cubes:

Sum of cubes: a3 + b3 = (a + b) (a2 − ab + b2 )

3. Look for factors that can be factored further.


4. Check by multiplying.

999
Chapter 6 Factoring and Solving by Factoring

Note

• If a binomial is both a difference of squares and a difference of cubes, then


first factor it as difference of squares and then as a sum and difference of
cubes to obtain a more complete factorization.
• Not all polynomials with integer coefficients factor. When this is the case,
we say that the polynomial is prime.

If an expression has a GCF, then factor this out first. Doing so is often overlooked
and typically results in factors that are easier to work with. Also, look for the
resulting factors to factor further; many factoring problems require more than one
step. A polynomial is completely factored when none of the factors can be factored
further.

Example 1: Factor: 6x 4 − 3x 3 − 24x 2 + 12x.

Solution: This four-term polynomial has a GCF of 3x . Factor this out first.

Now factor the resulting four-term polynomial by grouping.

The factor (x 2 − 4) is a difference of squares and can be factored further.

6.5 General Guidelines for Factoring Polynomials 1000


Chapter 6 Factoring and Solving by Factoring

Answer: 3x (2x − 1) (x + 2) (x − 2)

Example 2: Factor: 18x 3 y − 60x 2 y + 50xy .

Solution: This trinomial has a GCF of 2xy . Factor this out first.

the factors 9 = 3 ⋅ 3 and 25 = (−5) ⋅ (−5). These combine to generate the


The trinomial factor can be factored further using the trial and error method. Use

correct coefficient for the middle term: 3 (−5) + 3 (−5) = −15 − 15 = −30.

Check.

6.5 General Guidelines for Factoring Polynomials 1001


Chapter 6 Factoring and Solving by Factoring

Answer: 2xy(3x − 5)
2

Example 3: Factor: 5a3 b4 + 10a2 b3 − 75ab2.

Solution: This trinomial has a GCF of 5ab2. Factor this out first.

The resulting trinomial factor can be factored as follows:

Answer: 5ab2 (ab + 5) (ab − 3)

Try this! Factor: 3x 3 y − 12x 2 y 2 + 12xy 3 .

Answer: 3xy(x − 2y)


2

Video Solution

(click to see video)

Example 4: Factor: 16y 4 − 1.

6.5 General Guidelines for Factoring Polynomials 1002


Chapter 6 Factoring and Solving by Factoring

Solution: This binomial does not have a GCF. Therefore, begin factoring by
identifying it as a difference of squares.

Here a = 4y 2 and b = 1. Substitute into the formula for difference of squares.

The factor (4y 2 + 1) is a sum of squares and is prime. However, (4y 2 − 1) is a


difference of squares and can be factored further.

Answer: (4y 2 + 1) (2y + 1) (2y − 1)

Example 5: Factor: x 6 − 64y 6 .

Solution: This binomial is a difference of squares and a difference of cubes. When


this is the case, first factor it as a difference of squares.

We can write

6.5 General Guidelines for Factoring Polynomials 1003


Chapter 6 Factoring and Solving by Factoring

Each factor can be further factored either as a sum or difference of cubes,


respectively.

Therefore,

Answer: (x + 2y) (x 2 − 2xy + 4y 2 ) (x − 2y) (x 2 + 2xy + 4y 2 )

Example 6: Factor: x 2 − (2x − 1)2 .

Solution: First, identify this expression as a difference of squares.

Here use a = x and b = 2x − 1 in the formula for a difference of squares.

6.5 General Guidelines for Factoring Polynomials 1004


Chapter 6 Factoring and Solving by Factoring

Answer: (3x − 1) (−x + 1)

Try this! Factor: x 4 + 2x 3 + 27x + 54.

Answer: (x + 2) (x + 3) (x 2 − 3x + 9)

Video Solution

(click to see video)

KEY TAKEAWAYS

• Use the polynomial type to determine the method used to factor it.
• It is a best practice to look for and factor out the greatest common factor
(GCF) first. This will facilitate further factoring and simplify the process.
Be sure to include the GCF as a factor in the final answer.
• Look for resulting factors to factor further. It is often the case that
factoring requires more than one step.
• If a binomial can be considered as both a difference of squares and a
difference of cubes, then first factor it as a difference of squares. This
results in a more complete factorization.

6.5 General Guidelines for Factoring Polynomials 1005


Chapter 6 Factoring and Solving by Factoring

TOPIC EXERCISES

Part A: Mixed Factoring

Factor completely.

1. 2x 5 y 2 − 12x 4 y 3

2. 18x 5 y 3 − 6x 4 y 5

3. 5x 2 + 20x − 25

4. 4x 2 + 10x − 6

5. 24x 3 − 30x 2 − 9x

6. 30x 3 − 65x 2 + 10x

7. 6x 3 + 27x 2 − 9x

8. 21x 3 + 49x 2 − 28x

9. 5x 3 − 30x 2 − 15x + 90

10. 6x 4 + 24x 3 − 2x 2 − 8x

11. x 4 − 6x 3 + 8x − 48

12. x 4 − 5x 3 + 27x − 135

13. 4x 3 − 4x 2 − 9x + 9

14. 50x 3 + 25x 2 − 32x − 16

15. 2x 3 + 250

16. 3x 5 − 81x 2

6.5 General Guidelines for Factoring Polynomials 1006


Chapter 6 Factoring and Solving by Factoring

17. 2x 5 − 162x 3

18. 4x 4 − 36

19. x 4 + 16

20. x 3 +9

21. 72 − 2x 2

22. 5x 4 − 25x 2

23. 7x 3 − 14x

24. 36x 2 − 12x + 1

25. 25x 2 + 10x + 1

26. 250x 3 + 200x 4 + 40x 5

27. −7x 2 + 19x + 6

28. −8x 4 + 40x 3 − 50x 2

29. a4 − 16

30. 16a4 − 81b 4

31. y 5 + y4 − y − 1

32. 4y 5 + 2y 4 − 4y 2 − 2y

33. 3x 8 − 192x 2

34. 4x 7 + 4x

35. 4x 2 − 19xy + 12y 2

6.5 General Guidelines for Factoring Polynomials 1007


Chapter 6 Factoring and Solving by Factoring

36. 16x 2 − 66xy − 27y 2

37. 5x 5 − 3x 4 − 5x 3 + 3x 2
2
38. 4a2 b − 4a2 − 9b 2 + 9

39. 15a2 − 4ab − 4b 2

40. 6a2 − 25ab + 4b 2

41. 6x 2 + 5xy + 6y 2

42. 9x 2 + 5xy − 12y 2

43. (3x − 1) 2 − 64

44. (x − 5) − (x − 2) 2
2

45. (x + 1) 3 + 8

46. (x − 4) 3 − 27

47. (2x − 1) 2 − (2x − 1) − 12

48. (x − 4) 2 + 5 (x − 4) + 6

49. a3 b − 10a2 b 2 + 25ab 3


2
50. 2a3 b − 12a2 b + 18a
2
51. 15a2 b − 57ab − 12

52. −60x 3 + 4x 2 + 24x

53. −24x 5 + 78x 3 − 54x

6.5 General Guidelines for Factoring Polynomials 1008


Chapter 6 Factoring and Solving by Factoring

54. 9y 6 − 13y 4 + 4y 2

55. 36 − 15a − 6a2


2
56. 60ab + 5a2 b 2 − 5a3 b 2

57. x 4 −1

58. 16x 4 − 64

59. x 8 −1

60. 81x 8 −1

61. x 16 −1

62. x 12 −1

63. 54x 6 − 216x 4 − 2x 3 + 8x

64. 4a4 − 4a2 b 2 − a2 + b 2

65. 32y 3 + 32y 2 − 18y − 18

66. 3a3 + a2 b − 12ab − 4b 2

67. 18m 2 − 21mn − 9n 2

68. 5m 2 n 2 + 10mn − 15

69. The volume of a certain rectangular solid is given by the function


V(x) = x 3 − 2x 2 − 3x . Write the function in its factored form.

70. The volume of a certain right circular cylinder is given by the function
V(x) = 4πx 3 − 4πx 2 + πx . Write the function in its factored form.

Part B: Discussion Board

6.5 General Guidelines for Factoring Polynomials 1009


Chapter 6 Factoring and Solving by Factoring

71. First, factor the trinomial 24x 2 − 28x − 40 . Then factor out the
GCF. Discuss the significance of factoring out the GCF first. Do you obtain the
same result?

72. Discuss a plan for factoring polynomial expressions on an exam. What


should you be looking for and what should you be expecting?

6.5 General Guidelines for Factoring Polynomials 1010


Chapter 6 Factoring and Solving by Factoring

ANSWERS

1: 2x 4 y 2 (x − 6y)

3: 5 (x − 1) (x + 5)

5: 3x (2x − 3) (4x + 1)

7: 3x (2x 2 + 9x − 3)

9: 5 (x − 6) (x 2 − 3)

11: (x − 6) (x + 2) (x 2 − 2x + 4)

13: (x − 1) (2x − 3) (2x + 3)

15: 2 (x + 5) (x 2 − 5x + 25)

17: 2x 3 (x + 9) (x − 9)

19: Prime

21: 2 (6 + x) (6 − x)

23: 7x (x 2 − 2)

25: (5x + 1)
2

27: − (x − 3) (7x + 2)

29: (a2 + 4) (a + 2) (a − 2)

31: (y 2 + 1) (y − 1) (y + 1)
2

33: 3x 2 (x + 2) (x 2 − 2x + 4) (x − 2) (x 2 + 2x + 4)

6.5 General Guidelines for Factoring Polynomials 1011


Chapter 6 Factoring and Solving by Factoring

35: (x − 4y) (4x − 3y)

37: x 2 (5x − 3) (x + 1) (x − 1)

39: (3a − 2b) (5a + 2b)

41: Prime

43: 3 (x − 3) (3x + 7)

45: (x + 3) (x 2 + 3)

47: 2 (x + 1) (2x − 5)

49: ab(a − 5b)2

51: 3 (ab − 4) (5ab + 1)

53: −6x (x + 1) (x − 1) (2x + 3) (2x − 3)

55: −3 (a + 4) (2a − 3)

57: (x 2 + 1) (x + 1) (x − 1)

59: (x 4 + 1) (x 2 + 1) (x + 1) (x − 1)

61: (x 8 + 1) (x 4 + 1) (x 2 + 1) (x + 1) (x − 1)

63: 2x (x + 2) (x − 2) (3x − 1) (9x 2 + 3x + 1)

65: 2 (y + 1) (4y − 3) (4y + 3)

67: 3 (2m − 3n) (3m + n)

69: V(x) = x (x + 1) (x − 3)

6.5 General Guidelines for Factoring Polynomials 1012


Chapter 6 Factoring and Solving by Factoring

6.6 Solving Equations by Factoring

LEARNING OBJECTIVES

1. Verify solutions to quadratic equations.


2. Solve quadratic equations by factoring.
3. Determine a quadratic equation with given solutions.
4. Solve polynomial equations by factoring.

Solving Quadratic Equations by Factoring

Learning how to solve equations is one of our main goals in algebra. Up to this
point, we have solved linear equations, which are of degree 1. In this section, we
will learn a technique that can be used to solve certain equations of degree 2. A
quadratic equation20 is any equation that can be written in the standard form21

where a, b, and c are real numbers and a ≠ 0. The following are some examples of
quadratic equations, all of which will be solved in this section:

A solution of a quadratic equation in standard form is called a root22. Quadratic


equations can have two real solutions, one real solution, or no real solution. The
20. A polynomial equation with quadratic equation x 2 + x − 6 = 0 has two solutions, namely, x = −3 and x = 2 .
one variable of degree 2.

21. A quadratic equation written in


the form
ax 2 + bx + c = 0.
Example 1: Verify that x = −3 and x = 2 are solutions to x 2 + x − 6 = 0.
22. A solution to a quadratic
equation in standard form.

1013
Chapter 6 Factoring and Solving by Factoring

Solution: To verify solutions, substitute the values for x and then simplify to see if a
true statement results.

Answer: Both values produce true statements. Therefore, they are both solutions to
the equation.

Our goal is to develop algebraic techniques for finding solutions to quadratic


equations. The first technique requires the zero-product property23:

In other words, if any product is equal to zero, then one or both of the variable
factors must be equal to zero.

Example 2: Solve: (x − 8) (x + 7) = 0.

Solution: This equation consists of a product of two quantities equal to zero;


therefore, the zero-product property applies. One or both of the quantities must be
zero.

23. Any product is equal to zero if


and only if at least one of the
factors is zero.

6.6 Solving Equations by Factoring 1014


Chapter 6 Factoring and Solving by Factoring

To verify that these are solutions, substitute them for the variable x.

Notice that each solution produces a factor that is equal to zero.

Answer: The solutions are 8 and −7.

The quadratic equation may not be given in its factored form.

Example 3: Solve: x 2 + 3x − 10 = 0.

Solution: The goal is to produce a product that is equal to zero. We can do that by
factoring the trinomial on the left side of the equation.

6.6 Solving Equations by Factoring 1015


Chapter 6 Factoring and Solving by Factoring

Next, apply the zero-product property and set each factor equal to zero.

This leaves us with two linear equations, each of which can be solved for x .

Check the solutions by substituting into the original equation to verify that we
obtain true statements.

Answer: The solutions are −5 and 2.

Using the zero-product property after factoring a quadratic equation in standard


form is the key to this technique. However, the quadratic equation may not be
given in standard form, and so there may be some preliminary steps before
factoring. The steps required to solve by factoring24 are outlined in the following
example.

24. The process of solving an


equation that is equal to zero
by factoring it and then setting
each variable factor equal to
Example 4: Solve: 2x 2 + 10x + 20 = −3x + 5.
zero.

6.6 Solving Equations by Factoring 1016


Chapter 6 Factoring and Solving by Factoring

Solution:

Step 1: Express the quadratic equation in standard form. For the zero-product
property to apply, the quadratic expression must be equal to zero. Use the addition
and subtraction properties of equality to combine opposite-side like terms and
obtain zero on one side of the equation.

In this example, add 3x to and subtract 5 from both sides.

Step 2: Factor the quadratic expression.

Step 3: Apply the zero-product property and set each variable factor equal to zero.

Step 4: Solve the resulting linear equations.

6.6 Solving Equations by Factoring 1017


Chapter 6 Factoring and Solving by Factoring

Answer: The solutions are −5 and −3/2. The check is optional.

Example 5: Solve: 9x 2 + 1 = 6x.

Solution: Write this in standard form by subtracting 6x from both sides.

Once the equation is in standard form, equal to zero, factor.

This is a perfect square trinomial. Hence setting each factor equal to zero results in
a repeated solution.

A repeated solution is called a double root25 and does not have to be written twice.

Answer: The solution is 1/3.

Try this! Solve: x 2 − 3x = 28.

25. A root that is repeated twice. Answer: x = −4 or x = 7

6.6 Solving Equations by Factoring 1018


Chapter 6 Factoring and Solving by Factoring

Video Solution

(click to see video)

Not all quadratic equations in standard form are trinomials. We often encounter
binomials.

Example 6: Solve: x 2 − 9 = 0.

Solution: This quadratic equation is given in standard form, where the binomial on
the left side is a difference of squares. Factor as follows:

Next, set each factor equal to zero and solve.

Answer: The solutions are 3 and −3, which can also be written as ±3.

Example 7: Solve: 5x 2 = 15x.

Solution: By inspection, we see that x = 0 is a solution to this quadratic equation.


Since dividing by zero is undefined, we want to avoid dividing both sides of this
equation by x. In general, we wish to avoid dividing both sides of any equation by a
variable or an expression containing a variable. We will discuss this in more detail
later. The first step is to rewrite this equation in standard form with zero on one
side.

6.6 Solving Equations by Factoring 1019


Chapter 6 Factoring and Solving by Factoring

Next, factor the expression. Notice that the binomial on the left has a GCF of 5x .

Set each factor to equal to zero.

Answer: The solutions are 0 and 3.

Example 8: Solve: (2x + 1) (x + 5) = 11.

Solution: This quadratic equation appears to be factored; hence it might be


tempting to set each factor equal to 11. However, this would lead to incorrect
results. We must rewrite the equation in standard form, equal to zero, so that we
can apply the zero-product property.

6.6 Solving Equations by Factoring 1020


Chapter 6 Factoring and Solving by Factoring

Once it is in standard form, we can factor and then set each factor equal to zero.

Answer: The solutions are 1/2 and −6.

Example 9: Solve: 15x 2 − 25x + 10 = 0.

Solution: We begin by factoring out the GCF of 5. Then factor the resulting
trinomial.

Next, we set each variable factor equal to zero and solve for x.

Notice that the factor 5 is not a variable factor and thus did not contribute to the
solution set.

Answer: The solutions are 2/3 and 1.

6.6 Solving Equations by Factoring 1021


Chapter 6 Factoring and Solving by Factoring

Example 10: Factor: 52 x 2 + 76 x − 13 = .0

Solution: Clear the fractions by multiplying both sides of the equation by the LCD,
which is equal to 6.

At this point, we have an equivalent equation with integer coefficients and can
factor as usual. Begin with the factors of 15 and 2.

The coefficient of the middle term is 7 = 3 (−1) + 5 (2). Factor as follows:

Set each factor equal to zero and solve.

6.6 Solving Equations by Factoring 1022


Chapter 6 Factoring and Solving by Factoring

Answer: The solutions are −2/3 and 1/5.

Try this! Solve: 4x 2 − 9 = 0.

Answer: −3/2 and 3/2

Video Solution

(click to see video)


Finding Equations with Given Solutions

The zero-product property states,

And, in fact, the converse is true as well:

When this is the case, we can write the following:

We use this property to find equations, given the solutions. To do this, the steps for
solving by factoring are performed in reverse.

Example 11: Find a quadratic equation with solutions −7 and 2.

6.6 Solving Equations by Factoring 1023


Chapter 6 Factoring and Solving by Factoring

Solution: Given the solutions, we can determine two linear factors.

The product of these linear factors is equal to zero when x = −7 or x = 2 :

Multiply the binomials and present the equation in standard form.

Answer: x 2 + 5x − 14 = 0. We may check our equation by substituting the given


answers to see if we obtain a true statement. Also, the equation found above is not
unique and so the check becomes essential when our equation looks different from
someone else’s. This is left as an exercise.

Example 12: Find a quadratic equation with integer coefficients, given solutions 1/
2 and −3/4.

Solution: To avoid fractional coefficients, we first clear the fractions by multiplying


both sides by the denominator.

6.6 Solving Equations by Factoring 1024


Chapter 6 Factoring and Solving by Factoring

Apply the zero-product property and multiply.

Answer: 8x 2 + 2x − 3 = 0

Try this! Find a quadratic equation with integer coefficients, given solutions −1 and
2/3.

Answer: 3x 2 + x − 2 = 0

Video Solution

(click to see video)


Solving Polynomial Equations by Factoring

The zero-product property is true for any number of factors that make up an
equation. If an expression is equal to zero and can be factored into linear factors,
then we will be able to set each factor equal to zero and solve for each equation.

Example 13: Solve: 3x (x − 5) (3x − 2) = 0.

6.6 Solving Equations by Factoring 1025


Chapter 6 Factoring and Solving by Factoring

Solution: Set each variable factor equal to zero and solve.

Answer: The solutions are 0, 5, and 2/3.

Of course, we cannot expect the equation to be given in factored form.

Example 14: Solve: x 3 + 2x 2 − 9x − 18 = 0.

Solution: Begin by factoring the left side completely.

Set each factor equal to zero and solve.

Answer: The solutions are −2, −3, and 3.

6.6 Solving Equations by Factoring 1026


Chapter 6 Factoring and Solving by Factoring

Notice that the degree of the polynomial is 3 and we obtained three solutions. In
general, for any polynomial equation with one variable of degree n, the
fundamental theorem of algebra26 guarantees n real solutions or fewer. We have
seen that many polynomials do not factor. This does not imply that equations
involving these unfactorable polynomials do not have real solutions. In fact, many
polynomial equations that do not factor do have real solutions. We will learn how to
solve these types of equations as we continue in our study of algebra.

Try this! Solve: −10x 3 − 18x 2 + 4x = 0.

Answer: −2, 0, 1/5

Video Solution

(click to see video)

KEY TAKEAWAYS

• A polynomial can have at most a number of solutions equal to its degree.


Therefore, quadratic equations can have up to two real solutions.
• To solve a quadratic equation, first write it in standard form. Once the
quadratic expression is equal to zero, factor it and then set each variable
factor equal to zero. The solutions to the resulting linear equations are
the solutions to the quadratic equation.
• Not all quadratic equations can be solved by factoring. We will learn how
to solve quadratic equations that do not factor later in the course.
• To find a quadratic equation with given solutions, perform the process
of solving by factoring in reverse.
• If any polynomial is factored into linear factors and is set to zero, then
we can determine the solutions by setting each variable factor equal to
zero and solving each individually.

26. Guarantees that there will be


as many (or fewer) real
solutions to a polynomial with
one variable as its degree.

6.6 Solving Equations by Factoring 1027


Chapter 6 Factoring and Solving by Factoring

TOPIC EXERCISES

Part A: Solutions to Quadratic Equations

Determine whether the given set of values are solutions to the quadratic equation.

1. {−3, 5}; x 2 − 2x − 15 = 0

2. {7, −1}; x 2 − 6x − 7 = 0
1
3. {−1/2, 1/2}; x 2 − =0
4

9
4. {−3/4, 3/4}; x 2 − =0
16

5. {−3, 2}; x 2 −x−6=0

6. {−5, 1}; x 2 − 4x − 5 = 0

Solve.

7. (x − 3) (x + 2) = 0

8. (x + 5) (x + 1) = 0

9. (2x − 1) (x − 4) = 0

10. (3x + 1) (3x − 1) = 0

11. (x − 2) 2 = 0

12. (5x + 3) = 0
2

13. 7x (x − 5) = 0

14. −2x (2x − 3) = 0

15. (x − 12 ) (x + 34 ) = 0

6.6 Solving Equations by Factoring 1028


Chapter 6 Factoring and Solving by Factoring

16. (x + 58 ) (x − 38 ) = 0

17. ( x + 12 ) ( 16 x − 23 ) = 0
1
4

18. ( x − 3) = 0
1 2
5

19. −5 (x + 1) (x − 2) = 0

20. 12 (x − 7) (x − 6) = 0

21. (x + 5) (x − 1) = 0

22. (x + 5) (x + 1) = 0

23. −2 (3x − 2) (2x + 5) = 0

24. 5(7x − 8) 2 = 0

Part B: Solve by Factoring

Solve.

25. x 2 −x−6=0

26. x 2 + 3x − 10 = 0

27. y 2 − 10y + 24 = 0

28. y 2 + 6y − 27 = 0

29. x 2 − 14x + 40 = 0

30. x 2 + 14x + 49 = 0

31. x 2 − 10x + 25 = 0

6.6 Solving Equations by Factoring 1029


Chapter 6 Factoring and Solving by Factoring

32. 3x 2 + 2x − 1 = 0

33. 5x 2 − 9x − 2 = 0

34. 7y 2 + 20y − 3 = 0

35. 9x 2 − 42x + 49 = 0

36. 25x 2 + 30x + 9 = 0

37. 2y 2 +y−3=0

38. 7x 2 − 11x − 6 = 0

39. 2x 2 = −15x + 8

40. 8x − 5 = 3x 2

41. x 2 − 36 = 0

42. x 2 − 100 = 0

43. 4x 2 − 81 = 0

44. 49x 2 −4=0

45. x 2 =4

46. 9y 2 =1

47. 16y 2 = 25

48. 36x 2 = 25

49. 4x 2 − 36 = 0

50. 2x 2 − 18 = 0

6.6 Solving Equations by Factoring 1030


Chapter 6 Factoring and Solving by Factoring

51. 10x 2 + 20x = 0

52. −3x 2 + 6x = 0

53. 25x 2 = 50x

54. x 2 =0

55. (x + 1) 2 − 25 = 0

56. (x − 2) 2 − 36 = 0

57. 5x (x − 4) = −4 + x

58. (x − 1) (x − 10) = 22

59. (x − 3) (x − 5) = 24

60. −2x (x − 9) = x + 21

61. (x + 1) (6x + 1) = 2x

62. (x − 2) (x + 12) = 15x

63. (x + 1) (x + 2) = 2(x + 16)

64. (x − 9) (2x + 3) = 2(x − 9)

Clear the fractions by first multiplying both sides by the LCD and then solve.

65. 1
15
x2 + 1
3
x+ 2
5
=0

66. 1
14
x2 − 1
2
x+ 3
7
=0

67. 3
2
x2 − 2
3
=0

68. 5
2
x2 − 1
10
=0

6.6 Solving Equations by Factoring 1031


Chapter 6 Factoring and Solving by Factoring

69. 3
14
x2 − 21
2
=0

70. 1
3
x2 − 1
5
x=0

71. 1
32
x2 − 1
2
x+2=0

72. 1
3
x2 + 5
6
x− 1
2
=0

73. The sides of a square measure x + 3 units. If the area is 25 square units,
then find x.

74. The height of a triangle is 2 units more than its base. If the area is 40
square units, then find the length of the base.

75. The sides of a right triangle have measures that are consecutive integers.
Find the length of the hypotenuse. (Hint: The hypotenuse is the longest side.
Apply the Pythagorean theorem.)

76. The profit in dollars generated by producing and selling x custom lamps
is given by the function P(x) = −10x 2 + 800x − 12000 . How many
lamps must be sold and produced to break even? (Hint: We break even when
the profit is zero.)

Assuming dry road conditions and average reaction times, the safe stopping
distance, d in feet of an average car is given using the formula d = 201 v 2 + v,
where v represents the speed of the car in miles per hour. For each problem below,
given the stopping distance, determine the safe speed.

77. 15 feet

78. 40 feet

79. 75 feet

80. 120 feet

Part C: Finding Equations with Given Solutions

Find a quadratic equation with integer coefficients, given the following solutions.

6.6 Solving Equations by Factoring 1032


Chapter 6 Factoring and Solving by Factoring

81. −3, 1

82. −5, 3

83. −10, −3

84. −7, −4

85. −1, 0

86. 0, 3/5

87. −2, 2

88. −1/2, 1/2

89. −4, 1/3

90. 2/3, 2/5

91. −1/5, −2/3

92. −3/2, 3/4

93. 3, double root

94. −5, double root

Part D: Solving Polynomial Equations

Solve.

95. 7x (x + 5) (x − 9) = 0

96. (x − 1) (x − 2) (x − 3) = 0

97. −2x (x − 10) (x − 1) = 0

98. 8x(x − 4) 2 = 0

6.6 Solving Equations by Factoring 1033


Chapter 6 Factoring and Solving by Factoring

99. 4 (x + 3) (x − 2) (x + 1) = 0

100. −2 (3x + 1) (3x − 1) (x − 1) (x + 1) = 0

101. x 3 − x 2 − 2x = 0

102. 2x 3 + 5x 2 − 3x = 0

103. 5x 3 − 15x 2 + 10x = 0

104. −2x 3 + 2x 2 + 12x = 0

105. 3x 3 − 27x = 0

106. −2x 3 + 8x = 0

107. x 3 + x2 − x − 1 = 0

108. x 3 + 2x 2 − 16x − 32 = 0

109. 8x 3 − 4x 2 − 18x + 9 = 0

110. 12x 3 = 27x

Part E: Discussion Board Topics

111. Explain why 2 (x


+ 5) (x − 5) = 0 has two solutions and
2x (x + 5) (x − 5) = 0 has three solutions.

112. Make up your own quadratic equation and post it and the solutions on
the discussion board.

113. Explain, in your own words, how to solve a quadratic equation in


standard form.

6.6 Solving Equations by Factoring 1034


Chapter 6 Factoring and Solving by Factoring

ANSWERS

1: Yes

3: Yes

5: No

7: −2, 3

9: 1/2, 4

11: 2

13: 0, 5

15: −3/4, 1/2

17: −2, 4

19: −1, 2

21: −5, 1

23: −5/2, 2/3

25: −2, 3

27: 4, 6

29: 4, 10

31: 5

33: −1/5, 2

35: 7/3

37: −3/2, 1

6.6 Solving Equations by Factoring 1035


Chapter 6 Factoring and Solving by Factoring

39: −8, ½

41: −6, 6

43: −9/2, 9/2

45: −2, 2

47: −5/4, 5/4

49: −3, 3

51: −2, 0

53: 0, 2

55: −6, 4

57: 1/5, 4

59: −1, 9

61: −1/2, −1/3

63: −6, 5

65: −3, −2

67: −2/3, 2/3

69: ±7

71: 8

73: 2 units

75: 5 units

77: 10 miles per hour

6.6 Solving Equations by Factoring 1036


Chapter 6 Factoring and Solving by Factoring

79: 30 miles per hour

81: x 2 + 2x − 3 = 0

83: x 2 + 13x + 30 = 0

85: x 2 +x=0

87: x 2 −4=0

89: 3x 2 + 11x − 4 = 0

91: 15x 2 + 13x + 2 = 0

93: x 2 − 6x + 9 = 0

95: −5, 0, 9

97: 0, 1, 10

99: −3, −1, 2

101: −1, 0, 2

103: 0, 1, 2

105: −3, 0, 3

107: −1, 1

109: −3/2, 1/2, 3/2

6.6 Solving Equations by Factoring 1037


Chapter 6 Factoring and Solving by Factoring

6.7 Applications Involving Quadratic Equations

LEARNING OBJECTIVES

1. Set up and solve applications involving relationships between real


numbers.
2. Set up and solve applications involving geometric relationships
involving area and the Pythagorean theorem.
3. Set up and solve applications involving the height of projectiles.

Number Problems

The algebraic setups of the word problems that we have previously encountered led
to linear equations. When we translate the applications to algebraic setups in this
section, the setups lead to quadratic equations. Just as before, we want to avoid
relying on the “guess and check” method for solving applications. Using algebra to
solve problems simplifies the process and is more reliable.

Example 1: One integer is 4 less than twice another integer, and their product is 96.
Set up an algebraic equation and solve it to find the two integers.

Solution: First, identify the variables. Avoid two variables by using the relationship
between the two unknowns.

The key phrase, “their product is 96,” indicates that we should multiply and set the
product equal to 96.

1038
Chapter 6 Factoring and Solving by Factoring

Once we have the problem translated to a mathematical equation, we then solve. In


this case, we can solve by factoring. The first step is to write the equation in
standard form:

Next, factor completely and set each variable factor equal to zero.

The problem calls for two integers whose product is +96. The product of two
positive numbers is positive and the product of two negative numbers is positive.
Hence we can have two sets of solutions. Use 2n − 4 to determine the other
integers.

Answer: Two sets of integers solve this problem: {8, 12} and {−6, −16}. Notice that
(8)(12) = 96 and (−6)(−16) = 96; our solutions check out.

6.7 Applications Involving Quadratic Equations 1039


Chapter 6 Factoring and Solving by Factoring

With quadratic equations, we often obtain two solutions for the identified
unknown. Although it may be the case that both are solutions to the equation, they
may not be solutions to the problem. If a solution does not solve the original
application, then we disregard it.

Recall that consecutive odd and even integers both are separated by two units.

Example 2: The product of two consecutive positive odd integers is 99. Find the
integers.

Solution:

The key phase, “product…is 99,” indicates that we should multiply and set the
product equal to 99.

Rewrite the quadratic equation in standard form and solve by factoring.

6.7 Applications Involving Quadratic Equations 1040


Chapter 6 Factoring and Solving by Factoring

Because the problem asks for positive integers, n = 9 is the only solution. Back
substitute to determine the next odd integer.

Answer: The consecutive positive odd integers are 9 and 11.

Example 3: Given two consecutive positive odd integers, the product of the larger
and twice the smaller is equal to 70. Find the integers.

Solution:

The key phrase “twice the smaller” can be translated to 2n. The phrase “product…is
70” indicates that we should multiply this by the larger odd integer and set the
product equal to 70.

6.7 Applications Involving Quadratic Equations 1041


Chapter 6 Factoring and Solving by Factoring

Solve by factoring.

Because the problem asks for positive integers, n = 5 is the only solution. Back
substitute into n + 2 to determine the next odd integer.

Answer: The positive odd integers are 5 and 7.

Try this! The product of two consecutive positive even integers is 168. Find the
integers.

Answer: The positive even integers are 12 and 14.

Video Solution

(click to see video)


Geometry Problems

When working with geometry problems, it is helpful to draw a picture. Below are
some area formulas that you are expected to know. (Recall that π ≈ 3.14.)

6.7 Applications Involving Quadratic Equations 1042


Chapter 6 Factoring and Solving by Factoring

Area of a rectangle27: A = l ⋅ w

Area of a square28: A = s2

Area of a triangle29: A= 1
2
bh

Area of a circle30: A = πr2

Example 4: The floor of a rectangular room has a length that is 4 feet more than
twice its width. If the total area of the floor is 240 square feet, then find the
dimensions of the floor.
27. A = lw , where l represents
the length and w represents Solution:
the width.

28. A = s2 , where s represents


the length of each side.

29. A = 12 bh, where b


represents the length of the
base and h represents the
height.

30. A = πr2 , where r represents


the radius and the constant
π ≈ 3.14.

6.7 Applications Involving Quadratic Equations 1043


Chapter 6 Factoring and Solving by Factoring

Use the formula A = l ⋅ w and the fact that the area is 240 square feet to set up an
algebraic equation.

Solve by factoring.

At this point we have two possibilities for the width of the rectangle. However,
since a negative width is not defined, choose the positive solution, w = 10 . Back
substitute to find the length.

Answer: The width is 10 feet and the length is 24 feet.

6.7 Applications Involving Quadratic Equations 1044


Chapter 6 Factoring and Solving by Factoring

It is important to include the correct units in the final presentation of the answer.
In the previous example, it would not make much sense to say the width is 10. Make
sure to indicate that the width is 10 feet.

Example 5: The height of a triangle is 3 inches less than twice the length of its base.
If the total area of the triangle is 7 square inches, then find the lengths of the base
and height.

Solution:

Use the formula A = 1


2
bh and the fact that the area is 7 square inches to set up an
algebraic equation.

To avoid fractional coefficients, multiply both sides by 2 and then rewrite the
quadratic equation in standard form.

6.7 Applications Involving Quadratic Equations 1045


Chapter 6 Factoring and Solving by Factoring

Factor and then set each factor equal to zero.

In this case, disregard the negative answer; the length of the base is 7/2 inches long.
Use 2b − 3 to determine the height of the triangle.

Answer: The base measures 7/2 = 3½ inches and the height is 4 inches.

Try this! The base of a triangle is 5 units less than twice the height. If the area is 75
square units, then what is the length of the base and height?

Answer: The height is 10 units and the base is 15 units.

6.7 Applications Involving Quadratic Equations 1046


Chapter 6 Factoring and Solving by Factoring

Video Solution

(click to see video)

Recall that a right triangle is a triangle where one of the angles measures 90°. The
side opposite of the right angle is the longest side of the triangle and is called the
hypotenuse. The Pythagorean theorem31 gives us a relationship between the legs
and hypotenuse of any right triangle, where a and b are the lengths of the legs and c
is the length of the hypotenuse:

Given certain relationships, we use this theorem when determining the lengths of
sides of right triangles.

Example 6: The hypotenuse of a right triangle is 10 inches. If the short leg is 2


inches less than the long leg, then find the lengths of the legs.

Solution:

31. Given any right triangle with


Given that the hypotenuse measures 10 inches, substitute its value into the
legs measuring a and b units
and hypotenuse measuring c Pythagorean theorem and obtain a quadratic equation in terms of x.
units, then a2 + b2 = c2.

6.7 Applications Involving Quadratic Equations 1047


Chapter 6 Factoring and Solving by Factoring

Multiply and rewrite the equation in standard form.

Once it is in standard form, factor and set each variable factor equal to zero.

Because lengths cannot be negative, disregard the negative answer. In this case, the
long leg measures 8 inches. Use x − 2 to determine the length of the short leg.

Answer: The short leg measures 6 inches and the long leg measures 8 inches.

6.7 Applications Involving Quadratic Equations 1048


Chapter 6 Factoring and Solving by Factoring

Example 7: One leg of a right triangle measures 3 centimeters. The hypotenuse of


the right triangle measures 3 centimeters less than twice the length of the unknown
leg. Find the measure of all the sides of the triangle.

Solution:

To set up an algebraic equation, we use the Pythagorean theorem.

Solve by factoring.

Disregard 0. The length of the unknown leg is 4 centimeters. Use 2x − 3 to


determine the length of the hypotenuse.

6.7 Applications Involving Quadratic Equations 1049


Chapter 6 Factoring and Solving by Factoring

Answer: The sides of the triangle measure 3 centimeters, 4 centimeters, and 5


centimeters.

Try this! The hypotenuse of a right triangle measures 13 units. If one leg is 2 units
more than twice that of the other, then find the length of each leg.

Answer: The two legs measure 5 units and 12 units.

Video Solution

(click to see video)


Projectile Problems

The height of an object launched upward, ignoring the effects of air resistance, can
be modeled with the following formula:

Using function notation, which is more appropriate, we have

With this formula, the height can be calculated at any given time t after the object is
launched. The coefficients represent the following:

6.7 Applications Involving Quadratic Equations 1050


Chapter 6 Factoring and Solving by Factoring

1
− g The letter g represents the acceleration due to gravity.
2

v0 “v-naught” represents the initial velocity of the object.

“s-naught” represents the initial height from which the


s0
object is launched.

We consider only problems where the acceleration due to gravity can be expressed
as g = 32 ft/sec2. Therefore, in this section time will be measured in seconds and
the height in feet. Certainly though, the formula is valid using units other than
these.

Example 8: The height of a projectile launched upward at a speed of 32 feet/second


from a height of 128 feet is given by the function h (t) = −16t2 + 32t + 128. How
long does it take to hit the ground?

Solution: An inefficient method for finding the time to hit the ground is to simply
start guessing at times and evaluating. To do this, construct a chart.

6.7 Applications Involving Quadratic Equations 1051


Chapter 6 Factoring and Solving by Factoring

Use the table to sketch the height of the projectile over time.

We see that at 4 seconds, the projectile hits the ground. Note that when this occurs,
the height is equal to 0. Now we need to solve this problem algebraically. To find
the solution algebraically, use the fact that the height is 0 when the object hits the
ground. We need to find the time, t, when h (t) = 0.

6.7 Applications Involving Quadratic Equations 1052


Chapter 6 Factoring and Solving by Factoring

Solve the equation by factoring.

Now set each variable factor to zero.

As expected, the projectile hits the ground at t = 4 seconds. Disregard −2 as a


solution because negative time is not defined.

Answer: The projectile hits the ground 4 seconds after it is launched.

Example 9: The height of a certain book dropped from the top of a 144-foot
building is given by h (t) = −16t2 + 144. How long does it take to hit the ground?

Solution: Find the time t when the height h (t) = 0.

Answer: The book takes 3 seconds to hit the ground when dropped from the top of a
144-foot building.

6.7 Applications Involving Quadratic Equations 1053


Chapter 6 Factoring and Solving by Factoring

Try this! The height of a projectile, shot straight up into the air from the ground, is
given by h (t) = −16t2 + 80t. How long does it take to come back down to the
ground?

Answer: It will take 5 seconds to come back down to the ground.

Video Solution

(click to see video)

KEY TAKEAWAYS

• It is best to translate a word problem to a mathematical setup and then


solve using algebra. Avoid using the “guess and check” method of
solving applications in this section.
• When solving applications, check that your solutions make sense in the
context of the question. For example, if you wish to find the length of
the base of a triangle, then you would disregard any negative solutions.
• It is important to identify each variable and state in a sentence what
each variable represents. It is often helpful to draw a picture.

6.7 Applications Involving Quadratic Equations 1054


Chapter 6 Factoring and Solving by Factoring

TOPIC EXERCISES

Part A: Number Problems

Set up an algebraic equation and then solve.

1. One integer is five times another. If the product of the two integers is 80,
then find the integers.

2. One integer is four times another. If the product of the two integers is 36,
then find the integers.

3. An integer is one more than four times another. If the product of the two
integers is 39, then find the integers.

4. An integer is 3 more than another. If the product of the two integers is


130, then find the integers.

5. An integer is 2 less than twice another. If the product of the two integers
is 220, then find the integers.

6. An integer is 3 more than twice another. If the product of the two integers
is 90, then find the integers.

7. One integer is 2 units more than another. If the product of the two
integers is equal to five times the larger, then find the two integers.

8. A positive integer is 1 less than twice another. If the product of the two
integers is equal to fifteen times the smaller, then find the two integers.

9. A positive integer is 3 more than twice a smaller positive integer. If the


product of the two integers is equal to six times the larger, then find the
integers.

10. One positive integer is 3 more than another. If the product of the two
integers is equal to twelve times the smaller, then find the integers.

11. An integer is 3 more than another. If the product of the two integers is
equal to 2 more than four times their sum, then find the integers.

6.7 Applications Involving Quadratic Equations 1055


Chapter 6 Factoring and Solving by Factoring

12. An integer is 5 more than another. If the product of the two integers is
equal to 2 more than twice their sum, then find the integers.

13. The product of two consecutive positive even integers is 120. Find the
integers.

14. The product of two consecutive positive odd integers is 99. Find the
integers.

15. The product of two consecutive positive integers is 110. Find the
integers.

16. The product of two consecutive positive integers is 42. Find the integers.

17. The product of two consecutive positive odd integers is equal to 1 less
than seven times the sum of the integers. Find the integers.

18. The product of two consecutive positive even integers is equal to 22 more
than eleven times the sum of the integers. Find the integers.

19. The sum of the squares of two consecutive positive odd integers is 74.
Find the integers.

20. The sum of the squares of two consecutive positive even integers is 100.
Find the integers.

21. The sum of the squares of two consecutive positive integers is 265. Find
the integers.

22. The sum of the squares of two consecutive positive integers is 181. Find
the integers.

23. For two consecutive positive odd integers, the product of twice the
smaller and the larger is 126. Find the integers.

24. For two consecutive positive even integers, the product of the smaller
and twice the larger is 160. Find the integers.

Part B: Geometry Problems

Set up an algebraic equation and then solve.

6.7 Applications Involving Quadratic Equations 1056


Chapter 6 Factoring and Solving by Factoring

25. The width of a rectangle is 7 feet less than its length. If the area of the
rectangle is 170 square feet, then find the length and width.

26. The length of a rectangle is 2 feet more than its width. If the area of the
rectangle is 48 square feet, then find the length and width.

27. The width of a rectangle is 3 units less than the length. If the area is 70
square units, then find the dimensions of the rectangle.

28. The width of a rectangle measures one half of the length. If the area is 72
square feet, then find the dimensions of the rectangle.

29. The length of a rectangle is twice that of its width. If the area of the
rectangle is 72 square inches, then find the length and width.

30. The length of a rectangle is three times that of its width. If the area of
the rectangle is 75 square centimeters, then find the length and width.

31. The length of a rectangle is 2 inches more than its width. The area of the
rectangle is equal to 12 inches more than three times the perimeter. Find
the length and width of the rectangle.

32. The length of a rectangle is 3 meters more than twice the width. The area
of the rectangle is equal to 10 meters less than three times the perimeter.
Find the length and width of the rectangle.

33. A uniform border is to be placed around an 8-inch-by-10-inch picture. If


the total area including the border must be 224 square inches, then how
wide should the border be?

34. A 2-foot brick border is constructed around a square cement slab. If the
total area, including the border, is 121 square feet, then what are the
dimensions of the slab?

6.7 Applications Involving Quadratic Equations 1057


Chapter 6 Factoring and Solving by Factoring

35. The area of a picture frame including a 2-inch wide border is 99 square
inches. If the width of the inner area is 2 inches more than its length, then
find the dimensions of the inner area.

36. A box can be made by cutting out the corners and folding up the edges of
a square sheet of cardboard. A template for a cardboard box with a height of
2 inches is given. What is the length of each side of the cardboard sheet if
the volume of the box is to be 50 cubic inches?

37. The height of a triangle is 3 inches more than the length of its base. If the
area of the triangle is 44 square inches, then find the length of its base and
height.

38. The height of a triangle is 4 units less than the length of the base. If the
area of the triangle is 48 square units, then find the length of its base and
height.

39. The base of a triangle is twice that of its height. If the area is 36 square
centimeters, then find the length of its base and height.

40. The height of a triangle is three times the length of its base. If the area is
73½ square feet, then find the length of the base and height.

41. The height of a triangle is 1 unit more than the length of its base. If the
area is 5 units more than four times the height, then find the length of the
base and height of the triangle.

42. The base of a triangle is 4 times that of its height. If the area is 3 units
more than five times the height, then find the length of the base and height
of the triangle.

6.7 Applications Involving Quadratic Equations 1058


Chapter 6 Factoring and Solving by Factoring

43. The diagonal of a rectangle measures 5 inches. If the length is 1 inch


more than its width, then find the dimensions of the rectangle.

44. The diagonal of a rectangle measures 10 inches. If the width is 2 inches


less than the length, then find the area of the rectangle.

45. If the sides of a right triangle are consecutive even integers, then what
are their measures?

46. The hypotenuse of a right triangle is 13 units. If the length of one leg is 2
more than twice the other, then what are their lengths?

47. The shortest leg of a right triangle measures 9 centimeters and the
hypotenuse measures 3 centimeters more than the longer leg. Find the
length of the hypotenuse.

48. The long leg of a right triangle measures 24 centimeters and the
hypotenuse measures 4 centimeters more three times the short leg. Find the
length of the hypotenuse.

Part C: Projectile Problems

Set up an algebraic equation and then solve.

49. The height of a projectile launched upward at a speed of 32 feet/second


from a height of 48 feet is given by the function
h (t) = −16t 2 + 32t + 48. How long will it take the projectile to hit
the ground?

50. The height of a projectile launched upward at a speed of 16 feet/second


from a height of 192 feet is given by the function
h (t) = −16t 2 + 16t + 192 . How long will it take to hit the ground?

51. An object launched upward at a speed of 64 feet/second from a height of


80 feet. How long will it take the projectile to hit the ground?

52. An object launched upward at a speed of 128 feet/second from a height


of 144 feet. How long will it take the projectile to hit the ground?

6.7 Applications Involving Quadratic Equations 1059


Chapter 6 Factoring and Solving by Factoring

53. The height of an object dropped from the top of a 64-foot building is
given by h (t) = −16t 2 + 64. How long will it take the object to hit the
ground?

54. The height of an object dropped from an airplane at 1,600 feet is given by
h (t) = −16t 2 + 1,600 . How long will it take the object to hit the
ground?

55. An object is dropped from a ladder at a height of 16 feet. How long will it
take to hit the ground?

56. An object is dropped from a 144-foot building. How long will it take to hit
the ground?

57. The height of a projectile, shot straight up into the air from the ground
at 128 feet/second, is given by h (t) = −16t 2 + 128t . How long does it
take to come back down to the ground?

58. A baseball, tossed up into the air from the ground at 32 feet/second, is
given by h (t) = −16t 2 + 32t. How long does it take to come back down
to the ground?

59. How long will it take a baseball thrown into the air at 48 feet/second to
come back down to the ground?

60. A football is kicked up into the air at 80 feet/second. Calculate how long
will it hang in the air.

Part D: Discussion Board Topics

61. Research and discuss the life of Pythagoras.

62. If the sides of a square are doubled, then by what factor is the area
increased? Why?

63. Design your own geometry problem involving the area of a rectangle or
triangle. Post the question and a complete solution on the discussion board.

64. Write down your strategy for setting up and solving word problems.
Share your strategy on the discussion board.

6.7 Applications Involving Quadratic Equations 1060


Chapter 6 Factoring and Solving by Factoring

ANSWERS

1: {4, 20} or {−4, −20}

3: 3, 13

5: {11, 20} or {−22, −10}

7: {5, 7} or {−2, 0}

9: 6, 15

11: {7, 10} or {−2, 1}

13: 10, 12

15: 10, 11

17: 13, 15

19: 5, 7

21: 11, 12

23: 7, 9

25: Length: 17 feet; width: 10 feet

27: Length: 10 units; width: 7 units

29: Length: 12 inches; width: 6 inches

31: Length: 14 inches; width: 12 inches

33: 3 inches

35: 5 inches by 7 inches

37: Base: 8 inches; height: 11 inches

6.7 Applications Involving Quadratic Equations 1061


Chapter 6 Factoring and Solving by Factoring

39: Base: 12 centimeters; height: 6 centimeters

41: Base: 9 units; height: 10 units

43: 3 inches by 4 inches

45: 6 units, 8 units, and 10 units

47: 15 centimeters

49: 3 seconds

51: 5 seconds

53: 2 seconds

55: 1 second

57: 8 seconds

59: 3 seconds

6.7 Applications Involving Quadratic Equations 1062


Chapter 6 Factoring and Solving by Factoring

6.8 Review Exercises and Sample Exam

1063
Chapter 6 Factoring and Solving by Factoring

REVIEW EXERCISES

Introduction to Factoring

Determine the missing factor.

1. 12x 3 − 24x 2 + 4x = 4x ( ? )

2. 10y 4 − 35y 3 − 5y 2 = 5y 2 ( ? )

3. −18a5 + 9a4 − 27a3 = −9a3 ( ? )

4. −21x 2 y + 7xy 2 − 49xy = −7xy ( ? )

Factor out the GCF.

5. 22x 2 + 11x

6. 15y 4 − 5y 3

7. 18a3 − 12a2 + 30a

8. 12a5 + 20a3 − 4a

9. 9x 3 y 2 − 18x 2 y 2 + 27xy 2
5
10. 16a5 b c − 8a3 b 6 + 24a3 b 2 c

Factor by grouping.

11. x 2 + 2x − 5x − 10

12. 2x 2 − 2x − 3x + 3

13. x 3 + 5x 2 − 3x − 15

14. x 3 − 6x 2 + x − 6

6.8 Review Exercises and Sample Exam 1064


Chapter 6 Factoring and Solving by Factoring

15. x 3 − x 2 y − 2x + 2y
2
16. a2 b − 2a3 + 6ab − 3b 3

Factoring Trinomials of the Form x2 + bx + c

Are the following factored correctly? Check by multiplying.

17. x 2 + 5x + 6 = (x + 6) (x − 1)

18. x 2 + 3x − 10 = (x + 5) (x − 2)

19. x 2 + 6x + 9 = (x + 3) 2

20. x 2 − 6x − 9 = (x − 3) (x + 3)

Factor.

21. x 2 − 13x − 14

22. x 2 + 13x + 12

23. y 2 + 10y + 25

24. y 2 − 20y + 100

25. a2 − 8a − 48
2
26. b − 18b + 45

27. x 2 + 2x + 24

28. x 2 − 10x − 16

29. a2 + ab − 2b 2
2
30. a2 b + 5ab − 50

6.8 Review Exercises and Sample Exam 1065


Chapter 6 Factoring and Solving by Factoring

Factoring Trinomials of the Form ax2 + bx + c

Factor.

31. 5x 2 − 27x − 18

32. 3x 2 − 14x + 8

33. 4x 2 − 28x + 49

34. 9x 2 + 48x + 64

35. 6x 2 − 29x − 9

36. 8x 2 + 6x + 9

37. 60x 2 − 65x + 15

38. 16x 2 − 40x + 16

39. 6x 3 − 10x 2 y + 4xy 2

40. 10x 3 y − 82x 2 y 2 + 16xy 3

41. −y 2 + 9y + 36

42. −a2 − 7a + 98

43. 16 + 142x − 18x 2

44. 45 − 132x − 60x 2

Factoring Special Binomials

Factor completely.

45. x 2 − 81

6.8 Review Exercises and Sample Exam 1066


Chapter 6 Factoring and Solving by Factoring

46. 25x 2 − 36

47. 4x 2 − 49

48. 81x 2 −1

49. x 2 − 64y 2

50. 100x 2 y 2 −1

51. 16x 4 − y4

52. x 4 − 81y 4

53. 8x 3 − 125

54. 27 + y3

55. 54x 4 y − 2xy 4

56. 3x 4 y 2 + 24xy 5

57. 64x 6 − y6

58. x 6 +1

General Guidelines for Factoring Polynomials

Factor completely.

59. 8x 3 − 4x 2 + 20x
4
60. 50a4 b c + 5a3 b 5 c2

61. x 3 − 12x 2 − x + 12

62. a3 − 2a2 − 3ab + 6b

6.8 Review Exercises and Sample Exam 1067


Chapter 6 Factoring and Solving by Factoring

63. −y 2 − 15y + 16

64. x 2 − 18x + 72

65. 144x 2 − 25

66. 3x 4 − 48

67. 20x 2 − 41x − 9

68. 24x 2 + 14x − 20

69. a4 b − 343ab 4

70. 32x 7 y 2 + 4xy 8

Solving Equations by Factoring

Solve.

71. (x − 9) (x + 10) = 0

72. −3x (x + 8) = 0

73. 6 (x + 1) (x − 1) = 0

74. (x − 12) (x + 4) (2x − 1) = 0

75. x 2 + 5x − 50 = 0

76. 3x 2 − 13x + 4 = 0

77. 3x 2 − 12 = 0

78. 16x 2 −9=0

79. (x − 2) (x + 6) = 20

6.8 Review Exercises and Sample Exam 1068


Chapter 6 Factoring and Solving by Factoring

80. 2 (x − 2) (x + 3) = 7x − 9

81. 5
2
x2 − 20
3
x=0

82. 2
3
x2 − 5
12
x+ 1
24
=0

Find a quadratic equation with integer coefficients, given the following solutions.

83. −7, 6

84. 0, −10

85. −1/9, 1/2

86. ±3/2

Applications Involving Quadratic Equations

Set up an algebraic equation and then solve the following.

87. An integer is 4 less than twice another. If the product of the two integers
is 96, then find the integers.

88. The sum of the squares of two consecutive positive even integers is 52.
Find the integers.

89. A 20-foot ladder leaning against a wall reaches a height that is 4 feet
more than the distance from the wall to the base of the ladder. How high
does the ladder reach?

90. The height of an object dropped from the top of a 196-foot building is
given by h(t) = −16t 2 + 196 , where t represents the number of
seconds after the object has been released. How long will it take the object to
hit the ground?

91. The length of a rectangle is 1 centimeter less than three times the width.
If the area is 70 square centimeters, then find the dimensions of the
rectangle.

6.8 Review Exercises and Sample Exam 1069


Chapter 6 Factoring and Solving by Factoring

92. The base of a triangle is 4 centimeters more than twice the height. If the
area of the triangle is 80 square centimeters, then find the measure of the
base.

6.8 Review Exercises and Sample Exam 1070


Chapter 6 Factoring and Solving by Factoring

SAMPLE EXAM

2
1. Determine the GCF of the terms 25a2 b c , 50ab 4 , and 35a3 b 3 c2 .

24x 2 y 3 − 16x 3 y 2 + 8x 2 y = 8x 2 y ( )
2. Determine the missing factor:
? .

Factor.

3. 12x 5 − 15x 4 + 3x 2

4. x 3 − 4x 2 − 2x + 8

5. x 2 − 7x + 12

6. 9x 2 − 12x + 4

7. x 2 − 81

8. x 3 + 27y 3

Factor completely.

9. x 3 + 2x 2 − 4x − 8

10. x 4 −1

11. −6x 3 + 20x 2 − 6x

12. x 6 −1

Solve.

13. (2x + 1) (x − 7) = 0

14. 3x (4x − 3) (x + 1) = 0

15. x 2 − 64 = 0

6.8 Review Exercises and Sample Exam 1071


Chapter 6 Factoring and Solving by Factoring

16. x 2 + 4x − 12 = 0

17. 2
3
x2 + 8
9
x− 1
6
=0

18. (x − 5) (x − 3) = −1

19. 3x (x + 3) = 14x + 2

20. (3x + 1) (3x + 2) = 9x + 3

For each problem, set up an algebraic equation and then solve.

21. An integer is 4 less than twice another. If the product of the two integers
is 70, then find the integers.

22. The sum of the squares of two consecutive positive odd integers is 130.
Find the integers.

23. The length of a rectangle is 4 feet more than twice its width. If the area is
160 square feet, then find the dimensions of the rectangle.

24. The height of a triangle is 6 centimeters less than four times the length
of its base. If the area measures 27 square centimeters, then what is the
height of the triangle?

25. The height of a projectile launched upward at a speed of 64 feet/second


from a height of 36 feet is given by the function
h (t) = −16t 2 + 64t + 36. How long will it take the projectile to hit
the ground?

6.8 Review Exercises and Sample Exam 1072


Chapter 6 Factoring and Solving by Factoring

REVIEW EXERCISES ANSWERS

1: (3x 2 − 6x + 1)

3: (2a2 − a + 3)

5: 11x (2x + 1)

7: 6a (3a2 − 2a + 5)

9: 9xy 2 (x − 2x + 3)
2

11: (x + 2) (x − 5)

13: (x + 5) (x 2 − 3)

15: (x − y) (x 2 − 2)

17: No

19: Yes

21: (x − 14) (x + 1)

23: (y + 5)
2

25: (a − 12) (a + 4)

27: Prime

29: (a − b) (a + 2b)

31: (5x + 3) (x − 6)

33: (2x − 7) 2

6.8 Review Exercises and Sample Exam 1073


Chapter 6 Factoring and Solving by Factoring

35: Prime

37: 5 (3x − 1) (4x − 3)

39: 2x (3x − 2y) (x − y)

41: −1 (y − 12) (y + 3)

43: −2 (9x + 1) (x − 8)

45: (x + 9) (x − 9)

47: (2x + 7) (2x − 7)

49: (x + 8y) (x − 8y)

51: (4x 2 + y 2 ) (2x + y) (2x − y)

53: (2x − 5) (4x 2 + 10x + 25)

55: 2xy (3x − y) (9x 2 + 3xy + y 2 )

57: (2x + y) (4x 2 − 2xy + y 2 ) (2x − y) (4x 2 + 2xy + y 2 )

59: 4x (2x 2 − x + 5)

61: (x − 12) (x + 1) (x − 1)

63: −1 (y + 16) (y − 1)

65: (12x + 5) (12x − 5)

67: (4x − 9) (5x + 1)

69: ab (a − 7b) (a2 + 7ab + 49b 2 )

6.8 Review Exercises and Sample Exam 1074


Chapter 6 Factoring and Solving by Factoring

71: 9, −10

73: −1, 1

75: −10, 5

77: ±2

79: −8, 4

81: 0, 8/3

83: x 2 + x − 42 = 0

85: 18x 2 − 7x − 1 = 0

87: {8, 12} or {−6, −16}

89: 16 feet

91: Length: 14 centimeters; width: 5 centimeters

6.8 Review Exercises and Sample Exam 1075


Chapter 6 Factoring and Solving by Factoring

SAMPLE EXAM ANSWERS

2
1: 5ab

3: 3x 2 (4x − 5x + 1)
3 2

5: (x − 4) (x − 3)

7: (x + 9) (x − 9)

9: (x + 2) 2 (x − 2)

11: −2x (3x − 1) (x − 3)

13: −1/2, 7

15: ±8

17: −3/2, 1/6

19: −1/3, 2

21: {7, 10} or {−14, −5}

23: Width: 8 feet; length: 20 feet

25: 4½ sec

6.8 Review Exercises and Sample Exam 1076


Chapter 7
Rational Expressions and Equations

1077
Chapter 7 Rational Expressions and Equations

7.1 Simplifying Rational Expressions

LEARNING OBJECTIVES

1. Determine the restrictions to the domain of a rational expression.


2. Simplify rational expressions.
3. Simplify expressions with opposite binomial factors.
4. Simplify and evaluate rational functions.

Rational Expressions, Evaluating, and Restrictions

A rational number, or fraction ab, is a real number defined as a quotient of two


integers a and b, where b ≠ 0. Similarly, we define a rational expression1, or
algebraic fraction2 QP, as the quotient of two polynomials P and Q, where Q ≠ 0.
Some examples of rational expressions follow:

x+3
The example x−5 consists of linear expressions in both the numerator and
denominator. Because the denominator contains a variable, this expression is not
defined for all values of x.

x+3
Example 1: Evaluate x−5 for the set of x-values {−3, 4, 5}.

Solution: Substitute the values in for x.

1. The quotient QP of two


polynomials P and Q, where Q ≠
0.

2. Term used when referring to a


rational expression.

1078
Chapter 7 Rational Expressions and Equations

Answer: When x = −3 , the value of the rational expression is 0; when x = 4 , the


value of the rational expression is −7; and when x = 5 , the value of the rational
expression is undefined.

This example illustrates that variables are restricted to values that do not make the
denominator equal to 0. The domain of a rational expression3 is the set of real
numbers for which it is defined, and restrictions4 are the real numbers for which
the expression is not defined. We often express the domain of a rational expression
in terms of its restrictions.

x+7
Example 2: Find the domain of the following: .
2x 2 +x−6

Solution: In this example, the numerator x + 7 is a linear expression and the


denominator 2x 2 + x − 6 is a quadratic expression. If we factor the denominator,
then we will obtain an equivalent expression.

3. The set of real numbers for Because rational expressions are undefined when the denominator is 0, we wish to
which the rational expression
is defined.
find the values for x that make it 0. To do this, apply the zero-product property. Set
each factor in the denominator equal to 0 and solve.
4. The set of real numbers for
which a rational expression is
not defined.

7.1 Simplifying Rational Expressions 1079


Chapter 7 Rational Expressions and Equations

We conclude that the original expression is defined for any real number except 3/2
and −2. These two values are the restrictions to the domain.

It is important to note that −7 is not a restriction to the domain because the


expression is defined as 0 when the numerator is 0.

Answer: The domain consists of any real number x, where x ≠ 3


2
and x ≠ −2 .

We can express the domain of the previous example using notation as follows:

The restrictions to the domain of a rational expression are determined by the


denominator. Ignore the numerator when finding those restrictions.

7.1 Simplifying Rational Expressions 1080


Chapter 7 Rational Expressions and Equations

x 4 +x 3 −2x 2 −x
Example 3: Determine the domain: .
x 2 −1

Solution: To find the restrictions to the domain, set the denominator equal to 0 and
solve:

These two values cause the denominator to be 0. Hence they are restricted from the
domain.

Answer: The domain consists of any real number x, where x ≠ ±1 .

2
Example 4: Determine the domain: x −25
4
.

Solution: There is no variable in the denominator and thus no restriction to the


domain.

Answer: The domain consists of all real numbers, R.

Simplifying Rational Expressions

When simplifying fractions, look for common factors that cancel. For example,

7.1 Simplifying Rational Expressions 1081


Chapter 7 Rational Expressions and Equations

We say that the fraction 12/60 is equivalent to 1/5. Fractions are in simplest form if
the numerator and denominator share no common factor other than 1. Similarly,
when working with rational expressions, look for factors to cancel. For example,

The resulting rational expression is equivalent if it shares the same domain.


Therefore, we must make note of the restrictions and write

x+4 1
In words, (x−3)(x+4) is equivalent to x−3 , if x ≠ 3 and x ≠ −4 . We can verify this
by choosing a few values with which to evaluate both expressions to see if the
results are the same. Here we choose x = 7 and evaluate as follows:

It is important to state the restrictions before simplifying rational expressions


because the simplified expression may be defined for restrictions of the original. In
this case, the expressions are not equivalent. Here −4 is defined for the simplified
equivalent but not for the original, as illustrated below:

7.1 Simplifying Rational Expressions 1082


Chapter 7 Rational Expressions and Equations

2
Example 5: Simplify and state the restriction: 25x 3 .
15x

Solution: In this example, the expression is undefined when x is 0.

Therefore, the domain consists of all real numbers x, where x ≠ 0 . With this
understanding, we can cancel common factors.

5
Answer: 3x , where x ≠ 0

7.1 Simplifying Rational Expressions 1083


Chapter 7 Rational Expressions and Equations

3x(x−5)
(2x+1)(x−5)
Example 6: State the restrictions and simplify: .

Solution: To determine the restrictions, set the denominator equal to 0 and solve.

The domain consists of all real numbers except for −1/2 and 5. Next, we find an
equivalent expression by canceling common factors.

3x
Answer: 2x+1 , where x ≠ − 12 and x ≠ 5

Typically, rational expressions are not given in factored form. If this is the case,
factor first and then cancel. The steps are outlined in the following example.

3x+6
Example 7: State the restrictions and simplify: .
x 2 +x−2

Solution:

Step 1: Completely factor the numerator and denominator.

7.1 Simplifying Rational Expressions 1084


Chapter 7 Rational Expressions and Equations

Step 2: Determine the restrictions to the domain. To do this, set the denominator
equal to 0 and solve.

The domain consists of all real numbers except −2 and 1.

Step 3: Cancel common factors, if any.

3
Answer: x−1 , where x ≠ 1 and x ≠ −2

x 2 +7x−30
Example 8: State the restrictions and simplify: .
x 2 −7x+12

Solution: First, factor the numerator and denominator.

7.1 Simplifying Rational Expressions 1085


Chapter 7 Rational Expressions and Equations

Any value of x that results in a value of 0 in the denominator is a restriction. By


inspection, we determine that the domain consists of all real numbers except 4 and
3. Next, cancel common factors.

x+10
Answer: x−4 , where x ≠ 3 and x ≠ 4

It is important to remember that we can only cancel factors of a product. A common


mistake is to cancel terms. For example,

2
Try this! State the restrictions and simplify: x 2 −16 .
5x −20x

x+4
Answer: 5x , where x ≠ 0 and x ≠ 4

Video Solution

(click to see video)

In some examples, we will make a broad assumption that the denominator is


nonzero. When we make that assumption, we do not need to determine the
restrictions.

7.1 Simplifying Rational Expressions 1086


Chapter 7 Rational Expressions and Equations

xy+y 2 −3x−3y
Example 9: Simplify: . (Assume all denominators are nonzero.)
x 2 −y 2

Solution: Factor the numerator by grouping. Factor the denominator using the
formula for a difference of squares.

Next, cancel common factors.

y−3
Answer: x−y

Opposite Binomial Factors

Recall that the opposite of the real number a is −a. Similarly, we can define the
opposite of a polynomial P to be −P. We first consider the opposite of the binomial
a − b:

5. If given a binomial a − b, then This leads us to the opposite binomial property5:

− (a − b) = b − a.
the opposite is

7.1 Simplifying Rational Expressions 1087


Chapter 7 Rational Expressions and Equations

This is equivalent to factoring out a –1.

If a ≠ b, then we can divide both sides by (a − b) and obtain the following:

Example 10: State the restrictions and simplify: 3−x


x−3
.

Solution: By inspection, we can see that the denominator is 0 if x = 3 . Therefore, 3


is the restriction to the domain. Apply the opposite binomial property to the
numerator and then cancel.

Answer: 3−x
x−3
= −1, where x ≠ 3

Since addition is commutative, we have

7.1 Simplifying Rational Expressions 1088


Chapter 7 Rational Expressions and Equations

or

Take care not to confuse this with the opposite binomial property. Also, it is
important to recall that

In other words, show a negative fraction by placing the negative sign in the
numerator, in front of the fraction bar, or in the denominator. Generally, negative
denominators are avoided.

4−x 2
Example 11: Simplify and state the restrictions: .
x 2 +3x−10

Solution: Begin by factoring the numerator and denominator.

7.1 Simplifying Rational Expressions 1089


Chapter 7 Rational Expressions and Equations

x+2
Answer: − x+5 , where x ≠ 2 and x ≠ −5

2
Try this! Simplify and state the restrictions: 2x −7x−15 .
25−x
2

2x+3
Answer: − x+5 , where x ≠ ±5

Video Solution

(click to see video)


Rational Functions

Rational functions have the form

where p(x) and q(x) are polynomials and q(x) ≠ 0. The domain of a rational
function consists of all real numbers x such that the denominator q(x) ≠ 0.

Example 12:

2x 2 +5x−3
a. Simplify: r (x) = .
6x 2 +18x

b. State the domain.

c. Calculate r(−2).

Solution:

a. To simplify the rational function, first factor and then cancel.

7.1 Simplifying Rational Expressions 1090


Chapter 7 Rational Expressions and Equations

b. To determine the restrictions, set the denominator of the original function equal
to 0 and solve.

The domain consists of all real numbers x, where x ≠ 0 and x ≠ −3 .

c. Since −2 is not a restriction, substitute it for the variable x using the simplified
form.

7.1 Simplifying Rational Expressions 1091


Chapter 7 Rational Expressions and Equations

Answers:

a. r (x) = 2x−1
6x

b. The domain is all real numbers except 0 and −3.

c. r (−2) = 5
12

If a cost function6 C(x) represents the cost of producing x units, then the average
cost7 c(x) is the cost divided by the number of units produced.

Example 13: The cost in dollars of producing t-shirts with a company logo is given
by C (x) = 7x + 200, where x represents the number of shirts produced.
Determine the average cost of producing

a. 40 t-shirts

b. 250 t-shirts

c. 1,000 t-shirts

Solution: Set up a function representing the average cost.

6. A function that represents the


cost of producing a certain
number of units.

7. The total cost divided by the


number of units produced,
which can be represented by
C(x)
c(x) = x , where C(x) is a Next, calculate c(40), c(250), and c(1000).
cost function.

7.1 Simplifying Rational Expressions 1092


Chapter 7 Rational Expressions and Equations

Answers:

a. If 40 t-shirts are produced, then the average cost per t-shirt is $12.00.

b. If 250 t-shirts are produced, then the average cost per t-shirt is $7.80.

c. If 1,000 t-shirts are produced, then the average cost per t-shirt is $7.20.

KEY TAKEAWAYS

• Rational expressions usually are not defined for all real numbers. The
real numbers that give a value of 0 in the denominator are not part of
the domain. These values are called restrictions.
• Simplifying rational expressions is similar to simplifying fractions. First,
factor the numerator and denominator and then cancel the common
factors. Rational expressions are simplified if there are no common
factors other than 1 in the numerator and the denominator.
• Simplified rational expressions are equivalent for values in the domain
of the original expression. Be sure to state the restrictions if the
denominators are not assumed to be nonzero.

involve subtraction. Use − (a − b) = b − ato replace factors that


• Use the opposite binomial property to cancel binomial factors that

such as (a + b) = (b + a).
will then cancel. Do not confuse this with factors that involve addition,

7.1 Simplifying Rational Expressions 1093


Chapter 7 Rational Expressions and Equations

TOPIC EXERCISES

Part A: Rational Expressions

Evaluate for the given set of x-values.

5
1. x ; {−1, 0, 1}

4x
2. ; {−1, 0, 1}
3x 2

1
3. ; {−10, −9, 0}
x+9

x+6
4. ; {−6, 0, 5}
x−5

3x(x−2)
5. ; {0, 1/2, 2}
2x−1

9x 2 −1
6. x−7 ; {0, 1/3, 7}

5
7. ; {−3, 0, 3}
x 2 −9

x 2 −25
8. ; {−5, −4, 5}
x 2 −3x−10

9. Fill in the following chart:

10. Fill in the following chart:

7.1 Simplifying Rational Expressions 1094


Chapter 7 Rational Expressions and Equations

11. Fill in the following chart:

12. Fill in the following chart:

7.1 Simplifying Rational Expressions 1095


Chapter 7 Rational Expressions and Equations

An object’s weight depends on its height above the surface of earth. If an object
weighs 120 pounds on the surface of earth, then its weight in pounds, W, x miles
above the surface is approximated by the formula

120 ⋅ 4000 2
W=
(4000 + x) 2

For each problem below, approximate the weight of a 120-pound object at the given
height above the surface of earth. (1 mile = 5,280 feet)

13. 100 miles

14. 1,000 miles

15. 44,350 feet

16. 90,000 feet

The price to earnings ratio (P/E) is a metric used to compare the valuations of
similar publicly traded companies. The P/E ratio is calculated using the stock price
and the earnings per share (EPS) over the previous 12‑month period as follows:

price per share


P/E =
earnings per share

If each share of a company stock is priced at $22.40, then calculate the P/E ratio
given the following values for the earnings per share.

17. $1.40

18. $1.21

19. What happens to the P/E ratio when earnings decrease?

20. What happens to the P/E ratio when earnings increase?

7.1 Simplifying Rational Expressions 1096


Chapter 7 Rational Expressions and Equations

State the restrictions to the domain.

1
21.
3x

3x 2
22.
7x 5

3x(x+1)
23.
x+4

2x 2 (x−3)
24.
x−1

1
25.
5x−1

x−2
26.
3x−2

x−9
27.
5x(x−2)

(x−3)(x+6)
1
28.

x
29.
1−x 2

x 2 −9
30.
x 2 −36

1
31.
2x(x+3)(2x−1)

x−3
32.
(3x−1)(2x+3)

4x(2x+1)
33.
12x 2 +x−1

x−5
34.
3x 2 −15x

Part B: Simplifying Rational Expressions

State the restrictions and then simplify.

7.1 Simplifying Rational Expressions 1097


Chapter 7 Rational Expressions and Equations

5x 2
35.
20x 3

12x 6
36.
60x

3x 2 (x−2)
37.
9x(x−2)

20(x−3)(x−5)
38.
6(x−3)(x+1)

6x 2 (x−8)
39.
36x(x+9)(x−8)

16x 2 −1
40.
(4x+1) 2

9x 2 −6x+1
41.
(3x−1) 2

x−7
42.
x 2 −49

x 2 −64
43.
x 2 +8x

x+10
44.
x 2 −100

2x 3 −12x 2
45.
5x 2 −30x

30x 5 +60x 4
46.
2x 3 −8x

2x−1
47.
2x 2 +x−6

x 2 −x−6
48.
3x 2 −8x−3

6x 2 −25x+25
49.
3x 2 +16x−35

3x 2 +4x−15
50.
x 2 −9

7.1 Simplifying Rational Expressions 1098


Chapter 7 Rational Expressions and Equations

x 2 −10x+21
51.
x 2 −4x−21

x 3 −1
52.
x 2 −1

x 3 +8
53.
x 2 −4

x 4 −16
54.
x 2 −4

Part C: Simplifying Rational Expressions with Opposite Binomial Factors

State the restrictions and then simplify.

x−9
55.
9−x

3x−2
56.
2−3x

x+6
57.
6+x

3x+1
58.
1+3x

(2x−5)(x−7)
59.
(7−x)(2x−1)

(3x+2)(x+5)
(x−5)(2+3x)
60.

x 2 −4
61.
(2−x) 2

16−9x 2
62.
(3x+4) 2

4x 2 (10−x)
63.
3x 3 −300x

−2x+14
64.
x 3 −49x

7.1 Simplifying Rational Expressions 1099


Chapter 7 Rational Expressions and Equations

2x 2 −7x−4
65.
1−4x 2

9x 2 −4
66.
4x−6x 2

x 2 −5x−14
67.
7−15x+2x 2

2x 3 +x 2 −2x−1
68.
1+x−2x 2

x 3 +2x−3x 2 −6
69.
2+x 2

27+x 3
70.
x 2 +6x+9

64−x 3
71.
x 2 −8x+16

x 2 +4
72.
4−x 2

Simplify. (Assume all denominators are nonzero.)

−15x 3 y 2
5xy 2 (x+y)
73.

14x 7 y 2 (x−2y)
4

7x 8 y(x−2y)
74. 2

y+x
75.
x 2 −y 2

y−x
76.
x 2 −y 2

x 2 −y 2
(x−y)
77. 2

a2 −ab−6b 2
78.
a2 −6ab+9b 2

7.1 Simplifying Rational Expressions 1100


Chapter 7 Rational Expressions and Equations

2a2 −11a+12
79.
−32+2a2

a2 b−3a2
80.
3a2 −3ab

xy 2 −x+y 3 −y
81.
x−xy 2

x 3 −xy 2 −x 2 y+y 3
82.
x 2 −2xy+y 2

x 3 −27
83.
x 2 +3x+9

x 2 −x+1
84.
x 3 +1

Part D: Rational Functions

Calculate the following.

5x
85. f (x) = ;f (0), f (2), f (4)
x−3

x+7
86. f (x) = ;f (−1) , f (0), f (1)
x 2 +1

x3
87. g (x) = ; g (0) , g (2) , g (−2)
(x−2) 2

x 2 −9
88. g (x) = ; g (−2) , g (0) , g (2)
9−x 2

x3
89. g (x) = ; g (−1) , g (0) , g (1)
x 2 +1

; g (−1/5) , g (−1) , g (−5)


5x+1
90. g (x) = x 2 −25

State the restrictions to the domain and then simplify.

−3x 2 −6x
91. f (x) = x 2 +4x+4

7.1 Simplifying Rational Expressions 1101


Chapter 7 Rational Expressions and Equations

x 2 +6x+9
92. f (x) = 2x 2 +5x−3

9−x
93. g (x) = x 2 −81

x 3 −27
94. g (x) = 3−x

3x−15
95. g (x) = 10−2x

25−5x
96. g (x) = 4x−20

97. The cost in dollars of producing coffee mugs with a company logo is
given by C (x) = x + 40 , where x represents the number of mugs
produced. Calculate the average cost of producing 100 mugs and the average
cost of producing 500 mugs.

98. The cost in dollars of renting a moving truck for the day is given by
C (x) = 0.45x + 90 , where x represents the number of miles driven.
Calculate the average cost per mile if the truck is driven 250 miles in one
day.

99. The cost in dollars of producing sweat shirts with a custom design on the
back is given by C(x) = 1200 + (12 − 0.05x)x , where x represents
the number of sweat shirts produced. Calculate the average cost of
producing 150 custom sweat shirts.

100. The cost in dollars of producing a custom injected molded part is given
by C(x) = 500 + (3 − 0.001x)x , where x represents the number of
parts produced. Calculate the average cost of producing 1,000 custom parts.

Part E: Discussion Board

b−a
101. Explain why
a−b
= −1 and illustrate this fact by substituting some
numbers for the variables.

b+a
102. Explain why
a+b
= 1 and illustrate this fact by substituting some
numbers for the variables.

x
103. Explain why we cannot cancel x in the expression .
x+1

7.1 Simplifying Rational Expressions 1102


Chapter 7 Rational Expressions and Equations

ANSWERS

1: −5, undefined, 5

3: −1, undefined, 1/9

5: 0, undefined, 0

7: Undefined, −5/9, undefined

9:

11:

13: 114 pounds

15: 119.5 pounds

7.1 Simplifying Rational Expressions 1103


Chapter 7 Rational Expressions and Equations

17: 16

19: The P/E ratio increases.

21: x ≠0

23: x ≠ −4

25: x ≠ 1
5

27: x ≠ 0 and x ≠ 2

29: x ≠ ±1

31: x ≠ 0 , x ≠ −3 , and x ≠ 1
2

33: x ≠− 1
3
and x ≠ 1
4

1
35: ;x ≠0
4x

x
37: ;x ≠ 0, 2
3

x
39: ;x ≠ 0, −9, 8
6(x+9)

41: 1 ; x ≠ 1
3

x−8
43: x ;x ≠ 0, −8

2x
45: ;x ≠ 0, 6
5

2x−1
47: ;x ≠ −2, 3
2x 2 +x−6 2

2x−5
49: x+7 ; x ≠ −7, 5
3

x−3
51: ;x ≠ −3, 7
x+3

7.1 Simplifying Rational Expressions 1104


Chapter 7 Rational Expressions and Equations

x 2 −2x+4
53: ;x ≠ ±2
x−2

55: −1; x ≠9

57: 1; x ≠ −6
2x−5
59: − ;x ≠ 1
,7
2x−1 2

x+2
61: ;x ≠2
x−2

4x
63: − ;x ≠ ±10, 0
3(x+10)

x−4
65: ;x ≠± 1
1−2x 2

x+2
67: ;x ≠ 1
,7
2x−1 2

69: x − 3 ; none

16+4x+x 2
71: − ;x ≠4
x−4

3x 2
73: − x+y

1
75: x−y

x+y
77: x−y

2a−3
79:
2(4+a)

x+y
81: − x

83: x −3

85: f (0) = 0, f (2) = −10 , f (4) = 20

87: g (0) = 0, g (2) undefined, g (−2) = −1/2

7.1 Simplifying Rational Expressions 1105


Chapter 7 Rational Expressions and Equations

89: g (−1) = −1/2 , g (0) = 0, g (1) = 1/2


3x
91: f (x) =− ;x ≠ −2
x+2

1
93: g(x) =− ;x ≠ ±9
x+9

3
95: g(x) =− ;x ≠5
2

97: The average cost of producing 100 mugs is $1.40 per mug. The average
cost of producing 500 mugs is $1.08 per mug.

99: $12.50

7.1 Simplifying Rational Expressions 1106


Chapter 7 Rational Expressions and Equations

7.2 Multiplying and Dividing Rational Expressions

LEARNING OBJECTIVES

1. Multiply rational expressions.


2. Divide rational expressions.
3. Multiply and divide rational functions.

Multiplying Rational Expressions

When multiplying fractions, we can multiply the numerators and denominators


together and then reduce, as illustrated:

Multiplying rational expressions is performed in a similar manner. For example,

In general, given polynomials P, Q, R, and S, where Q ≠ 0 and S ≠ 0 , we have

In this section, assume that all variable expressions in the denominator are nonzero
unless otherwise stated.

1107
Chapter 7 Rational Expressions and Equations

2 20y 4
Example 1: Multiply: 12x3 ⋅ .
5y 3 6x

Solution: Multiply numerators and denominators and then cancel common factors.

8y
Answer: x

Example 2: Multiply: x−3


x+5
⋅ x+5
x+7
.

Solution: Leave the product in factored form and cancel the common factors.

Answer: x−3
x+7

15x 2 y 3 x(2x−1)
Example 3: Multiply: (2x−1) ⋅ 2 .
3x y(x+3)

7.2 Multiplying and Dividing Rational Expressions 1108


Chapter 7 Rational Expressions and Equations

Solution: Leave the polynomials in the numerator and denominator factored so


that we can cancel the factors. In other words, do not apply the distributive
property.

5xy 2
Answer: x+3

Typically, rational expressions will not be given in factored form. In this case, first
factor all numerators and denominators completely. Next, multiply and cancel any
common factors, if there are any.

x+5
Example 4: Multiply: x−5 ⋅ x−5 .
x 2 −25

Solution: Factor the denominator x 2 − 25 as a difference of squares. Then


multiply and cancel.

7.2 Multiplying and Dividing Rational Expressions 1109


Chapter 7 Rational Expressions and Equations

Keep in mind that 1 is always a factor; so when the entire numerator cancels out,
make sure to write the factor 1.

1
Answer: x−5

x 2 +3x+2 x 2 −7x+12
Example 5: Multiply:
x 2 −5x+6
⋅ x 2 +8x+7
.

Solution:

It is a best practice to leave the final answer in factored form.

(x+2)(x−4)
Answer: (x−2)(x+7)

−2x 2 +x+3 3x−6


Example 6: Multiply:
x 2 +2x−8
⋅ x 2 +x
.

Solution: The trinomial −2x 2 + x + 3 in the numerator has a negative leading


coefficient. Recall that it is a best practice to first factor out a −1 and then factor the
resulting trinomial.

7.2 Multiplying and Dividing Rational Expressions 1110


Chapter 7 Rational Expressions and Equations

3(2x−3)
Answer: − x(x+4)

2
Example 7: Multiply: 7−x
2
⋅ x +10x+21
x +3x 2
.
x −49

Solution: We replace 7 − x with −1 (x − 7) so that we can cancel this factor.

Answer: − 1x

7.2 Multiplying and Dividing Rational Expressions 1111


Chapter 7 Rational Expressions and Equations

2 2
Try this! Multiply: x8−x
−64
⋅ x 2x+x
+9x+8
.

Answer: −x

Video Solution

(click to see video)


Dividing Rational Expressions

To divide two fractions, we multiply by the reciprocal of the divisor, as illustrated:

Dividing rational expressions is performed in a similar manner. For example,

In general, given polynomials P, Q, R, and S, where Q ≠ 0, R ≠ 0 , and S ≠ 0 , we


have

8x 5 y 20xy 4
Example 8: Divide:
25z 6
÷ 15z 3
.

Solution: First, multiply by the reciprocal of the divisor and then cancel.

7.2 Multiplying and Dividing Rational Expressions 1112


Chapter 7 Rational Expressions and Equations

4
Answer: 6x3 3
25y z

x+2 x+3
Example 9: Divide:
x 2 −4
÷ x−2
.

Solution: After multiplying by the reciprocal of the divisor, factor and cancel.

1
Answer: x+3

7.2 Multiplying and Dividing Rational Expressions 1113


Chapter 7 Rational Expressions and Equations

2 x 2 +9x+14
Example 10: Divide: x 2 −6x−16 ÷ 2 .
x +4x−21 x −8x+15

Solution: Begin by multiplying by the reciprocal of the divisor. After doing so,
factor and cancel.

(x−8)(x−5)
Answer:
(x+7) 2

9−4x 2
Example 11: Divide: x+2 ÷ (2x − 3).

Solution: Just as we do with fractions, think of the divisor (2x − 3) as an algebraic


fraction over 1.

7.2 Multiplying and Dividing Rational Expressions 1114


Chapter 7 Rational Expressions and Equations

2x+3
Answer: − x+2

4x 2 +7x−2 1−4x
Try this! Divide:
25x 2
÷ 100x 4
.

Answer: −4x 2 (x + 2)

Video Solution

(click to see video)


Multiplying and Dividing Rational Functions

The product and quotient of two rational functions can be simplified using the
techniques described in this section. The restrictions to the domain of a product
consist of the restrictions of each function.

Example 12: Calculate (f ⋅ g) (x) and determine the restrictions to the domain.

7.2 Multiplying and Dividing Rational Expressions 1115


Chapter 7 Rational Expressions and Equations

Solution: In this case, the domain of f (x) consists of all real numbers except 0, and
the domain of g(x) consists of all real numbers except 1/4. Therefore, the domain of
the product consists of all real numbers except 0 and 1/4. Multiply the functions
and then simplify the result.

Answer: (f ⋅ g) (x) = − 5x , where x ≠ 0,


4x+1 1
4

The restrictions to the domain of a quotient will consist of the restrictions of each
function as well as the restrictions on the reciprocal of the divisor.

Example 13: Calculate (f /g) (x) and determine the restrictions.

Solution:

7.2 Multiplying and Dividing Rational Expressions 1116


Chapter 7 Rational Expressions and Equations

In this case, the domain of f (x) consists of all real numbers except 3 and 8, and the
domain of g(x) consists all real numbers except 3. In addition, the reciprocal of g(x)
has a restriction of −8. Therefore, the domain of this quotient consists of all real
numbers except 3, 8, and −8.

Answer: (f /g) (x) = 1, where x ≠ 3, 8, − 8

KEY TAKEAWAYS

• After multiplying rational expressions, factor both the numerator and


denominator and then cancel common factors. Make note of the
restrictions to the domain. The values that give a value of 0 in the
denominator are the restrictions.
• To divide rational expressions, multiply by the reciprocal of the divisor.
• The restrictions to the domain of a product consist of the restrictions to
the domain of each factor.
• The restrictions to the domain of a quotient consist of the restrictions to
the domain of each rational expression as well as the restrictions on the
reciprocal of the divisor.

7.2 Multiplying and Dividing Rational Expressions 1117


Chapter 7 Rational Expressions and Equations

TOPIC EXERCISES

Part A: Multiplying Rational Expressions

Multiply. (Assume all denominators are nonzero.)

2x 9
1.
3
⋅ 4x 2

−5x 3 y2
2. y ⋅ 25x

5x 2 4y 2
3.
2y
⋅ 15x 3

16a4 49b
4. ⋅ 32a3
7b 2

x−6 24x 2
5.
12x 3
⋅ x−6

x+10 x−2
6.
2x−1
⋅ x+10

(y−1)
2
1
7.
y+1
⋅ y−1

y 2 −9 2y−3
8.
y+3
⋅ y−3

2a−5 2a+5
9.
a−5
⋅ 4a2 −25

2a2 −9a+4
10.
a2 −16
⋅ (a2 + 4a)

2x 2 +3x−2 2x
11. ⋅ x+2
(2x−1) 2

9x 2 +19x+2 x 2 −4x+4
12.
4−x 2
⋅ 9x 2 −8x−1

x 2 +8x+16 x 2 −3x−4
13.
16−x 2
⋅ x 2 +5x+4

7.2 Multiplying and Dividing Rational Expressions 1118


Chapter 7 Rational Expressions and Equations

x 2 −x−2 x 2 +2x−15
14.
x 2 +8x+7
⋅ x 2 −5x+6

x+1 3−x
15.
x−3
⋅ x+5

2x−1 x+6
16.
x−1
⋅ 1−2x

9+x 3
17.
3x+1
⋅ x+9

1 5x+2
18.
2+5x
⋅ 5x

100−y 2 25y 2
19.
y−10
⋅ y+10

3y 3 36y 2 −25
20.
6y−5
⋅ 5+6y

3a2 +14a−5 3a+1


21.
a2 +1
⋅ 1−9a2

4a2 −16a 1−16a2


22.
4a−1
⋅ 4a2 −15a−4

23.
x+9
−x 2 +14x−45
⋅ (x 2 − 81)

24.
1
2+5x
⋅ (25x 2 + 20x + 4)

x 2 +x−6 2x 2 −8
25.
3x 2 +15x+18
⋅ x 2 −4x+4

5x 2 −4x−1 25x 2 −10x+1


26.
5x 2 −6x+1
⋅ 3−75x 2

Part B: Dividing Rational Expressions

Divide. (Assume all denominators are nonzero.)

5x 15x 2
27.
8
÷ 4

7.2 Multiplying and Dividing Rational Expressions 1119


Chapter 7 Rational Expressions and Equations

3 −15
28.
8y
÷ 2y 2

5x 9
3y 3
29.
25x 10
9y 5

12x 4 y 2
21z 5
30.
6x 3 y 2
7z 3

(x−4) 2 x−4
31.
30x 4
÷ 15x

5y 4 10y 5
÷
10(3y−5) 2(3y−5)
32. 2 3

x 2 −9
33.
5x
÷ (x − 3)

÷ (8 + y)
y 2 −64
34.
8y

(a−8)2 a−8
35.
2a2 +10a
÷ a

3 12ab(a−2b)
36. 24a2 b (a − 2b) ÷ 5

x 2 +7x+10 1
37.
x 2 +4x+4
÷ x 2 −4

2x 2 −x−1 1
38.
2x 2 −3x+1
÷ 4x 2 −1

y+1 y 2 −1
39.
y 2 −3y
÷ y 2 −6y+9

9−a2 2a2 −10a


40.
a2 −8a+15
÷ a2 −10a+25

a2 −3a−18 a2 +a−6
41.
2a2 −11a−6
÷ 2a2 −a−1

7.2 Multiplying and Dividing Rational Expressions 1120


Chapter 7 Rational Expressions and Equations

y 2 −7y+10
y 2 +5y−14
42.
2y 2 −9y−5
y 2 +14y+49

6y 2 +y−1
4y 2 +4y+1
43.
3y 2 +2y−1
2y 2 −7y−4

x 2 −7x−18 x 2 −81
44.
x 2 +8x+12
÷ x 2 +12x+36

÷ (b − 2a)
4a2 −b 2 2
45.
b+2a

÷ (y − x)
x 2 −y 2 2
46. y+x

5y 2 (y−3) 25y(3−y)
47.
4x 3
÷ 2x 2

15x 3 25x 6
3(y+7)
÷
9(7+y)
48. 2

3x+4 7x
49.
x−8
÷ 8−x

3x−2 2−3x
50.
2x+1
÷ 3x

(7x−1) 2 28x 2 −11x+1


51.
4x+1
÷ 1−4x

4x 2−x
52. ÷ x 2 −4
(x+2) 2

÷ (b − a)
a2 −b 2 2
53. a

÷ (2b 2 + ab − a2 )
(a−2b)2
54.
2b

x 2 −6x+9 9−x 2
55.
x 2 +7x+12
÷ x 2 +8x+16

7.2 Multiplying and Dividing Rational Expressions 1121


Chapter 7 Rational Expressions and Equations

2x 2 −9x−5 1−4x+4x 2
56.
25−x 2
÷ −2x 2 −9x+5

3x 2 −16x+5
100−4x 2
57.
9x 2 −6x+1
3x 2 +14x−5

10x 2 −25x−15
x 2 −6x+9
58.
9−x 2
2
x +6x+9

Recall that multiplication and division are to be performed in the order they appear
from left to right. Simplify the following.

1 x−1 x−1
59.
x2
⋅ x+3
÷ x3

x−7 1 x−7
60.
x+9
⋅ x3
÷ x

x+1 x x2
61.
x−2
÷ x−5
⋅ x+1

x+4 x−3 x+4


62.
2x+5
÷ 2x+5
⋅ x−3

2x−1 x−4 x−4


63.
x+1
÷ x 2 +1
⋅ 2x−1

4x 2 −1 2x−1 3x+2
64.
3x+2
÷ x+5
⋅ 2x+1

Part C: Multiplying and Dividing Rational Functions

Calculate (f ⋅ g) (x) and determine the restrictions to the domain.

1 1
65. f (x) = x and g(x) = x−1

x+1
66. f (x) = x−1
and g(x) = x2 − 1

3x+2 x 2 −4
67. f (x) = and g(x) =
x+2 (3x+2) 2

7.2 Multiplying and Dividing Rational Expressions 1122


Chapter 7 Rational Expressions and Equations

(x−6)
2
(1−3x) 2
68. f (x) = and g(x) =
x−6 9x 2 −1

25x 2 −1 x 2 −9
69. f (x) = and g(x) =
x 2 +6x+9 5x+1

x 2 −49 4x 2 −4x+1
70. f (x) = and g(x) = 7−x
2x 2 +13x−7

Calculate (f /g) (x) and state the restrictions.

1 x−2
71. f (x) = x and g(x) = x−1

(5x+3)
2
5x+3
72. f (x) = and g(x) =
x2 6−x

5−x x 2 −25
73. f (x) = and g(x) =
(x−8) 2 x−8

x 2 −2x−15 2x 2 −5x−3
74. f (x) = and g(x) =
x 2 −3x−10 x 2 −7x+12

3x 2 +11x−4 x 2 −2x+1
75. f (x) = and g(x) =
9x 2 −6x+1 3x 2 −4x+1

36−x 2 x 2 −12x+36
76. f (x) = and g(x) =
x 2 +12x+36 x 2 +4x−12

Part D: Discussion Board Topics

77. In the history of fractions, who is credited for the first use of the fraction
bar?

78. How did the ancient Egyptians use fractions?

1 x−7
79. Explain why x = 7 is a restriction to x ÷ .
x−2

7.2 Multiplying and Dividing Rational Expressions 1123


Chapter 7 Rational Expressions and Equations

ANSWERS

3
1:
2x

2y
3:
3x

2
5: x

y−1
7:
y+1

1
9:
a−5

2x
11:
2x−1

13: −1

x+1
15: −
x+5

3
17:
3x+1

19: −25y 2

a+5
21: −
a2 +1

(x+9) 2
23: −
x−5

25: 2/3

1
27:
6x

3y 2
29:
5x

x−4
31:
2x 3

7.2 Multiplying and Dividing Rational Expressions 1124


Chapter 7 Rational Expressions and Equations

x+3
33:
5x

2(a+5)
a−8
35:

37: (x + 5) (x − 2)

y−3
y(y−1)
39:

a−1
41:
a−2

y−4
43:
y+1

1
45:
2a−b

y
47: −
10x

3x+4
49: − 7x

7x−1
51: −
4x+1

a+b
53: −
a(b−a)

(x−3)(x+4)
55: −
(x+3) 2

57: −1/4

x
59:
x+3

x(x−5)
61:
x−2

x 2 +1
63:
x+1

7.2 Multiplying and Dividing Rational Expressions 1125


Chapter 7 Rational Expressions and Equations

65: (f ⋅ g) (x) = 1
;x ≠ 0, 1
x(x−1)

67: (f ⋅ g) (x) = x−2


;x ≠ −2, − 2
3x+2 3

69: (f ⋅ g) (x) =
(x−3)(5x−1)
;x ≠ −3, − 1
x+3 5

71: (f /g) (x)


x−1
= x(x−2)
;x ≠ 0, 1, 2

73: (f /g) (x)


(x−8)(x+5)
1
=− ;x ≠ ±5, 8

75: (f /g) (x)


(x+4)
= (x−1)
;x ≠ 1
3
,1

7.2 Multiplying and Dividing Rational Expressions 1126


Chapter 7 Rational Expressions and Equations

7.3 Adding and Subtracting Rational Expressions

LEARNING OBJECTIVES

1. Add and subtract rational expressions with common denominators.


2. Add and subtract rational expressions with unlike denominators.
3. Add and subtract rational functions.

Adding and Subtracting with Common Denominators

Adding and subtracting rational expressions is similar to adding and subtracting


fractions. Recall that if the denominators are the same, we can add or subtract the
numerators and write the result over the common denominator.

When working with rational expressions, the common denominator will be a


polynomial. In general, given polynomials P, Q, and R, where Q ≠ 0, we have the
following:

In this section, assume that all variable factors in the denominator are nonzero.

Example 1: Add: 3y + 7y.

1127
Chapter 7 Rational Expressions and Equations

Solution: Add the numerators 3 and 7, and write the result over the common
denominator, y.

Answer: 10
y

x−5 1
Example 2: Subtract: 2x−1 − 2x−1.

Solution: Subtract the numerators x − 5 and 1, and write the result over the
common denominator, 2x − 1 .

x−6
Answer: 2x−1

2x+7 x+10
( )(x−3) ( )(x−3)
Example 3: Subtract: − .
x+5 x+5

Solution: We use parentheses to remind us to subtract the entire numerator of the


second rational expression.

7.3 Adding and Subtracting Rational Expressions 1128


Chapter 7 Rational Expressions and Equations

1
Answer: x+5

2x 2 +10x+3 x 2 +6x+5 x−4


Example 4: Simplify:
x 2 −36
− x 2 −36
+ .
x 2 −36

Solution: Subtract and add the numerators. Make use of parentheses and write the
result over the common denominator, x 2 − 36.

Answer: x−1
x−6

7.3 Adding and Subtracting Rational Expressions 1129


Chapter 7 Rational Expressions and Equations

x 2 +1 x 2 −2x
Try this! Subtract:
2x 2 −7x−4
− 2x 2 −7x−4
.

1
Answer: x−4

Video Solution

(click to see video)


Adding and Subtracting with Unlike Denominators

To add rational expressions with unlike denominators, first find equivalent


expressions with common denominators. Do this just as you have with fractions. If
the denominators of fractions are relatively prime, then the least common
denominator (LCD) is their product. For example,

Multiply each fraction by the appropriate form of 1 to obtain equivalent fractions


with a common denominator.

The process of adding and subtracting rational expressions is similar. In general,


given polynomials P, Q, R, and S, where Q ≠ 0 and S ≠ 0 , we have the following:

7.3 Adding and Subtracting Rational Expressions 1130


Chapter 7 Rational Expressions and Equations

In this section, assume that all variable factors in the denominator are nonzero.

Example 5: Add: 1x + 1y.

Solution: In this example, the LCD = xy . To obtain equivalent terms with this
y
common denominator, multiply the first term by y and the second term by xx .

y+x
Answer: xy

Example 6: Subtract: 1y − y−3


1
.

Solution: Since the LCD = y(y − 3) , multiply the first term by 1 in the form of
(y−3) y
and the second term by y .
(y−3)

7.3 Adding and Subtracting Rational Expressions 1131


Chapter 7 Rational Expressions and Equations

3
y(y−3)
Answer: −

It is not always the case that the LCD is the product of the given denominators.
Typically, the denominators are not relatively prime; thus determining the LCD
requires some thought. Begin by factoring all denominators. The LCD is the product
of all factors with the highest power. For example, given

there are three base factors in the denominator: x , (x + 2), and (x − 3). The
highest powers of these factors are x 3 , (x + 2)2 , and (x − 3)1 . Therefore,

The general steps for adding or subtracting rational expressions are illustrated in
the following example.

7.3 Adding and Subtracting Rational Expressions 1132


Chapter 7 Rational Expressions and Equations

x 3
Example 7: Subtract:
x 2 +4x+3
− x 2 −4x−5
.

Solution:

Step 1: Factor all denominators to determine the LCD.

The LCD is (x + 1) (x + 3) (x − 5).

Step 2: Multiply by the appropriate factors to obtain equivalent terms with a


(x−5)
common denominator. To do this, multiply the first term by and the second
(x−5)
(x+3)
term by (x+3)
.

Step 3: Add or subtract the numerators and place the result over the common
denominator.

Step 4: Simplify the resulting algebraic fraction.

7.3 Adding and Subtracting Rational Expressions 1133


Chapter 7 Rational Expressions and Equations

(x−9)
(x+3)(x−5)
Answer:

x 2 −9x+18 x
Example 8: Subtract:
x 2 −13x+36
− x−4
.

Solution: It is best not to factor the numerator, x 2 − 9x + 18, because we will


most likely need to simplify after we subtract.

7.3 Adding and Subtracting Rational Expressions 1134


Chapter 7 Rational Expressions and Equations

18
Answer: (x−4)(x−9)

1 1
Example 9: Subtract:
x 2 −4
− 2−x
.

Solution: First, factor the denominators and determine the LCD. Notice how the
opposite binomial property is applied to obtain a more workable denominator.

The LCD is (x + 2) (x − 2). Multiply the second term by 1 in the form of


(x+2)
(x+2)
.

7.3 Adding and Subtracting Rational Expressions 1135


Chapter 7 Rational Expressions and Equations

Now that we have equivalent terms with a common denominator, add the
numerators and write the result over the common denominator.

x+3
Answer: (x+2)(x−2)

y−1 y+1 y 2 −5
Example 10: Simplify: y+1 − y−1 + 2 .
y −1

Solution: Begin by factoring the denominator.

We can see that the LCD is (y + 1) (y − 1). Find equivalent fractions with this
denominator.

7.3 Adding and Subtracting Rational Expressions 1136


Chapter 7 Rational Expressions and Equations

Next, subtract and add the numerators and place the result over the common
denominator.

Finish by simplifying the resulting rational expression.

y−5
Answer: y−1

x 5
Try this! Simplify: −2 + 1+x − 1−x.
x 2 −1

x+3
Answer: x−1

7.3 Adding and Subtracting Rational Expressions 1137


Chapter 7 Rational Expressions and Equations

Video Solution

(click to see video)

Rational expressions are sometimes expressed using negative exponents. In this


case, apply the rules for negative exponents before simplifying the expression.

Example 11: Simplify: y −2 + (y − 1) .


−1

Solution: Recall that x −n = x1n . We begin by rewriting the negative exponents as


rational expressions.

y 2 +y−1
y 2 (y−1)
Answer:

7.3 Adding and Subtracting Rational Expressions 1138


Chapter 7 Rational Expressions and Equations

Adding and Subtracting Rational Functions

We can simplify sums or differences of rational functions using the techniques


learned in this section. The restrictions of the result consist of the restrictions to
the domains of each function.

Example 12: Calculate (f + g) (x), given f (x) = x+3


1 1
and g (x) = x−2 , and state
the restrictions.

Solution:

Here the domain of f consists of all real numbers except −3, and the domain of g
consists of all real numbers except 2. Therefore, the domain of f + g consists of all
real numbers except −3 and 2.

2x+1
Answer: (x+3)(x−2) , where x ≠ −3, 2

7.3 Adding and Subtracting Rational Expressions 1139


Chapter 7 Rational Expressions and Equations

Example 13: Calculate (f − g) (x), given f (x) = 2


x(x−1)
and g (x) = x−3
x−5
, and state
x −25
the restrictions to the domain.

Solution:

The domain of f consists of all real numbers except 5 and −5, and the domain of g
consists of all real numbers except 5. Therefore, the domain of f − g consists of all
real numbers except −5 and 5.

3
Answer: − x+5 , where x ≠ ±5

7.3 Adding and Subtracting Rational Expressions 1140


Chapter 7 Rational Expressions and Equations

KEY TAKEAWAYS

• When adding or subtracting rational expressions with a common


denominator, add or subtract the expressions in the numerator and
write the result over the common denominator.
• To find equivalent rational expressions with a common denominator,
first factor all denominators and determine the least common multiple.
Then multiply numerator and denominator of each term by the
appropriate factor to obtain a common denominator. Finally, add or
subtract the expressions in the numerator and write the result over the
common denominator.
• The restrictions to the domain of a sum or difference of rational
functions consist of the restrictions to the domains of each function.

7.3 Adding and Subtracting Rational Expressions 1141


Chapter 7 Rational Expressions and Equations

TOPIC EXERCISES

Part A: Adding and Subtracting with Common Denominators

Simplify. (Assume all denominators are nonzero.)

3 7
1. x + x

9 10
2. x − x

x 3
3. y − y

4 6
4.
x−3
+ x−3

7 x
5.
2x−1
− 2x−1

8 3x
6.
3x−8
− 3x−8

2 x−11
7.
x−9
+ x−9

y+2 y+3
8.
2y+3
− 2y+3

2x−3 x−4
9.
4x−1
− 4x−1

2x 3x+4 x−2
10.
x−1
− x−1
+ x−1

1 2y−9 13−5y
11.
3y
− 3y
− 3y

−3y+2 y+7 3y+4


12.
5y−10
+ 5y−10
− 5y−10

x 3
13.
(x+1)(x−3)
− (x+1)(x−3)

3x+5 x+6
(2x−1)(x−6) (2x−1)(x−6)
14. −

7.3 Adding and Subtracting Rational Expressions 1142


Chapter 7 Rational Expressions and Equations

x 6
15.
x 2 −36
+ x 2 −36

x 9
16.
x 2 −81
− x 2 −81

x 2 +2 x−22
17.
x 2 +3x−28
+ x 2 +3x−28

x 3−x
18.
2x 2 −x−3
− 2x 2 −x−3

Part B: Adding and Subtracting with Unlike Denominators

Simplify. (Assume all denominators are nonzero.)

1 1
19.
2
+ 3x

1 1
20.
5x 2
− x

1 3
21.
12y 2
+ 10y 3

1 1
22. x − 2y

1
23. y −2

3
24.
y+2
−4

2
25.
x+4
+2

2 1
26. y − y2

3 1
27.
x+1
+ x

1 2
28.
x−1
− x

1 1
29.
x−3
+ x+5

7.3 Adding and Subtracting Rational Expressions 1143


Chapter 7 Rational Expressions and Equations

1 1
30.
x+2
− x−3

x 2
31.
x+1
− x−2

2x−3 x
32.
x+5
− x−3

y+1 y−1
33.
y−1
+ y+1

3y−1 y+4
34.
3y
− y−2

2x−5 2x+5
35.
2x+5
− 2x−5

2 2x+1
36.
2x−1
− 1−2x

3x+4 2
37.
x−8
− 8−x

1 1
38.
y−1
+ 1−y

2x 2 x+15
39.
x 2 −9
+ 9−x 2

x 1 15−x
40.
x+3
+ x−3
− (x+3)(x−3)

2x 1 2(x−1)
41.
3x−1
− 3x+1
+ (3x−1)(3x+1)

(2x+1)(x−5)
4x x 16x−3
42.
2x+1
− x−5
+

x 2 4
43.
3x
+ x−2
+ 3x(x−2)

18(x−2)
(x+6)(x−6)
−2x 3x
44.
x+6
− 6−x

(x+5)(x−7)
x 1 25−7x
45.
x+5
− x−7

7.3 Adding and Subtracting Rational Expressions 1144


Chapter 7 Rational Expressions and Equations

x 2
46.
x 2 −2x−3
+ x−3

1 x2
47.
x+5
− x 2 −25

5x−2 2
48.
x 2 −4
− x−2

1 6x−3
49.
x+1
− x 2 −7x−8

3x 1
50.
9x 2 −16
− 3x+4

2x 1
51.
x 2 −1
+ x 2 +x

x(4x−1) x
52.
2x 2 +7x−4
− 4+x

3x 2 2x
53.
3x 2 +5x−2
− 3x−1

2x 11x+4
54.
x−4
− x 2 −2x−8

x 6x−24
55.
2x+1
+ 2x 2 −7x−4

1 1
56.
x 2 −x−6
+ x 2 −3x−10

x 3
57.
x 2 +4x+3
− x 2 −4x−5

y+1 y
58.
2y 2 +5y−3
− 4y 2 −1

y−1 2
59.
y 2 −25
− y 2 −10y+25

3x 2 +24 12
60.
x 2 −2x−8
− x−4

4x 2 +28 28
61.
x 2 −6x−7
− x−7

7.3 Adding and Subtracting Rational Expressions 1145


Chapter 7 Rational Expressions and Equations

a a2 −9a+18
62.
4−a
+ a2 −13a+36

3a−12 a+2
63.
a2 −8a+16
− 4−a

a2 −14 5
64.
2a2 −7a−4
− 1+2a

1 x 3
65.
x+3
− x 2 −6x+9
+ x 2 −9

3x 2x 23x−10
66. x+7 − x−2
+ x 2 +5x−14

x+3 x−1 x(x+11)


67.
x−1
+ x+2
− x 2 +x−2

−2x 4 4(x+5)
68.
3x+1
− x−2
+ 3x 2 −5x−2

x−1 x+3 3(x+5)


69.
4x−1
− 2x+3
− 8x 2 +10x−3

3x 2 6x 2 −5x−9
70.
2x−3
− 2x+3
− 4x 2 −9

1 1 2
71.
y+1
+ y + y 2 −1

1 1 1
72. y − y+1
+ y−1

−2
73. 5 + 2 −1
−1
74. 6 + 4 −2

75. x −1 + y −1

76. x −2 − y −1

77. (2x − 1) −1 − x −2

78. (x − 4) −1 − (x + 1) −1

7.3 Adding and Subtracting Rational Expressions 1146


Chapter 7 Rational Expressions and Equations

79. 3x 2 (x − 1) −1 − 2x

80. 2(y − 1) − (y − 1)
−2 −1

Part C: Adding and Subtracting Rational Functions

Calculate (f + g) (x) and (f − g) (x) and state the restrictions to the


domain.

1 1
81. f (x) = and g (x) =
3x x−2

1 1
82. f (x) = and g (x) =
x−1 x+5

x 1
83. f (x) = and g (x) =
x−4 4−x

x 1
84. f (x) = and g (x) =
x−5 2x−3

x−1 4
85. f (x) = and g (x) =
x 2 −4 x 2 −6x−16

5
86. f (x) = and g (x) = 3x + 4
x+2

Calculate (f + f ) (x) and state the restrictions to the domain.

1
87. f (x) = x

1
88. f (x) = 2x

x
89. f (x) = 2x−1

1
90. f (x) = x+2

Part D: Discussion Board

1 2 3
91. Explain to a classmate why this is incorrect:
x2
+ x2
= 2x 2
.

7.3 Adding and Subtracting Rational Expressions 1147


Chapter 7 Rational Expressions and Equations

92. Explain to a classmate how to find the common denominator when


adding algebraic expressions. Give an example.

7.3 Adding and Subtracting Rational Expressions 1148


Chapter 7 Rational Expressions and Equations

ANSWERS

10
1: x

x−3
3: y

7−x
5:
2x−1

7: 1

x+1
9:
4x−1

y−1
11: y

1
13:
x+1

1
15:
x−6

x+5
17: x+7

3x+2
19:
6x

5y+18
21:
60y 3

1−2y
23: y

2(x+5)
25:
x+4

4x+1
27:
x(x+1)

2(x+1)
(x−3)(x+5)
29:

7.3 Adding and Subtracting Rational Expressions 1149


Chapter 7 Rational Expressions and Equations

x 2 −4x−2
31:
(x−2)(x+1)

2(y 2 +1)
(y+1)(y−1)
33:

(2x+5)(2x−5)
40x
35: −

3(x+2)
37:
x−8

2x+5
39:
x+3

2x+1
41:
3x+1

x 2 +4x+4
43:
3x(x−2)

x−6
45: x−7

−x 2 +x−5
(x+5)(x−5)
47:

5
49: −
x−8

2x−1
51:
x(x−1)

x(x−4)
53:
(x+2)(3x−1)

x+6
55:
2x+1

x−9
(x−5)(x+3)
57:

y 2 −8y−5
(y+5)(y−5)
59: 2

7.3 Adding and Subtracting Rational Expressions 1150


Chapter 7 Rational Expressions and Equations

4x
61:
x+1

a+5
63:
a−4

6x
65: −
(x+3)(x−3) 2

x−7
67:
x+2

−x−5
69:
4x−1

2y−1
y(y−1)
71:

27
73:
50

x+y
75: xy

(x−1) 2
77:
x 2 (2x−1)

x(x+2)
79:
x−1

81: (f + g) (x) = 3x(x−2) (


− g) (x) = −
2(2x−1) 2(x+1)
; f ;x ≠ 0, 2
3x(x−2)

83: (f + g) (x) = x−4 (


x−1
; f − g) (x) = x+1
x−4
;x ≠4

85: (f
+ g) (x) = (x+2)(x−2)(x−8) (
− g) (x) =
x(x−5) x 2 −13x+16
; f ;
(x+2)(x−2)(x−8)
x ≠ −2, 2, 8

87: (f + f ) (x) = 2
x;x ≠0

89: (f + f ) (x) = 2x
;x ≠ 1
2x−1 2

7.3 Adding and Subtracting Rational Expressions 1151


Chapter 7 Rational Expressions and Equations

7.4 Complex Rational Expressions

LEARNING OBJECTIVES

1. Simplify complex rational expressions by multiplying the numerator by


the reciprocal of the divisor.
2. Simplify complex rational expressions by multiplying numerator and
denominator by the least common denominator (LCD).

Definitions

A complex fraction8 is a fraction where the numerator or denominator consists of


one or more fractions. For example,

Simplifying such a fraction requires us to find an equivalent fraction with integer


numerator and denominator. One way to do this is to divide. Recall that dividing
fractions involves multiplying by the reciprocal of the divisor.

An alternative method for simplifying this complex fraction involves multiplying


both the numerator and denominator by the LCD of all the given fractions. In this
case, the LCD = 4.

8. A fraction where the


numerator or denominator
consists of one or more
fractions.

1152
Chapter 7 Rational Expressions and Equations

A complex rational expression9 is defined as a rational expression that contains


one or more rational expressions in the numerator or denominator or both. For
example,

We simplify a complex rational expression by finding an equivalent fraction where


the numerator and denominator are polynomials. As illustrated above, there are
two methods for simplifying complex rational expressions, and we will outline the
steps for both methods. For the sake of clarity, assume that variable expressions
used as denominators are nonzero.

Method 1: Simplify Using Division

We begin our discussion on simplifying complex rational expressions using division.


Before we can multiply by the reciprocal of the divisor, we must simplify the
numerator and denominator separately. The goal is to first obtain single algebraic
fractions in the numerator and the denominator. The steps for simplifying a
complex algebraic fraction are illustrated in the following example.

1
2
+ 1x
Example 1: Simplify: 1
.
4
− 12
x

Solution:
9. A rational expression where
the numerator or denominator Step 1: Simplify the numerator and denominator. The goal is to obtain a single
consists of one or more
rational expressions.
algebraic fraction divided by another single algebraic fraction. In this example, find

7.4 Complex Rational Expressions 1153


Chapter 7 Rational Expressions and Equations

equivalent terms with a common denominator in both the numerator and


denominator before adding and subtracting.

At this point we have a single algebraic fraction divided by a single algebraic


fraction.

Step 2: Multiply the numerator by the reciprocal of the divisor.

Step 3: Factor all numerators and denominators completely.

Step 4: Cancel all common factors.

7.4 Complex Rational Expressions 1154


Chapter 7 Rational Expressions and Equations

2x
Answer: x−2

1 1
x − x−2
Example 2: Simplify: 4
.
x 2 −2x

Solution:

7.4 Complex Rational Expressions 1155


Chapter 7 Rational Expressions and Equations

Answer: − 12

1− 4x − 21
x2
Example 3: Simplify 15
.
1− 2x −
x2

Solution: The LCD of the rational expressions in both the numerator and
denominator is x 2 . Multiply by the appropriate factors to obtain equivalent terms
with this as the denominator and then subtract.

7.4 Complex Rational Expressions 1156


Chapter 7 Rational Expressions and Equations

We now have a single rational expression divided by another single rational


expression. Next, multiply the numerator by the reciprocal of the divisor and then
factor and cancel.

Answer: x−7
x−5

1
1−
x2
Example 4: Simplify: 1
.
x −1

7.4 Complex Rational Expressions 1157


Chapter 7 Rational Expressions and Equations

Solution:

x+1
Answer: − x

1 1
81

x2
Try this! Simplify: 1
.
9
+ 1x

x−9
Answer: 9x

Video Solution

(click to see video)


Method 2: Simplify Using the LCD

An alternative method for simplifying complex rational expressions involves


clearing the fractions by multiplying the expression by a special form of 1. In this

7.4 Complex Rational Expressions 1158


Chapter 7 Rational Expressions and Equations

method, multiply the numerator and denominator by the least common


denominator (LCD) of all given fractions.

1
2
+ 1x
Example 5: Simplify: 1
.
4
− 12
x

Solution:

Step 1: Determine the LCD of all the fractions in the numerator and denominator.
In this case, the denominators of the given fractions are 2, x , 4, and x 2 . Therefore,
the LCD is 4x 2 .

Step 2: Multiply the numerator and denominator by the LCD. This step should clear
the fractions in both the numerator and denominator.

This leaves us with a single algebraic fraction with a polynomial in the numerator
and in the denominator.

Step 3: Factor the numerator and denominator completely.

7.4 Complex Rational Expressions 1159


Chapter 7 Rational Expressions and Equations

Step 4: Cancel all common factors.

2x
Answer: x−2

Note

This was the same problem that we began this section with, and the results
here are the same. It is worth taking the time to compare the steps involved
using both methods on the same problem.

15
1− 2x −
x2
Example 6: Simplify: 14 5
.
3− x − x2

Solution: Considering all of the denominators, we find that the LCD is x 2 .


Therefore, multiply the numerator and denominator by x 2 :

7.4 Complex Rational Expressions 1160


Chapter 7 Rational Expressions and Equations

At this point, we have a rational expression that can be simplified by factoring and
then canceling the common factors.

x+3
Answer: 3x+1

It is important to point out that multiplying the numerator and denominator by the
same nonzero factor is equivalent to multiplying by 1 and does not change the
problem. Because x 2 = 1, we can multiply the numerator and denominator by x 2
2

x
in the previous example and obtain an equivalent expression.

1 3
x+1
+ x−3
Example 7: Simplify: 2 1
.
x−3
− x+1

7.4 Complex Rational Expressions 1161


Chapter 7 Rational Expressions and Equations

Solution: The LCM of all the denominators is (x + 1) (x − 3). Begin by multiplying


the numerator and denominator by these factors.

4x
Answer: x+5

1
y − 14
Try this! Simplify: 1 1
.

16 y2

4y
Answer: − y+4

Video Solution

(click to see video)

7.4 Complex Rational Expressions 1162


Chapter 7 Rational Expressions and Equations

KEY TAKEAWAYS

• Complex rational expressions can be simplified into equivalent


expressions with a polynomial numerator and polynomial denominator.
• One method of simplifying a complex rational expression requires us to
first write the numerator and denominator as a single algebraic fraction.
Then multiply the numerator by the reciprocal of the divisor and
simplify the result.
• Another method for simplifying a complex rational expression requires
that we multiply it by a special form of 1. Multiply the numerator and
denominator by the LCM of all the denominators as a means to clear the
fractions. After doing this, simplify the remaining rational expression.
• An algebraic fraction is reduced to lowest terms if the numerator and
denominator are polynomials that share no common factors other than
1.

7.4 Complex Rational Expressions 1163


Chapter 7 Rational Expressions and Equations

TOPIC EXERCISES

Part A: Complex Rational Expressions

Simplify. (Assume all denominators are nonzero.)

1
2
1. 5
4

7
8
2. 5
4

10
3
3. 20
9

4
− 21
4. 8
7

2
3
5. 5
6

7
4
6. 14
3

1− 32
7. 5
4
− 13

1
2
−5
8. 1
2
+ 13

1+ 32
9.
1− 14

2− 12
10.
1+ 34

7.4 Complex Rational Expressions 1164


Chapter 7 Rational Expressions and Equations

5x 2
x+1
11. 25x
x+1

7+x
7x
12. x+7
14x 2

3y
x
13.
y2
x−1

5a2
b−1
14.
15a3
(b−1)2

1+ 1x
15.
2− 1x

2
x +1
16.
3− 1x

2
3y
−4
17.
6− 1y

5
y − 12
18. 10−y
y2

1
5
− 1x
19. 1 1

25 x2

1
x + 15
20. 1 1

25 x2

1 1
x −3
21. 1
9
− 12
x

7.4 Complex Rational Expressions 1165


Chapter 7 Rational Expressions and Equations

1
4
+ 1x
22. 1 1
− 16
x2

1
16−
x2
23. 1
x −4

2− 1y
24. 1
1−
4y 2

1
x + 1y
25. 1 1

y2 x2

1
2x
− 43
26. 1
− 16
4x 2 9

2 1

25 2x 2
27. 1 1
5
− 2x

4 1

25 4x 2
28. 1 1
5
+ 4x

1
y − 1x
29.
4− xy2

1
ab
+2
30. 1 1
a+b

1
y + 1x
31. xy

3x
32. 1
3
− 1x

1− 4x − 21
x2
33. 15
1− 2x −
x2

7.4 Complex Rational Expressions 1166


Chapter 7 Rational Expressions and Equations

1− 3x − 4
x2
34. 16
1−
x2

3− 2x1 − 1
2x 2
35.
2− 2x + 1
2x 2

1
2
− 5x + 12
x2
36. 1
2
− 6x + 18
x2

1 4
x −
3x 2
37. 8 16
3− x + 3x 2

3 1
1+ 10x −
10x 2
38. 3 1
5
− 10x
− 12
5x

x−1
39. 5
1+ 4x −
x2

2− 2x5 − 3
x2
40.
4x+3

1
x−3
+ 2x
41. 1 3
x − x−3

1 1
+
4x−5 x2
42. 1 1
+
x2 3x−10

1 4
x+5
+ x−2
43. 2 1
x−2
− x+5

3 2
x−1
− x+3
44. 2 1
x+3
+ x−3

x 2
x+1
− x+3
45. x 1
3x+4
+ x+1

7.4 Complex Rational Expressions 1167


Chapter 7 Rational Expressions and Equations

x 2
x−9
+ x+1
46. x 1
7x−9
− x+1

x 1
3x+2
− x+2
47. x 2
x+2
− x+2

x 1
x−4
+ x+2
48. x 1
3x+4
+ x+2

a3 −8b 3
27
49.
a−2b

27a3 +b 3
ab
50.
3a+b

1 1
+
b3 a3
51. 1 1
b
+ a

1 1

b3 a3
52. 1 1
a − b

x 2 +y 2
xy +2
53.
x 2 −y 2
2xy

x 4y
y +4+ x
54. x 2y
y +3+ x

1
55. 1 +
1+ 12

1
56. 2 −
1+ 13

1
57. 1
1+ 1+x

7.4 Complex Rational Expressions 1168


Chapter 7 Rational Expressions and Equations

x+1
x
58. 1
1− x+1

1− 1x
59.
x− 1x

1
x −x
60. x−1
x2

Part B: Discussion Board Topics

61. Choose a problem from this exercise set and clearly work it out on paper,
explaining each step in words. Scan your page and post it on the discussion
board.

62. Explain why we need to simplify the numerator and denominator to a


single algebraic fraction before multiplying by the reciprocal of the divisor.

63. Two methods for simplifying complex rational expressions have been
presented in this section. Which of the two methods do you feel is more
efficient, and why?

7.4 Complex Rational Expressions 1169


Chapter 7 Rational Expressions and Equations

ANSWERS

2
1:
5

3
3:
2

5: 4/5

7: −6/11

10
9:
3

x
11:
5

3(x−1)
13: xy

x+1
15:
2x−1

2
17: −
3

5x
19:
x+5

3x
21: −
x+3

4x+1
23: − x

xy
25: x−y

2x+5
27:
5x

x−y
29:
4xy−2

x+y
31:
x 2y2

7.4 Complex Rational Expressions 1170


Chapter 7 Rational Expressions and Equations

x−7
33:
x−5

3x+1
35:
2x−1

1
37:
3x−4

x2
39:
x+5

3(x−2)
41: −
2x+3

5x+18
43:
x+12

(x−1)(3x+4)
45:
(x+2)(x+3)

x+1
47:
3x+2

a2 +2ab+4b 2
49:
27

a2 −ab+b 2
51:
a2 b 2

2(x+y)
53: x−y

5
55:
3

x+1
57:
x+2

1
59:
x+1

7.4 Complex Rational Expressions 1171


Chapter 7 Rational Expressions and Equations

7.5 Solving Rational Equations

LEARNING OBJECTIVES

1. Solve rational equations.


2. Solve literal equations, or formulas, involving rational expressions.

Solving Rational Equations

A rational equation10 is an equation containing at least one rational expression.


Rational expressions typically contain a variable in the denominator. For this
reason, we will take care to ensure that the denominator is not 0 by making note of
restrictions and checking our solutions.

Solve rational equations by clearing the fractions by multiplying both sides of the
equation by the least common denominator (LCD).

Example 1: Solve: 5x − 13 = 1x.

Solution: We first make a note that x ≠ 0 and then multiply both sides by the LCD,
3x:

Check your answer by substituting 12 for x to see if you obtain a true statement.
10. An equation containing at least
one rational expression.

1172
Chapter 7 Rational Expressions and Equations

Answer: The solution is 12.

After multiplying both sides of the previous example by the LCD, we were left with a
linear equation to solve. This is not always the case; sometimes we will be left with a
quadratic equation.

1 3
Example 2: Solve: 2 − x(x+1) = x+1.

Solution: In this example, there are two restrictions, x ≠ 0 and x ≠ −1 . Begin by


multiplying both sides by the LCD, x(x + 1) .

7.5 Solving Rational Equations 1173


Chapter 7 Rational Expressions and Equations

After distributing and dividing out the common factors, a quadratic equation
remains. To solve it, rewrite it in standard form, factor, and then set each factor
equal to 0.

Check to see if these values solve the original equation.

Answer: The solutions are −1/2 and 1.

7.5 Solving Rational Equations 1174


Chapter 7 Rational Expressions and Equations

Up to this point, all of the possible solutions have solved the original equation.
However, this may not always be the case. Multiplying both sides of an equation by
variable factors may lead to extraneous solutions11, which are solutions that do
not solve the original equation. A complete list of steps for solving a rational
equation is outlined in the following example.

x 2 5
Example 3: Solve: x+2 + x 2 +5x+6
= x+3
.

Solution:

Step 1: Factor all denominators and determine the LCD.

The LCD is (x + 2) (x + 3).

Step 2: Identify the restrictions. In this case, they are x ≠ −2 and x ≠ −3 .

Step 3: Multiply both sides of the equation by the LCD. Distribute carefully and then
simplify.

11. A solution that does not solve


the original equation.

7.5 Solving Rational Equations 1175


Chapter 7 Rational Expressions and Equations

Step 4: Solve the resulting equation. Here the result is a quadratic equation.
Rewrite it in standard form, factor, and then set each factor equal to 0.

Step 5: Check for extraneous solutions. Always substitute into the original
equation, or the factored equivalent. In this case, choose the factored equivalent to
check:

Here −2 is an extraneous solution and is not included in the solution set. It is


important to note that −2 is a restriction.

7.5 Solving Rational Equations 1176


Chapter 7 Rational Expressions and Equations

Answer: The solution is 4.

If this process produces a solution that happens to be a restriction, then disregard it


as an extraneous solution.

x 3 7x
Try this! Solve: x−5 + x+2 = x 2 −3x−10
.

Answer: −3

Video Solution

(click to see video)

Sometimes all potential solutions are extraneous, in which case we say that there is
no solution to the original equation. In the next two examples, we demonstrate two
ways in which a rational equation can have no solutions.

Example 4: Solve: 3x − 2
x+2
= 1
x+2
.
2 x −4

Solution: To identify the LCD, first factor the denominators.

Multiply both sides by the least common denonominator (LCD), (x + 2) (x − 2),


distributing carefully.

7.5 Solving Rational Equations 1177


Chapter 7 Rational Expressions and Equations

The equation is a contradiction and thus has no solution.

Answer: No solution, ∅

x 4 36
Example 5: Solve: x−4 − x+5 = x 2 +x−20
.

Solution: First, factor the denominators.

multiply by the LCD, (x − 4) (x + 5).


Take note that the restrictions are x ≠ 4 and x ≠ −5 . To clear the fractions,

7.5 Solving Rational Equations 1178


Chapter 7 Rational Expressions and Equations

Both of these values are restrictions of the original equation; hence both are
extraneous.

Answer: No solution, ∅

1 x 4x
Try this! Solve: x+1 + x−3 = x 2 −2x−3
.

Answer: ∅

Video Solution

(click to see video)

It is important to point out that this technique for clearing algebraic fractions only
works for equations. Do not try to clear algebraic fractions when simplifying expressions.
As a reminder, we have

7.5 Solving Rational Equations 1179


Chapter 7 Rational Expressions and Equations

Expressions are to be simplified and equations are to be solved. If we multiply the


expression by the LCD, x (2x + 1) , we obtain another expression that is not
equivalent.

Literal Equations

Literal equations, or formulas, are often rational equations. Hence the techniques
described in this section can be used to solve for particular variables. Assume that
all variable expressions in the denominator are nonzero.

Example 6: Solve for x: z = x−5


y .

Solution: The goal is to isolate x. Assuming that y is nonzero, multiply both sides by
y and then add 5 to both sides.

Answer: x = yz + 5

7.5 Solving Rational Equations 1180


Chapter 7 Rational Expressions and Equations

Example 7: Solve for c: 1c = 1a + 1b.

Solution: In this example, the goal is to isolate c. We begin by multiplying both


sides by the LCD, a ⋅ b ⋅ c, distributing carefully.

On the right side of the equation, factor out c.

Next, divide both sides of the equation by the quantity (b + a).

ab
Answer: c = b+a

y+1
Try this! Solve for y: x = y−1 .

x+1
Answer: y = x−1

7.5 Solving Rational Equations 1181


Chapter 7 Rational Expressions and Equations

Video Solution

(click to see video)

KEY TAKEAWAYS

• Begin solving rational equations by multiplying both sides by the LCD.


The resulting equivalent equation can be solved using the techniques
learned up to this point.
• Multiplying both sides of a rational equation by a variable expression
introduces the possibility of extraneous solutions. Therefore, we must
check the solutions against the set of restrictions. If a solution is a
restriction, then it is not part of the domain and is extraneous.
• When multiplying both sides of an equation by an expression, distribute
carefully and multiply each term by that expression.
• If all of the resulting solutions are extraneous, then the original
equation has no solutions.

7.5 Solving Rational Equations 1182


Chapter 7 Rational Expressions and Equations

TOPIC EXERCISES

Part A: Rational Equations

Solve.

1 1 1
1.
2
+ x = 8

1 1 2
2.
3
− x = 9

1 2 1
3.
3x
− 3
= x

2 1 3
4.
5x
− x = 10

1
5.
2x+1
=5

3
6.
3x−1
+4=5

2x−3 2
7.
x+5
= x+5

5x x−1
8.
2x−1
= 2x−1

5 6
9. x−7 = x−9

5 3
10.
x+5
= x+1

x 6
11.
6
− x =0

5 x
12. x + 5
= −2

x 2
13.
x+12
= x

2x 1
14.
x+5
= 6−x

7.5 Solving Rational Equations 1183


Chapter 7 Rational Expressions and Equations

1 x
15. x + 2x+1
=0

9x 4
16.
3x−1
− x =0

2 48
17. 1 − x = x2

9 5
18. 2 − x = x2

12 12
19. 1 + x = x−2

3x−5 1
20. 1 − =− x
x(3x−4)

x 14
21.
2
= x+3

3x x+1
22.
2
= 3−x

−3x+3
23. 6 = x−1

12 6(4−x)
24.
x−2
=2+ x−2

2x 3(x−1)
25. 2 + =
x−3 x−3

(x−1)(6x−1)
x 1 x
26.
x−1
+ 6x−1
=

12 1 2
27.
x 2 −81
= x+9
− x−9

14 2 3
28.
x 2 −49
= x−7
− x+7

6x 4 3x
29.
x+3
+ x−3
= x 2 −9

3x 17 48
30.
x+2
− x−2
=− x 2 −4

31. x −1 +3=0

7.5 Solving Rational Equations 1184


Chapter 7 Rational Expressions and Equations

32. 4 − y −1 = 0

33. y −2 −4=0

34. 9x −2 −1=0

35. 3(x − 1) −1 + 5 = 0

36. 5 − 2(3x + 1) −1 = 0
2 2
37. 3 + =
x−3 x−3

1 1
38. x = x+1

x x+1
39.
x+1
= x

3x−1 x
40.
3x
= x+3

4x−7 3x−2
41.
x−5
= x−5

x 1
42.
x 2 −9
= x−3

3x+4 2
43.
x−8
− 8−x
=1

1 6
44. x = x(x+3)

3 1 13
45. x = x+1
+ x(x+1)

x 3 9x
46.
x−1
− 4x−1
= (4x−1)(x−1)

1 x 2
47.
x−4
+ x−2
= x 2 −6x+8

x x−1 5
48.
x−5
+ x 2 −11x+30
= x−6

7.5 Solving Rational Equations 1185


Chapter 7 Rational Expressions and Equations

x 6 5
49.
x+1
− 5x 2 +4x−1
=− 5x−1

−8 2(x+2) 1
50.
x 2 −4x−12
+ x 2 +4x−60
= x+2

x 20 4
51.
x+2
− x 2 −x−6
=− x−3

x+7 x−1 4
52.
x−1
+ x+1
= x 2 −1

x−1 x−3 −x+5


53.
x−3
+ x−1
= x−3

x−2 x−5 8−x


54.
x−5
− x−2
= x−5

x+7 81 9
55.
x−2
− x 2 +5x−14
= x+7

x 5x+30
56.
x−6
+1= 36−x 2

2x 4 −7
57.
x+1
− 4x−3
= 4x 2 +x−3

x−5 5 5x
58.
x−10
+ x−5
=− x 2 −15x+50

5 x+1 5
59.
x 2 +5x+4
+ x 2 +3x−4
= x 2 −1

1 x−9 1
60.
x 2 −2x−63
+ x 2 +10x+21
= x 2 −6x−27

4 2(x−2) x+2
61.
x 2 −4
+ x 2 −4x−12
= x 2 −8x+12

x+2 x+2 x−1


62.
x 2 −5x+4
+ x 2 +x−2
= x 2 −2x−8

6x 11x+1 6x
63.
x−1
− 2x 2 −x−1
= 2x+1

8x 4x 1
64.
2x−3
+ 2x 2 −7x+6
= x−2

Part B: Literal Equations

7.5 Solving Rational Equations 1186


Chapter 7 Rational Expressions and Equations

Solve for the indicated variable.

D
65. Solve for r: t= r.

2A
66. Solve for b: h= b
.

I
67. Solve for P: t= Pr
.

C
68. Solve for π : r= .

1 1 1
69. Solve for c: a = b
+ c.

y−y 1
70. Solve for y: m= x−x 1 .

71. Solve for w: P = 2(l + w).

72. Solve for t: A = P(1 + rt).


1
73. Solve for m: s= n+m .

S
74. Solve for S: h= 2πr
− r.
x
75. Solve for x: y= x+2
.

2x+1
76. Solve for x: y= 5x
.

1 1 1
77. Solve for R: R = R1
+ R2
.

1 1 1
78. Solve for S 1 : = S1
+ S2
.
f

Part C: Discussion Board

79. Explain why multiplying both sides of an equation by the LCD sometimes
produces extraneous solutions.

7.5 Solving Rational Equations 1187


Chapter 7 Rational Expressions and Equations

80. Explain the connection between the technique of cross multiplication


and multiplying both sides of a rational equation by the LCD.

81. Explain how we can tell the difference between a rational expression and
a rational equation. How do we treat them differently?

7.5 Solving Rational Equations 1188


Chapter 7 Rational Expressions and Equations

ANSWERS

1: −8/3

3: −1

5: −2/5

7: 5/2

9: −3

11: −6, 6

13: −4, 6

15: −1

17: −6, 8

19: −4, 6

21: −7, 4

23: ∅

25: ∅

27: −39

29: 4/3, 3/2

31: −1/3

33: −1/2, 1/2

35: 2/5

37: ∅

7.5 Solving Rational Equations 1189


Chapter 7 Rational Expressions and Equations

39: −1/2

41: ∅

43: −7

45: 5

47: −1

49: ∅

51: −4

53: 5/3

55: ∅

57: 1/2

59: −6, 4

61: 10

63: 1/3

D
65: r = t

I
67: P = tr

ab
69: c = b−a

P−2l
71: w = 2

1−sn
73: m = s

2y
75: x = 1−y

7.5 Solving Rational Equations 1190


Chapter 7 Rational Expressions and Equations

R1 R2
77: R = R 1 +R 2

7.5 Solving Rational Equations 1191


Chapter 7 Rational Expressions and Equations

7.6 Applications of Rational Equations

LEARNING OBJECTIVES

1. Solve applications involving relationships between real numbers.


2. Solve applications involving uniform motion (distance problems).
3. Solve work-rate applications.

Number Problems

Recall that the reciprocal12 of a nonzero number n is 1/n. For example, the
reciprocal of 5 is 1/5 and 5 ⋅ 1/5 = 1. In this section, the applications will often
involve the key word “reciprocal.” When this is the case, we will see that the
algebraic setup results in a rational equation.

Example 1: A positive integer is 4 less than another. The sum of the reciprocals of
the two positive integers is 10/21. Find the two integers.

Solution: Begin by assigning variables to the unknowns.

Next, use the reciprocals 1n and n−4


1
to translate the sentences into an algebraic
equation.

12. The reciprocal of a nonzero


number n is 1/n.

1192
Chapter 7 Rational Expressions and Equations

We can solve this rational expression by multiplying both sides of the equation by
the least common denominator (LCD). In this case, the LCD is 21n(n − 4).

Solve the resulting quadratic equation.

The question calls for integers and the only integer solution is n = 7. Hence
disregard 6/5. Use the expression n − 4 to find the smaller integer.

7.6 Applications of Rational Equations 1193


Chapter 7 Rational Expressions and Equations

Answer: The two positive integers are 3 and 7. The check is left to the reader.

Example 2: A positive integer is 4 less than another. If the reciprocal of the smaller
integer is subtracted from twice the reciprocal of the larger, then the result is 1/30.
Find the two integers.

Solution:

Set up an algebraic equation.

Solve this rational expression by multiplying both sides by the LCD. The LCD is
30n(n − 4).

7.6 Applications of Rational Equations 1194


Chapter 7 Rational Expressions and Equations

Here we have two viable possibilities for the larger integer. For this reason, we will
we have two solutions to this problem.

As a check, perform the operations indicated in the problem.

7.6 Applications of Rational Equations 1195


Chapter 7 Rational Expressions and Equations

Answer: Two sets of positive integers solve this problem: {6, 10} and {20, 24}.

Try this! The difference between the reciprocals of two consecutive positive odd
integers is 2/15. Find the integers.

Answer: The integers are 3 and 5.

Video Solution

(click to see video)


Uniform Motion Problems

Uniform motion13 problems, also referred to as distance problems, involve the


formula

where the distance, D, is given as the product of the average rate, r, and the time, t,
traveled at that rate. If we divide both sides by the average rate, r, then we obtain
13. Described by the formula the formula
D = rt, where the distance, D,
is given as the product of the
average rate, r, and the time, t,
traveled at that rate.

7.6 Applications of Rational Equations 1196


Chapter 7 Rational Expressions and Equations

For this reason, when the unknown quantity is time, the algebraic setup for
distance problems often results in a rational equation. Similarly, when the
unknown quantity is the rate, the setup also may result in a rational equation.

We begin any uniform motion problem by first organizing our data with a chart.
Use this information to set up an algebraic equation that models the application.

Example 5: Mary spent the first 120 miles of her road trip in traffic. When the
traffic cleared, she was able to drive twice as fast for the remaining 300 miles. If the
total trip took 9 hours, then how fast was she moving in traffic?

Solution: First, identify the unknown quantity and organize the data.

To avoid introducing two more variables for the time column, use the formula
t = Dr. Here the time for each leg of the trip is calculated as follows:

Use these expressions to complete the chart.

7.6 Applications of Rational Equations 1197


Chapter 7 Rational Expressions and Equations

The algebraic setup is defined by the time column. Add the times for each leg of the
trip to obtain a total of 9 hours:

We begin solving this equation by first multiplying both sides by the LCD, 2x.

7.6 Applications of Rational Equations 1198


Chapter 7 Rational Expressions and Equations

Answer: Mary averaged 30 miles per hour in traffic.

Example 6: A passenger train can travel, on average, 20 miles per hour faster than a
freight train. If the passenger train covers 390 miles in the same time it takes the
freight train to cover 270 miles, then how fast is each train?

Solution: First, identify the unknown quantities and organize the data.

Next, organize the given data in a chart.

Use the formula t = Dr to fill in the time column for each train.

7.6 Applications of Rational Equations 1199


Chapter 7 Rational Expressions and Equations

Because the trains travel the same amount of time, finish the algebraic setup by
equating the expressions that represent the times:

Solve this equation by first multiplying both sides by the LCD, x(x + 20) .

Use x + 20 to find the speed of the passenger train.

Answer: The speed of the passenger train is 65 miles per hour and the speed of the
freight train is 45 miles per hour.

7.6 Applications of Rational Equations 1200


Chapter 7 Rational Expressions and Equations

Example 7: Brett lives on the river 8 miles upstream from town. When the current
is 2 miles per hour, he can row his boat downstream to town for supplies and back
in 3 hours. What is his average rowing speed in still water?

Solution:

Rowing downstream, the current increases his speed, and his rate is x + 2 miles per
hour. Rowing upstream, the current decreases his speed, and his rate is x − 2 miles
per hour. Begin by organizing the data in the following chart:

Use the formula t = Dr to fill in the time column for each leg of the trip.

7.6 Applications of Rational Equations 1201


Chapter 7 Rational Expressions and Equations

The algebraic setup is defined by the time column. Add the times for each leg of the
trip to obtain a total of 3 hours:

Solve this equation by first multiplying both sides by the LCD, (x + 2) (x − 2).

Next, solve the resulting quadratic equation.

Use only the positive solution, x = 6 miles per hour.

Answer: His rowing speed is 6 miles per hour.

7.6 Applications of Rational Equations 1202


Chapter 7 Rational Expressions and Equations

Try this! Dwayne drove 18 miles to the airport to pick up his father and then
returned home. On the return trip he was able to drive an average of 15 miles per
hour faster than he did on the trip there. If the total driving time was 1 hour, then
what was his average speed driving to the airport?

Answer: His average speed driving to the airport was 30 miles per hour.

Video Solution

(click to see video)


Work-Rate Problems

The rate at which a task can be performed is called a work rate14. For example, if a
painter can paint a room in 8 hours, then the task is to paint the room, and we can
write

In other words, the painter can complete 18 of the task per hour. If he works for less
than 8 hours, then he will perform a fraction of the task. For example,

Obtain the amount of the task completed by multiplying the work rate by the
amount of time the painter works. Typically, work-rate problems involve people
14. The rate at which a task can be working together to complete tasks. When this is the case, we can organize the data
performed. in a chart, just as we have done with distance problems.

7.6 Applications of Rational Equations 1203


Chapter 7 Rational Expressions and Equations

Suppose an apprentice painter can paint the same room by himself in 10 hours.
Then we say that he can complete 101 of the task per hour. Let t represent the time it
takes both of the painters, working together, to paint the room.

To complete the chart, multiply the work rate by the time for each person. The
portion of the room each can paint adds to a total of 1 task completed. This is
represented by the equation obtained from the first column of the chart:

This setup results in a rational equation that can be solved for t by multiplying both
sides by the LCD, 40.

7.6 Applications of Rational Equations 1204


Chapter 7 Rational Expressions and Equations

Therefore, the two painters, working together, complete the task in 4 49 hours.

In general, we have the following work-rate formula15:

1 1
Here t1
and t2
are the individual work rates and t is the time it takes to complete
one task working together. If we factor out the time, t, and then divide both sides by
t, we obtain an equivalent work-rate formula:

1 1
15. t ⋅ t + t ⋅ t = 1, where t1
1 2 1
and t1 are the individual work
2
rates and t is the time it takes In summary, we have the following equivalent work-rate formulas:
to complete the task working
together.

7.6 Applications of Rational Equations 1205


Chapter 7 Rational Expressions and Equations

Example 3: Working alone, Billy’s dad can complete the yard work in 3 hours. If
Billy helps his dad, then the yard work takes 2 hours. How long would it take Billy
working alone to complete the yard work?

Solution: The given information tells us that Billy’s dad has an individual work rate
of 13 task per hour. If we let x represent the time it takes Billy working alone to
complete the yard work, then Billy’s individual work rate is 1x , and we can write

Working together, they can complete the task in 2 hours. Multiply the individual
work rates by 2 hours to fill in the chart.

7.6 Applications of Rational Equations 1206


Chapter 7 Rational Expressions and Equations

The amount of the task each completes will total 1 completed task. To solve for x,
we first multiply both sides by the LCD, 3x.

Answer: It takes Billy 6 hours to complete the yard work alone.

Of course, the unit of time for the work rate need not always be in hours.

7.6 Applications of Rational Equations 1207


Chapter 7 Rational Expressions and Equations

Example 4: Working together, two construction crews can build a shed in 5 days.
Working separately, the less experienced crew takes twice as long to build a shed
than the more experienced crew. Working separately, how long does it take each
crew to build a shed?

Solution:

Working together, the job is completed in 5 days. This gives the following setup:

The first column in the chart gives us an algebraic equation that models the
problem:

Solve the equation by multiplying both sides by 2x.

7.6 Applications of Rational Equations 1208


Chapter 7 Rational Expressions and Equations

To determine the time it takes the less experienced crew, we use 2x:

Answer: Working separately, the experienced crew takes 7½ days to build a shed,
and the less experienced crew takes 15 days to build a shed.

Try this! Joe’s garden hose fills the pool in 12 hours. His neighbor has a thinner
hose that fills the pool in 15 hours. How long will it take to fill the pool using both
hoses?

Answer: It will take both hoses 6 23 hours to fill the pool.

Video Solution

(click to see video)

7.6 Applications of Rational Equations 1209


Chapter 7 Rational Expressions and Equations

KEY TAKEAWAYS

• In this section, all of the steps outlined for solving general word
problems apply. Look for the new key word “reciprocal,” which
indicates that you should write the quantity in the denominator of a
fraction with numerator 1.
• When solving distance problems where the time element is unknown,
D
use the equivalent form of the uniform motion formula, t = r , to avoid
introducing more variables.
• When solving work-rate problems, multiply the individual work rate by
the time to obtain the portion of the task completed. The sum of the
portions of the task results in the total amount of work completed.

7.6 Applications of Rational Equations 1210


Chapter 7 Rational Expressions and Equations

TOPIC EXERCISES

Part A: Number Problems

Use algebra to solve the following applications.

1. A positive integer is twice another. The sum of the reciprocals of the two
positive integers is 3/10. Find the two integers.

2. A positive integer is twice another. The sum of the reciprocals of the two
positive integers is 3/12. Find the two integers.

3. A positive integer is twice another. The difference of the reciprocals of the


two positive integers is 1/8. Find the two integers.

4. A positive integer is twice another. The difference of the reciprocals of the


two positive integers is 1/18. Find the two integers.

5. A positive integer is 2 less than another. If the sum of the reciprocal of the
smaller and twice the reciprocal of the larger is 5/12, then find the two
integers.

6. A positive integer is 2 more than another. If the sum of the reciprocal of


the smaller and twice the reciprocal of the larger is 17/35, then find the two
integers.

7. The sum of the reciprocals of two consecutive positive even integers is 11/
60. Find the two even integers.

8. The sum of the reciprocals of two consecutive positive odd integers is 16/
63. Find the integers.

9. The difference of the reciprocals of two consecutive positive even integers


is 1/24. Find the two even integers.

10. The difference of the reciprocals of two consecutive positive odd integers
is 2/99. Find the integers.

11. If 3 times the reciprocal of the larger of two consecutive integers is


subtracted from 2 times the reciprocal of the smaller, then the result is 1/2.
Find the two integers.

7.6 Applications of Rational Equations 1211


Chapter 7 Rational Expressions and Equations

12. If 3 times the reciprocal of the smaller of two consecutive integers is


subtracted from 7 times the reciprocal of the larger, then the result is 1/2.
Find the two integers.

13. A positive integer is 5 less than another. If the reciprocal of the smaller
integer is subtracted from 3 times the reciprocal of the larger, then the
result is 1/12. Find the two integers.

14. A positive integer is 6 less than another. If the reciprocal of the smaller
integer is subtracted from 10 times the reciprocal of the larger, then the
result is 3/7. Find the two integers.

Part B: Uniform Motion Problems

Use algebra to solve the following applications.

15. James can jog twice as fast as he can walk. He was able to jog the first 9
miles to his grandmother’s house, but then he tired and walked the
remaining 1.5 miles. If the total trip took 2 hours, then what was his average
jogging speed?

16. On a business trip, an executive traveled 720 miles by jet aircraft and
then another 80 miles by helicopter. If the jet averaged 3 times the speed of
the helicopter and the total trip took 4 hours, then what was the average
speed of the jet?

17. Sally was able to drive an average of 20 miles per hour faster in her car
after the traffic cleared. She drove 23 miles in traffic before it cleared and
then drove another 99 miles. If the total trip took 2 hours, then what was
her average speed in traffic?

18. Harry traveled 15 miles on the bus and then another 72 miles on a train.
If the train was 18 miles per hour faster than the bus and the total trip took
2 hours, then what was the average speed of the train?

19. A bus averages 6 miles per hour faster than a trolley. If the bus travels 90
miles in the same time it takes the trolley to travel 75 miles, then what is the
speed of each?

20. A passenger car averages 16 miles per hour faster than the bus. If the bus
travels 56 miles in the same time it takes the passenger car to travel 84
miles, then what is the speed of each?

7.6 Applications of Rational Equations 1212


Chapter 7 Rational Expressions and Equations

21. A light aircraft travels 2 miles per hour less than twice as fast as a
passenger car. If the passenger car can travel 231 miles in the same time it
takes the aircraft to travel 455 miles, then what is the average speed of
each?

22. Mary can run 1 mile per hour more than twice as fast as Bill can walk. If
Bill can walk 3 miles in the same time it takes Mary to run 7.2 miles, then
what is Bill’s average walking speed?

23. An airplane traveling with a 20-mile-per-hour tailwind covers 270 miles.


On the return trip against the wind, it covers 190 miles in the same amount
of time. What is the speed of the airplane in still air?

24. A jet airliner traveling with a 30-mile-per-hour tailwind covers 525 miles
in the same amount of time it is able to travel 495 miles after the tailwind
eases to 10 miles per hour. What is the speed of the airliner in still air?

25. A boat averages 16 miles per hour in still water. With the current, the
boat can travel 95 miles in the same time it travels 65 miles against it. What
is the speed of the current?

26. A river tour boat averages 7 miles per hour in still water. If the total
24-mile tour downriver and 24 miles back takes 7 hours, then how fast is the
river current?

27. If the river current flows at an average 3 miles per hour, then a tour boat
makes the 9-mile tour downstream with the current and back the 9 miles
against the current in 4 hours. What is the average speed of the boat in still
water?

28. Jane rowed her canoe against a 1-mile-per-hour current upstream 12


miles and then returned the 12 miles back downstream. If the total trip took
5 hours, then at what speed can Jane row in still water?

29. Jose drove 15 miles to pick up his sister and then returned home. On the
return trip, he was able to average 15 miles per hour faster than he did on
the trip to pick her up. If the total trip took 1 hour, then what was Jose’s
average speed on the return trip?

30. Barry drove the 24 miles to town and then back in 1 hour. On the return
trip, he was able to average 14 miles per hour faster than he averaged on the
trip to town. What was his average speed on the trip to town?

7.6 Applications of Rational Equations 1213


Chapter 7 Rational Expressions and Equations

31. Jerry paddled his kayak upstream against a 1-mile-per-hour current for
12 miles. The return trip downstream with the 1-mile-per-hour current took
1 hour less time. How fast can Jerry paddle the kayak in still water?

32. It takes a light aircraft 1 hour more time to fly 360 miles against a
30-mile-per-hour headwind than it does to fly the same distance with it.
What is the speed of the aircraft in calm air?

Part C: Work-Rate Problems

Use algebra to solve the following applications.

33. James can paint the office by himself in 7 hours. Manny paints the office
in 10 hours. How long will it take them to paint the office working together?

34. Barry can lay a brick driveway by himself in 12 hours. Robert does the
same job in 10 hours. How long will it take them to lay the brick driveway
working together?

35. Jerry can detail a car by himself in 50 minutes. Sally does the same job in
1 hour. How long will it take them to detail a car working together?

36. Jose can build a small shed by himself in 26 hours. Alex builds the same
small shed in 2 days. How long would it take them to build the shed working
together?

37. Allison can complete a sales route by herself in 6 hours. Working with an
associate, she completes the route in 4 hours. How long would it take her
associate to complete the route by herself?

38. James can prepare and paint a house by himself in 5 days. Working with
his brother, Bryan, they can do it in 3 days. How long would it take Bryan to
prepare and paint the house by himself?

39. Joe can assemble a computer by himself in 1 hour. Working with an


assistant, he can assemble a computer in 40 minutes. How long would it take
his assistant to assemble a computer working alone?

40. The teacher’s assistant can grade class homework assignments by herself
in 1 hour. If the teacher helps, then the grading can be completed in 20

7.6 Applications of Rational Equations 1214


Chapter 7 Rational Expressions and Equations

minutes. How long would it take the teacher to grade the papers working
alone?

41. A larger pipe fills a water tank twice as fast as a smaller pipe. When both
pipes are used, they fill the tank in 5 hours. If the larger pipe is left off, then
how long would it take the smaller pipe to fill the tank?

42. A newer printer can print twice as fast as an older printer. If both
printers working together can print a batch of flyers in 45 minutes, then
how long would it take the newer printer to print the batch working alone?

43. Working alone, Henry takes 9 hours longer than Mary to clean the
carpets in the entire office. Working together, they clean the carpets in 6
hours. How long would it take Mary to clean the office carpets if Henry were
not there to help?

44. Working alone, Monique takes 4 hours longer than Audrey to record the
inventory of the entire shop. Working together, they take inventory in 1.5
hours. How long would it take Audrey to record the inventory working
alone?

45. Jerry can lay a tile floor in 3 hours less time than Jake. If they work
together, the floor takes 2 hours. How long would it take Jerry to lay the
floor by himself?

46. Jeremy can build a model airplane in 5 hours less time than his brother.
Working together, they need 6 hours to build the plane. How long would it
take Jeremy to build the model airplane working alone?

47. Harry can paint a shed by himself in 6 hours. Jeremy can paint the same
shed by himself in 8 hours. How long will it take them to paint two sheds
working together?

48. Joe assembles a computer by himself in 1 hour. Working with an


assistant, he can assemble 10 computers in 6 hours. How long would it take
his assistant to assemble 1 computer working alone?

49. Jerry can lay a tile floor in 3 hours, and his assistant can do the same job
in 4 hours. If Jerry starts the job and his assistant joins him 1 hour later,
then how long will it take to lay the floor?

7.6 Applications of Rational Equations 1215


Chapter 7 Rational Expressions and Equations

50. Working alone, Monique takes 6 hours to record the inventory of the
entire shop, while it takes Audrey only 4 hours to do the same job. How long
will it take them working together if Monique leaves 2 hours early?

7.6 Applications of Rational Equations 1216


Chapter 7 Rational Expressions and Equations

ANSWERS

1: {5, 10}

3: {4, 8}

5: {6, 8}

7: {10, 12}

9: {6, 8}

11: {1, 2} or {−4, −3}

13: {4, 9} or {15, 20}

15: 6 miles per hour

17: 46 miles per hour

19: Trolley: 30 miles per hour; bus: 36 miles per hour

21: Passenger car: 66 miles per hour; aircraft: 130 miles per hour

23: 115 miles per hour

25: 3 miles per hour

27: 6 miles per hour

29: 40 miles per hour

31: 5 miles per hour

33: 4 2
17
hours

35: 27 3
11
minutes

37: 12 hours

7.6 Applications of Rational Equations 1217


Chapter 7 Rational Expressions and Equations

39: 2 hours

41: 15 hours

43: 9 hours

45: 3 hours

47: 6 6
7
hours

49: 2 1
7
hours

7.6 Applications of Rational Equations 1218


Chapter 7 Rational Expressions and Equations

7.7 Variation

LEARNING OBJECTIVES

1. Solve applications involving direct variation.


2. Solve applications involving inverse variation.
3. Solve applications involving joint variation.

Direct Variation

Consider a freight train moving at a constant speed of 30 miles per hour. The
equation that expresses the distance traveled at that speed in terms of time is given
by

After 1 hour the train has traveled 30 miles, after 2 hours the train has traveled 60
miles, and so on. We can construct a chart and graph this relation.

1219
Chapter 7 Rational Expressions and Equations

In this example, we can see that the distance varies over time as the product of the
constant rate, 30 miles per hour, and the variable, t. This relationship is described
as direct variation16 and 30 is called the variation constant. In addition, if we
divide both sides of D = 30t by t we have

In this form, it is reasonable to say that D is proportional to t, where 30 is the


constant of proportionality. In general, we have

Key words Translation

“y varies directly as x”

y = kx

“y is directly proportional17 to x”
16. Describes two quantities x and
y that are constant multiples of
each other: y = kx .

17. Used when referring to direct


variation.

7.7 Variation 1220


Chapter 7 Rational Expressions and Equations

Key words Translation

“y is proportional to x”

Here k is nonzero and is called the constant of variation18 or the constant of


proportionality19.

Example 1: The circumference of a circle is directly proportional to its diameter,


and the constant of proportionality is π. If the circumference is measured to be 20
inches, then what is the radius of the circle?

Solution:

Use the fact that “the circumference is directly proportional to the diameter” to
write an equation that relates the two variables.

We are given that “the constant of proportionality is π,” or k = π. Therefore, we


write

18. The nonzero multiple k, when Now use this formula to find d when the circumference is 20 inches.
quantities vary directly or
inversely.

19. Used when referring to the


constant of variation.

7.7 Variation 1221


Chapter 7 Rational Expressions and Equations

The radius of the circle, r, is one-half of its diameter.

Answer: The radius is 10


π inches, or approximately 3.18 inches.

Typically, we will not be given the constant of variation. Instead, we will be given
information from which it can be determined.

Example 2: An object’s weight on earth varies directly to its weight on the moon. If
a man weighs 180 pounds on earth, then he will weigh 30 pounds on the moon. Set
up an algebraic equation that expresses the weight on earth in terms of the weight
on the moon and use it to determine the weight of a woman on the moon if she
weighs 120 pounds on earth.

Solution:

7.7 Variation 1222


Chapter 7 Rational Expressions and Equations

We are given that the “weight on earth varies directly to the weight on the moon.”

To find the constant of variation k, use the given information. A 180-pound man on
earth weighs 30 pounds on the moon, or y = 180 when x = 30 .

Solve for k.

Next, set up a formula that models the given information.

This implies that a person’s weight on earth is 6 times her weight on the moon. To
answer the question, use the woman’s weight on earth, y = 120 pounds, and solve
for x.

7.7 Variation 1223


Chapter 7 Rational Expressions and Equations

Answer: The woman weighs 20 pounds on the moon.

Inverse Variation

Next, consider the relationship between time and rate,

If we wish to travel a fixed distance, then we can determine the average speed
required to travel that distance in a given amount of time. For example, if we wish
to drive 240 miles in 4 hours, we can determine the required average speed as
follows:

The average speed required to drive 240 miles in 4 hours is 60 miles per hour. If we
wish to drive the 240 miles in 5 hours, then determine the required speed using a
similar equation:

In this case, we would only have to average 48 miles per hour. We can make a chart
and view this relationship on a graph.

7.7 Variation 1224


Chapter 7 Rational Expressions and Equations

This is an example of an inverse relationship. We say that r is inversely proportional


to the time t, where 240 is the constant of proportionality. In general, we have

Key words Translation

“y varies inversely20 as x”

k
y= x

“y is inversely proportional21 to x”

Again, k is nonzero and is called the constant of variation or the constant of


proportionality.

Example 3: If y varies inversely as x and y = 5 when x = 2 , then find the constant


20. Describes two quantities x and of proportionality and an equation that relates the two variables.
y, where one variable is
directly proportional to the
reciprocal of the other:
Solution: If we let k represent the constant of proportionality, then the statement
y= k
x .
“y varies inversely as x” can be written as follows:
21. Used when referring to inverse
variation.

7.7 Variation 1225


Chapter 7 Rational Expressions and Equations

Use the given information, y = 5 when x = 2 , to find k.

Solve for k.

Therefore, the formula that models the problem is

Answer: The constant of proportionality is 10, and the equation is y = 10


x .

Example 4: The weight of an object varies inversely as the square of its distance
from the center of earth. If an object weighs 100 pounds on the surface of earth
(approximately 4,000 miles from the center), then how much will it weigh at 1,000
miles above earth’s surface?

Solution:

7.7 Variation 1226


Chapter 7 Rational Expressions and Equations

Since “w varies inversely as the square of d,” we can write

Use the given information to find k. An object weighs 100 pounds on the surface of
earth, approximately 4,000 miles from the center. In other words, w = 100 when d =
4,000:

Solve for k.

Therefore, we can model the problem with the following formula:

To use the formula to find the weight, we need the distance from the center of
earth. Since the object is 1,000 miles above the surface, find the distance from the
center of earth by adding 4,000 miles:

To answer the question, use the formula with d = 5,000.

7.7 Variation 1227


Chapter 7 Rational Expressions and Equations

Answer: The object will weigh 64 pounds at a distance 1,000 miles above the surface
of earth.

Joint Variation

Lastly, we define relationships between multiple variables. In general, we have

Vocabulary Translation

“y varies jointly22 as x and z”

y = kxz

“y is jointly proportional23 to x and z”

Here k is nonzero and is called the constant of variation or the constant of


proportionality.

Example 5: The area of an ellipse varies jointly as a, half of the ellipse’s major axis,
and b, half of the ellipse’s minor axis. If the area of an ellipse is 300π cm2 , where
22. Describes a quantity y that a = 10 cm and b = 30 cm, then what is the constant of proportionality? Give a
varies directly as the product
of two other quantities x and z: formula for the area of an ellipse.
y = kxz .
23. Used when referring to joint
variation.

7.7 Variation 1228


Chapter 7 Rational Expressions and Equations

Solution: If we let A represent the area of an ellipse, then we can use the statement
“area varies jointly as a and b” to write

To find the constant of variation, k, use the fact that the area is 300π when a = 10
and b = 30.

Therefore, the formula for the area of an ellipse is

Answer: The constant of proportionality is π, and the formula for the area is
A = abπ .

Try this! Given that y varies directly as the square of x and inversely to z, where y =
2 when x = 3 and z = 27, find y when x = 2 and z = 16.

Answer: 3/2

7.7 Variation 1229


Chapter 7 Rational Expressions and Equations

Video Solution

(click to see video)

KEY TAKEAWAY

• The setup of variation problems usually requires multiple steps. First,


identify the key words to set up an equation and then use the given
information to find the constant of variation k. After determining the
constant of variation, write a formula that models the problem. Once a
formula is found, use it to answer the question.

7.7 Variation 1230


Chapter 7 Rational Expressions and Equations

TOPIC EXERCISES

Part A: Variation Problems

Translate the following sentences into a mathematical formula.

1. The distance, D, an automobile can travel is directly proportional to the


time, t, that it travels at a constant speed.

2. The extension of a hanging spring, d, is directly proportional to the


weight, w, attached to it.

3. An automobile’s breaking distance, d, is directly proportional to the


square of the automobile’s speed, v.

4. The volume, V, of a sphere varies directly as the cube of its radius, r.

5. The volume, V, of a given mass of gas is inversely proportional to the


pressure, p, exerted on it.

6. The intensity, I, of light from a light source is inversely proportional to


the square of the distance, d, from the source.

7. Every particle of matter in the universe attracts every other particle with
a force, F, that is directly proportional to the product of the masses, m 1 and
m 2 , of the particles and inversely proportional to the square of the
distance, d, between them.

8. Simple interest, I, is jointly proportional to the annual interest rate, r, and


the time, t, in years a fixed amount of money is invested.

9. The period, T, of a pendulum is directly proportional to the square root of


its length, L.

10. The time, t, it takes an object to fall is directly proportional to the square
root of the distance, d, it falls.

Construct a mathematical model given the following.

11. y varies directly as x, and y = 30 when x = 6.

7.7 Variation 1231


Chapter 7 Rational Expressions and Equations

12. y varies directly as x, and y = 52 when x = 4.

13. y is directly proportional to x, and y = 12 when x = 3.

14. y is directly proportional to x, and y = 120 when x = 20.

15. y varies directly as x, and y = 14 when x = 10.

16. y varies directly as x, and y = 2 when x = 8.

17. y varies inversely as x, and y = 5 when x = 7.

18. y varies inversely as x, and y = 12 when x = 2.

19. y is inversely proportional to x, and y = 3 when x = 9.

20. y is inversely proportional to x, and y = 21 when x = 3.

21. y varies inversely as x, and y = 2 when x = 1/8.

22. y varies inversely as x, and y = 3/2 when x = 1/9.

23. y varies jointly as x and z, where y = 8 when x = 4 and z = 1/2.

24. y varies jointly as x and z, where y = 24 when x = 1/3 and z = 9.

25. y is jointly proportional to x and z, where y = 2 when x = 1 and z = 3.

26. y is jointly proportional to x and z, where y = 15 when x = 3 and z = 7.

27. y varies jointly as x and z, where y = 2/3 when x = 1/2 and z = 12.

28. y varies jointly as x and z, where y = 5 when x = 3/2 and z = 2/9.

29. y varies directly as the square of x, where y = 45 when x = 3.

30. y varies directly as the square of x, where y = 3 when x = 1/2.

31. y is inversely proportional to the square of x, where y = 27 when x = 1/3.

7.7 Variation 1232


Chapter 7 Rational Expressions and Equations

32. y is inversely proportional to the square of x, where y = 9 when x = 2/3.

33. y varies jointly as x and the square of z, where y = 54 when x = 2 and z = 3.

34. y varies jointly as x and the square of z, where y = 6 when x = 1/4 and z =
2/3.

35. y varies jointly as x and z and inversely as the square of w, where y = 30


when x = 8, z = 3, and w = 2.

36. y varies jointly as x and z and inversely as the square of w, where y = 5


when x = 1, z = 3, and w = 1/2.

37. y varies directly as the square root of x and inversely as z, where y = 12


when x = 9 and z = 5.

38. y varies directly as the square root of x and inversely as the square of z,
where y = 15 when x = 25 and z = 2.

39. y varies directly as the square of x and inversely as z and the square of w,
where y = 14 when x = 4, w = 2, and z = 2.

40. y varies directly as the square root of x and inversely as z and the square
of w, where y = 27 when x = 9, w = 1/2, and z = 4.

Part B: Variation Problems

Applications involving variation.

41. Revenue in dollars is directly proportional to the number of branded


sweat shirts sold. If the revenue earned from selling 25 sweat shirts is
$318.75, then determine the revenue if 30 sweat shirts are sold.

42. The sales tax on the purchase of a new car varies directly as the price of
the car. If an $18,000 new car is purchased, then the sales tax is $1,350. How
much sales tax is charged if the new car is priced at $22,000?

43. The price of a share of common stock in a company is directly


proportional to the earnings per share (EPS) of the previous 12 months. If
the price of a share of common stock in a company is $22.55 and the EPS is

7.7 Variation 1233


Chapter 7 Rational Expressions and Equations

published to be $1.10, then determine the value of the stock if the EPS
increases by $0.20.

44. The distance traveled on a road trip varies directly with the time spent
on the road. If a 126-mile trip can be made in 3 hours, then what distance
can be traveled in 4 hours?

45. The circumference of a circle is directly proportional to its radius. If the


circumference of a circle with radius 7 centimeters is measured as 14π
centimeters, then find the constant of proportionality.

46. The area of circle varies directly as the square of its radius. If the area of
a circle with radius 7 centimeters is determined to be 49π square
centimeters, then find the constant of proportionality.

47. The surface area of a sphere varies directly as the square of its radius.
When the radius of a sphere measures 2 meters, the surface area measures
16π square meters. Find the surface area of a sphere with radius 3 meters.

48. The volume of a sphere varies directly as the cube of its radius. When the
radius of a sphere measures 3 meters, the volume is 36π cubic meters. Find
the volume of a sphere with radius 1 meter.

49. With a fixed height, the volume of a cone is directly proportional to the
square of the radius at the base. When the radius at the base measures 10
centimeters, the volume is 200 cubic centimeters. Determine the volume of
the cone if the radius of the base is halved.

50. The distance, d, an object in free fall drops varies directly with the square
of the time, t, that it has been falling. If an object in free fall drops 36 feet in
1.5 seconds, then how far will it have fallen in 3 seconds?

Hooke’s law suggests that the extension of a hanging spring is directly proportional
to the weight attached to it. The constant of variation is called the spring constant.

7.7 Variation 1234


Chapter 7 Rational Expressions and Equations

Figure 7.1
Robert Hooke
(1635–1703)

51. If a hanging spring is stretched 5 inches when a 20‑pound weight is


attached to it, then determine its spring constant.

52. If a hanging spring is stretched 3 centimeters when a 2-kilogram weight


is attached to it, then determine the spring constant.

53. If a hanging spring is stretched 3 inches when a 2‑pound weight is


attached, then how far will it stretch with a 5-pound weight attached?

54. If a hanging spring is stretched 6 centimeters when a 4-kilogram weight


is attached to it, then how far will it stretch with a 2-kilogram weight
attached?

The breaking distance of an automobile is directly proportional to the square of its


speed.

55. If it takes 36 feet to stop a particular automobile moving at a speed of 30


miles per hour, then how much breaking distance is required if the speed is
35 miles per hour?

56. After an accident, it was determined that it took a driver 80 feet to stop
his car. In an experiment under similar conditions, it takes 45 feet to stop
the car moving at a speed of 30 miles per hour. Estimate how fast the driver
was moving before the accident.

Boyle’s law states that if the temperature remains constant, the volume, V, of a given
mass of gas is inversely proportional to the pressure, p, exerted on it.

7.7 Variation 1235


Chapter 7 Rational Expressions and Equations

Figure 7.2
Robert Boyle
(1627–1691)

Portrait of Robert
Boyle, from
http://commons.wikim
edia.org/wiki/
File:Robert_boyle.jpg.

57. A balloon is filled to a volume of 216 cubic inches on a diving boat under
1 atmosphere of pressure. If the balloon is taken underwater approximately
33 feet, where the pressure measures 2 atmospheres, then what is the
volume of the balloon?

58. If a balloon is filled to 216 cubic inches under a pressure of 3


atmospheres at a depth of 66 feet, then what would the volume be at the
surface, where the pressure is 1 atmosphere?

59. To balance a seesaw, the distance from the fulcrum that a person must
sit is inversely proportional to his weight. If a 72-pound boy is sitting 3 feet
from the fulcrum, then how far from the fulcrum must a 54-pound boy sit to
balance the seesaw?

60. The current, I, in an electrical conductor is inversely proportional to its


resistance, R. If the current is 1/4 ampere when the resistance is 100 ohms,
then what is the current when the resistance is 150 ohms?

61. The number of men, represented by y, needed to lay a cobblestone


driveway is directly proportional to the area, A, of the driveway and
inversely proportional to the amount of time, t, allowed to complete the job.
Typically, 3 men can lay 1,200 square feet of cobblestone in 4 hours. How

7.7 Variation 1236


Chapter 7 Rational Expressions and Equations

many men will be required to lay 2,400 square feet of cobblestone given 6
hours?

62. The volume of a right circular cylinder varies jointly as the square of its
radius and its height. A right circular cylinder with a 3-centimeter radius
and a height of 4 centimeters has a volume of 36π cubic centimeters. Find a
formula for the volume of a right circular cylinder in terms of its radius and
height.

63. The period, T, of a pendulum is directly proportional to the square root


of its length, L. If the length of a pendulum is 1 meter, then the period is
approximately 2 seconds. Approximate the period of a pendulum that is 0.5
meter in length.

64. The time, t, it takes an object to fall is directly proportional to the square
root of the distance, d, it falls. An object dropped from 4 feet will take 1/2
second to hit the ground. How long will it take an object dropped from 16
feet to hit the ground?

Newton’s universal law of gravitation states that every particle of matter in the
universe attracts every other particle with a force, F, that is directly proportional to
the product of the masses, m 1 and m 2 , of the particles and inversely proportional
to the square of the distance, d, between them. The constant of proportionality is
called the gravitational constant.

Figure 7.3
Sir Isaac Newton
(1643–1724)

7.7 Variation 1237


Chapter 7 Rational Expressions and Equations

Source: Portrait of
Isaac Newton by Sir
Godfrey Kneller, from
http://commons.wikim
edia.org/wiki/
File:GodfreyKneller-
IsaacNewton-1689.jpg.

65. If two objects with masses 50 kilograms and 100 kilograms are 1/2 meter
apart, then they produce approximately 1.34 × 10 −6 newtons (N) of
force. Calculate the gravitational constant.

66. Use the gravitational constant from the previous exercise to write a
formula that approximates the force, F, in newtons between two masses m 1
and m 2 , expressed in kilograms, given the distance d between them in
meters.

67. Calculate the force in newtons between earth and the moon, given that
the mass of the moon is approximately 7.3 × 10 22 kilograms, the mass of
24
earth is approximately 6.0× 10 kilograms, and the distance between
11
them is on average 1.5 × 10 meters.

68. Calculate the force in newtons between earth and the sun, given that the
mass of the sun is approximately 2.0 × 10 30 kilograms, the mass of earth
24
is approximately 6.0
× 10 kilograms, and the distance between them is
8
on average 3.85 × 10 meters.

69. If y varies directly as the square of x, then how does y change if x is


doubled?

70. If y varies inversely as square of t, then how does y change if t is doubled?

71. If y varies directly as the square of x and inversely as the square of t, then
how does y change if both x and t are doubled?

7.7 Variation 1238


Chapter 7 Rational Expressions and Equations

ANSWERS

1: D = kt

3: d = kv 2
k
5: V = p

m 1 ⋅m 2
7: F =k
d2

⎯⎯

9: T = k√ L

11: y = 5x

13: y = 4x

15: y = 7
5
x

35
17: y = x

27
19: y = x

1
21: y = 4x

23: y = 4xz

25: y = 2
3
xz

27: y = 1
9
xz

29: y = 5x 2
3
31: y = x2

33: y = 3xz 2

7.7 Variation 1239


Chapter 7 Rational Expressions and Equations

5xz
35: y = w2

20√x
37: y = z

7x 2
39: y = w2 z

41: $382.50

43: $26.65

45: 2π

47: 36π square meters

49: 50 cubic centimeters

51: 1/4

53: 7.5 inches

55: 49 feet

57: 108 cubic inches

59: 4 feet

61: 4 men

63: 1.4 seconds

65: 6.7 × 10 −11 N m 2 /kg 2

67: 1.98 × 10 20 N

69: y changes by a factor of 4

71: y remains unchanged

7.7 Variation 1240


Chapter 7 Rational Expressions and Equations

7.8 Review Exercises and Sample Exam

1241
Chapter 7 Rational Expressions and Equations

REVIEW EXERCISES

Simplifying Rational Expressions

Evaluate for the given set of x-values.

25
1. ; {−5, 0, 5}
2x 2

x−4
2. ; {1/2, 2, 4}
2x−1

1
3. ; {−3, 0, 3}
x 2 +9

x+3
4. ; {−3, 0, 3}
x 2 −9

State the restrictions to the domain.

5
5. x

1
6.
x(3x+1)

x+2
7.
x 2 −25

x−1
8.
(x−1)(2x−3)

State the restrictions and simplify.

x−8
9.
x 2 −64

3x 2 +9x
10.
2x 3 −18x

x 2 −5x−24
11.
x 2 −3x−40

2x 2 +9x−5
12.
4x 2 −1

7.8 Review Exercises and Sample Exam 1242


Chapter 7 Rational Expressions and Equations

x 2 −144
13.
12−x

8x 2 −10x−3
14.
9−4x 2

x−3
15. Given f (x) = , find f (−3) , f (0), and f (3).
x 2 +9

x 2 −2x−24
16. Simplify g (x) = and state the restrictions.
2x 2 −9x−18

Multiplying and Dividing Rational Expressions

Multiply. (Assume all denominators are nonzero.)

3x 5 x−3
17.
x−3
⋅ 9x 2

12y 2 (2y−1)
(2y−1)
18. ⋅ 3y
y3

3x 2 x 2 −4x+4
19.
x−2
⋅ 5x 3

x 2 −8x+15 12x 2
20.
9x 5
⋅ x−3

x 2 −36 2x 2 +10x
21.
x 2 −x−30
⋅ x 2 +5x−6

9x 2 +11x+2 9x−2
22.
4−81x 2

(x+1) 2

Divide. (Assume all denominators are nonzero.)

9x 2 −25 3x+5
23.
5x 3
÷ 15x 4

4x 2 2x
24.
4x 2 −1
÷ 2x−1

3x 2 −13x−10 9x 2 +12x+4
25.
x 2 −x−20
÷ x 2 +8x+16

7.8 Review Exercises and Sample Exam 1243


Chapter 7 Rational Expressions and Equations

2x 2 +xy−y 2 4x 2 −y 2
26.
x 2 +2xy+y 2
÷ 3x 2 +2xy−y 2

27.
2x 2 −6x−20
8x 2 +17x+2
÷ (8x 2 − 39x − 5)

28.
12x 2 −27x 4
15x 4 +10x 3
÷ (3x 2 + x − 2)

25y 2 −1 10y 2
5y 4 (y−2)
1
⋅ ÷
(y−2)
29.
5y−1 2

10x 4 5x 2 x−1
30.
1−36x 2
÷ 6x 2 −7x+1
⋅ 2x

, calculate (f ⋅ g) (x)
16x 2 −9 x 2 +3x−10
31. Given f (x) = and g (x) =
x+5 4x 2 +5x−6
and state the restrictions.

, calculate (f /g)
x+7 x 2 −49
32. Given f (x) = and g (x) = (x) and
5x−1 25x 2 −5x
state the restrictions.

Adding and Subtracting Rational Expressions

Simplify. (Assume all denominators are nonzero.)

5x 3
33. y − y

x 3
34.
x 2 −x−6
− x 2 −x−6

2x 1
35.
2x+1
+ x−5

3 1−2x
36. x−7 + x2

7x 2
37.
4x 2 −9x+2
− x−2

5 20−9x
38.
x−5
+ 2x 2 −15x+25

7.8 Review Exercises and Sample Exam 1244


Chapter 7 Rational Expressions and Equations

x 2 5(x−3)
39.
x−5
− x−3
− x 2 −8x+15

3x x−4 12(2−x)
40.
2x−1
− x+4
+ 2x 2 +7x−4

1 1
41.
x 2 +8x−9
− x 2 +11x+18

4 3
42.
x 2 +13x+36
+ x 2 +6x−27

y+1 1 2y
43.
y+2
− 2−y
+ y 2 −4

1 1 2
44. y − 1−y
− y 2 −1

, calculate (f + g) (x) and


x+1 x
45. Given f (x) = and g (x) =
2x−5 x+1
state the restrictions.

, calculate (f − g) (x) and


x+1 2
46. Given f (x) = and g (x) =
3x x−8
state the restrictions.

Complex Fractions

Simplify.

4− 2x
47. 2x−1
3x

1
3
− 3y1
48. 1
5
− 5y1

1
6
+ 1x
49. 1 1

36 x2

1 1
100

x2
50. 1 1
10
− x

7.8 Review Exercises and Sample Exam 1245


Chapter 7 Rational Expressions and Equations

x 2
x+3
− x+1
51. x 1
x+4
+ x+3

3 1
x − x−5
52. 5
x+2
− 2x

35
1− 12
x + x2
53. 25
1−
x2

2− 15
x +
25
x2
54.
2x−5

Solving Rational Equations

Solve.

6 2
55.
x−6
= 2x−1

x x+2
56.
x−6
= x−2

1 2 1
57.
3x
− 9
= x

2 3 1
58.
x−5
+ 5
= x−5

x 4 10
59.
x−5
+ x+5
=− x 2 −25

2 12 2−3x 2
60. x − 2x+3
= 2x 2 +3x

x+1 x−6
61.
2(x−2)
+ x =1

5x+2 x
62.
x+1
− x+4
=4

x 1 4x−7
63.
x+5
+ x−4
= x 2 +x−20

7.8 Review Exercises and Sample Exam 1246


Chapter 7 Rational Expressions and Equations

2 x 2(3−4x)
64.
3x−1
+ 2x+1
= 6x 2 +x−1

x 1 2x
65.
x−1
+ x+1
= x 2 −1

2x 1 4−7x
66.
x+5
− 2x−3
= 2x 2 +7x−15

1 1 1
67. Solve for a: a = b
+ c.

2y−1
68. Solve for y: x= 3y
.

Applications of Rational Equations

Use algebra to solve the following applications.

69. A positive integer is twice another. The sum of the reciprocals of the two
positive integers is 1/4. Find the two integers.

70. If the reciprocal of the smaller of two consecutive integers is subtracted


from three times the reciprocal of the larger, the result is 3/10. Find the
integers.

71. Mary can jog, on average, 2 miles per hour faster than her husband,
James. James can jog 6.6 miles in the same amount of time it takes Mary to
jog 9 miles. How fast, on average, can Mary jog?

72. Billy traveled 140 miles to visit his grandmother on the bus and then
drove the 140 miles back in a rental car. The bus averages 14 miles per hour
slower than the car. If the total time spent traveling was 4.5 hours, then
what was the average speed of the bus?

73. Jerry takes twice as long as Manny to assemble a skateboard. If they work
together, they can assemble a skateboard in 6 minutes. How long would it
take Manny to assemble the skateboard without Jerry’s help?

74. Working alone, Joe completes the yard work in 30 minutes. It takes Mike
45 minutes to complete work on the same yard. How long would it take them
working together?

7.8 Review Exercises and Sample Exam 1247


Chapter 7 Rational Expressions and Equations

Variation

Construct a mathematical model given the following.

75. y varies directly with x, and y = 12 when x = 4.

76. y varies inversely as x, and y = 2 when x = 5.

77. y is jointly proportional to x and z, where y = 36 when x = 3 and z = 4.

78. y is directly proportional to the square of x and inversely proportional to


z, where y = 20 when x = 2 and z = 5.

79. The distance an object in free fall drops varies directly with the square of
the time that it has been falling. It is observed that an object falls 16 feet in 1
second. Find an equation that models the distance an object will fall and use
it to determine how far it will fall in 2 seconds.

80. The weight of an object varies inversely as the square of its distance from
the center of earth. If an object weighs 180 pounds on the surface of earth
(approximately 4,000 miles from the center), then how much will it weigh at
2,000 miles above earth’s surface?

7.8 Review Exercises and Sample Exam 1248


Chapter 7 Rational Expressions and Equations

SAMPLE EXAM

Simplify and state the restrictions.

15x 3 (3x−1) 2
1.
3x(3x−1)

x 2 −144
2.
x 2 +12x

x 2 +x−12
3.
2x 2 +7x−4

9−x 2
4.
(x−3) 2

Simplify. (Assume all variables in the denominator are positive.)

5x x−5
5.
x 2 −25
⋅ 25x 2

x 2 +x−6 3x 2 −5x−2
6.
x 2 −4x+4
⋅ x 2 −9

x 2 −4x−12 x−6
7.
12x 2
÷ 6x

2x 2 −7x−4 2x 2 +7x+3
8.
6x 2 −24x
÷ 10x 2 +30x

1 1
9.
x−5
+ x+5

x 8 12x
10.
x+1
− 2−x
− x 2 −x−2

1
y + 1x
11. 1 1

y2 x2

9
1− 6x +
x2
12.
2− 5x − 3
x2

7.8 Review Exercises and Sample Exam 1249


Chapter 7 Rational Expressions and Equations

, calculate (f ⋅ g) (x) and


x 2 −81 4x−3
13. Given f (x) = and g (x) =
(4x−3) 2 x−9
state the restrictions.

, calculate (f − g) (x) and


x 1
14. Given f (x) = and g (x) =
x−5 3x−5
state the restrictions.

Solve.

1 1
15.
3
+ x =2

1 3
16.
x−5
= 2x−3

9 20
17. 1 − x + =0
x2

x+2 1 4(x+1)
18.
x−2
+ x+2
= x 2 −4

x 1 3x−10
19.
x−2
− x−3
= x 2 −5x+6

5 x 9x−4
20.
x+4
− 4−x
= x 2 −16

120
21. Solve for r: P= 1+3r
.

Set up an algebraic equation and then solve.

22. An integer is three times another. The sum of the reciprocals of the two
integers is 1/3. Find the two integers.

23. Working alone, Joe can paint the room in 6 hours. If Manny helps, then
together they can paint the room in 2 hours. How long would it take Manny
to paint the room by himself?

24. A river tour boat averages 6 miles per hour in still water. With the
current, the boat can travel 17 miles in the same time it can travel 7 miles
against the current. What is the speed of the current?

25. The breaking distance of an automobile is directly proportional to the


square of its speed. Under optimal conditions, a certain automobile moving

7.8 Review Exercises and Sample Exam 1250


Chapter 7 Rational Expressions and Equations

at 35 miles per hour can break to a stop in 25 feet. Find an equation that
models the breaking distance under optimal conditions and use it to
determine the breaking distance if the automobile is moving 28 miles per
hour.

7.8 Review Exercises and Sample Exam 1251


Chapter 7 Rational Expressions and Equations

REVIEW EXERCISES ANSWERS

1: 1/2, undefined, 1/2

3: 1/18, 1/9, 1/18

5: x ≠0

7: x ≠ ±5
1
9: ;x ≠ ±8
x+8

x+3
11: ;x ≠ −5, 8
x+5

13: − (x + 12) ; x ≠ 12

15: f (−3) = − 1
3
, f (0) = − 1
3
, f (3) = 0

x3
17:
3

3(x−2)
19:
5x

2x
21:
x−1

23: 3x (3x − 5)

x+4
25:
3x+2

2
27:
(8x+1) 2

(5y+1)(y−2)
29:
50y 6

31: (f ⋅ g) (x) =
(4x+3)(x−2)
;x ≠ −5, − 2, 3
x+2 4

7.8 Review Exercises and Sample Exam 1252


Chapter 7 Rational Expressions and Equations

5x−3
33: y

2x 2 −8x+1
(2x+1)(x−5)
35:

1
37: −
4x−1

x−5
39:
x−3

3
41:
(x−1)(x+2)(x+9)

y
43:
y−2

45: (f + g) (x) = 3x 2 −3x+1


(2x−5)(x+1)
;x ≠ −1, 5
2

47: 6

6x
49:
x−6

(x−3)(x+4)
51:
(x+1)(x+2)

x−7
53:
x+5

55: −3/5

57: −3

59: −10, 1

61: 3, 8

63: 3

65: Ø

7.8 Review Exercises and Sample Exam 1253


Chapter 7 Rational Expressions and Equations

bc
67: a = b+c

69: 6, 12

71: 7.5 miles per hour

73: 9 minutes

75: y = 3x

77: y = 3xz

79: d = 16t 2 ; 64 feet

7.8 Review Exercises and Sample Exam 1254


Chapter 7 Rational Expressions and Equations

SAMPLE EXAM ANSWERS

1: 5x 2 (3x − 1) ; x ≠ 0, 1
3

x−3
3: ;x ≠ −4, 1
2x−1 2

5x(x+5)
1
5:

x+2
7:
2x

( )(x+5)
2x
9:
x−5

xy
11: x−y

13: (f ⋅ g) (x) = x+9


;x ≠ 3
,9
4x−3 4

15: 3/5

17: 4, 5

19: 4

40 1
21: r = P
− 3

23: 3 hours

25: y = 1
49
x 2 ; 16 feet

7.8 Review Exercises and Sample Exam 1255


Chapter 8
Radical Expressions and Equations

1256
Chapter 8 Radical Expressions and Equations

8.1 Radicals

LEARNING OBJECTIVES

1. Find square roots.


2. Find cube roots.
3. Find nth roots.
4. Simplify expressions using the product and quotient rules for radicals.

Square Roots

The square root1 of a number is that number that when multiplied by itself yields
the original number. For example, 4 is a square root of 16, because 42 = 16. Since
(−4)2 = 16, we can say that −4 is a square root of 16 as well. Every positive real
number has two square roots, one positive and one negative. For this reason, we use
the radical sign √ to denote the principal (nonnegative) square root2 and a
negative sign in front of the radical −√ to denote the negative square root.

Zero is the only real number with one square root.

If the radicand3, the number inside the radical sign, is nonnegative and can be
1. The number that, when
multiplied by itself, yields the factored as the square of another nonnegative number, then the square root of the
original number. number is apparent. In this case, we have the following property:
2. The positive square root of a
real number, denoted with the
symbol √ .

3. The expression a within a


n ⎯⎯
radical sign, √ a.

1257
Chapter 8 Radical Expressions and Equations

Example 1: Find the square root.

⎯⎯⎯⎯
a. √36

⎯⎯⎯⎯⎯⎯
b. √144

⎯⎯⎯⎯⎯⎯⎯
c. √0.04

d. √ 19
⎯⎯⎯

Solution:

⎯⎯⎯⎯ ⎯⎯⎯⎯
a. √36 = √62 = 6

⎯⎯⎯⎯⎯⎯ ⎯⎯⎯⎯⎯⎯
b. √144 = √122 = 12

⎯⎯⎯⎯⎯⎯⎯ ⎯⎯⎯⎯⎯⎯⎯⎯⎯
c. √0.04 = √(0.2)2 = 0.2

d. √ 19 = √( 13 ) = 13
⎯⎯⎯ ⎯⎯⎯⎯⎯⎯⎯2⎯

Example 2: Find the negative square root.

⎯⎯
a. −√4

⎯⎯
b. −√1

Solution:

⎯⎯ ⎯⎯⎯⎯
a. −√4 = −√22 = −2

8.1 Radicals 1258


Chapter 8 Radical Expressions and Equations

⎯⎯ ⎯⎯⎯⎯
b. −√1 = −√12 = −1

The radicand may not always be a perfect square. If a positive integer is not a
⎯⎯
perfect square, then its square root will be irrational. For example, √2 is an
irrational number and can be approximated on most calculators using the square
root button.

Next, consider the square root of a negative number. To determine the square root
of −9, you must find a number that when squared results in −9:

However, any real number squared always results in a positive number:

The square root of a negative number is currently left undefined. For now, we will
⎯⎯⎯⎯⎯
state that √−9 is not a real a number.

Cube Roots

The cube root4 of a number is that number that when multiplied by itself three
times yields the original number. Furthermore, we denote a cube root using the
symbol √ 3 , where 3 is called the index5. For example,

4. The number that, when used as


a factor with itself three times,
yields the original number; it is
denoted with the symbol √ 3 .

5. The positive integer n in the


notation √n that is used to
The product of three equal factors will be positive if the factor is positive and
negative if the factor is negative. For this reason, any real number will have only
indicate an nth root.

8.1 Radicals 1259


Chapter 8 Radical Expressions and Equations

one real cube root. Hence the technicalities associated with the principal root do
not apply. For example,

In general, given any real number a, we have the following property:

When simplifying cube roots, look for factors that are perfect cubes.

Example 3: Find the cube root.

⎯⎯⎯⎯
a. √27
3

⎯⎯⎯⎯
b. √64
3

⎯⎯
c. √0
3

d. √
⎯⎯⎯
3 1
8

Solution:

⎯⎯⎯⎯ ⎯⎯⎯⎯
a. √27 = √33 = 3
3 3

⎯⎯⎯⎯ ⎯⎯⎯⎯
b. √64 = √43 = 4
3 3

⎯⎯ ⎯⎯⎯⎯
c. √0 = √03 = 0
3 3

8.1 Radicals 1260


Chapter 8 Radical Expressions and Equations

d. √ = √( 12 ) = 12
⎯⎯⎯ ⎯⎯⎯⎯⎯⎯⎯3⎯
3 1 3
8

Example 4: Find the cube root.

⎯⎯⎯⎯⎯
a. √−8
3

⎯⎯⎯⎯⎯
b. √−1
3

c. √
⎯⎯⎯⎯⎯⎯⎯
1
3
− 27

Solution:

⎯⎯⎯⎯⎯ ⎯⎯⎯⎯⎯⎯⎯⎯⎯
a. √−8 = √(−2)3 = −2
3 3

⎯⎯⎯⎯⎯ ⎯⎯⎯⎯⎯⎯⎯⎯⎯
b. √−1 = √(−1)3 = −1
3 3

c. √ =√ (− 3 ) = − 3
⎯⎯⎯⎯⎯⎯⎯ ⎯⎯⎯⎯⎯⎯⎯⎯⎯⎯⎯
1 1 3 1
3
− 27 3

It may be the case that the radicand is not a perfect cube. If an integer is not a
⎯⎯
perfect cube, then its cube root will be irrational. For example, √2 is an irrational
3

number which can be approximated on most calculators using the root button.
Depending on the calculator, we typically type in the index prior to pushing the
button and then the radicand as follows:

Therefore, we have

8.1 Radicals 1261


Chapter 8 Radical Expressions and Equations

nth Roots

For any integer n ≥ 2, we define the nth root6 of a positive real number as that
number that when raised to the nth power yields the original number. Given any
nonnegative real number a, we have the following property:

Here n is called the index and an is called the radicand. Furthermore, we can refer
n ⎯⎯
to the entire expression √ a as a radical7. When the index is an integer greater
than 3, we say “fourth root”, “fifth root”, and so on. The nth root of any number is
apparent if we can write the radicand with an exponent equal to the index.

Example 5: Find the nth root.

⎯⎯⎯⎯
a. √81
4

⎯⎯⎯⎯
b. √32
5

⎯⎯
c. √1
7

d. √
⎯⎯⎯⎯
4 1
16

Solution:

⎯⎯⎯⎯ ⎯⎯⎯⎯
a. √81 = √34 = 3
4 4

⎯⎯⎯⎯ ⎯⎯⎯⎯
b. √32 = √25 = 2
5 5

6. The number that, when raised


to the nth power, yields the
⎯⎯ ⎯⎯⎯⎯
c. √1 = √17 = 1
7 7
original number.

7. Used when referring to an


n ⎯⎯
expression of the form √ a.

8.1 Radicals 1262


Chapter 8 Radical Expressions and Equations

d. √ = √( 12 ) = 12
⎯⎯⎯⎯ ⎯⎯⎯⎯⎯⎯⎯4⎯
4 1 4
16

If the index is n = 2, then the radical indicates a square root and it is customary to
write the radical without the index, as illustrated below:

We have already taken care to define the principal square root of a number. At this
point, we extend this idea to nth roots when n is even. For example, 3 is a fourth
root of 81, because 34 = 81. And since (−3)4 = 81, we can say that −3 is a fourth
root of 81 as well. Hence we use the radical sign √
n to denote the principal
(nonnegative) nth root8 when n is even. In this case, for any real number a, we use
the following property:

For example,

The negative nth root, when n is even, will be denoted using a negative sign in front
of the radical −√
n .

We have seen that the square root of a negative number is not real because any real
number, when squared, will result in a positive number. In fact, a similar problem
arises for any even index:
8. The positive nth root when n is
even.

8.1 Radicals 1263


Chapter 8 Radical Expressions and Equations

Here the fourth root of −81 is not a real number because the fourth power of any
real number is always positive.

Example 6: Simplify.

⎯⎯⎯⎯⎯⎯⎯
a. √−16
4

⎯⎯⎯⎯
b. −√16
4

Solution:

a. The radicand is negative and the index is even. Therefore, there is no real
number that when raised to the fourth power is −16.

b. Here the radicand is positive. Furthermore, 16 = 24 , and we can simplify as


follows:

When n is odd, the same problems do not occur. The product of an odd number of
positive factors is positive and the product of an odd number of negative factors is

8.1 Radicals 1264


Chapter 8 Radical Expressions and Equations

negative. Hence when the index n is odd, there is only one real nth root for any real
number a. And we have the following property:

Example 7: Find the nth root.

⎯⎯⎯⎯⎯⎯⎯
a. √−32
5

⎯⎯⎯⎯⎯
b. √−1
7

Solution:

a. √−32 = √(−2)5 = −2
⎯⎯⎯⎯⎯⎯⎯ ⎯⎯⎯⎯⎯⎯⎯⎯⎯
5 5

⎯⎯⎯⎯⎯ ⎯⎯⎯⎯⎯⎯⎯⎯⎯
b. √−1 = √(−1)7 = −1
7 7

⎯⎯⎯⎯⎯⎯
Try this! Find the fourth root: √625.
4

Answer: 5

Video Solution

(click to see video)

Summary: When n is odd, the nth root is positive or negative depending on the
sign of the radicand.

8.1 Radicals 1265


Chapter 8 Radical Expressions and Equations

When n is even, the nth root is positive or not real depending on the sign of the
radicand.

Simplifying Using the Product and Quotient Rule for Radicals

It will not always be the case that the radicand is a perfect power of the given index.
If not, we use the following two properties to simplify them. If a and b represent
positive real numbers, then we have

n ⎯⎯⎯⎯⎯⎯⎯ n ⎯⎯ n ⎯⎯
Product rule for radicals9: √ a⋅b=√ a⋅√ b

√b =
⎯⎯a⎯ n a

Quotient rule for radicals10: n
n
√ b

A radical is simplified11 if it does not contain any factor that can be written as a
perfect power of the index.
⎯⎯⎯⎯⎯⎯⎯ ⎯⎯ ⎯⎯
9. √a ⋅ b = √ a ⋅ √b, where
n n n

a and b represent positive real


numbers.

√b
⎯a⎯⎯ n a
√ ⎯⎯⎯⎯
10. n
= n , where a and b Example 8: Simplify: √12.
√ b
represent positive real
numbers.
Solution: Here 12 can be written as 4 ⋅ 3, where 4 is a perfect square.
11. A radical where the radicand
does not consist of any factor
that can be written as a perfect
power of the index.

8.1 Radicals 1266


Chapter 8 Radical Expressions and Equations

We can verify our answer on a calculator:

Also, it is worth noting that

⎯⎯
Answer: 2√3

⎯⎯⎯⎯⎯⎯
Example 9: Simplify: √135.

Solution: Begin by finding the largest perfect square factor of 135.

Therefore,

8.1 Radicals 1267


Chapter 8 Radical Expressions and Equations

⎯⎯⎯⎯
Answer: 3√15

Example 10: Simplify: √ 121


50
⎯⎯⎯⎯⎯⎯
.

Solution: Begin by finding the prime factorizations of both 50 and 121. This will
enable us to easily determine the largest perfect square factors.

Therefore,

5√2
Answer: 11

⎯⎯⎯⎯⎯⎯
Example 11: Simplify: √162.
3

Solution: Use the prime factorization of 162 to find the largest perfect cube factor:

8.1 Radicals 1268


Chapter 8 Radical Expressions and Equations

Replace the radicand with this factorization and then apply the product rule for
radicals.

We can verify our answer on a calculator.

⎯⎯
Answer: 3 √6
3

⎯⎯⎯⎯
Try this! Simplify: 2 √96.
3

⎯⎯⎯⎯
Answer: 4 √12
3

Video Solution

(click to see video)

⎯⎯⎯⎯⎯⎯⎯
Example 12: Simplify: √−96.
5

Solution: Here we note that the index is odd and the radicand is negative; hence
the result will be negative. We can factor the radicand as follows:

8.1 Radicals 1269


Chapter 8 Radical Expressions and Equations

Then simplify:

⎯⎯
Answer: −2 √3
5

Example 13: Simplify: √


⎯⎯⎯⎯⎯⎯⎯
8
3
− 64 .

Solution: In this case, consider the equivalent fraction with −8 = (−2)3 in the
numerator and then simplify.

Answer: −1/2

8.1 Radicals 1270


Chapter 8 Radical Expressions and Equations

⎯⎯⎯⎯⎯⎯⎯⎯
Try this! Simplify √−108.
3

⎯⎯
Answer: −3 √4
3

Video Solution

(click to see video)

KEY TAKEAWAYS

• The square root of a number is that number that when multiplied by


itself yields the original number. When the radicand a is positive,
⎯⎯⎯⎯
√ a2 = a. When the radicand is negative, the result is not a real
number.
• The cube root of a number is that number that when used as a factor
with itself three times yields the original number. The cube root may be
positive or negative depending on the sign of the radicand. Therefore,
⎯⎯⎯⎯
for any real number a, we have the property √ a3 = a.
3

• When working with nth roots, n determines the definition that applies.
⎯⎯⎯⎯ ⎯⎯⎯⎯
We use √ an = awhen n is odd and √ an = |a|when n is even.
n n

When n is even, the negative nth root is denoted with a negative sign in
front of the radical sign.
• To simplify square roots, look for the largest perfect square factor of the
radicand and then apply the product or quotient rule for radicals.
• To simplify cube roots, look for the largest perfect cube factor of the
radicand and then apply the product or quotient rule for radicals.
• To simplify nth roots, look for the factors that have a power that is equal
to the index n and then apply the product or quotient rule for radicals.
Typically, the process is streamlined if you work with the prime
factorization of the radicand.

8.1 Radicals 1271


Chapter 8 Radical Expressions and Equations

TOPIC EXERCISES

Part A: Radicals

Simplify.

⎯⎯⎯⎯
1. √ 81

⎯⎯⎯⎯⎯⎯
2. √ 100

⎯⎯⎯⎯
3. √ 64

⎯⎯⎯⎯⎯⎯
4. √ 121

⎯⎯
5. √ 0

⎯⎯
6. √ 1

⎯⎯⎯⎯⎯⎯⎯
7. √ 0.25

⎯⎯⎯⎯⎯⎯⎯
8. √ 0.01

⎯⎯⎯⎯⎯⎯⎯
9. √ 1.21

⎯⎯⎯⎯⎯⎯⎯
10. √ 2.25

11. √
⎯1⎯⎯
4

12. √
⎯⎯⎯⎯
1

36

13. √
⎯25
⎯⎯⎯⎯
16

14. √
⎯⎯⎯⎯
9

25

8.1 Radicals 1272


Chapter 8 Radical Expressions and Equations

⎯⎯⎯⎯⎯⎯⎯
15. √ −25

⎯⎯⎯⎯⎯
16. √ −9

⎯⎯⎯⎯
17. −√ 36

⎯⎯⎯⎯
18. −√ 81

⎯⎯⎯⎯⎯⎯
19. −√ 100

⎯⎯
20. −√ 1

⎯⎯⎯⎯
21. √ 27
3

⎯⎯⎯⎯⎯⎯
22. √ 125
3

⎯⎯⎯⎯
23. √ 64
3

⎯⎯
24. √ 8
3

25. √
⎯1⎯⎯
3
8

26. √
⎯⎯⎯⎯
1

3
64

27. √
⎯⎯⎯⎯
8

3
27

28. √
⎯⎯⎯⎯⎯
64

3
125

⎯⎯⎯⎯⎯⎯⎯⎯⎯
29. √ 0.001
3

⎯⎯⎯⎯⎯⎯⎯⎯⎯
30. √ 1,000
3

8.1 Radicals 1273


Chapter 8 Radical Expressions and Equations

⎯⎯⎯⎯⎯
31. √ −1
3

⎯⎯⎯⎯⎯
32. √ −8
3

⎯⎯⎯⎯⎯⎯⎯
33. √ −27
3

⎯⎯⎯⎯⎯⎯⎯
34. √ −64
3

35. √
⎯⎯⎯⎯⎯1⎯
3
− 8

36. √
⎯⎯⎯⎯⎯⎯⎯
27

3
− 64

37. √
⎯⎯⎯⎯⎯⎯⎯8 ⎯
3
− 27

38. √
⎯⎯⎯⎯⎯⎯⎯⎯
1

3
− 125

⎯⎯⎯⎯
39. √ 81
4

⎯⎯⎯⎯⎯⎯
40. √ 625
4

⎯⎯⎯⎯
41. √ 16
4

⎯⎯⎯⎯⎯⎯⎯⎯⎯⎯⎯⎯
42. √ 10,000
4

⎯⎯⎯⎯
43. √ 32
5

⎯⎯
44. √ 1
5

⎯⎯⎯⎯⎯⎯
45. √ 243
5

⎯⎯⎯⎯⎯⎯⎯⎯⎯⎯⎯⎯⎯⎯
46. √ 100,000
5

8.1 Radicals 1274


Chapter 8 Radical Expressions and Equations

⎯⎯⎯⎯
47. −√ 16
4

⎯⎯
48. −√ 1
6

⎯⎯⎯⎯⎯⎯⎯
49. √ −32
5

⎯⎯⎯⎯⎯
50. √ −1
5

⎯⎯⎯⎯⎯
51. √ −1

⎯⎯⎯⎯⎯⎯⎯
52. √ −16
4

⎯⎯⎯⎯⎯⎯⎯
53. −5 √ −27
3

⎯⎯⎯⎯⎯
54. −2 √ −8
3

⎯⎯⎯⎯⎯⎯⎯⎯⎯⎯⎯⎯
55. 5 √ −1,000
3

⎯⎯⎯⎯⎯⎯⎯⎯⎯
56. 3 √ −243
5

⎯⎯⎯⎯⎯⎯⎯
57. 10 √ −16
4

⎯⎯⎯⎯⎯⎯⎯
58. 2 √ −64
6

⎯⎯⎯⎯
59. 3√ 25

⎯⎯
60. 6√ 4

⎯⎯⎯⎯
61. 2 √ 27
3

⎯⎯⎯⎯⎯⎯
62. 8 √ 243
5

⎯⎯
63. −7 √ 8
3

8.1 Radicals 1275


Chapter 8 Radical Expressions and Equations

⎯⎯⎯⎯⎯⎯
64. −4 √ 625
4

⎯⎯⎯⎯⎯⎯⎯⎯⎯⎯⎯⎯⎯⎯
65. 6 √ 100,000
5

⎯⎯⎯⎯⎯⎯
66. 5 √ 128
7

Part B: Simplifying Radicals

Simplify.

⎯⎯⎯⎯
67. √ 32

⎯⎯⎯⎯⎯⎯
68. √ 250

⎯⎯⎯⎯
69. √ 80

⎯⎯⎯⎯⎯⎯
70. √ 150

⎯⎯⎯⎯⎯⎯
71. √ 160

⎯⎯⎯⎯
72. √ 60

⎯⎯⎯⎯⎯⎯
73. √ 175

⎯⎯⎯⎯⎯⎯
74. √ 216

⎯⎯⎯⎯⎯⎯
75. 5√ 112

⎯⎯⎯⎯⎯⎯
76. 10√ 135

77. √
⎯50
⎯⎯⎯⎯
49

⎯⎯⎯⎯⎯⎯
78. −2√ 120

8.1 Radicals 1276


Chapter 8 Radical Expressions and Equations

⎯⎯⎯⎯⎯⎯
79. −3√ 162

80. √
⎯8⎯⎯
9

81. √
⎯⎯⎯⎯⎯
45

121

82. √
⎯96
⎯⎯⎯⎯
81

⎯⎯⎯⎯
83. √ 54
3

⎯⎯⎯⎯
84. √ 24
3

⎯⎯⎯⎯
85. √ 48
3

⎯⎯⎯⎯
86. √ 81
3

⎯⎯⎯⎯
87. √ 40
3

⎯⎯⎯⎯⎯⎯
88. √ 120
3

⎯⎯⎯⎯⎯⎯
89. √ 162
3

⎯⎯⎯⎯⎯⎯
90. √ 500
3

91. √
⎯⎯⎯⎯⎯
54

3
125

92. √
⎯⎯⎯⎯⎯
40

3
343

⎯⎯⎯⎯⎯⎯⎯
93. 5 √ −48
3

⎯⎯⎯⎯⎯⎯⎯⎯⎯
94. 2 √ −108
3

8.1 Radicals 1277


Chapter 8 Radical Expressions and Equations

⎯⎯⎯⎯
95. 8 √ 96
4

⎯⎯⎯⎯⎯⎯
96. 7 √ 162
4

⎯⎯⎯⎯⎯⎯
97. √ 160
5

⎯⎯⎯⎯⎯⎯
98. √ 486
5

99. √
⎯224
⎯⎯⎯⎯⎯
5
243

100. √
⎯⎯⎯⎯
5

5
32

Simplify. Give the exact answer and the approximate answer rounded to the nearest
hundredth.

⎯⎯
101. √ 8

⎯⎯⎯⎯⎯⎯
102. √ 200

⎯⎯⎯⎯
103. √ 45

⎯⎯⎯⎯
104. √ 72

105. √
⎯3⎯⎯
4

106. √
⎯5⎯⎯
9

107. √
⎯32
⎯⎯⎯⎯
25

108. √
⎯48
⎯⎯⎯⎯
49

⎯⎯⎯⎯
109. √ 80
3

8.1 Radicals 1278


Chapter 8 Radical Expressions and Equations

⎯⎯⎯⎯⎯⎯
110. √ 320
3

⎯⎯⎯⎯
111. √ 48
3

⎯⎯⎯⎯⎯⎯
112. √ 270
3

Rewrite the following as a radical expression with coefficient 1.

⎯⎯⎯⎯
113. 2√ 15

⎯⎯
114. 3√ 7

⎯⎯⎯⎯
115. 5√ 10

⎯⎯
116. 10√ 3

⎯⎯
117. 2 √ 7
3

⎯⎯
118. 3 √ 6
3

⎯⎯
119. 2 √ 5
4

⎯⎯
120. 3 √ 2
4

121. The formula for the area A of a square is A = s2 . If the area is 18


square units, then what is the length of each side?

122. Calculate the length of a side of a square with an area of 60 square


centimeters.

123. The formula for the volume V of a cube is V = s3 . If the volume of a


cube is 112 cubic units, then what is the length of each side?

124. Calculate the length of a side of a cube with a volume of 54 cubic


centimeters.

8.1 Radicals 1279


Chapter 8 Radical Expressions and Equations

Part C: Discussion Board

125. Explain why there are two square roots for any nonzero real number.

126. Explain why there is only one cube root for any real number.

127. What is the square root of 1, and what is the cube root of 1? Explain
why.

⎯⎯⎯⎯⎯ ⎯⎯⎯⎯⎯
128. Explain why √ −1 is not a real number and why √ −1 is a real
3

number.

8.1 Radicals 1280


Chapter 8 Radical Expressions and Equations

ANSWERS

1: 9

3: 8

5: 0

7: 0.5

9: 1.1

11: 1/2

13: 5/4

15: Not a real number

17: −6

19: −10

21: 3

23: 4

25: 1/2

27: 2/3

29: 0.1

31: −1

33: −3

35: −1/2

37: −2/3

8.1 Radicals 1281


Chapter 8 Radical Expressions and Equations

39: 3

41: 2

43: 2

45: 3

47: −2

49: −2

51: Not a real number

53: 15

55: −50

57: Not a real number

59: 15

61: 6

63: −14

65: 60

⎯⎯
67: 4√ 2

⎯⎯
69: 4√ 5

⎯⎯⎯⎯
71: 4√ 10

⎯⎯
73: 5√ 7

⎯⎯
75: 20√ 7

8.1 Radicals 1282


Chapter 8 Radical Expressions and Equations

5√2
77: 7

⎯⎯
79: −27√ 2

3√5
81:
11

⎯⎯
83: 3 √ 2
3

⎯⎯
85: 2 √ 6
3

⎯⎯
87: 2 √ 5
3

⎯⎯
89: 3 √ 6
3

3
3√ 2
91:
5

⎯⎯
93: −10 √ 6
3

⎯⎯
95: 16 √ 6
4

⎯⎯
97: 2 √ 5
5

5
2√ 7
99:
3

⎯⎯
101: 2√ 2 ≈ 2.83
⎯⎯
103: 3√ 5 ≈ 6.71

√3
105:
2
≈ 0.87

4√2
107:
5
≈ 1.13

8.1 Radicals 1283


Chapter 8 Radical Expressions and Equations

⎯⎯⎯⎯
109: 2 √ 10 ≈ 4.31
3

⎯⎯
111: 2 √ 6 ≈ 3.63
3

⎯⎯⎯⎯
113: √ 60

⎯⎯⎯⎯⎯⎯
115: √ 250

⎯⎯⎯⎯
117: √ 56
3

⎯⎯⎯⎯
119: √ 80
4

⎯⎯
121: 3√ 2 units

⎯⎯⎯⎯
123: 2 √ 14 units
3

8.1 Radicals 1284


Chapter 8 Radical Expressions and Equations

8.2 Simplifying Radical Expressions

LEARNING OBJECTIVES

1. Simplify radical expressions using the product and quotient rule for
radicals.
2. Use formulas involving radicals.
3. Evaluate given square root and cube root functions.

Simplifying Radical Expressions

An algebraic expression that contains radicals is called a radical expression12. We


use the product and quotient rules to simplify them.

⎯⎯⎯⎯⎯⎯
Example 1: Simplify: √
3
8y 3 .

⎯⎯⎯⎯
a = a when n is odd.
n n
Solution: Use the fact that √

Answer: 2y

⎯⎯⎯⎯⎯⎯
Example 2: Simplify: √9x 2 .

⎯⎯⎯⎯
a = |a| when n is even.
n n
Solution: The square root has index 2; use the fact that √

12. An algebraic expression that


contains radicals.

1285
Chapter 8 Radical Expressions and Equations

Since x is a variable, it may represent a negative number. Thus we need to ensure


that the result is positive by including the absolute value operator.

Answer: 3 |x|

Important Note

Typically, at this point beginning algebra texts note that all variables are
assumed to be positive. If this is the case, then x in the previous example is
positive and the absolute value operator is not needed. The example can be
simplified as follows:

⎯⎯⎯⎯⎯⎯ ⎯⎯⎯⎯⎯⎯⎯⎯
√9x 2 = √32 x 2
⎯⎯⎯⎯ ⎯⎯⎯⎯
= √32 ⋅ √x 2
= 3x

In this section, we will assume that all variables are positive. This allows us to
focus on calculating nth roots without the technicalities associated with the
principal nth root problem. For this reason, we will use the following property
for the rest of the section:

n ⎯⎯⎯⎯
√ an = a, if a≥0 nth root

When simplifying radical expressions, look for factors with powers that match the
index.

⎯⎯⎯⎯⎯⎯⎯⎯⎯⎯⎯
Example 3: Simplify: √18x 3 y 4 .

8.2 Simplifying Radical Expressions 1286


Chapter 8 Radical Expressions and Equations

Solution: Begin by determining the square factors of 18, x 3 , and y 4 .

Make these substitutions and then apply the product rule for radicals and simplify.

⎯⎯⎯⎯
Answer: 3xy 2 √2x

Example 4: Simplify: √ 4a6 .


⎯⎯⎯⎯5⎯
b

Solution: Begin by determining the square factors of 4, a5 , and b6 .

Make these substitutions and then apply the product rule for radicals and simplify.

8.2 Simplifying Radical Expressions 1287


Chapter 8 Radical Expressions and Equations

2a2 √a
Answer:
b3

⎯⎯⎯⎯⎯⎯⎯⎯⎯⎯⎯
Example 5: Simplify: √
3
80x 5 y 7 .

Solution: Begin by determining the cubic factors of 80, x 5 , and y 7 .

Make these substitutions and then apply the product rule for radicals and simplify.

⎯⎯⎯⎯⎯⎯⎯⎯⎯⎯
Answer: 2xy 2 ⋅ √
3
10x 2 y

8.2 Simplifying Radical Expressions 1288


Chapter 8 Radical Expressions and Equations

√y z .
⎯⎯⎯⎯⎯
9x 6

3
Example 6: Simplify 3 9

Solution: The coefficient 9 = 32 and thus does not have any perfect cube factors. It
will be left as the only remaining radicand because all of the other factors are cubes,
as illustrated below:

Replace the variables with these equivalents, apply the product and quotient rule
for radicals, and then simplify.

3
x 2 ⋅√ 9
Answer:
yz 3

8.2 Simplifying Radical Expressions 1289


Chapter 8 Radical Expressions and Equations

⎯⎯⎯⎯⎯⎯⎯⎯⎯⎯⎯
Example 7: Simplify: √81a4 b5 .
4

Solution: Determine all factors that can be written as perfect powers of 4. Here it is
important to see that b5 = b4 ⋅ b. Hence the factor b will be left inside the radical.

⎯⎯
Answer: 3ab ⋅ √b
4

⎯⎯⎯⎯⎯⎯⎯⎯⎯⎯⎯⎯⎯⎯⎯⎯⎯
Example 8: Simplify: √
5
−32x 3 y 6 z 5 .

Solution: Notice that the variable factor x cannot be written as a power of 5 and
thus will be left inside the radical. In addition, for y 6 = y 5 ⋅ y ; the factor y will be
left inside the radical as well.

⎯⎯⎯⎯⎯⎯
Answer: −2yz ⋅ √
5
x3y

⎯⎯⎯⎯⎯⎯⎯⎯⎯⎯⎯⎯⎯⎯⎯⎯⎯⎯
Try this! Simplify: √192x 6 y 7 z 12 . (Assume all variables are positive.)

⎯⎯⎯⎯
Answer: 8x 3 y 3 z 6 √3y

8.2 Simplifying Radical Expressions 1290


Chapter 8 Radical Expressions and Equations

Video Solution

(click to see video)

Tip

To easily simplify an nth root, we can divide the powers by the index.

⎯⎯⎯⎯
√a6 = a3 , which is a6÷2 = a3
3 ⎯⎯⎯6⎯
√ b = b2 , which is b6÷3 = b2
6 ⎯⎯⎯⎯
√ c6 = c , which is c6÷6 = c1

If the index does not divide into the power evenly, then we can use the quotient
and remainder to simplify. For example,

⎯⎯⎯⎯
√a5 = a2 ⋅ √⎯⎯a, which is a5÷2 = a2 r 1
3 ⎯⎯⎯5⎯ 3 ⎯⎯⎯⎯
√ b = b ⋅ √b2 , which is b5÷3 = b1 r 2
5 ⎯⎯⎯⎯ ⎯ 5 ⎯⎯⎯⎯
√ c14 = c2 ⋅ √c4 , which is c14÷5 = c2 r 4

The quotient is the exponent of the factor outside of the radical, and the
remainder is the exponent of the factor left inside the radical.

Formulas Involving Radicals

We next review the distance formula. Given two points (x 1 , y 1 ) and (x 2 , y 2 ),

The distance, d, between them is given by the following formula:

8.2 Simplifying Radical Expressions 1291


Chapter 8 Radical Expressions and Equations

Distance formula13: d = √(x 2 − x 1 )2 + (y 2 − y 1 )


⎯⎯⎯⎯⎯⎯⎯⎯⎯⎯⎯⎯⎯⎯⎯⎯⎯⎯⎯⎯⎯⎯⎯⎯⎯⎯⎯⎯⎯⎯⎯⎯⎯⎯⎯2⎯

Recall that this formula was derived from the Pythagorean theorem.

Example 9: Calculate the distance between (−4, 7) and (2, 1).

Solution: Use the distance formula with the following points.

It is a good practice to include the formula in its general form before substituting
values for the variables; this improves readability and reduces the probability of
making errors.

13. Given two points (x 1 , y 1 ) and


(x 2 , y 2 ), calculate the
distance d between them using

√(x 2 − x 1 ) + (y 2 − y 1 ) .
the formula d =
⎯⎯⎯⎯⎯⎯⎯⎯⎯⎯⎯⎯⎯⎯⎯⎯⎯⎯⎯⎯⎯⎯⎯⎯⎯⎯⎯⎯⎯⎯⎯⎯⎯⎯⎯⎯2⎯
2

8.2 Simplifying Radical Expressions 1292


Chapter 8 Radical Expressions and Equations

⎯⎯
Answer: 6√2 units

Example 10: The period, T, of a pendulum in seconds is given by the formula

where L represents the length of the pendulum in feet. If the length of a pendulum
measures 6 feet, then calculate the period rounded off to the nearest tenth of a
second.

8.2 Simplifying Radical Expressions 1293


Chapter 8 Radical Expressions and Equations

Solution: Substitute 6 for L and then simplify.

8.2 Simplifying Radical Expressions 1294


Chapter 8 Radical Expressions and Equations

Answer: The period is approximately 2.7 seconds.

Square Root and Cube Root Functions

We begin with the square root function14:

We know that the square root is not a real number when the radicand x is negative.
Therefore, we conclude that the domain consists of all real numbers greater than or
equal to 0. Here we choose 0 and some positive values for x, calculate the
corresponding y-values, and plot the resulting ordered pairs.

After plotting the points, we can then sketch the graph of the square root function.

⎯⎯
14. The function f (x) = √x .

8.2 Simplifying Radical Expressions 1295


Chapter 8 Radical Expressions and Equations

⎯⎯⎯⎯⎯⎯⎯⎯
Example 11: Given the function f (x) = √x + 2 , find f (−2), f (2), and f (6).

Solution: Replace x with each of the given values.

⎯⎯
Answer: f (−2) = 0, f (2) = 2, and f (6) = 2√2

Next, consider the cube root function15:


3 ⎯⎯
15. The function f (x) = √ x.

8.2 Simplifying Radical Expressions 1296


Chapter 8 Radical Expressions and Equations

Since the cube root could be either negative or positive, we conclude that the
domain consists of all real numbers. For completeness, choose some positive and
negative values for x, as well as 0, and then calculate the corresponding y-values.

Plot the points and sketch the graph of the cube root function.

Example 12: Given the function g(x) = √x − 1, find g (−7), g (0), and g (55).
3 ⎯⎯⎯⎯⎯⎯⎯⎯

8.2 Simplifying Radical Expressions 1297


Chapter 8 Radical Expressions and Equations

Solution: Replace x with each of the given values.

Answer: g (−7) = −2, g (0) = −1, and g (55) = 3 √2


3 ⎯⎯

KEY TAKEAWAYS

• In beginning algebra, we typically assume that all variable expressions


within the radical are positive. This allows us to focus on simplifying
radicals without the technical issues associated with the principal nth
root.
• To simplify radical expressions, look for factors of the radicand with
powers that match the index. If found, they can be simplified by
applying the product and quotient rules for radicals, as well as the
⎯⎯⎯⎯
property √ an = a, where a is positive.
n

8.2 Simplifying Radical Expressions 1298


Chapter 8 Radical Expressions and Equations

TOPIC EXERCISES

Part A: Simplifying Radical Expressions

Simplify. (Assume all variables represent positive numbers.)

⎯⎯⎯⎯⎯⎯⎯⎯
1. √ 36a2

⎯⎯⎯⎯⎯⎯⎯⎯⎯2⎯
2. √ 121b

⎯⎯⎯⎯⎯⎯⎯⎯
3. √ x 2 y 2

⎯⎯⎯⎯⎯⎯⎯⎯⎯⎯⎯⎯⎯⎯⎯
4. √ 25x 2 y 2 z 2

⎯⎯⎯⎯⎯⎯⎯⎯⎯⎯
5. √ 180x 3

⎯⎯⎯⎯⎯⎯⎯⎯⎯⎯
6. √ 150y 3

⎯⎯⎯⎯⎯⎯⎯⎯⎯⎯2⎯
7. √ 49a3 b

⎯⎯⎯⎯⎯⎯⎯⎯⎯⎯
3

8. √ 4a4 b c
⎯⎯⎯⎯⎯⎯⎯⎯⎯⎯⎯⎯
9. √ 45x 5 y 3

⎯⎯⎯⎯⎯⎯⎯⎯⎯⎯⎯⎯
10. √ 50x 6 y 4

⎯⎯⎯⎯⎯⎯⎯⎯⎯⎯⎯⎯⎯
11. √ 64r2 s6 t 5

⎯⎯⎯⎯⎯⎯⎯⎯⎯⎯⎯⎯⎯⎯⎯
12. √ 144r8 s6 t 2

⎯⎯⎯⎯⎯⎯⎯⎯⎯⎯⎯⎯2⎯
13. √ (x + 1)
⎯⎯⎯⎯⎯⎯⎯⎯⎯⎯⎯⎯⎯⎯2⎯
14. √ (2x + 3)
⎯⎯⎯⎯⎯⎯⎯⎯⎯⎯⎯⎯⎯⎯⎯⎯2⎯
15. √ 4(3x − 1)

8.2 Simplifying Radical Expressions 1299


Chapter 8 Radical Expressions and Equations

⎯⎯⎯⎯⎯⎯⎯⎯⎯⎯⎯⎯⎯⎯⎯⎯2⎯
16. √ 9(2x + 3)

√ 25y 2
⎯⎯⎯⎯⎯⎯
9x 3

17.

√ 9y 4
⎯4x
⎯⎯⎯⎯5⎯
18.

19. √
⎯⎯⎯⎯⎯⎯
m7

36n 4

20. √
⎯147m
⎯⎯⎯⎯⎯⎯⎯9 ⎯
n6

21. √
⎯2r
⎯⎯⎯⎯⎯
2 s5

25t 4

22. √
⎯36r
⎯⎯⎯⎯⎯5⎯
s2 t 6

⎯⎯⎯⎯⎯⎯⎯⎯
23. √ 27a3
3

⎯⎯⎯⎯⎯⎯⎯⎯⎯3⎯
24. √ 125b
3

⎯⎯⎯⎯⎯⎯⎯⎯⎯⎯⎯⎯⎯⎯
25. √ 250x 4 y 3
3

⎯⎯⎯⎯⎯⎯⎯⎯⎯⎯⎯⎯5⎯
26. √ 162a3 b
3

⎯⎯⎯⎯⎯⎯⎯⎯⎯⎯⎯⎯⎯⎯⎯
27. √ 64x 3 y 6 z 9
3

⎯⎯⎯⎯⎯⎯⎯⎯⎯⎯⎯⎯⎯⎯⎯
28. √ 216x 12 y 3
3

⎯⎯⎯⎯⎯⎯⎯⎯⎯⎯
29. √ 8x 3 y 4
3

⎯⎯⎯⎯⎯⎯⎯⎯⎯⎯⎯⎯
30. √ 27x 5 y 3
3

⎯⎯⎯⎯⎯⎯⎯⎯⎯⎯
5 6

31. √ a4 b
3
c

8.2 Simplifying Radical Expressions 1300


Chapter 8 Radical Expressions and Equations

⎯⎯⎯⎯⎯⎯⎯⎯⎯⎯
5 3

32. √ a7 b
3
c

√ 27y 3
⎯⎯⎯⎯⎯⎯
8x 4

3
33.

√ 125y 6
⎯⎯⎯⎯⎯⎯⎯⎯
x5

3
34.

⎯⎯⎯⎯⎯⎯⎯⎯⎯⎯⎯⎯⎯⎯⎯⎯⎯⎯
35. √ 360r5 s12 t 13
3

⎯⎯⎯⎯⎯⎯⎯⎯⎯⎯⎯⎯⎯⎯⎯
36. √ 540r3 s2 t 9
3

⎯⎯⎯⎯⎯⎯⎯⎯
37. √ 81x 4
4

⎯⎯⎯⎯⎯⎯⎯⎯
38. √ x 4 y 4
4

⎯⎯⎯⎯⎯⎯⎯⎯⎯⎯⎯⎯
39. √ 16x 4 y 8
4

⎯⎯⎯⎯⎯⎯⎯⎯⎯⎯⎯⎯⎯
40. √ 81x 12 y 4
4

⎯⎯⎯⎯⎯⎯⎯⎯⎯⎯
5 6

41. √ a4 b
4
c
⎯⎯⎯⎯⎯⎯⎯⎯⎯⎯
4 6 8

42. √ 5
4
ac
⎯⎯⎯⎯⎯⎯⎯⎯⎯⎯
43. √ 128x 6
4

⎯⎯⎯⎯⎯⎯⎯⎯⎯⎯
44. √ 243y 7
4

45. √
⎯32m
⎯⎯⎯⎯⎯⎯⎯
10

5
n5

46. √
⎯⎯⎯⎯⎯⎯⎯
7 9⎯
5 3 m
n 10

⎯⎯⎯⎯⎯⎯
47. −3√ 4x 2

8.2 Simplifying Radical Expressions 1301


Chapter 8 Radical Expressions and Equations

⎯⎯⎯⎯⎯⎯
48. 7√ 9y 2

⎯⎯⎯⎯⎯⎯⎯⎯
49. −5x√ 4x 2 y

⎯⎯⎯⎯⎯⎯⎯⎯⎯⎯⎯⎯
50. −3y√ 16x 3 y 2

⎯⎯⎯⎯⎯⎯3⎯
51. 12ab√ a5 b

⎯⎯⎯⎯⎯⎯⎯⎯2⎯
52. 6a2 b√ 9a7 b

3 ⎯⎯⎯⎯⎯⎯
53. 2x ⋅ √ 8x 6
3 ⎯⎯⎯⎯⎯⎯⎯⎯
54. −5x 2 ⋅ √ 27x 3
3 ⎯⎯⎯⎯⎯⎯⎯⎯⎯⎯⎯⎯
55. 2ab ⋅ √ −8a4 b 5
3 ⎯⎯⎯⎯⎯⎯⎯⎯⎯⎯⎯⎯⎯⎯
56. 5a2 b ⋅ √ −27a3 b 3

Rewrite the following as a radical expression with coefficient 1.

⎯⎯⎯⎯
57. 5√ 2x

⎯⎯⎯⎯
58. 2√ 3y

⎯⎯
59. 2x√ 3

⎯⎯
60. 3y√ 2

⎯⎯⎯⎯⎯⎯
61. ab√ 10a

2 ⎯⎯
62. 2ab √a
⎯⎯⎯⎯⎯
63. m 2 n√ mn

8.2 Simplifying Radical Expressions 1302


Chapter 8 Radical Expressions and Equations

⎯⎯⎯⎯
64. 2m 2 n 3 √ 3n

⎯⎯⎯⎯
65. 5 √ 2x
3

⎯⎯⎯⎯
66. 3 √
3
5y

3 ⎯⎯
67. 2x ⋅√ 3

3 ⎯⎯
68. 3y ⋅√ 2

Assume that the variable could represent any real number and then simplify.

⎯⎯⎯⎯⎯⎯
69. √ 4x 2

⎯⎯⎯⎯⎯⎯⎯⎯
70. √ 25y 2

⎯⎯⎯⎯⎯⎯
71. √ 8y 3
3

⎯⎯⎯⎯⎯⎯⎯⎯⎯⎯
72. √ 125a3
3

⎯⎯⎯⎯⎯⎯⎯⎯
73. √ 64x 4
4

⎯⎯⎯⎯⎯⎯⎯⎯
74. √ 81y 4
4

⎯⎯⎯⎯⎯⎯⎯⎯
75. √ 36a4

⎯⎯⎯⎯⎯⎯⎯⎯⎯⎯
76. √ 100a8

⎯⎯⎯⎯⎯⎯
77. √ 4a6

⎯⎯⎯⎯⎯
78. √ a10

⎯⎯⎯⎯⎯⎯⎯⎯⎯⎯5⎯
79. √ 18a4 b

8.2 Simplifying Radical Expressions 1303


Chapter 8 Radical Expressions and Equations

⎯⎯⎯⎯⎯⎯⎯⎯⎯⎯3⎯
80. √ 48a5 b

⎯⎯⎯⎯⎯⎯⎯⎯⎯⎯⎯⎯⎯⎯
81. √ 128x 6 y 8
6

⎯⎯⎯⎯⎯⎯⎯⎯⎯⎯
7 8

82. √ a6 b
6
c

Part B: Formulas Involving Radicals

The y-intercepts for any graph will have the form (0, y), where y is a real number.
Therefore, to find y-intercepts, set x = 0 and solve for y. Find the y-intercepts for the
following.

⎯⎯⎯⎯⎯⎯⎯⎯⎯
83. y = √x + 4 − 1
⎯⎯⎯⎯⎯⎯⎯⎯⎯
84. y = √x + 1 − 3

3 ⎯⎯⎯⎯⎯⎯⎯⎯⎯
85. y =√ x−1 +2

3 ⎯⎯⎯⎯⎯⎯⎯⎯⎯
86. y =√ x+1 −3

Use the distance formula to calculate the distance between the given two points.

87. (5, −7) and (3, −8)

88. (−9, 7) and (−8, 4)

89. (−3, −4) and (3, −6)

90. (−5, −2) and (1, −6)

91. (−1, 1) and (−4, 10)

92. (8, −3) and (2, −12)

Factor the radicand and then simplify. (Assume that all expressions are positive.)

⎯⎯⎯⎯⎯⎯⎯⎯⎯⎯⎯⎯⎯⎯⎯⎯⎯⎯⎯
93. √ x 2 − 6x + 9

8.2 Simplifying Radical Expressions 1304


Chapter 8 Radical Expressions and Equations

⎯⎯⎯⎯⎯⎯⎯⎯⎯⎯⎯⎯⎯⎯⎯⎯⎯⎯⎯⎯⎯⎯⎯
94. √ x 2 − 10x + 25
⎯⎯⎯⎯⎯⎯⎯⎯⎯⎯⎯⎯⎯⎯⎯⎯⎯⎯⎯⎯⎯⎯⎯
95. √ 4x 2 + 12x + 9
⎯⎯⎯⎯⎯⎯⎯⎯⎯⎯⎯⎯⎯⎯⎯⎯⎯⎯⎯⎯⎯
96. √ 9x 2 + 6x + 1

97. The speed of a vehicle before the brakes were applied can be estimated
by the length of the skid marks left on the road. On dry pavement, the speed,
⎯⎯
v, in miles per hour can be estimated by the formula v = 5√ d , where d
represents the length of the skid marks in feet. Estimate the speed of a
vehicle before applying the brakes on dry pavement if the skid marks left
behind measure 36 feet.

98. The radius, r, of a sphere can be calculated using the formula


3
√ 6π 2 V
r= 2π
, where V represents the sphere’s volume. What is the radius of
a sphere if the volume is 36π cubic centimeters?

The period, T, of a pendulum in seconds is given by the formula

⎯⎯⎯⎯⎯⎯
√ 32
L
T = 2π

where L represents the length in feet. Calculate the period, given the following
lengths. Give the exact value and the approximate value rounded off to the nearest
tenth of a second.

99. 8 feet

100. 32 feet

101. 1/2 foot

102. 1/8 foot

The time, t, in seconds that an object is in free fall is given by the formula

8.2 Simplifying Radical Expressions 1305


Chapter 8 Radical Expressions and Equations


√s
t=
4

where s represents the distance it has fallen in feet. Calculate the time it takes an
object to fall, given the following distances. Give the exact value and the
approximate value rounded off to the nearest tenth of a second.

103. 48 feet

104. 80 feet

105. 192 feet

106. 288 feet

Part C: Radical Functions

Given the function, calculate the following.

⎯⎯⎯⎯⎯⎯⎯⎯⎯
107. f (x) = √ x − 1 , find f (1) , f (2) , and f (5)
⎯⎯⎯⎯⎯⎯⎯⎯⎯
108. f (x) = √ x + 5 , find f (−5) , f (−1) , and f (20)
⎯⎯
109. f (x) = √ x + 3, find f (0) , f (1) , and f (16)
⎯⎯
110. f (x) = √ x − 5, find f (0) , f (1) , and f (25)

3 ⎯⎯
111. g(x) =√ x , find g(−1) , g(0) , and g(1)

3 ⎯⎯⎯⎯⎯⎯⎯⎯⎯
112. g(x) =√ x + 7 , find g(−15) , g(−7) , and g(20)

3 ⎯⎯
113. g(x) =√ x − 2, find g(−1) , g(0) , and g(8)

3 ⎯⎯⎯⎯⎯⎯⎯⎯⎯
114. g(x) =√ x − 1 + 2, find g(0) , g(2) , and g(9)

For each function, fill in the table.

8.2 Simplifying Radical Expressions 1306


Chapter 8 Radical Expressions and Equations

⎯⎯⎯⎯⎯⎯⎯⎯⎯
115. f (x) = √x + 1

⎯⎯⎯⎯⎯⎯⎯⎯⎯
116. f (x) = √x − 2

3 ⎯⎯
117. f (x) =√ x +1

3 ⎯⎯⎯⎯⎯⎯⎯⎯⎯
118. f (x) =√ x+2

8.2 Simplifying Radical Expressions 1307


Chapter 8 Radical Expressions and Equations

Part D: Discussion Board

⎯⎯⎯⎯
119. Give a value for x such that √ x 2 ≠ x. Explain why it is important to
assume that the variables represent positive numbers.

120. Research and discuss the accomplishments of Christoph Rudolff. What


is he credited for?

121. Research and discuss the methods used for calculating square roots
before the common use of electronic calculators.

122. What is a surd, and where does the word come from?

8.2 Simplifying Radical Expressions 1308


Chapter 8 Radical Expressions and Equations

ANSWERS

1: 6a

3: xy

⎯⎯⎯⎯
5: 6x√ 5x

⎯⎯
7: 7ab√ a

⎯⎯⎯⎯⎯⎯
9: 3x 2 y√ 5xy

11: 8rs3 t 2 √ t

13: x +1

15: 2 (3x − 1)

3x√x
17:
5y

m 3 √m
19:
6n 2

rs2 √2s
21:
5t 2

23: 3a

3 ⎯⎯⎯⎯
25: 5xy ⋅√ 2x

27: 4xy 2 z 3

3 ⎯⎯
29: 2xy ⋅√ y

3 ⎯⎯⎯⎯⎯⎯
31: abc2 ⋅ √ ab 2

8.2 Simplifying Radical Expressions 1309


Chapter 8 Radical Expressions and Equations

3 x
2x⋅√
33:
3y

3 ⎯⎯⎯⎯⎯⎯⎯⎯
35: 2rs4 t 4 ⋅ √ 45r2 t

37: 3x

39: 2xy 2

4 ⎯⎯⎯⎯⎯⎯
41: abc ⋅ √ bc2
4 ⎯⎯⎯⎯⎯⎯
43: 2x ⋅ √ 8x 2

2m 2
45: n

47: −6x

49: −10x 2 √ y
⎯⎯

2 ⎯⎯⎯⎯
51: 12a3 b √ ab

53: 4x 3

2 3 ⎯⎯⎯⎯⎯⎯
55: −4a2 b ⋅ √ ab 2
⎯⎯⎯⎯⎯⎯
57: √ 50x

⎯⎯⎯⎯⎯⎯⎯⎯
59: √ 12x 2

⎯⎯⎯⎯⎯⎯⎯⎯⎯⎯2⎯
61: √ 10a3 b

⎯⎯⎯⎯⎯⎯⎯⎯⎯
63: √ m 5 n 3

⎯⎯⎯⎯⎯⎯⎯⎯
65: √ 250x
3

8.2 Simplifying Radical Expressions 1310


Chapter 8 Radical Expressions and Equations

⎯⎯⎯⎯⎯⎯⎯⎯
67: √ 24x 3
3

69: 2 |x|

71: 2y

73: 2 |x|

75: 6a2

77: 2 ||a3 ||

2 ⎯⎯⎯⎯
79: 3a2 b √ 2b

6 ⎯⎯⎯⎯⎯2⎯
81: 2 ||xy|| ⋅√ 2y

83: (0, 1)

85: (0, 1)

⎯⎯
87: √ 5

⎯⎯⎯⎯
89: 2√ 10

⎯⎯⎯⎯
91: 3√ 10

93: x −3

95: 2x +3

97: 30 miles per hour

99: π ≈ 3.1 seconds

101: π/4 ≈ 0.8 seconds


⎯⎯
103: √ 3 ≈ 1.7 seconds

8.2 Simplifying Radical Expressions 1311


Chapter 8 Radical Expressions and Equations

⎯⎯
105: 2√ 3 ≈ 3.5 seconds

107: f (1) = 0, f (2) = 1, and f (5) = 2

109: f (0) = 3, f (1) = 4, and f (16) = 7

111: g(−1) = −1 , g(0) = 0 , and g(1) = 1

113: g(−1) = −3 , g(0) = −2 , and g(8) = 0

115:

117:

8.2 Simplifying Radical Expressions 1312


Chapter 8 Radical Expressions and Equations

8.3 Adding and Subtracting Radical Expressions

LEARNING OBJECTIVES

1. Add and subtract like radicals.


2. Simplify radical expressions involving like radicals.

Adding and Subtracting Radical Expressions

Adding and subtracting radical expressions is similar to adding and subtracting like
terms. Radicals are considered to be like radicals16, or similar radicals17, when
⎯⎯ ⎯⎯
they share the same index and radicand. For example, the terms 3√5 and 4√5
contain like radicals and can be added using the distributive property as follows:

Typically, we do not show the step involving the distributive property and simply
write

When adding terms with like radicals, add only the coefficients; the radical part
remains the same.

⎯⎯ ⎯⎯
Example 1: Add: 3√2 + 2√2.

16. Radicals that share the same Solution: The terms contain like radicals; therefore, add the coefficients.
index and radicand.

17. Term used when referring to


like radicals.

1313
Chapter 8 Radical Expressions and Equations

⎯⎯
Answer: 5√2

Subtraction is performed in a similar manner.

⎯⎯ ⎯⎯
Example 2: Subtract: 2√7 − 3√7.

Solution:

⎯⎯
Answer: −√7

If the radicand and the index are not exactly the same, then the radicals are not
similar and we cannot combine them.

⎯⎯ ⎯⎯ ⎯⎯ ⎯⎯
Example 3: Simplify: 10√5 + 6√2 − 9√5 − 7√2.

Solution:

8.3 Adding and Subtracting Radical Expressions 1314


Chapter 8 Radical Expressions and Equations

⎯⎯ ⎯⎯
We cannot simplify any further because √5 and √2 are not like radicals; the
radicands are not the same.

⎯⎯ ⎯⎯
Answer: √5 − √2

Caution
⎯⎯ ⎯⎯ ⎯⎯⎯⎯⎯⎯⎯⎯
It is important to point out that √5 − √2 ≠ √5 − 2. We can verify this by
calculating the value of each side with a calculator.

⎯⎯ ⎯⎯ ⎯⎯⎯⎯⎯⎯⎯⎯
In general, note that √
n
a ± √b ≠ √
n
a ± b.
n

⎯⎯ ⎯⎯ ⎯⎯ ⎯⎯
Example 4: Simplify: 3 √6 + 2√6 − √6 − 3√6.
3 3

Solution:

⎯⎯ ⎯⎯
We cannot simplify any further because √6 and √6 are not like radicals; the
3

indices are not the same.

⎯⎯ ⎯⎯
Answer: 2 √6 − √6
3

8.3 Adding and Subtracting Radical Expressions 1315


Chapter 8 Radical Expressions and Equations

Often we will have to simplify before we can identify the like radicals within the
terms.

⎯⎯⎯⎯ ⎯⎯⎯⎯
Example 5: Subtract: √12 − √48.

Solution: At first glance, the radicals do not appear to be similar. However, after
simplifying completely, we will see that we can combine them.

⎯⎯
Answer: −2√3

⎯⎯⎯⎯ ⎯⎯⎯⎯ ⎯⎯ ⎯⎯⎯⎯


Example 6: Simplify: √20 + √27 − 3√5 − 2√12.

Solution:

⎯⎯ ⎯⎯
Answer: −√5 − √3

8.3 Adding and Subtracting Radical Expressions 1316


Chapter 8 Radical Expressions and Equations

⎯⎯⎯⎯ ⎯⎯
Try this! Subtract: 2√50 − 6√8.

⎯⎯
Answer: −2√2

Video Solution

(click to see video)

Next, we work with radical expressions involving variables. In this section, assume
all radicands containing variable expressions are not negative.

⎯⎯⎯⎯ ⎯⎯⎯⎯ ⎯⎯⎯⎯


Example 7: Simplify: −6 √2x − √3x + 7 √2x.
3 3 3

Solution:

We cannot combine any further because the remaining radical expressions do not
share the same radicand; they are not like radicals. Note that
3 ⎯⎯⎯⎯ 3 ⎯⎯⎯⎯ 3 ⎯⎯⎯⎯⎯⎯⎯⎯⎯⎯⎯⎯
√ 2x − √ 3x ≠ √ 2x − 3x.

⎯⎯⎯⎯ ⎯⎯⎯⎯
Answer: √2x − √3x
3 3

We will often find the need to subtract a radical expression with multiple terms. If
this is the case, remember to apply the distributive property before combining like
terms.

8.3 Adding and Subtracting Radical Expressions 1317


Chapter 8 Radical Expressions and Equations

Example 8: Simplify: (9√x − 2√y ) − (10√x + 7√y ).


⎯⎯ ⎯⎯ ⎯⎯ ⎯⎯

Solution:

⎯⎯ ⎯⎯
Answer: −√x − 9√y

Until we simplify, it is often unclear which terms involving radicals are similar.

2y − (√ 54y − √16).
⎯⎯⎯⎯ ⎯⎯⎯⎯⎯⎯ ⎯⎯⎯⎯
Example 9: Simplify: 5 √
3 3 3

Solution:

⎯⎯⎯⎯ ⎯⎯
Answer: 2 √ 2y + 2 √2
3 3

⎯⎯⎯⎯⎯⎯⎯⎯⎯⎯⎯ ⎯⎯⎯⎯⎯⎯ ⎯⎯⎯⎯⎯⎯⎯⎯⎯


Example 10: Simplify: 2a√125a2 b − a2 √80b + 4√20a4 b.

8.3 Adding and Subtracting Radical Expressions 1318


Chapter 8 Radical Expressions and Equations

Solution:

⎯⎯⎯⎯
Answer: 14a2 √5b

Try this! Simplify: √45x 3 − (√20x 3 − √80x ).


⎯⎯⎯⎯⎯⎯⎯⎯ ⎯⎯⎯⎯⎯⎯⎯⎯ ⎯⎯⎯⎯⎯⎯

⎯⎯⎯⎯ ⎯⎯⎯⎯
Answer: x√5x + 4√5x

Video Solution

(click to see video)

8.3 Adding and Subtracting Radical Expressions 1319


Chapter 8 Radical Expressions and Equations

Tip

Take careful note of the differences between products and sums within a
radical.

Products Sums
⎯⎯⎯⎯⎯⎯⎯⎯ ⎯⎯⎯⎯⎯⎯⎯⎯⎯⎯⎯⎯
√ x 2 y 2 = xy √x 2 + y 2 ≠ x + y
3 ⎯⎯⎯⎯⎯⎯⎯⎯ 3 ⎯⎯⎯⎯⎯⎯⎯⎯⎯⎯⎯⎯
√ x 3 y 3 = xy √ x 3 + y3 ≠ x + y

⎯⎯⎯⎯⎯⎯⎯ ⎯⎯ ⎯⎯
The property √a ⋅ b = √ a ⋅ √b says that we can simplify radicals when the
n n n

operation in the radicand is multiplication. There is no corresponding property


for addition.

KEY TAKEAWAYS

• Add and subtract terms that contain like radicals just as you do like
terms. If the index and radicand are exactly the same, then the radicals
are similar and can be combined. This involves adding or subtracting
only the coefficients; the radical part remains the same.
• Simplify each radical completely before combining like terms.

8.3 Adding and Subtracting Radical Expressions 1320


Chapter 8 Radical Expressions and Equations

TOPIC EXERCISES

Part A: Adding and Subtracting Like Radicals

Simplify.

⎯⎯ ⎯⎯
1. 9√ 3 + 5√ 3
⎯⎯ ⎯⎯
2. 12√ 6 + 3√ 6
⎯⎯ ⎯⎯
3. 4√ 5 − 7√ 5
⎯⎯⎯⎯ ⎯⎯⎯⎯
4. 3√ 10 − 8√ 10
⎯⎯ ⎯⎯ ⎯⎯
5. √ 6 − 4√ 6 + 2√ 6
⎯⎯⎯⎯ ⎯⎯⎯⎯ ⎯⎯⎯⎯
6. 5√ 10 − 15√ 10 − 2√ 10
⎯⎯ ⎯⎯ ⎯⎯ ⎯⎯
7. 13√ 7 − 6√ 2 − 5√ 7 + 5√ 2
⎯⎯⎯⎯ ⎯⎯⎯⎯ ⎯⎯⎯⎯ ⎯⎯⎯⎯
8. 10√ 13 − 12√ 15 + 5√ 13 − 18√ 15

− (4√ 3 − 3√ 5 )
⎯⎯ ⎯⎯ ⎯⎯
9. 6√ 5

− (6√ 6 + √ 2 )
⎯⎯ ⎯⎯ ⎯⎯
10. −12√ 2

( ) ( 5)
⎯⎯ ⎯⎯⎯⎯ ⎯⎯⎯⎯ ⎯⎯
11. 2 √ 5 − 3 √ 10 − √ 10 + 3 √

( ) ( 15 )
⎯⎯ ⎯⎯⎯⎯ ⎯⎯ ⎯⎯⎯⎯
12. −8 √ 3 + 6 √ 15 − √ 3 − √

⎯⎯ 3 ⎯⎯ 3 ⎯⎯
13. 4 √ 6 − 3√ 5 + 6√ 6
3

⎯⎯⎯⎯ 3 ⎯⎯⎯⎯ 3 ⎯⎯⎯⎯


14. √ 10 + 5√ 10 − 4 √ 10
3

8.3 Adding and Subtracting Radical Expressions 1321


Chapter 8 Radical Expressions and Equations

(7 √ 9 − 4 √ 3 ) − (√ 9 − 3 √ 3 )
3 ⎯⎯ 3 ⎯⎯ 3 ⎯⎯ 3 ⎯⎯
15.

( ) ( 25 )
3 ⎯⎯ 3 ⎯⎯⎯⎯ 3 ⎯⎯ 3 ⎯⎯⎯⎯
16. −8 √ 5 + √ 25 − 2 √ 5 + 6 √

Simplify. (Assume all radicands containing variable expressions are positive.)

⎯⎯ ⎯⎯
17. 9√ x + 7√ x

18. −8√ y
⎯⎯ + 4 ⎯⎯
√y

19. 7x√ y
⎯⎯ − 3x ⎯⎯ ⎯⎯
√ y + x√ y
⎯⎯ ⎯⎯ ⎯⎯
20. 10y 2 √ x − 12y 2 √ x − 2y 2 √ x
⎯⎯⎯⎯ ⎯⎯ ⎯⎯⎯⎯ ⎯⎯
21. 2√ ab − 5√ a + 6√ ab − 10√ a

22. −3x√ y
⎯⎯ + 6 ⎯⎯ ⎯⎯ ⎯⎯
√ y − 4x√ y − 7√ y

(3√ xy − 7√ xy )
23. 5√ xy
⎯⎯⎯⎯ − ⎯⎯⎯⎯ ⎯⎯⎯⎯

− (2a√ b − 4√ ab)
⎯⎯ ⎯⎯ ⎯⎯⎯⎯
24. −8a√ b

( ) ( 3x )
⎯⎯⎯⎯ ⎯⎯⎯⎯ ⎯⎯⎯⎯ ⎯⎯⎯⎯
25. 3 √ 2x − √ 3x − √ 2x − 7 √

( ) ( 2y )
⎯⎯ ⎯⎯⎯⎯ ⎯⎯ ⎯⎯⎯⎯
26. √ y − 4 √ 2y − √ y − 5 √

⎯⎯ 3 ⎯⎯
27. 5 √
3
x − 12 √ x

28. −2 √ y ⎯⎯ − 3 3 ⎯⎯
y
3

5 ⎯⎯⎯⎯ 5 ⎯⎯⎯⎯ 5 ⎯⎯⎯⎯


29. a ⋅√ 3b + 4a ⋅ √ 3b − a ⋅ √ 3b

8.3 Adding and Subtracting Radical Expressions 1322


Chapter 8 Radical Expressions and Equations

⎯⎯⎯⎯ 4 ⎯⎯⎯⎯ 4 ⎯⎯⎯⎯


30. −8 √ ab + 3√ ab − 2 √ ab
4

⎯⎯⎯⎯ 3 ⎯⎯⎯⎯ ⎯⎯⎯⎯ 3 ⎯⎯⎯⎯


31. 6√ 2a − 4√ 2a + 7√ 2a − √ 2a
⎯⎯⎯⎯ 3 ⎯⎯⎯⎯ 5 ⎯⎯⎯⎯ 3 ⎯⎯⎯⎯
32. 4 √ 3a +√ 3a − 9 √ 3a + √ 3a
5

(√ 4xy − √ xy ) − (2 √ 4xy − √ xy )
4 ⎯⎯⎯⎯⎯⎯ 3 ⎯⎯⎯⎯ 4 ⎯⎯⎯⎯⎯⎯ 3 ⎯⎯⎯⎯
33.

(5 √ 6y − 5√ y ) − (2 √ 6y + 3√ y )
6 ⎯⎯⎯⎯ ⎯⎯ 6 ⎯⎯⎯⎯ ⎯⎯
34.

Part B: Adding and Subtracting Rational Expressions

Simplify.

⎯⎯⎯⎯ ⎯⎯⎯⎯
35. √ 75 − √ 12
⎯⎯⎯⎯ ⎯⎯⎯⎯
36. √ 24 − √ 54
⎯⎯⎯⎯ ⎯⎯⎯⎯ ⎯⎯
37. √ 32 + √ 27 − √ 8
⎯⎯⎯⎯ ⎯⎯⎯⎯ ⎯⎯⎯⎯
38. √ 20 + √ 48 − √ 45
⎯⎯⎯⎯ ⎯⎯⎯⎯ ⎯⎯⎯⎯ ⎯⎯⎯⎯
39. √ 28 − √ 27 + √ 63 − √ 12
⎯⎯⎯⎯ ⎯⎯⎯⎯ ⎯⎯⎯⎯ ⎯⎯⎯⎯
40. √ 90 + √ 24 − √ 40 − √ 54
⎯⎯⎯⎯ ⎯⎯⎯⎯ ⎯⎯⎯⎯⎯⎯ ⎯⎯
41. √ 45 − √ 80 + √ 245 − √ 5
⎯⎯⎯⎯⎯⎯ ⎯⎯⎯⎯ ⎯⎯⎯⎯ ⎯⎯
42. √ 108 + √ 48 − √ 75 − √ 3

− (√ 27 − √ 72 )
⎯⎯ ⎯⎯⎯⎯ ⎯⎯⎯⎯
43. 4√ 2

− (√ 20 − √ 50 )
⎯⎯ ⎯⎯⎯⎯ ⎯⎯⎯⎯
44. −3√ 5

8.3 Adding and Subtracting Radical Expressions 1323


Chapter 8 Radical Expressions and Equations

⎯⎯⎯⎯ 3 ⎯⎯⎯⎯
45. √ 16 −√ 54
3

⎯⎯⎯⎯ 3 ⎯⎯⎯⎯
46. √ 81 −√ 24
3

⎯⎯⎯⎯⎯⎯ 3 ⎯⎯⎯⎯ 3 ⎯⎯
47. √ 135 +√ 40 − √ 5
3

⎯⎯⎯⎯⎯⎯ 3 ⎯⎯⎯⎯ 3 ⎯⎯
48. √ 108 −√ 32 − √ 4
3

⎯⎯⎯⎯ ⎯⎯⎯⎯
49. 2√ 27 − 2√ 12
⎯⎯⎯⎯ ⎯⎯⎯⎯
50. 3√ 50 − 4√ 32
⎯⎯⎯⎯⎯⎯ ⎯⎯⎯⎯ ⎯⎯⎯⎯
51. 3√ 243 − 2√ 18 − √ 48
⎯⎯⎯⎯⎯⎯ ⎯⎯⎯⎯ ⎯⎯⎯⎯
52. 6√ 216 − 2√ 24 − 2√ 96
⎯⎯⎯⎯ ⎯⎯⎯⎯ ⎯⎯⎯⎯ ⎯⎯⎯⎯
53. 2√ 18 − 3√ 75 − 2√ 98 + 4√ 48
⎯⎯⎯⎯ ⎯⎯⎯⎯ ⎯⎯⎯⎯ ⎯⎯⎯⎯⎯⎯
54. 2√ 45 − √ 12 + 2√ 20 − √ 108

( ) ( 54 )
⎯⎯⎯⎯⎯⎯ ⎯⎯⎯⎯ ⎯⎯⎯⎯ ⎯⎯⎯⎯
55. 2 √ 363 − 3 √ 96 − 7 √ 12 − 2 √

( ) ( 40 )
⎯⎯⎯⎯⎯⎯ ⎯⎯⎯⎯⎯⎯ ⎯⎯⎯⎯ ⎯⎯⎯⎯
56. 2 √ 288 + 3 √ 360 − 2 √ 72 − 7 √

⎯⎯⎯⎯ 3 ⎯⎯⎯⎯⎯⎯ 3 ⎯⎯⎯⎯


57. 3 √ 54 + 5√ 250 − 4 √ 16
3

⎯⎯⎯⎯⎯⎯ 3 ⎯⎯⎯⎯⎯⎯ 3 ⎯⎯⎯⎯⎯⎯


58. 4 √ 162 − 2√ 384 − 3 √ 750
3

Simplify. (Assume all radicands containing variable expressions are positive.)

⎯⎯⎯⎯⎯⎯ ⎯⎯⎯⎯
59. √ 81b + √ 4b
⎯⎯⎯⎯⎯⎯⎯⎯ ⎯⎯
60. √ 100a + √a

8.3 Adding and Subtracting Radical Expressions 1324


Chapter 8 Radical Expressions and Equations

⎯⎯⎯⎯⎯⎯⎯⎯ ⎯⎯⎯⎯⎯⎯⎯⎯⎯⎯
61. √ 9a2 b − √ 36a2 b
⎯⎯⎯⎯⎯⎯⎯⎯ ⎯⎯⎯⎯⎯⎯⎯⎯
62. √ 50a2 − √ 18a2
⎯⎯⎯⎯⎯⎯ ⎯⎯⎯⎯ ⎯⎯ ⎯⎯⎯⎯
63. √ 49x − √ 9y + √ x − √ 4y
⎯⎯⎯⎯ ⎯⎯⎯⎯⎯⎯ ⎯⎯⎯⎯⎯⎯
64. √ 9x + √ 64y − √ 25x − √ ⎯⎯
y

− (3√ 16y − 2√ 18x )


⎯⎯⎯⎯ ⎯⎯⎯⎯⎯⎯ ⎯⎯⎯⎯⎯⎯
65. 7√ 8x

− (3√ 32y − √ 81y )


⎯⎯⎯⎯⎯⎯ ⎯⎯⎯⎯⎯⎯ ⎯⎯⎯⎯⎯⎯
66. 2√ 64y

⎯⎯⎯⎯⎯⎯⎯⎯⎯ ⎯⎯⎯⎯ ⎯⎯⎯⎯⎯⎯⎯


67. 2√ 9m 2 n − 5m√ 9n + √ m 2 n
⎯⎯⎯⎯⎯⎯⎯⎯⎯⎯⎯ ⎯⎯⎯⎯⎯ ⎯⎯⎯⎯⎯
68. 4√ 18n 2 m − 2n√ 8m + n√ 2m
⎯⎯⎯⎯⎯⎯⎯⎯ ⎯⎯⎯⎯⎯⎯⎯⎯ ⎯⎯⎯⎯⎯⎯⎯⎯⎯⎯ ⎯⎯⎯⎯⎯⎯
69. √ 4x 2 y − √ 9xy 2 − √ 16x 2 y + √ y 2 x
⎯⎯⎯⎯⎯⎯⎯⎯⎯⎯⎯⎯ ⎯⎯⎯⎯⎯⎯⎯⎯⎯⎯ ⎯⎯⎯⎯⎯⎯⎯⎯⎯⎯⎯⎯ ⎯⎯⎯⎯⎯⎯⎯⎯⎯⎯
70. √ 32x 2 y 2 + √ 12x 2 y − √ 18x 2 y 2 − √ 27x 2 y

71. (√ 9x 2 y − √ 16y ) − (√ 49x 2 y − 4√ ⎯⎯


y)
⎯⎯⎯⎯⎯⎯⎯⎯ ⎯⎯⎯⎯⎯⎯ ⎯⎯⎯⎯⎯⎯⎯⎯⎯⎯

72. (√ 72x 2 y 2 − √ 18x 2 y ) − (√ 50x 2 y 2 + x√ 2y )


⎯⎯⎯⎯⎯⎯⎯⎯⎯⎯⎯⎯ ⎯⎯⎯⎯⎯⎯⎯⎯⎯⎯ ⎯⎯⎯⎯⎯⎯⎯⎯⎯⎯⎯⎯ ⎯⎯⎯⎯

⎯⎯⎯⎯⎯⎯⎯⎯⎯⎯⎯ ⎯⎯⎯⎯⎯⎯⎯⎯⎯⎯⎯ ⎯⎯⎯⎯⎯⎯⎯⎯⎯⎯⎯


73. √ 12m 4 n − m√ 75m 2 n + 2√ 27m 4 n
⎯⎯⎯⎯⎯⎯⎯⎯⎯⎯⎯ ⎯⎯⎯⎯⎯⎯⎯⎯⎯⎯⎯ ⎯⎯⎯⎯⎯⎯⎯⎯⎯
74. 5n√ 27mn 2 + 2√ 12mn 4 − n√ 3mn 2
⎯⎯⎯⎯⎯⎯⎯⎯⎯⎯ ⎯⎯⎯⎯⎯⎯⎯⎯ ⎯⎯⎯⎯⎯⎯⎯⎯⎯⎯⎯⎯
75. 2√ 27a3 b − a√ 48ab − a√ 144a3 b
⎯⎯⎯⎯⎯⎯⎯⎯⎯⎯ ⎯⎯⎯⎯⎯⎯⎯⎯⎯⎯⎯⎯ ⎯⎯⎯⎯⎯⎯⎯⎯
76. 2√ 98a4 b − 2a√ 162a2 b + a√ 200b
⎯⎯⎯⎯⎯⎯⎯⎯ 3 ⎯⎯⎯⎯⎯⎯
77. √ 125a −√ 27a
3

8.3 Adding and Subtracting Radical Expressions 1325


Chapter 8 Radical Expressions and Equations

⎯⎯⎯⎯⎯⎯⎯⎯⎯⎯⎯⎯ 3 ⎯⎯⎯⎯⎯⎯⎯⎯
78. √ 1000a2 − √ 64a2
3

3 ⎯⎯⎯⎯⎯⎯ 3 ⎯⎯⎯⎯⎯⎯⎯⎯ 3 ⎯⎯⎯⎯⎯⎯


79. 2x ⋅√ 54x − 2 √ 16x 4 + 5 √ 2x 4

3 ⎯⎯⎯⎯⎯⎯⎯⎯ 3 ⎯⎯⎯⎯⎯⎯⎯⎯⎯⎯ 3 ⎯⎯
80. x ⋅ √ 54x 3 − √ 250x 6 + x 2 ⋅ √ 2
⎯⎯⎯⎯⎯⎯⎯⎯ 4 ⎯⎯⎯⎯⎯⎯⎯⎯
81. √ 16y 2 +√ 81y 2
4

⎯⎯⎯⎯⎯⎯⎯⎯ 5 ⎯⎯⎯4⎯
82. √ 32y 4 −√ y
5

⎯⎯⎯⎯⎯⎯⎯⎯ 4 ⎯⎯⎯⎯⎯⎯⎯⎯⎯⎯ 4 ⎯⎯⎯⎯⎯⎯


83. √ 32a3 − √ 162a3 + 5 √ 2a3
4

⎯⎯⎯⎯⎯⎯⎯⎯⎯⎯ 4 ⎯⎯⎯⎯⎯⎯⎯⎯ 4 ⎯⎯⎯⎯


84. √ 80a4 b + √ 5a4 b − a ⋅ √
4
5b
⎯⎯⎯⎯⎯⎯⎯⎯ 3 ⎯⎯⎯⎯ 3 ⎯⎯⎯⎯⎯⎯⎯⎯⎯⎯
85. √ 27x 3 8x − √ 125x 3
3
+√
⎯⎯⎯⎯⎯⎯ 3 ⎯⎯⎯⎯⎯⎯⎯⎯ 3 ⎯⎯⎯⎯⎯⎯
86. √ 24x −√ 128x − √ 81x
3

⎯⎯⎯⎯⎯⎯⎯⎯⎯⎯ 3 ⎯⎯⎯⎯⎯⎯⎯⎯ 3 ⎯⎯⎯⎯⎯⎯⎯⎯ 3 ⎯⎯


87. √ 27x 4 y −√ 8xy 3 + x ⋅ √ 64xy − y ⋅ √ x
3

⎯⎯⎯⎯⎯⎯⎯⎯⎯⎯⎯⎯ 3 ⎯⎯⎯⎯⎯⎯⎯⎯ 3 ⎯⎯⎯⎯⎯⎯⎯⎯⎯⎯⎯⎯ 3 ⎯⎯


88. √ 125xy 3 +√ 8x 3 y − √ 216xy 3 + 10x ⋅ √ y
3

89. (√ 162x 4 y 250x 4 y 2 ) − (√ 384x 4 y )


⎯⎯⎯⎯⎯⎯⎯⎯⎯⎯⎯⎯ 3 ⎯⎯⎯⎯⎯⎯⎯⎯⎯⎯⎯⎯⎯⎯ 3 ⎯⎯⎯⎯⎯⎯⎯⎯⎯⎯ 3 ⎯⎯⎯⎯⎯⎯⎯⎯⎯⎯⎯⎯
3
−√ 2x 4 y 2 − √

90. (√ 32x 2 y 6 243x 6 y 2 ) − (√ xy 2 )


⎯⎯⎯⎯⎯⎯⎯⎯⎯⎯⎯⎯ 5 ⎯⎯⎯⎯⎯⎯⎯⎯⎯⎯⎯⎯⎯⎯ 5 ⎯⎯⎯⎯⎯⎯⎯⎯ 5 ⎯⎯⎯⎯⎯⎯
5
−√ x 2y6 − x ⋅ √

Part C: Discussion Board

91. Choose values for x and y and use a calculator to show that
⎯⎯⎯⎯⎯⎯⎯⎯⎯ ⎯⎯ ⎯⎯
√ x + y ≠ √ x + √ y.

92. Choose values for x and y and use a calculator to show that
⎯⎯⎯⎯⎯⎯⎯⎯⎯⎯⎯⎯
√x 2 + y 2 ≠ x + y .

8.3 Adding and Subtracting Radical Expressions 1326


Chapter 8 Radical Expressions and Equations

ANSWERS

⎯⎯
1: 14√ 3

⎯⎯
3: −2√ 5

⎯⎯
5: −√ 6

⎯⎯ ⎯⎯
7: 8√ 7 − √2
⎯⎯ ⎯⎯
9: 9√ 5 − 4√ 3
⎯⎯ ⎯⎯⎯⎯
11: −√ 5 − 4√ 10
⎯⎯ 3 ⎯⎯
13: 10 √ 6 − 3√ 5
3

⎯⎯ 3 ⎯⎯
15: 6 √ 9 −√ 3
3

⎯⎯
17: 16√ x

19: 5x√ y
⎯⎯

⎯⎯⎯⎯ ⎯⎯
21: 8√ ab − 15√ a

23: 9√ xy
⎯⎯⎯⎯

⎯⎯⎯⎯ ⎯⎯⎯⎯
25: 2√ 2x + 6√ 3x
⎯⎯
27: −7 √
3
x

5 ⎯⎯⎯⎯
29: 4a ⋅√ 3b
⎯⎯⎯⎯ 3 ⎯⎯⎯⎯
31: 13√ 2a − 5√ 2a

8.3 Adding and Subtracting Radical Expressions 1327


Chapter 8 Radical Expressions and Equations

⎯⎯⎯⎯⎯⎯
33: −√
4
4xy
⎯⎯
35: 3√ 3

⎯⎯ ⎯⎯
37: 2√ 2 + 3√ 3
⎯⎯ ⎯⎯
39: 5√ 7 − 5√ 3
⎯⎯
41: 5√ 5

⎯⎯ ⎯⎯
43: 10√ 2 − 3√ 3
⎯⎯
45: −√ 2
3

⎯⎯
47: 4 √ 5
3

⎯⎯
49: 2√ 3

⎯⎯ ⎯⎯
51: 23√ 3 − 6√ 2
⎯⎯ ⎯⎯
53: −8√ 2 + √3
⎯⎯ ⎯⎯
55: 8√ 3 − 6√ 6
⎯⎯
57: 26 √ 2
3

⎯⎯
59: 11√ b

⎯⎯
61: −3a√ b

⎯⎯
63: 8√ x − 5√ ⎯⎯
y

⎯⎯⎯⎯
65: 20√ 2x − 12√ ⎯⎯
y

8.3 Adding and Subtracting Radical Expressions 1328


Chapter 8 Radical Expressions and Equations

⎯⎯
67: −8m√ n

69: −2x√ y
⎯⎯ − 2y ⎯⎯
√x

71: −4x√ y
⎯⎯

⎯⎯⎯⎯
73: 3m 2 √ 3n

⎯⎯⎯⎯⎯⎯ ⎯⎯⎯⎯
75: 2a√ 3ab − 12a2 √ ab
⎯⎯
77: 2 √
3
a

3 ⎯⎯⎯⎯
79: 7x ⋅√ 2x
⎯⎯⎯⎯
81: 5 √ y 2
4

⎯⎯⎯⎯⎯⎯
83: 4 √ 2a3
4

3 ⎯⎯
85: −2x + 2√ x

3 ⎯⎯⎯⎯ 3 ⎯⎯
87: 7x ⋅√ xy − 3y ⋅ √ x

3 ⎯⎯⎯⎯⎯⎯ 3 ⎯⎯⎯⎯⎯⎯⎯⎯
89: 7x ⋅√ 6xy − 6x ⋅ √ 2xy 2

8.3 Adding and Subtracting Radical Expressions 1329


Chapter 8 Radical Expressions and Equations

8.4 Multiplying and Dividing Radical Expressions

LEARNING OBJECTIVES

1. Multiply radical expressions.


2. Divide radical expressions.
3. Rationalize the denominator.

Multiplying Radical Expressions

When multiplying radical expressions with the same index, we use the product rule
for radicals. If a and b represent positive real numbers,

⎯⎯ ⎯⎯
Example 1: Multiply: √2 ⋅ √6.

Solution: This problem is a product of two square roots. Apply the product rule for
radicals and then simplify.

⎯⎯
Answer: 2√3

1330
Chapter 8 Radical Expressions and Equations

⎯⎯ ⎯⎯
Example 2: Multiply: √9 ⋅ √6.
3 3

Solution: This problem is a product of cube roots. Apply the product rule for
radicals and then simplify.

⎯⎯
Answer: 3 √2
3

Often there will be coefficients in front of the radicals.

⎯⎯ ⎯⎯
Example 3: Multiply: 2√3 ⋅ 5√2.

Solution: Using the product rule for radicals and the fact that multiplication is
commutative, we can multiply the coefficients and the radicands as follows.

Typically, the first step involving the application of the commutative property is
not shown.

⎯⎯
Answer: 10√6

8.4 Multiplying and Dividing Radical Expressions 1331


Chapter 8 Radical Expressions and Equations

⎯⎯⎯⎯ ⎯⎯⎯⎯⎯⎯⎯⎯
Example 4: Multiply: −2 √5x ⋅ 3 √25x 2 .
3 3

Solution:

Answer: −30x

Use the distributive property when multiplying rational expressions with more
than one term.

Example 5: Multiply: 4√3 (2√3 − 3√6).


⎯⎯ ⎯⎯ ⎯⎯

⎯⎯
Solution: Apply the distributive property and multiply each term by 4√3.

8.4 Multiplying and Dividing Radical Expressions 1332


Chapter 8 Radical Expressions and Equations

⎯⎯
Answer: 24 − 36√2

Example 6: Multiply: √4x 2 (√2x − 5 √4x 2 ).


3 ⎯⎯⎯⎯⎯⎯ 3 ⎯⎯⎯⎯ 3 ⎯⎯⎯⎯⎯⎯

Solution: Apply the distributive property and then simplify the result.

⎯⎯⎯⎯
Answer: 2x − 10x ⋅ √2x
3

The process for multiplying radical expressions with multiple terms is the same
process used when multiplying polynomials. Apply the distributive property,
simplify each radical, and then combine like terms.

Example 7: Multiply: (√5 + 2) (√5 − 4).


⎯⎯ ⎯⎯

Solution: Begin by applying the distributive property.

8.4 Multiplying and Dividing Radical Expressions 1333


Chapter 8 Radical Expressions and Equations

⎯⎯
Answer: −3 − 2√5

Example 8: Multiply: (3√x − √y ) .


⎯⎯ 2
⎯⎯

Solution:

⎯⎯⎯⎯
Answer: 9x − 6√xy + y

8.4 Multiplying and Dividing Radical Expressions 1334


Chapter 8 Radical Expressions and Equations

Try this! Multiply: (2√3 + 5√2) (√3 − 2√6).


⎯⎯ ⎯⎯ ⎯⎯ ⎯⎯

⎯⎯ ⎯⎯ ⎯⎯
Answer: 6 − 12√2 + 5√6 − 20√3

Video Solution

(click to see video)

The expressions (a + b) and (a − b) are called conjugates18. When multiplying


conjugates, the sum of the products of the inner and outer terms results in 0.

Example 9: Multiply: (√2 + √5) (√2 − √5).


⎯⎯ ⎯⎯ ⎯⎯ ⎯⎯

Solution: Apply the distributive property and then combine like terms.

Answer: −3

It is important to note that when multiplying conjugate radical expressions, we


obtain a rational expression. This is true in general and is often used in our study of
algebra.

18. The factors (a + b) and


(a − b) are conjugates.

8.4 Multiplying and Dividing Radical Expressions 1335


Chapter 8 Radical Expressions and Equations

Therefore, for nonnegative real numbers a and b, we have the following property:

Dividing Radical Expressions (Rationalizing the Denominator)

To divide radical expressions with the same index, we use the quotient rule for
radicals. If a and b represent nonnegative numbers, where b ≠ 0, then we have

√80
Example 10: Divide: .
√10

Solution: In this case, we can see that 10 and 80 have common factors. If we apply
the quotient rule for radicals and write it as a single square root, we will be able to
reduce the fractional radicand.

8.4 Multiplying and Dividing Radical Expressions 1336


Chapter 8 Radical Expressions and Equations

⎯⎯
Answer: 2√2

√16x 5 y 4
Example 11: Divide: .
√2xy

Solution:

⎯⎯⎯⎯
Answer: 2x 2 y√2y

3
√ 54a3 b5
Example 12: Divide: 3
.
√ 16a2 b2

Solution:

8.4 Multiplying and Dividing Radical Expressions 1337


Chapter 8 Radical Expressions and Equations

3b⋅√3a
Answer: 2

When the divisor of a radical expression contains a radical, it is a common practice


to find an equivalent expression where the denominator is a rational number.
Finding such an equivalent expression is called rationalizing the denominator19.

To do this, multiply the fraction by a special form of 1 so that the radicand in the
denominator can be written with a power that matches the index. After doing this,
simplify and eliminate the radical in the denominator. For example,

19. The process of determining an


equivalent radical expression
with a rational denominator.

8.4 Multiplying and Dividing Radical Expressions 1338


Chapter 8 Radical Expressions and Equations

Remember, to obtain an equivalent expression, you must multiply the numerator


and denominator by the exact same nonzero factor.

√3
Example 13: Rationalize the denominator: .
√2

Solution: The goal is to find an equivalent expression without a radical in the


√2
denominator. In this example, multiply by 1 in the form .
√2

√6
Answer: 2

1
Example 14: Rationalize the denominator: .
2√3x

Solution: The radicand in the denominator determines the factors that you need to
√3x
use to rationalize it. In this example, multiply by 1 in the form .
√3x

8.4 Multiplying and Dividing Radical Expressions 1339


Chapter 8 Radical Expressions and Equations

√3x
Answer: 6x

Typically, we will find the need to reduce, or cancel, after rationalizing the
denominator.

5√2
Example 15: Rationalize the denominator: .
√5ab

√5ab
Solution: In this example, we will multiply by 1 in the form .
√5ab

8.4 Multiplying and Dividing Radical Expressions 1340


Chapter 8 Radical Expressions and Equations

Notice that a and b do not cancel in this example. Do not cancel factors inside a
radical with those that are outside.

√10ab
Answer: ab

Try this! Rationalize the denominator: √ 3b


4a
⎯⎯⎯⎯
.

2√3ab
Answer: 3b

Video Solution

(click to see video)

Up to this point, we have seen that multiplying a numerator and a denominator by


a square root with the exact same radicand results in a rational denominator. In
general, this is true only when the denominator contains a square root. However,
this is not the case for a cube root. For example,

Note that multiplying by the same factor in the denominator does not rationalize it.
√x 2
3
In this case, if we multiply by 1 in the form of , then we can write the radicand in
√x 2
3

the denominator as a power of 3. Simplifying the result then yields a rationalized


denominator. For example,

8.4 Multiplying and Dividing Radical Expressions 1341


Chapter 8 Radical Expressions and Equations

Therefore, to rationalize the denominator of radical expressions with one radical


term in the denominator, begin by factoring the radicand of the denominator. The
factors of this radicand and the index determine what we should multiply by.
Multiply numerator and denominator by the nth root of factors that produce nth
powers of all the factors in the radicand of the denominator.

1
Example 16: Rationalize the denominator: 3
.
√25

⎯⎯⎯⎯
Solution: The radical in the denominator is equivalent to √52 . To rationalize the
3

⎯⎯⎯⎯
denominator, it should be √53 . To obtain this, we need one more factor of 5.
3

35

Therefore, multiply by 1 in the form of 35
.

3
√ 5
Answer: 5

√ 2b2 .
⎯27a
⎯⎯⎯⎯⎯
3
Example 17: Rationalize the denominator:

√22 b
3
Solution: In this example, we will multiply by 1 in the form .
√22 b
3

8.4 Multiplying and Dividing Radical Expressions 1342


Chapter 8 Radical Expressions and Equations

3
3√ 4ab
Answer: 2b

1
Example 18: Rationalize the denominator: 5
.
√ 4x 3

√23 x 2
5
Solution: In this example, we will multiply by 1 in the form .
√23 x 2
5

5
√ 8x 2
Answer: 2x

When two terms involving square roots appear in the denominator, we can
rationalize it using a very special technique. This technique involves multiplying

8.4 Multiplying and Dividing Radical Expressions 1343


Chapter 8 Radical Expressions and Equations

the numerator and the denominator of the fraction by the conjugate of the
denominator. Recall that multiplying a radical expression by its conjugate produces
a rational number.

1
Example 19: Rationalize the denominator: .
√ √2
3 −

⎯⎯ ⎯⎯
Solution: In this example, the conjugate of the denominator is √3 + √2.
(√3+√2)
(√3+√2)
Therefore, multiply by 1 in the form .

⎯⎯ ⎯⎯
Answer: √3 + √2

Notice that the terms involving the square root in the denominator are eliminated
by multiplying by the conjugate. We can use the property

(√a + √b) (√a − √b) = a − bto expedite the process of multiplying the
⎯⎯ ⎯⎯ ⎯⎯ ⎯⎯

expressions in the denominator.

8.4 Multiplying and Dividing Radical Expressions 1344


Chapter 8 Radical Expressions and Equations

√2−√6
Example 20: Rationalize the denominator: .
√2+√6

√2−√6
Solution: Multiply by 1 in the form .
√2−√6

⎯⎯
Answer: −2 + √3

√x +√y
Example 21: Rationalize the denominator: .
√x −√y

√x −√y
Solution: In this example, we will multiply by 1 in the form .
√x −√y

8.4 Multiplying and Dividing Radical Expressions 1345


Chapter 8 Radical Expressions and Equations

x−2√xy +y
Answer: x−y

3√5+5
Try this! Rationalize the denominator: .
2√5−3

19√5+45
Answer: 11

Video Solution

(click to see video)

8.4 Multiplying and Dividing Radical Expressions 1346


Chapter 8 Radical Expressions and Equations

KEY TAKEAWAYS

• To multiply two single-term radical expressions, multiply the


coefficients and multiply the radicands. If possible, simplify the result.
• Apply the distributive property when multiplying radical expressions
with multiple terms. Then simplify and combine all like radicals.
• Multiplying a two-term radical expression involving square roots by its
conjugate results in a rational expression.
• It is common practice to write radical expressions without radicals in
the denominator. The process of finding such an equivalent expression
is called rationalizing the denominator.
• If an expression has one term in the denominator involving a radical,
then rationalize it by multiplying numerator and denominator by the
nth root of factors of the radicand so that their powers equal the index.
• If a radical expression has two terms in the denominator involving
square roots, then rationalize it by multiplying the numerator and
denominator by its conjugate.

8.4 Multiplying and Dividing Radical Expressions 1347


Chapter 8 Radical Expressions and Equations

TOPIC EXERCISES

Part A: Multiplying Radical Expressions

Multiply. (Assume all variables are nonnegative.)

⎯⎯ ⎯⎯
1. √ 3 ⋅ √5
⎯⎯ ⎯⎯
2. √ 7 ⋅ √3
⎯⎯ ⎯⎯
3. √ 2 ⋅ √6
⎯⎯ ⎯⎯⎯⎯
4. √ 5 ⋅ √ 15
⎯⎯ ⎯⎯
5. √ 7 ⋅ √7
⎯⎯⎯⎯ ⎯⎯⎯⎯
6. √ 12 ⋅ √ 12
⎯⎯ ⎯⎯⎯⎯
7. 2√ 5 ⋅ 7√ 10
⎯⎯⎯⎯ ⎯⎯
8. 3√ 15 ⋅ 2√ 6

( 5)
⎯⎯ 2
9. 2 √

( )
⎯⎯ 2
10. 6√ 2

⎯⎯⎯⎯ ⎯⎯⎯⎯
11. √ 2x ⋅ √ 2x
⎯⎯⎯⎯ ⎯⎯⎯⎯
12. √ 5y ⋅ √ 5y
⎯⎯⎯⎯ ⎯⎯⎯⎯
13. √ 3a ⋅ √ 12
⎯⎯⎯⎯ ⎯⎯⎯⎯
14. √ 3a ⋅ √ 2a

8.4 Multiplying and Dividing Radical Expressions 1348


Chapter 8 Radical Expressions and Equations

⎯⎯⎯⎯ ⎯⎯⎯⎯
15. 4√ 2x ⋅ 3√ 6x
⎯⎯⎯⎯⎯⎯ ⎯⎯⎯⎯
16. 5√ 10y ⋅ 2√ 2y
⎯⎯ 3 ⎯⎯⎯⎯
17. √ 5 ⋅√ 25
3

⎯⎯ 3 ⎯⎯
18. √ 4 ⋅√ 2
3

⎯⎯ 3 ⎯⎯⎯⎯
19. √ 4 ⋅√ 10
3

⎯⎯⎯⎯ 3 ⎯⎯
20. √ 18 ⋅√ 6
3

( )( 6)
3 ⎯⎯ 3 ⎯⎯
21. 5 √ 9 2 √

( )( 4)
3 ⎯⎯ 3 ⎯⎯
22. 2 √ 4 3 √

( )
3 ⎯⎯
3
23. 2 √ 2

( )
⎯⎯ 3
24. 3 √ 4
3

⎯⎯⎯⎯⎯⎯ 3 ⎯⎯⎯⎯
25. √ 3a2
3
⋅√ 9a
⎯⎯⎯⎯ 3 ⎯⎯⎯⎯⎯⎯⎯⎯
26. √ 7b
3
⋅ √ 49b 2
⎯⎯⎯⎯⎯⎯ 3 ⎯⎯⎯⎯⎯⎯
27. √ 6x 2 ⋅ √ 4x 2
3

⎯⎯⎯⎯⎯⎯ 3 ⎯⎯⎯⎯⎯2⎯
28. √
3
12y ⋅√ 9y
⎯⎯⎯⎯⎯⎯⎯⎯⎯⎯ 3 ⎯⎯⎯⎯⎯⎯⎯⎯⎯⎯⎯⎯
29. √ 20x 2 y ⋅√ 10x 2 y 2
3

⎯⎯⎯⎯⎯⎯⎯⎯ 3 ⎯⎯⎯⎯⎯⎯⎯⎯⎯⎯⎯⎯
30. √
3
63xy ⋅√ 12x 4 y 2

8.4 Multiplying and Dividing Radical Expressions 1349


Chapter 8 Radical Expressions and Equations

(3 − √ 5 )
⎯⎯ ⎯⎯
31. √ 5

( 2)
⎯⎯ ⎯⎯ ⎯⎯
32. √ 2 √ 3 − √

( 3)
⎯⎯ ⎯⎯ ⎯⎯
33. 3√ 7 2 √ 7 − √

( 10 )
⎯⎯ ⎯⎯⎯⎯
34. 2√ 5 6 − 3√

( 2)
⎯⎯ ⎯⎯ ⎯⎯
35. √ 6 √ 3 − √

( 3)
⎯⎯⎯⎯ ⎯⎯ ⎯⎯
36. √ 15 √ 5 + √

(√ x + √ xy )
⎯⎯ ⎯⎯ ⎯⎯⎯⎯
37. √ x

(√ xy + √ y )
38. √ y
⎯⎯ ⎯⎯⎯⎯ ⎯⎯

( 10b )
⎯⎯⎯⎯⎯⎯ ⎯⎯⎯⎯⎯⎯ ⎯⎯⎯⎯⎯⎯
39. √ 2ab √ 14a − 2 √

(5√ 2a − √ 3b )
⎯⎯⎯⎯⎯⎯ ⎯⎯⎯⎯ ⎯⎯⎯⎯
40. √ 6ab

(√ 2 − √ 5 ) (√ 3 + √ 7 )
⎯⎯ ⎯⎯ ⎯⎯ ⎯⎯
41.

( )( 7)
⎯⎯ ⎯⎯ ⎯⎯ ⎯⎯
42. √ 3 + √ 2 √ 5 − √

( ) ( 6 + 1)
⎯⎯ ⎯⎯
43. 2 √ 3 − 4 3√

( )( 3)
⎯⎯ ⎯⎯
44. 5 − 2 √ 6 7 − 2 √

8.4 Multiplying and Dividing Radical Expressions 1350


Chapter 8 Radical Expressions and Equations

( 3)
⎯⎯ ⎯⎯ 2
45. √ 5 − √

( )
⎯⎯ ⎯⎯ 2
46. √ 7 − √ 2

( )( 2)
⎯⎯ ⎯⎯ ⎯⎯ ⎯⎯
47. 2 √ 3 + √ 2 2 √ 3 − √

( )( 7)
⎯⎯ ⎯⎯ ⎯⎯ ⎯⎯
48. √ 2 + 3 √ 7 √ 2 − 3 √

( )
⎯⎯ ⎯⎯⎯⎯ 2
49. √ a − √ 2b

(√ )
⎯⎯⎯⎯ 2
50. ab + 1

⎯⎯
51. What are the perimeter and area of a rectangle with length of 5√ 3
⎯⎯
centimeters and width of 3√ 2 centimeters?

⎯⎯
52. What are the perimeter and area of a rectangle with length of 2√ 6
⎯⎯
centimeters and width of √ 3 centimeters?

⎯⎯
53. If the base of a triangle measures 6√ 2 meters and the height measures
⎯⎯
3√ 2 meters, then what is the area?
⎯⎯
54. If the base of a triangle measures 6√ 3 meters and the height measures
⎯⎯
3√ 6 meters, then what is the area?

Part B: Dividing Radical Expressions

Divide.

√75
55.
√3

8.4 Multiplying and Dividing Radical Expressions 1351


Chapter 8 Radical Expressions and Equations

√360
56.
√10

√72
57.
√75

√90
58.
√98

√90x 5
59.
√2x

√96y 3
60.
√3y

√162x 7 y 5
61.
√2xy

√363x 4 y 9
62.
√3xy

3
√ 16a5 b 2
63. 3
√ 2a2 b 2

3
√ 192a2 b 7
64. 3
√ 2a2 b 2

Rationalize the denominator.

1
65.
√5

1
66.
√6

√2
67.
√3

8.4 Multiplying and Dividing Radical Expressions 1352


Chapter 8 Radical Expressions and Equations

√3
68.
√7

5
69.
2√10

3
70.
5√6

√3 −√5
71.
√3

√6 −√2
72.
√2

1
73.
√7x

1
74.
√3y

a
75.
5√ab

3b 2
76.
2√3ab

2
77. 3
√ 36

14
78. 3
√ 7

1
79. 3
√ 4x

1
80. 3
√3y 2

3
9x⋅√ 2
81. 3
√ 9xy 2

8.4 Multiplying and Dividing Radical Expressions 1353


Chapter 8 Radical Expressions and Equations

5y 2 ⋅√
3 x
82. 3
√ 5x 2 y

3a
83. 3
2 √3a2 b 2

25n
84. 3
3 √25m 2 n

3
85. 5
√ 27x 2 y

2
86. 5
√16xy 2

ab
87. 5
√ 9a3 b

abc
88. 5
√ab 2 c3

3
89.
√ −3
10

2
90.
√6 −2

1
91.
√ √3
5 +

1
92.
√7 −√2

√3
93.
√3 +√6

√5
94.
√5 +√15

10
95.
5−3√5

8.4 Multiplying and Dividing Radical Expressions 1354


Chapter 8 Radical Expressions and Equations

−2√2
96.
4−3√2

√3 +√5
97.
√3 −√5

√10 −√2
98.
√10 +√2

2√3 −3√2
99.
4√3 +√2

6√5 +2
100.
2√5 −√2

x+√y
101.
x−√y

x−√y
102.
x+√y

√a−√b
103.
√a+√b

√ab+√2
104.
√ab−√2

√x
105.
5−2√x

1
106.
√x −y

Part C: Discussion

107. Research and discuss some of the reasons why it is a common practice
to rationalize the denominator.

108. Explain in your own words how to rationalize the denominator.

8.4 Multiplying and Dividing Radical Expressions 1355


Chapter 8 Radical Expressions and Equations

ANSWERS

⎯⎯⎯⎯
1: √ 15

⎯⎯
3: 2√ 3

5: 7

⎯⎯
7: 70√ 2

9: 20

11: 2x

⎯⎯
13: 6√ a

⎯⎯
15: 24x√ 3

17: 5

⎯⎯
19: 2 √ 5
3

⎯⎯
21: 30 √ 2
3

23: 16

25: 3a

3 ⎯⎯⎯⎯
27: 2x ⋅√ 3x

3 ⎯⎯⎯⎯⎯⎯
29: 2xy ⋅√ 25x
⎯⎯
31: 3√ 5 −5
⎯⎯⎯⎯
33: 42 − 3√ 21

8.4 Multiplying and Dividing Radical Expressions 1356


Chapter 8 Radical Expressions and Equations

⎯⎯ ⎯⎯
35: 3√ 2 − 2√ 3

37: x + x√ ⎯⎯
y

⎯⎯⎯⎯ ⎯⎯⎯⎯
39: 2a√ 7b − 4b√ 5a
⎯⎯ ⎯⎯⎯⎯ ⎯⎯⎯⎯ ⎯⎯⎯⎯
41: √ 6 + √ 14 − √ 15 − √ 35
⎯⎯ ⎯⎯ ⎯⎯
43: 18√ 2 + 2√ 3 − 12√ 6 − 4
⎯⎯⎯⎯
45: 8 − 2√ 15

47: 10

⎯⎯⎯⎯⎯⎯
49: a − 2√ 2ab + 2b

( )
⎯⎯ ⎯⎯ ⎯⎯
51: Perimeter: 10 √ 3 + 6 √ 2 centimeters; area: 15√ 6 square
centimeters

53: 18 square meters

55: 5

2√6
57:
5

⎯⎯
59: 3x 2 √ 5

61: 9x 3 y 2

63: 2a

√5
65:
5

√6
67:
3

8.4 Multiplying and Dividing Radical Expressions 1357


Chapter 8 Radical Expressions and Equations

√10
69:
4

3−√15
71:
3

√7x
73: 7x

√ab
75:
5b

3
√ 6
77:
3

3
√ 2x 2
79:
2x

3
3√ 6x 2 y
81: y

3
√ 9ab
83:
2b

5
√ 9x 3 y 4
85: xy

5
√ 27a2 b 4
87:
3

⎯⎯⎯⎯
89: 3√ 10 +9

√5 −√3
91:
2

⎯⎯
93: −1 + √2

−5−3√5
95:
2

⎯⎯⎯⎯
97: −4 − √ 15

8.4 Multiplying and Dividing Radical Expressions 1358


Chapter 8 Radical Expressions and Equations

15−7√6
99:
23

x 2 +2x√y +y
101:
x 2 −y

a−2√ab+b
103:
a−b

5√x +2x
105:
25−4x

8.4 Multiplying and Dividing Radical Expressions 1359


Chapter 8 Radical Expressions and Equations

8.5 Rational Exponents

LEARNING OBJECTIVES

1. Write expressions with rational exponents in radical form.


2. Write radical expressions with rational exponents.
3. Perform operations and simplify expressions with rational exponents.
4. Perform operations on radicals with different indices.

Definition of Rational Exponents

So far, exponents have been limited to integers. In this section, we will define what
rational (or fractional) exponents20 mean and how to work with them. All of the
rules for exponents developed up to this point apply. In particular, recall the
product rule for exponents. Given any rational numbers m and n, then

For example, if we have an exponent of 12, then the product rule for exponents
implies the following:

Here 51/2 is one of two equal factors of 5; hence it is a square root of 5, and we can
write

Furthermore, we can see that 21/3 is one of three equal factors of 2.


20. The fractional exponent m/n
that indicates a radical with
index n and exponent m:
n ⎯⎯⎯⎯
am/n = √ am .

1360
Chapter 8 Radical Expressions and Equations

Therefore, 21/3 is the cube root of 2, and we can write

This is true in general, given any nonzero real number a,

In other words, the denominator of a fractional exponent determines the index of


an nth root.

Example 1: Rewrite as a radical.

a. 71/2

b. 71/3

Solution:

⎯⎯
a. 71/2 = √7

⎯⎯
b. 71/3 = √7
3

Example 2: Rewrite as a radical and then simplify.

8.5 Rational Exponents 1361


Chapter 8 Radical Expressions and Equations

a. 811/2

b. 811/4

Solution:

⎯⎯⎯⎯
a. 811/2 = √81 = 9

⎯⎯⎯⎯ ⎯⎯⎯⎯
b. 811/4 = √81 = √34 = 3
4 4

Example 3: Rewrite as a radical and then simplify.

a. (125x 3 )
1/3

b. (−32y 10 )
1/5

Solution:

a.

b.

8.5 Rational Exponents 1362


Chapter 8 Radical Expressions and Equations

Next, consider fractional exponents where the numerator is an integer other than
1. For example, consider the following:

This shows that 52/3 is one of three equal factors of 52 . In other words, 52/3 is the
cube root of 52 and we can write:

In general, given any real number a,

An expression with a rational exponent is equivalent to a radical where the


denominator is the index and the numerator is the exponent. Any radical
expression can be written with a rational exponent, which we call exponential
form21.

Example 4: Rewrite as a radical.

a. 72/5

b. 23/4
21. An equivalent expression
written using a rational
exponent. Solution:

8.5 Rational Exponents 1363


Chapter 8 Radical Expressions and Equations

⎯⎯⎯⎯ ⎯⎯⎯⎯
a. 72/5 = √72 = √49
5 5

⎯⎯⎯⎯ ⎯⎯
b. 23/4 = √23 = √8
4 4

Example 5: Rewrite as a radical and then simplify.

a. 82/3

b. (32)3/5

Solution:

a.

b. We can often avoid very large integers by working with their prime factorization.

8.5 Rational Exponents 1364


Chapter 8 Radical Expressions and Equations

Given a radical expression, we will be asked to find the equivalent in exponential


form. Assume all variables are positive.

⎯⎯⎯⎯
Example 6: Rewrite using rational exponents: √x 2 .
3

Solution: Here the index is 3 and the power is 2. We can write

Answer: x 2/3

⎯⎯⎯⎯
Example 7: Rewrite using rational exponents: √
6
y3 .

Solution: Here the index is 6 and the power is 3. We can write

Answer: y 1/2

It is important to note that the following are equivalent.

8.5 Rational Exponents 1365


Chapter 8 Radical Expressions and Equations

In other words, it does not matter if we apply the power first or the root first. For
example, we can apply the power before the root:

Or we can apply the nth root before the power:

The results are the same.

Example 8: Rewrite as a radical and then simplify: (−8)2/3 .

Solution: Here the index is 3 and the power is 2. We can write

Answer: 4

Try this! Rewrite as a radical and then simplify: 253/2 .

Answer: 125

Video Solution

(click to see video)

8.5 Rational Exponents 1366


Chapter 8 Radical Expressions and Equations

Some calculators have a caret button ˆ . If so, we can calculate approximations


for radicals using it and rational exponents. For example, to calculate
⎯⎯
√2 = 21/2 = 2 ^ (1/2) ≈ 1.414, we would type

3 ⎯⎯⎯⎯
To calculate √22 = 22/3 = 2 ^ (2/3) ≈ 1.587, we would type

Operations Using the Rules of Exponents

In this section, we review all of the rules of exponents, which extend to include
rational exponents. If given any rational numbers m and n, then we have

Product rule: x m ⋅ x n = x m+n

xm
Quotient rule: = x m−n , x ≠ 0
x n

(x ) = x
m n m⋅n
Power rule:

(xy) = x y
n n n
Power rule for a product:

(y )
n
x xn
Power rule for a quotient: = n ,y≠0
y

1
Negative exponents: x −n =
xn

8.5 Rational Exponents 1367


Chapter 8 Radical Expressions and Equations

Zero exponent: x 0 = 1, x ≠ 0

These rules allow us to perform operations with rational exponents.

Example 9: Simplify: 22/3 ⋅ 21/6 .

Solution:

Answer: 25/6

1/2
Example 10: Simplify: x 1/3 .
x

Solution:

Answer: x 1/6

8.5 Rational Exponents 1368


Chapter 8 Radical Expressions and Equations

Example 11: Simplify: (y 3/4 )


2/3
.

Solution:

Answer: y 1/2

Example 12: Simplify: (16a4 b8 )


3/4
.

Solution:

Answer: 8a3 b6

Example 13: Simplify: 25−3/2 .

Solution:

8.5 Rational Exponents 1369


Chapter 8 Radical Expressions and Equations

Answer: 1/125

(8a b )
3/4 3 2/3

Try this! Simplify: .


a1/3

Answer: 4a1/6 b2

Video Solution

(click to see video)


Radical Expressions with Different Indices

To apply the product or quotient rule for radicals, the indices of the radicals
involved must be the same. If the indices are different, then first rewrite the
radicals in exponential form and then apply the rules for exponents.

⎯⎯ ⎯⎯
Example 14: Multiply: √2 ⋅ √2.
3

Solution: In this example, the index of each radical factor is different. Hence the
product rule for radicals does not apply. Begin by converting the radicals into an

8.5 Rational Exponents 1370


Chapter 8 Radical Expressions and Equations

equivalent form using rational exponents. Then apply the product rule for
exponents.

⎯⎯⎯⎯
Answer: √25
6

3
√ 4
Example 15: Divide: 5
.
√ 2

Solution: In this example, the index of the radical in the numerator is different
from the index of the radical in the denominator. Hence the quotient rule for
radicals does not apply. Begin by converting the radicals into an equivalent form
using rational exponents and then apply the quotient rule for exponents.

⎯⎯⎯⎯
Answer: √27
15

8.5 Rational Exponents 1371


Chapter 8 Radical Expressions and Equations

Example 16: Simplify: √ √4 .


⎯⎯⎯⎯⎯⎯⎯⎯⎯
3

Solution: Here the radicand of the square root is a cube root. After rewriting this
expression using rational exponents, we will see that the power rule for exponents
applies.

⎯⎯
Answer: √2
3

KEY TAKEAWAYS

• When converting fractional exponents to radicals, use the numerator as


the power and the denominator as the index of the radical.
• All the rules of exponents apply to expressions with rational exponents.

8.5 Rational Exponents 1372


Chapter 8 Radical Expressions and Equations

TOPIC EXERCISES

Part A: Rational Exponents

Express using rational exponents.

⎯⎯
1. √ 6

⎯⎯⎯⎯
2. √ 10

⎯⎯⎯⎯
3. √ 11
3

⎯⎯
4. √ 2
4

⎯⎯⎯2⎯
5. √ 5
3

⎯⎯⎯3⎯
6. √ 2
4

⎯⎯
7. √
5
x
⎯⎯
8. √
6
x
⎯⎯⎯⎯
9. √ x 7
6

⎯⎯⎯⎯
10. √ x 4
5

Express in radical form.

1/2
11. 2

1/3
12. 5

2/3
13. 7

3/5
14. 2

8.5 Rational Exponents 1373


Chapter 8 Radical Expressions and Equations

15. x 3/4

16. x 5/6

17. x −1/2

18. x −3/4

19. ( x )
1 −1/3

20. ( x )
1 −3/5

Write as a radical and then simplify.

1/2
21. 25

1/2
22. 36

1/2
23. 121

1/2
24. 144

25. (
4)
1 1/2

26. (
9)
4 1/2

−1/2
27. 4

−1/2
28. 9

29. (
4)
1 −1/2

30. (
16 )
1 −1/2

8.5 Rational Exponents 1374


Chapter 8 Radical Expressions and Equations

1/3
31. 8

1/3
32. 125

33. (
27 )
1 1/3

34. (
125 )
8 1/3

1/3
35. (−27)

36. (−64)
1/3

1/4
37. 16

1/4
38. 625

−1/4
39. 81

−1/4
40. 16

1/5
41. 100, 000

1/5
42. (−32)

43. (
32 )
1 1/5

44. (
243 )
1 1/5

3/2
45. 9

3/2
46. 4

5/3
47. 8

2/3
48. 27

8.5 Rational Exponents 1375


Chapter 8 Radical Expressions and Equations

3/2
49. 16

2/5
50. 32

51. (
16 )
1 3/4

52. (
81 )
1 3/4

2/3
53. (−27)

4/3
54. (−27)

3/5
55. (−32)

4/5
56. (−32)

Use a calculator to approximate an answer rounded to the nearest hundredth.

3/4
57. 2

2/3
58. 3

1/5
59. 5

1/7
60. 7

3/2
61. (−9)

3/2
62. −9

63. Explain why (−4)^(3/2) gives an error on a calculator and −4^(3/2) gives
an answer of −8.

64. Marcy received a text message from Mark asking her how old she was. In
response, Marcy texted back “125^(2/3) years old.” Help Mark determine
how old Marcy is.

8.5 Rational Exponents 1376


Chapter 8 Radical Expressions and Equations

Part B: Rational Exponents

Perform the operations and simplify. Leave answers in exponential form.

2/3
65. 2 ⋅ 2 4/3
3/2
66. 3 ⋅ 3 1/2
1/2
67. 5 ⋅ 5 1/3
1/6
68. 2 ⋅ 2 3/4

69. y 1/4 ⋅ y 2/5

70. x 1/2 ⋅ x 1/4

5 7/3
71.
5 1/3

2 9/2
72.
2 1/2

2a2/3
73.
a1/6

3b 1/2
74.
b 1/3

75. (8 )
1/2 2/3

76. (3 )
6 2/3

77. (x 2/3 )
1/2

78. (y 3/4 )
4/5

79. (4x 2 y 4 )
1/2

8.5 Rational Exponents 1377


Chapter 8 Radical Expressions and Equations

80. (9x 6 y 2 )
1/2

81. (2x 1/3 y 2/3 )


3

82. (8x 3/2 y 1/2 )


2

( a1/2 )
4/3
a3/4
83.

( b1/10 )
10/3
b 4/5
84.

( y4 )
1/2
4x 2/3
85.

( y9 )
1/3
27x 3/4
86.

y 1/2 ⋅y 2/3
87.
y 1/6

x 2/5 ⋅x 1/2
88.
x 1/10

xy
89.
x 1/2 y 1/3

x 5/4 y
90.
xy 2/5

49a5/7 b 3/2
91.
7a3/7 b 1/4

16a5/6 b 5/4
92.
8a1/2 b 2/3

(9x y )
2/3 6 3/2

93.
x 1/2 y

8.5 Rational Exponents 1378


Chapter 8 Radical Expressions and Equations

(125x y )
3 3/5 2/3

94.
xy 1/3

(27a b )
1/4 3/2 2/3

95.
a1/6 b 1/2

(25a b )
2/3 4/3 3/2

96.
a1/6 b 1/3

Part C: Mixed Indices

Perform the operations.

⎯⎯ 5 ⎯⎯
97. √ 9 ⋅√ 3
3

⎯⎯ 5 ⎯⎯⎯⎯
98. √ 5 ⋅√ 25
⎯⎯ 3 ⎯⎯
99. √ x ⋅√ x

100. √ y
⎯⎯ ⋅ 4 ⎯⎯
y

⎯⎯⎯⎯ 4 ⎯⎯
101. √ x 2
3
⋅√ x
⎯⎯⎯⎯ 3 ⎯⎯
102. √ x 3
5
⋅√ x

3
√ 100
103.
√10

5
√ 16
104. 3
√ 4

3 2
√a
105.
√a

5 4
√b
106. 3
√ b

8.5 Rational Exponents 1379


Chapter 8 Radical Expressions and Equations

3 2
√x
107. 5 3
√x

4 3
√x
108. 3 2
√x

109. √ √ 16
⎯⎯⎯⎯⎯⎯⎯⎯
⎯⎯⎯⎯⎯
5

110. √ √ 9
⎯⎯⎯⎯⎯⎯⎯⎯⎯
3

111. √ √ 2
⎯⎯⎯⎯⎯⎯⎯⎯⎯
35

112. √ √ 5
⎯⎯⎯⎯⎯⎯⎯⎯⎯
35

113. √ √ 7
⎯⎯⎯⎯⎯⎯⎯⎯⎯
3

114. √ √ 3
⎯⎯⎯⎯⎯⎯⎯⎯⎯
3

Part D: Discussion Board

115. Who is credited for devising the notation for rational exponents? What
are some of his other accomplishments?

116. When using text, it is best to communicate nth roots using rational
exponents. Give an example.

8.5 Rational Exponents 1380


Chapter 8 Radical Expressions and Equations

ANSWERS

1/2
1: 6

1/3
3: 11

2/3
5: 5

7: x 1/5

9: x 7/6

⎯⎯
11: √ 2

⎯⎯⎯2⎯
13: √ 7
3

⎯⎯⎯⎯
15: √ x 3
4

1
17:
√x

⎯⎯
19: √
3
x

21: 5

23: 11

25: 1/2

27: 1/2

29: 2

31: 2

33: 1/3

35: −3

8.5 Rational Exponents 1381


Chapter 8 Radical Expressions and Equations

37: 2

39: 1/3

41: 10

43: 1/2

45: 27

47: 32

49: 64

51: 1/8

53: 9

55: −8

57: 1.68

59: 1.38

61: Not a real number

63: In the first expression, the square root of a negative number creates an
error condition on the calculator. The square root of a negative number is
not real. In the second expression, because of the order of operations, the
negative sign is applied to the answer after 4 is raised to the (3/2) power.

65: 4

5/6
67: 5

69: y 13/20

71: 25

73: 2a1/2

8.5 Rational Exponents 1382


Chapter 8 Radical Expressions and Equations

75: 2

77: x 1/3

79: 2xy 2

81: 8xy 2

83: a1/3

2x 1/3
85:
y2

87: y

89: x 1/2 y 2/3

91: 7x 2/7 y 5/4

93: 27x 1/2 y 8

1/2
95: 9b

⎯⎯⎯⎯
13

97: √ 3
15

⎯⎯⎯⎯
99: √ x 5
6

⎯⎯⎯⎯⎯⎯
101: √ x 11
12

⎯⎯⎯⎯
103: √ 10
6

⎯⎯
105: √
6
a
⎯⎯
107: √
15
x
⎯⎯
109: √ 4
5

8.5 Rational Exponents 1383


Chapter 8 Radical Expressions and Equations

⎯⎯
111: √ 2
15

⎯⎯
113: √ 7
6

8.5 Rational Exponents 1384


Chapter 8 Radical Expressions and Equations

8.6 Solving Radical Equations

LEARNING OBJECTIVES

1. Solve equations involving square roots.


2. Solve equations involving cube roots.

Radical Equations

A radical equation22 is any equation that contains one or more radicals with a
variable in the radicand. Following are some examples of radical equations, all of
which will be solved in this section:

We begin with the squaring property of equality23; given real numbers a and b, we
have the following:

In other words, equality is retained if we square both sides of an equation.

22. Any equation that contains one


or more radicals with a The converse, on the other hand, is not necessarily true:
variable in the radicand.

23. Given real numbers a and b,


where a = b, then a2 = b2.

1385
Chapter 8 Radical Expressions and Equations

This is important because we will use this property to solve radical equations.
Consider a very simple radical equation that can be solved by inspection:

use of the squaring property of equality and the fact that (√a) = √a2 = a when
Here we can see that x = 9 is a solution. To solve this equation algebraically, make
⎯⎯ 2 ⎯⎯⎯⎯

a is positive. Eliminate the square root by squaring both sides of the equation as
follows:

⎯⎯
As a check, we can see that √9 = 3 as expected. Because the converse of the
squaring property of equality is not necessarily true, solutions to the squared
equation may not be solutions to the original. Hence squaring both sides of an
equation introduces the possibility of extraneous solutions24, or solutions that do
not solve the original equation. For this reason, we must check the answers that
result from squaring both sides of an equation.

⎯⎯⎯⎯⎯⎯⎯⎯
Example 1: Solve: √x − 1 = 5.

Solution: We can eliminate the square root by applying the squaring property of
equality.

24. A solution that does not solve


the original equation.

8.6 Solving Radical Equations 1386


Chapter 8 Radical Expressions and Equations

Next, we must check.

Answer: The solution is 26.

⎯⎯⎯⎯⎯⎯⎯⎯⎯⎯
Example 2: Solve: √5 − 4x = x.

Solution: Begin by squaring both sides of the equation.

You are left with a quadratic equation that can be solved by factoring.

8.6 Solving Radical Equations 1387


Chapter 8 Radical Expressions and Equations

Since you squared both sides, you must check your solutions.

After checking, you can see that x = −5 was extraneous; it did not solve the
original radical equation. Disregard that answer. This leaves x = 1 as the only
solution.

Answer: The solution is x = 1 .

In the previous two examples, notice that the radical is isolated on one side of the
equation. Typically, this is not the case. The steps for solving radical equations
involving square roots are outlined in the following example.

⎯⎯⎯⎯⎯⎯⎯⎯⎯⎯
Example 3: Solve: √2x − 5 + 4 = x.

8.6 Solving Radical Equations 1388


Chapter 8 Radical Expressions and Equations

Solution:

Step 1: Isolate the square root. Begin by subtracting 4 from both sides of the
equation.

Step 2: Square both sides. Squaring both sides eliminates the square root.

Step 3: Solve the resulting equation. Here you are left with a quadratic equation
that can be solved by factoring.

Step 4: Check the solutions in the original equation. Squaring both sides introduces
the possibility of extraneous solutions; hence the check is required.

8.6 Solving Radical Equations 1389


Chapter 8 Radical Expressions and Equations

After checking, we can see that x = 3 is an extraneous root; it does not solve the
original radical equation. This leaves x = 7 as the only solution.

Answer: The solution is x = 7 .

⎯⎯⎯⎯⎯⎯⎯⎯
Example 4: Solve: 3√x + 1 − 2x = 0.

Solution: Begin by isolating the term with the radical.

Despite the fact that the term on the left side has a coefficient, it is still considered
isolated. Recall that terms are separated by addition or subtraction operators.

8.6 Solving Radical Equations 1390


Chapter 8 Radical Expressions and Equations

Solve the resulting quadratic equation.

Since we squared both sides, we must check our solutions.

After checking, we can see that x = − 34 was extraneous.

Answer: The solution is 3.

Sometimes both of the possible solutions are extraneous.

8.6 Solving Radical Equations 1391


Chapter 8 Radical Expressions and Equations

⎯⎯⎯⎯⎯⎯⎯⎯⎯⎯⎯⎯
Example 5: Solve: √4 − 11x − x + 2 = 0.

Solution: Begin by isolating the radical.

Since we squared both sides, we must check our solutions.

Since both possible solutions are extraneous, the equation has no solution.

Answer: No solution, Ø

8.6 Solving Radical Equations 1392


Chapter 8 Radical Expressions and Equations

The squaring property of equality extends to any positive integer power n. Given
real numbers a and b, we have the following:

along with the fact that (√ a) = √


This is often referred to as the power property of equality25. Use this property,
⎯⎯ n ⎯⎯⎯⎯
n
a = a, when a is positive, to solve radical
n n

equations with indices greater than 2.

⎯⎯⎯⎯⎯⎯⎯⎯⎯⎯
Example 6: Solve: √x 2 + 4 − 2 = 0.
3

Solution: Isolate the radical and then cube both sides of the equation.

Check.

25. Given any positive integer n


and real numbers a and b,
n
where a = b, then an = b .

8.6 Solving Radical Equations 1393


Chapter 8 Radical Expressions and Equations

Answer: The solutions are −2 and 2.

⎯⎯⎯⎯⎯⎯⎯⎯⎯⎯
Try this! Solve: √2x − 1 + 2 = x.

Answer: x = 5 (x = 1 is extraneous)

Video Solution

(click to see video)

It may be the case that the equation has two radical expressions.

⎯⎯⎯⎯⎯⎯⎯⎯⎯⎯ ⎯⎯⎯⎯⎯⎯⎯⎯⎯⎯
Example 7: Solve: √3x − 4 = √2x + 9.

Solution: Both radicals are considered isolated on separate sides of the equation.

8.6 Solving Radical Equations 1394


Chapter 8 Radical Expressions and Equations

Check x = 13 .

Answer: The solution is 13.

⎯⎯⎯⎯⎯⎯⎯⎯⎯⎯⎯⎯⎯⎯⎯⎯⎯⎯ ⎯⎯⎯⎯⎯⎯⎯⎯⎯⎯
Example 8: Solve: √x 2 + x − 14 = √x + 50.
3 3

Solution: Eliminate the radicals by cubing both sides.

8.6 Solving Radical Equations 1395


Chapter 8 Radical Expressions and Equations

Check.

Answer: The solutions are −8 and 8.

We will learn how to solve some of the more advanced radical equations in the next
course, Intermediate Algebra.

⎯⎯⎯⎯⎯⎯⎯⎯⎯⎯ ⎯⎯⎯⎯⎯⎯⎯⎯
Try this! Solve: √3x + 1 = 2√x − 3.

Answer: 13

Video Solution

(click to see video)

8.6 Solving Radical Equations 1396


Chapter 8 Radical Expressions and Equations

KEY TAKEAWAYS

• Solve equations involving square roots by first isolating the radical and
then squaring both sides. Squaring a square root eliminates the radical,
leaving us with an equation that can be solved using the techniques
learned earlier in our study of algebra. However, squaring both sides of
an equation introduces the possibility of extraneous solutions, so check
your answers in the original equation.
• Solve equations involving cube roots by first isolating the radical and
then cubing both sides. This eliminates the radical and results in an
equation that may be solved with techniques you have already
mastered.

8.6 Solving Radical Equations 1397


Chapter 8 Radical Expressions and Equations

TOPIC EXERCISES

Part A: Solving Radical Equations

Solve.

⎯⎯
1. √ x =2
⎯⎯
2. √ x =7
⎯⎯
3. √ x +7=8
⎯⎯
4. √ x +4=9
⎯⎯
5. √ x +6=3
⎯⎯
6. √ x +2=1
⎯⎯
7. 5√ x −1=0
⎯⎯
8. 3√ x −2=0
⎯⎯⎯⎯⎯⎯⎯⎯⎯
9. √ x −3 =3
⎯⎯⎯⎯⎯⎯⎯⎯⎯
10. √ x +5 =6
⎯⎯⎯⎯⎯⎯⎯⎯⎯⎯⎯
11. √ 3x +1 =2
⎯⎯⎯⎯⎯⎯⎯⎯⎯⎯⎯
12. √ 5x −4 =4
⎯⎯⎯⎯⎯⎯⎯⎯⎯⎯⎯
13. √ 7x + 4 + 6 = 11
⎯⎯⎯⎯⎯⎯⎯⎯⎯⎯⎯
14. √ 3x − 5 + 9 = 14
⎯⎯⎯⎯⎯⎯⎯⎯⎯
15. 2√ x −1 −3=0

8.6 Solving Radical Equations 1398


Chapter 8 Radical Expressions and Equations

⎯⎯⎯⎯⎯⎯⎯⎯⎯
16. 3√ x +1 −2=0
⎯⎯
17. √
3
x =2
⎯⎯
18. √
3
x =5
⎯⎯⎯⎯⎯⎯⎯⎯⎯⎯⎯
19. √ 2x +9 =3
3

⎯⎯⎯⎯⎯⎯⎯⎯⎯⎯⎯⎯⎯
20. √ 4x − 11 = 1
3

⎯⎯⎯⎯⎯⎯⎯⎯⎯⎯⎯
21. √ 5x +7 +3=1
3

⎯⎯⎯⎯⎯⎯⎯⎯⎯⎯⎯
22. √ 3x −6 +5=2
3

⎯⎯⎯⎯⎯⎯⎯⎯⎯
23. 2 √ x +2 −1=0
3

⎯⎯⎯⎯⎯⎯⎯⎯⎯⎯⎯
24. 2 √ 2x −3 −1=0
3

⎯⎯⎯⎯⎯⎯⎯⎯⎯⎯⎯⎯⎯ ⎯⎯⎯⎯⎯⎯⎯⎯⎯
25. √ 8x + 11 = 3√ x + 1
⎯⎯⎯⎯⎯⎯⎯⎯⎯⎯⎯ ⎯⎯⎯⎯⎯⎯⎯⎯⎯⎯⎯⎯⎯⎯⎯⎯
26. 2√ 3x − 4 = √ 2 (3x + 1)
⎯⎯⎯⎯⎯⎯⎯⎯⎯⎯⎯⎯⎯⎯⎯⎯ ⎯⎯⎯⎯⎯⎯⎯⎯⎯⎯⎯⎯⎯
27. √ 2 (x + 10) = √ 7x − 15
⎯⎯⎯⎯⎯⎯⎯⎯⎯⎯⎯⎯⎯⎯ ⎯⎯⎯⎯⎯⎯⎯⎯⎯
28. √ 5 (x − 4) = √ x + 4
⎯⎯⎯⎯⎯⎯⎯⎯⎯⎯⎯ 3 ⎯⎯⎯⎯
29. √ 5x −2 =√ 4x
3

⎯⎯⎯⎯⎯⎯⎯⎯⎯⎯⎯⎯⎯⎯ 3 ⎯⎯⎯⎯⎯⎯⎯⎯⎯⎯⎯⎯⎯⎯
30. √ 9 (x − 1) = √ 3 (x + 7)
3

⎯⎯⎯⎯⎯⎯⎯⎯⎯⎯⎯ 3 ⎯⎯⎯⎯⎯⎯⎯⎯⎯⎯⎯⎯⎯
31. √ 3x +1 =√ 2(x − 1)
3

⎯⎯⎯⎯ 3 ⎯⎯⎯⎯⎯⎯⎯⎯⎯⎯⎯⎯⎯
32. √ 9x =√ 3(x − 6)
3

8.6 Solving Radical Equations 1399


Chapter 8 Radical Expressions and Equations

⎯⎯⎯⎯⎯⎯⎯⎯⎯⎯⎯⎯⎯
33. √ 4x + 21 = x
⎯⎯⎯⎯⎯⎯⎯⎯⎯⎯⎯
34. √ 8x +9 =x
⎯⎯⎯⎯⎯⎯⎯⎯⎯⎯⎯⎯⎯⎯⎯⎯
35. √ 4 (2x − 3) = x
⎯⎯⎯⎯⎯⎯⎯⎯⎯⎯⎯⎯⎯⎯⎯⎯
36. √ 3 (4x − 9) = x
⎯⎯⎯⎯⎯⎯⎯⎯⎯
37. 2√ x −1 =x
⎯⎯⎯⎯⎯⎯⎯⎯⎯⎯⎯
38. 3√ 2x −9 =x
⎯⎯⎯⎯⎯⎯⎯⎯⎯⎯⎯
39. √ 9x +9 =x+1
⎯⎯⎯⎯⎯⎯⎯⎯⎯⎯⎯⎯⎯
40. √ 3x + 10 = x + 4
⎯⎯⎯⎯⎯⎯⎯⎯⎯
41. √ x −1 =x−3
⎯⎯⎯⎯⎯⎯⎯⎯⎯⎯⎯
42. √ 2x −5 =x−4
⎯⎯⎯⎯⎯⎯⎯⎯⎯⎯⎯⎯⎯
43. √ 16 − 3x = x − 6
⎯⎯⎯⎯⎯⎯⎯⎯⎯⎯⎯
44. √ 7 − 3x = x − 3
⎯⎯⎯⎯⎯⎯⎯⎯⎯⎯⎯⎯⎯
45. 3√ 2x + 10 = x + 9
⎯⎯⎯⎯⎯⎯⎯⎯⎯⎯⎯
46. 2√ 2x +5 =x+4
⎯⎯⎯⎯⎯⎯⎯⎯⎯
47. 3√ x −1 −1=x
⎯⎯⎯⎯⎯⎯⎯⎯⎯⎯⎯
48. 2√ 2x +2 −1=x
⎯⎯⎯⎯⎯⎯⎯⎯⎯⎯⎯⎯⎯⎯⎯
49. √ 10x + 41 − 5 = x

8.6 Solving Radical Equations 1400


Chapter 8 Radical Expressions and Equations

⎯⎯⎯⎯⎯⎯⎯⎯⎯⎯⎯⎯⎯⎯
50. √ 6 (x + 3) − 3 = x
⎯⎯⎯⎯⎯⎯⎯⎯⎯⎯⎯⎯⎯⎯⎯⎯⎯⎯⎯⎯⎯
51. √ 8x 2 − 4x + 1 = 2x
⎯⎯⎯⎯⎯⎯⎯⎯⎯⎯⎯⎯⎯⎯⎯⎯⎯⎯⎯⎯⎯⎯⎯
52. √ 18x 2 − 6x + 1 = 3x
⎯⎯⎯⎯⎯⎯⎯⎯⎯
53. 5√ x +2 =x+8
⎯⎯⎯⎯⎯⎯⎯⎯⎯⎯⎯⎯⎯⎯
54. 4√ 2 (x + 1) = x + 7
⎯⎯⎯⎯⎯⎯⎯⎯⎯⎯⎯⎯⎯
55. √ x 2 − 25 = x
⎯⎯⎯⎯⎯⎯⎯⎯⎯⎯⎯
56. √ x 2 +9=x
⎯⎯⎯⎯⎯⎯⎯⎯⎯⎯⎯⎯⎯
57. 3 + √ 6x − 11 = x
⎯⎯⎯⎯⎯⎯⎯⎯⎯⎯⎯
58. 2 + √ 9x − 8 = x
⎯⎯⎯⎯⎯⎯⎯⎯⎯⎯⎯⎯⎯
59. √ 4x + 25 − x = 7
⎯⎯⎯⎯⎯⎯⎯⎯⎯⎯⎯⎯⎯
60. √ 8x + 73 − x = 10
⎯⎯⎯⎯⎯⎯⎯⎯⎯⎯⎯
61. 2√ 4x + 3 − 3 = 2x
⎯⎯⎯⎯⎯⎯⎯⎯⎯⎯⎯
62. 2√ 6x + 3 − 3 = 3x
⎯⎯⎯⎯⎯⎯⎯⎯⎯⎯⎯⎯⎯⎯⎯
63. 2x − 4 = √ 14 − 10x
⎯⎯⎯⎯⎯⎯⎯⎯⎯⎯⎯⎯⎯⎯⎯
64. 3x − 6 = √ 33 − 24x
⎯⎯⎯⎯⎯⎯⎯⎯⎯⎯⎯⎯⎯
65. √ x 2
3
− 24 = 1
⎯⎯⎯⎯⎯⎯⎯⎯⎯⎯⎯⎯⎯
66. √ x 2
3
− 54 = 3

8.6 Solving Radical Equations 1401


Chapter 8 Radical Expressions and Equations

⎯⎯⎯⎯⎯⎯⎯⎯⎯⎯⎯⎯⎯
67. √ x 2 + 6x + 1 = 4
3

⎯⎯⎯⎯⎯⎯⎯⎯⎯⎯⎯⎯⎯
68. √ x 2 + 2x + 5 = 7
3

⎯⎯⎯⎯⎯⎯⎯⎯⎯⎯⎯⎯⎯⎯⎯⎯⎯⎯⎯⎯⎯⎯⎯⎯⎯
69. √ 25x 2
3
− 10x − 7 = −2
⎯⎯⎯⎯⎯⎯⎯⎯⎯⎯⎯⎯⎯⎯⎯⎯⎯⎯⎯⎯⎯⎯⎯⎯⎯
70. √ 9x 2
3
− 12x − 23 = −3

− 15x + 25 = √(x + 5) (x − 5)
⎯⎯⎯⎯⎯⎯⎯⎯⎯⎯⎯⎯⎯⎯⎯⎯⎯⎯⎯⎯⎯⎯⎯⎯⎯ ⎯⎯⎯⎯⎯⎯⎯⎯⎯⎯⎯⎯⎯⎯⎯⎯⎯⎯⎯⎯⎯⎯⎯⎯⎯⎯
71. √ 2x 2

− 4x + 4 = √x (5 − x)
⎯⎯⎯⎯⎯⎯⎯⎯⎯⎯⎯⎯⎯⎯⎯⎯⎯⎯⎯ ⎯⎯⎯⎯⎯⎯⎯⎯⎯⎯⎯⎯⎯⎯⎯
72. √ x 2

73. √ 2 (x 2 + 3x − 20) = √
⎯⎯⎯⎯⎯⎯⎯⎯⎯⎯⎯⎯⎯⎯⎯⎯⎯⎯⎯⎯⎯⎯⎯⎯⎯⎯⎯ 3 ⎯⎯⎯⎯⎯⎯⎯⎯⎯⎯⎯⎯2⎯
3
(x + 3)

74. √ 3x + 3x + 40 = √ (x − 5)
⎯⎯⎯⎯⎯⎯⎯⎯⎯⎯⎯⎯⎯⎯⎯⎯⎯⎯⎯⎯⎯⎯⎯ ⎯⎯⎯⎯⎯⎯⎯⎯⎯⎯⎯⎯⎯2⎯
3 2 3

75. x 1/2 − 10 = 0

76. x 1/2 −6=0

77. x 1/3 +2=0

78. x 1/3 +4=0

79. (x − 1) 1/2 − 3 = 0

80. (x + 2) 1/2 − 6 = 0

81. (2x − 1) 1/3 + 3 = 0

82. (3x − 1) 1/3 − 2 = 0

83. (4x + 15)


1/2
− 2x = 0

8.6 Solving Radical Equations 1402


Chapter 8 Radical Expressions and Equations

84. (3x + 2) 1/2 − 3x = 0

85. (2x + 12) 1/2 − x = 6

86. (4x + 36)


1/2
−x=9

87. 2(5x + 26)


1/2
= x + 10

88. 3(x − 1) 1/2 = x + 1

89. The square root of 1 less than twice a number is equal to 2 less than the
number. Find the number.

90. The square root of 4 less than twice a number is equal to 6 less than the
number. Find the number.

91. The square root of twice a number is equal to one-half of that number.
Find the number.

92. The square root of twice a number is equal to one-third of that number.
Find the number.

93. The distance, d, measured in miles, a person can see an object is given by
the formula

⎯⎯⎯⎯
√ 6h
d=
2

where h represents the person’s height above sea level, measured in feet.
How high must a person be to see an object 5 miles away?

94. The current, I, measured in amperes, is given by the formula

⎯⎯⎯⎯
√R
P
I=

8.6 Solving Radical Equations 1403


Chapter 8 Radical Expressions and Equations

where P is the power usage, measured in watts, and R is the resistance,


measured in ohms. If a light bulb requires 1/2 ampere of current and uses 60
watts of power, then what is the resistance of the bulb?

The period, T, of a pendulum in seconds is given by the formula

⎯⎯⎯⎯⎯⎯
√ 32
L
T = 2π

where L represents the length in feet. For each problem below, calculate the length of
a pendulum, given the period. Give the exact value and the approximate value
rounded off to the nearest tenth of a foot.

95. 1 second

96. 2 seconds

97. 1/2 second

98. 1/3 second

The time, t, in seconds an object is in free fall is given by the formula


√s
t=
4

where s represents the distance in feet the object has fallen. For each problem below,
calculate the distance an object falls, given the amount of time.

99. 1 second

100. 2 seconds

101. 1/2 second

102. 1/4 second

8.6 Solving Radical Equations 1404


Chapter 8 Radical Expressions and Equations

The x-intercepts for any graph have the form (x, 0), where x is a real number.
Therefore, to find x-intercepts, set y = 0 and solve for x. Find the x-intercepts for each
of the following.

⎯⎯⎯⎯⎯⎯⎯⎯⎯
103. y = √x − 3 − 1
⎯⎯⎯⎯⎯⎯⎯⎯⎯
104. y = √x + 2 − 3

3 ⎯⎯⎯⎯⎯⎯⎯⎯⎯
105. y =√ x−1 +2

3 ⎯⎯⎯⎯⎯⎯⎯⎯⎯
106. y =√ x+1 −3

Part B: Discussion Board

107. Discuss reasons why we sometimes obtain extraneous solutions when


solving radical equations. Are there ever any conditions where we do not
need to check for extraneous solutions? Why?

8.6 Solving Radical Equations 1405


Chapter 8 Radical Expressions and Equations

ANSWERS

1: 4

3: 1

5: Ø

7: 1/25

9: 12

11: 1

13: 3

15: 13/4

17: 8

19: 9

21: −3

23: −15/8

25: 2

27: 7

29: 2

31: −3

33: 7

35: 2, 6

37: 2

8.6 Solving Radical Equations 1406


Chapter 8 Radical Expressions and Equations

39: −1, 8

41: 5

43: Ø

45: −3, 3

47: 2, 5

49: −4, −4

51: 1/2

53: 2, 7

55: Ø

57: 10

59: −6, −4

61: −1/2, 3/2

63: Ø

65: −5, 5

67: −9, 3

69: 1/5

71: 5, 10

73: −7, 7

75: 100

77: −8

8.6 Solving Radical Equations 1407


Chapter 8 Radical Expressions and Equations

79: 10

81: −13

83: 5/2

85: −6, −4

87: −2, 2

89: 5

91: 8

93: 16 2
3
feet

95: 8/π 2 ≈ 0.8 foot

97: 2/π 2 ≈ 0.2 foot

99: 16 feet

101: 4 feet

103: (4, 0)

105: (−7, 0)

8.6 Solving Radical Equations 1408


Chapter 8 Radical Expressions and Equations

8.7 Review Exercises and Sample Exam

1409
Chapter 8 Radical Expressions and Equations

REVIEW EXERCISES

(Assume all variables represent nonnegative numbers.)

Radicals

Simplify.

⎯⎯⎯⎯
1. √ 36

2. √
⎯⎯⎯⎯
4

25

⎯⎯⎯⎯⎯⎯⎯
3. √ −16

⎯⎯
4. −√ 9

⎯⎯⎯⎯⎯⎯
5. √ 125
3

⎯⎯⎯⎯⎯
6. 3 √ −8
3

7. √
⎯⎯⎯⎯
1

3
64

⎯⎯⎯⎯⎯⎯⎯
8. −5 √ −27
3

⎯⎯⎯⎯
9. √ 40

⎯⎯⎯⎯
10. −3√ 50

11. √
⎯98
⎯⎯⎯⎯
81

12. √
⎯⎯⎯⎯⎯
1

121

⎯⎯⎯⎯⎯⎯
13. 5 √ 192
3

8.7 Review Exercises and Sample Exam 1410


Chapter 8 Radical Expressions and Equations

⎯⎯⎯⎯⎯⎯⎯
14. 2 √ −54
3

Simplifying Radical Expressions

Simplify.

⎯⎯⎯⎯⎯⎯⎯⎯
15. √ 49x 2

⎯⎯⎯⎯⎯⎯⎯⎯⎯⎯2⎯
16. √ 25a2 b

⎯⎯⎯⎯⎯⎯⎯⎯⎯⎯⎯⎯
17. √ 75x 3 y 2

⎯⎯⎯⎯⎯⎯⎯⎯⎯⎯⎯⎯⎯⎯⎯
18. √ 200m 4 n 3

√ 25y 2
⎯18x
⎯⎯⎯⎯⎯3⎯
19.


⎯108x
⎯⎯⎯⎯⎯⎯⎯3⎯
20.
49y 4

⎯⎯⎯⎯⎯⎯⎯⎯⎯⎯
21. √ 216x 3
3

⎯⎯⎯⎯⎯⎯⎯⎯⎯⎯⎯⎯⎯⎯⎯⎯⎯
22. √ −125x 6 y 3
3

⎯⎯⎯⎯⎯⎯⎯⎯⎯⎯⎯⎯⎯⎯
5 3

23. √ 27a7 b
3
c
⎯⎯⎯⎯⎯⎯⎯⎯⎯⎯⎯⎯⎯⎯
24. √ 120x 9 y 4
3

Use the distance formula to calculate the distance between the given two points.

25. (5, −8) and (2, −10)

26. (−7, −1) and (−6, 1)

27. (−10, −1) and (0, −5)

28. (5, −1) and (−2, −2)

8.7 Review Exercises and Sample Exam 1411


Chapter 8 Radical Expressions and Equations

Adding and Subtracting Radical Expressions

Simplify.

⎯⎯ ⎯⎯
29. 8√ 3 + 3√ 3
⎯⎯⎯⎯ ⎯⎯⎯⎯
30. 12√ 10 − 2√ 10
⎯⎯ ⎯⎯ ⎯⎯ ⎯⎯
31. 14√ 3 + 5√ 2 − 5√ 3 − 6√ 2
⎯⎯⎯⎯ ⎯⎯ ⎯⎯⎯⎯ ⎯⎯
32. 22√ ab − 5a√ b + 7√ ab − 2a√ b

− (3√ x + 2√ ⎯⎯
y)
⎯⎯ ⎯⎯
33. 7√ x

( ) ( x)
⎯⎯ ⎯⎯ ⎯⎯ ⎯⎯
34. 8y √ x − 7x √ y − 5x √ y − 12y √

⎯⎯⎯⎯ ⎯⎯⎯⎯ ⎯⎯⎯⎯ ⎯⎯⎯⎯


35. √ 45 + √ 12 − √ 20 − √ 75
⎯⎯⎯⎯ ⎯⎯⎯⎯ ⎯⎯⎯⎯ ⎯⎯⎯⎯
36. √ 24 − √ 32 + √ 54 − 2√ 32
⎯⎯⎯⎯⎯⎯ ⎯⎯⎯⎯⎯⎯ ⎯⎯⎯⎯ ⎯⎯⎯⎯⎯⎯
37. 2√ 3x 2 + √ 45x − x√ 27 + √ 20x
⎯⎯⎯⎯⎯⎯⎯⎯ ⎯⎯⎯⎯⎯⎯⎯⎯⎯ ⎯⎯⎯⎯⎯⎯⎯⎯⎯⎯ ⎯⎯⎯⎯⎯⎯⎯⎯
38. 5√ 6a2 b + √ 8a2 b 2 − 2√ 24a2 b − a√ 18b 2

− (x√ 16y 3 − 2√ 9x 2 y 3 )
⎯⎯⎯⎯⎯⎯⎯⎯ ⎯⎯⎯⎯⎯⎯⎯⎯ ⎯⎯⎯⎯⎯⎯⎯⎯⎯⎯
39. 5y√ 4x 2 y

( ) ( )
⎯⎯⎯⎯⎯⎯⎯ ⎯⎯⎯⎯⎯⎯⎯⎯⎯⎯
2 c − 3a√ 16b 2 c −
⎯⎯⎯⎯⎯⎯⎯⎯⎯⎯⎯⎯⎯ ⎯⎯⎯⎯⎯
2 b 2 c − 9b√ a2 c
40. 2b√ 9a √ 64a

⎯⎯⎯⎯⎯⎯⎯⎯ 3 ⎯⎯⎯⎯⎯⎯⎯⎯⎯⎯⎯ 3 ⎯⎯⎯⎯


41. √ 216x −√ 125xy − √ 8x
3

⎯⎯⎯⎯⎯⎯⎯⎯⎯⎯ 3 ⎯⎯⎯⎯ 3 ⎯⎯⎯⎯⎯⎯


42. √ 128x 3 54 + 3 √ 2x 3
3
− 2x ⋅ √
⎯⎯⎯⎯⎯⎯⎯⎯ 3 ⎯⎯⎯⎯ 3 ⎯⎯⎯⎯⎯⎯⎯⎯⎯⎯ 3 ⎯⎯
43. √ 8x 3 y − 2x ⋅ √ 8y + √ 27x 3 y + x ⋅ √ y
3

8.7 Review Exercises and Sample Exam 1412


Chapter 8 Radical Expressions and Equations

⎯⎯⎯⎯⎯⎯⎯⎯⎯⎯ 3 ⎯⎯⎯⎯⎯⎯⎯⎯ 3 ⎯⎯⎯⎯⎯⎯ 3 ⎯⎯


44. √ 27a3 b − 3 √ 8ab 3 + a ⋅ √
3
64b − b ⋅ √ a

Multiplying and Dividing Radical Expressions

Multiply.

⎯⎯ ⎯⎯
45. √ 3 ⋅ √6

( 5)
⎯⎯ 2
46. 3 √

(√ 3 − √ 6 )
⎯⎯ ⎯⎯ ⎯⎯
47. √ 2

( 6)
⎯⎯ ⎯⎯ 2
48. √ 2 − √

( )( 5)
⎯⎯ ⎯⎯
49. 1 − √ 5 1 + √

(2√ 3 + √ 5 ) (3√ 2 − 2√ 5 )
⎯⎯ ⎯⎯ ⎯⎯ ⎯⎯
50.

⎯⎯⎯⎯⎯⎯ 3 ⎯⎯⎯⎯
51. √ 2a2
3
⋅√ 4a
⎯⎯⎯⎯⎯⎯⎯⎯⎯⎯ 3 ⎯⎯⎯⎯⎯⎯⎯⎯⎯
52. √ 25a2 b ⋅ √ 5a2 b 2
3

Divide.

√72
53.
√4

10√48
54.
√64

√98x 4 y 2
55.
√36x 2

8.7 Review Exercises and Sample Exam 1413


Chapter 8 Radical Expressions and Equations

3
√ 81x 6 y 7
56. 3
√ 8y 3

Rationalize the denominator.

2
57.
√7

√6
58.
√3

14
59.
√2x

1
60.
2√15

1
61. 3
√ 2x 2

5a2 b
62. 3
√5ab 2

1
63.
√3 −√2

√2 −√6
64.
√2 +√6

Rational Exponents

Express in radical form.

1/2
65. 7

2/3
66. 3

67. x 4/5

68. y −3/4

8.7 Review Exercises and Sample Exam 1414


Chapter 8 Radical Expressions and Equations

Write as a radical and then simplify.

1/2
69. 4

1/2
70. 50

2/3
71. 4

1/3
72. 81

73. (
4)
1 3/2

74. (
216 )
1 −1/3

Perform the operations and simplify. Leave answers in exponential form.

1/2
75. 3 ⋅ 3 3/2
1/2
76. 2 ⋅ 2 1/3

4 3/2
77.
4 1/2

9 3/4
78.
9 1/4

79. (36x 4 y 2 )
1/2

80. (8x 6 y 9 )
1/3

( a1/2 )
2/5
a4/3
81.

( y2 )
1/2
16x 4/3
82.

Solving Radical Equations

8.7 Review Exercises and Sample Exam 1415


Chapter 8 Radical Expressions and Equations

Solve.

⎯⎯
83. √ x =5
⎯⎯⎯⎯⎯⎯⎯⎯⎯⎯⎯
84. √ 2x −1 =3
⎯⎯⎯⎯⎯⎯⎯⎯⎯
85. √ x −8 +2=5
⎯⎯⎯⎯⎯⎯⎯⎯⎯
86. 3√ x − 5 − 1 = 11
⎯⎯⎯⎯⎯⎯⎯⎯⎯⎯⎯ ⎯⎯⎯⎯⎯⎯⎯⎯⎯⎯⎯⎯⎯
87. √ 5x − 3 = √ 2x + 15
⎯⎯⎯⎯⎯⎯⎯⎯⎯⎯⎯⎯⎯
88. √ 8x − 15 = x
⎯⎯⎯⎯⎯⎯⎯⎯⎯⎯⎯
89. √ x + 41 = x − 1
⎯⎯⎯⎯⎯⎯⎯⎯⎯⎯⎯
90. √ 7 − 3x = x − 3
⎯⎯⎯⎯⎯⎯⎯⎯⎯⎯⎯⎯⎯⎯
91. 2 (x + 1) = √ 2 (x + 1)

92. √ x (x + 6) = 4
⎯⎯⎯⎯⎯⎯⎯⎯⎯⎯⎯⎯⎯⎯⎯

⎯⎯⎯⎯⎯⎯⎯⎯⎯⎯⎯⎯⎯⎯⎯⎯⎯⎯
93. √ x (3x + 10) = 2
3

⎯⎯⎯⎯⎯⎯⎯⎯⎯⎯⎯⎯⎯
94. √ 2x 2
3
−x +4=5
⎯⎯⎯⎯⎯⎯⎯⎯⎯⎯⎯⎯⎯⎯⎯⎯⎯⎯⎯⎯⎯⎯⎯⎯⎯⎯ 3 ⎯⎯⎯⎯⎯⎯⎯⎯⎯⎯⎯⎯⎯
95. √ 3 (x + 4) (x + 1) = √ 5x + 37
3

⎯⎯⎯⎯⎯⎯⎯⎯⎯⎯⎯⎯⎯⎯⎯⎯⎯⎯⎯⎯⎯⎯⎯ 3 ⎯⎯⎯⎯⎯⎯⎯⎯⎯⎯⎯⎯2⎯
96. √ 3x 2 − 9x + 24 = √ (x + 2)
3

97. y 1/2 −3=0

98. y 1/3 +3=0

8.7 Review Exercises and Sample Exam 1416


Chapter 8 Radical Expressions and Equations

99. (x − 5)
1/2
−2=0

100. (2x − 1) 1/3 − 5 = 0

8.7 Review Exercises and Sample Exam 1417


Chapter 8 Radical Expressions and Equations

SAMPLE EXAM

In problems 1–18, assume all variables represent nonnegative numbers.

1. Simplify.

⎯⎯⎯⎯⎯⎯
a. √ 100
⎯⎯⎯⎯⎯⎯⎯⎯⎯
b. √ −100
⎯⎯⎯⎯⎯⎯
c. −√ 100

2. Simplify.

3 ⎯⎯⎯⎯
a. √ 27
3 ⎯⎯⎯⎯⎯⎯⎯
b. √ −27
3 ⎯⎯⎯⎯
c. −√ 27

3. √
⎯128
⎯⎯⎯⎯⎯
25

4. √
⎯192
⎯⎯⎯⎯⎯
3
125

⎯⎯⎯⎯⎯⎯⎯⎯⎯⎯⎯⎯⎯⎯
5. 5√ 12x 2 y 3 z

⎯⎯⎯⎯⎯⎯⎯⎯⎯⎯⎯⎯⎯⎯⎯⎯⎯
6. √ 250x 2 y 3 z 5
3

Perform the operations.

⎯⎯⎯⎯ ⎯⎯⎯⎯⎯⎯ ⎯⎯⎯⎯ ⎯⎯⎯⎯


7. 5√ 24 − √ 108 + √ 96 − 3√ 27

− (x√ 200y − √ 18x 2 y )


⎯⎯⎯⎯⎯⎯⎯⎯ ⎯⎯⎯⎯⎯⎯⎯⎯ ⎯⎯⎯⎯⎯⎯⎯⎯⎯⎯
8. 3√ 8x 2 y

( b)
⎯⎯⎯⎯ ⎯⎯⎯⎯ ⎯⎯
9. 2√ ab 3√ 2a − √

(√ √ )
⎯⎯ 2
10. x − 2 ⎯⎯
y

8.7 Review Exercises and Sample Exam 1418


Chapter 8 Radical Expressions and Equations

Rationalize the denominator.

10
11.
√2x

1
12. 3
√ 4xy 2

1
13.
√x +5

√2 −√3
14.
√2 +√3

Perform the operations and simplify. Leave answers in exponential form.

2/3
15. 2 ⋅ 2 1/6

10 4/5
16.
10 1/3

17. (121a4 b )
2 1/2

(9y x )
1/3 6 1/2

18.
y 1/6

Solve.

⎯⎯
19. √ x −7=0
⎯⎯⎯⎯⎯⎯⎯⎯⎯⎯⎯
20. √ 3x +5 =1
⎯⎯⎯⎯⎯⎯⎯⎯⎯⎯⎯
21. √ 2x −1 +2=x
⎯⎯⎯⎯⎯⎯⎯⎯⎯⎯⎯⎯⎯⎯⎯
22. √ 31 − 10x = x − 4
⎯⎯⎯⎯⎯⎯⎯⎯⎯⎯⎯⎯⎯⎯⎯⎯⎯⎯⎯⎯⎯⎯⎯⎯⎯⎯⎯ ⎯⎯⎯⎯⎯⎯⎯⎯⎯⎯⎯⎯⎯⎯⎯⎯
23. √ (2x + 1) (3x + 2) = √ 3 (2x + 1)

8.7 Review Exercises and Sample Exam 1419


Chapter 8 Radical Expressions and Equations

24. √ x (2x − 15) = 3


⎯⎯⎯⎯⎯⎯⎯⎯⎯⎯⎯⎯⎯⎯⎯⎯⎯⎯⎯
3

T = 2π √ 32
25. The period, T, of a pendulum in seconds is given the formula
⎯⎯⎯⎯
L

, where L represents the length in feet. Calculate the length
of a pendulum if the period is 1½ seconds. Round off to the nearest tenth.

8.7 Review Exercises and Sample Exam 1420


Chapter 8 Radical Expressions and Equations

REVIEW EXERCISES ANSWERS

1: 6

3: Not a real number

5: 5

7: 1/4

⎯⎯⎯⎯
9: 2√ 10

7√2
11:
9

⎯⎯
13: 20 √ 3
3

15: 7x

⎯⎯⎯⎯
17: 5xy√ 3x

3x√2x
19:
5y

21: 6x

3 ⎯⎯⎯⎯⎯⎯
23: 3a2 bc ⋅ √ ab 2
⎯⎯⎯⎯
25: √ 13

⎯⎯⎯⎯
27: 2√ 29

⎯⎯
29: 11√ 3

⎯⎯ ⎯⎯
31: 9√ 3 − √2
⎯⎯
33: 4√ x − 2√ ⎯⎯
y

8.7 Review Exercises and Sample Exam 1421


Chapter 8 Radical Expressions and Equations

⎯⎯ ⎯⎯
35: √ 5 − 3√ 3
⎯⎯ ⎯⎯⎯⎯
37: −x√ 3 + 5√ 5x

39: 12xy√ y
⎯⎯

⎯⎯ 3 ⎯⎯⎯⎯
41: 4 √
3
x − 5√ xy

3 ⎯⎯
43: 2x ⋅√ y

⎯⎯
45: 3√ 2

⎯⎯ ⎯⎯
47: √ 6 − 2√ 3

49: −4

51: 2a

⎯⎯
53: 3√ 2

7xy√2
55:
6

2√7
57: 7

7√2x
59: x

3
√ 4x
61:
2x

⎯⎯ ⎯⎯
63: √ 3 + √2
⎯⎯
65: √ 7

⎯⎯⎯⎯
67: √ x 4
5

8.7 Review Exercises and Sample Exam 1422


Chapter 8 Radical Expressions and Equations

69: 2

⎯⎯
71: 2 √ 2
3

73: 1/8

75: 9

77: 4

79: 6x 2 y

81: a1/3

83: 25

85: 17

87: 6

89: 8

91: −1/2, −1

93: 2/3, −4

95: −5, 5/3

97: 9

99: 9

8.7 Review Exercises and Sample Exam 1423


Chapter 8 Radical Expressions and Equations

SAMPLE EXAM ANSWERS

1:

a. 10
b. Not a real number
c. −10

8√2
3:
5

⎯⎯⎯⎯⎯⎯
5: 10xy√ 3yz

⎯⎯ ⎯⎯
7: 14√ 6 − 15√ 3
⎯⎯⎯⎯ ⎯⎯
9: 6a√ 2b − 2b√ a

5√2x
11: x

√x −5
13:
x−25

5/6
15: 2

17: 11a2 b

19: 49

21: 5

23: −1/2, 1/3

25: 1.8 feet

8.7 Review Exercises and Sample Exam 1424


Chapter 9
Solving Quadratic Equations and Graphing Parabolas

1425
Chapter 9 Solving Quadratic Equations and Graphing Parabolas

9.1 Extracting Square Roots

LEARNING OBJECTIVE

1. Solve quadratic equations by extracting square roots.

Extracting Square Roots

Recall that a quadratic equation is in standard form1 if it is equal to 0:

where a, b, and c are real numbers and a ≠ 0. A solution to such an equation is


called a root2. Quadratic equations can have two real solutions, one real solution, or
no real solution. If the quadratic expression on the left factors, then we can solve it
by factoring. A review of the steps used to solve by factoring follow:

Step 1: Express the quadratic equation in standard form.

Step 2: Factor the quadratic expression.

Step 3: Apply the zero-product property and set each variable factor equal to 0.

Step 4: Solve the resulting linear equations.

For example, we can solve x 2 − 4 = 0 by factoring as follows:

1. Any quadratic equation in the


form ax 2 + bx + c = 0,
where a, b, and c are real
numbers and a ≠ 0.

2. A solution to a quadratic
equation in standard form.

1426
Chapter 9 Solving Quadratic Equations and Graphing Parabolas

The two solutions are −2 and 2. The goal in this section is to develop an alternative
method that can be used to easily solve equations where b = 0, giving the form

The equation x 2 − 4 = 0 is in this form and can be solved by first isolating x 2 .

If we take the square root of both sides of this equation, we obtain the following:

Here we see that x = −2 and x = 2 are solutions to the resulting equation. In


general, this describes the square root property3; for any real number k,

The notation “±” is read “plus or minus” and is used as compact notation that
⎯⎯ ⎯⎯
indicates two solutions. Hence the statement x = ±√k indicates that x = −√k or
⎯⎯
x = √k . Applying the square root property as a means of solving a quadratic
equation is called extracting the roots4.

3. For any real number k, if


⎯⎯ Example 1: Solve: x 2 − 25 = 0.
x 2 = k, then x = ±√k .
4. Applying the square root
property as a means of solving Solution: Begin by isolating the square.
a quadratic equation.

9.1 Extracting Square Roots 1427


Chapter 9 Solving Quadratic Equations and Graphing Parabolas

Next, apply the square root property.

Answer: The solutions are −5 and 5. The check is left to the reader.

Certainly, the previous example could have been solved just as easily by factoring.
However, it demonstrates a technique that can be used to solve equations in this
form that do not factor.

Example 2: Solve: x 2 − 5 = 0.

Solution: Notice that the quadratic expression on the left does not factor. We can
extract the roots if we first isolate the leading term, x 2 .

Apply the square root property.

9.1 Extracting Square Roots 1428


Chapter 9 Solving Quadratic Equations and Graphing Parabolas

For completeness, check that these two real solutions solve the original quadratic
equation. Generally, the check is optional.

⎯⎯ ⎯⎯
Answer: The solutions are −√5 and √5.

Example 3: Solve: 4x 2 − 45 = 0.

Solution: Begin by isolating x 2 .

Apply the square root property and then simplify.

9.1 Extracting Square Roots 1429


Chapter 9 Solving Quadratic Equations and Graphing Parabolas

3√5 3√5
Answer: The solutions are − 2
and 2
.

Sometimes quadratic equations have no real solution.

Example 4: Solve: x 2 + 9 = 0.

Solution: Begin by isolating x 2 .

After applying the square root property, we are left with the square root of a
negative number. Therefore, there is no real solution to this equation.

Answer: No real solution

Reverse this process to find equations with given solutions of the form ±k.

9.1 Extracting Square Roots 1430


Chapter 9 Solving Quadratic Equations and Graphing Parabolas

⎯⎯ ⎯⎯
Example 5: Find an equation with solutions −2√3 and 2√3.

Solution: Begin by squaring both sides of the following equation:

Lastly, subtract 12 from both sides and present the equation in standard form.

Answer: x 2 − 12 = 0

Try this! Solve: 9x 2 − 8 = 0.

2√2 2√2
Answer: x = − 3
or x = 3

Video Solution

(click to see video)

Consider solving the following equation:

To solve this equation by factoring, first square x + 2 and then put it in standard
form, equal to zero, by subtracting 25 from both sides.

9.1 Extracting Square Roots 1431


Chapter 9 Solving Quadratic Equations and Graphing Parabolas

Factor and then apply the zero-product property.

The two solutions are −7 and 3.

When an equation is in this form, we can obtain the solutions in fewer steps by
extracting the roots.

Example 6: Solve: (x + 2)2 = 25.

Solution: Solve by extracting the roots.

At this point, separate the “plus or minus” into two equations and simplify each
individually.

9.1 Extracting Square Roots 1432


Chapter 9 Solving Quadratic Equations and Graphing Parabolas

Answer: The solutions are −7 and 3.

In addition to fewer steps, this method allows us to solve equations that do not
factor.

Example 7: Solve: (3x + 3)2 − 27 = 0.

Solution: Begin by isolating the square.

Next, extract the roots and simplify.

Solve for x.

9.1 Extracting Square Roots 1433


Chapter 9 Solving Quadratic Equations and Graphing Parabolas

⎯⎯ ⎯⎯
Answer: The solutions are −1 − √3 and −1 + √3.

Example 8: Solve: 9(2x − 1)2 − 8 = 0.

Solution: Begin by isolating the square factor.

Apply the square root property and solve.

9.1 Extracting Square Roots 1434


Chapter 9 Solving Quadratic Equations and Graphing Parabolas

3−2√2 3+2√2
Answer: The solutions are 6
and 6
.

Try this! Solve: 3(x − 5) − 2 = 0.


2

15±√6
Answer: 3

Video Solution

(click to see video)

Example 9: The length of a rectangle is twice its width. If the diagonal measures 2
feet, then find the dimensions of the rectangle.

9.1 Extracting Square Roots 1435


Chapter 9 Solving Quadratic Equations and Graphing Parabolas

Solution:

The diagonal of any rectangle forms two right triangles. Thus the Pythagorean
theorem applies. The sum of the squares of the legs of a right triangle is equal to the
square of the hypotenuse:

Solve.

9.1 Extracting Square Roots 1436


Chapter 9 Solving Quadratic Equations and Graphing Parabolas

Here we obtain two solutions, w = − 2


and w = 2
. Since the problem asked for a
√5 √5

length of a rectangle, we disregard the negative answer. Furthermore, we will


rationalize the denominator and present our solutions without any radicals in the
denominator.

Back substitute to find the length.

4√5 2√5
Answer: The length of the rectangle is 5
feet and the width is 5
feet.

KEY TAKEAWAYS

• Solve equations of the form ax 2 + c = 0by extracting the roots.


• Extracting roots involves isolating the square and then applying the
square root property. After applying the square root property, you have
two linear equations that each can be solved. Be sure to simplify all
radical expressions and rationalize the denominator if necessary.

9.1 Extracting Square Roots 1437


Chapter 9 Solving Quadratic Equations and Graphing Parabolas

TOPIC EXERCISES

Part A: Extracting Square Roots

Solve by factoring and then solve by extracting roots. Check answers.

1. x 2 − 36 = 0

2. x 2 − 81 = 0

3. 4y 2 −9=0

4. 9y 2 − 25 = 0

5. (x − 2) 2 − 1 = 0

6. (x + 1) 2 − 4 = 0

7. 4(y − 2) − 9 = 0
2

8. 9(y + 1) − 4 = 0
2

9. −3(t − 1)2 + 12 = 0

10. −2(t + 1)2 + 8 = 0

11. (x − 5) − 25 = 0
2

12. (x + 2) 2 − 4 = 0

Solve by extracting the roots.

13. x 2 = 16

14. x 2 =1

9.1 Extracting Square Roots 1438


Chapter 9 Solving Quadratic Equations and Graphing Parabolas

15. y 2 =9

16. y 2 = 64
1
17. x 2 = 4

1
18. x 2 = 9

19. y 2 = 0.25

20. y 2 = 0.04

21. x 2 = 12

22. x 2 = 18

23. 16x 2 =9

24. 4x 2 = 25

25. 2t 2 =1

26. 3t 2 =2

27. x 2 − 100 = 0

28. x 2 − 121 = 0

29. y 2 +4=0

30. y 2 +1=0
4
31. x 2 − =0
9

9
32. x 2 − =0
25

9.1 Extracting Square Roots 1439


Chapter 9 Solving Quadratic Equations and Graphing Parabolas

33. y 2 − 0.09 = 0

34. y 2 − 0.81 = 0

35. x 2 −7=0

36. x 2 −2=0

37. x 2 −8=0

38. t 2 − 18 = 0

39. x 2 +8=0

40. x 2 + 125 = 0

41. 16x 2 − 27 = 0

42. 9x 2 −8=0

43. 2y 2 −3=0

44. 5y 2 −2=0

45. 3x 2 −1=0

46. 6x 2 −3=0

47. (x + 7) 2 − 4 = 0

48. (x + 9) 2 − 36 = 0

49. (2y − 3) − 81 = 0
2

50. (2y + 1) − 25 = 0
2

9.1 Extracting Square Roots 1440


Chapter 9 Solving Quadratic Equations and Graphing Parabolas

51. (x − 5) − 20 = 0
2

52. (x + 1) 2 − 28 = 0

53. (3t + 2)2 − 6 = 0

54. (3t − 5) − 10 = 0
2

55. 4(y + 2) − 3 = 0
2

56. 9(y − 7) − 5 = 0
2

57. 4(3x + 1) 2 − 27 = 0

58. 9(2x − 3) 2 − 8 = 0

59. 2(3x − 1) 2 + 3 = 0

60. 5(2x − 1) 2 − 3 = 0

61. 3(y − 23 ) −
2 3
2
=0

62. 2(3y − 13 ) −
2 5
2
=0

Find a quadratic equation in standard form with the following solutions.

63. ±7

64. ±13

⎯⎯
65. ±√ 7

⎯⎯
66. ±√ 3

⎯⎯
67. ±3√ 5

9.1 Extracting Square Roots 1441


Chapter 9 Solving Quadratic Equations and Graphing Parabolas

⎯⎯
68. ±5√ 2

⎯⎯
69. 1 ± √2
⎯⎯
70. 2 ± √3

Solve and round off the solutions to the nearest hundredth.

71. 9x (x + 2) = 18x + 1

72. x 2 = 10 (x 2 − 2) − 5

73. (x + 3) (x − 7) = 11 − 4x

74. (x − 4) (x − 3) = 66 − 7x

75. (x − 2) 2 = 67 − 4x

76. (x + 3) 2 = 6x + 59

77. (2x + 1) (x + 3) − (x + 7) = (x + 3) 2

78. (3x − 1) (x + 4) = 2x (x + 6) − (x − 3)

Set up an algebraic equation and use it to solve the following.

79. If 9 is subtracted from 4 times the square of a number, then the result is
3. Find the number.

80. If 20 is subtracted from the square of a number, then the result is 4. Find
the number.

81. If 1 is added to 3 times the square of a number, then the result is 2. Find
the number.

82. If 3 is added to 2 times the square of a number, then the result is 12. Find
the number.

9.1 Extracting Square Roots 1442


Chapter 9 Solving Quadratic Equations and Graphing Parabolas

83. If a square has an area of 8 square centimeters, then find the length of
each side.

84. If a circle has an area of 32π square centimeters, then find the length of
the radius.

85. The volume of a right circular cone is 36π cubic centimeters when the
height is 6 centimeters. Find the radius of the cone. (The volume of a right
circular cone is given by V = 1
3
πr2 h.)

86. The surface area of a sphere is 75π square centimeters. Find the radius
of the sphere. (The surface area of a sphere is given by SA = 4πr2 .)

87. The length of a rectangle is 6 times its width. If the area is 96 square
inches, then find the dimensions of the rectangle.

88. The base of a triangle is twice its height. If the area is 16 square
centimeters, then find the length of its base.

89. A square has an area of 36 square units. By what equal amount will the
sides have to be increased to create a square with double the given area?

90. A circle has an area of 25π square units. By what amount will the radius
have to be increased to create a circle with double the given area?

91. If the sides of a square measure 1 unit, then find the length of the
diagonal.

92. If the sides of a square measure 2 units, then find the length of the
diagonal.

93. The diagonal of a square measures 5 inches. Find the length of each side.

94. The diagonal of a square measures 3 inches. Find the length of each side.

95. The length of a rectangle is twice its width. If the diagonal measures 10
feet, then find the dimensions of the rectangle.

96. The length of a rectangle is twice its width. If the diagonal measures 8
feet, then find the dimensions of the rectangle.

9.1 Extracting Square Roots 1443


Chapter 9 Solving Quadratic Equations and Graphing Parabolas

97. The length of a rectangle is 3 times its width. If the diagonal measures 5
meters, then find the dimensions of the rectangle.

98. The length of a rectangle is 3 times its width. If the diagonal measures 2
feet, then find the dimensions of the rectangle.

99. The height in feet of an object dropped from a 9‑foot ladder is given by
h(t) = −16t 2 + 9, where t represents the time in seconds after the
object has been dropped. How long does it take the object to hit the ground?
(Hint: The height is 0 when the object hits the ground.)

100. The height in feet of an object dropped from a 20‑foot platform is given
by h(t) = −16t 2 + 20 , where t represents the time in seconds after the
object has been dropped. How long does it take the object to hit the ground?

101. The height in feet of an object dropped from the top of a 144-foot
building is given by h(t) = −16t 2 + 144 , where t is measured in
seconds.

a. How long will it take to reach half of the distance to the ground, 72 feet?

b. How long will it take to travel the rest of the distance to the ground?

Round off to the nearest hundredth of a second.

102. The height in feet of an object dropped from an airplane at 1,600 feet is
given by h(t) = −16t 2 + 1,600 , where t is in seconds.

a. How long will it take to reach half of the distance to the ground?

b. How long will it take to travel the rest of the distance to the ground?

Round off to the nearest hundredth of a second.

Part B: Discussion Board

103. Create an equation of your own that can be solved by extracting the
root. Share it, along with the solution, on the discussion board.

9.1 Extracting Square Roots 1444


Chapter 9 Solving Quadratic Equations and Graphing Parabolas

104. Explain why the technique of extracting roots greatly expands our
ability to solve quadratic equations.

105. Explain in your own words how to solve by extracting the roots.

106. Derive a formula for the diagonal of a square in terms of its sides.

9.1 Extracting Square Roots 1445


Chapter 9 Solving Quadratic Equations and Graphing Parabolas

ANSWERS

1: −6, 6

3: −3/2, 3/2

5: 1, 3

7: 1/2, 7/2

9: −1, 3

11: 0, 10

13: ±4

15: ±3

17: ±1/2

19: ±0.5

⎯⎯
21: ±2√ 3

23: ±3/4

√2
25: ±
2

27: ±10

29: No real solution

31: ±2/3

33: ±0.3

⎯⎯
35: ±√ 7

9.1 Extracting Square Roots 1446


Chapter 9 Solving Quadratic Equations and Graphing Parabolas

⎯⎯
37: ±2√ 2

39: No real solution

3√3
41: ±
4

√6
43: ±
2

√3
45: ±
3

47: −9, −5

49: −3, 6

⎯⎯
51: 5 ± 2√ 5

−2±√6
53:
3

−4±√3
55:
2

−2±3√3
57:
6

59: No real solution

4±3√2
61:
6

63: x 2 − 49 = 0

65: x 2 −7=0

67: x 2 − 45 = 0

69: x 2 − 2x − 1 = 0

9.1 Extracting Square Roots 1447


Chapter 9 Solving Quadratic Equations and Graphing Parabolas

71: ±0.33

73: ±5.66

75: ±7.94

77: ±3.61

⎯⎯ ⎯⎯
79: −√ 3 or √ 3

81: −
√3 √3
3
or 3

⎯⎯
83: 2√ 2 centimeters

⎯⎯
85: 3√ 2 centimeters

87: Length: 24 inches; width: 4 inches

⎯⎯
89: −6 + 6√ 2 ≈ 2.49 units
⎯⎯
91: √ 2 units

5√2
93: 2
inches

⎯⎯ ⎯⎯
95: Length: 4√ 5 feet; width: 2√ 5 feet

3√10 √10
97: Length: 2
meters; width: 2
meters

99: 3/4 second

101: a. 2.12 seconds; b. 0.88 second

9.1 Extracting Square Roots 1448


Chapter 9 Solving Quadratic Equations and Graphing Parabolas

9.2 Completing the Square

LEARNING OBJECTIVE

1. Solve quadratic equations by completing the square.

Completing the Square

In this section, we will devise a method for rewriting any quadratic equation of the
form

in the form

This process is called completing the square5. As we have seen, quadratic


equations in this form can easily be solved by extracting roots. We begin by
examining perfect square trinomials:

The last term, 9, is the square of one-half of the coefficient of x. In general, this is
true for any perfect square trinomial of the form x 2 + bx + c.

5. The process of rewriting a

(x − p) = q.
quadratic equation in the form
2

1449
Chapter 9 Solving Quadratic Equations and Graphing Parabolas

In other words, any trinomial of the form x 2 + bx + c will be a perfect square


trinomial if

Note

It is important to point out that the leading coefficient must be equal to 1 for
this to be true.

Example 1: Complete the square: x 2 + 8x + ? = (x + ? ) .


2

Solution: In this example, the coefficient of the middle term b = 8, so find the value
that completes the square as follows:

The value that completes the square is 16.

9.2 Completing the Square 1450


Chapter 9 Solving Quadratic Equations and Graphing Parabolas

Answer: x 2 + 8x + 16 = (x + 4)2

Example 2: Complete the square: x 2 + 3x + ? = (x + ? ) .


2

Solution: Here b = 3, so find the value that will complete the square as follows:

The value 9/4 completes the square:

= (x + 32 )
2
Answer: x 2 + 3x + 9
4

We can use this technique to solve quadratic equations. The idea is to take any
quadratic equation in standard form and complete the square so that we can solve
it by extracting roots. The following are general steps for solving a quadratic
equation with a leading coefficient of 1 in standard form by completing the square.

9.2 Completing the Square 1451


Chapter 9 Solving Quadratic Equations and Graphing Parabolas

Example 3: Solve by completing the square: x 2 + 14x + 46 = 0.

Solution:

Step 1: Add or subtract the constant term to obtain the equation in the form
x 2 + bx = c . In this example, subtract 46 to move it to the right side of the
equation.

Step 2: Use ( b2 ) to determine the value that completes the square. Here b = 14:
2

Step 3: Add ( b2 ) to both sides of the equation and complete the square.
2

Step 4: Solve by extracting roots.

9.2 Completing the Square 1452


Chapter 9 Solving Quadratic Equations and Graphing Parabolas

⎯⎯ ⎯⎯
Answer: The solutions are −7 − √3 or −7 + √3. The check is optional.

Example 4: Solve by completing the square: x 2 − 18x + 72 = 0.

Solution: Begin by subtracting 72 from both sides.

Next, find the value that completes the square using b = −18.

To complete the square, add 81 to both sides, complete the square, and then solve
by extracting the roots.

At this point, separate the “plus or minus” into two equations and solve each.

9.2 Completing the Square 1453


Chapter 9 Solving Quadratic Equations and Graphing Parabolas

Answer: The solutions are 6 and 12.

Note that in the previous example the solutions are integers. If this is the case, then
the original equation will factor.

If it factors, we can solve it by factoring. However, not all quadratic equations will
factor.

Example 5: Solve by completing the square: x 2 + 10x + 1 = 0.

Solution: Begin by subtracting 1 from both sides of the equation.

Here b = 10, and we determine the value that completes the square as follows:

9.2 Completing the Square 1454


Chapter 9 Solving Quadratic Equations and Graphing Parabolas

To complete the square, add 25 to both sides of the equation.

Factor and then solve by extracting roots.

⎯⎯ ⎯⎯
Answer: The solutions are −5 − 2√6 and −5 + 2√6.

Sometimes quadratic equations do not have real solutions.

Example 6: Solve by completing the square: x 2 − 2x + 3 = 0.

Solution: Begin by subtracting 3 from both sides of the equation.

9.2 Completing the Square 1455


Chapter 9 Solving Quadratic Equations and Graphing Parabolas

Here b = −2, and we have

Therefore,

At this point we see that extracting the root leads to the square root of a negative
number.

Answer: No real solution

Try this! Solve by completing the square: x 2 − 2x − 27 = 0.

⎯⎯
Answer: x = 1 ± 2√7

Video Solution

(click to see video)

The coefficient of x is not always divisible by 2.

9.2 Completing the Square 1456


Chapter 9 Solving Quadratic Equations and Graphing Parabolas

Example 7: Solve by completing the square: x 2 + 3x − 2 = 0.

Solution: Begin by adding 2 to both sides.

Use b = 3 to find the value that completes the square:

To complete the square, add 9/4 to both sides of the equation.

Solve by extracting roots.

9.2 Completing the Square 1457


Chapter 9 Solving Quadratic Equations and Graphing Parabolas

−3±√17
Answer: The solutions are 2
.

So far, all of the examples have had a leading coefficient of 1. The formula ( b2 )
2

determines the value that completes the square only if the leading coefficient is 1. If
this is not the case, then simply divide both sides by the leading coefficient.

Example 8: Solve by completing the square: 2x 2 + 5x − 1 = 0.

Solution: Notice that the leading coefficient is 2. Therefore, divide both sides by 2
before beginning the steps required to solve by completing the square.

9.2 Completing the Square 1458


Chapter 9 Solving Quadratic Equations and Graphing Parabolas

Begin by adding 1/2 to both sides of the equation.

Here b = 5/2, and we can find the value that completes the square as follows:

To complete the square, add 25/16 to both sides of the equation.

Next, solve by extracting roots.

9.2 Completing the Square 1459


Chapter 9 Solving Quadratic Equations and Graphing Parabolas

−5±√33
Answer: The solutions are 4
.

Try this! Solve: 2x 2 − 2x − 3 = 0.

1±√7
Answer: x = 2

Video Solution

(click to see video)

KEY TAKEAWAYS

• Solve any quadratic equation by completing the square.

first convert the equation to the form (x − p) = q.


• You can apply the square root property to solve an equation if you can
2

x 2 + bx = c. Then add the value ( b2 )


• To complete the square, first make sure the equation is in the form
2
to both sides and factor.
• The process for completing the square always works, but it may lead to
some tedious calculations with fractions. This is the case when the
middle term, b, is not divisible by 2.

9.2 Completing the Square 1460


Chapter 9 Solving Quadratic Equations and Graphing Parabolas

TOPIC EXERCISES

Part A: Completing the Square

Complete the square.

+ 6x + ? = (x + ? )
2
1. x 2

+ 8x + ? = (x + ? )
2
2. x 2

− 2x + ? = (x − ? )
2
3. x 2

− 4x + ? = (x − ? )
2
4. x 2

+ 7x + ? = (x + ? )
2
5. x 2

+ 3x + ? = (x + ? )
2
6. x 2

x + ? = (x + ? )
2 2
7. x 2 + 3

x + ? = (x + ? )
4 2
8. x 2 + 5

x + ? = (x + ? )
3 2
9. x 2 + 4

x + ? = (x + ? )
5 2
10. x 2 + 3

Solve by factoring and then solve by completing the square. Check answers.

11. x 2 + 2x − 8 = 0

12. x 2 − 8x + 15 = 0

13. y 2 + 2y − 24 = 0

9.2 Completing the Square 1461


Chapter 9 Solving Quadratic Equations and Graphing Parabolas

14. y 2 − 12y + 11 = 0

15. t 2 + 3t − 28 = 0

16. t 2 − 7t + 10 = 0

17. 2x 2 + 3x − 2 = 0

18. 3x 2 −x−2=0

19. 2y 2 −y−1=0

20. 2y 2 + 7y − 4 = 0

Solve by completing the square.

21. x 2 + 6x − 1 = 0

22. x 2 + 8x + 10 = 0

23. x 2 − 2x − 7 = 0

24. x 2 − 6x − 3 = 0

25. x 2 − 2x + 4 = 0

26. x 2 − 4x + 9 = 0

27. t 2 + 10t − 75 = 0

28. t 2 + 12t − 108 = 0

29. x 2 − 4x − 1 = 15

30. x 2 − 12x + 8 = −10

31. y 2 − 20y = −25

9.2 Completing the Square 1462


Chapter 9 Solving Quadratic Equations and Graphing Parabolas

32. y 2 + 18y = −53

33. x 2 − 0.6x − 0.27 = 0

34. x 2 − 1.6x − 0.8 = 0

35. x 2 − 2
3
x− 1
3
=0

36. x 2 − 4
5
x− 1
5
=0

37. x 2 +x−1=0

38. x 2 +x−3=0

39. y 2 + 3y − 2 = 0

40. y 2 + 5y − 3 = 0

41. x 2 + 3x + 5 = 0

42. x 2 +x+1=0
11
43. x 2 − 7x + =0
2

3
44. x 2 − 9x + =0
2

45. t 2 − 1
2
t−1=0

46. t 2 − 1
3
t−2=0

47. x 2 − 1.7x − 0.0875 = 0

48. x 2 + 3.3x − 1.2775 = 0

49. 4x 2 − 8x − 1 = 0

9.2 Completing the Square 1463


Chapter 9 Solving Quadratic Equations and Graphing Parabolas

50. 2x 2 − 4x − 3 = 0

51. 3x 2 + 6x + 1 = 0

52. 5x 2 + 10x + 2 = 0

53. 3x 2 + 2x − 3 = 0

54. 5x 2 + 2x − 5 = 0

55. 4x 2 − 12x − 15 = 0

56. 2x 2 + 4x − 43 = 0

57. 2x 2 − 4x + 10 = 0

58. 6x 2 − 24x + 42 = 0

59. 2x 2 −x−2=0

60. 2x 2 + 3x − 1 = 0

61. 3x 2 + 2x − 2 = 0

62. 3x 2 −x−1=0

63. x (x + 1) − 11 (x − 2) = 0

64. (x + 1) (x + 7) − 4 (3x + 2) = 0

65. y 2 = (2y + 3) (y − 1) − 2 (y − 1)

66. (2y + 5) (y − 5) − y (y − 8) = −24

67. (t + 2)2 = 3 (3t + 1)

68. (3t + 2) (t − 4) − (t − 8) = 1 − 10t

9.2 Completing the Square 1464


Chapter 9 Solving Quadratic Equations and Graphing Parabolas

Solve by completing the square and round off the solutions to the nearest hundredth.

69. (2x − 1) 2 = 2x

70. (3x − 2) 2 = 5 − 15x

71. (2x + 1) (3x + 1) = 9x + 4

72. (3x + 1) (4x − 1) = 17x − 4

73. 9x (x − 1) − 2 (2x − 1) = −4x

74. (6x + 1) − 6 (6x + 1) = 0


2

Part B: Discussion Board

75. Research and discuss the Hindu method for completing the square.

76. Explain why the technique for completing the square described in this
section requires that the leading coefficient be equal to 1.

9.2 Completing the Square 1465


Chapter 9 Solving Quadratic Equations and Graphing Parabolas

ANSWERS

+ 6x + 9 = (x + 3)
2
1: x 2

− 2x + 1 = (x − 1)
2
3: x 2

5: x 2 + 7x + 49
4
= (x + 2)
7 2

7: x 2 + 2
3
x+ 1
9
= (x + 3)
1 2

9: x 2 + 3
4
x+ 9
64
= (x + 8)
3 2

11: −4, 2

13: −6, 4

15: −7, 4

17: 1/2, −2

19: −1/2, 1

⎯⎯⎯⎯
21: −3 ± √ 10
⎯⎯
23: 1 ± 2√ 2

25: No real solution

27: −15, 5

⎯⎯
29: 2 ± 2√ 5
⎯⎯
31: 10 ± 5√ 3

33: −0.3, 0.9

9.2 Completing the Square 1466


Chapter 9 Solving Quadratic Equations and Graphing Parabolas

35: −1/3, 1

−1±√5
37:
2

−3±√17
39:
2

41: No real solution

7±3√3
43:
2

1±√17
45:
4

47: −0.05, 1.75

2±√5
49:
2

−3±√6
51:
3

−1±√10
53:
3

3±2√6
55:
2

57: No real solution

1±√17
59:
4

−1±√7
61:
3

⎯⎯
63: 5 ± √3

1±√5
65:
2

9.2 Completing the Square 1467


Chapter 9 Solving Quadratic Equations and Graphing Parabolas

5±√21
67:
2

69: 0.19, 1.31

71: −0.45, 1.12

73: 0.33, 0.67

9.2 Completing the Square 1468


Chapter 9 Solving Quadratic Equations and Graphing Parabolas

9.3 Quadratic Formula

LEARNING OBJECTIVE

1. Solve quadratic equations with real solutions using the quadratic


formula.

The Quadratic Formula

In this section, we will develop a formula that gives the solutions to any quadratic
equation in standard form. To do this, we begin with a general quadratic equation
in standard form and solve for x by completing the square. Here a, b, and c are real
numbers and a ≠ 0:

Determine the constant that completes the square: take the coefficient of x, divide it
by 2, and then square it.

Add this to both sides of the equation and factor.

1469
Chapter 9 Solving Quadratic Equations and Graphing Parabolas

Solve by extracting roots.

This derivation gives us a formula that solves any quadratic equation in standard
form. Given ax 2 + bx + c = 0, where a, b, and c are real numbers and a ≠ 0, then
the solutions can be calculated using the quadratic formula6:

6. The formula
−b±√b 2 −4ac
x= 2a
, which
gives the solutions to any
quadratic equation in the form
ax 2 + bx + c = 0, where a, Consider the quadratic equation 2x 2 − 7x + 3 = 0. It can be solved by factoring as
b, and c are real numbers and
a ≠ 0. follows:

9.3 Quadratic Formula 1470


Chapter 9 Solving Quadratic Equations and Graphing Parabolas

The solutions are 1/2 and 3. The following example shows that we can obtain the
same results using the quadratic formula.

Example 1: Solve using the quadratic formula: 2x 2 − 7x + 3 = 0.

Solution: Begin by identifying a, b, and c as the coefficients of each term.

Substitute these values into the quadratic formula and then simplify.

9.3 Quadratic Formula 1471


Chapter 9 Solving Quadratic Equations and Graphing Parabolas

Separate the “plus or minus” into two equations and simplify each individually.

Answer: The solutions are 1/2 and 3.

Of course, if the quadratic factors, then it is a best practice to solve it by factoring.


However, not all quadratic polynomials factor; nevertheless, the quadratic formula
provides us with a means to solve such equations.

Example 2: Solve using the quadratic formula: 5x 2 + 2x − 1 = 0.

Solution: Begin by identifying a, b, and c.

Substitute these values into the quadratic formula.

9.3 Quadratic Formula 1472


Chapter 9 Solving Quadratic Equations and Graphing Parabolas

−1±√6
Answer: The solutions are 5
.

Often terms are missing. When this is the case, use 0 as the coefficient.

Example 3: Solve using the quadratic formula: x 2 − 18 = 0.

Solution: Think of this equation with the following coefficients:

Here

9.3 Quadratic Formula 1473


Chapter 9 Solving Quadratic Equations and Graphing Parabolas

Substitute these values into the quadratic formula.

⎯⎯
Answer: The solutions are ±3√2.

Since the coefficient of x was 0, we could have solved the equation by extracting the
roots. As an exercise, solve the previous example using this method and verify that
the results are the same.

Example 4: Solve using the quadratic formula: 9x 2 − 12x + 4 = 0.

Solution: In this case,

9.3 Quadratic Formula 1474


Chapter 9 Solving Quadratic Equations and Graphing Parabolas

Substitute these values into the quadratic formula and then simplify.

In this example, notice that the radicand of the square root is 0. This results in only
one solution to this quadratic equation. Normally, we expect two solutions. When
we find only one solution, the solution is called a double root. If we solve this
equation by factoring, then the solution appears twice.

Answer: 2/3, double root

9.3 Quadratic Formula 1475


Chapter 9 Solving Quadratic Equations and Graphing Parabolas

Example 5: Solve using the quadratic formula: x 2 + x + 1 = 0.

Solution: In this case,

Substitute these values into the quadratic formula.

The solution involves the square root of a negative number; hence the solutions are
not real. This quadratic equation has two nonreal solutions and will be discussed in
further detail as we continue in our study of algebra. For now, simply state that the
equation does not have real solutions.

Answer: No real solutions

Try this! Solve: x 2 − 2x − 2 = 0.

⎯⎯
Answer: 1 ± √3

Video Solution

(click to see video)

9.3 Quadratic Formula 1476


Chapter 9 Solving Quadratic Equations and Graphing Parabolas

It is important to place the quadratic equation in standard form before using the
quadratic formula.

Example 6: Solve using the quadratic formula: (2x + 1) (2x − 1) = 24x + 8.

Solution: Begin by using the distributive property to expand the left side and
combining like terms to obtain an equation in standard form, equal to 0.

Once the equation is in standard form, identify a, b, and c. Here

Substitute these values into the quadratic formula and then simplify.

9.3 Quadratic Formula 1477


Chapter 9 Solving Quadratic Equations and Graphing Parabolas

6±3√5
Answer: The solutions are 2
.

Try this! Solve: 3x (x − 2) = 1.

3±2√3
Answer: 3

Video Solution

(click to see video)

9.3 Quadratic Formula 1478


Chapter 9 Solving Quadratic Equations and Graphing Parabolas

KEY TAKEAWAYS

• Use the quadratic formula to solve any quadratic equation in standard


form.
• To solve any quadratic equation, first rewrite in standard form,
ax 2 + bx + c = 0, substitute the appropriate coefficients into the
−b±√b 2 −4ac
quadratic formula, x = , and then simplify.
2a

9.3 Quadratic Formula 1479


Chapter 9 Solving Quadratic Equations and Graphing Parabolas

TOPIC EXERCISES

Part A: Quadratic Formula

Identify the coefficients a, b, and c used in the quadratic formula. Do not solve.

1. x 2 −x+5=0

2. x 2 − 3x − 1 = 0

3. 3x 2 − 10 = 0

4. −y 2 +5=0

5. 5t 2 − 7t = 0

6. −y 2 +y=0

7. −x 2 + x = −6

8. −2x 2 − x = −15

9. (3x + 1) (2x + 5) = 19x + 4

10. (4x + 1) (2x + 1) = 16x + 4

Solve by factoring and then solve using the quadratic formula. Check answers.

11. x 2 − 10x + 24 = 0

12. x 2 − 3x − 18 = 0

13. t 2 + 6t + 5 = 0

14. t 2 + 9t + 14 = 0

15. 2x 2 − 7x − 4 = 0

9.3 Quadratic Formula 1480


Chapter 9 Solving Quadratic Equations and Graphing Parabolas

16. 3x 2 −x−2=0

17. −2x 2 −x+3=0

18. −6x 2 +x+1=0

19. y 2 − 2y + 1 = 0

20. y 2 −1=0

Use the quadratic formula to solve the following.

21. x 2 − 6x + 4 = 0

22. x 2 − 4x + 1 = 0

23. x 2 + 2x − 5 = 0

24. x 2 + 4x − 6 = 0

25. t 2 − 4t − 1 = 0

26. t 2 − 8t − 2 = 0

27. −y 2 +y+1=0

28. −y 2 − 3y + 2 = 0

29. −x 2 + 16x − 62 = 0

30. −x 2 + 14x − 46 = 0

31. 2t 2 − 4t − 3 = 0

32. 4t 2 − 8t − 1 = 0

33. −4y 2 + 12y − 9 = 0

9.3 Quadratic Formula 1481


Chapter 9 Solving Quadratic Equations and Graphing Parabolas

34. −25x 2 + 10x − 1 = 0

35. 3x 2 + 6x + 2 = 0

36. 5x 2 + 10x + 2 = 0

37. 9t 2 + 6t − 11 = 0

38. 8t 2 + 8t + 1 = 0

39. x 2 −2=0

40. x 2 − 18 = 0

41. 9x 2 −3=0

42. 2x 2 −5=0

43. y 2 +9=0

44. y 2 +1=0

45. 2x 2 =0

46. x 2 =0

47. −2y 2 + 5y = 0

48. −3y 2 + 7y = 0

49. t 2 −t =0

50. t 2 + 2t = 0

51. x 2 − 0.6x − 0.27 = 0

52. x 2 − 1.6x − 0.8 = 0

9.3 Quadratic Formula 1482


Chapter 9 Solving Quadratic Equations and Graphing Parabolas

53. y 2 − 1.4y − 0.15 = 0

54. y 2 − 3.6y + 2.03 = 0

55. 1
2
t 2 + 5t + 3
2
=0

56. −t 2 + 3t − 3
4
=0

57. 3y 2 + 1
2
y− 1
3
=0

58. −2y 2 + 1
3
y+ 1
2
=0

59. 2x 2 − 10x + 3 = 4

60. 3x 2 + 6x + 1 = 8

61. −2y 2 = 3(y − 1)

62. 3y 2 = 5(2y − 1)

63. (t + 1)2 = 2t + 7

64. (2t − 1)2 = 73 − 4t

65. (x + 5) (x − 1) = 2x + 1

66. (x + 7) (x − 2) = 3 (x + 1)

67. x (x + 5) = 3 (x − 1)

68. x (x + 4) = −7

69. (5x + 3) (5x − 3) − 10 (x − 1) = 0

70. (3x + 4) (3x − 1) − 33x = −20

9.3 Quadratic Formula 1483


Chapter 9 Solving Quadratic Equations and Graphing Parabolas

71. 27y (y + 1) + 2 (3y − 2) = 0

72. 8 (4y 2 + 3) − 3 (28y − 1) = 0

73. (x + 2) 2 − 2 (x + 7) = 4 (x + 1)

74. (x + 3) 2 − 10 (x + 5) = −2 (x + 1)

Part B: Discussion Board

75. When talking about a quadratic equation in standard form,


ax 2 + bx + c = 0, why is it necessary to state that a ≠ 0? What
happens if a is equal to 0?

76. Research and discuss the history of the quadratic formula and solutions
to quadratic equations.

9.3 Quadratic Formula 1484


Chapter 9 Solving Quadratic Equations and Graphing Parabolas

ANSWERS

1: a = 1, b = −1 , and c = 5

3: a = 3, b = 0, and c = −10

5: a = 5, b = −7 , and c = 0

7: a = −1, b = 1, and c = 6

9: a = 6, b = −2 , and c = 1

11: 4, 6

13: −5, −1

15: −1/2, 4

17: −3/2, 1

19: 1, double root

⎯⎯
21: 3 ± √5
⎯⎯
23: −1 ± √6
⎯⎯
25: 2 ± √5

1±√5
27:
2

⎯⎯
29: 8 ± √2

2±√10
31:
2

33: 3/2, double root

9.3 Quadratic Formula 1485


Chapter 9 Solving Quadratic Equations and Graphing Parabolas

−3±√3
35:
3

−1±2√3
37:
3

⎯⎯
39: ±√ 2

√3
41: ±
3

43: No real solutions

45: 0, double root

47: 0, 5/2

49: 0, 1

51: −0.3, 0.9

53: −0.1, 1.5

⎯⎯⎯⎯
55: −5 ± √ 22

−1±√17
57:
12

5±3√3
59:
2

−3±√33
61:
4

⎯⎯
63: ±√ 6

⎯⎯
65: −1 ± √7

67: No real solutions

9.3 Quadratic Formula 1486


Chapter 9 Solving Quadratic Equations and Graphing Parabolas

69: 1/5, double root

71: −4/3, 1/9

⎯⎯⎯⎯
73: 1 ± √ 15

9.3 Quadratic Formula 1487


Chapter 9 Solving Quadratic Equations and Graphing Parabolas

9.4 Guidelines for Solving Quadratic Equations and Applications

LEARNING OBJECTIVES

1. Use the discriminant to determine the number and type of solutions to


any quadratic equation.
2. Develop a general strategy for solving quadratic equations.
3. Solve applications involving quadratic equations.

Discriminant

If given a quadratic equation in standard form, ax 2 + bx + c = 0, where a, b, and c


are real numbers and a ≠ 0, then the solutions can be calculated using the
quadratic formula:

The solutions are rational, irrational, or not real. We can determine the type and
number of solutions by studying the discriminant7, the expression inside the
radical, b2 − 4ac. If the value of this expression is negative, then the equation has
no real solutions. If the discriminant is positive, then we have two real solutions.
And if the discriminant is 0, then we have one real solution.

Example 1: Determine the type and number of solutions: x 2 − 10x + 30 = 0.

Solution: We begin by identifying a, b, and c. Here

7. The expression inside the


radical of the quadratic
2
formula, b − 4ac. Substitute these values into the discriminant and simplify.

1488
Chapter 9 Solving Quadratic Equations and Graphing Parabolas

Since the discriminant is negative, we conclude that there are no real solutions.

Answer: No real solution

If we use the quadratic formula in the previous example, we find that a negative
radicand stops the process of simplification and shows that there is no real solution.

Note

We will study quadratic equations with no real solutions as we progress in our


study of algebra.

Example 2: Determine the type and number of solutions: 7x 2 − 10x + 1 = 0.

Solution: Here

9.4 Guidelines for Solving Quadratic Equations and Applications 1489


Chapter 9 Solving Quadratic Equations and Graphing Parabolas

Substitute these values into the discriminant:

Since the discriminant is positive, we can conclude that there are two real solutions.

Answer: Two real solutions

If we use the quadratic formula in the previous example, we find that a positive
radicand in the quadratic formula leads to two real solutions.

9.4 Guidelines for Solving Quadratic Equations and Applications 1490


Chapter 9 Solving Quadratic Equations and Graphing Parabolas

5−3√2 5+3√2
The two real solutions are 7 and 7 . Note that these solutions are irrational;
we can approximate the values on a calculator.

Example 3: Determine the type and number of solutions: 2x 2 − 7x − 4 = 0.

Solution: In this example,

Substitute these values into the discriminant and simplify.

Since the discriminant is positive, we conclude that there are two real solutions.
Furthermore, since the discriminant is a perfect square, we obtain two rational
solutions.

Answer: Two real solutions

We could solve the previous quadratic equation using the quadratic formula as
follows:

9.4 Guidelines for Solving Quadratic Equations and Applications 1491


Chapter 9 Solving Quadratic Equations and Graphing Parabolas

Note that if the discriminant is a perfect square, then we could have factored the
original equation.

Given the special condition where the discriminant is 0, we obtain only one
solution, a double root.

Example 4: Determine the type and number of solutions: 9x 2 − 6x + 1 = 0.

Solution: Here a = 9, b = −6, and c = 1, and we have

9.4 Guidelines for Solving Quadratic Equations and Applications 1492


Chapter 9 Solving Quadratic Equations and Graphing Parabolas

Since the discriminant is 0, we conclude that there is only one real solution, a
double root.

Answer: One real solution

Since 0 is a perfect square, we can solve the equation above by factoring.

Here 1/3 is a solution that occurs twice; it is a double root.

In summary, if given any quadratic equation in standard form, ax 2 + bx + c = 0,


where a, b, and c are real numbers and a ≠ 0, then we have the following:

Positive discriminant: b 2 − 4ac > 0 Two real solutions

Zero discriminant: b 2 − 4ac = 0 One real solution

Negative discriminant: b 2 − 4ac < 0 No real solution

As we will see, knowing the number and type of solutions ahead of time helps us
determine which method is best for solving a quadratic equation.

9.4 Guidelines for Solving Quadratic Equations and Applications 1493


Chapter 9 Solving Quadratic Equations and Graphing Parabolas

Try this! Determine the number and type of solutions: 3x 2 − 5x + 4 = 0.

Answer: No real solution

Video Solution

(click to see video)


General Guidelines for Solving Quadratic Equations

Use the coefficients of a quadratic equation to help decide which method is most
appropriate for solving it. While the quadratic formula always works, it is
sometimes not the most efficient method. Given any quadratic equation in standard
form, ax 2 + bx + c = 0, general guidelines for determining the method for
solving it follow:

1. If c = 0, then factor out the GCF and solve by factoring.


2. If b = 0, then solve by extracting the roots.
3. If a, b, and c are all nonzero, then determine the value for the
discriminant, b2 − 4ac:

a. If the discriminant is a perfect square, then solve by factoring.


b. If the discriminant is not a perfect square, then solve using the
quadratic formula.

a. If the discriminant is positive, we obtain two real solutions.


b. If the discriminant is negative, then there is no real solution.

Example 5: Solve: 15x 2 − 5x = 0.

Solution: In this case, c = 0 and we can solve by factoring out the GCF.

9.4 Guidelines for Solving Quadratic Equations and Applications 1494


Chapter 9 Solving Quadratic Equations and Graphing Parabolas

Answer: The solutions are 0 and 1/3.

Example 6: Solve: 3x 2 − 5 = 0.

Solution: In this case, b = 0 and we can solve by extracting the roots.

√15
Answer: The solutions are ± 3
.

Example 7: Solve: 9x 2 − 6x − 7 = 0.

9.4 Guidelines for Solving Quadratic Equations and Applications 1495


Chapter 9 Solving Quadratic Equations and Graphing Parabolas

Solution: Begin by identifying a, b, and c as the coefficients of each term. Here

Substitute these values into the discriminant and then simplify.

Since the discriminant is positive and not a perfect square, use the quadratic
formula and expect two real solutions.

1±2√2
Answer: The solutions are 3
.

9.4 Guidelines for Solving Quadratic Equations and Applications 1496


Chapter 9 Solving Quadratic Equations and Graphing Parabolas

Example 8: Solve: 4x (x − 2) = −7.

Solution: Begin by rewriting the quadratic equation in standard form.

Here

Substitute these values into the discriminant and then simplify.

Since the discriminant is negative, the solutions are not real numbers.

Answer: No real solution

Example 9: Solve: (3x + 5) (3x + 7) = 6x + 10.

Solution: Begin by rewriting the quadratic equation in standard form.

9.4 Guidelines for Solving Quadratic Equations and Applications 1497


Chapter 9 Solving Quadratic Equations and Graphing Parabolas

Substitute a = 9, b = 30, and c = 25 into the discriminant.

Since the discriminant is 0, solve by factoring and expect one real solution, a double
root.

Answer: The solution is −5/3.

Try this! Solve: 5x 2 + 2x − 7 = 2x − 3.

2√5
Answer: ± 5

9.4 Guidelines for Solving Quadratic Equations and Applications 1498


Chapter 9 Solving Quadratic Equations and Graphing Parabolas

Video Solution

(click to see video)


Applications Involving Quadratic Equations

In this section, the algebraic setups usually consist of a quadratic equation where
the solutions may not be integers.

Example 10: The height of a triangle is 2 inches less than twice the length of its
base. If the total area of the triangle is 11 square inches, then find the lengths of the
base and height. Round answers to the nearest hundredth.

Solution:

Use the formula A = 1


2
bh and the fact that the area is 11 square inches to set up an
algebraic equation.

9.4 Guidelines for Solving Quadratic Equations and Applications 1499


Chapter 9 Solving Quadratic Equations and Graphing Parabolas

To rewrite this quadratic equation in standard form, first distribute 1


2
x.

Use the coefficients, a = 1, b = −1, and c = −11, to determine the type of solutions.

Since the discriminant is positive, expect two real solutions.

In this problem, disregard the negative solution and consider only the positive
solution.

Back substitute to find the height.

9.4 Guidelines for Solving Quadratic Equations and Applications 1500


Chapter 9 Solving Quadratic Equations and Graphing Parabolas

1+3√5
Answer: The base measures ≈ 3.85inches and the height is
⎯⎯ 2

−1 + 3√5 ≈ 5.71 inches.

Example 11: The sum of the squares of two consecutive positive integers is 481.
Find the integers.

Solution:

The algebraic setup follows:

Rewrite the quadratic equation in standard form.

9.4 Guidelines for Solving Quadratic Equations and Applications 1501


Chapter 9 Solving Quadratic Equations and Graphing Parabolas

When the coefficients are large, sometimes it is less work to use the quadratic
formula instead of trying to factor it. In this case, a = 1, b = 1, and c = −240.
Substitute into the quadratic formula and then simplify.

Since the problem calls for positive integers, disregard the negative solution and
choose n = 15.

9.4 Guidelines for Solving Quadratic Equations and Applications 1502


Chapter 9 Solving Quadratic Equations and Graphing Parabolas

Answer: The positive integers are 15 and 16.

KEY TAKEAWAYS

• Determine the number and type of solutions to any quadratic equation


2
in standard form using the discriminant, b − 4ac. If the discriminant
is negative, then the solutions are not real. If the discriminant is
positive, then the solutions are real. If the discriminant is 0, then there
is only one solution, a double root.
• Choose the appropriate method for solving a quadratic equation based
on the value of its discriminant. While the quadratic formula will solve
any quadratic equation, it may not be the most efficient method.
• When solving applications, use the key words and phrases to set up an
algebraic equation that models the problem. In this section, the setup
typically involves a quadratic equation.

9.4 Guidelines for Solving Quadratic Equations and Applications 1503


Chapter 9 Solving Quadratic Equations and Graphing Parabolas

TOPIC EXERCISES

Part A: Using the Discriminant

Calculate the discriminant and use it to determine the number and type of solutions.
Do not solve.

1. x 2 + 2x + 3 = 0

2. x 2 − 2x − 3 = 0

3. 3x 2 − 1x − 2 = 0

4. 3x 2 − 1x + 2 = 0

5. 9y 2 +2=0

6. 9y 2 −2=0

7. 5x 2 +x=0

8. 5x 2 −x=0

9. 1
2
x 2 − 2x + 5
2
=0

10. 1
2
x2 − x − 1
2
=0

11. −x 2 − 2x + 4 = 0

12. −x 2 − 4x + 2 = 0

13. 4t 2 − 20t + 25 = 0

14. 9t 2 − 6t + 1 = 0

Part B: Solving

9.4 Guidelines for Solving Quadratic Equations and Applications 1504


Chapter 9 Solving Quadratic Equations and Graphing Parabolas

Choose the appropriate method to solve the following.

15. x 2 − 2x − 3 = 0

16. x 2 + 2x + 3 = 0

17. 3x 2 −x−2=0

18. 3x 2 −x+2=0

19. 9y 2 +2=0

20. 9y 2 −2=0

21. 5x 2 +x=0

22. 5x 2 −x=0

23. 1
2
x 2 − 2x + 5
2
=0

24. 1
2
x2 − x − 1
2
=0

25. −x 2 − 2x + 4 = 0

26. −x 2 − 4x + 2 = 0

27. 4t 2 − 20t + 25 = 0

28. 9t 2 − 6t + 1 = 0

29. y 2 − 4y − 1 = 0

30. y 2 − 6y − 3 = 0

31. 25x 2 +1=0

32. 36x 2 +4=0

9.4 Guidelines for Solving Quadratic Equations and Applications 1505


Chapter 9 Solving Quadratic Equations and Graphing Parabolas

33. 5t 2 −4=0

34. 2t 2 −9=0

35. 1
2
x2 − 9
4
x+1=0

36. 3x 2 + 1
2
x− 1
6
=0

37. 36y 2 = 2y

38. 50y 2 = −10y

39. x (x − 6) = −29

40. x (x − 4) = −16

41. 4y (y + 1) = 5

42. 2y (y + 2) = 3

43. −3x 2 = 2x + 1

44. 3x 2 + 4x = −2

45. 6(x + 1) 2 = 11x + 7

46. 2(x + 2) 2 = 7x + 11

47. 9t 2 = 4 (3t − 1)

48. 5t (5t − 6) = −9

49. (x + 1) (x + 7) = 3

50. (x − 5) (x + 7) = 14

Part C: Applications

9.4 Guidelines for Solving Quadratic Equations and Applications 1506


Chapter 9 Solving Quadratic Equations and Graphing Parabolas

Set up an algebraic equation and use it to solve the following.

Number Problems

51. A positive real number is 2 less than another. When 4 times the larger is
added to the square of the smaller, the result is 49. Find the numbers.

52. A positive real number is 1 more than another. When twice the smaller is
subtracted from the square of the larger, the result is 4. Find the numbers.

53. A positive real number is 6 less than another. If the sum of the squares of
the two numbers is 38, then find the numbers.

54. A positive real number is 1 more than twice another. If 4 times the
smaller number is subtracted from the square of the larger, then the result
is 21. Find the numbers.

Geometry Problems

Round off your answers to the nearest hundredth.

55. The area of a rectangle is 60 square inches. If the length is 3 times the
width, then find the dimensions of the rectangle.

56. The area of a rectangle is 6 square feet. If the length is 2 feet more than
the width, then find the dimensions of the rectangle.

57. The area of a rectangle is 27 square meters. If the length is 6 meters less
than 3 times the width, then find the dimensions of the rectangle.

58. The area of a triangle is 48 square inches. If the base is 2 times the height,
then find the length of the base.

59. The area of a triangle is 14 square feet. If the base is 4 feet more than 2
times the height, then find the length of the base and the height.

60. The area of a triangle is 8 square meters. If the base is 4 meters less than
the height, then find the length of the base and the height.

61. The perimeter of a rectangle is 54 centimeters and the area is 180 square
centimeters. Find the dimensions of the rectangle.

9.4 Guidelines for Solving Quadratic Equations and Applications 1507


Chapter 9 Solving Quadratic Equations and Graphing Parabolas

62. The perimeter of a rectangle is 50 inches and the area is 126 square
inches. Find the dimensions of the rectangle.

63. George maintains a successful 6-meter-by-8-meter garden. Next season


he plans on doubling the planting area by increasing the width and height
by an equal amount. By how much must he increase the length and width?

64. A uniform brick border is to be constructed around a 6-foot-by-8-foot


garden. If the total area of the garden, including the border, is to be 100
square feet, then find the width of the brick border.

Pythagorean Theorem

⎯⎯
65. If the sides of a square measure 10√ 6 units, then find the length of the
diagonal.

⎯⎯⎯⎯
66. If the diagonal of a square measures 3√ 10 units, then find the length
of each side.

⎯⎯
67. The diagonal of a rectangle measures 6√ 3 inches. If the width is 4
inches less than the length, then find the dimensions of the rectangle.

⎯⎯
68. The diagonal of a rectangle measures 2√ 3 inches. If the width is 2
inches less than the length, then find the dimensions of the rectangle.

69. The top of a 20-foot ladder, leaning against a building, reaches a height
of 18 feet. How far is the base of the ladder from the wall? Round off to the
nearest hundredth.

70. To safely use a ladder, the base should be placed about 1/4 of the ladder’s
length away from the wall. If a 20-foot ladder is to be safely used, then how
high against a building will the top of the ladder reach? Round off to the
nearest hundredth.

71. The diagonal of a television monitor measures 32 inches. If the monitor


has a 3:2 aspect ratio, then determine its length and width. Round off to the
nearest hundredth.

9.4 Guidelines for Solving Quadratic Equations and Applications 1508


Chapter 9 Solving Quadratic Equations and Graphing Parabolas

72. The diagonal of a television monitor measures 52 inches. If the monitor


has a 16:9 aspect ratio, then determine its length and width. Round off to the
nearest hundredth.

Business Problems

73. The profit in dollars of running an assembly line that produces custom
uniforms each day is given by the function
P(t) = −40t 2 + 960t − 4,000 , where t represents the number of
hours the line is in operation.

a. Calculate the profit on running the assembly line for 10 hours a day.

b. Calculate the number of hours the assembly line should run in order to
break even. Round off to the nearest tenth of an hour.

74. The profit in dollars generated by producing and selling x custom lamps
is given by the function P(x) = −10x 2 + 800x − 12,000 .

a. Calculate the profit on the production and sale of 35 lamps.

b. Calculate the number of lamps that must be sold to profit $3,000.

75. If $1,200 is invested in an account earning an annual interest rate r, then


the amount A that is in the account at the end of 2 years is given by the
2
formula A = 1,200(1 + r) . If at the end of 2 years the amount in the
account is $1,335.63, then what was the interest rate?

76. A manufacturing company has determined that the daily revenue, R, in


thousands of dollars depends on the number, n, of palettes of product sold
according to the formula R = 12n − 0.6n 2 . Determine the number of
palettes that must be sold in order to maintain revenues at $60,000 per day.

Projectile Problems

77. The height of a projectile launched upward at a speed of 32 feet/second


from a height of 128 feet is given by the function
h (t) = −16t 2 + 32t + 128 .

a. What is the height of the projectile at 1/2 second?

9.4 Guidelines for Solving Quadratic Equations and Applications 1509


Chapter 9 Solving Quadratic Equations and Graphing Parabolas

b. At what time after launch will the projectile reach a height of 128 feet?

78. The height of a projectile launched upward at a speed of 16 feet/second


from a height of 192 feet is given by the function
h (t) = −16t 2 + 16t + 192 .

a. What is the height of the projectile at 3/2 seconds?

b. At what time will the projectile reach 128 feet?

79. The height of an object dropped from the top of a 144-foot building is
given by h (t) = −16t 2 + 144 . How long will it take to reach a point
halfway to the ground?

80. The height of a projectile shot straight up into the air at 80 feet/second
from the ground is given by h (t) = −16t 2 + 80t. At what time will the
projectile reach 95 feet?

Part D: Discussion Board

81. Discuss the strategy of always using the quadratic formula to solve
quadratic equations.

82. List all of the methods that we have learned so far to solve quadratic
equations. Discuss the pros and cons of each.

9.4 Guidelines for Solving Quadratic Equations and Applications 1510


Chapter 9 Solving Quadratic Equations and Graphing Parabolas

ANSWERS

1: −8, no real solution

3: 25, two real solutions

5: −72, no real solution

7: 1, two real solutions

9: −1, no real solution

11: 20, two real solutions

13: 0, one real solution

15: −1, 3

17: −2/3, 1

19: No real solution

21: −1/5, 0

23: No real solution

⎯⎯
25: −1 ± √5

27: 5/2

⎯⎯
29: 2 ± √5

31: No real solution

2√5
33: ±
5

35: 1/2, 4

9.4 Guidelines for Solving Quadratic Equations and Applications 1511


Chapter 9 Solving Quadratic Equations and Graphing Parabolas

37: 0, 1/18

39: No real solution

−1±√6
41:
2

43: No real solution

45: −1/2, 1/3

47: 2/3

⎯⎯
49: −4 ± 2√ 3
⎯⎯ ⎯⎯
51: 3√ 5 and 3√ 5 −2
⎯⎯⎯⎯ ⎯⎯⎯⎯
53: 3 + √ 10 and −3 + √ 10

55: Length: 13.42 inches; width: 4.47 inches

57: Length: 6.48 meters; width: 4.16 meters

59: Height: 2.87 feet; base: 9.74 feet

61: Length: 15 centimeters; width: 12 centimeters

63: 2.85 meters

⎯⎯
65: 20√ 3 units

⎯⎯ ⎯⎯
67: Length: 2 + 5√ 2 inches; width: −2 + 5√ 2 inches
⎯⎯⎯⎯
69: 2√ 19 ≈ 8.72 feet

71: Length: 26.63 inches; width: 17.75 inches

73: a. $1,600; b. 5.4 hours and 18.6 hours

9.4 Guidelines for Solving Quadratic Equations and Applications 1512


Chapter 9 Solving Quadratic Equations and Graphing Parabolas

75: 5.5%

77: a. 140 feet; b. 0 seconds and 2 seconds

79: 2.12 seconds

9.4 Guidelines for Solving Quadratic Equations and Applications 1513


Chapter 9 Solving Quadratic Equations and Graphing Parabolas

9.5 Graphing Parabolas

LEARNING OBJECTIVES

1. Graph a parabola.
2. Find the intercepts and vertex of a parabola.
3. Find the vertex of a parabola by completing the square.

The Graph of a Quadratic Equation

We know that any linear equation with two variables can be written in the form
y = mx + b and that its graph is a line. In this section, we will see that any
quadratic equation of the form y = ax 2 + bx + c has a curved graph called a
parabola8.

Two points determine any line. However, since a parabola is curved, we should find
more than two points. In this text, we will determine at least five points as a means
to produce an acceptable sketch. To begin, we graph our first parabola by plotting
points. Given a quadratic equation of the form y = ax 2 + bx + c, x is the
independent variable and y is the dependent variable. Choose some values for x and
then determine the corresponding y-values. Then plot the points and sketch the
graph.

8. The graph of any quadratic Example 1: Graph by plotting points: y = x 2 − 2x − 3 .


equation
y = ax 2 + bx + c, where a,
b, and c are real numbers and Solution: In this example, choose the x-values {−2, −1, 0, 1, 2, 3, 4} and calculate the
a ≠ 0. corresponding y-values.

1514
Chapter 9 Solving Quadratic Equations and Graphing Parabolas

Plot these points and determine the shape of the graph.

Answer:

9. The point that defines the


minimum or maximum of a
parabola.

10. The vertical line through the


When graphing, we want to include certain special points in the graph. The y-
b
vertex, x = − 2a , about
intercept is the point where the graph intersects the y-axis. The x-intercepts are the
which the parabola is points where the graph intersects the x-axis. The vertex9 is the point that defines
symmetric. the minimum or maximum of the graph. Lastly, the line of symmetry10 (also called

9.5 Graphing Parabolas 1515


Chapter 9 Solving Quadratic Equations and Graphing Parabolas

the axis of symmetry11) is the vertical line through the vertex, about which the
parabola is symmetric.

For any parabola, we will find the vertex and y-intercept. In addition, if the x-
intercepts exist, then we will want to determine those as well. Guessing at the x-
values of these special points is not practical; therefore, we will develop techniques
that will facilitate finding them. Many of these techniques will be used extensively
as we progress in our study of algebra.

Given a quadratic equation of the form y = ax 2 + bx + c, find the y-intercept by


setting x = 0 and solving. In general, y = a(0)2 + b (0) + c = c, and we have

11. A term used when referencing


the line of symmetry.

9.5 Graphing Parabolas 1516


Chapter 9 Solving Quadratic Equations and Graphing Parabolas

Next, recall that the x-intercepts, if they exist, can be found by setting y = 0 . Doing
this, we have 0 = a2 + bx + c, which has general solutions given by the quadratic
−b±√b2 −4ac
formula, x = 2a
. Therefore, the x-intercepts have this general form:

Using the fact that a parabola is symmetric, we can determine the vertical line of
symmetry using the x-intercepts. To do this, we find the x-value midway between
the x-intercepts by taking an average as follows:

Therefore, the line of symmetry is the vertical line:

We can use the line of symmetry to find the x-value of the vertex. The steps for
graphing a parabola are outlined in the following example.

9.5 Graphing Parabolas 1517


Chapter 9 Solving Quadratic Equations and Graphing Parabolas

Example 2: Graph: y = −x 2 − 2x + 3 .

Solution:

Step 1: Determine the y-intercept. To do this, set x = 0 and solve for y.

The y-intercept is (0, 3).

Step 2: Determine the x-intercepts. To do this, set y = 0 and solve for x.

Here when y = 0, we obtain two solutions. There are two x-intercepts, (−3, 0) and
(1, 0).

Step 3: Determine the vertex. One way to do this is to use the equation for the line
b
of symmetry, x = − 2a , to find the x-value of the vertex. In this example, a = −1 and
b = −2:

9.5 Graphing Parabolas 1518


Chapter 9 Solving Quadratic Equations and Graphing Parabolas

Substitute −1 into the original equation to find the corresponding y-value.

The vertex is (−1, 4).

Step 4: Determine extra points so that we have at least five points to plot. In this
example, one other point will suffice. Choose x = −2 and find the corresponding y-
value.

Our fifth point is (−2, 3).

Step 5: Plot the points and sketch the graph. To recap, the points that we have
found are

y-intercept: (0, 3)

x-intercept: (−3, 0) and (1, 0)

Vertex: (−1, 4)

Extra point: (−2, 3)

9.5 Graphing Parabolas 1519


Chapter 9 Solving Quadratic Equations and Graphing Parabolas

Answer:

The parabola opens downward. In general, use the leading coefficient to determine
whether the parabola opens upward or downward. If the leading coefficient is
negative, as in the previous example, then the parabola opens downward. If the
leading coefficient is positive, then the parabola opens upward.

9.5 Graphing Parabolas 1520


Chapter 9 Solving Quadratic Equations and Graphing Parabolas

All quadratic equations of the form y = ax 2 + bx + c have parabolic graphs with


y-intercept (0, c). However, not all parabolas have x intercepts.

Example 3: Graph: y = 2x 2 + 4x + 5 .

Solution: Because the leading coefficient 2 is positive, note that the parabola opens
upward. Here c = 5 and the y-intercept is (0, 5). To find the x-intercepts, set y = 0.

In this case, a = 2, b = 4, and c = 5. Use the discriminant to determine the number and
type of solutions.

Since the discriminant is negative, we conclude that there are no real solutions.
Because there are no real solutions, there are no x-intercepts. Next, we determine
the x-value of the vertex.

9.5 Graphing Parabolas 1521


Chapter 9 Solving Quadratic Equations and Graphing Parabolas

Given that the x-value of the vertex is −1, substitute into the original equation to
find the corresponding y-value.

The vertex is (−1, 3). So far, we have only two points. To determine three more,
choose some x-values on either side of the line of symmetry, x = −1. Here we choose
x-values −3, −2, and 1.

To summarize, we have

y-intercept: (0, 5)

x-intercepts: None

Vertex: (−1, 3)

Extra points: (−3, 11), (−2, 5), (1, 11)

Plot the points and sketch the graph.

Answer:

9.5 Graphing Parabolas 1522


Chapter 9 Solving Quadratic Equations and Graphing Parabolas

Example 4: Graph: y = −2x 2 + 12x − 18.

Solution: Note that a = −2: the parabola opens downward. Since c = −18, the y-
intercept is (0, −18). To find the x-intercepts, set y = 0.

Solve by factoring.

9.5 Graphing Parabolas 1523


Chapter 9 Solving Quadratic Equations and Graphing Parabolas

Here x = 3 is a double root, so there is only one x-intercept, (3, 0). From the original
equation, a = −2, b = 12, and c = −18. The x-value of the vertex can be calculated as
follows:

Given that the x-value of the vertex is 3, substitute into the original equation to find
the corresponding y-value.

Therefore, the vertex is (3, 0), which happens to be the same point as the x-
intercept. So far, we have only two points. To determine three more, choose some x-
values on either side of the line of symmetry, x = 3 in this case. Choose x-values 1, 5,
and 6.

To summarize, we have

y-intercept: (0, −18)

9.5 Graphing Parabolas 1524


Chapter 9 Solving Quadratic Equations and Graphing Parabolas

x-intercept: (3, 0)

Vertex: (3, 0)

Extra points: (1, −8), (5, −8), (6, −18)

Plot the points and sketch the graph.

Answer:

Example 5: Graph: y = x 2 − 2x − 1 .

Solution: Since a = 1, the parabola opens upward. Furthermore, c = −1, so the y-


intercept is (0, −1). To find the x-intercepts, set y = 0.

In this case, solve using the quadratic formula with a = 1, b = −2, and c = −1.

9.5 Graphing Parabolas 1525


Chapter 9 Solving Quadratic Equations and Graphing Parabolas

Here we obtain two real solutions for x, and thus there are two x-intercepts:

Approximate values using a calculator:

Use the approximate answers to place the ordered pair on the graph. However, we
will present the exact x-intercepts on the graph. Next, find the vertex.

9.5 Graphing Parabolas 1526


Chapter 9 Solving Quadratic Equations and Graphing Parabolas

Given that the x-value of the vertex is 1, substitute into the original equation to find
the corresponding y-value.

The vertex is (1, −2). We need one more point.

To summarize, we have

y-intercept: (0, −1)


⎯⎯ ⎯⎯
x-intercepts: (1 − √ 2 , 0) and (1 + √ 2 , 0)
Vertex: (1, −2)

Extra point: (2, −1)

Plot the points and sketch the graph.

Answer:

9.5 Graphing Parabolas 1527


Chapter 9 Solving Quadratic Equations and Graphing Parabolas

Try this! Graph: y = 9x 2 − 5.

Answer:

9.5 Graphing Parabolas 1528


Chapter 9 Solving Quadratic Equations and Graphing Parabolas

Video Solution

(click to see video)


Finding the Maximum and Minimum

It is often useful to find the maximum and/or minimum values of functions that
model real-life applications. To find these important values given a quadratic
function, we use the vertex. If the leading coefficient a is positive, then the parabola
opens upward and there will be a minimum y-value. If the leading coefficient a is
negative, then the parabola opens downward and there will be a maximum y-value.

Example 6: Determine the maximum or minimum: y = −4x 2 + 24x − 35.

Solution: Since a = −4, we know that the parabola opens downward and there will be
a maximum y-value. To find it, we first find the x-value of the vertex.

The x-value of the vertex is 3. Substitute this value into the original equation to find
the corresponding y-value.

9.5 Graphing Parabolas 1529


Chapter 9 Solving Quadratic Equations and Graphing Parabolas

The vertex is (3, 1). Therefore, the maximum y-value is 1, which occurs when x = 3,
as illustrated below:

Note

The graph is not required to answer this question.

Answer: The maximum is 1.

Example 7: Determine the maximum or minimum: y = 4x 2 − 32x + 62 .

Solution: Since a = +4, the parabola opens upward and there is a minimum y-value.
Begin by finding the x-value of the vertex.

9.5 Graphing Parabolas 1530


Chapter 9 Solving Quadratic Equations and Graphing Parabolas

Substitute x = 4 into the original equation to find the corresponding y-value.

The vertex is (4, −2). Therefore, the minimum y-value of −2 occurs when x = 4, as
illustrated below:

Answer: The minimum is −2.

9.5 Graphing Parabolas 1531


Chapter 9 Solving Quadratic Equations and Graphing Parabolas

Try this! Determine the maximum or minimum: y = (x − 3)2 − 9.

Answer: The minimum is −9.

Video Solution

(click to see video)

A parabola, opening upward or downward (as opposed to sideways), defines a


function and extends indefinitely to the right and left as indicated by the arrows.
Therefore, the domain (the set of x-values) consists of all real numbers. However,
the range (the set of y-values) is bounded by the y-value of the vertex.

Example 8: Determine the domain and range: y = x 2 − 4x + 3 .

Solution: First, note that since a = 1 is positive, the parabola opens upward. Hence
there will be a minimum y-value. To find that value, find the x-value of the vertex:

Then substitute into the equation to find the corresponding y-value.

The vertex is (2, −1). The range consists of the set of y-values greater than or equal
to the minimum y-value −1.

9.5 Graphing Parabolas 1532


Chapter 9 Solving Quadratic Equations and Graphing Parabolas

Answer: Domain: R = (−∞, ∞); range: [−1, ∞)

Example 9: The height in feet of a projectile is given by the function


h(t) = −16t2 + 72t, where t represents the time in seconds after launch. What is
the maximum height reached by the projectile?

Solution: Here a = −16, and the parabola opens downward. Therefore, the y-value
of the vertex determines the maximum height. Begin by finding the x-value of the
vertex:

The maximum height will occur in 9/4 = 2¼ seconds. Substitute this time into the
function to determine the height attained.

9.5 Graphing Parabolas 1533


Chapter 9 Solving Quadratic Equations and Graphing Parabolas

Answer: The maximum height of the projectile is 81 feet.

Finding the Vertex by Completing the Square

In this section, we demonstrate an alternate approach for finding the vertex. Any
quadratic equation y = ax 2 + bx + c can be rewritten in the form

In this form, the vertex is (h, k).

Example 10: Determine the vertex: y = −4(x − 3)2 + 1.

Solution: When the equation is in this form, we can read the vertex directly from
the equation.

Here h = 3 and k = 1.

Answer: The vertex is (3, 1).

9.5 Graphing Parabolas 1534


Chapter 9 Solving Quadratic Equations and Graphing Parabolas

Example 11: Determine the vertex: y = 2(x + 3)2 − 2.

Solution: Rewrite the equation as follows before determining h and k.

Here h = −3 and k = −2.

Answer: The vertex is (−3, −2).

Often the equation is not given in this form. To obtain this form, complete the
square.

Example 12: Rewrite in y = a(x − h) + k form and determine the vertex:


2

y = x 2 + 4x + 9 .

Solution: Begin by making room for the constant term that completes the square.

The idea is to add and subtract the value that completes the square, ( b2 ) , and then
2

factor. In this case, add and subtract ( 42 ) = (2)2 = 4.


2

9.5 Graphing Parabolas 1535


Chapter 9 Solving Quadratic Equations and Graphing Parabolas

Adding and subtracting the same value within an expression does not change it.
Doing so is equivalent to adding 0. Once the equation is in this form, we can easily
determine the vertex.

Here h = −2 and k = 5.

Answer: The vertex is (−2, 5).

If there is a leading coefficient other than 1, then we must first factor out the
leading coefficient from the first two terms of the trinomial.

Example 13: Rewrite in y = a(x − h) + k form and determine the vertex:


2

y = 2x 2 − 4x + 8 .

Solution: Since a = 2, factor this out of the first two terms in order to complete the
square. Leave room inside the parentheses to add a constant term.

9.5 Graphing Parabolas 1536


Chapter 9 Solving Quadratic Equations and Graphing Parabolas

( 2 ) = (−1) = 1. Add and subtract 1 and factor as follows:


Now use −2 to determine the value that completes the square. In this case,
−22 2

In this form, we can easily determine the vertex.

Here h = 1 and k = 6.

Answer: The vertex is (1, 6).

Try this! Rewrite in y = a(x − h) + k form and determine the vertex:


2

y = −2x 2 − 12x + 3.

9.5 Graphing Parabolas 1537


Chapter 9 Solving Quadratic Equations and Graphing Parabolas

Answer: y = −2(x + 3)2 + 21; vertex: (−3, 21)

Video Solution

(click to see video)

KEY TAKEAWAYS

• The graph of any quadratic equation y = ax 2 + bx + c, where a, b,


and c are real numbers and a ≠ 0, is called a parabola.
• When graphing parabolas, find the vertex and y-intercept. If the x-
intercepts exist, find those as well. Also, be sure to find ordered pair
solutions on either side of the line of symmetry, x = − 2a b
.
• Use the leading coefficient, a, to determine if a parabola opens upward
or downward. If a is positive, then it opens upward. If a is negative, then
it opens downward.
• The vertex of any parabola has an x-value equal to − 2a b
. After finding
the x-value of the vertex, substitute it into the original equation to find
the corresponding y-value. This y-value is a maximum if the parabola
opens downward, and it is a minimum if the parabola opens upward.
• The domain of a parabola opening upward or downward consists of all
real numbers. The range is bounded by the y-value of the vertex.

equation in the form y = a(x − h) + k. When in this form, the


• An alternate approach to finding the vertex is to rewrite the quadratic
2

vertex is (h, k) and can be read directly from the equation. To obtain this
form, take y = ax 2 + bx + c and complete the square.

9.5 Graphing Parabolas 1538


Chapter 9 Solving Quadratic Equations and Graphing Parabolas

TOPIC EXERCISES

Part A: The Graph of Quadratic Equations

Does the parabola open upward or downward? Explain.

1. y = x 2 − 9x + 20

2. y = x 2 − 12x + 32

3. y = −2x 2 + 5x + 12

4. y = −6x 2 + 13x − 6

5. y = 64 − x 2

6. y = −3x + 9x 2

Determine the x- and y-intercepts.

7. y = x 2 + 4x − 12

8. y = x 2 − 13x + 12

9. y = 2x 2 + 5x − 3

10. y = 3x 2 − 4x − 4

11. y = −5x 2 − 3x + 2

12. y = −6x 2 + 11x − 4

13. y = 4x 2 − 25

14. y = 9x 2 − 49

15. y = x2 − x + 1

9.5 Graphing Parabolas 1539


Chapter 9 Solving Quadratic Equations and Graphing Parabolas

16. y = 5x 2 + 15x

Find the vertex and the line of symmetry.

17. y = −x 2 + 10x − 34

18. y = −x 2 − 6x + 1

19. y = −4x 2 + 12x − 7

20. y = −9x 2 + 6x + 2

21. y = 4x 2 − 1

22. y = x 2 − 16

Graph. Find the vertex and the y-intercept. In addition, find the x-intercepts if they
exist.

23. y = x 2 − 2x − 8

24. y = x 2 − 4x − 5

25. y = −x 2 + 4x + 12

26. y = −x 2 − 2x + 15

27. y = x 2 − 10x

28. y = x 2 + 8x

29. y = x2 − 9

30. y = x 2 − 25

31. y = 1 − x2

32. y = 4 − x2

9.5 Graphing Parabolas 1540


Chapter 9 Solving Quadratic Equations and Graphing Parabolas

33. y = x 2 − 2x + 1

34. y = x 2 + 4x + 4

35. y = −4x 2 + 12x − 9

36. y = −4x 2 − 4x + 3

37. y = x2 − 2

38. y = x2 − 3

39. y = −4x 2 + 4x − 3

40. y = 4x 2 + 4x + 3

41. y = x 2 − 2x − 2

42. y = x 2 − 6x + 6

43. y = −2x 2 + 6x − 3

44. y = −4x 2 + 4x + 1

45. y = x 2 + 3x + 4

46. y = −x 2 + 3x − 4

47. y = −2x 2 + 3

48. y = −2x 2 − 1

49. y = 2x 2 + 4x − 3

50. y = 3x 2 + 2x − 2

Part B: Maximum or Minimum

9.5 Graphing Parabolas 1541


Chapter 9 Solving Quadratic Equations and Graphing Parabolas

Determine the maximum or minimum y-value.

51. y = −x 2 − 6x + 1

52. y = −x 2 − 4x + 8

53. y = 25x 2 − 10x + 5

54. y = 16x 2 − 24x + 7

55. y = −x 2

56. y = 1 − 9x 2

57. y = 20x − 10x 2

58. y = 12x + 4x 2

59. y = 3x 2 − 4x − 2

60. y = 6x 2 − 8x + 5

Given the following quadratic functions, determine the domain and range.

61. f (x) = 3x 2 + 30x + 50

62. f (x) = 5x 2 − 10x + 1

63. g(x) = −2x 2 + 4x + 1

64. g(x) = −7x 2 − 14x − 9

65. The height in feet reached by a baseball tossed upward at a speed of 48


feet/second from the ground is given by the function
h (t) = −16t 2 + 48t, where t represents time in seconds. What is the
baseball’s maximum height and how long will it take to attain that height?

9.5 Graphing Parabolas 1542


Chapter 9 Solving Quadratic Equations and Graphing Parabolas

66. The height of a projectile launched straight up from a mound is given by


the function h(t) = −16t 2 + 96t + 4, where t represents seconds after
launch. What is the maximum height?

67. The profit in dollars generated by producing and selling x custom lamps
is given by the function P(x) = −10x 2 + 800x − 12,000 . What is
the maximum profit?

68. The revenue in dollars generated from selling a particular item is


modeled by the formula R(x) = 100x − 0.0025x 2 , where x
represents the number of units sold. What number of units must be sold to
maximize revenue?

69. The average number of hits to a radio station website is modeled by the
formula f (x) = 450t 2 − 3,600t + 8,000 , where t represents the
number of hours since 8:00 a.m. At what hour of the day is the number of
hits to the website at a minimum?

70. The value in dollars of a new car is modeled by the formula


V(t) = 125t 2 − 3,000t + 22,000 , where t represents the number of
years since it was purchased. Determine the minimum value of the car.

71. The daily production costs in dollars of a textile manufacturing company


producing custom uniforms is modeled by the formula
C(x) = 0.02x 2 − 20x + 10,000 , where x represents the number of
uniforms produced.

a. How many uniforms should be produced to minimize the daily production


costs?

b. What is the minimum daily production cost?

72. The area of a certain rectangular pen is given by the formula


A = 14w − w 2 , where w represents the width in feet. Determine the
width that produces the maximum area.

Part C: Vertex by Completing the Square

Determine the vertex.

9.5 Graphing Parabolas 1543


Chapter 9 Solving Quadratic Equations and Graphing Parabolas

= −(x − 5) + 3
2
73. y

74. y = −2(x − 1) 2 + 7

75. y = 5(x + 1) 2 + 6

76. y = 3(x + 4) 2 + 10

77. y = −5(x + 8) 2 − 1

78. y = (x + 2) 2 − 5

= a(x − h) + k form and determine the vertex.


2
Rewrite in y

79. y = x 2 − 14x + 24

80. y = x 2 − 12x + 40

81. y = x 2 + 4x − 12

82. y = x 2 + 6x − 1

83. y = 2x 2 − 12x − 3

84. y = 3x 2 − 6x + 5

85. y = −x 2 + 16x + 17

86. y = −x 2 + 10x

Graph.

87. y = x2 − 1

88. y = x2 + 1

9.5 Graphing Parabolas 1544


Chapter 9 Solving Quadratic Equations and Graphing Parabolas

89. y = (x − 1) 2

90. y = (x + 1) 2

91. y = (x − 4) 2 − 9

92. y = (x − 1) 2 − 4

93. y = −2(x + 1) 2 + 8

94. y = −3(x + 2) 2 + 12

95. y = −5(x − 1) 2

96. y = −(x + 2) 2

97. y = −4(x − 1) 2 − 2

98. y = 9(x + 1) 2 + 2

= (x + 5) − 15
2
99. y

= 2(x − 5) − 3
2
100. y

101. y = −2(x − 4) 2 + 22

102. y = 2(x + 3) 2 − 13

Part D: Discussion Board

103. Write down your plan for graphing a parabola on an exam. What will
you be looking for and how will you present your answer? Share your plan
on the discussion board.

104. Why is any parabola that opens upward or downward a function?


Explain to a classmate how to determine the domain and range.

9.5 Graphing Parabolas 1545


Chapter 9 Solving Quadratic Equations and Graphing Parabolas

ANSWERS

1: Upward

3: Downward

5: Downward

7: x-intercepts: (−6, 0), (2, 0); y-intercept: (0, −12)

9: x-intercepts: (−3, 0), (1/2, 0); y-intercept: (0, −3)

11: x-intercepts: (−1, 0), (2/5, 0); y-intercept: (0, 2)

13: x-intercepts: (−5/2, 0), (5/2, 0); y-intercept: (0, −25)

15: x-intercepts: none; y-intercept: (0, 1)

17: Vertex: (5, −9); line of symmetry: x =5

19: Vertex: (3/2, 2); line of symmetry: x = 3/2

21: Vertex: (0, −1); line of symmetry: x =0

23:

25:

9.5 Graphing Parabolas 1546


Chapter 9 Solving Quadratic Equations and Graphing Parabolas

27:

29:

31:

33:

9.5 Graphing Parabolas 1547


Chapter 9 Solving Quadratic Equations and Graphing Parabolas

35:

37:

39:

41:

9.5 Graphing Parabolas 1548


Chapter 9 Solving Quadratic Equations and Graphing Parabolas

43:

45:

47:

49:

9.5 Graphing Parabolas 1549


Chapter 9 Solving Quadratic Equations and Graphing Parabolas

51: Maximum: y = 10

53: Minimum: y = 4

55: Maximum: y = 0

57: Maximum: y = 10

59: Minimum: y = −10/3

61: Domain: R; range: [−25, ∞)

63: Domain: R; range: (−∞, 3]

65: The maximum height of 36 feet occurs after 1.5 seconds.

67: $4,000

69: 12:00 p.m.

71: a. 500 uniforms; b. $5,000

73: (5, 3)

75: (−1, 6)

77: (−8, −1)

79: y = (x − 7) 2 − 25 ; vertex: (7, −25)

81: y = (x + 2) 2 − 16 ; vertex: (−2, −16)

9.5 Graphing Parabolas 1550


Chapter 9 Solving Quadratic Equations and Graphing Parabolas

83: y = 2(x − 3) 2 − 21 ; vertex: (3, −21)

85: y = −(x − 8) 2 + 81 ; vertex: (8, 81)

87:

89:

91:

93:

9.5 Graphing Parabolas 1551


Chapter 9 Solving Quadratic Equations and Graphing Parabolas

95:

97:

99:

101:

9.5 Graphing Parabolas 1552


Chapter 9 Solving Quadratic Equations and Graphing Parabolas

9.6 Introduction to Complex Numbers and Complex Solutions

LEARNING OBJECTIVES

1. Perform operations with complex numbers.


2. Solve quadratic equations with complex solutions.

Introduction to Complex Numbers

Up to this point, the square root of a negative number has been left undefined. For
⎯⎯⎯⎯⎯
example, we know that √−9 is not a real a number.

There is no real number that when squared results in a negative number. We begin
the resolution of this issue by defining the imaginary unit12, i, as the square root of
−1.

To express a square root of a negative number in terms of the imaginary unit i, we


use the following property, where a represents any nonnegative real number:

With this we can write

⎯⎯⎯⎯⎯
12. Defined as i = √−1 and
i2 = −1.

1553
Chapter 9 Solving Quadratic Equations and Graphing Parabolas

⎯⎯⎯⎯⎯
If √−9 = 3i, then we would expect that 3i squared equals −9:

Therefore, the square root of any negative real number can be written in terms of
the imaginary unit. Such numbers are often called imaginary numbers13.

Example 1: Rewrite in terms of the imaginary unit i.

⎯⎯⎯⎯⎯
a. √−4

⎯⎯⎯⎯⎯
b. √−5

⎯⎯⎯⎯⎯
c. √−8

Solution:

⎯⎯⎯⎯⎯ ⎯⎯⎯⎯⎯⎯⎯⎯⎯⎯ ⎯⎯⎯⎯⎯ ⎯⎯


a. √−4 = √−1 ⋅ 4 = √−1 ⋅ √4 = i ⋅ 2 = 2i

⎯⎯⎯⎯⎯ ⎯⎯⎯⎯⎯⎯⎯⎯⎯⎯ ⎯⎯⎯⎯⎯ ⎯⎯ ⎯⎯


b. √−5 = √−1 ⋅ 5 = √−1 ⋅ √5 = i√5

⎯⎯⎯⎯⎯ ⎯⎯⎯⎯⎯⎯⎯⎯⎯⎯⎯⎯⎯⎯ ⎯⎯⎯⎯⎯ ⎯⎯ ⎯⎯ ⎯⎯ ⎯⎯


c. √−8 = √−1 ⋅ 4 ⋅ 2 = √−1 ⋅ √4 ⋅ √2 = i ⋅ 2 ⋅ √2 = 2i√2

13. The square roots of any


negative real numbers.

9.6 Introduction to Complex Numbers and Complex Solutions 1554


Chapter 9 Solving Quadratic Equations and Graphing Parabolas

Notation Note

When an imaginary number involves a radical, place i in front of the radical.


Consider the following:

⎯⎯ ⎯⎯
2i√2 = 2√2 i

Since multiplication is commutative, these numbers are equivalent. However,


⎯⎯
in the form 2√2i, the imaginary unit i is often misinterpreted to be part of the
radicand. To avoid this confusion, it is a best practice to place the i in front of
⎯⎯
the radical and use 2i√2.

A complex number14 is any number of the form

where a and b are real numbers. Here a is called the real part15 and b is called the
imaginary part16. For example, 3 − 4i is a complex number with a real part, 3, and
an imaginary part, −4. It is important to note that any real number is also a complex
number. For example, the real number 5 is also a complex number because it can be
written as 5 + 0i with a real part of 5 and an imaginary part of 0. Hence the set of
real numbers, denoted R, is a subset of the set of complex numbers, denoted C.

Adding and subtracting complex numbers is similar to adding and subtracting like
terms. Add or subtract the real parts and then the imaginary parts.

14. Numbers of the form a + bi, Example 2: Add: (3 − 4i) + (2 + 5i).


where a and b are real
numbers.
Solution: Add the real parts and then add the imaginary parts.
15. The real number a of a complex
number a + bi.

16. The real number b of a complex


number a + bi.

9.6 Introduction to Complex Numbers and Complex Solutions 1555


Chapter 9 Solving Quadratic Equations and Graphing Parabolas

Answer: 5 + i

To subtract complex numbers, subtract the real parts and subtract the imaginary
parts. This is consistent with the use of the distributive property.

Example 3: Subtract: (3 − 4i) − (2 + 5i).

Solution: Distribute the negative one and then combine like terms.

Answer: 1 − 9i

The distributive property also applies when multiplying complex numbers. Make
use of the fact that i2 = −1 to resolve the result into standard form: a + bi.

Example 4: Multiply: 5i (3 − 4i).

Solution: Begin by applying the distributive property.

9.6 Introduction to Complex Numbers and Complex Solutions 1556


Chapter 9 Solving Quadratic Equations and Graphing Parabolas

Answer: 20 + 15i

Example 5: Multiply: (3 − 4i) (4 + 5i).

Solution:

Answer: 32 − i

Given a complex number a + bi, its complex conjugate17 is a − bi. We next explore
the product of complex conjugates.

17. Two complex numbers whose Example 6: Multiply: (3 − 4i) (3 + 4i).


real parts are the same and
imaginary parts are opposite. If
given a + bi, then its complex Solution:
conjugate is a − bi.

9.6 Introduction to Complex Numbers and Complex Solutions 1557


Chapter 9 Solving Quadratic Equations and Graphing Parabolas

Answer: 25

In general, the product of complex conjugates18 follows:

Note that the result does not involve the imaginary unit; hence the result is real.
This leads us to the very useful property:

To divide complex numbers, we apply the technique used to rationalize the


denominator. Multiply the numerator and denominator (dividend and divisor) by
the conjugate of the denominator. The result can then be resolved into standard
form, a + bi.

1
Example 7: Divide: 1−2i .
18. The real number that results
from multiplying complex

(a + bi) (a − bi) = a + b in
conjugates: Solution: In this example, the conjugate of the denominator is 1 + 2i. Multiply by 1
2 2
. the form (1+2i)
(1+2i)
.

9.6 Introduction to Complex Numbers and Complex Solutions 1558


Chapter 9 Solving Quadratic Equations and Graphing Parabolas

To express this complex number in standard form, write each term over the
common denominator 5.

Answer: 1
5
+ 2
5
i

Example 8: Divide: 3−4i


3+2i
.

Solution:

9.6 Introduction to Complex Numbers and Complex Solutions 1559


Chapter 9 Solving Quadratic Equations and Graphing Parabolas

1
Answer: 13 − 18
13
i

5+5i
Try this! Divide: 1−3i .

Answer: −1 + 2i

Video Solution

(click to see video)


Quadratic Equations with Complex Solutions

Now that complex numbers are defined, we can complete our study of solutions to
quadratic equations. Often solutions to quadratic equations are not real.

Example 9: Solve using the quadratic formula: x 2 − 2x + 5 = 0

Solution: Begin by identifying a, b, and c. Here

9.6 Introduction to Complex Numbers and Complex Solutions 1560


Chapter 9 Solving Quadratic Equations and Graphing Parabolas

Substitute these values into the quadratic formula and then simplify.

Check these solutions by substituting them into the original equation.

Answer: The solutions are 1 − 2i and 1 + 2i.

9.6 Introduction to Complex Numbers and Complex Solutions 1561


Chapter 9 Solving Quadratic Equations and Graphing Parabolas

The equation may not be given in standard form. The general steps for solving
using the quadratic formula are outlined in the following example.

Example 10: Solve: (2x + 1) (x − 3) = x − 8.

Solution:

Step 1: Write the quadratic equation in standard form.

Step 2: Identify a, b, and c for use in the quadratic formula. Here

Step 3: Substitute the appropriate values into the quadratic formula and then
simplify.

9.6 Introduction to Complex Numbers and Complex Solutions 1562


Chapter 9 Solving Quadratic Equations and Graphing Parabolas

Answer: The solution is 3


2
± 12 .iThe check is optional.

Example 11: Solve: x (x + 2) = −19.

Solution: Begin by rewriting the equation in standard form.

Here a = 1, b = 2, and c = 19. Substitute these values into the quadratic formula.

9.6 Introduction to Complex Numbers and Complex Solutions 1563


Chapter 9 Solving Quadratic Equations and Graphing Parabolas

⎯⎯ ⎯⎯
Answer: The solutions are −1 − 3i√2 and −1 + 3i√2.

Notation Note

Consider the following:

⎯⎯ ⎯⎯
−1 + 3i√2 = −1 + 3√2 i
⎯⎯
Both numbers are equivalent and −1 + 3√2i is in standard form, where the
⎯⎯
real part is −1 and the imaginary part is 3√2. However, this number is often
⎯⎯
expressed as −1 + 3i√2, even though this expression is not in standard form.
Again, this is done to avoid the possibility of misinterpreting the imaginary
unit as part of the radicand.

9.6 Introduction to Complex Numbers and Complex Solutions 1564


Chapter 9 Solving Quadratic Equations and Graphing Parabolas

Try this! Solve: (2x + 3) (x + 5) = 5x + 4.

−4±i√6 √6
Answer: 2
= −2 ± 2
i

Video Solution

(click to see video)

KEY TAKEAWAYS

• The result of adding, subtracting, multiplying, and dividing complex


numbers is a complex number.
• Use complex numbers to describe solutions to quadratic equations that
are not real.

9.6 Introduction to Complex Numbers and Complex Solutions 1565


Chapter 9 Solving Quadratic Equations and Graphing Parabolas

TOPIC EXERCISES

Part A: Introduction to Complex Numbers

Rewrite in terms of i.

⎯⎯⎯⎯⎯⎯⎯
1. √ −64

⎯⎯⎯⎯⎯⎯⎯
2. √ −81

⎯⎯⎯⎯⎯⎯⎯
3. √ −20

⎯⎯⎯⎯⎯⎯⎯
4. √ −18

⎯⎯⎯⎯⎯⎯⎯
5. √ −50

⎯⎯⎯⎯⎯⎯⎯
6. √ −48

⎯⎯⎯⎯⎯⎯⎯
7. −√ −45

⎯⎯⎯⎯⎯
8. −√ −8

9. √ −
⎯⎯⎯⎯⎯1⎯
4

10. √ −
⎯⎯⎯⎯⎯2⎯
9

Perform the operations.

11. (3 + 5i) + (7 − 4i)

12. (6 − 7i) + (−5 − 2i)

13. (−8 − 3i) + (5 + 2i)

9.6 Introduction to Complex Numbers and Complex Solutions 1566


Chapter 9 Solving Quadratic Equations and Graphing Parabolas

14. (−10 + 15i) + (15 − 20i)

15. ( 12 + 3
4
i) + ( 16 − 1
8
i)

16. ( 25 − 1
6
i) + ( 101 − 3
2
i)

17. (5 + 2i) − (8 − 3i)

18. (7 − i) − (−6 − 9i)

19. (−9 − 5i) − (8 + 12i)

20. (−11 + 2i) − (13 − 7i)

21. ( 141 + 3
2
i) − ( 47 − 3
4
i)

22. ( 38 − 1
3
i) − ( 12 − 1
2
i)

23. 2i (7 − 4i)

24. 6i (1 − 2i)

25. −2i (3 − 4i)

26. −5i (2 − i)

27. (2 + i) (2 − 3i)

28. (3 − 5i) (1 − 2i)

29. (1 − i) (8 − 9i)

30. (1 + 5i) (5 + 2i)

31. (4 + 3i)2

9.6 Introduction to Complex Numbers and Complex Solutions 1567


Chapter 9 Solving Quadratic Equations and Graphing Parabolas

32. (2 − 5i)
2

33. (4 − 2i) (4 + 2i)

34. (6 + 5i) (6 − 5i)

35. ( 12 + 2
3
i) ( 13 − 1
2
i)

36. ( 23 − 1
3
i) ( 12 − 3
2
i)

1
37.
5+4i

1
38.
3−4i

20i
39.
1−3i

10i
40.
1−2i

10−5i
41.
3−i

4−2i
42.
2−2i

5+10i
43.
3+4i

2−4i
44.
5+3i

1+2i
45.
2−3i

3−i
46.
4−5i

Part B: Complex Roots

Solve by extracting the roots and then solve by using the quadratic formula. Check
answers.

9.6 Introduction to Complex Numbers and Complex Solutions 1568


Chapter 9 Solving Quadratic Equations and Graphing Parabolas

47. x 2 +9=0

48. x 2 +1=0

49. 4t 2 + 25 = 0

50. 9t 2 +4=0

51. 4y 2 +3=0

52. 9y 2 +5=0

53. 3x 2 +2=0

54. 5x 2 +3=0

55. (x + 1) 2 + 4 = 0

56. (x + 3) 2 + 9 = 0

Solve using the quadratic formula.

57. x 2 − 2x + 10 = 0

58. x 2 − 4x + 13 = 0

59. x 2 + 4x + 6 = 0

60. x 2 + 2x + 9 = 0

61. y 2 − 6y + 17 = 0

62. y 2 − 2y + 19 = 0

63. t 2 − 5t + 10 = 0

64. t 2 + 3t + 4 = 0

9.6 Introduction to Complex Numbers and Complex Solutions 1569


Chapter 9 Solving Quadratic Equations and Graphing Parabolas

65. −x 2 + 10x − 29 = 0

66. −x 2 + 6x − 10 = 0

67. −y 2 −y−2=0

68. −y 2 + 3y − 5 = 0

69. −2x 2 + 10x − 17 = 0

70. −8x 2 + 20x − 13 = 0

71. 3y 2 − 2y + 4 = 0

72. 5y 2 − 4y + 3 = 0

73. 2x 2 + 3x + 2 = 0

74. 4x 2 + 2x + 1 = 0

75. 2x 2 − 1
2
x+ 1
4
=0

76. 3x 2 − 2
3
x+ 1
3
=0

77. 2x (x − 1) = −1

78. x (2x + 5) = 3x − 5

79. 3t (t − 2) + 4 = 0

80. 5t (t − 1) = t − 4

81. (2x + 3) 2 = 16x + 4

82. (2y + 5) − 12 (y + 1) = 0
2

83. −3 (y + 3) (y − 5) = 5y + 46

9.6 Introduction to Complex Numbers and Complex Solutions 1570


Chapter 9 Solving Quadratic Equations and Graphing Parabolas

84. −2 (y − 4) (y + 1) = 3y + 10

85. 9x (x − 1) + 3 (x + 2) = 1

86. 5x (x + 2) − 6 (2x − 1) = 5

87. 3 (t − 1) − 2t (t − 2) = 6t

88. 3 (t − 3) − t (t − 5) = 7t

89. (2x + 3) (2x − 3) − 5 (x 2 + 1) = −9

90. 5 (x + 1) (x − 1) − 3x 2 = −8

Part C: Discussion Board

91. Explore the powers of i. Share your discoveries on the discussion board.

92. Research and discuss the rich history of imaginary numbers.

93. Research and discuss real-world applications involving complex


numbers.

9.6 Introduction to Complex Numbers and Complex Solutions 1571


Chapter 9 Solving Quadratic Equations and Graphing Parabolas

ANSWERS

1: 8i

⎯⎯
3: 2i√ 5

⎯⎯
5: 5i√ 2

⎯⎯
7: −3i√ 5

i
9:
2

11: 10 +i

13: −3 −i

15: 2
3
+ 5
8
i

17: −3 + 5i

19: −17 − 17i

21: − 1
2
+ 9
4
i

23: 8 + 14i

25: −8 − 6i

27: 7 − 4i

29: −1 − 17i

31: 7 + 24i

33: 20

35: 1
2
− 1
36
i

9.6 Introduction to Complex Numbers and Complex Solutions 1572


Chapter 9 Solving Quadratic Equations and Graphing Parabolas

37: 5
41
− 4
41
i

39: −6 + 2i

41: 7
2
− 1
2
i

43: 11
5
− 2
5
i

45: − 4
13
+ 7
13
i

47: ±3i

5i
49: ±
2

i√3
51: ±
2

i√6
53: ±
3

55: −1 ± 2i

57: 1 ± 3i
⎯⎯
59: −2 ± i√ 2
⎯⎯
61: 3 ± 2i√ 2

63: 5
2
± √15
2
i

65: 5 ± 2i

67: − ± i
1 √7
2 2

69: 5
2
± 3
2
i

71: 1
3
± √11
3
i

9.6 Introduction to Complex Numbers and Complex Solutions 1573


Chapter 9 Solving Quadratic Equations and Graphing Parabolas

73: − ± i
3 √7
4 4

75: 1
8
± √7
8
i

77: 1
2
± 1
2
i

79: 1 ± i
√3
3

81: 1
2
±i

83: 1
6
± √11
6
i

85: 1
3
± 2
3
i

87: 1
4
± √23
4
i
⎯⎯
89: ±i√ 5

9.6 Introduction to Complex Numbers and Complex Solutions 1574


Chapter 9 Solving Quadratic Equations and Graphing Parabolas

9.7 Review Exercises and Sample Exam

1575
Chapter 9 Solving Quadratic Equations and Graphing Parabolas

REVIEW EXERCISES

Extracting Square Roots

Solve by extracting the roots.

1. x 2 − 16 = 0
9
2. y 2 = 4

3. x 2 − 27 = 0

4. x 2 + 27 = 0

5. 3y 2 − 25 = 0

6. 9x 2 −2=0

7. (x − 5) − 9 = 0
2

8. (2x − 1) 2 − 1 = 0

9. 16(x − 6) − 3 = 0
2

10. 2(x + 3) 2 − 5 = 0

11. (x + 3) (x − 2) = x + 12

12. (x + 2) (5x − 1) = 9x − 1

Find a quadratic equation in standard form with the given solutions.

⎯⎯
13. ±√ 2

⎯⎯
14. ±2√ 5

9.7 Review Exercises and Sample Exam 1576


Chapter 9 Solving Quadratic Equations and Graphing Parabolas

Completing the Square

Complete the square.

15. x 2 − 6x+? = (x−?) 2

16. x 2 − x+? = (x−?) 2

Solve by completing the square.

17. x 2 − 12x + 1 = 0

18. x 2 + 8x + 3 = 0

19. y 2 − 4y − 14 = 0

20. y 2 − 2y − 74 = 0

21. x 2 + 5x − 1 = 0

22. x 2 − 7x − 2 = 0

23. 2x 2 +x−3=0

24. 5x 2 + 9x − 2 = 0

25. 2x 2 − 16x + 5 = 0

26. 3x 2 − 6x + 1 = 0

27. 2y 2 + 10y + 1 = 0

28. 5y 2 +y−3=0

29. x (x + 9) = 5x + 8

30. (2x + 5) (x + 2) = 8x + 7

9.7 Review Exercises and Sample Exam 1577


Chapter 9 Solving Quadratic Equations and Graphing Parabolas

Quadratic Formula

Identify the coefficients a, b, and c used in the quadratic formula. Do not solve.

31. x 2 −x+4=0

32. −x 2 + 5x − 14 = 0

33. x 2 −5=0

34. 6x 2 +x=0

Use the quadratic formula to solve the following.

35. x 2 − 6x + 6 = 0

36. x 2 + 10x + 23 = 0

37. 3y 2 −y−1=0

38. 2y 2 − 3y + 5 = 0

39. 5x 2 − 36 = 0

40. 7x 2 + 2x = 0

41. −x 2 + 5x + 1 = 0

42. −4x 2 − 2x + 1 = 0

43. t 2 − 12t − 288 = 0

44. t 2 − 44t + 484 = 0

45. (x − 3) 2 − 2x = 47

46. 9x (x + 1) − 5 = 3x

9.7 Review Exercises and Sample Exam 1578


Chapter 9 Solving Quadratic Equations and Graphing Parabolas

Guidelines for Solving Quadratic Equations and Applications

Use the discriminant to determine the number and type of solutions.

47. −x 2 + 5x + 1 = 0

48. −x 2 +x−1=0

49. 4x 2 − 4x + 1 = 0

50. 9x 2 −4=0

Solve using any method.

51. x 2 + 4x − 60 = 0

52. 9x 2 + 7x = 0

53. 25t 2 −1=0

54. t 2 + 16 = 0

55. x 2 −x−3=0

56. 9x 2 + 12x + 1 = 0

57. 4(x − 1) 2 − 27 = 0

58. (3x + 5) − 4 = 0
2

59. (x − 2) (x + 3) = 6

60. x (x − 5) = 12

61. (x + 1) (x − 8) + 28 = 3x

62. (9x − 2) (x + 4) = 28x − 9

9.7 Review Exercises and Sample Exam 1579


Chapter 9 Solving Quadratic Equations and Graphing Parabolas

Set up an algebraic equation and use it to solve the following.

63. The length of a rectangle is 2 inches less than twice the width. If the area
measures 25 square inches, then find the dimensions of the rectangle. Round
off to the nearest hundredth.

64. An 18-foot ladder leaning against a building reaches a height of 17 feet.


How far is the base of the ladder from the wall? Round to the nearest tenth
of a foot.

65. The value in dollars of a new car is modeled by the function


V (t) = 125t 2 − 3,000t + 22,000 , where t represents the number of
years since it was purchased. Determine the age of the car when its value is
$22,000.

66. The height in feet reached by a baseball tossed upward at a speed of 48


feet/second from the ground is given by the function
h (t) = −16t 2 + 48t, where t represents time in seconds. At what time
will the baseball reach a height of 16 feet?

Graphing Parabolas

Determine the x- and y-intercepts.

67. y = 2x 2 + 5x − 3

68. y = x 2 − 12

69. y = 5x 2 − x + 2

70. y = −x 2 + 10x − 25

Find the vertex and the line of symmetry.

71. y = x 2 − 6x + 1

72. y = −x 2 + 8x − 1

73. y = x 2 + 3x − 1

9.7 Review Exercises and Sample Exam 1580


Chapter 9 Solving Quadratic Equations and Graphing Parabolas

74. y = 9x 2 − 1

Graph. Find the vertex and the y-intercept. In addition, find the x-intercepts if they
exist.

75. y = x 2 + 8x + 12

76. y = −x 2 − 6x + 7

77. y = −2x 2 − 4

78. y = x 2 + 4x

79. y = 4x 2 − 4x + 1

80. y = −2x 2

81. y = −2x 2 + 8x − 7

82. y = 3x 2 − 1

Determine the maximum or minimum y-value.

83. y = x 2 − 10x + 1

84. y = −x 2 + 12x − 1

85. y = −5x 2 + 6x

86. y = 2x 2 − x − 1

87. The value in dollars of a new car is modeled by the function


V (t) = 125t 2 − 3,000t + 22,000 , where t represents the number of
years since it was purchased. Determine the age of the car when its value is
at a minimum.

88. The height in feet reached by a baseball tossed upward at a speed of 48


feet/second from the ground is given by the function

9.7 Review Exercises and Sample Exam 1581


Chapter 9 Solving Quadratic Equations and Graphing Parabolas

h (t) = −16t 2 + 48t, where t represents time in seconds. What is the


maximum height of the baseball?

Introduction to Complex Numbers and Complex Solutions

Rewrite in terms of i.

⎯⎯⎯⎯⎯⎯⎯
89. √ −36

⎯⎯⎯⎯⎯⎯⎯
90. √ −40

91. √ −
⎯⎯⎯⎯⎯⎯⎯8 ⎯
25

92. −√ −
⎯⎯⎯⎯⎯1⎯
9

Perform the operations.

93. (2 − 5i) + (3 + 4i)

94. (6 − 7i) − (12 − 3i)

95. (2 − 3i) (5 + i)

4−i
96.
2−3i

Solve.

97. 9x 2 + 25 = 0

98. 3x 2 +1=0

99. y 2 −y+5=0

100. y 2 + 2y + 4

101. 4x (x + 2) + 5 = 8x

9.7 Review Exercises and Sample Exam 1582


Chapter 9 Solving Quadratic Equations and Graphing Parabolas

102. 2 (x + 2) (x + 3) = 3 (x 2 + 13)

9.7 Review Exercises and Sample Exam 1583


Chapter 9 Solving Quadratic Equations and Graphing Parabolas

SAMPLE EXAM

Solve by extracting the roots.

1. 4x 2 −9=0

2. (4x + 1) 2 − 5 = 0

Solve by completing the square.

3. x 2 + 10x + 19 = 0

4. x 2 −x−1=0

Solve using the quadratic formula.

5. −2x 2 +x+3=0

6. x 2 + 6x − 31 = 0

Solve using any method.

7. (5x + 1) (x + 1) = 1

8. (x + 5) (x − 5) = 65

9. x (x + 3) = −2

10. 2(x − 2) 2 − 6 = 3x 2

Set up an algebraic equation and solve.

11. The length of a rectangle is twice its width. If the diagonal measures
⎯⎯
6√ 5 centimeters, then find the dimensions of the rectangle.

12. The height in feet reached by a model rocket launched from a platform is
given by the function h(t) = −16t 2 + 256t + 3 , where t represents
time in seconds after launch. At what time will the rocket reach 451 feet?

9.7 Review Exercises and Sample Exam 1584


Chapter 9 Solving Quadratic Equations and Graphing Parabolas

Graph. Find the vertex and the y-intercept. In addition, find the x-intercepts if they
exist.

13. y = 2x 2 − 4x − 6

14. y = −x 2 + 4x − 4

15. y = 4x 2 − 9

16. y = x 2 + 2x − 1

17. Determine the maximum or minimum y-value:


y = −3x 2 + 12x − 15 .

18. Determine the x- and y-intercepts: y = x 2 + x + 4.

19. Determine the domain and range: y = 25x 2 − 10x + 1 .

20. The height in feet reached by a model rocket launched from a platform is
given by the function h(t) = −16t 2 + 256t + 3 , where t represents
time in seconds after launch. What is the maximum height attained by the
rocket.

21. A bicycle manufacturing company has determined that the weekly


revenue in dollars can be modeled by the formula R = 200n − n 2 , where
n represents the number of bicycles produced and sold. How many bicycles
does the company have to produce and sell in order to maximize revenue?

⎯⎯⎯⎯⎯⎯⎯
22. Rewrite in terms of i: √ −60 .

4−2i
23. Divide: .
4+2i

Solve.

24. 25x 2 +3=0

25. −2x 2 + 5x − 1 = 0

9.7 Review Exercises and Sample Exam 1585


Chapter 9 Solving Quadratic Equations and Graphing Parabolas

REVIEW EXERCISES ANSWERS

1: ±16

⎯⎯
3: ±3√ 3

5√3
5: ±
3

7: 2, 8

24±√3
9:
4

⎯⎯
11: ±3√ 2

13: x 2 −2=0

15: x 2 − 6x + 9 = (x − 3) 2
⎯⎯⎯⎯
17: 6 ± √ 35
⎯⎯
19: 2 ± 3√ 2

−5±√29
21:
2

23: −3/2, 1

8±3√6
25:
2

−5±√23
27:
2

⎯⎯
29: −2 ± 2√ 3

31: a = 1, b = −1 , and c = 4

9.7 Review Exercises and Sample Exam 1586


Chapter 9 Solving Quadratic Equations and Graphing Parabolas

33: a = 1, b = 0, and c = −5
⎯⎯
35: 3 ± √3

1±√13
37:
6

6√5
39: ±
5

5±√29
41:
2

43: −12, 24

⎯⎯
45: 4 ± 3√ 6

47: Two real solutions

49: One real solution

51: −10, 6

53: ±1/5

1±√13
55:
2

2±3√3
57:
2

59: −4, 3

⎯⎯
61: 5 ± √5

63: Length: 6.14 inches; width: 4.07 inches

65: It is worth $22,000 new and when it is 24 years old.

67: x-intercepts: (−3, 0), (1/2, 0); y-intercept: (0, −3)

9.7 Review Exercises and Sample Exam 1587


Chapter 9 Solving Quadratic Equations and Graphing Parabolas

69: x-intercepts: none; y-intercept: (0, 2)

71: Vertex: (3, −8); line of symmetry: x =3

73: Vertex: (−3/2, −13/4); line of symmetry: x =− 3


2

75:

77:

79:

81:

9.7 Review Exercises and Sample Exam 1588


Chapter 9 Solving Quadratic Equations and Graphing Parabolas

83: Minimum: y = −24

85: Maximum: y = 9/5

87: The car will have a minimum value 12 years after it is purchased.

89: 6i

2i√2
91:
5

93: 5 −i

95: 13 − 13i
5i
97: ±
3

1 √19
99:
2
± 2
i

i√5
101: ±
2

9.7 Review Exercises and Sample Exam 1589


Chapter 9 Solving Quadratic Equations and Graphing Parabolas

SAMPLE EXAM ANSWERS

1: ± 3
2

⎯⎯
3: −5 ± √6

5: −1, 3/2

7: −6/5, 0

9: −2, −1

11: Length: 12 centimeters; width: 6 centimeters

13:

15:

17: Maximum: y = −3

19: Domain: R; range: [0, ∞)

21: To maximize revenue, the company needs to produce and sell 100
bicycles a week.

9.7 Review Exercises and Sample Exam 1590


Chapter 9 Solving Quadratic Equations and Graphing Parabolas

3 4
23:
5
− 5
i

5±√17
25:
4

9.7 Review Exercises and Sample Exam 1591


Chapter 10
Appendix: Geometric Figures

LEARNING OBJECTIVE

1.

1592
Chapter 10 Appendix: Geometric Figures

10.1 Plane

Area (A) is measured in square units, perimeter (P) is measured in units, and
circumference (C) is measured in units.

Square

Rectangle

Parallelogram

1593
Chapter 10 Appendix: Geometric Figures

Trapezoid

Triangle

Circle

10.1 Plane 1594


Chapter 10 Appendix: Geometric Figures

10.2 Solid

Volume (V) is measured in cubic units and surface area (SA) is measured in square
units.

Cube

Rectangular Solid

1595
Chapter 10 Appendix: Geometric Figures

Right Circular Cylinder

Right Circular Cone

Sphere

10.2 Solid 1596

You might also like